Shortcuts & Tips in Quantitativ - Disha Experts

You might also like

Download as pdf or txt
Download as pdf or txt
You are on page 1of 735

Tips & Techniques

in English
for Competitive Exams

SSC/ Banking/ Insurance / Railways/ CDS/ NDA/


Hotel Mgmt./ B.Ed/ GATE
EBD_7275
• Corporate Office : 45, 2nd Floor, Maharishi Dayanand Marg, Corner
Market, Malviya Nagar, New Delhi-110017
Tel. : 011-49842349 / 49842350

How to access the ebook(s)?


Educore

INSTRUCTIONS
1. Mail your Order ID at ebooks.support@aiets.co.in to get the
16 digit Access Code.
2. Go to http://www.educoreonline.com/register.htm
3. Enter your details along with your 16 digit Access Code.
4. Click Register & you would be successfully redirected to the
Login Page.
Note: If you are already registered with us, you just have to
login (http://www.educoreonline.com/login.htm) & enter
your new 16 Character Unique Code under your Account
Section.
5. Login with your registered email ID & password.
6. You can now view you e-book(s) under your Library.
7. You can read your e-books either Online or Offline. For offline,
simply download our Educore App once & download the e-
books inside the app. Educore App is available for Windows
Desktop, IOS & Android.
8. Educore works best in Desktop, Laptop, 7" & 10" tablets.
9. Contact us at support@educoreonline.com for any further
assistance.
Note: This app is not accessible on Mobiles.

DISHA PUBLICATION
ALL RIGHTS RESERVED
© Copyright Publisher

No part of this publication may be reproduced in any form without prior permission
of the publisher. The author and the publisher do not take any legal responsibility
for any errors or misrepresentations that might have crept in. We have tried and
made our best efforts to provide accurate up-to-date information in this book.

For further information about books from DISHA,


Log on to www.dishapublication.com or email to info@dishapublication.com
PREFACE
About the Book…..
Having command of English grammar isn’t impossible. With a few
essential grammar tips and techniques, you can overcome many of the
obstacles that the aspirants of various competitive exams face. Disha’s
‘Tips & Techniques in English for Competitive Exams’ contains all
those tools that help you improve your English in various competitive
exams.
Disha’s ‘Tips & Techniques in English for Competitive Exams’ covers
the entire syllabus for gaining clear concepts in English that consists
of Fundamental Grammar including Tenses, Modals, Parts of Speech,
Voices, Narrations, Phrasal Verbs, Question Tags, Transformation of
Sentences, Clause Analysis, Idioms & Phrases, Spelling Rules and
Contractions, etc., Vocabulary, Reading Comprehension, Parajumbles,
Cloze Test, Spotting Errors, Sentence Completion, Passage Completion
and many more.
‘Tips & Techniques in English for Competitive Exams’ by Disha is a
must-have book for thorough preparation in SSC, Banking, Insurance,
Railways, CDS, Hotel Management, B.Ed., Gate and various other
competitive exams.
The book ‘Tips & Techniques in English for Competitive Exams’ by
Disha has been designed to explore most of the tips and techniques
required for understanding English grammar and avoiding common
mistakes with an ample number of examples of Common Errors in
English and their detailed explanation .
As the book provides the information taken from sources believed to be
valid and most reliable, it can prove a comprehensive guide to learning
English Grammar and Composition.
Disha Experts
EBD_7275
Index
FUNDAMENTAL GRAMMAR 1-75
m Tenses 1
m Modals 8
m Infinitives, Gerunds and Participles 10
m Parts of Speech 21
m Articles 24
m Voices 26
m Narration 30
m Kinds of Sentences & Clauses 35
m Phrasal Verbs 38
m Question Tags 41
m Idioms and Phrases 43
m Sentence Structure 48
m Punctuation 58
m Contraction 63
m Common Errors 67
VOCABULARY 76-103
READING COMPREHENSION 104-112
m Objective Comprehension 104
PARAJUMBLES 113-119
m Rearranging Words or Sentences 113
CLOZE TEST 120-124
m Filling in Paragraph Gaps or Numbered Gaps 120
SPOTTING ERRORS 125-131
SENTENCE COMPLETION 132-137
PASSAGE COMPLETION 138-140
REVISION EXERCISES 141-184

E-book 1: Practice Exercises with Hints and Solutions


E-book 2: 10 Practice Sets
E-book 3: Past Solved Papers
FUNDAMENTAL GRAMMAR

Chapter

1 TENSES

Tense is the form taken by a verb to indicate time and continuance or completeness
of an action or event.
Tenses are of three main types viz. Present Tense, Past Tense and Future Tense. Further,
each of these is sub-divided into four categories i.e. Simple Tense, Continuous Tense,
Perfect Tense and Perfect Continuous Tense.

(I) PRESENT TENSE


It expresses an action that is currently going on or habitually performed, or a state
that currently or generally exists.
Examples: She plays hockey and then goes home.
He goes to dance classes.
I get up every day at four o’clock in the morning.
Sun rises in the east.
(1) Simple Present Tense – It is used to describe universal truths, habits,
unchanging situations and scheduled activities.
Examples for repeated or regular actions in the present time period:
 I take the tram to the office.
 Prakash works eight hours every day.
 The train to Mumbai leaves at 10 P.M.
Examples for facts:
 We belong to India.
 Sun sets in the west.
 The president of the USA lives in the White House.
Examples for habits:
 They travel to their farmhouse every weekend.
 She brushes her teeth twice a day.
 I get up early every day.
Formation of Simple Present Tense:
 The first person (I) takes the first form of verb like- I go and I work there
etc.
 The second person (You) takes the first form of verb like- You come and
You run etc.
 In the third person singular number, the verb always ends with ‘s’ like-
He wants, She gives, She thinks and She runs etc. In case the verb is
ending with ‘y’, it changes to ‘ies’ like- She flies and He cries etc.
 In the third person plural number, the first form of verb is used like- They
come and They go etc.
EBD_7275
2 tenses
(2) Present Continuous Tense – It is used for those actions which are happening
now or are unfinished. This tense is also used when the action is temporary
and it is also known as Present Progressive Tense.
Examples:
 He is weeping.
 She is talking with the guests.
 The baby is sleeping in the crib.
Present Continuous Tense is also used to express something not happening right
now or will not happen in the near future, for example- You are not watching the
game, She is not sitting over there and I am not going to the meeting after work
etc. Moreover, The Present Continuous Tense is also used in questions as well, for
example- Is he laughing?, Are you coming? and Are they listening to the teacher?
etc.
(3) Present Perfect Tense – It is used to indicate the completion of an activity or
an action that occurred at some point in the past. Though, the time of the action
is not exactly known, this tense is mostly used to refer to actions completed in
the immediate past (not a very long time ago).
Examples:
 I have eaten my meal.
 I have finished cooking.
 He has bought a car.
Note:
In the present perfect sentences, the past participle of 3rd form of verb is used with
the auxiliary verbs ‘has’ or ‘have ‘depending upon the subject of the sentence. For
example, if the subject of the sentence is ‘She, He, It or a singular noun’ then the
auxiliary verb ‘has’ is used and when the subject of the sentence is ‘They, You or
a plural noun’ then the auxiliary verb ‘have’ is used.
Examples:
 She has qualified the exam.
 They have helped us.
 You have done a good job.
 He has not started a business.
 It has come.
(4) Present Perfect Continuous Tense – It is used for an action which started in
the past and is continuing at the present time. A time reference is also used in
the sentence to show the time of action. The specific words ‘since’ and ‘for’
are used to show the time of action.
‘Since’ is used if the exact starting time of action is known like - since Sunday
and since 6 A.M etc. and on the other hand, ‘For’ is used to express the amount
of time like - for 10 days and for six months etc.
tenses 3
The auxiliary verbs ‘have been’ or ‘has been’ is used depending upon the
subject of the sentence. If the subject of the sentence is ‘She, He, It or a
singular noun’ then the auxiliary verb ‘has been’ is used and if the subject of
the sentence is ‘They, You or a plural noun’ then the auxiliary verb ‘have been’
is used.
Structure: Subject + Auxiliary verb + Main verb + Time-reference and Subject +
Have been/ Has been + Present Participle (verb+ing) + Time-reference
Examples:
 He has been living in the USA since 1990.
 She has been working in this company since 2002.
 He has been studying this book for three months.
 They have been waiting for me for three hours.
 Ravi has been writing for this newspaper since 25th May, 2007.
 I have been watching the movie for two hours.
Some examples of interrogative sentences are:
 Has she been working as a professor for five years?
 Have they been making a noise for two hours?
 Has she been writing the report since 1st March, 2016?
 Has your mother been teaching you since 2001?

(II) PAST TENSE


It expresses an action or event that has happened or a state that previously existed.
Examples:
 He went home yesterday.
 The work was finished on 4th June.
 She worked in a sugar factory.
 My father believed in superstitions.

(1) Simple Past Tense – It is used to talk about a completed action in a time
before now. The time of the action can be in the recent past or the distant past.
Examples for an action completed in the past:
 The steamer sailed yesterday.
 He went home some time back.
 She used to carry an umbrella.
(2) Past Continuous Tense – It is used to mention an ongoing action of the past
or an action that continued sometime in the past. It is also known as Past
Progressive Tense. In these sentences, 1st form of the verb + ing and auxiliary
verbs ‘was’ or ‘were’ is used depending upon the subject of the sentence. If
the subject of the sentence is ‘I, She, He or a singular noun then the auxiliary
verb ‘was’ is used and if the subject of the sentence is ‘You, They or a plural
noun then the auxiliary verb ‘were’ is used.
EBD_7275
4 tenses
Examples:
 He was waiting for his mother.
 She was riding a bike.
 The dog was barking at them.
 I was planning for the holidays.
 They were eating their meal.
 You were not preparing for the exam.
(3) Past Perfect Tense – It is used to show that something happened before another
action in the past or simply, it is used to express two actions that happened
in the past. Moreover, in this case, it is necessary to show which action/event
happened earlier than the other.
Examples:
 I had done my homework when Hari came to see me.
 They lost many games because they had not practised enough.
 You had studied French before you moved to Italy.
 When I reached the station, the train had departed.
 I had just gone out when it started raining.
(4) Past Perfect Continuous Tense – It is used when an action/event that began
before a certain point of time in the past and was continuing at the given point
of time in the sentence. The sentence includes a ‘time-reference’ i.e. ‘since’
and ‘for’ to show when the action started in past or for how long the action
was continued in the past.
Sentence structure:
Subject + Auxiliary verb + Main verb + Time-reference
Subject + Had been + Present Participle + Object + Time-reference
Examples:
 She had been watching the game for two hours.
 He had been working for a newspaper for seven years.
 I had been applying for jobs since May 2013.
 She had been teaching since October, 2010
 Had she been waiting for her husband for three years.

(III) FUTURE TENSE

It expresses an action/event that has not yet happened or a state that does not yet
exist.
Examples:
 I will go there.
 They will not play football.
 I shall meet him if he calls me.
tenses 5
(1) Simple Future Tense – It is used to express an action that will occur or
happen in the future.
Examples:
 I will buy a laptop at the end of this month.
 We will shift to a new apartment the next week.
 My father will buy me a bicycle on my birthday.
 He will leave for Canada day after tomorrow.
 She will get admission in a new school.
(2) Future Continuous Tense – It is used to express an on-going or continued
action which will occur at some time in the future. In these sentences, the first
form of the verb + ing is used along with the auxiliary verbs ‘will be or ‘shall
be’.
Examples:
 He will be singing a song for the audience.
 I shall be reading the paper then.
 He will be meeting us next week.
 I will be writing a report.
(3) Future Perfect Tense – It is used to indicate the completion of an action/event
in the future. In these sentences, the third form of the verb is used with the
auxiliary verbs ‘will have’ or ‘shall have’.
Examples:
 I shall have written my exercise by that time.
 He will have completed his project by Sunday.
 I will have taken my lunch.
 He would have finished his task.
(4) Future Perfect Continuous Tense – It is used to indicate an action represented
as being in progress over a period of time that will end in the future. Time
period is generally mentioned along with it. ‘Since’ or ‘for’ is used in the
sentence for time-reference. In these sentences, first form of the verb + ing
is used along with the auxiliary verbs ‘will have been’ or ‘shall have been’.
Examples:
 By next July, we shall have been living here for four years.
 The child will have been sleeping since 10 P.M.
 The doctor will have been treating patients for three years.
 You will have been using my bike for six months.
 The company will not have been advertising posts for two years.
EBD_7275
6 tenses
TENSES IN A NUTSHELL:

Tenses Present Past Future


Rule : Subject + V1 Subject + V2 Subject + will +
Form Form V1 Form
When to Universal Truths, Activity started Only when there
Simple be used : planned and in the past, is possibility
scheduled activi- got over in the of the activity
ties, description past. happening, not
of routines. certainity.
Rule : Subject + is/am/ Subject + was/ Subject + Will
are + ING Form were + ING be + ING Form
Form
Continuous When to When an Activ- An activity is There is a
be used : ity started some going on. Be- certainity of the
time ago, is still fore it is over, Activity happen-
continuing while it is interrupted ing in the future.
speaking. by another
activity.
Rule : Subject + Has/ Subject + has + Subject + Will
Have + V3 Form V3 Form have + V3 Form
When to When an Activity An activity Refers to Activ-
be used : is over but it still started in the ity 1, which
Perfect has an effect on past and was will have been
the present. concluded. An- completed, by
other Activity the time Activity
also happened. 2 happens.
REFERS TO
ACTIVITY 1.
Rule : Subect + has Subject + had Subject + Will
been + ING Form been + ING have been + ING
+ Since/for Form + since/ Form + since/for
for
Perfect
Continuous When to An Activity NOT USED NOT USED
be used : started in the past IN CONVER- IN CONVER-
but we are not SATIONAL SATIONAL
sure when it will ENGLISH ENGLISH
conclude/end.
tenses 7
Additional notes:
 Events occurring at the same time must be given in the same tense.
Examples
When he fainted, his brother was with him; When he was writing his report,
his mother was preparing meal for him, etc.
 Will or Shall can’t be used twice in the same sentence even if both the actions
refer to future tense.
Examples
I shall come if he will call me. (WRONG)
I shall come if he calls me. (RIGHT)
 With the phrase ‘as if ‘or ‘as though’, the past tense and plural form of the
verb should be used.
Examples - He behaves as if he were the owner.
It looks as if they have had a shock.
It looks as though you’ve not met him before.
 With the word ‘wish’, four verbs are used namely, were, had, could and
would.
‘Were’ is used when the wish seems to be unrealizable like, I wish I were a king.
‘Had’ is used when our wish is lament over the past happening like, I wish I had
accepted that offer.
‘Would’ is used when we refer to the future like I wish I would get a ticket.
‘Could’ is used when we wish that something which has already happened should
have happened otherwise like, He did not go because he was busy yesterday, I
wish he could go with you.
EBD_7275
Chapter

2 MODALS

Modals are verbs that are used to indicate modality i.e. the mood or the attitude of
the speaker which may be likelihood, ability, obligation, request, wish, duty and
suggestion etc. The commonly used modals are can, could, may, might, would,
shall, should, need, must and ought etc.
MODALS USAGE EXAMPLES
Can To express ability I can speak Russian fluently.
Can To request permission Can I open the door?
Could To request politely Could you please do it for me?
May To express possibility It may rain.
May To request permission May I come in?
May Wish or Prayer May you live long!
May Purpose He works hard so that he may pass.
Might Less possibility She might be sleeping now.
Must To express obligation You must leave now.
Must To express strong belief He must be over 80 years old.
Must Logical certainity Living alone in such a big city must
be difficult.
Should To give advice You should stop smoking.
Should Duty/Obligation We should obey the laws.
Should To express probability She should be in the temple.
Would To request or Offer Would you like to have a cup of tea?
Would Habitual action or Past She would study at noon.
Routine

Would Wish I would be glad to help you.


Modals 9

Ought To express moral We ought to love our parents.


* (It is obligation
stronger than
both should
and must.)

Will Wish, Request, Demand Will you please pick up the phone?
Will Prediction, Assumption I think it will rain on Monday.
Will Promise I will quit smoking.
Will To express habits Her mom is strange; she will sit for
hours without talking.

Need not To express an action He need not go there again.


which is not necessary
EBD_7275
Chapter
INFINITIVES, GERUNDS
3 AND PARTICIPLES

Infinitives, Gerunds and Participles are verb forms that perform peculiar functions,
other than principal verbs in sentences. They are called verbals.

INFINITIVES

Infinitives are, basically, to+verb phrases functioning as nouns.


They follow verbs such as agree, begin, continue, decide, fail, hesitate, hope,
intend, learn, neglect, offer, plan, prefer, pretend, promise, refuse, remember, start,
try that require no agent of action; and advise, allow, convince, remind, encourage,
force, hire, teach, instruct, invite, permit, tell, implore, incite, appoint, order which
need an agent of action.
FORM: To + verb
Stemming from the To + verb formula, infinitives appear in various forms as
under:
(i) To + V1 [present infinitive, active voice]
Examples:
 Most children hate to study.
 Try as you may, you are not cut out to succeed.
 He promised to deliver but his credibility was suspect.
 The employees were asked to furnish the details of their assets.
 This team is expected to win the match.
(ii)To + be + V2 [present infinitive, passive voice]
Examples:
 Young and old alike crave to be appreciated.
 Some great men walk the earth to be revered forever.
 Though extremely talented, he was destined to be doomed.
 Derek bolted like a bullet even as Sam appeared to be hurt.
 Having prepared an excellent meal, he was sure to be lauded by the
guests.
(iii) To + have + V3 [perfect infinitive, active voice]
Examples:
 Mother seems to have forgotten to shut the door.
 I remember to have met her in your birthday party.
 He claims to have seen them sneak into the premises.
infinitives, gerunds and participles 11
 The duo is reported to have stolen all the valuables in the house.
 They are believed to have spoken the truth.
(iv) To + have been + V3 [perfect infinitive, passive voice]
Examples:
 He seemed to have been warned of dire consequences.
 She appeared to have been taken for a ride.
 The witness looked to have been harassed by the police.
 The sky appeared to have been washed blue after the downpour.
 They admitted to have been mistaken in viewing her negatively.
(v) To + be + V1-ing [continuous infinitive]
Examples:
 You seem to be enjoying the show very much.
 The traffic seems to be delaying the arrival of the minister.
 He is too artless to be lying through his teeth.
 Terrorists are perceived to be seeking attention through their misdeeds.
(vi) To + have been + V1-ing [perfect continuous infinitive]
Examples:
 The clerk confessed to have been helping the embezzlers.
 The fugitive was reported to have been living with a nomadic tribe.
 Failed and defeated, he was suspected to have been contemplating
suicide.
 The movie looked to have been doing well before protesters brought it
down.
 They appear to have been going the wrong way.
[Note: Passive voice is not possible in continuous forms]
(vii) [To] Verb [bare infinitive or direct infinitive]
Bare infinitives associate with specific words as discussed below:
`` the verbs make, bid, see, hear, feel, know, watch, help, let, have, expect
with the agent of the action
Examples:
 He made them [to] see the truth behind the apparent.
 They bade him [to] ascend the stage and speak.
 It made him [to] feel the pain of the sufferers.
 The incident helped them [to] know their servants as human beings.
 You helped me [to] overcome my depression.
 I watched the ship [to] disappear down the horizon.
 I saw them [to] leave in a huff.
 On his request, the administration let him [to] preside over the conference.
 I will not have you [to] scold him for no fault of his.
EBD_7275
12 infinitives, gerunds and participles
`` need and dare only as auxiliaries in negative and interrogative forms
Examples:
 Mother, you need not [to] worry for me.
 Need you [to] work so hard?
 Dare he [to] cross the forest in the dark?
 I dare not [to] question his authority.
`` and, but, or, except, as... as, than
Examples:
 He did nothing more than [to] play the whole day.
 The banished people had nothing to do except [to] curse their fate.
 He liked to travel as much as [to] wander in the forests.
 Would you want to finish the work or [to] have your dinner first?
 The activists do nothing but [to] make people aware of the anomalies.
 Isn’t it foolish to fight and [to] complain of injuries?
`` better, rather, , sooner... than, as soon...as in combination with would or
had
Examples:
 The invitees would better/had better [to] leave as it is quite late.
 He would rather/had rather [to] start if he hopes to meet the deadline.
 She would as soon [to] sing as [to] dance.
 I would sooner [to] fight and die than [to] surrender.
{Note: All these phrases are synonyms of ‘prefer’, which itself takes a
‘gerund’ instead of an ‘infinitive’; e.g., Soldiers prefer dying for the country.}

SOME SPECIAL ADDITIONS WITH INFINITIVES

`` How + infinitive
This construction is followed by show, know, teach, learn, ask, tell,
remember, forget, discover, and find, etc., to express manner or method.
Examples:
 Let me show you how to operate this new device.
 They know how to deal with the situation.
 The class is learning (how) to create a story in dialogues.
 This should be enough to teach him how to respect his elders.
 We shall ask the instructor how to go about it.
 So far so good, now tell me how to conclude the dissertation.
 I am sorry; I do not remember how to solve this kind of problems.
 Curiously, he forgot how to unravel the maze he had himself created.
infinitives, gerunds and participles 13
 The mission discovered how to harvest relevant data from the junk.
 He is desperate to find out how to impress the girl he fancies.
`` Noun + infinitive
Preposition is essential in some infinitives following a noun to complete the
sense.
Examples:
 As long as you are engaged, please give me a book to read.
(no preposition)
 Can I have a glass of water to drink? (no preposition)
 I am afraid, I have no pen to write with. (preposition required)
 They are looking for a penthouse to live in. (preposition required)
`` Adjective + infinitive
Too is used before the adjective to convey a negative sense; Enough is used
after the adjective to convey a positive meaning.
Examples:
 I am too tired to venture any further.
 This is too good to be true.
 She is smart enough to deal with any untoward incident.
 The foundation should be strong enough to bear the impact of any tremor.

ERRONEOUS USE OF THE SPLIT INFINITIVE

Insertion of an adverb within an infinitive is an oft-committed mistake that


must be avoided.
Examples:
 I request you to kindly grant me the said leave. (incorrect)
 I request you kindly to grant me the said leave. (correct)
 The audience was instructed to quietly take their seats. (incorrect)
 The audience was instructed to take their seats quietly. (correct)
FUNCTION: As a Noun (subject of verb, object of verb, object of preposition,
subjective or
objective complement), Adjective, or Adverb.
As subject of verb :
 To err is human; to forgive, divine.
 To wait and watch is the only option we have.
 To pass the buck is easy.
As object of verb :
 The president-elect promised to deliver on the promises.
 The college staff threatened to strike work.
 He manages to balance work and home effortlessly.
EBD_7275
14 infinitives, gerunds and participles
As object of preposition :
 The invaders had no option but to surrender.
 Vagabonds love nothing except to wander.
 He lives a pauper only to die a millionaire.
As objective complement :
 Alia asked her to leave as it was getting dark.
 They wanted him to lead the team.
 No student wished the examinations to stay.
As subjective complement
 My parents’ desire is to see me well settled in life.
 The mark of a true man is to behave courteously.
 The delegation called to apprise him of the situation.
As adjective
 Napoleon nurtured the ambition to rule the world.
 Man is driven by the desire to make a name for himself.
 The play is too satiric to earn critical acclaim.
As adverb
 We must work to make this happen.
 These kids are anxious to learn. (adverb)
 He was pained to see the widespread destruction. (adverb)

GERUNDS

Gerunds are the V1-ing form of verbs functioning as nouns.


FORM: V1-ing
Retaining the V1-ing form, gerunds appear in specific constructions as under:
(i) Direct gerund
Direct gerunds are often preceded by
`` verbs such as consider, propose, practise, risk, regret, resist, prefer, enjoy,
love, like, dislike, hate, detest, start, finish, begin, stop, miss, etc.
Examples:
 I have requested them to consider using the new technology.
 Will you stop screaming like that, please?
 It is high time they began accepting the change.
`` phrases such as can’t (or couldn’t) help and it is no use/good
Examples:
 They couldn’t help praising her effusively.
 She can’t help lying through her teeth.
 It is no use crying over spilt milk.
 It is no good trying to reform this serial offender.
infinitives, gerunds and participles 15
`` verbs such as forgive, pardon, hate, like, miss and phrasal verbs, such as
don’t mind, fed up of, give up, in combination with a noun in the possessive
case( Ravi’s, my mother’s, boys’, etc.) or a possessive adjective (my, his,
her, their, etc.)
Examples:
 Teachers hate students’ speaking out of turn.
 Please forgive their trespassing for one last time.
 Nobody misses my singing as much as she does.
 He doesn’t mind his father’s scolding so much.
 I am fed up of his frowning and scowling.
 She seems to have given up her masquerading.
(ii) preposition + gerund
`` Gerunds mostly appear in this very construction usually with prepositions
other than ‘to’.
 Some phrases that precede gerunds are:
 aim at, abstain from, bent on, break free from, call for, confident of,
dissuade from, desirous of, end up, fond of, harm in, intent on, interested
in,
 keep on, prohibit from, refrain from, sick of, take pride in, wary of, yearn
for,
 zealous in
Examples:
 These aspirants are desirous of making a name for themselves.
 The delegation is wary of trying after a series of setbacks.
 There is no harm in accepting one’s mistake.
 They seem to be intent on proving him wrong.
 They yearned for having a glimpse of the star.
`` The preposition ‘for’ + gerund is used to convey reason/cause or purpose.
Use of to (i.e., infinitive form) in such cases is erroneous and must be avoided.
Examples:
Reason/cause :
 He is infamous for creating disturbance during office hours. (correct)
 He is infamous to create disturbance during office hours. (incorrect)
Purpose :
 This is for ensuring the safety of inmates. (correct)
 This is to ensure the safety of the inmates. (incorrect)
`` Gerunds follow the preposition ‘to’ only when they are preceded by phrasal
verbs containing to, such as given to, accustomed to, prone to, used to, boil
down to, in addition to, with a view to, look forward to, etc.
EBD_7275
16 infinitives, gerunds and participles
Examples:
 We were used to driving through the rough countryside.
 The poor are prone to contracting diseases.
 The guests looked forward to hosting the next get-together.
FUNCTION: As a Noun (subject of verb, object of verb, subjective complement
or object of preposition).
As subject of verb :
 Hiking in the hilly terrains is his favourite pastime.
 Smoking is injurious to health.
 Seeing eye to eye (gerund phrase) is a prerequisite for any association.
As object of verb :
 All the members of the council supported voting.
 Jaden wanted his parents to accept his dancing.
 She detested my gifting her friend a watch (gerund phrase).
Gerund as subjective complement :
 What the locality urgently needs is desilting of its drains (gerund phrase).
 Every employee knows performing as the success mantra (gerund
phrase).
Gerund as object of preposition:
 The law incriminates one for impersonaing.
 The accountant was neck deep in embezzling.
Distinguishing between infinitives and gerunds
 adjectives are mostly followed by infinitives rather than gerunds
 all prepositions (excluding but and except) are followed only by gerunds
 specific verbs are followed either by gerunds or, infinitives
 infinitives as well as gerunds can be subjects of verbs; only their forms
distinguish them, e.g.
To keep clean is a step closer to God.
Keeping clean is a step closer to God.
 objects (of verbs) referring to people(nouns/pronouns) are followed only
by infinitives

TIPS AND TECHNIQUES


 Identify the word preceding the verbal.
 If it is an adjective, an infinitive should follow.
 If it is a verb, recall which verbal follows that particular verb.
 If it is a simple verb with an agent of action, an infinitive should follow.
 If it is a preposition other than ‘but’ or ‘except’, a gerund should follow. In
case of ‘but’ or ‘except’, a direct infinitive should follow.
 If it is a phrasal verb with ‘to’, a gerund should follow.
 If it is a noun/pronoun as the object of a verb, an infinitive should follow.
infinitives, gerunds and participles 17
In some cases, a gerund is interchangeable with an infinitive
He works for providing smart solutions.
He works to provide smart solutions.

PARTICIPLES
Participles are the present participle verb forms (V1-ing); past participle verb forms
(V3); or perfect participle verb forms (having + V3); functioning as adjectives.
They appear as absolute participles or participial phrases. As phrases, they include
other words that modify the state or action expressed.
FORMS: (i) V1-ing [present participle]
Absolute participles :
Examples:
 The breaking news about the arrest of the celebrity took everyone by
surprise.
 She had known him since his singing days.
 All the brouhaha about their scathing remarks soon died down.
Phrase participles :
Examples:
 The hurricane, wreaking havoc in the city, struck the homeless the hardest.
 Strolling in the lawn, I saw a deadly cobra slipping away into the bushes.
 The mendicant, wandering unaware, was easily nabbed by the police.
(ii) V3 [past participle]
Absolute participles :
Examples:
 The broken window pane gave a glimpse of the penury inside.
 The need of the hour is an overhaul of the distorted system.
 Everyone present vouched for the uncorrupted innocence of the accused.
Phrase participles :
Examples:
 The minister ordered an enquiry, in reference to the destroyed network.
 Undeterred by his singed hands, he continued to rescue people from the
fire.
 He revealed, he had acted on instinct, unmindful of the given outcome.
(iii) Having + V3 [perfect participle]
 Perfect participles generally appear in phrases.
EBD_7275
18 infinitives, gerunds and participles
Phrase participles :
Examples:
 Having completed her work, she went out to play with her friends.
 The management acknowledged having released the festival bonus.
 He boasted of having won every match he had played till date.
FUNCTION: As Adjectives
As direct adjectives :
 As direct adjectives, participles qualify nouns/pronouns and are
replaceable by
adjective clauses.
Examples:
 The intruder sneaked into the house, dodging the perambulating
watchman.
(Here, the present participle perambulating qualifies the noun watchman;
and can be replaced with the adjective clause who was perambulating)
 He didn’t mind buying a used vehicle if it was in a working condition.
(Here, the past participle used qualifies the noun vehicle; and can be
replaced with the adjective clause which had been used)
As adverbial adjectives :
 As adverbial adjectives, participles qualify nouns/pronouns and are
replaceable by
 adjective clauses or adverb clauses.
Replaceable by adjective clause
Examples:
 He came across a farm girl carrying a bundle of hay.
 He came across a farm girl who was carrying a bundle of hay.
 Having played 200 matches, the cricketer announced retirement.
 The cricketer who had played 200 matches announced retirement.
Replaceable by adverb clause of time :
Example:
 Ferrying passengers across the river, the boatman would sing folksongs.
 As he ferried his passengers across the river, the boatman would sing
folksongs.
Replaceable by adverb clause of condition :
Examples:
 You may ruin your chances by doing this.
 You may ruin your chances if you do this.
 But for your saving the situation, we had lost it.
infinitives, gerunds and participles 19
 If you had not saved the situation, we had lost it.
 Without/not keeping a standby, one is likely to be left high and dry.
 If one doesn’t keep a standby, one is likely to be left high and dry.
Replaceable by adverb clause of reason :
Example:
 Being careless, they were in for a rebuke.
 They were in for a rebuke, as/since/for/because they were careless.
Replaceable by adverb clause of contrast :
Examples:
 Despite/in spite of showing potential, he was not considered.
 Even though/although/though he showed potential, he was not considered.
 For all/ notwithstanding its smart showcasing, the proposal was rejected.
 Even though/although/though it was showcased smartly, the proposal
was rejected.
Participles are also replaceable by coordinate clauses :
Examples:
 Everyone gathered there laughing merrily.
 Everyone gathered there and they were laughing merrily.

ERRONEOUS USE OF A DANGLING OR MISRELATED PARTICIPLE :


Participles must connect to their subjects. If a particle is without a subject
(dangling), or attached to the wrong subject (misrelated participle), it distorts
the meaning of the sentence.
Examples:
 Being a bright sunny day, we had a game of cricket. (incorrect)
 The participle being is dangling or is misrelated to ‘we’, the wrong
subject.
 Its subject is ‘a bright sunny day’; so, the pronoun ‘it’ should be used.
 It being a bright sunny day, we had a game of cricket. (correct)
 Taking a stroll in the garden, a snake bit him. (incorrect)
The participle taking is attached to ‘a snake’, the wrong subject.
Its subject is ‘he’.
 Taking a stroll in the garden, he was bitten by a snake. (correct)
 Bullying everyone, Rajiv’s classmates avoided him. (incorrect)
 The participle bullying is attached to ‘Rajiv’s classmates’, the wrong
subject.
 Its subject is Rajiv; so, the pronoun ‘his’ should be used.
 Bullying everyone, Rajiv was avoided by his classmates. (correct)
EBD_7275
20 infinitives, gerunds and participles
DISTINGUISHING BETWEEN PARTICIPLES AND GERUNDS
Both, participles and gerunds, have V1-ing form. They are distinguished by their
function.
Gerunds function as nouns, participles, as adjectives.
Examples:
 Organising is a difficult task. (Gerund)
 He was an organising genius. (Participle)
 He refrains from sleeping late into the day. (Gerund)
 She found her dog sleeping under the tree. (Participle)
 They were pulled up for creating trouble. (Gerund)
 Children love any kind of creating game. (Participle)

TIPS AND TECHNIQUES


 Identify whether the –ing verb is finite or non-finite ( a verbal).
 If it is not preceded by any form of the auxiliary ‘be’ (is/am/are/was/ were/
has been/have been/ had been), it is a verbal.
 Check the position of the verbal to identify its function.
 If it precedes a verb, it is the subject of the verb or a noun; therefore, a
gerund.
 If it follows a verb, it is the object of the verb or a noun; therefore, a gerund.
 If it follows a preposition, it is a noun; therefore, a gerund.
 If it precedes or follows an article, it is an adjective; therefore, a participle.
 If it precedes or follows a noun/pronoun, it is an adjective; therefore, a
participle.
 If it is part of an adverbial which can be replaced with a clause, it is a
participle.
Chapter

4 PARTS OF SPEECH

A part of speech is a linguistic category of words which explains how a word is


used in a sentence. It is also called lexical categories, grammatical categories or
word classes. There are eight parts of a speech viz. Noun, Pronoun, Adjective,
Verb, Adverb, Preposition, Conjunction and Interjection.
Noun: A noun is a naming word .i.e. it is used to name a person, place, thing,
quality or action. Examples: India, Rajeev, Tiger, Water and Sand, Happiness etc.
Usage in sentences:
 I live in Australia.
 The sand is hot.
 Philip is my elder brother.
 Gold is a precious metal.
 It makes me sad to see you looking so unhappy.
Pronoun: It is a word used in place of a noun or noun phrase to avoid repetition.
Example – I, You, We, She, It, They, Those and Either etc.
Usage in sentences:
 I am taller than he is.
 This is my notebook.
 Whom did you meet?
 The watch that I bought is very costly.
 Either of the two brothers is staying here.
Adjective: It is a word naming an attribute of a noun or adds to the meaning of a
noun.
Example – Sweet, Red, Heavy, Tall and Beautiful etc.
Usage in sentences:
 The box is too heavy to lift.
 This house is bigger than that one.
 She is the smartest in the class.
 She has five children.
 Skinny cats are not necessarily healthy.
EBD_7275
22 Parts of Speech
Verb: It is a word or phrase that describes an action, condition or experience.
Example – Run, Teach, Feel, Keep and Come etc.
Usage in sentence:
 He teaches in our college.
 Austin and Monika ride the bus to school every morning.
 He believes in fairies and unicorns.
 She is writing her exams.
 We saw the man with long hair waiting for the train.
Adverb: It is a word or a phrase that modifies the meaning of an adjective, verb,
or other adverb expressing manner, place, time, or degree. It is also used to
modify whole sentence. Example – Slowly, Wilfully, Lightly, Very, Randomly and
Truthfully etc.
Usage in sentence:
 He is very intelligent.
 The old lady was walking slowly.
 He returned immediately.
 She did the work wholeheartedly.
 She completely rejected his proposal.
 I sort of felt betrayed by you.
Preposition: It is a word or a group of words that is placed before a noun or a
pronoun to indicate, place, direction and method etc. Examples – In, On, Above,
Over, Under, To and Across etc.
Usage in sentence:
 The glass is on the table.
 The dog jumped over the wall.
 He is standing beside Manisha.
 At noon, I went to my dad’s office to surprise him.
 During the winter, I always spend my time playing video games.
Conjunction: It is a word used to connect clauses or sentences or to coordinate
words in the same cause. Examples – And, If, But, Or, Lest and That etc.
Usage in sentence:
 You must work or starve.
 My elder brother and my cousin have gone for hunting.
Parts of Speech 23
 He was annoyed that he was contradicted.
 I will call you after I arrive here.
 Many people like him because he is honest and hardworking.
 Go to the party but don’t fight with anyone.
Interjection: It is the word used to express emotions. An interjection is generally
punctuated with an exclamation mark. Example – Oh, Alas, Wait, Ah ha, Gosh and
Hurray, etc.
Usage in sentence:
 Wow! That was impressive.
 Oh, really? I don’t agree.
 Wait! Don’t throw the pen out of the window.
 Nice! You got the highest marks in the exam.
 Oh god! He again failed in the test.
EBD_7275
Chapter

5 ARTICLES

Articles are words used before only nouns to define their (nouns) uses in the
context of the sentence. It defines whether something is specific or unspecific.
There are only three types of articles namely, A, An and The. ‘A’ and ‘An’ are
Indefinite articles and ‘The’ is a Definite article. The choice to use A and An is
determined primarily by sound.
 ‘A’ is used before a word beginning with a consonant and a vowel giving
the sound of a consonant. Example – A boy, A child, A book, A university
student, A uniform and A one eyed man etc.
 ‘An’ is used before a word beginning with a vowel, with a mute ‘h’ and
a consonant pronounced with the sound of a vowel. Example – An ant,
An apple, An hour, An honest man, An M.L.A, An X-ray machine and
An L.L.B. student.
 ‘A’/‘An’ is used before a singular/countable common noun when it is
mentioned for the first time representing no particular person or thing.
Example – He needs a book, Twelve inches make a foot, A beggar is
standing outside the gate, Please, give me a pen, etc.
 ‘A’/‘An’ is used before a singular countable noun which is used to
single out some person/something as a representative of a class of thing,
animals and persons. Example – A cow is a useful animal. A son should
be obedient, etc.
 ‘A’/‘An’ is used before a person who is unknown to the speaker. Example
– There was a Mr. Sharma speaking on the topic in the seminar. A Miss
Singh was the chief guest.
 ‘A’/‘An’ is used before a verb used as nouns. Example – For a talk, For a
drive and Have a taste, etc.
 ‘A’/‘An’ is used before certain phrases like – In a hurry, Feel a shame,
Take a fancy, Make a noise, etc.
 ‘The’ is used when we talk of a particular person or a thing which is
already mentioned or known. Example – The person (who is mentioned
earlier) has not come yet. Let us study the chapter carefully, etc.
 ‘The’ is used‘ before proper nouns specifically, before the names of
historical buildings, planets, mountain ranges, rivers, gulfs, newspapers,
hotels, trains and oceans, etc. Example – The Indian ocean, The Taj
Mahal, The USA, The Sahara desert, The Moon, etc.
Articles 25

Note:
 There is no plural of ‘A’ and ‘An’. ‘Some’ or ‘Any’ is used in case a
plural is required. Example – Some cows/ any cows’ some apples/any
apples, etc.
 When indefinite articles i.e. ‘A’ and ‘An’ are used before proper nouns,
they become common nouns. For example – She is a shylock. (A miser)
 We use ‘The’ before a proper noun for the sake of comparison. For
example – Samudra Gupta was the Napoleon of India.
EBD_7275
Chapter
VOICES
6
Voice is the quality of a verb that indicates whether its subject acts (active voice)
or is acted upon (passive voice). Therefore, a verb is said to be an action on the part
of a doer/subject. The distinction between active and passive voice applies only on
transitive verb. Verbs are either active or passive in voice.
In Active Voice, the sentence begins with a subject.
Examples:
He will go to the restroom.
He has bought a plot in Mumbai.
We are going to watch a movie.
My mother read the novel in one day.
Two ministers are signing the treaty.
No one responded to her marketing advertisement.
The professor will give you instructions during the exam.
The team will celebrate its victory tomorrow.
The Kangaroo carried her baby in her pouch.
The actor posted the video on Facebook.

In Passive Voice, the sentence begins with the object.


Examples:
The restroom will be visited by him.
A plot has been bought by him in Mumbai.
A movie is going to be watched by us.
The novel was read by my mother in one day.
The treaty is being signed by two ministers.
Her marketing advertisement was not responded by anyone.
Instructions will be given to you by the professor during the exam.
The victory will be celebrated by the team tomorrow.
The baby was carried by the Kangaroo in her pouch.
The video was posted on Facebook by the actor.
voices 27
Important Rules:
 The place of subject and object are interchanged in the sentence.
 We use only the 3rd form of the verb or Past Participle as the main verb
in Passive Voice.
 When a sentence in Simple Present Tense is changed into passive voice
then ‘is’, ‘am’ or ‘are’ is used.

Examples:
Active Voice Passive Voice
She writes a letter. A letter is written by her.
Does he write a letter? Is a letter written by him?
They sell vegetables. Vegetables are sold by them.
My friend helps me. I am helped by my friend.

 When a sentence in Present Continuous Tense is changed into passive


voice then ‘is being’, ‘am being’ or ‘are being’ is used.

Examples:
Active Voice Passive Voice
She is writing a letter. A letter is being written by her.
They are eating bananas. Bananas are being eaten by them.
You are disturbing me. I am being disturbed by you.
My friend is helping me. I am being helped by my friend.

 When a sentence in Present Perfect Tense is changed into passive voice


then ‘has been’ or ‘have been’ is used.

Examples:
Active Voice Passive Voice
He has completed the work. The work has been completed by her.
She has cooked the food. The food has been cooked by her.
I have made some cookies. Some cookies have been made by me.
Has he done the work? Has the work been done by him?

 When a sentence in Simple Past Tense is changed into passive voice then
‘was’ or ‘were’ is used.
EBD_7275
28 voices
Examples:
Active Voice Passive Voice
She bought a car. A car was bought by her.
Did she buy a car? Was a car bought by her?
I helped them. They were helped by me.
He decorated the walls. The walls were decorated by him.
 When a sentence in Past Continuous Tense is changed into passive
voice then ‘was being’ or ‘were being’ is used.
Examples:
Active Voice Passive Voice
He was cleaning the floor. The floor was being cleaned by him.
Was she wearing a shirt? Was a shirt being worn by her?
Girls were singing songs. Songs were being sung by girls.
They were eating sweets. Sweets were being eaten by them.
 When a sentence in Past Perfect Tense is changed into passive voice
then ‘had been’ is used.
Examples:
Active Voice Passive Voice
They had won the game. The game had been won by them.
Had they won the game? Had the game been won by them?
He had collected donations. Donations had been collected by him.
They had not done the work. The work had not been done by them.

 When a sentence in Simple Future Tense is changed into passive voice


then ‘will be’ is used.
Examples:
Active Voice Passive Voice
She will write a poem. A poem will be written by her.
You will receive the letter. The letter will be received by you.
They will help the poor boy. The poor boy will be helped by them.
My wife will buy crockery. Crockery will be bought by my wife.
voices 29
 When a sentence in Future Perfect Tense is changed into passive voice
then ‘will have been’ is used.
Examples:
Active Voice Passive Voice
She will have received the letter. The letter will have been received by her.
She will not have received the The parcel will not have been received by
parcel. her.
Will he have delivered the money? Will the money have been delivered by
him?
He will have done the work. The work will have been done by him.
 The sentences of the following tenses can’t be changed to Passive
Voices:
• Present Perfect Continuous Tense
• Past Perfect Continuous Tense
• Future Perfect Continuous Tense
• Future Continuous Tense
• Sentences having Intransitive verbs.
EBD_7275
Chapter
NARRATION
7
The art of conveying the words of the speaker is called Narration. Narration is of
two types viz. Direct Speech and Indirect Speech.
(1) Direct Speech – It is a kind of speech which is conveyed by some other
person exactly in the words spoken by the speaker. In this form, the actual
words of the speaker are put in inverted commas.
Examples :
The President said, “We will become a developed nation in the coming ten
years.” In this example, The President is the conveyer/reporter, said is the
reporting verb and “We will become a developed nation in the coming ten
years.” is the reported speech.
(2) Indirect Speech – It is a kind of speech in which some other person
reports what the speaker said in his own words without quoting the exact
words i.e. the actual words of the speaker are transformed and said in a
simple manner by using certain conjunctions in place of commas and
making necessary changes in the verbs and the pronoun of the reported
speech.
Examples :
Direct speech – Monika said, “I am suffering from fever.
Indirect speech – Monika said that she was suffering from fever.
Important Rules to convert a Direct Speech to Indirect Speech :

RULES OF CHANGING DIRECT INTO INDIRECT SPEECH


Changes in Tenses : The past perfect and past perfect continuous tenses do not
change.
Direct Speech Indirect Speech

Simple Present Changes To "I always drink tea", He said that he always
Simple Past he said drank tea.
Present Continuous Changes To "I am reading a book", She said that she was
Past Continuous she said. reading a book.
Present Perfect Changes To Past She said, "He has She said that he had
Perfect finished his work" finished his work.
Narration 31

Present Perfect Continuous "I have been to He told me that he had


Changes To Past Perfect England", he told me. been to England.
Continuous
Simple Past Changes To Past "Bill arrived on He said that Bill had
Perfect Saturday", he said. arrived on Saturday.
Past Perfect Changes To Past "I had just come back He said that he had
Perfect (No Change In Tense) from work," he said. just come back from
work.
Past Continuous Changes To "We were living in They told us that they
Past Perfect Continuous Hong Kong", they had been living in
told us. Hong Kong.
Future Changes To Present "I will be in Italy on She said that she
Conditional Saturday", she said would be in Italy on
Saturday.
Future Continuous Changes To He said, "I'll be He said that he would
Conditional Continuous visiting mother next be visiting mother next
Monday." Monday.
"I will be in Italy on She said that she
Future Changes To Present Saturday", she said would be in Italy on
Conditional Saturday.
Future Continuous Changes To He said, "I'll be He said that he would
Conditional Continuous visiting mother next be visiting mother next
Monday." Monday.

Words expressing nearness in time or places are generally changed into words
expressing distance.

Direct Speech Indirect Speech

Here There
Today that day

Change of place this morning that morning


and time Yesterday the day before
Tomorrow the next day
next week the following week
next month the following month
EBD_7275
32 Narration

Direct Speech Indirect Speech


Now Then
Change of place
and time Ago Before
Thus So
Last Night the night before
This That
These Those
Hither Thither
Hence Thence
Come Go

CHANGES IN PRONOUNS
The pronouns of the Direct Speech are changed where necessary, according to their
relations with the reporter and his hearer, rather than with the original speaker.

Direct Speech Indirect Speech

The first person of the He says, "I am in fifth He says that he is in


reported speech changes class." fifth class.
according to the subject of
reporting speech.
The second person of He says to them, "You He tells them that they
reported speech changes have completed your have completed their
according to the object of job." job.
reporting speech.

The third person of the She says, "She is in ninth She says that she is in
reported speech doesn't class." ninth class.
change.

CHANGES IN MODALS
Direct Speech Indirect Speech

CAN changes into He said, "I can touch the He said that he could touch
COULD ceiling". the ceiling.
MAY changes into He said, "I may buy a He said that he might buy a
MIGHT house" house.
Narration 33

MUST changes into He said, "I must resign He said that he had to resign
HAD TO from the job" from the job.
These Modals Do Not Change: Would, could, might, should, ought to.
Would She said, "she would apply She said that she would apply
for a visa" for a visa.
Could He said, "I could climb the He said that he could climb
ladder." the ladder.
Might Tom said, "I might help Tom said that he might help
him". him.
Should She said, "I should go to She said that she should go to
the pub." the pub.
Ought to She said to me, "you ought She said to me that I ought to
to wait for her." wait for her.

CHANGES IN IMPERATIVE SENTENCES


Imperative sentences consist any of these four: Order, request, advice and suggestion.
Mood in Direct Speech Reporting verb in indirect verb
Order ordered
Request requested / entreated
Advice advised / urged
Never told, advised or forbade (No need of "not"
after "forbade")
Direction directed
Suggestion suggested to
Warning warned
(If a person is addressed directly) called

CHANGES IN EXCLAMATORY SENTENCES


Exclamatory sentences express emotions. Interjections such as Hurrah, wow, alas,
oh, ah are used to express emotions.
Rules of conversion of Exclamatory Direct Speech into Indirect Speech
1. Exclamatory sentence changes into assertive sentence.
2. Interjections are removed.
3. Exclamation mark changes into full stop.
4. Wh- words like ‘what’ and ‘how’ are removed and before the adjective of
reported speech we put ‘very.’
EBD_7275
34 Narration

Mood in Direct Speech Reporting verb in indirect verb


sorrow Exclaimed with sorrow/ grief/ exclaimed
sorrowfully or cried out
happiness exclaimed with joy/ delight/ exclaimed joyfully
surprise exclaimed with surprise/ wonder/ astonishment
appreciation applauded

RULES OF CONVERSION OF INTERROGATIVE DIRECT SPEECH

Changes Direct Speech Indirect Speech Condition


Reporting Verb said/ said to Asked, enquired or demanded.
Joining Clause If sentence begins with joining clause should be if or
auxiliary verb whether.
If sentence begins with no conjunction is used as
"wh-" questions "question-word" itself act as
joining clause.
Punctuation Question Mark Full Stop
Helping Verbs sentences is expressing do/does is removed from sentence.
positive feeling
if 'No' is used in do/does is changed into did.
interrogative sentences
Did or has/have Had
Chapter
KINDS OF SENTENCES
8 & CLAUSES

In the English language, there is considerable flexibility in sentence construction.


Using various sentence patterns produces speech and writing which interests the
reader. Variety in sentence construction also contributes to well-organized content.

ENGLISH SENTENCE CONSTRUCTION

(1) SIMPLE SENTENCE


A simple sentence has the most basic elements that make it a sentence: a
subject, a verb, and a completed thought.
Example:
She bought a big cake for her brother’s birthday. Rina waited for her sister.
Rishi and Saumya took the bus.
(2) COMPLEX SENTENCE
A complex sentence contains one independent clause and one or more
dependent clause. In a complex sentence, one idea is generally more important
than the other one and the more important idea is put in the independent clause
while the less important idea is put in the dependent clause.
Independent Clause + Dependent Clause
Dependent Clause + , + Independent Clause

Example:
If you are not good at English, it is pointless to apply for content writing.
When he saw the Mother coming, he sneaked out of the house. Shimla
which is very crowded in this month of the season is not worth visiting.
In all the above sentences, the independent clause is in bold.
IMPORTANT: There are three kinds of dependent clauses used in complex
sentences: adverb, adjective and noun.
1. A dependent adverb clause begins with an adverbial subordinator such as
when, while, because, even though, so that, etc.
2. A dependent adjective clause begins with a relative pronoun such as who,
whom, which, whose, or a relative adverb where, when, and why.
3. A dependent noun clause begins with that, a wh-question word, whether, and
if.
EBD_7275
36 Kinds of Sentences & clauses
(3) COMPOUND SENTENCE
The compound sentence contains two or more independent clauses but no
subordinate clauses. The two independent clauses are joined by a comma (,)
followed by a conjunction (for, and, nor, but…). They may also be joined by a
semicolon (;), a semicolon followed by a linking adverb (therefore, however,
because, since…), or a colon (:).
Example:
I don’t know where she lives, and no one knew her whereabouts. (conjunction)
Alexander the Great conquered the Achaemenid Empire of Persia; he began
his Indian campaign in 326 BC. (semicolon)
Joe wanted to stay with his mother; however, his father refused to keep him in
the house any more. (linking adverb)
You must have heard the news: nobody is getting bonus this year! (colon)

COMPOUND SENTENCES WITH COORDINATORS

Independent Clause + , + Coordinators + Independent Clause

The two independent clauses are joined by a comma and one of the seven
coordinating conjunctions: for, and, nor, but, or, yet, and so. The following sentences
illustrate their meanings.
Examples:
The Japanese have the longest life expectancy of any other people, for their
diet is extremely healthy. (for expresses reason)
Many Americans, on the other hand, do not take a healthy diet, nor do they
get enough exercise. (nor joins two equal negative independent clauses)

COMPOUND SENTENCES WITH LINKING ADVERBS (CONJUNCTIVE ADVERBS)

Independent Clause; + Conjunctive Adverb, + Independent Clause


The two independent clauses are joined by a semicolon (;), a conjunctive
adverb and a comma. Conjunctive adverbs express the relationship of the
second clause to the first clause.
Examples:
Technical colleges offer courses for skill development for professionals;
moreover, they prepare them to get a decent job in the industry. (equal related
ideas)
Kinds of Sentences & clauses 37
COMPOUND SENTENCES WITH SEMICOLON

Independent Clause + ; + Independent Clause


The two independent clauses are joined by a semicolon (;). Use a semicolon
only when the two independent clauses are closely related and the relationship is
implied.
Examples:
Her older sister studies medicines; her younger sister studies philosophy.
The Berlin Wall’s construction in 1961 surprised the world; its destruction in
1989 stunned it.
(4) COMPOUND-COMPLEX SENTENCES
A compound-complex sentence is a combination of two or more independent
clauses and one or more dependent clauses.
Examples:
After she finished college, she wanted to work in family’s business, but her
parents sent her abroad for further study.
When the light went out, Hina was writing the essay, and Rishi was watching
TV.
Types of Sentences
There are four types of sentences:
(1) Declarative Sentences: Sentences that make statements. Examples: Rishi and
Kavya are neighbours. She has one brother and one sister. We are going to
cinema tonight.
(2) Interrogative Sentences: Sentences that ask questions. Examples: Where are
you going? Where will you go in summer holidays? Can you show me your
passport?
(3) Imperative Sentences: Sentences that give commands or make requests.
Examples: Get lost! Mind your business! Go and do something.
(4) Exclamatory Sentences: Sentences that are in the form of exclamations.
Examples: What a good idea! How wonderful! How pretty the kitten is!
EBD_7275
Chapter

9 PHRASAL VERBS

A phrasal verb is an idiomatic phrase consisting of a verb and other elements,


either an adverb or a preposition. Typically, their meaning is not obvious from the
meaning of the individual words themselves. For example – blow up, abide by,
call off, put on, etc.
EXAMPLES OF USAGE OF PHRASAL VERBS IN SENTENCES:
Take off ( to leave the ground): The plane will take off at 1 P.M.
Hand out ( to distribute): He volunteered at the shelter where he handed out
blankets.
Back up ( to give support or encouragement, make to make a copy of ): The
rest of the students backed her up when she complained about the teacher to the
principal.
Don’t forget to back up your data before you format your phone.
Get away ( to escape): The thieves got away in a stolen car.
Hurry up ( to be quick or act speedily): Hurry up, guys, we have to complete the
work before midnight.

BELOW IS A LIST OF SOME VERY IMPORTANT PHRASAL VERBS WITH


THEIR MEANINGS:
 Ache for (want something or someone a lot)
 Auction off (sell something in an auction)
 Back away (to move backwards in fear or dislike)
 Barge in/ into (enter a place and interrupt)
 Be onto (pursue, be aware of someone’s true nature)
 Beaver away (work hard)
 Bliss out (to be extremely relaxed and happy)
 Bolster up (give support, reinforce or strengthen)
 Bone up (study hard for a reason)
 Bring forth (produce something or make something known or visible)
 Buoy up (make something feel more positive)
 Butter up (praise or flatter someone excessively)
 Chicken out (be too afraid to do something)
 Chip away at (gradually reduce something to make it less powerful and
effective etc.)
 Chow down (eat)
 Clamp down on (restrict or try to stop something/ to act strictly to prevent
something)
 Clock up (win, score or achieve results/ ruin or spoil something)
Phrasal Verbs 39
 Conjure up (create a picture or memory in someone’s mind)
 Dash off (leave somewhere quickly)
 Die down (to calm down or become less strong)
 Divvy out (divide or share)
 Dole out (give out or distribute)
 Doze off (fall asleep)
 Drag on (to last longer than expected)
 Drone on (talk boringly for a long time)
 Ease off (to reduce, become less severe or slow down)
 Ebb away (disappear gradually)
 Fend for oneself (take care of oneself without help from other people)
 Ferret about (search for something)
 Gloss over (try to minimize the importance of something)
 Gnaw at (trouble, worry or annoy someone)
 Hive off (separate part of a company or service, often by selling it)
 Iron out (to resolve by discussion, to eliminate differences)
 Jockey into (persuade or deceive someone into doing something)
 Keel over (turn upside down)
 Naff off (get lost or go away - used as imperative)
 Nag at (repeatedly criticize someone verbally)
 Nod off (to fall asleep)
 Nut out (find an answer to a problem)
 Own up (to admit or confess something)
 Peck at (eat very small amounts)
 Pootle along (travel in a leisurely way)
 Psych up (prepare someone mentally)
 Queer up (mess up or ruin)
 Sally forth (leave somewhere safe or comfortable)
 Scuzz up (spill, ruin or contaminate)
 Slimmer down (become calmer or make less noise)
 Snarl up (entangle)
 Stick up for (to defend)
 Stomp off (leave somewhere angrily)
 Trot off (leave)
 Trudge through (do something slowly, with difficulty, unwillingly or
considerable effort)
 Use up (To finish a product)
 Usher in (be at mark or celebrate an important point in time)
 Vamp up (make something more exciting and attractive etc.)
 Wade into (become embroiled or involved in a situation without thinking or
planning usually)
 Wear out (to become unusual, to become very tired)
 Wiggle out (avoid doing)
 Yammer on (talk continuously, especially if it is an annoying way)
 Zero in on (direct or focus attention on)
EBD_7275
40 Phrasal Verbs

IMPORTANT TIPS AND TECHNIQUES FOR PHRASAL VERB

 Frankly stating, there is no short-cut trick to solve a question based on


phrasal verb. Learning them and regular reading habits can help you get
good marks in this section.
 Keep on learning phrasal verbs and once you learn a phrasal verb, try to
use it often while speaking or writing so that you don’t forget its usage and
meaning. Do it with every phrasal verb that you learn or mug up.
 Whenever you are reading or listening in English, take a note of any
interesting and new phrasal verb that you come across. This helps you
remember it easier and stick in your memory for a longer time.
 Be aware that one of the special features of phrasal verbs is that some of them
have many different meanings. For example, you can say pick something
up from the floor; you can pick up a language or bad habits; the weather
can pick up; you can pick up a bargain; a radio can pick up a signal; the
economy can pick up. Sometimes the meaning is clearly related, at others
more literal and metaphorical.
Chapter

10 QUESTION TAGS

A question tag is a short question at the end of a statement, mostly while speaking.
Question tags are used when asking for agreement or confirmation. For example
– The weather is nice outside, isn’t it?, You are coming from the market, aren’t
you?, He didn’t steal the money, did he? and You remembered to feed the dog,
didn’t you? etc.
They are formed with the auxiliary or modal verb from the statement and the
appropriate subject. It should be noted that usually, a positive sentence is
followed by a negative question tag and vice-versa.
Examples:

POSITIVE SENTENCES, WITH NEGATIVE QUESTION TAGS

Present simple ‘be’ She’s Italian, isn’t she?


Present simple other verbs They live in London, don’t they?
Present continuous We’re working tomorrow, aren’t we?
Past simple ‘be’ It was cold yesterday, wasn’t it?
Past simple other verbs He went to the party last night, didn’t he?
Past continuous We were waiting at the station, weren’t we?
Present perfect They’ve been to Japan, haven’t they?
Present perfect continuous She’s been studying a lot recently, hasn’t she?
Past perfect He had forgotten his wallet, hadn’t he?
Past perfect continuous We’d been working, hadn’t we?
Future simple She’ll come at six, won’t she?
Future continuous They’ll be arriving soon, won’t they?
Future perfect They’ll have finished before nine, won’t they?
Future perfect continuous She’ll have been cooking all day, won’t she?
Modals He can help, can’t he?
Modals John must stay, mustn’t he?
EBD_7275
42 Question Tags
NEGATIVE SENTENCES, WITH POSITIVE QUESTION TAGS

Present simple ‘be’ We aren’t late, are we?


Present simple other verbs She doesn’t have any children, does she?
Present continuous The bus isn’t coming, is it?
Past simple ‘be’ She wasn’t at home yesterday, was she?
Past simple other verbs They didn’t go out last Sunday, did they?
Past continuous You weren’t sleeping, were you?
Present perfect She hasn’t eaten all the cake, has she?
Present perfect continuous He hasn’t been running in this weather, has he?
Past perfect We hadn’t been to London before, had we?
Past perfect continuous You hadn’t been sleeping, had you?
Future simple They won’t be late, will they?
Future continuous He’ll be studying tonight, won’t he?
Future perfect She won’t have left work before six, will she?
Future perfect continuous He won’t have been travelling all day, will he?
Modals She can’t speak Arabic, can she?
Modals They mustn’t come early, must they?

Exceptions (Some verbs/expressions have different question tags):


 Question tag after ‘I am’ is ‘aren’t I?’

Example :
I am in charge of the department, aren’t I?
 Positive imperative – Stop daydreaming, will you/won’t you?
 Negative imperative – Don’t stop singing, will you?
 “Let’s” – Let’s go to the party, shall we?

TIPS AND TECHNIQUES FOR QUESTION TAGS


 Usually if the main clause in positive, the question tag is negative and if
the main clause is negative, it is positive.
 If the main clause has an auxiliary verb in it, you use the same verb in the
tag question. If there is no auxiliary verb (in the present simple and past
simple) use do/does/did (just like when you make a normal question).
Chapter
IDIOMS AND PHRASES
11
Idioms and Phrases are two useful elements of linguistics which are often consid-
ered similar to each other and most of the time used interchangeably. However,
they are different based on their meaning and readability.

A phrase is a small group of words standing together as a conceptual unit, while


an idiom (also called idiomatic expression) is an expression, word, or phrase
that has a figurative meaning conventionally understood by native speakers. This
meaning is different from the literal meaning of the idiom’s individual elements. In
other words, idioms don’t mean exactly what the words say. They have, however,
hidden meaning. So, it can be said that any idiom is a phrase but all phrases are not
necessarily idioms. Example – ‘Raining cats and dogs’ is both idiom and phrase.
‘A herd of dogs’ is a phrase but not an idiom.

Examples of usage of Idioms and Phrases:

¾¾ On the tip of my tongue which means you are almost able to remember
something, but you can’t.
Her name was on the tip of my tongue, but I couldn’t remember it.

¾¾ A piece of cake which means very easy.


The exam was a piece of cake; I finished it in 25 minutes.
¾¾ Fool’s paradise which means a state of happiness based on a person’s not
knowing about or denying the existence of potential trouble.
He was living in a fool’s paradise, refusing to accept that he was in huge
debt.
¾¾ Do you know what my friend said? He’s an audio expert and he said that
we need to improve it. I don’t know, what do you think?
¾¾ Bring up which means raise children and mention a topic.
Nowadays, it is very difficult to bring up children.
The matter was brought up to the authorities by the gatekeeper.
EBD_7275
44 Idioms and Phrases
Given below are two lists of important Idioms and Phrases with
their meanings:

IDIOMS MEANING
A hot potato Speak of a current issue which many people are
talking about and which is usually disputed
A penny for your A way of asking what someone is thinking
thoughts
Add insult to injury To further a loss with mockery or indignity; to worsen
an unfavourable situation
A dime a dozen Something common
At the drop of hat Without any hesitation; instantly.
Back to the drawing When an attempt fails and it’s time to start all over.
board
Be glad to see the back Be happy when a person leaves.
of
Beat around the bush Avoiding the main topic
Best thing since sliced A good invention; a good idea or plan.
bread
Blessing in disguise Something good that isn’t recognized at first.
Break a leg Good luck
Caught between two When someone finds it difficult to choose between
stools two alternatives.
Cross that bridge when Deal with a problem if and when it becomes
you come to it necessary, not before.
Curiosity killed the cat Being inquisitive can lead you into an unpleasant
situation.
Cut corners When something is done badly to save money.
Cut the mustard To succeed; to come up to expectations; adequate
enough to compete or participate.
Cut somebody some Don’t be so critical.
slack
Devil’s advocate To present a counter argument.
Don’t give up the day You are not very good at something; You could
job definitely not do it professionally.
Idioms and Phrases 45

Elvis has left the The show has come to an end; it’s all over.
building
Far cry from Very different from
Feel a bit under the Feeling slightly ill.
weather
Get your act together Work better or leave
Hear it on the grapevine To hear rumours about something or someone.
Hit the nail on the head Do or say something exactly right.
Hit the sack/sheets/hay To go to bed.
It takes two to tango Actions or communications need more than one
person.
Jump on the bandwagon Join a popular trend or activity.
Last straw The final problem in a series of problems.
Method to my madness An assertion that, despite one’s approach seeming
random, there actually is structure to it.
Off one’s rocker Crazy, demented, out of one’s mind, in a confused or
befuddled state of mind, senile.
On the wall When someone understands the situation well.
Picture paints a A visual presentation is far more descriptive than
thousand words words.
Put wool over other To deceive someone into thinking well of them.
people’s eyes
See eye to eye To say that two or more people agree on something.
Sit on the fence When someone does not want to choose or make a
decision.
Speak of the devil When the person you have just been talking about
arrives.
Steal someone’s To take credit for something someone else did.
thunder
Take with a grain of salt This means not to take what someone says too
seriously.
Whole nine yards Everything
Wouldn’t be caught Would never like to do something
dead
Your guess is as good To have no idea, do not know the answer to a question.
as mine
EBD_7275
46 Idioms and Phrases

PHRASES MEANING
Add up Add up in number or quantity, To make sense, seem
reasonable
Beef up To make changes or an improvement.
Black out To faint, lose consciousness
Blow up Explode
Boil down to To be summarized as
Break in on Interrupt (a conversation)
Catch on Become popular
Chicken out To refrain from doing something because of fear.
Come about When something happens or occurs
Come over To visit
Do over Repeat a job; To do something again in order to
improve or correct mistakes.
Doze off To go to sleep unintentionally
Drop by Visit without appointment
Fall behind To move slower than others
Flip out To become very mad or lose control over your
emotions.
Get along with Have a good relationship with; To have good
interactions with others.
Get by survive; To pass someone or something
Goof around To waste time doing silly or unimportant things.
Gross out To be disgusted with someone or something.
Hit on To suddenly have a solution to a problem or an
interesting idea.
Look over Examine, check
Look up to Respect; To admire
Monkey around with To try to play with or repair a device that you have no
true knowledge about.
Narrow down To reduce the number of options or possibilities.
Idioms and Phrases 47

Nod off To fall asleep


Pet up to To encourage or persuade someone to do something.
Put past Not to be surprised by a person’s actions. (Always
negatively)
Put up with Tolerate
Sneak in/into To enter a place quietly to avoid being seen or heard.
Stick up for To defend
Walk out on Abandon
Watch out To be aware of someone or something.
Wrap up To cover something or end something

IMPORTANT TIPS AND TECHNIQUES FOR IDIOMS AND PHRASES

¾¾ There is no short-cut trick to solve a question based on idioms and phrases.


Learning them and regular reading habits are the only options that will help
you get good marks in this section.
¾¾ Keep learning idioms and phrases and try to use them often while speaking
or writing so that you don’t forget their usage and meaning. Do it with every
idiom/phrase that you learn or mug up.
¾¾ Whenever you are reading or listening in English and come across any
interesting and new idiom/phrase, make a note of it. This helps you remember
them easily and stick in your memory.
EBD_7275
Chapter
SENTENCE
12 STRUCTURE
Sentences are the basic building blocks of our language. We use them to converse.
And correct conversation means correct sentence usage. Thus, we need to under-
stand different sentence structure so as to avoid grammatical errors and converse
and write better. Understanding the sentence structures is essential as a lot of rules
are based on them. Competitive exams always include questions on common er-
rors/ spotting errors in sentences.
There are 4 sentence structures. But before we proceed, let us recall what clauses
are. A clause is a group of words that contains at least a subject and a verb. A
clause may form part of a sentence or it may be a complete sentence in itself. For
example -
I have a pen made of plastic.
Here “I” is the subject and “have” is the verb.
There are two types of clauses: Independent clause and Dependent clause. For
example:
When I was in Agra, I visited many famous tourist places.
The part before the comma is incomplete without the second part. If we pick
only this part i.e. “When I was in Agra” it does not tell what happened when the
author was in Agra. Thus it is DEPENDENT on the next part and hence is called
Dependent Clause
Now look at the second part. It makes complete sense without the first one. Thus it
is INDEPENDENT in existence of the previous part. This means without the pre-
vious part, it is a complete sentence in itself. Thus, it is called Independent Clause.
IMPORTANT: In order to be a complete sentence, a group of words should
contain a subject and a verb, and should state a complete thought. Now, let us
understand the 4 sentence structures.
1. Simple Sentence: A simple sentence has one independent clause that has a
subject and a verb and states a complete thought.
I won a race.
She gifted me a watch.
Deepavali is celebrated across India.
2. Compound Sentence: A compound sentence contains at least two indepen-
dent clauses joined by a coordinating conjunction or a semicolon. A coordinating
conjunction joins entities of equal ranks such as words, phrases, or clauses. [Most
used coordinating conjunctions are F = for, A = and, N = nor, B = but, O = or, Y
= yet, S = so]
Sentence Structure 49
It was raining outside, yet the children wanted to play outside.
She likes to go to parties, but he likes to attend conferences.
Ramesh is a good boy, but his brother is extremely cunning.
3. Complex Sentence: A complex sentence contains an independent clause and
a dependent clause or a phrase joined by subordinating conjunctions. A complex
sentence establishes a relation such as cause and effect, sequence, or outcome rela-
tion between the independent clause and the dependent clause.
She is studying because she has an exam tomorrow. [The clause in boldface is
dependent clause as it shows the reason for the first clause]
While I was playing the piano, my wife was reading a book.[The clause in bold-
face is dependent clause as it shows the reason for the first clause]
4. Compound-Complex Sentence: A Compound-complex sentence contains at
least two independent clauses and at least one subordinate clause. The independent
clause is joined by coordinating conjunction to a complex sentence [combination
of independent and dependent clause]
I was watching my favourite show, but when I heard my mother coming, I
quickly switched it off.
[Notice that the boldface part is a complex sentence and is joined to an indepen-
dent clause]
The principal wanted to scold the students, but the teacher did not, after they
apologized sincerely. [Notice that the boldface part is a complex sentence and is
joined to an independent clause]

TRANSFORMATION OF SENTENCES

Sentences can be classified into affirmative, negative, imperative, interrogative,


assertive (declarative) and exclamatory sentences. They can be transformed into
different forms without changing the meaning of the sentence. This process is
known as transformation of sentences.

1. Affirmative Sentence
Affirmative sentence means the sentences which are used to describe any general
action, event, speech, or expression.

Examples:
All students have done their homework today.
The train is always on time.
EBD_7275
50 Sentence Structure
2. Assertive or Declarative Sentence
A sentence that makes a statement or assertion is called an assertive or declarative
sentence. Assertive sentence ends with a period.

Examples:
She goes to market. The baby likes to play with toys. They are singing a song.
3. Interrogative Sentence
A sentence that asks a question is called an interrogative sentence. Interrogative sentence
ends with question mark.

Examples:
What are you doing? Do you use your car everyday?
4. Imperative Sentence
A sentence that expresses a request, command or advice is called an imperative sentence.

Examples:
Turn off the light. (an order); Please do me a favour. (a request).
5. Exclamatory Sentence
A sentence that expresses strong feelings or emotions is called an exclamatory sentence.
These sentences express surprise, joy, sorrow, appreciation, love excitement, frustration,
anger etc. An exclamatory sentence ends with exclamation mark.

Examples :
What a beautiful picture it is! How nicely she is dancing! That is wonderful!
Hurrah! We have won the debate!
Transformation Rules - Affirmative to Negative Sentence

Affirmative Negative
S.No. Sentence Sentence Tips Example
structure structure
1. Subject Subject If there is always in an Affirmative: He
+always + + never affirmative sentence, never is always on
verb + ext. opposite verb will be used for ever/always time.
+ ext. in negative sentence and Negative:- He
OR verb /adjective/adverb will was never
OR be opposite meaning. bothered about
Subject + such incidents.
auxiliary Subject +
verb + auxiliary verb
always + never +
+ verb/ opposite verb/
adjective/ adjective/
adverb + ext. adverb + ext.
Sentence Structure 51

2. Subject + Subject + If there is present tense Affirmative:-He


verb + too + verb + so + before too, ‘can’ will be is too weak to
adjective + adjective + used after that and if there walk.
to + verb + that + subject is past tense before too, Negative:- He is
ext. + can/ could ‘could’ will be used after so weak that he
+ not + verb that. Again when there is for cannot walk.
+ ext. + noun/ pronoun after too+
adjective, the noun/ pronoun
after for will be the subject
after that.
3. Subject + Subject + In negative sentence as -as is Affirmative: She
verb + as + auxiliary replaced by not less-than. is as tall as her
adjective + verb + not + sister.
as + noun/ verb + less + Negative: She is
pronoun. adjective + not less tall than
than + noun/ her sister.
pronoun.
4. Subject + Subject + To change an affirmative Affirmative:- I
auxiliary auxiliary sentence having an auxiliary shall remember
verb + verb/ verb + not + verb with a verb/ adjective/ those lovely
adjective/ opposite verb adverb into a negative days.
adverb + ext. / adjective / sentence, add not after the Negative:-I shall
adverb + ext. auxiliary verb and use the not forget those
opposite meaning of verb/ lovely days.
adjective/ adverb in negative
sentence.
5. Subject + Subject + If no auxiliary verb in an Affirmative:-He
verb + ext. auxiliary verb affirmative sentence, to do remembered
to do + not + verb will be used as auxiliary her.
opposite verb verb to make it a negative Negative:-He
+ ext. sentence. After to do verb did not forget
not will be added along with her.
the opposite verb. This to
do verb will be according
to the tense of the verb of
affirmative sentence.
6. Subject + Subject + To change a negative Affirmative:-
must + verb cannot but + sentence into an affirmative We must assist
+ ext. verb + ext. sentence having must, use them financially.
cannot but/cannot help Negative:-We
Subject + for must. In this case, basic cannot but assist
cannot help + form of verb will be used them financially.
verb-ing + ext after cannot but and present
We cannot help
participle (verb-ing) will be
assisting them
used after cannot help.
financially.
EBD_7275
52 Sentence Structure

7. Every + There is no + To change an affirmative Affirmative:


noun/ body/ noun/ body/ sentence having every + Everybody hates
one + verb + one + but + noun/ body/ one into a a bigmouth.
ext. verb + ext negative sentence, we can Negative:-
use there is no for every, then There is no
we have to put the word after one but hates a
every and next we should bigmouth.
use but before verb+ ext.
8. As soon as No sooner To change an affirmative Affirmative:-
+ subject + had + subject sentence having as soon as As soon as he
verb (past), + verb(past with two clauses, no sooner reached the
subject + participle) + had will be used for as soon station, the train
verb(past) + than + subject as in negative sentence. started.
ext. + verb(past) Than must be used between Negative:- No
+ ext. the two clauses. sooner had he
reached the
station than the
rain started.
9. Only + None but + Here only is used before a Affirmative:-
subject + subject + verb person, but alone is used Only God can
verb + ext. + ext. after the person word and help us.
auxiliary verb. To change Negative:- None
an affirmative sentence into but God can
a negative sentence where help us.
only is before a person or
alone is after the person and
auxiliary verb, we have to
start the negative sentence
with none but for only/
alone.
10. Subject + None but When the subjective word Affirmative:-
auxiliary + subject + is a person word i.e he/ she/ He was alone
verb + alone auxiliary verb I/ you/ they/ we/ any proper arrested in the
+ ext. + ext. noun. case.
Negative:- None
but he was
arrested in the
case.
11. Only + Nothing but + When only is used before Affirmative:-
subject + subject + verb an object/thing, nothing Only the US
verb + ext. + ext. but will be put to make a has the power to
negative sentence from an veto the verdict.
OR OR affirmative sentence. Negative:- None
Nothing but is used in but the US has
Subject + Subject negative sentence for only the power to
verb + only + verb + when there is an object/thing veto the verdict.
+ object + nothing but + after only in affirmative
ext. object + ext. sentence.
Sentence Structure 53

12. Subject + Subject When only is used before Affirmative:- I


verb + only + verb + number/ age, not more / am only 25 year
+ number/ not more / less than will be used in old.
age + ext. less than + negative sentence for only in Negative:- I am
number/age affirmative sentence. not more/less
+ ext. than 25 years
old.

Transformation Rules - Assertive to Interrogative Sentence


Assertive Interrogative
S.no Sentence Sentence Tips Example
structure structure
1 Subject + auxiliary verb When an auxiliary verb in the Assertive:- We shall
auxiliary + n’t + subject assertive sentence, n’t added go to the picnic.
verb + ext. + ext.? and placed before the subject in Interrogative:-Shan’t
the interrogative sentence we go to the picnic?
2 Subject + auxiliary verb When the assertive sentence Assertive:- We can
auxiliary + subject + is a negative sentence, the not concentrate on the
verb + not ext? negative word will be deleted subject.
+ ext. in interrogative sentence and Interrogative:-Can’t
then it will be started with only we concentrate on the
auxiliary verb. subject?
3 Subject + to do + n’t + When there is no auxiliary Assertive:- He
verb + ext. subject + verb verb in the assertive sentence/ watches the TV.
+ ext? affirmative sentence, to change Interrogative:-
it into interrogative sentence Doesn’t he watch the
to do verb will be used as an TV?
auxiliary verb and n’t will
be added after the to do verb
and to do with n’t will be
placed before the subject in the
interrogative sentence.
4 Subject + to do verb + When there is ‘never’ used in Assertive:- I never
never + subject + ever the assertive sentence, to make play chess.
verb + ext. + verb + ext.? an interrogative sentence to do Interrogative:-Do I
verb will be used as Question ever play chess?
word and ‘never’ is replaced by
‘ever’.
5 subject + auxiliary verb When there is ‘nothing’ used Assertive:- I have
auxiliary + subject in the assertive sentence, to nothing to do with it.
verb + + verb + make it interrogative sentence, Interrogative:- Do I
verb + anything + sentence starts with auxiliary have anything to do
nothing + ext.? verb and ‘nothing’ is replaced with it?
ext. by ‘anything’.
EBD_7275
54 Sentence Structure
When there is everybody/all/ Assertive:-
everybody/ everyone used in the assertive
Who + to do Everybody condemns
all/ sentence, to make it interrogative
verb + n’t + a thief.
6 everyone + sentence, everybody/all/everyone
verb + ext.? Interrogative:-Who
verb + ext. is replaced by ‘who + to do
verb’ and n’t will have to add doesn’t condemn a
after the to do verb. thief?
When there is ‘Nobody’ used
in the assertive sentence, to Assertive:- Nobody
Nobody + Who +
make it interrogative sentence, can reach the answer
auxiliary auxiliary
‘Nobody is replaced by ‘Who’. accurately.
verb verb+ verb +
+verb+ ext.? When there is ‘Nobody’ used Interrogative:-Who
7
ext. in the assertive sentence, to can reach the answer
auxiliary verb accurately?
make it interrogative sentence,
+ anybody +
sentence starts with auxiliary Can anybody reach
verb+ ext.?
verb and ‘nobody’ is replaced the answer accurately?
by ‘anybody’
When there is ‘none/no one
used in the assertive sentence, to
none/ who + make it interrogative sentence, Assertive:- None/
no one + auxiliary ‘none/ no one’ is replaced by No one can touch the
auxiliary verb+ verb + ‘who’. ceiling.
8 verb+ verb ext.?
+ ext. When there is none/no one used Interrogative:-Who
auxiliary verb in the assertive sentence, to can touch the ceiling?
+ anyone + make it interrogative sentence, Can any one touch the
ext.? sentence starts with auxiliary ceiling?
verb and ‘none/no’ one is
replaced by ‘anyone’.

Transformation Rules - Assertive to Imperative Sentence


Assertive Imperative
S.no Sentence Sentence Tips Example
structure structure
Assertive:- You
You + auxiliary Verb + ext. When the assertive sentence consists should exercise
verb + verb + auxiliary verb and no negative word daily.
1 ext. then to make it imperative remove
subject and auxiliary verb from Imperative:-
sentence. Do the exercise
daily.
When the assertive sentence consists Assertive:-You
you +auxiliary Do + not + auxiliary verb and negative word do not roam
verb + not + verb + ext. ‘not’ then to make it imperative around.
2 verb + ext. remove subject and auxiliary verb Imperative:-
from sentence and start sentence Do not roam
with ‘Do Not’. around.
Sentence Structure 55
When the assertive sentence consists Assertive:-You
of auxiliary verb ‘should’ and should never
you + should
Never + negative word ‘never’ , to make it tell a lie.
3 + never + verb
verb + ext. imperative remove subject and aux-
+ ext. Imperative:-
iliary verb from sentence and start
sentence with ‘Never’ Never tell a lie.
When the assertive sentence consists Assertive:-She
I/We/He/She/ Let + me/
of subject other then ‘you’ and no sings a song.
Noun + verb us/ him/
auxiliary verb then to make it imper-
4 + ext. her/ them/ Imperative:-
ative use object form of pronoun if
noun + verb Let her sing a
subject is pronoun and start sentence
+ ext. song.
with ‘Let’
If the assertive sentence consist of
Let me/ us/ pronoun as subject other then ‘you’ Assertive:-We
I/we/he/she/
him/ her/ and auxiliary verb and also negative do not do it.
they/ + auxil-
5 them + not word ‘not’ then to make it impera-
iary verb + not Imperative:-
+ verb + tive use object form of pronoun and
+ verb + ext. Let us not do it
ext. start sentence with ‘Let’ and place
‘not’ after subject.
Assertive:-
When the assertive sentence consists John does not
Noun + auxil- Let not + of noun as subject and auxiliary verb go there.
6 iary verb + not noun + verb and also negative word ‘not’ then
+ verb + ext. + ext. to make it imperative start sentence Impera-
with ‘Let not’. tive:- Let not
John go there.
Transformation Rules - Assertive to Exclamatory Sentence
Assertive Exclamatory
Sr.no Sentence Sentence struc- Tips Example
structure ture
Assertive:- It is
In exclamatory sentence, a a very interesting
Subject + verb What + a/ an +
very is replaced by what +a/movie.
+ a + very adjective/ adverb
1 an and these are used after
+ adjective/ + ext. + subject + Exclamatory:-
verb and before adjective/
adverb + ext. verb! What an interest-
adverb.
ing movie it is!
Subject + verb Assertive:- The
how + adjective/ In exclamatory sentence , picture looks
+ very + adjec-
adverb + ext. + very is replaced by how and very nice.
2 tive/ adverb +
subject + verb! it is used after verb. Exclamatory:-
ext.
How nice the
picture looks!
Assertive:- I
Subject + wish
If + subject + For subject + wish, if is used wish I had
+ subject + walked on the
verb + ext.! in exclamatory sentence.
3 verb + ext. moon.
Exclamatory:-If
I had walked on
the moon!
EBD_7275
56 Sentence Structure
Assertive:-We
wish we could
Subject + wish would that + For subject + wish + could,
subject + could + if /would that is used in live in that man-
+ subject +
sion.
4 could + verb verb + ext! exclamatory sentence
+ ext. Exclamatory:-
Would that we
could live in that
mansion!
Transformation Rules - Exclamatory to Assertive Sentence
Exclama- Assertive
Sr.no tory Sentence Sentence Tips Example
structure structure
1 what + a/an + Subject + verb In assertive sentence, what Exclamatory:-
adjective/ ad- + a + very + a/an is replaced by very What a good chef
verb + subject + adjective/ and these are used after he is!
+ verb! adverb. verb and before adjective/ Assertive:- He is a
adverb. good chef.
2 How + adjec- Subject + verb In assertive sentence, how Exclamatory:-
tive/ adverb + + very + adjec- is replaced by very and it is How nicely you
subject + verb! tive/ adverb. used after verb. arranged the meet-
ing!
Assertive:-You ar-
ranged the meeting
very nicely.
3 Hurrah! It is a matter Hurrah is replaced by it is a Exclamatory:-
Subject + verb of joy that + matter of joy Hurrah! we have
+ ext. subject + verb won the debate.
+ ext. Assertive:- It is a
matter of joy that
we have won the
debate.
4 Alas! Subject It is a matter of Alas is replaced by it is a Exclamatory:-
+ verb + ext. sorrow that + matter of sorrow. Alas! I am disap-
subject + verb pointed.
+ ext. Assertive:-It is a
matter of sorrow
that I am disap-
pointed.
5 If + subject Subject + For ‘If were/verb(past)’ Exclamatory:-
+ were/ wish + subject , subject + wish + were/ If I danced with
verb(past) + + were/ verb(past) is used in asser- Amanda!
ext.! verb(past) + tive sentence. Assertive:-I wish
ext. I danced with
Amanda.
Sentence Structure 57
6 Were/Had + Subject + wish For ‘Were/ Had’, subject + Exclamatory:-
subject + ext.! + subject + wish + were/had is used in Were I a child
were/had + ext. assertive sentence. again!
Assertive:-I wish I
were a child again.
7 Would that + Subject + wish For if /would that subject Exclamatory:-
subject + could + subject + + wish is used in assertive Would that we
+ verb + ext.! could + verb sentence. could have a
+ ext. holiday today!
Assertive:-We
wish we could have
a holiday today.
EBD_7275
Chapter

13 PUNCTUATION

Punctuations are marks such as full stop, comma and brackets, used in writing to
separate sentences and their elements and to clarify meaning. When we speak, we
use pauses and the pitch of the voice to express ourselves clearly but in written
English, punctuations are vital to disambiguate the meaning of sentences.
For example – “A new born, without its mother, is nothing.” Here, the emphasis is
on the mother. “New born: without it, mother is nothing.” Here, the emphasis is on
the new born. So, from these two examples, we can understand how punctuations
play a significant role in clarifying the meaning of a sentence. Punctuations consist
of both rules and conventions which should be followed while writing. Below
are some common punctuation marks with their rules and conventions along with
examples that will help you understand the usage of punctuation marks in English
language.
Full stop (.)

¾¾ Full stops are used to mark the end of a sentence.


Example
He is a nice person. She works in a sugar mill.
¾¾ Full stops are also used in initials for personal names.
Example
F.W.Taylor, David M. Joshua
¾¾ Full stops are also used in abbreviations.
Example
Dr. (doctor), Prof. (professor), R.A.M (Random Access Memory) and W.T.C
(World Trade Center) etc.

Question mark (?)


¾¾ Question marks are used to make clear what is said is a question.
Example
Do you need a mobile phone? Are you coming from the market?

Exclamation mark (!)


¾¾ Exclamation marks are used to indicate an exclamative clause or expression
in informal writing.
Punctuation 59
Example :
Listen! Don’t go out of the room. Oh my God! We have won the match.
Note: When we want to indicate something shocking or extremely important in
informal w riting, we use more than one exclamation mark.
Example :
Oh no!!! I am again late for the office. Oh my gosh!!! He is no more.
Comma (,)
¾¾ Commas are used to separate a list of similar words or phrases.
Examples :
He was friendlier, more talkative and more open than last time I met him.
It is advised to write in simple, clear and accurate words.
¾¾ We use commas to separate words or phrases that mark where the voice would
pause slightly.
Examples :
I can’t tell you now. However, I will tell you tomorrow.
He has, in fact, lost all his money in gambling.
¾¾ When main clauses are separated by and, or, but, we don’t normally use a
comma if the clauses have the same subject. However, we sometimes use
commas if the clauses have different subjects.
Examples :
She was very friendly and invited us to her villa in Spain. (Same subject)
Cricketers these days earn more money but they are fitter and play many more
matches. (Same subject)
It was an expensive gift to buy, but we decided it was worth the money.
(Different subjects)
¾¾ We use commas to separate the clauses when a subordinate clause comes
before the main clause. However, we do not always do this in short sentences.
Examples :
If you get lost in the museum, please don’t hesitate to call us.
If you get lost just call us.
¾¾ Commas are used to mark non-defining clauses. Such clauses normally add
extra, non-essential information about the noun or noun phrase.
Examples :
The bus, which arrived after ten minutes, took all the children to the picnic.
India, which has become a technology hub, is a very different country now.
EBD_7275
60 Punctuation
¾¾ We commonly separate question tags, yes-no response and ‘please’ expres-
sion with commas.
Examples
He is a good boy, isn’t he?
Yes, I want to have some coffee.
Please, shut the door.
¾¾ Commas are also used to separate vocatives, discourse markers and interjec-
tions.
Examples :
Pick up the phone, Joseph, can you? Thanks. (Vocative)
Well, what do you think we should do in the matter? (Discourse marker)
Great, that sounds really exciting. (Interjection)
¾¾ We use commas to show that direct speech is following or has just occurred.
Examples :
He said, “I am watching a movie”.
Note: When the direct speech is first, we use a comma before the closing of
the quotation marks. Example – “We will not go”, he said impatiently.

Colon (:)
¾¾ Colons are used to introduce lists.
Examples :
There are three main reasons for the success of the Indian Cricket team:
correct guidance, hard work and self-belief.
¾¾ Colons are also used to indicate a subtitle or to indicate a subdivision of a
topic.
Examples :
1965: Stories from the Second Indo-Pak War
The world is a stage: play your role well.
Semi-colon (;)
¾¾ We use semi-colons instead of full stops to separate two main clauses. In such
cases, the clauses are related in meaning but are separated grammatically.
Examples :
English is spoken throughout South America; in Brazil, the main language is
Portuguese.
Mary was hurt; she knew he only said it to upset her.
Punctuation 61
¾¾ Semi-colons can replace ‘and’ or ‘but’. They denote a pause that’s longer than
a comma but shorter than a full stop.
Note: Semi-colons are not commonly used in contemporary English.

Quotation mark (‘….’ and “.…”)


¾¾ Quotation marks are used in direct speech; we enclose what is said within a
pair of single or double quotation marks.
Example :
The teacher said, “Have you all done the homework?” or
The teacher said: ‘Have you all done the homework?’
¾¾ We use single quotation marks to draw attention to a word.
Examples :
I am very disappointed by his ‘apology’. I don’t think he meant it at all.
¾¾ Single quotation marks are also used to refer to the titles of books, newspapers,
magazines, movies and poems etc.
Examples :
An interesting article has been published in ‘The Times of India’.
‘Life of a pie’ is an award winning movie.

Dashes (-)
¾¾ A dash or hyphen is used to link words together.
Examples
Sub-part, non-verbal, first-class and week-end etc.
Other Punctuation Marks
¾¾ We use brackets to make an aside or a point which is not a part of the main
flow of the sentence. Even if we remove the words between the brackets, the
sentence would still make the same sense.
Examples :
Mahatma Gandhi (father of nation) has written the book ‘My experiment with
truth’.
Mahatma Gandhi has written the book ‘My experiment with truth’.
¾¾ The apostrophe (‘) has two main uses:
(1) To indicate possession or ownership.
EBD_7275
62 Punctuation
Examples :
The boy’s watch was beautiful. Ram’s mother has come to the school.
(2) To indicate where a letter is omitted.
Examples:
I’ll come today which means I will come today
The time is 4 o’clock which means the time is 4 of the clock.
Note: People usually get confused between ‘its’ and ‘it’s’. The examples
below will clearly show the difference between these two and their usage.
¾¾ Use “it’s” when you mean it is or it has.
Examples :
It’s a pleasant day.
It’s your right to vote in the election.
It’s been great getting to know you.
¾¾ Use “its” when you are using it as a possessive.
Examples :
The dog has hurt its paws.
The company celebrated its silver jubilee.
The sugar is in its container.

TIPS AND TECHNIQUES FOR PUNCTUATIONS


There is no short-cut and nothing to mug up in punctuations; all you need to do
is read books, newspapers, magazines and good blogs as much as you can so
that you get to see plethora of punctuations and their uses in different ways This
will help you hone your skills and knowledge of punctuations.
All the rules and conventions mentioned above are very important to keep in
mind, so, it is advised that try to write sentences and use lot of punctuations
while writing and check them later on whether you have correctly used them or
not. Doing this will enable you gain confidence in the usage of punctuations.
After following these tips, you will be able to pick wrongly used punctuations
just by having a glance over the sentences.
Chapter

14 CONTRACTION

A contraction is an abbreviated version of a word or words made by shortening and


combining two words. We make contractions with auxiliary verbs, and also with
be and have when they are not auxiliary verbs. When we make a contraction,
we commonly put an apostrophe in place of a missing letter. For example – can’t
(can+not), don’t (do+not) and haven’t (have+not) etc.
We use contraction in both speaking and writing; however, these are usually not
appropriate in formal writing.

Examples of uses of contractions in sentences:


¾¾ He didn’t go to school yesterday.
¾¾ Please, don’t open the window now.
¾¾ I haven’t eaten.
¾¾ Isn’t he a good boy?
¾¾ Let’s celebrate Arthur’s birthday.
¾¾ She’ll do the work before time.
¾¾ You shouldn’t have broken the glass in anger.
¾¾ Finally, we’re going for the picnic.
The following are the most common contractions:

AM
I am I’m I’m a teacher.

ARE
You are you’re You’re funny.
We are we’re We’re happy.
They are they’re They’re going to the market.
Who are who’re Who’re you?
EBD_7275
64 Contraction

HAVE
I have I’ve I’ve had too much drink.
You have you’ve You’ve been a good sister.
We have we’ve We’ve been to London.
They have they’ve They’ve been on the swings.
Could have could’ve He could’ve helped the poor boy.
Would have would’ve We would’ve worn a sweater.
Should have should’ve She should’ve put on a hat.
Might have might’ve I might’ve asked for another glass of water.
Who have who’ve Who’ve you spoken to?
There have there’ve There’ve been a number of visitors today.

IS, HAS
He is/ has he’s He’s a nice boy.
She is/ has she’s She’s a nice girl.
It is/ has it’s It’s a pleasant day.
What is/ has what’s What’s for lunch?
That is/ has that’s That’s good news.
Who is/ has who’s Who’s coming tonight?
There is/has there’s There’s no water in the vase.
Here is/has here’s Here’s one apple in the fridge.
One is/has one’s One’s needed to represent our family.

WILL (or SHALL)


I will I’ll I’ll see you soon.
You will you’ll You’ll be on time, right?
She will she’ll She’ll be late.
He will he’ll He’ll be early.
It will it’ll It’ll be here soon.
We will we’ll We’ll see you later.
They will they’ll They’ll get there first.
That will that’ll That’ll be great.
Contraction 65

There will there’ll There’ll be lots to see.


This will this’ll This’ll be fun.
What will what’ll What’ll we do?
Who will who’ll Who’ll be there?

WOULD, HAD
I would/ had I’d I’d like a glass of water.
You would/ had you’d I wish you’d let me come along with you.
He would/ had he’d He’d like a biscuit.
She would/ had she’d She’d like milk.
We would/ had we’d We’d go there soon.
They would/ had they’d They’d like something to drink.
It would/ had it’d It’d be troublesome.
There would/ had there’d There’d be a little delay.
What would/ had what’d What’d you expect?
Who would/ had who’d Who’d have known?
That would/ had that’d That’d be great.

US
Let us let’s Let’s help the old man.

Negative Contractions

NOT
Cannot can’t I can’t come today.
Do not don’t Don’t touch the wire.
Is not isn’t It isn’t safe to drive without a helmet.
Will not won’t I won’t enter the room unless asked.
Should not shouldn’t I shouldn’t go first.
EBD_7275
66 Contraction

Could not couldn’t I couldn’t be the last one.


Would not wouldn’t I wouldn’t want to be third.
Are not aren’t Aren’t you Manish’s cousin?
Does not doesn’t He doesn’t want to come with us.
Was not wasn’t He wasn’t paying attention.
Were not weren’t They weren’t afraid.
Has not hasn’t He hasn’t called yet.
Have not haven’t I haven’t received the mail.
Had not hadn’t I hadn’t thought of that.
Must not mustn’t I mustn’t get too upset.
Did not didn’t He didn’t know what to do at that moment.
Might not mightn’t I mightn’t do it again.
Need not needn’t You needn’t worry about me, I’ll be fine.

TIPS & TECHNIQUES FOR CONTRACTIONS

To be frank, there is no short-cut and nothing to mug up in contractions; all you


need to do is read books, newspapers and magazines etc. as much as you can so
that you get to different contractions and their uses.
Chapter

15 COMMON ERRORS

English grammar is a majorly important part of English section in almost all


entrance examinations. A number of students face lot of problems while solving
the questions based on common errors because most of the time, they are unaware
of the rules of grammar which leads to committing very common mistakes.
To enhance your grammar knowledge and help you overcome committing common
mistakes, below are given some examples along with explanations that will be
very helpful and handy to you.

¾¾ Only I and my friend were at the concert. (WRONG)


Only my friend and I were at the concert. (RIGHT)
(Pronouns order – I and me come last when more than one pronoun is used
in a phrase, you comes next to last, and third-person pronoun comes first.)
¾¾ Everybody will get their share. (WRONG)
Everybody will get his share. (RIGHT)
(‘Everybody’ may sound like ‘a lot of people’, but it is a singular noun and
takes a singular verb.)
¾¾ Most of the students found difficult to comprehend his speech. (WRONG)
Most of the students found it difficult to comprehend his speech. (RIGHT)
¾¾ Don’t pride on your victory. (WRONG)
Don’t pride yourself on your victory. (RIGHT)
(Without the reflexive pronoun ‘yourself’, it would be impossible for the
reader to know who you pride on.)
¾¾ I have a good news for you. (WRONG)
I have good news for you. (RIGHT)
(‘News’ is uncountable, which means that not only it is followed by a singular
verb, but you also cannot say ‘a news’).
¾¾ The boys leave the school at five o’clock. (WRONG)
The boys leave school at five o’clock. (RIGHT)
(Daily routine, not leaving for any specific purpose)
EBD_7275
68 Common Errors
¾¾ They realized where their weak points were and how to get rid of them.
(WRONG)
They realized where their weak points were and how they could get rid of
them (RIGHT)
(A fragment with a missing subject; hence, an appropriate subject to form an
independent clause should be included.)
¾¾ While doing the work, there are obstacles ahead. (WRONG)
While doing the work, they met with obstacles. (RIGHT)
(Needs to revise a dangling modifier by naming the appropriate doer of the
action as the subject of the main clause.)
¾¾ He cannot set a foot in my house. (WRONG)
He cannot set foot in my house. (RIGHT)
(The idiom is ‘set foot in’ and not ‘set a foot in’)
¾¾ Andamans are a group of islands. (WRONG)
The Andamans are a group of islands. (RIGHT)
(‘The’ is used with the places consisting many islands; for example – The
Bahamas, The Maldives, The Philippines and The West Indies etc.)
¾¾ She pretended to not recognize the man in the market. (WRONG)
She pretended not to recognize the man in the market. (RIGHT)
¾¾ She neither speaks English nor French. (WRONG)
She speaks neither English nor French. (RIGHT)
(Here, ‘neither’ modifies the verb ‘speak’ whereas, it should modify the
language ‘English’.)
¾¾ Fire broke out in our neighbourhood. (WRONG)
A fire broke out in our neighbourhood. (RIGHT)
(While fire, as a substance, is uncountable, but ‘a fire broke out’ gives a mental
image of a single fire.)
¾¾ His heart sank and could hardly stand. (WRONG)
His heart sank and he could hardly stand. (RIGHT)
(A fragment with a missing subject; hence, an appropriate subject to form an
independent clause should be included.)
Common Errors 69
¾¾ All I know is my friend is right. (WRONG)
All I know is that my friend is right. (RIGHT)
(Here, the verb ‘is’ connected with reporting can be followed by a that-clause
acting as the direct object.)
¾¾ He has not yet gone to the bed. (WRONG)
He has not yet gone to bed. (RIGHT)
(‘Go to bed’ means to lie down to sleep, to put oneself in one’s bed while ‘go
to the bed’ means not necessarily preparing for sleep.
¾¾ I saw him to play. (WRONG)
I saw him play. (RIGHT)
(Here, use of the preposition ‘to’ is inappropriate. There are two patterns of
this kind of sentences: 1) I saw him play = I saw him play throughout the
game. 2) I saw him playing = I saw him on the field but I don’t know whether
he actually finished the action.)
¾¾ The answer of this question is not so easy. (WRONG)
The answer to this question is not so easy. (RIGHT)
(The answer to the question is the normal grammatical form like key to the
problem.)
¾¾ Water composes of hydrogen and oxygen. (WRONG)
Water is composed of hydrogen and oxygen. (RIGHT)
(‘Be composed’ of something means to be formed from various things like, air
is composed mainly of nitrogen and oxygen.)
¾¾ I didn’t see him too. (WRONG)
I didn’t see him either. (RIGHT)
(‘Either’ is used in negative sentences to add an agreeing thought: Tom doesn’t
speak French. Sam doesn’t speak French either.)
¾¾ The polar bear is not used to live in hot places. (WRONG)
The polar bear is not used to living in hot places. (RIGHT)
(‘Be used to’ is followed by a noun (or pronoun) or the gerund – the ‘ing’ form
of a verb like, I can’t get used to getting up early.)
¾¾ Nobody is bound to suffering. (WRONG)
Nobody is bound to suffer. (RIGHT)
(‘Bound’ after verb + ‘to’ infinitive means certain or extremely likely to
happen like, you’re bound to forget people’s name often.)
EBD_7275
70 Common Errors
¾¾ Have you read the Shakespeare’s Macbeth? (WRONG)
Have you read Shakespeare’s Macbeth? (RIGHT)
¾¾ She rarely goes to theatre. (WRONG)
She rarely goes to the theatre. (RIGHT)
(‘The’, the definite article is used because you’re talking about something
‘definite’.)
¾¾ Adams invited me to a party. (WRONG)
The Adams invited me to a party. (RIGHT)
(If you are referring some members of a family, you make it plural by adding
‘the’ in the beginning.)
¾¾ Shiela felt sorry about the street children but she did not voice. (WRONG)
Shiela felt sorry about the street children but she did not voice it. (RIGHT)
(The sentence is incomplete without the objective case ‘it’.)
¾¾ He was astonished by her sudden resignation. (WRONG)
He was astonished at her sudden resignation. (RIGHT)
(If you are astonished by something, you are very surprised about it: I was
astonished by his stupidity. Here, contextual use is ‘at’.)
¾¾ The judge has decided the case. (WRONG)
The judge has decided upon the case. (RIGHT)
(To make a judgement about some aspect of someone or something; ‘Upon’ is
formal and less commonly used than on.)
¾¾ Do you want to try these new skates? (WRONG)
Do you want to try out these new skates? (RIGHT)
(‘Try out’ means ‘to undergo a competitive qualifying test; to test or use
something experimentally’).
¾¾ Tom’s project is more perfect than Sam’s. (WRONG)
Tom’s project is better than/superior to Sam’s. (RIGHT)
(There are words like ‘square’ and not ‘more square’ or round and not ‘more
round’ similarly, it is perfect, unique or not unique etc.)
¾¾ This machine cost me hundred dollars. (WRONG)
This machine cost me a hundred dollars. (RIGHT)
(If you spell out dollars, it would be usual to also spell out a hundred or one
hundred: You owe me a hundred dollars or you owe me $100.)
Common Errors 71
¾¾ He has good knowledge of Zoology. (WRONG)
He has a good knowledge of Zoology. (RIGHT)
¾¾ French are industrious and frugal. (WRONG)
The French are industrious and frugal. (RIGHT)
(French means of, relating to, or characteristic of France or its people or
culture; The French denotes people of France or the natives/inhabitants of
France collectively.)
¾¾ David has just been commissioned as the captain. (WRONG)
David has just been commissioned as captain. (RIGHT)
¾¾ He will be great help for you. (WRONG)
He will be of great help to you. (RIGHT)
(‘Of great help to you should be used as it is grammatically correct.)
¾¾ Both of them did not win the election. (WRONG)
Neither of them won the election. (RIGHT)
(Additive phrases (both, also, too, etc.) are not used when overall meaning of
them is to negate.)
¾¾ He told that he would be visiting Father on weekend. (WRONG)
He said that he would be visiting Father on weekend. (RIGHT)
(Use ‘tell’ with a personal object and ‘said’ when there is no personal object.)
¾¾ Hardly the sun had risen when we set out. (WRONG)
Hardly had the sun risen when we set out. (RIGHT) or
The sun had hardly risen when we set out. (RIGHT)
(If hardly, scarcely, barely and no sooner are in the initial position, the subject
and auxiliary are inverted: Hardly had we arrived home when the doorbell
rang or we had hardly arrived home when the doorbell rang.)
¾¾ Not only she speaks English but also Chinese. (WRONG)
She speaks not only English but also Chinese. (RIGHT)
(For proper parallel structure, the verb following the subject must precede
‘not only’ so that it applies to both parallel phrases – English and Chinese.)
¾¾ Suppose, if she arrives late, you will miss the train. (WRONG)
Suppose she arrives late, you will miss the train. (RIGHT)
(Use suppose, supposing and what if + present verb form to make suggestions
about what might happen. Suppose and if do not come together.)
EBD_7275
72 Common Errors
¾¾ He did good in the exams. (WRONG)
He did well in the exams. (RIGHT)
(Good is an adjective and a verb cannot be modified by an adjective. Well is
an adverb and it goes after the verb or verb + object.)
¾¾ It was bitter cold that night. (WRONG)
It was bitterly cold that night. (RIGHT)
(‘Cold’ itself is an adjective; hence, can be modified by an adverb ‘bitterly’
and not by adjective i.e. ‘bitter’)
¾¾ The movie is too interesting. (WRONG)
The movie is very interesting. (RIGHT)
(‘Too’ means more than enough; do not use in the sense of very or much.)
¾¾ No one writes as neat as Susan does. (WRONG)
No one writes as neatly as Susan does. (RIGHT)
(‘Neat’ is an adjective which can’t modify a verb i.e. ‘write’; ‘neatly’ meaning
‘with neatness’ is an adverb which correctly modifies the verb ‘write’.)
¾¾ The grandmother is living miserly. (WRONG)
The grandmother is living in a miserly way. (RIGHT)
(Not all words ending in-ly are adverbs like miserly is an adjective.)
¾¾ The receptionist sat on her desk. (WRONG)
The receptionist sat at her desk. (RIGHT)
¾¾ Everybody must conform with the rules. (WRONG)
Everybody must conform to the rules. (RIGHT)
(‘Conform’ means to comply with rules, standards, or laws’; conform to
hygiene regulations; in some special usages ‘conform with’ is used: changes
have to conform with international classifications.)
¾¾ The Himalayas are covered by snow. (WRONG)
The Himalayas are covered with/in snow. (RIGHT)
‘Covered by’ usually means that the covering actually hides the thing that is
covered)
¾¾ Divide the apple in four parts. (WRONG)
Divide the apple into four parts. (RIGHT)
(To or cause to separate into parts or groups: divide students into small
discussion groups; Book divided into various chapters etc.)
Common Errors 73
¾¾ This is an exception of the rule. (WRONG)
This is an exception to the rule. (RIGHT)
¾¾ My leg is paining. (WRONG)
There’s/I’ve got a pain in my leg. (RIGHT)
(Use pain as noun and precede it by have or feel.)
¾¾ They behaved cowardly. (WRONG)
They behaved in a cowardly manner. (RIGHT)
(Cowardly, silly and miserly are all adjectives which can’t modify verbs)
¾¾ A 80% majority agree to the decision made by the principal. (WRONG)
A 80% majority agrees to the decision made by the principal. (RIGHT)
(Here, the word ‘majority’ is used for a collective group, hence, it should be
treated as singular.)
¾¾ She does not know swimming. (WRONG)
She does not know to swim. (RIGHT)
¾¾ Open your book on page 15. (WRONG)
Open your book at page 15. (RIGHT)
¾¾ She died from Cancer. (WRONG)
She died of Cancer. (RIGHT)
¾¾ He lives besides my house. (WRONG)
He lives beside my house. (RIGHT)
(Besides and beside have different meanings. Beside shows position whereas
besides means ‘in addition’.)
¾¾ I have lost my patience. (WRONG)
I have lost patience. (RIGHT)
¾¾ You have no influence on him. (WRONG)
You have no influence over him. (RIGHT)
¾¾ He insisted to go there. (WRONG)
He insisted on going there. (RIGHT)
¾¾ We reached safely. (WRONG)
We reached safe. (RIGHT)
EBD_7275
74 Common Errors
¾¾ There were less people. (WRONG)
There were fewer people. (RIGHT)
¾¾ He had no other alternative. (WRONG)
He had no alternative. (RIGHT)
¾¾ He shouted not as loud as the rest of the people. (WRONG)
He shouted not as loudly as the rest of the people. (RIGHT)
(The adverb ‘loudly’ should be used.)
¾¾ Verbal orders shall not be obeyed. (WRONG)
Oral orders shall not be obeyed. (RIGHT)
(Use of the adjective verbal is inappropriate in the context of the sentence.
¾¾ You have paid the bill, isn’t it? (WRONG)
You have paid the bill, haven’t you) (RIGHT)
¾¾ Varun threatened to divorce her often. (WRONG)
Varun often threatened to divorce her. (RIGHT)
(If you put a modifier in a different place in the sentence, it means something
different.)
¾¾ You should learn how to cope up. (WRONG)
You should learn how to cope. (RIGHT)
¾¾ There is nothing such as luck. (WRONG)
There is no such thing as luck. (RIGHT)
¾¾ The maid almost washed all of the utensils. (WRONG)
The maid washed almost all of the utensils. (RIGHT)
(You should be very careful in placing the adverb in the sentence as it has a
different meaning.)
¾¾ After the party, I will return to my quarter. (WRONG)
After the party, I will return to my quarters. (RIGHT)
¾¾ It is more cold now. (WRONG)
It is colder now. (RIGHT)
(The comparative adjective ‘colder’ should be used).
¾¾ He was late for office and fired by his boss. (WRONG)
He was late for office and was fired by his boss. (RIGHT)
(Parallel verb phrase ‘was’ should be used)
Common Errors 75

TIPS AND TECHNIQUES IN COMMON ERRORS

¾¾ Read the sentence all the way through. Even if you think the error is in part
B, make sure to read the entire sentence as this will help prevent you from
falling into traps.
¾¾ Always check for subject-verb agreement. The verb needs to take a form
that matches the subject.
¾¾ You are not done only by picking the error because it sounds weird or
because it’s not the word you would use. You should be able to explain to
yourself what error the answer you picked contains. If you can’t do so, it’s
probably not the right choice.
¾¾ Lastly, read English newspapers and articles etc. as much as you can so that
you get accustomed to English language. Reading habits, many times, help
you pick the errors just by having a glance over the sentence.
EBD_7275
VOCABULARY

Chapter

16 VOCABULARY

A set of all the words that exist in a particular language or subject is vocabulary.
Learning vocabulary is a very important part of learning a language. The more
words you know, the more you will be able to understand what you hear and read;
and the better you will be able to say what you want to speak or write. In this
chapter, various sections like Synonyms, Antonyms, Analogies, Homophones and
Foreign words and Expression etc. have been given to enhance your vocabulary
which will help you in the coming examinations.

One word – A small collection


Abdicate – Renounce a throne or high office
Accomplice – One associated with another especially in wrong-doing
Acoustics – Science of the production, transmission, reception and effects of sound
Actuary – One who calculates insurance and annuity premium etc.
Amnesty – General pardon
Abattoir – A building where animals are killed for meat
Agnostic – One who believes that nothing can be known about God
Alimony – Compensatory allowance given to wife after divorce
Altruist – One who is habitually kind to others
Anecdote – A short interesting or amusing story
Backwater – A part of river out of the main stream, where the water does not move
Barbecue – A metal flame on which meat etc. is cooked over an open fire
Bibliography – A list of writings on a subject
Biennial – Happening once every two years
Blue blood – The quality of being a noble person by birth
Bonsai – The art of growing a plant in a pot that is prevented from reaching its
natural size
Boulevard – A broad street having trees on each side
Bourgeois – Belonging to the middle class
Bric-a-brac – Small object kept for decoration
Vocabulary 77
Bullion – Bars of Gold or Silver
Cabal – A small group of people who make secret plans for political action
Cannibal – One who eats human flesh
Catch-22 – A situation from which one is prevented from escaping by something
that is part of the situation itself.
Charlatan – One who deceives others by falsely claiming to have a skill
Celibacy – One who does not indulge in carnal pleasure
Cloak-and-Dagger – Stories that deal with adventure and exciting mystery
Coagulate – Change from a liquid into a solid by chemical action
Colonnade – A row of pillars supporting a roof or arches
Consortium – A combination of several companies, banks etc. for a common
purpose
Contretemps – An unlucky and unexpected event, socially uncomfortable position
with someone
Defeatism – the practice of thinking in a way that shows an expectation of being
unsuccessful
Deja vu – The feeling of remembering something that in fact one is experiencing
for the first time
Dragnet – A system of connected actions and methods for caching criminals
Dregs – Sediment in a liquid that sinks to the bottom and is thrown away
Drudgery – Hard uninteresting work
Dally – Waste time
Dawdle – Take one’s time; proceed slowly
Dearth – An insufficient quantity or number
Demeanor – The way a person behaves toward other people
Diffident – Lacking self-confidence
Enigmatic – That which is mysterious and very hard to understand
Empirical – Based on practical experience of the world we see and feel
Entomology – The scientific study of insects
Epicurean – Lover of physical/material
Egalitarian – Favouring social equality
Expressionism – A style of painting which expresses feelings rather than describing
objects and experiences
Enamor – Attract
Ennui – The feeling of being bored by something tedious
EBD_7275
78 Vocabulary
Epoch – A period marked by distinctive character
Exculpate – Pronounce not guilty of criminal charges
Febrile – Of or caused by fever
Felony – A serious crime such as murder or armed robbery
Fluvial – Of, found in, or produced by rivers
Foible – A small rather strange and stupid personal habit
Foray – A sudden rush into enemy country
Freckle – A small flat brown spot on the skin
Fumigate – To clear of disease, bacteria etc. by means of chemical smoke
Feign – Give a false appearance of
Founder- Walk with great difficulty
Forlorn – Marked by or showing hopelessness
Graffiti – Drawing or writing on a wall
Grange – A large country house with farm buildings
Guinea pig – A person who is subject of some kind of test
Gentry – The most powerful members of a society
Germane – Relevant and appropriate
Gibberish – Unintelligible talking
Gibe – An aggressive remark directed at a person
Gird – Prepare oneself for action or a confrontation
Gloat – Dwell on with satisfaction
Gripe - Complain
Halitosis – A condition in which one has bad health
Headstrong – Determined to do what one wants in spite of all advice
Heirloom – A valuable object passed on for generations
Hinterland – The inner part of a country
Hothead – One who does things too quickly, without thinking
Haggle – An instance of intense argument (as in bargaining)
Helm – A position of leadership
Hireling – A person who works only for money
Humongous – Very large
Hoard – Save up as for future use
Idolatry – The worship of idols
Vocabulary 79
Implacable – Impossible to satisfy, change, or make less angry
Incorporeal – Without a body or form
Innate – Being talented through inherited qualities
Inseminate – To put male seed into a female
Intelligentsia – Those who are highly educated and often concern themselves with
ideas and new developments
Intestate – Not having made a will
Invective – A forceful attacking speech used for blaming someone
Imperious – Having or showing arrogant superiority
Impromptu – Without advance preparation
Juxtapose – To place side by side or close together
Jaded – Bored or apathetic after experiencing too much of something
Jaunt – A journey taken for pleasure
Jaunty – Having a cheerful, lively, and self-confident air
Jeer - Laugh at which contempt and derision
Jejune – Lacking interest or significance or impact
Jeopardize – Pose a threat to; present a danger to
Jest – Activity characterized by good humour
Jettison – Throw away, of something encumbering
Jibe – An aggressive remark directed at a person
Ken – Range of what one can know or understand
Kindle – Cause to start burning
Knave – A deceitful and unreliable scoundrel
Knoll – A small natural hill
Kimono – A long loose garment made of silk
Kudos – An expression of approval and commendation
Killjoy – Someone who spoils the pleasure of others
Knack – A special way of doing something
Kindred – Group of people related by blood or marriage
Kleptomaniac – Someone with an irrational urge to steal
Lackey – One who behaves like a servant by always obeying
Labile – Readily undergoing change or breakdown
Lachrymose – Showing sorrow
Lacklustre – Lacking brilliance or vitality
EBD_7275
80 Vocabulary
Laconic – Brief and to the point
Lament – Express grief verbally
Languor – A feeling of lack of interest or energy
Leery – Openly distrustful and unwilling to confide
Lethargic – Deficient in alertness or activity
Limpid – Clear and bright
Malady – Any unwholesome or desperate condition
Malcontent – One who is dissatisfied with the existing state of affairs
Misnomer – A name wrongly or mistakenly applied
Morbid – Having or expressing a strong interest in sad or unpleasant things
Malleable – Easily influenced
Manumit – Free from slavery or servitude
Maudlin – Effusively or insincerely emotional
Maul – Injure badly
Melancholy – Characterized by or causing or expressing sadness
Melee – A noisy riotous fight
Namesake – A person or thing that has the same name as another
Necromancy – The practice which claims to learn about the future by talking with
the dead
Nemesis – Just and unavoidable punishment
Newfangled – New (idea or machine etc.) but neither necessary nor better
Nihilism – The belief that nothing has meaning or value
Nadir – The lowest or most unsuccessful point in a situation
Narcissist – Someone who is excessively self-centered
Nefarious – Extremely wicked
Nestle – Move or arrange oneself in a comfortable and cosy position
Nether – Lower
Obtrude – To be pushed or push oneself into undue prominence
Obviate – To clear away or provide for, as an objection or difficulty
Odoriferous – Having a smell
Ostracism – The state of not being included in a group
Obdurate – Showing unfeeling resistance to tender feelings
Oblique – Not direct, explicit, or straightforward
Obloquy – State of disgrace resulting from public abuse
Vocabulary 81
Obsolete – No longer in use
Obtuse – Slow to learn or understand; lacking intellectual acuity
Ogle – Look at with amorous intentions
Panacea – A solution or remedy for all difficulties or diseases
Panache – Being able to do things in a confidant and elegant way
Panegyric – A speech or a piece of writing praising somebody or something
Perdition – A state of eternal punishment and damnation into which an unrepentant
person passes after death
Perjury – A lie told on purpose in court
Placate – Make someone less angry or hostile
Platonic – A friendly, not sexual, relationship between a man and a woman
Plebeian – Of the lower social class
Portend – Be a sign or warning that (something, especially something momentous
or calamitous) is likely to happen
Putsch – A sudden secretly planned attempt to remove a government by force
Quixotic – Trying to do the impossible, often so as to help others, while getting
oneself into danger
Quadruped- An animal especially a mammal having four limbs
Quaff – To swallow hurriedly or greedily or in one draught
Quaint – Attractively old-fashioned
Qualm – Uneasiness about the fitness of an action
Quandary – A situation from which extrication is difficult
Quash – Put down by force or intimidation
Quell – Suppress or crush completely
Quibble – Argue over petty things
Quirk – A strange attitude or habit
Raconteur – One who is good at telling stories in an interesting way
Recumbent – Lying down on the back or side
Rescind – To make void, as an act, by the enacting authority or by a superior
authority
Retribution – A justly deserved penalty
Rancour – A feeling of deep and bitter anger and ill-will
Ravenous – Extremely hungry
Rebuke – An act or expression of criticism and censure
EBD_7275
82 Vocabulary
Recoil – Draw back as with fear or pain
Refurbish – Make brighter and prettier
Relegate – Assign to a lower position
Sapient – Wise and full of deep knowledge
Snippet – A short piece from something spoken or written
Souvenir – An object kept as a reminder of something
Stoic – One who is indifferent to joys or sorrows
Stratagem – A trick to deceive an enemy
Superannuated – Too old for work
Surreal – Having a strange dreamlike unreal quality
Sardonic – Disdainfully or ironically humorous
Satiate – Fill to satisfaction
Spurn –Reject with contempt
Transient – Lasting only for a short time
Tacit – Implied by or inferred from actions or statements
Taciturn – Habitually reserved and uncommunicative
Tantamount – Being essentially equal to something
Taper – Diminish gradually
Temerity – Fearless daring
Terse – Brief and to the point
Threshold – The entrance for passing through a room or building
Throng – A large gathering of people
Toil – Work hard
Underling – A person of low rank in relation to another
Ubiquitous – Being present everywhere at once
Ultimatum – A final peremptory demand
Umbrage – A feeling of anger caused by being offended
Uncanny – Surpassing the ordinary or normal
Unctuous – Unpleasantly and excessively suave or ingratiating
Underscore – Give extra weight to
Unfledged – Young and inexperienced
Unfounded – Without a basis in reason or fact
Unison – The act of occurring together or simultaneously
Vocabulary 83
Valise – A small bag used while travelling
Vacillate – Be undecided about something
Vacuous – Devoid of intelligence
Vagary – An unexpected and inexplicable change in something
Vainglorious – Feeling self-importance
Valour – Great courage in the face of danger, especially in battle
Vanity – The trait of being unduly conceited
Vanquish – Come out better in a competition, race, or conflict
Vantage – Place or situation affording some benefit
Vapid – Lacking significance or liveliness or spirit or zest
Weakling – One who lacks physical strength or strength of character
Wretch – An unfortunate or unhappy person
Waggish – Witty or joking
Waif – A homeless child especially one forsaken or orphaned
Wail – A cry of sorrow and grief
Waive – Do without or cease to hold or adhere to
Wallow – Devote oneself entirely to something
Wane – A gradual decline (in size/strength/power/number)
Waver – Be unsure or weak
Weary – physically and mentally fatigued
Xenophobia – Fear of strange or foreign people and customs etc.
Yearn –Desire strongly or persistently
Yahoo – A person who is not intelligent or intelligent in culture
Yelp – A sharp high-pitched cry
Yeoman – A free man who cultivates his own land
Yoke – Become joined or linked together
Yokel – A person who is not intelligent or interested in culture
Yonder – Distant but within sight
Yore – Time long past
Yuppie – A young man in a professional job with a high income
Yell – A loud, sharp cry of pin, surprise or delight
Zeitgeist – The intellectual and moral tendencies that characterize any age or epoch
Zany – Ludicrous or foolish
EBD_7275
84 Vocabulary
Zeal – A feeling of strong eagerness
Zealot – A fervent and even militant proponent of something
Zealous – Marked by active interest and enthusiasm
Zenith – The point above the observer directly opposite the nadir
Zephyr – A slight wind
Zest – Vigorous and enthusiastic enjoyment
Zonk – Hit or strike

Foreign words and phrases


Foreign words and phrases are generally not asked directly. But the knowledge of
foreign words and phrases will help you in reading comprehension and other types
of common questions. So, make yourself familiar with the common foreign words
and phrases.
• Ab initio : from the beginning.
• Addenda : ‘list of additions’. (addenda to a book)
• Ad valorem : according to value.
• Ad infinitum : to infinity.
• A la carte : according to the bill of fare. (a la carte dishes are
available)
• Alter ego : the other self, intimate friend, (Kissinger was the
alter ego of Nixon)
• Amende honorable : satisfactory apology, reparation.
• Amour propre : self love
• Ancien regime : a political or social system that has been displaced
by another.
• A posteriori : empirical
• A priori : from cause to effect, presumptive. (every science
cannot be taught a priori)
• Apropos : in respect of
• An couran : fully acquainted with matters.
• Au fait : completely familiar with
• Au revoir : until we meet again (to say au revoir at parting)
• Avant propos : preliminary matter, preface
• Beau ideal : the ideal of perfection.
• Beaumonde : the world of fashion.
• Beaux esprits : men of wit.
Vocabulary 85
• Bete noire : a special aversion (Uncle Symond was my
father’s bête noire)
• Bon voyage : a good voyage or journey to you
• Casus belli : that which causes or justifies war.
• Cause celebre : a celebrated or notorious case in law
• Charge d’ affaires : diplomat inferior in rank to an Ambassador but
acting on his behalf in his absence.
• Chef d’ oeuvre : masterpiece (Mona Lisa is Vinci’s Chef-d’
oeuvre)
• Circa : about (‘circa 1930’)
• Contretemps : an unexpected or untoward event; a hitch
• Corrigenda : a list of errors (in a book)
• Coup d’etat : violent change in government.
• Coup de grace : a finishing stroke. (The coup de grace of the
Russian Revolution was the total annihilation of
the Czar family)
• Cul-de-sac : a blind alley (The failure of the Policy of non-
alignment in 1962 saw our foreign policy reach a
cul-de-sac)
• De facto : actual or actually (de facto recognition to a state)
• Dejure : from the law, by law.
• Denovo : anew, again (trial of a case)
• Denouement : the end of a plot (in play)
• De profundis : out of the depths
• Dernier resort : last resort
• Detente : easing of strained relations especially between
states / countries
• Dramatis personae : characters of a drama or play
• Enfant terrible : a terrible child; one who makes disconcerting
remarks
• En rapport : in harmony
• Entourage : friends, group of people accompanying a
dignitary.
• Errata : list of errors
• Esprit de corps : the animating spirit of a collective body, as a
regiment.
• Eureka : a cry of joy or satisfaction when one finds or
discovers something.
EBD_7275
86 Vocabulary
• Ex-officio : in virtue of his office.
• Expose : a statement
• Expost facto : acting retrospectively
• Fait accompli : a thing already done.
• Faux pas : a false step; slip in behaviour
• Hoi polloi : the rabble, ordinary people
• Inextenso : unextended small in extension
• Ipso facto : by that very fact.
• Laissez faire : non interference
• Mal-a-propos : ill-timed
• Mutatis mutandis : with the necessary changes (rules will come into
force mutatis mutandis)
• Noblesse oblige : rank imposes obligation.
• Nota bene : note well
• Par excellence : pre-eminently.
• Pari passu : side by side.
• Per se : by itself.
• Piece de resistance : the main dish of a meal.
• Poste restante : to remain in the post office till called for. (said of
letters)
• Pro bone publico : for the good of the public
• Protégé : one under the protection of another. (S.Vietnam is
US’s protege).
• Quid pro quo : an equivalent, something in return.
• Raisond’etre : the reason for a thing’s existence.
• Sanctum sanctorum : holy of holies. (temple, church etc.)
• Seiratim : in a series
• Sub rosa : under the rose; confidentially
• Sui gereris : in a class by itself
• Summon bonum : the chief good.
• Terra incognita : an unknown country
• Tour deforce : a notable feat or strength of skill.
• Ultra vires : beyond one’s authority
• Vox populi, vox die : The voice of the people is The voice of God.
• Zeitgeist : spirit of the age
Vocabulary 87
One word with different meanings
Multiple meaning words are those which we use for different meanings in different
contexts. The same word can be used as noun, adjective or verb etc. Below is list
of some common words that have different meanings.
1. Ream
¾¾ A pile of paper
¾¾ To juice a citrus fruit
2. Mean
¾¾ Average
¾¾ Not nice
3. Tender
¾¾ Gentle
¾¾ To pay money
4. Harbour
¾¾ A sheltered area of water that is deep enough so that ships can anchor
there
¾¾ To provide shelter
5. Hatch
¾¾ The process of a baby bird coming out of its egg
¾¾ An opening in the floor, ceiling, or wall of a ship or aircraft
6. Caper
¾¾ A pickled flower bud used to flavour food
¾¾ A prank, or a playful activity for amusement
7. Carp
¾¾ A type of fish
¾¾ To find fault or complain about small matters
8. Flake
¾¾ A small particle or piece of something
¾¾ To become flabby or languid
9. Nick
¾¾ A small cut or small notch
¾¾ To steal (slang)
10. Page
¾¾ A sheet of paper, on one side of it, in a book
¾¾ A boy who acts as a servant, sometimes wearing a uniform.
Homophones
A homophone is a word which is pronounced the same as another word but differs
in meaning. Below are given few examples of homophones.
EBD_7275
88 Vocabulary

Air, Heir Lead, Led


Allowed, Aloud Loan, Lone
Alter, Altar Made, Maid
Ate, Eight Peace, Piece
Band, Banned Pain, Pane
Bean, Been Pair, Pear
Blew, Blue Raise, Rays
Brake, Break Sight, Site
Cell, Sell Tail, Tale
Ceiling, Sealing Throne, Thrown
Check, Cheque Steal, Steel
Coarse, Course Son, Sun
Die, Dye Tide, Tied
Groan, Grown To, Too, Two
Hair, Hare Vain, Vane, Vein
Hall, Haul Waist, Waste
Heel, Heal We, Wee
Knew, New Wait, Weight

Synonyms and Antonyms


A synonym is a word that means exactly or nearly the same as another word
or phrase in the same language, for example – shut is a synonym of close. An
antonym is a word opposite in meaning to another word, for example – bad is an
antonym of good.

Following are some words and against each word are given the list of Synonyms
and Antonyms separately:
¾¾ Allure
Synonym: Tempt, Entice, Fascinate
Antonym: Repulse, Deter, Repel, Distaste
¾¾ Apparent
Synonym: Distinct, Evident, Obvious, Perceptible
Antonym: Obscure, Indistinct, Masked, Doubtful
¾¾ Axiom
Synonym: Maxim, Truth, Saying, Dictum
Antonym: Blunder, Absurdity, Irrelevant, Ridiculousness
¾¾ Astute
Synonym: Clever, Intelligent, Wise, Brilliant
Antonym: Dull, Shallow, Unintelligent, Solid
Vocabulary 89
¾¾ Abash
Synonym: Discourage, Confound, Embarrass, Discompose
Antonym: Encourage, Uphold, Hearten, Embolden
¾¾ Bleak
Synonym: Dismal, Gloomy, Chilly, Dreary
Antonym: Bright, Pleasant, Cheerful, Balmy
¾¾ Brittle
Synonym: Frail, Fragile, Breakable, Delicate
Antonym: Tough, Enduring, Strong, Unbreakable
¾¾ Benevolence
Synonym: Humanity, Generosity, Charity, Liberality
Antonym: Malevolence, Inhumanity, Malignity, Unkindness
¾¾ Barbarous
Synonym: Savage, Uncivilized, Untamed, Brutal
Antonym: Cultured, Humane, Refined, Gentle
¾¾ Bewitching
Synonym: Magical, Fascinating, Tantalising, Spell binding
Antonym: Repulsive, Repugnant, Nauseating, Disgusting
¾¾ Corpulent
Synonym: Obese, Ugly, Fat, Awkward
Antonym: Thin, Slim, Lean, Delicate
¾¾ Consternation
Synonym: Fear, Disappointment, Dismay, Hopelessness
Antonym: Repose, Peace, Calm, Fearless
¾¾ Concede
Synonym: Yield, Assent, Permit, Sanction
Antonym: Reject, Deny, Dissent, Disallow
¾¾ Commodious
Synonym: Convenient, Suitable, Roomy, Comfortable
Antonym: Inconvenient, Unsuitable, Uncomfortable, Confined
¾¾ Chastise
Synonym: Punish, Admonish, Scold, Reprove
Antonym: Cheer, Comfort, Encourage, Stimulate
¾¾ Despicable
Synonym: Contemptible, Worthless, Shameless, Base
Antonym: Worthy, Decent, Honourable, Respectable
¾¾ Disdain
Synonym: Detest, Despise, Scorn, Loathe
Antonym: Approve, Praise, Love, laud
¾¾ Defray
Synonym: Meet, Bear, Spend, Pay
Antonym: Decline, Declaim, Refuse, Abjure
EBD_7275
90 Vocabulary
¾¾ Dainty
Synonym: Elegant, Delicate, Refined Exquisite
Antonym: Clumsy, Coarse, Unpleasant, Insipid
¾¾ Deplore
Synonym: Lament, Bemoan, Complain, Regret
Antonym: Cheer, Rejoice, Applaud, Celebrate
¾¾ Exult
Synonym: Brag, Triumph, Rejoice, Applaud
Antonym: Lament, Deplore, Bemoan, Grieve
¾¾ Equivocal
Synonym: Uncertain, Vague, Hazy, Ambiguous
Antonym: Lucid, Obvious, Clear, Plain
¾¾ Encumbrance
Synonym: Hindrance, Burden, Obstacle, Impediment
Antonym: Incentive, Stimulant, Patronize, Vantage
¾¾ Earnest
Synonym: Ardent, Sincere, Resolute, Determined
Antonym: Unheeding, Frivolous, Negligent, Careless
¾¾ Enjoin
Synonym: Direct, Counsel, Exhort, Command
Antonym: Prohibit, Forbid, Revolt, Dissuade
¾¾ Fabricate
Synonym: Construct, Produce, Build, Manipulate
Antonym: Destroy, Wreck, Dismantle, Demolish
¾¾ Frantic
Synonym: Violent, Agitated, Frenzied, Wild
Antonym: Subdued, Gentle, Lucid, Coherent
¾¾ Fleeting
Synonym: Transient, Temporary, Ephemeral, Transitory
Antonym: Enduring, Perpetual, Eternal, Unceasing
¾¾ Fickle
Synonym: Wavering, Unreliable, Unsteady, Volatile
Antonym: Resolute, Determined, Inalterable, Invariable
¾¾ Feud
Synonym: Strife, Quarrel, Row, Contention
Antonym: Fraternity, Harmony, Reconciliation, Recompose
¾¾ Guile
Synonym: Cunning, Deceit, Duplicity, Chicanery
Antonym: Honesty, Frankness, Sincerity Integrity
¾¾ Grisly
Synonym: Disgusting, Atrocious, Monstrous, Loathsome
Antonym: Pleasing, Beautiful, Attractive, Alluring
Vocabulary 91
¾¾ Gaudy
Synonym: Garish, Brilliant, Glittering, Tawdry
Antonym: Dull, Faded, Sober, Solemn
¾¾ Genial
Synonym: Cheerful, Pleasant, Joyful, Affable
Antonym: Sullen, Dismal, Morose, Melancholy
¾¾ Grudge
Synonym: Hatred, Aversion, Unwilling, Objection
Antonym: Benevolence, Affection, Goodwill, Kindness
¾¾ Hustle
Synonym: Hurry, Bustle, Haste, Activity
Antonym: Quiet, Lull, Idle, Motionless
¾¾ Haughty
Synonym: Arrogant, Pompous, Obstinate, Imperious
Antonym: Humble, Submissive, Modest, Inoffensive
¾¾ Hapless
Synonym: Unfortunate, Ill-fated, Hostile, Doomed
Antonym: Fortunate, Lucky, Favoured, Satisfied
¾¾ Haggard
Synonym: Exhausted, Emaciated, Lean, Gaunt
Antonym: Exuberant, Active, Robust, Lively
¾¾ Heretic
Synonym: Nonconformist, Secularist, Dissident, Offender
Antonym: Conformable, Adaptable, Religious, Believer
¾¾ Intimidate
Synonym: Frighten, Dictate, Hopeless, Irresolute
Antonym: Console, Encourage, Appease, Hearten
¾¾ Intrepid
Synonym: Brave, Courageous, Valorous Chivalrous
Antonym: Scared, Frightened, Timid, Cowardly
¾¾ Innocuous
Synonym: Salutary, Wholesome, Innocent, Harmless
Antonym: Deleterious, Baneful, Insanitary, Injurious
¾¾ Impute
Synonym: Attribute, Ascribe, Charge, Indict
Antonym: Exculpate, Support, Excuse, Vindicate
¾¾ Indigence
Synonym: Privation, Destitution, Insolvency, Penury
Antonym: Affluence, Abundance, Opulence, Luxury
¾¾ Just
Synonym: Honest, Impartial, Righteous, Upright
Antonym: Unfair, Unequal, Discriminatory, Unseasonable
EBD_7275
92 Vocabulary
¾¾ Jubilant
Synonym: Rejoicing, Triumphant, Gay, Cheerful
Antonym: Melancholy, Depressing, Gloomy, Despondent
¾¾ Jovial
Synonym: Frolicsome, Cheerful, Merry, Exultant
Antonym: Morose, Solemn, Malcontent, Sad
¾¾ Jaded
Synonym: Tired, Exhausted, Fatigued, Languish
Antonym: Renewal, Recreation, Restoration, Refreshed
¾¾ Jejune
Synonym: Dull, Boring, Uninteresting, Monotonous
Antonym: Interesting, Exciting, Piquant, Thrilling
¾¾ Kindred
Synonym: Relation, Species, Relative, Affinity
Antonym: Unrelated, Dissimilar, Heterogeneous, Disparate
¾¾ Keen
Synonym: Sharp, Eager, Acute, Poignant
Antonym: Vapid, Insipid, Blunt, Undesiring
¾¾ Knave
Synonym: Dishonest, Scoundrel, Vagabond, Rogue
Antonym: Paragon, Innocent, Benefactor, Idealist
¾¾ Knell
Synonym: Death knell, Last blow, Demolish, Suppress
Antonym: Reconstruction, Rediscovery, Procreation, Resurrection
¾¾ Knotty
Synonym: Complicated, Difficult, Arduous, Onerous
Antonym: Simple, Manageable, Tractable, Flexible
¾¾ Luscious
Synonym: Palatable, Delicious, Delectable, Delightful
Antonym: Unsavoury, Tart, Sharp, Sour
¾¾ Luxuriant
Synonym: Profusion, Abundant, Dense, Plentiful
Antonym: Scanty, Meagre, Inadequate, Deficient
¾¾ Lucid
Synonym: Sound, Rational, Sane, Coherent
Antonym: Obscure, Hidden, Incomprehensible, Unintelligible
¾¾ Linger
Synonym: Loiter, Prolong, Hesitate, Delay
Antonym: Hasten, Quicken, Dart, Hurry
¾¾ Languid
Synonym: Pensive, Lethargic, Exhausted, Fatigued
Antonym: Lively, Animated, Refreshed, Restored
Vocabulary 93
¾¾ Mutinous
Synonym: Recalcitrant, Insurgent, Unruly, Revolutionary
Antonym: Submissive, Faithful, Complaint, Loyal
¾¾ Murky
Synonym: Dusky, Dreary, Dismal, Bleak
Antonym: Bright, Shining, Luminous, Radiant
¾¾ Mollify
Synonym: Appease, Assuage, Relieve, Mitigate
Antonym: Irritate, Aggravate, Infuriate, Exasperate
¾¾ Mettle
Synonym: Courage, Determination, Stamina, Spirit
Antonym: Timidity, Fear, Cowardice, Diffident
¾¾ Mendacity
Synonym: Falsehood, Deception, Perjury, Perfidious
Antonym: Probity, Honesty, Veracity, Candour
¾¾ Noxious
Synonym: Baneful, Injurious, Pernicious, Disastrous
Antonym: Healing, Profitable, Innocuous, Salubrious
¾¾ Nimble
Synonym: Prompt, Brisk, Lively, Agile
Antonym: Sluggish, Languid, Weary, Tardy
¾¾ Nullify
Synonym: Cancel, Annual, Obliterate, Invalidate
Antonym: Confirm, Uphold, Empower, Endorse
¾¾ Nauseous
Synonym: Unsavoury, Loathsome, Abominable, Repellent
Antonym: Worthy, Commendable, Upright, Inoffensive
¾¾ Novice
Synonym: Tyro, Beginner, Debutant, Apprentice
Antonym: Veteran, Ingenious, Experienced, Mentor
¾¾ Overwrought
Synonym: Excited, Agitated, Fervent, Intense
Antonym: Quiet, Tranquil, Composed, Cool
¾¾ Oversight
Synonym: Omission, Error, Fault, Slip
Antonym: Precision, Observance, Circumspection, Caution
¾¾ Ostentation
Synonym: Display, Pretension, Vaunt, Pomposity
Antonym: Modesty, Constraint, Diffidence, Economy
¾¾ Odious
Synonym: Abhorrent, Obnoxious, Prejudice, Malevolent
Antonym: Engaging, Fascinating, Endearing, Captivating
EBD_7275
94 Vocabulary
¾¾ Obstinate
Synonym: Stubborn, Resolute, Unyielding, Obdurate
Antonym: Submissive, Obedient, Amenable, Pliable
¾¾ Puerile
Synonym: Shallow, Immature, Childish, Trivial
Antonym: Wise, Farsighted, Profound, Sensible
¾¾ Prudent
Synonym: Cautious, Discreet, Judicious, Circumspect
Antonym: Impetuous, Unwise, Reckless, Rash
¾¾ Protract
Synonym: Prolong, Delay, Stretch, Procrastinate
Antonym: Abbreviate, Curtail, Abridge, Compress
¾¾ Pillage
Synonym: Ransack, Ravage, Despoil, Plunder
Antonym: Recompense, Recover, Redeem, Restoration
¾¾ Pompous
Synonym: Haughty, Arrogant, Flamboyant, Florid
Antonym: Unpretentious, Humble, Coy, Modest
¾¾ Quell
Synonym: Subdue, Reduce, Suppress, Extinguish
Antonym: Exacerbate, Agitate, Foment, Instigate
¾¾ Quaint
Synonym: Queer, Strange, Odd, Ridiculous
Antonym: Familiar, Usual, Common, Normal
¾¾ Quibble
Synonym: Equivocate, Prevaricate, Evade, Dissemble
Antonym: Unfeigned, Plain, Scrupulous, Conscientious
¾¾ Quash
Synonym: Abrogate, Annual, Cancel, Revoke
Antonym: Uphold, Empower, Authorize, Permit
¾¾ Quarantine
Synonym: Isolate, Separate, Seclude, Screened
Antonym: Gregarious, Amiable, Sociable, Companionable
¾¾ Rustic
Synonym: Pastoral, Bucolic, Rural, Uncivilised
Antonym: Cultured, Refined, Urban, Urbane
¾¾ Rout
Synonym: Defeat, Overthrow, Vanquish, Subjugate
Antonym: Succumb, Withdraw, Retreat, Consolidate
¾¾ Rescind
Synonym: Annual, Abrogate, Revoke, Repeal
Antonym: Delegate, Permit, Authorize, Propose
Vocabulary 95
¾¾ Repugnant
Synonym: Hostile, Offensive, Disagreeable, Distasteful
Antonym: Agreeable, Pleasant, Friendly, Tasteful
¾¾ Raze
Synonym: Demolish, Destroy, Annihilate, Dismantle
Antonym: Restore, Construct, Repair, Build
¾¾ Sway
Synonym: Influence, Control, Command, power
Antonym: Impotence, Futility, Disability, Incapacity
¾¾ Sycophant
Synonym: Parasite, Flatterer, Cringing, Servile
Antonym: Devoted, Loyal, Truthful, Faithful
¾¾ Subvert
Synonym: Overthrow, Suppress, Demolish, Sabotage
Antonym: Accomplish, Sustain, Generate, Organize
¾¾ Squalid
Synonym: Dirty, Soiled, Filthy, Odious
Antonym: Attractive, Tidy, Polished, Spruce
¾¾ Saucy
Synonym: Impudent, Insolent, Brazen, Impertinent
Antonym: Modest, Humble, Esteem, Meek
¾¾ Trivial
Synonym: Trifling, Insignificant, Frivolous, Worthless
Antonym: Significant, Important, Consequential, Essential
¾¾ Throng
Synonym: Assembly, Gathering, Congregation, Crowd
Antonym: Dispersion, Scattering, Handful, Sparsely
¾¾ Trite
Synonym: Ordinary, Commonplace, Stale, Hackneyed
Antonym: Interesting, Extraordinary, Becoming, Proper
¾¾ Temerity
Synonym: Boldness, Audacity, Imprudence, Indiscretion
Antonym: Discretion, prudence, Caution, Wisdom
¾¾ Taciturn
Synonym: Reserved, Uncommunicative, Silent, Reticent
Antonym: Talkative, Loquacious, Garrulous, Extrovert
¾¾ Utterly
Synonym: Completely, Entirely, Extremely, Wholly
Antonym: Deficient, Incomplete, Insufficient, Partial
¾¾ Usurp
Synonym: Seize, Wrest, Encroach, Coup
Antonym: Restore, Compensate, Reinstate, Grant
EBD_7275
96 Vocabulary
¾¾ Unseemly
Synonym: Undesirable, Inappropriate, Uncouth, awkward
Antonym: Becoming, Acceptable, Decorous, Admirable
¾¾ Ungainly
Synonym: Clumsy, Unskilled, Immature, Slovenly
Antonym: Active, Expert, Skilful, Dexterous
¾¾ Uncouth
Synonym: Awkward, Ungraceful, Inelegant, Vulgar
Antonym: Elegant, Graceful, Distinguished, Shapely
¾¾ Vouch
Synonym: Confirm, Consent, Approve, Endorse
Antonym: Repudiate, prohibit, Recant, Retract
¾¾ Vicious
Synonym: Corrupt, Obnoxious, Degraded, Demoralized
Antonym: Noble, Virtuous, Innocent, Undefiled
¾¾ Vanity
Synonym: Conceit, Pretension, Immodesty, Pride
Antonym: Modesty, Humility, Meek, Bashful
¾¾ Valour
Synonym: Bravery, Prowess, Heroism, Chivalry
Antonym: Fear, Cowardice, Unmanliness
¾¾ Valiant
Synonym: Brave, Gallant, Courageous, Chivalrous
Antonym: Fearful, Afraid, Coward, Dastardly
¾¾ Wary
Synonym: Cautious, Circumspect, Prudent, Chary
Antonym: Heedless, Negligent, Impulsive, Reckless
¾¾ Withhold
Synonym: Reserve, Restrain, Hamper, Retard
Antonym: Emancipate, Liberate, Dispense, Release
¾¾ Wane
Synonym: Decline, Dwindle, Decrease, Deteriorate
Antonym: Ameliorate, Rise, Revive, Wax
¾¾ Wayward
Synonym: Volatile, Capricious, Unstable, Inconstant
Antonym: Stable, Determined, Resolute, Straight
¾¾ Wilful
Synonym: Stubborn, Obstinate, Obdurate, Inexorable
Antonym: Amenable, Irresolute, Pliable, Yielding
¾¾ Yawn
Synonym: Gape, Sleepy, Slumber, Doze
Antonym: Active, Close, Brisk, Wakeful
Vocabulary 97
¾¾ Yoke
Synonym: Connect, Harness, Hitch, Shackle
Antonym: Liberate, Release, Detach, Disconnect
¾¾ Yield
Synonym: Surrender, Abdicate, Succumb, Consent
Antonym: Resist, Protect, Prohibit, Forbid
¾¾ Yell
Synonym: Shout, Shriek, Exclaim, Gesticulate
Antonym: Suppress, Whisper, Muffled, Muted
¾¾ Yearn
Synonym: Languish, Crave, Require, Pine
Antonym: Content, Unwanted, Satisfied, Gratified
¾¾ Zest
Synonym: Delight, Enthusiasm, Energetic
Antonym: Disgust, Passive, Detriment, Languid
¾¾ Zigzag
Synonym: Oblique, Crooked, Winding, Wayward
Antonym: Straight, Even, Direct, Unbent
¾¾ Zenith
Synonym: Summit, Apex, Maximum, Pinnacle
Antonym: Nadir, Base, Floor, Bottom
¾¾ Zeal
Synonym: Eagerness, Fervour, Enthusiasm, Ardour
Antonym: Apathy, Lethargy, Indifference, Reluctant
¾¾ Zealot
Synonym: Fanatic, Partisan, Bigot, Chauvinist
Antonym: Tolerant, Liberal, Blasphemy, Impious
Spelling Errors
Questions based on English spelling errors are asked in many exams. At times,
we commit mistakes by choosing the wrong option considering it to be the right
spelling of the word as we are in habit of using the wrongly spelt words since we
started writing English. Spellings can be tricky because spellings are not based on
phonetics and also because there are many exceptions.
Below is a table of some incorrect words with their correct spellings. You should
read and analyze them thoroughly so that you do not commit mistake while writing.

INCORRECT CORRECT
Abscence Absence
Accomodate Accommodate
Arguement Argument
EBD_7275
98 Vocabulary

Assasination Assassination
Athiest Atheist
Attendence Attendance
Bizzare Bizarre
Calender, Calander Calendar
Commitee Committee
Concious Conscious
Convinient Convenient
Copywrite Copyright
Deterioreit Deteriorate
Embarass Embarrass
Explaination Explanation
Foreward Foreword
Foriegn Foreign
Fourty Forty
Grammer Grammar
Guidence Guidance
Heros Heroes
Hypocracy Hypocrisy
Interupt Interrupt
Lollypop Lollipop
Managable Manageable
Mispell Misspell
Neice Niece
Noticable Noticeable
Occured Occurred
Opthamologist Ophthalmologist
Pavillion Pavilion
Persue Pursue
Posession Possession
Vocabulary 99

Preceeding Preceding
Priviledge Privilege
Pronounciation Pronunciation
Que Queue
Questionaire Questionnaire
Reccomend Recommend
Rediculous Ridiculous
Rythm Rhythm
Seige Siege
Seperate Separate
Sincerly Sincerely
Supercede Supersede
Tatoo Tattoo
Tendancy Tendency
Tommorrow Tomorrow
Tounge Tongue
Truely Truly
Untill Until
Vaccum Vacuum
Vegeterian Vegetarian
Wierd Weird
Yachet Yacht

Analogies
Analogy, literally means a comparison or a comparable similarity. A student has
to find a pair of words in the same relation or a similar relation as that of the given
pair of words. Analogy is, in a sense, a test of vocabulary since you need to know
the meaning of the words given, but in a broader sense, it is a test of reasoning
ability. To know the meaning of the words will not be enough if one is not able to
understand clearly what the relation between the pairs of words is. Therefore, there
are two things that are important to attempt a question on analogy:
(i) meaning of all given words
(ii) relationship between the given pairs of words
EBD_7275
100 Vocabulary
It is more convenient and time saving to first figure out the relation between the
given pair and then compare it with the relations between the pairs in the options
given for choice. Consider the following example
Pen : Write : : Book : ?
Now, first determine the relation between the first two words, it is that of purpose,
pen is used to write. Then determine the other word which will be in the same
relation to the third word. Book is used to read, then
Pen : Write : : Book : Read
There are different kinds of relationships that could be drawn from daily usage but
some common relationships are given below:
1. CAUSE : EFFECT
Liquor : Intoxication → Liquor causes intoxication
Wound : Pain → wound causes pain.
In this relation, the first word is the cause for the second and the second is the
result of the first
2. PURPOSE
Bottle : Cork → a cork is used to close a bottle
Dress : Cloth → cloth is used to make a dress
In this relation, one word is used for another i.e. there is a purpose between
the two
3. OBJECT : ACTION
Gun : Fire → you fire a gun
Violin : Play → you play a violin
In this, one term is an object and the other one is an action undertaken with
the help of that object.
4. ACTION : OBJECT
Foment : Riot → you foment a riot
Wear : Clothes → you wear clothes
This is opposite to the previous relation. Here, the first word is the action and
the second the object with which that action is done.
5. PART : WHOLE
Book : Literature → a book is a part of the larger body of literature
Ship : Fleet → ship is a part of the collection called fleet
In this relation, the first word will in the same way be a constituent of a bigger
body represented by the second word.
6. SYNONYMS
Abundant : Ample → ample means the same as abundant
Skilled : Adroit → the two words are synonymous, i.e., they mean the same
This relation is when both the words are synonyms.
7. ANTONYMS :
Abstinence : Indulgence → Indulgence means the opposite of Abstinence
Legitimate : Unlawful → Legitimate means legal which is the opposite of
unlawful.
In this relation, the two words are opposite to each other in meaning.
Vocabulary 101
8. SECONDARY SYNONYMS :
Callous : Indifference → The synonym of callous will be indifferent, since
both words are adjectives but rather the noun form, indifference, has been
given in the relation
Brainwave: Inspired → The synonym of Brainwave is inspiration, but instead
the second word in this relation is Inspired - the one who has inspiration.
In this relation, the two words are not directly synonymous but a slight change
of the part of speech has been made in the second word.
9. WORKER: ARTICLE CREATED
Carpenter : Furniture → carpenter makes wooden furniture
Compose : Music → a composer composes or creates music
In this relation, the first word is the doer and the second is the professional
work done by the first.
10. SYMBOL : QUALITY
Olive leaf : Peace → an olive leaf is a symbol of peace.
Red : Passion → the colour red symbolises passion.
In this relation the first word is a symbol, and the second is the meaning
represented by the symbol.
11. CLASS : MEMBER
mammal : Man → man belongs to the class of mammals.
Doggerel : Poem → Doggerel is a class of poem which is bad in quality.
In this relation the first word is a member belonging to the class denoted by
the second word.
12. ACTION : SIGNIFICANCE
Blush : Embarrassment → if one blushes, that signifies that the person is
embarrassed.
Spasm : Pain → a spasm indicates that the person is in pain.
In this relation, the first word is an action and the second is what that action
signifies.

Although, most of the questions asked in a competitive exam can be solved with
the help of the given relationships, for subtle questions, a student should apply
reasoning to figure out the relation between the given words. Following are certain
tips that would help a student to attempt analogy questions.
 TIP 1
The first and foremost step while attempting an analogy question
should be to define the relationship. To avoid any errors, first define the
relationship on paper or in your mind before searching for options. Once
you have defined the relationship analyse the given pairs in the light of
the relationship.
EBD_7275
102 Vocabulary
(1) ANXIOUS : REASSURANCE
→ resentful : gratitude
→ perplexed : classification
→ insured : imagination
→ vociferous : suppression
First, the relationship can be defined as ‘need’ i.e. an anxious person needs
reassurance and then you can check the given pairs to find out that ‘a perplexed
person needs classification’. Thus, this will be the right analogy.

(2) SIMMER : BOIL


→ Cook : Fry
→ Chill : Freeze
→ Roast : Stew
→ Slice : Cut
Now, establish the relation between the two given words. It is that of degree.
Simmer is the lower degree of boil. Just as chill is the lower degree of freeze.

 TIP 2
Always be careful about apparent and easy similarity. These are only to
deceive the student as you would be attracted by these options. Always
confirm all the options and be highly careful while considering an obvious
answer.

e.g. STUTTER : SPEECH


→ Blare : hearing
→ Aroma : smelling
→ Astigmatism : sight
→ Novocaine : Touch
Stutter is a defect of speech, so the relation between the two is that of defect.
But Blare and hearing are closely related since blare means a harsh sound.
This may attract the student, but this is not a relation of defect. This relation
is in the third option, astigmatism is a defect of sight. So, always avoid giving
into the temptation of obviously correct answers.

 TIP 3
Sometimes, a word has two meanings, while what may first come to your
mind will be the more frequent use of that word. If you cannot find a logical
relation between the two words. Go beyond the obvious meaning and link
the word with the other meaning of the second word.
Vocabulary 103

MAROON : SAILOR
→ Red : Ship
→ Crimson : Flower
→ Stranded : Tourist
→ Colour : Dress
Maroon also has two meanings the colour ‘maroon’ and the verb maroon
which means being left alone or abandoned. Obviously the second meaning
will make a logical relation with sailor, a sailor is marooned just as a tourist
is stranded.

TIPS AND TECHNIQUES FOR VOCABULARY

¾¾ Read regularly from a variety of sources like newspapers, magazines and blogs
etc. as this habit will help you learn new words as well usages of words in different
contexts.
¾¾ Make a target of learning a new word daily. This technique is used by many people
and they have successfully enhanced their vocabulary.
¾¾ You should play word games like – Crossword puzzles, Anagrams, Word jumbles
and Scrabbles etc. This method will help you discover new words.
¾¾ Engage yourself in talking with people in English whenever you get an opportunity
so that you may practice the words that you have learnt. Apart from this, it will also
help you learn new words during the conversation and in addition help you enhance
your fluency in spoken English.
EBD_7275
READING COMPREHENSION

Chapter
OBJECTIVE
17 COMPREHENSION
Reading Comprehension contains a lot of questions in almost all competitive
examinations and thus, it is very important to attempt the questions of this section.
Reading comprehension is the most favoured section in English because it doesn’t
require prior knowledge to solve RCs (Reading Comprehensions). Greater chances
of high accuracy in it act as a game changer in clearing the cut off in English
section.

In this chapter, we will discuss some tips and techniques to easily approach RCs
with high accuracy.

¾¾ You need to have ample RCs practice on computer to pass this section
with flying colours because while attempting RCs online, you can’t
underline important points and mark different areas in the passage and
therefore, you need to hone your skills by practising regularly.
¾¾ Quickly skim through the passage before you read the passage thorough-
ly or attempt the questions.
¾¾ Before you start reading the passage, go through the questions that need
to be answered. This will give you a fair idea about what the passage
talks about. Once you start reading the passage, you can start locating the
answers to questions.
¾¾ It is very important to make inferences while reading the passage because
most of the questions are not asked directly. Your understanding of the
passage and its theme is of utmost importance as it helps you to eliminate
the wrong options and pick the right one.
¾¾ Before reading the entire passage, first, read the first and last paragraph
of the RC to have an idea what the author is saying in the paragraph. This
will help you in having an overview of the whole passage.
¾¾ When you are attempting a question based on phrase, just read the two-
three lines above and below that phrase to have an idea what is implicit
from that phrase.
objective Comprehension 105
¾¾ If there are questions on vocabulary then you should attempt them first as
it is quite easy to pick the antonyms and synonyms. You don’t have to an-
swer the questions in the order they appear to you because in the exams,
you have option to skip the questions and move to the next one and again
come back to the previous ones as per your choice.
¾¾ In most of the cases, elimination of choices works better than selection of
choices. You should try to eliminate those options which are broad, nar-
row, odd and irrelevant to the question asked. This strategy will help you
reach the correct choice easily along with high accuracy.
¾¾ Mark the answers only if you are sure of it and make sure you don’t go
for wild guess.
¾¾ Never use your past knowledge about the topic to answer any question.
¾¾ Read articles in newspapers on regular basis to improve your reading
speed and vocabulary.
Examples:

DIRECTIONS (QS. 1 - 10): Read the following passage carefully and


answer the questions given below it. Certain words/ phrases have
been printed in bold to help you locate them while answering some
of the questions.

The Indian education sector is one of the largest sunrise sectors


contributing to the country’s economic and social growth. The Indian
education system, considered as one of the largest in the world, is divided
into two major segments of core and non-core businesses. While, schools
and higher education form the core group, the non-core business consists
of pre-schools, vocational training and coaching classes. The education
sector in India is evolving, led by the emergence of new niche sectors
like vocational training, finishing schools, child-skill enhancement and
e-learning. India has emerged as a strong potential market for investments
in training and education sector, due to its favourable demographics
(young population) and being a services-driven economy. Indian
education sector’s market size in Financial Year, 2012-13 estimated to
be USD 71.2 billion is expected to increase to USD 109.8 billion by
Financial Year 2015-2016 due to the expected strong demand for quality
education. The market grew at a CAGR of 16.5% during Financial Year
2015-2016.
EBD_7275
106 objective Comprehension
Education has been made an important and integral part of the national
development efforts. The tremendous increase in the number of students
and of educational institutions has given rise to the term ‘education
explosion’. No doubt, this has resulted in serious problems such as
inadequacy of financial resources and infrastructure and dilution of
personal attention to the education and character-formation of the
students. Also, there is the unwanted side-effect of enormous increase
in the number of educated unemployed. However, we cannot overlook
the advantages of education explosion in India. Mere increase in the
percentage of literate people does not indicate a qualitative change in
the educational standards of the people and a substantial improvement in
manpower resources of India. Unemployment problem in India cannot
be blamed on the availability of large masses of educated people in
India. Uncertainty and vacillation have marked the government’s policy
regarding the medium of education in India. While the government
policy in this respect has not changed, a significant increase in the
number of schools-primary and secondary-imparting education through
the English medium is a significant development. Thousands of nursery
schools that have mushroomed since the last decade purport to impart
education to infants through English. This is an unwanted development
which has been deprecated by educationalists and political leaders.
Regarding the medium of instruction in colleges and universities, many
State Governments have already decided, in principle, to switch over to
the regional language. However, the implementation in this respect has
remained very slow.

Today, virtually, every university in India is offering correspondence


courses for different degrees and diplomas. In fact, correspondence
education has opened new vistas for the educational system which could
not successfully meet the challenging problem of providing infrastructure
for multitudes of new entrants into the portals of higher education. The
public demand for higher education was initially met through evening
colleges; now correspondence education has come to the rescue of the
worried education administrators.
objective Comprehension 107
1. Which of the following facts is not true regarding the Indian education sector
as per the passage?
(a) It is still in the process of development.
(b) It is one of the contributors to India’s growth.
(c) There has been a recent trend towards the adoption of regional languages
as the medium of instruction.
(d) Mushrooming of schools imparting English education has been
appreciated.
(e) The number of educated unemployed has increased.
2. As per the passage, India’s education system has been able to attract
investments because of
A. The demographic factor.
B. The Indian economy being service-driven.
C. Indian democratic governance being an attractive issue.
(a) Only A (b) Only B
(c) Only C (d) Both A and B
(e) Both B and C
3. As per the passage, which of the following explains the term ‘education
explosion’?
A. Huge investment in the education sector.
B. Pro-active government policy towards the education sector.
C. Spurt in the number of students and educational institutions.
(a) Both A and B (b) Only C
(c) Both B and C (d) Only B
(e) All of the above statements are correct
4. Which of the following can be inferred from the passage?
A. Increase in literacy level signifies a qualitative increase in educational
attainment of people.
B. Literacy levels are closely related to improvement in manpower
resources.
C. The existence of educated people does not necessarily contribute to the
problem of unemployment.
(a) Only A (b) Only B
(c) Only C (d) Both A and B
(e) All A, B and C
EBD_7275
108 objective Comprehension
5. According to the passage, which of the following statement(s) is/are correct?
(a) Increase in English medium schools is a welcome sign for the Indian
education sector.
(b) Increase in English medium schools in India is an insignificant, though
wanted development.
(c) Correspondence education has proved to be a panacea in terms of
educating people without proper infrastructure.
(d) The implementation of regional languages as medium of instruction has
been quite fast.
(e) The prospects for future growth of India’s education sector look bleak.
6. Which of the following statements cannot be said to be the highlights of the
passage?
A. India’s education sector is marked by increase in the number of educated
people paralleled by simultaneous growth in unemployment.
B. There are both positive and negative aspects of the education explosion
in India.
C. The government policy towards education has been that of clarity marked
by sincere efforts.
(a) Only A (b) Only B
(c) Only C (d) All A, B and C
(e) None of these
DIRECTIONS (Q.7-8): Choose the word which is most similar in meaning to the
words printed in bold as used in the passage.
7. Dilution
(a) Thickening (b) Concentration
(c) Extension (d) Diminution
(e) Development
8. Emergence
(a) Disappear (b) Rise
(c) Abandonment (d) Fall
(e) Lessening
objective Comprehension 109
DIRECTION (Qs.9-10): Choose the word which is most opposite in meaning to
the words printed in bold as used in the passage.
9. Deprecate
(a) Derogate (b) Frown
(c) Object (d) Commend
(e) Disparage
10. Purport
(a) Insignificant (b) Connotation
(c) Acceptation (d) Intention
(e) Purpose

ANSWERS & EXPLANATION

1. (d) All the sentences except sentence D are correct. It is clearly mentioned
in the second last paragraph that thousands of nursery schools that have
mushroomed since the last decade is an unwanted development which
has been deprecated by educationalists and political leaders.
2. (d) It is clearly mentioned in the first paragraph as “India has emerged as a
strong potential market for investments in training and education sector,
due to its favourable demographics (young population) and being a
services-driven economy.”
3. (c) ‘Education explosion’ as mentioned in the paragraph means a tremendous
rise in the number of educational institutions and students.
4. (c) Option (c) can be inferred from the second paragraph where it says that
unemployment problem in India can’t be blamed on the availability of
large masses of educated people in India.
5. (c) Option (c) can be inferred from the last paragraph where it says that
correspondence education has opened new vistas for the education
system which could not be met earlier because of the challenges of
providing necessary infrastructure for it.
6. (c) Option (a) and (b) are clearly mentioned in the passage. Option (c) is
contradictory to the lines mentioned in the second last paragraph where it
says that uncertainty and vacillation have marked the government policy
towards medium of education in India.
7. (d) Dilution means weakening in force, content or value. Diminution means
reduction is size, content or importance.
EBD_7275
110 objective Comprehension
8. (b) Emergence means the process of becoming visible or coming into
existence or prominence. Rise means to move from lower position to
higher position or go up.
9. (d) Deprecate means to express disapproval. Commend means to praise or
approve.
10. (d) Purport used as a verb in the passage means to claim, profess or pretend
falsely. Intention in general is clear as expressed.

NEW PATTERN READING COMPREHENSION


DIRECTIONS (1-5): Read the following passage divided into number of
paragraphs carefully and answer the questions that follow it.
Paragraph 1: Judiciary has become the centre of controversy, in the recent past,
on account of the sudden ‘Me’ in the level of judicial intervention. The area of
judicial intervention has been steadily expanding through the device of public
interest litigation. The judiciary has shed its pro-status-quo approach and taken
upon itself the duty to enforce the basic rights of the poor and vulnerable sections
of society, by progressive interpretation and positive action. The Supreme Court
has developed new methods of dispensing justice to the masses through the public
interest litigation.
Paragraph 2: Former Chief Justice PN. Bhagwat, under whose leadership public
interest litigation attained a new dimension comments that “the Supreme Court
has developed several new commitments. It has carried forward participative
justice. It has laid just standards of procedure. It has made justice more accessible
to citizens”. The term ‘judicial activism’ is intended to refer to, and cover, the
action of the court in excess of, and beyond the power of judicial review. From one
angle it is said to be an act in excess of, or without, jurisdiction. The Constitution
does not confer any authority or jurisdiction for ‘activism’ as such on the Court.
Paragraph 3: Judicial activism refers to the interference of the judiciary in
the legislative and executive fields. It mainly occurs due to the non-activity of
the other organs of the government. Judicial activism is a way through which
relief is provided to the disadvantaged and aggrieved citizens. Judicial activism
is providing a base for policy making in competition with the legislature and
executive. Judicial activism is the rendering of decisions, which are in tune with
the temper and tempo of the times.
Paragraph 4: In short, judicial activism means that instead of judicial restraint,
the Supreme Court and other lower courts become activists and compel the
authority to act and sometimes also direct the government regarding policies and
also matters of administration.
objective Comprehension 111
Paragraph 5: Judicial activism has arisen mainly due to the failure of the executive
and legislatures to act. Secondly, it has arisen also due to the fact that there is a
doubt that the legislature and executive have failed to deliver the goods. Thirdly,
it occurs because the entire system has been plagued by ineffectiveness and
inactiveness. The violation of basic human rights has also led to judicial activism.
Finally, due to the misuse and abuse of some of the provisions of the Constitution,
judicial activism has gained significance.
1. What does the author want to convey in Paragraph 1?
(I) Certain personal issues and agendas in recent past in the level of judicial
intervention have put a question mark on the credibility of the apex court.
(II) The Supreme Court is very concerned about the under-privileged sections
of the society and thus, has come up with an innovative idea to dispensing
justice.
(III) Public Interest Litigation is a boon for the poor and vulnerable sections of the
society as far as the enforcement of their basic rights is concerned.
(a) Only (I) (b) Only (II)
(c) Both (II) and (III) (d) All (I), (II) and (III)
(e) None of these
2. What is the meaning of the sentence ‘From one angle, it is said to be an act in
excess of, or without jurisdiction’ in the context of paragraph 2?
(I) Judicial activism can be exercised by the Supreme Court as it is beyond
the power of judicial review.
(II) Judicial activism does not find any mention in the constitution as far as
its authority or jurisdiction is concerned.
(III) The constitution has not limited the authority or jurisdiction to ‘activism’
hence, it can be exercised over the courts as per the say of the Supreme
Court.
(a) Only (I) (b) Only (II)
(c) Only (III) (d) Both (II) and (III)
(e) Both (I) and (II)
3. What can’t be said about judicial activism in accordance to paragraph 3?
(a) It emerges when the legislative and the judiciary do not do justice with
their work.
(b) It has the right to put question mark on the legislative and executive part
of the government.
(c) It is exercised in accordance with the demand of the situation.
(d) It has given the citizenry ample power to demand for their rights from the
judiciary.
(e) It is giving a foundation for policy making in competition with the other
organs of the government.
EBD_7275
112 objective Comprehension
4. Which of the following statements does not relate to Paragraph 5?
(a) Violation of basic human rights has led to judicial activism.
(b) There is a suspicion on the executive and the legislative organ of the
government as far as their working is concerned.
(c) Judicial activism has carried forward participative justice thereby
justifying that the entire system has been plagued by ineffectiveness and
inactiveness.
(d) Some of the constitutional provisions have been misused.
(e) The failure of the other organs of the government has led to the emergence
of judicial activism.
5. What does the author mean by the phrase ‘judicial restraint’ in Paragraph 4?
(I) The limiting of the exercise of their own power by the courts.
(II) The restriction of power which Supreme Court can impose on other
lower courts.
(III) The liberation of judicial power of the Supreme Court and other courts.
(a) Only (I) (b) Only (II)
(c) Only (III) (d) Both (I) and (II)
(e) All (I), (II) and (II)

ANSWER KEY

1. (e)
2. (b)
3. (d)
4. (c)
5. (a)
PARAJUMBLES

Chapter
REARRANGING WORDS
18 OR SENTENCES

Parajumbles is the lexical term used for the kind of questions wherein the
sentences of a paragraph are jumbled and the examinee is required to figure out
the logical sequence of the sentences that would render the paragraph meaningful
and grammatically correct.
Such questions basically pertain to rearrangement of a given set of sentences. At
times, instead of sentences of a paragraph, phrases of a complex sentence may be
jumbled for the candidate to arrange logically.
I. Let us first discuss the single sentence with its parts jumbled.
THINGS TO BEAR IN MIND/ STRATEGIES TO EMPLOY:
1. Every sentence has a subject and a predicate. So, locating the subject
will give headway in arranging the parts of the sentence sequentially. The
subject of a sentence is a noun or a pronoun.
2. Nouns are always mentioned first and later get replaced by pronouns.
e.g. Raman told Mona that he trusted her.
(noun) (noun) (pronoun) (pronoun)
Heena promised herself that she would not lie again.
(noun) (pronoun) (pronoun)
3. The predicate contains the verb of the subject. So, locate the verb.
4. A sentence may be either in active or in passive voice. In the active
voice, the sentence follows the structure, subject+verb+object+prepositi
on.
e.g. Adit helped them in their work.
(subject) (verb) (object)
In the passive voice, the ‘by phrase’ containing the doer of the action
occurs towards the end of the sentence
e.g. They were helped in their work by Adit.
or at the end of a clause with correlative conjunction.
e.g. She was so spoilt by her parents that she threw tantrums every now
and then.
EBD_7275
114 rearranging words or sentences
5. Connectors occur generally in the middle or alternatively, in the
beginning of the sentence.
e.g. If you are happy, you will be healthy.
You will be healthy if you are happy.
That he is ill is no news.
It is no news that he is ill.
6. Definite article the comes later to refer to the noun already mentioned. As
an opener, it can come only with singularly known nouns like sun, moon,
monuments, newspapers, trains etc.
EXAMPLES
Example 1.
P: where Gandhi Ji stood
Q: our long discussions about socialism
R: had left me
S: rather bewildered and confused as to just
(a) PSQR (b) QRSP
(c) SPQR (d) PSRQ
Step 1. Looking for the subject
Observing the parts, we can eliminate R and S as they are parts of the predicate.
So, option c) is ruled out. If we take P is the opening part, it may be followed by
R and S but Q doesn’t fit in the sequence. So, options a) and d) get eliminated.
Therefore, the opening part, containing the subject, will be Q.
Step 2. Arranging the predicate
Q may be followed by R or S.
The sequence QRSP makes good, complete sense.
The sequence QSP... is unable to include R.
Therefore, the sequence QRSP which is option b) is the correct one.
Example 2.
1: which is essential for its complete understanding
2: the five page introduction to the script places it in the right perspective
3: and gives the uninitiated reader
rearranging words or sentences 115
4: a background to the film
(a) 3214 (b) 2341
(c) 1324 (d) 4231
Step 1. Locating the subject
Parts 1 and 3 are eliminated as they start with connectors which means they are
parts of the predicate. So, options a) and c) get ruled out.
Step 2. Arranging the predicate
If we take 2 as the opening part, the sequence may be 2341, which makes complete
sense. With 4 as the first part, the sequence may be 413... but including 2 gives an
absurd sense. 4 cannot be followed by 3 or 2, hence, d) is eliminated.
Therefore option b) is the correct one.
Example 3.
1: environmental and psychological stress and strain
2: he said that
3: the development of positive attitudes to cope with
4: what the world needed today was
(a) 1432 (b) 2341
(c) 4213 (d) 2431
Step 1. Locating the subject
By observation, any of the four parts may be the subject, provided they can take a
suitable predicate for themselves.
Step 2. Arranging the predicate
None of the other three fits as predicate to 1. So, option a) is eliminated.
2 as the subject may take the sequence 2431 which makes correct sense. It may
take 2314 or 2134, both of which are meaningless. So, option b) is ruled out.
With either 3 or 4 as the subject, only 1 can follow it and the rest do not fit. Hence,
option c) is wrong.
Therefore, option d) is correct.
II. Now, we shall discuss the jumbled paragraph, wherein the sentences of the
paragraph are given in a random order and need to be sequenced logically.
EBD_7275
116 rearranging words or sentences

THINGS TO BEAR IN MIND/ STRATEGIES TO FOLLOW:


1. Every passage will have a central theme. It helps to identify it and
sequence the sentences logically in accordance with the structure of
(i) Premise
(ii) Support
(iii) Example
(iv) Progression
(v) Conclusion
Often, the opening sentence starts with ‘it is...’
2. The opening sentence will have a noun rather than a pronoun. That is, it
will introduce a person, place, body, group or any other entity.
3. Sentences bearing personal pronouns, that is, you, he, she, it, him,
her, they or demonstrative pronouns this, that, these, those will always
come later. They establish pairs with other noun-bearing sentence of the
paragraph.
4. Adjectives showing a degree of comparison, for example better, more,
worse etc., establish link with other sentences.
5. Some links or pairs of sentences of the given paragraph can be identified
from the options. These may be based on the subject, time sequence
(then, later, next, before, after etc.), noun-pronoun sequence, etc.
6. Sometimes, a sentence stands as an example of another. It will always
come later than the sentence for which it is working as an example.
7. Signal words of support, for example and, also, as well, beside, to, in
fact, moreover, likewise, similarly, additionally, furthermore, etc.;
and of contrast, for example although, yet, despite, in spite of, instead of,
while, whereas, on the other hand, on the contrary, nevertheless, none
the less, however, still, ironically, surprisingly, paradoxically etc. are
never the opening sentence. They follow the sentence that they support
or contrast.
8. Words such as therefore, because, consequently, hence, thus, given that,
in order to, when/if...then, so/such...that, accordingly etc. Also show
pairs of cause and effect sentences, where one causes or determines
another which follows it logically.
9. While the whole paragraph will have to be rearranged when TITA (type
in the answer) response is required this need not be done in multiple-
choice type. The best course is to eliminate options and zero in on the
right one as fast as one can.
rearranging words or sentences 117
EXAMPLES BASED ON LATEST PATTERN
1. If sentence (B) “The Finance Ministry’s warning to potential investors in
bitcoin and other cryptocurrencies has come at a time when a new, seemingly
attractive investment area has opened up that few have enough information
about.” is the first sentence, what is the order of other sentences after
rearrangement?
(A) One of the main reasons for this volatility is speculation and the entry
into the market of a large number of people lured by the prospect of quick
and easy profits.
(B) The Finance Ministry’s warning to potential investors in bitcoin and other
cryptocurrencies has come at a time when a new, seemingly attractive
investment area has opened up that few have enough information about.
(C) A number of investors, daunted by the high price of bitcoin, have put their
money into less well-established and often spurious cryptocurrencies,
only to lose it all.
(D) Investment in bitcoin and other cryptocurrencies increased tremendously
in India over the past year, but most new users know close to nothing
of the technology, or how to verify the genuineness of a particular
cryptocurrency.
(E) The price of bitcoin, the most popular of all cryptocurrencies, not
only shot up by well over 1000% over the course of the last year but also
fluctuated wildly.
(F) The government’s caution comes on top of three warnings issued by the
Reserve Bank of India since 2013.
(a) CDEFA (b) EAFDC
(c) DCAEF (d) ECDAF
(e) FEDAC
2. If sentence (C) “Clinical trials involving human subjects have long been a
flashpoint between bioethicists and clinical research organisations (CROs)
in India.” is the first sentence, what is the order of other sentences after
rearrangement?
(A) Such over-volunteering occurs more frequently in bioequivalence
studies, which test the metabolism of generics in healthy subjects.
(B) Landmark amendments to the Drugs and Cosmetics Act in 2013 led to
better protection of vulnerable groups such as illiterate people, but more
EBD_7275
118 rearranging words or sentences
regulation is needed to ensure truly ethical research.
(C) Clinical trials involving human subjects have long been a flashpoint
between bioethicists and clinical research organisations (CROs) in India.
(D) The big problem plaguing clinical research is an over-representation of
low-income groups among trial subjects.
(E) While CROs have argued that more rules will stifle the industry, the truth
is that ethical science is often better science.
(F) Sometimes CROs recruit them selectively, exploiting financial need and
medical ignorance, at other times people over-volunteer for the money.
(a) ABDFE (b) BDEAF
(c) DFAEB (d) BEDFA
(e) DFABE
3. If sentence (A) “The fresh round of economic sanctions imposed unanimously
by the UN Security Council on North Korea is a predictable response to
mounting international frustration over the nuclear stand-off. ” is the first
sentence, what is the order of other sentences after rearrangement?
(A) The fresh round of economic sanctions imposed unanimously by the UN
Security Council on North Korea is a predictable response to mounting
international frustration over the nuclear stand-off.
(B) Despite the crippling nature of the curbs, there is some good news on this
imbroglio.
(C) The measures come days after the U.S., echoing suspicions in other
countries, charged the North Korean government with the world-wide
‘WannaCry’ cyber attacks in May.
(D) As on previous occasions, Beijing and Moscow were able to impress
upon the Security Council the potentially destabilising and hence
counterproductive impact of extreme measures.
(E) The sanctions include an 89% curb on refined petroleum imports into
North Korea, stringent inspections of ships transferring fuel to the
country, and the expulsion of thousands of North Koreans in other
countries within two years.
(F) This is significant given the intercontinental ballistic missile that
Pyongyang launched in November.
(a) CEBDF (b) BDCEF
(c) CFEBD (d) BEDFC
(e) DFCBE
rearranging words or sentences 119

ANSWERS & EXPLANATION

1. (b) The first sentence talks about the fact that only few investors have idea
about bitcoins and other cryptocurrencies (which seems an attractive
investment area), so, the finance ministry has warned the potential
investors about it. Sentence E will follow the first sentence because it
says that ‘bitcoin not only shot up well over by 1000%......’ which justifies
‘attractive investment area’ and forms a link. Now, we are left with only
option (b) and (d) to choose from. When we consider the sentence F,
we can see that this line seems to be a part somewhere in the middle
of the paragraph, also, the first line starts with a warning, therefore, it
must justify the consequences of the investment in bitcoins and other
cryptocurrencies which is justified by sentence C. Hence, option (b) is
the correct choice.
2. (d) After reading all the sentences carefully, we see that sentence A and F
should go one after another as both talk about ‘over-volunteer’. Moreover,
sentence A will follow sentence F because of the presence of the word
‘such’ which signifies that the subject of the sentence has already been
discussed in the previous sentence. So, we have option (c), (d) and (e) to
choose from. Considering sentence D which talks about ‘a big problem’,
we find that it can’t be the second sentence as no problem of any sort
has been dealt in the first sentence, so, option (c) and (e) gets eliminated.
Hence, by elimination method, we can conclude that option (d) is the
correct choice.
3. (a) After reading all the sentences carefully, we see that sentence B and E
should go one after another as both talk about ‘curb’. Moreover, sentence
B will follow sentence E because of the presence of ‘the crippling nature
of the curb’ which signifies that the curb has already been discussed in
the previous line. So, we have only option (a) and (c) to choose from.
Sentence E will follow sentence C because the first line deals with
‘economic sanction’ making ACEB form a link. Hence, option (a) is the
correct choice.
EBD_7275
CLOZE TEST

Chapter
FILLING IN PARAGRAPH GAPS
19 OR NUMBERED GAPS

A cloze test passage is a reading comprehension with some words removed. The
candidate has to find the correct words from the options to fill the blanks to make
the sentence grammatically as well as contextually correct. A cloze test passage
has a definite structure, logical pattern and chronological order which help in
maintaining a unified tone throughout.
Example 1:
_____ (1) _____ stringent anti-pollution laws, mass awareness levels in India
about the need to _____ (2) _____ the environment are low. Which is ______
(3) ____ many people insist that mere laws won’t do: what we ____ (4) ____ need
are “environment conscious” citizens.
1. (a) Despite (b) Having
(c) Enacting (d) Adopting
(e) Although
2. (a) contaminate (b) clear
(c) filter (d) protect
(e) pollute
3. (a) resulting (b) why
(c) obvious (d) as
(e) because
4. (a) actually (b) don’t
(c) hardly (d) perfectly
(e) seldom

ANSWERS & EXPLANATION

1. (a) The passage should start with a negative conjunction as the succeeding
phrases states that mass awareness levels are low even though we have
strict laws. Hence, option (a) is correct.
filling in paragraph gaps or numbered gaps 121
2. (d) Option (e) and (a) are eliminated because after reading the sentence, it
is understood that there are stringent pollution laws and even though we
have laws, we couldn’t secure the environment. Option (d) is correct as
the sentence reads as ‘need to protect the environment are low’.
3. (b) Option (b) is the correct choice. Option (a), (c) and (d) get eliminated
because they are irrelevant in the context of the sentence. Between ‘why’
and ‘because’, ‘because’ can be eliminated as it is used as a negative
conjunction. Hence, option (b) is the correct choice.
4. (a) Option (b) and (c) get eliminated as they give negative connotation when
substituted in the given context. Among options (a), (d) and (e), option
(a) with the word ‘actually fits right into the sentence and is the best
suited option. Hence, option (a) is the correct choice.
Example 2:
Reliance Infrastructure today reported 6 per cent _1_ in consolidated net profit at
` 382 crore for the July September quarter _2_ a dip in operating income. Total
operating income declined to ` 5,515 crore in the quarter _3_. ` 5,729 crore in
the same quarter _4_ fiscal, Anil Ambani led Reliance Group Company _5_ in a
statement.
1. (a) extra (b) chance
(c) rise (d) develop
(e) waste
2. (a) admiration (b) despite
(c) slight (d) indeed
(e) sight
3. (a) against (b) consistent
(c) favour (d) similar
(e) different
4. (a) previous (b) future
(c) following (d) further
(e) current
5. (a) written (b) secret
(c) said (d) thought
(e) told

ANSWER KEY

1. (c)
2. (b)
3. (a)
4. (a)
5. (c)
EBD_7275
122 filling in paragraph gaps or numbered gaps

NEW PATTERN BASED EXAMPLE


Example 3:
The Goods and Services Tax (GST) is being __1__ (framed) as the single-biggest
economic reform since the economic liberalisation of 1991. Even critics of the tax,
who complain about its complex four-slab rate structure, agree that it is a step in
the right direction. The primary reason is that it __2__ (does away) with the present
system of multiple Central and State taxes, replacing it with a much simpler tax
system. Another supposed benefit of GST is that it is a tax on consumption, which
replaces the current web of __3__ (running) taxes in the production chain that
__4__ (lowers) prices and distorts production. In the process, it is said, the new tax
system does away with the barriers to free trade __5__ (into) and between States,
effectively turning India into a single free market for goods and services.
For __6__ (psychoanalysts), there is good reason to doubt all these claimed
benefits of the GST. One, a nationwide tax such as the GST will __7__ (form) to
a higher tax burden as it reduces tax competition. Earlier, States which were keen
to __8__ (put in) investment and labour from each other had a reason to cut taxes.
Now, the Centre, which will __9__ (increase) no tax competion except from the
rest of the world, can __10__ (terminate) rates at whim. This will encourage tax
rate increases that are detrimental to growth. Two, the number of taxes does not
necessarily reflect the actual burden.
1. (a) turned (b) flaunted
(c) biggest (d) oldest
(e) No change required
2. (a) do over (b) does over
(c) done over (d) done up
(e) No change required
3. (a) giving (b) devastating
(c) cascading (d) lower
(e) No change required
4. (a) controls (b) remarket
(c) decrease (d) increase
(e) No change required
5. (a) in (b) on
(c) within (d) onto
(e) No change required
filling in paragraph gaps or numbered gaps 123
6. (a) suptics (b) astrologists
(c) people (d) predicament
(e) No change required
7. (a) promote (b) diminish
(c) administer (d) lead
(e) No change required
8. (a) attract (b) commit
(c) advice (d) adore
(e) No change required
9. (a) arrange (b) conflict
(c) fare (d) advocate
(e) No change required
10. (a) determine (b) admire
(c) desire (d) cultivate
(e) No change required

ANSWER KEY

1. (b)
2. (e)
3. (c)
4. (d)
5. (c)
6. (a)
7. (d)
8. (a)
9. (c)
10. (a)

IMPORTANT TIPS AND TECHNIQUES


¾¾ Read the entire passage slowly and thoroughly without filling up the
blanks. This will help you to understand the idea or the theme of the
passage which will help you later on while choosing the options to fill
the blanks.
¾¾ It is very important to understand the tone of the passage as this will
help you eliminate the irrelevant options.
EBD_7275
124 filling in paragraph gaps or numbered gaps

¾¾ Emphasize on linking the sentences together because in the passage,


all the sentences are connected to each other. Do not make a mistake
of treating each sentence like an individual or an independent sentence.
Try to come up with logical connections that link up the sentences be-
cause this will be very helpful in picking up the correct option.
¾¾ You will often come across a blank that has more than one correct op-
tion. List out all these options and try them one by one. Use the one that
seems most fitting. Instead of getting confused, think of words that are
appropriate not only to the given sentence but also fit the context of the
entire passage.
¾¾ Sometimes, you may not be able to decide between two words. In this
case, if you see a word in the options that is frequently used with the
words around the blank, then pick that option.
¾¾ The knowledge of how prepositions are used will surely come handy.
There are times when looking at preposition alone can help you pick the
correct option.
¾¾ It is always advised to look at the sentences that come before and after
the sentence that has blank in it. By doing so, quite often you will get a
confirmation or some sort of clue regarding the most appropriate word
to fill.
¾¾ Read articles, especially in newspapers and magazines to improve your
language. When you read more, you develop an idea of using various
words in different contexts. You also get to read lot of idioms and phras-
es that prove to be very helpful while picking up the correct choice.
SPOTTING ERRORS

Chapter
SPOTTING ERRORS
20
Despite possessing a good command of the English language and considering
ourselves well-versed in it, we many times end up making the silliest of errors
in grammar. Every English exam/test contains questions on ‘spotting errors’
to test the grammatical knowledge of the candidate. Thus, in order to enhance
your grammatical knowledge and make you aware of common errors that you
usually commit while speaking and writing, we are giving very important tips and
techniques to spot errors.
¾¾ Some nouns are singular in form, but they are used as plural nouns and always
take a plural verb. Examples of such nouns are – Police, People, Company,
Cattle and Peasantry etc.
Sentence examples:
Police has reached the crime spot. (INCORRECT)
Police have reached the crime spot, (CORRECT)

The cattle is grazing in the ground. (INCORRECT)


The cattle are grazing in the ground. (CORRECT)
¾¾ Some nouns are always used in plural form and always take a plural verb.
Examples of such nouns are – Spectacles, Scissors, Trousers, Premises and
Alms etc.
Sentence examples:
Where is my trousers? (INCORRECT)
Where are my trousers? (CORRECT)

The scissors is on the rack. (INCORRECT)


The scissors are on the rack. (CORRECT)
¾¾ There are nouns that indicate length, measure, money, weight or number and
when they are preceded by a numeral, they remain unchanged in form so long
as they are followed by another noun or pronoun. Examples of such nouns
are – Year, Pair Foot Meter and Million etc.
EBD_7275
126 Spotting Errors
Sentence examples:
This is a ten-meters cloth. (INCORRECT)
This is a ten-meter cloth. (CORRECT)

He has completed a three-years degree course. (INCORRECT)


He has completed a three-year degree course. (CORRECT)
¾¾ When a number is followed by a noun denoting measure, length, money,
weight or number, but these are not followed by another noun or pronoun
then they take the plural form.
Sentence examples:
This mat is five yard long. (INCORRECT)
This mat is five yards long. (CORRECT)

The weight of the machine was eleven kilogram. (INCORRECT)


The weight of the machine was eleven kilograms. (CORRECT)
¾¾ Collective nouns such as public, team, jury, committee, audience and com-
pany etc. are used both as singular as well as plural depending on the mean-
ing. When these words indicate a unit, the verb is singular; otherwise the verb
will be plural.
Sentence examples:
The jury was divided in this case. (INCORRECT)
The jury were divided in this case. (CORRECT)
¾¾ A pronoun must agree with its antecedent in person, number and gender.
Examples:
Every student must bring his identity-card.
All employees must do their work in the given time.
Each of the girls should carry her water-bottle.
¾¾ The pronoun ‘one’ must be followed by ‘one’s’.
Sentence example:
One must complete his task in time. (INCORRECT)
One must complete one’s task in time. (CORRECT)

One should respect his elders. (INCORRECT)


One should respect one’s elders. (CORRECT)
Spotting Errors 127
¾¾ ‘One of’ always takes a plural noun after it.
Sentence examples:
This is one of the best moment of my life. (INCORRECT)
This is one of the best moments of my life. (CORRECT)

One of my friend is an engineer. (INCORRECT)


One of my friends is an engineer. (CORRECT)
¾¾ Question tags are always the opposite of the sentence which means if the sen-
tence is positive, the question tag will be negative and vice-versa.
Sentence examples:
You were quarreling, were you? (INCORRECT)
You were quarreling, weren’t you? (CORRECT)

She did this, did she? (INCORRECT)


She did this, didn’t she? (CORRECT)

EXAMPLES ON NEW PATTERN


The following questions consist of a sentence which is divided into three parts
which contain grammatical errors in one or more than one part of the sentence. If
there is an error in any part of the sentence, find the correct alternatives to replace
those parts from the three options given below each question to make the sentence
grammatically correct. If there is an error in any part of the sentence and none of
the alternatives is correct to replace that part, then choose (d) i.e. None of the (I),
(II) and (III) as your answer. If the given sentence is grammatically correct or does
not require any correction, choose (e) i.e. No correction required as your answer.
1. Nearly four years after the Supreme Court recognizes the rights of
transgender person,(I)/ and a few months after the approval of a policy
by the State Cabinet, (II)/ the first marriage of a transgender person was
registered yesterday. (III)
(I) Nearly four years after the Supreme Court recognized the rights of
transgender person,
(II) and a few month after the approval of a policy by the State Cabinet,
(III) the first marriage of a transgender person were registered yesterday.
(a) Only (I) (b) Only (II)
(c) Both (I) and (II) (d) None of the (I), (II) and (III)
(e) No correction required
EBD_7275
128 Spotting Errors
2. The government on Wednesday announced the detailed contours of the
recapitalising plan for public sector banks (I)/ it announced in October 2017,
including that a reforms package across six themes that cover 30 action points
(II)/ such as customer responsiveness, responsible banking, and credit off
take. (III)
(I) The government on Wednesday announced the detailed contours of the
recapitalisation plan for public sector banks
(II) It had announced in October 2017, including that a reform package
across six themes that cover 30 action points
(III) such as customer responsiveness, responsible banking, and credit off
take.
(a) Only (I) (b) Only (II)
(c) Both (I) and (II) (d) None of the (I), (II) and (III)
(e) No correction required
3. The centre has decided to borrow an additional sum of money in the last three
months (I)/ of this financial year, a move that some economists said could
result in (II)/ the government missing its budgeted fiscal deficit target. (III)
(I) The centre has decided to borrow additional sum of money in the last
three months
(II) of this financial year, a move that some economists said may result in
(III) government missing its budget fiscal deficit target.
(a) Only (II) (b) Only (III)
(c) Both (II) and (III) (d) None of the (I), (II) and (III)
(e) No correction required
4. Given the recent media reports about fresh transgressions by the Chinese
troops on the Line of Actual Control (LAC) since the Doklam crisis, (I)/ it is
now safe to say that the high-octane national security rhetoric by the Indian
government (II)/ has done little to strengthen India’s military stance at the
disputed border. (III)
(I) Given the recent media reports about fresh transgressions by the Chinese
troops on the Line of Actual Control (LAC) since the Doklam crisis,
(II) it is now safe to say that the high-octane national security rhetoric by the
Indian government
(III) has done little to strengthen India’s military stance on the disputed border.
(a) Only (I) (b) Only (II)
(c) Only (III) (d) None of the (I), (II) and (III)
(e) No correction required
Spotting Errors 129
5. The company will set up ten laboratories in Northern India to test, verify and
calibrate (I)/ the work and reference standards of different types of balances,
(II)/ weights and measuring equipments used in shops or establishments. (III)
(I) The company will set up ten laboratories in Northern India to test, verify
and calibrate
(II) the working and reference standards of different types of balances,
(III) weights and measuring equipments used in shops or establishments.
(a) Only (I) (b) Only (II)
(c) Both (I) and (III) (d) None of the (I), (II) and (III)
(e) No correction required
6. To the translation and interpretation of the Scriptures men might bring a fal-
lible judgment, (I)/ but this would be assisted by (II)/ the direct action of the
Spirit of God in proportion to their faith. (III)
(I) To the translation and interpretation of the Scriptures men might bring fal-
lible judgments,
(II) but this will be assisted by
(III) the direct action of the Spirit of God on proportion to their faith.
(a) Only (I) (b) Only (II)
(c) Both (I) and (III) (d) None of the (I), (II) and (III)
(e) No correction required
7. It is a true fact that in the next 5-6 years, (I)/ by when the high-speed trains
would start running, (II)/ the project will more than prove itself even to its
wildest of detractors. (III)
(I) It is a fact that in the next 5-6 years,
(II) by then the high-speed trains would start running,
(III) the project will more than prove itself even to its wildest of detractor.
(a) Only (I) (b) Only (II)
(c) Both (I) and (III) (d) None of the (I), (II) and (III)
(e) No correction required
8. Raman was a good teacher but despite this fact, (I)/ he had no influence (II)/
on his pupils. (III)
(I) Despite of Raman’s being a good teacher,
(II) he didn’t had any influence
(III) over his pupils
(a) Only (I) (b) Only (II)
(c) Only (III) (d) None of the (I), (II) and (III)
(e) No correction required
EBD_7275
130 Spotting Errors
9. Though the place was very scary, (I)/ he insisted to go there but we were for-
tunate (II)/ that we reached our home safely. (III)
(I) In spite the place was very scary,
(II) he insisted to going there but we were fortunate
(III) that we reached our home safe.
(a) Only (I) (b) Only (II)
(c) Both (II) and (III) (d) None of the (I), (II) and (III)
(e) No correction required
10. Since he was a translator, it was duck soup (I)/ for him to translate 15 pages
from (II)/ Hindi to English in a single day. (III)
(I) Him being a translator, it was duck soup
(II) to translate 15 page from
(III) Hindi into English in a single day.
(a) Only (I) (b) Only (II)
(c) Only (III) (d) None of the (I), (II) and (III)
(e) No correction required

ANSWERS & EXPLANATION

1. (a) Part (I) of the question is incorrect due to the use of the word ‘recognizes’,
which should be replaced with the past tense word ‘recognized’ as it goes
with the context of the sentence and makes the sentence grammatically
correct.
2. (c) In part (I), ‘recapitalising’ should be replaced with ‘recapitalisation’ as
noun should be used and not verb in the context of the sentence. In part
(II), ‘had’ should be used before ‘announced’ because when we describe
an action with reference to a timeframe, or an event of the past, we use
past perfect tense. Part (III) is grammatically correct hence, it doesn’t
require any correction.
3. (e)
4. (c) Part (III) of the question is incorrect. ‘on’ preposition should be used
instead of ‘at’ in the context of the sentence.
5. (b) Part (II) of the question is incorrect. ‘work’ which is a noun is incorrect
in the context of the sentence and therefore, it should be replaced with
‘working’ which is an adjective to make the sentence grammatically cor-
rect.
Spotting Errors 131
6. (e)
7. (a) ‘True’ and ‘fact’ should not be used together as they make the sentence
superfluous hence, part (I) in the question should be replaced by option
(I).
8. (c) ‘On’ preposition should be replaced with ‘over’ because ‘on’ is incorrect
in the context of the sentence whereas, ‘over’ makes the sentence gram-
matically correct.
9. (c) ‘Go’ which is a verb in part (II) of the sentence should be replaced with
‘going’ which is a noun and ‘safely’ which is an adverb in part (III) of the
sentence should be replaced with ‘safe’ which is an adjective to make the
sentence grammatically correct.
10. (c) ‘To’ preposition in part (III) of the sentence is incorrect so, it should be
replaced by the preposition ‘into’ to make the sentence grammatically
correct.
EBD_7275
SENTENCE COMPLETION

Chapter
SENTENCE
21 COMPLETION
Sentence completion is a type of test in which a word or two are removed and
you are to select the most appropriate word(s) from the given options so that the
sentence remains grammatically and contextually correct. This section tests your
vocabulary skills and reading practice. Your abilities to understand the main idea
of the sentence and the logical structure of the sentence are also tested.

Examples:

1. Because of his __________, David’s father-in-law felt very welcome and


comfortable staying at his house for the weekend.
(a) animosity (b) hospitality
(c) determination (d) wittiness
(e) severity
2. Although the warring partners had settled a lot of disputes, past experience
made them ________ to express optimism that the talks would be a success.
(a) scornful (b) reticent
(c) ambivalent (d) rash
(e) hateful
3. It became abundantly clear that there was nothing more that could be done
to save the _______ business, as years of irreversible and poor decisions had
been contributing to its slow decay.
(a) myopic (b) monumental
(c) mutable (d) moribund
(e) motley
4. Because the Parliament has the votes to override a presidential veto, the Presi-
dent has no choice but to _______.
(a) object (b) repel
(c) capitulate (d) abstain
(e) compromise
5. Because the man had told so many _______ tales about seeing tigers, none of
his friends believed him when he actually did see a tiger.
(a) fallacious (b) verifiable
(c) scrupulous (d) fictitious
(e) concrete
Sentence Completion 133
6. Although some think the term ‘bug’ and ‘insect’ are _______, the former term
actually refers to _______ group of insects.
(a) parallel, an identical
(b) precise, an exact
(c) interchangeable, a particular
(d) exclusive, a separate
(e) similar, concise
7. His __________ sense of humour caused more __________ than he must
have intended.
(a) debunk, sobriety
(b) wry, confusion
(c) prominent, impudence
(d) incorrigible, paucity
(e) wise, deter
8. Psychology has slowly evolved into a/an __________ scientific discipline
that now functions ________ with the same privileges and responsibility as
other sciences.
(a) independent, autonomously
(b) unusual, alone
(c) uncontrolled, dominantly
(d) inactive, homogeneously
(e) sanction solely

ANSWERS & EXPLANATION

1. (b) The sentence has positive connotation - David’s father-in-law felt


welcome and comfortable. In addition, the transition ‘because’ indicates
that something that belongs to David has caused his father-in-law to feel
welcome and comfortable. ‘Animosity’ and ‘severity’ have a negative
connotation and ‘determination’ has a neutral connotation. ‘Hospitality’
and ‘wittiness’ both have positive connotations, but ‘hospitality’ best fits
the context of the sentence. Hence, option (b) is the correct choice.
2. (b) The word ‘although’ sets up a contrast between what has occurred (suc-
cess on some issues) and what can be expected to occur (success for
the whole talks). Hence, the partners are reluctant to express optimism.
‘Reticent’ is synonymous to ‘reluctant’ hence, option (b) is the correct
choice. All other options are either in contrast or irrelevant to the context
of the sentence.
3. (d) To figure out what the missing word is, try to predict its definition by us-
ing keywords from the prompt. Here, the keywords are found in the noun
clause “slow decay.” The prompt hinges on a cause-and-effect relation-
ship with the first clause, including the missing word, being the effect
of the “irreversible and poor decisions” of the second clause. Thus, the
EBD_7275
134 Sentence Completion
missing word must relate to the effect of a “slow decay.” A slow decay
would lead to death, so the missing word must mean dying or near death.
Because moribund means dying, hence, choice (d) is correct. Other op-
tions do not fit here because they are irrelevant to the theme of the sen-
tence hence get eliminated.
4. (e) Since the Parliament has the votes to override a presidential veto, the
President is left with no other choice than to accept what the Parliament
says or compromise with it in order to make the best of situation. Hence,
option (e) is the correct choice. All other options are either vague or out
of context of the sentence.
5. (d) The missing word must mean untrue or false, since then it would be a
surprise to find him telling a story about “actually” seeing a tiger. Since
fictitious means made up or unreal, choice (d) is correct. Option (a) is
incorrect because fallacious means deceptive or misleading. While this
could work in context for the missing word, the prompt (actually) only
implies that the man’s stories were imagined, not that they were inten-
tionally deceptive. There is, thus, not enough information given in the
prompt to support this choice. (b) is incorrect because verifiable means
able to be confirmed. As such, this does not work in context. This word
would imply that the man’s usual tales were rooted in reality. (c) is incor-
rect because scrupulous means principled. This word would not describe
either the man or his tales, as his tales are apparently not rooted in reality.
(e) is incorrect because concrete means real, the exact opposite of the
boy’s usual tales. If the man’s tales were typically authentic, his friends
would be more likely to believe them.
6. (c) The word ‘although’ indicates that the two parts of the sentence are in
contrast with each other. Although most people think about the terms
‘bug’ and ‘insect’ one way, something else is actually true about the
terms. Option (c) logically completes the sentence, indicating that while
most people think the terms are ‘interchangeable,’ the term ‘bug’ actually
refers to ‘a particular’ group of insects.
7. (b) The clue word in this sentence is “caused”. ‘Wry’ means dry or twisted
humour and confusion would certainly be a potential unintended effect
from this speaking style. Hopefully, you are able to use the process of
elimination on many of these answer choices as the dual words double
the chances you will know the meaning of at least one of the words and
be able to eliminate the answer choice if it does not fit. The correct an-
swer is the option (b).
8. (a) The sentence contains attributive interpretation of the scientific dis-
cipline of the equivalent scientific discipline of pre-modifier modified
content, so space should be filled with ‘independent’ as autonomous is
independent, therefore, the answer is option (a) whereas, all other options
are incorrect.
Sentence Completion 135
EXAMPLES OF THE LATEST PATTERN OF QUESTIONS
DIRECTION (1-5): In question given below there are two statements, each
statement consists of two blanks. You have to choose the option which provides the
correct set of words that fits both the blanks in both the statements appropriately
and in the same order making them meaningful and grammatically correct.

1. (A) Consumers in India are already beginning to feel the ______ as petrol
and diesel prices have ______ multi-year highs.
(B) It was her hard ______ that saved his life as he was about to get _____ by
the fast moving truck.
(a) tweak, grown (b) pinch, hit
(c) squeeze, knocked (d) burden, buffeted
(e) cramp, bumped
2. (A) The ______ of actors at the ______ concert was nothing new.
(B) At the _______, the thieves made their ______ plan to execute in the
very next day.
(a) get-together, plummet
(b) meeting, grisly
(c) tryst, appalling
(d) amour, grotesque
(e) rendezvous, nocturnal
3. (A) His tough ______ with the dealer made sure that he was very concerned
about the proper ______ of his family in the new city.
(B) In the current state of Brexit _____, a spirit of reasonable _____ could
well define the future.
(a) dealings, settlement
(b) arguments, compliance
(c) negotiations, accommodation
(d) talks, conniption
(e) dispute, leniency
4. (A) The ______, which are generally conducted between January and March,
had a late start this year, and were less _____ than were seen in recent
years.
(B) The military ______ administered by the Indian army showed ______
use of indigenously built fighter jets.
(a) manoeuvre, extensive
(b) tactic, stunning
(c) exercise, capacious
(d) activity, amorphous
(e) cessation, huge
EBD_7275
136 Sentence Completion
5. (A) His love for cars never ______ because even in his old age, he used to
______ short his spending to buy the latest one in the market.
(B) Indian gold demand ______ as jewellers expected import tax ______ in
budget.
(a) diminished, slit
(b) dwindled, rip
(c) waned, cut
(d) drooped, chip
(e) decreased, praise

ANSWER KEY

11. (b)
12. (e)
13. (c)
14. (a)
15. (c)

IMPORTANT TIPS AND TECHNIQUES TO SOLVE


SENTENCE COMPLETION
¾¾ It is advised not to look at the options straightaway. At first, read the sentence
and think of a word that fits in the blank appropriately. This strategy will
help you find the missing word easily because when you read the sentence,
you catch the theme, tone and context of the sentence.
¾¾ When you look at the options, make sure you find the option that best
replaces the word that you had thought of initially. Ensure that the meaning
of the sentence is intact. Once you have placed the likely option, do check
that the sentence gives out a plausible meaning.
¾¾ Concentrate on eliminating the options rather than finding the correct
one. In sentence completion test, your knowledge and understanding of
vocabulary and common idioms-phrases of English language is tested. You
must consider all of the choices before you confirm your answer, even if
your predicted answer is among the choices. The difference between the
best answer and the second best answer is sometimes very subtle. When you
think that you have the correct answer, read the entire sentence to yourself,
using your best choice.
¾¾ Keeping a close eye on grammar rules can sometimes help you find the
correct word easily. For instance, if the article ‘an’ comes before the blank
then you can immediately go for the option that begins with a vowel or a
word the pronunciation of which sounds like a vowel.
Sentence Completion 137
¾¾ You should look for indicators in the sentence; if any. Indicators tell you what
is coming up. They indicate that the part of the sentence is either drawing
a contrast with something stated previously or supporting something stated
previously. Examples of some contrast indicators are – But, Despite,
Although, However, Nevertheless and Yet etc. Examples of some support
indicators are – And, For, Likewise, Also, Furthermore, Moreover and In
addition, etc. Examples of cause and effect indicators are – Thus, Therefore,
Hence, Because, If...then, etc.
¾¾ Lastly, read English newspapers and magazines regularly as this will
enhance your grammatical understanding and you will also come to see
how various words are used in different contexts. Reading habits, at times,
help you get the correct word just by having a quick glance at the options.
EBD_7275
PASSAGE COMPLETION

Chapter
PASSAGE
22 COMPLETION
Passage completion is a test in which a paragraph is given with an omitted line and
you have to find this omitted line from the given options.
For example:
1. Women’s boxing is yet to be recognized as an Olympic support, ________. If
that happens, the dream of most of the tough girls may come true.
(A) The International Boxing Association has been campaigning to include it
as an event in the 2008 Beijing Olympics.
(B) Though boxing is a very tough sport many women are seen willing to
take up professional boxing now-a-days.
(C) Even some state governments are now willing to employ the women pu-
gilists.
(a) Only A (b) Only B
(c) Both A and C (d) Both B and C
(e) All A, B and C
2. Although the share of agriculture in the overall GDP has declined from around
40 per cent in 1980-81 to below 20 per cent in 2006-07, its importance to the
Indian economy can hardly be over-emphasized. __________ . In the context
of ensuring food security and promoting inclusive growth, strategies to revi-
talize agriculture has become highly relevant.
(A) Fiscal deficits as a proportion of the GDP have come down but are still
high by global standards.
(B) Infrastructure deficiencies can hold back further grown in the agriculture
sector.
(C) Recent Volatility in agricultural production has had its impact not only on
economic but on price stability as well.
(a) Only A (b) Only B
(c) Only C (d) Both B and C
(e) None of these
3. The audiences for crosswords and Sudoku, understandably, overlap greatly,
but there are differences, too. A crossword attracts a more literary person,
while Sudoku appeals to a keenly logical mind. Some crossword enthusiasts
turn up their noses at Sudoku because they feel it lacks depth. A good cross-
word requires vocabulary, knowledge, mental flexibility and sometimes even
a sense of humour to complete. It touches numerous areas of life and provides
an ‘Aha!’ or two along the way. __________
Passage Completion 139
(a) Sudoku, on the other hand, is just a logical exercise, each one similar to
the last.
(b) Sudoku, incidentally, is growing faster in popularity than crosswords,
even among the literati.
(c) Sudoku, on the other hand, can be attempted and enjoyed even by chil-
dren.
(d) Sudoku, however, is not exciting in any sense of the term.
(e) Sudoku, whereas, gives better enthusiasm.
4. Mental illness is often stigmatized. __________ It is not as obviously fatal as
many physical illnesses. But it still takes a heavy human and economic toll.
That is why, it is important that politicians make good on their promises and
that ordinary people dig deep, too.
(a) Though the brain is extraordinarily complex, further scientific break-
throughs can be expected.
(b) Post-traumatic stress disorder was only defined in 1980; understanding
of that condition has jumped forward in the past few years, as have the
treatments for it.
(c) Past investigations into early interventions in psychosis have since repaid
themselves many times over.
(d) Many illnesses afflict the old disproportionately, but mental illness tends
to strike the young, undermining productivity.
(e) It lacks an effective lobby to match the groups that represent victims of
cancer and heart disease.

ANSWERS & EXPLANATION

1. (a) The first sentence says that women’s boxing has not been recognized as
an Olympic sport. Sentence A follows with the idea that the Indian Box-
ing Association is campaigning towards it being included in the 2008
Olympics. Taking up boxing as a profession or getting a government job
as mentioned in sentences B and C cannot be a great ‘dream’ to be ful-
filled which is the idea stated in the last sentence.
2. (c) The opening sentence highlights the importance of agriculture in the In-
dian economy. Sentence C follows emphasizing the fact that volatility in
agriculture production can have various undesirable impacts. Hence, we
have to revitalize agricultural field and this is the idea given in the last
sentence. The other two sentences talk about ‘fiscal deficits’ and infra-
structure deficiencies in the agricultural sector which are irrelevant in the
given context.
3. (a) The paragraph, in the beginning, highlights the main points of difference
between Sudoku and a crossword. Therefore, we are looking for a con-
trast, which is not present in options (b) and (d). Option (c) is too specific,
which is why we pick Option (a).
EBD_7275
140 Passage Completion
4. (e) In the next statement after the blank, mental illness is compared with
physical illness and is said that politicians and people should take this
matter seriously. Now in option (e) also, mental illness is compared with
cancer and heart disease that are physical illness and it is said that it lacks
effective lobby to match.

TIPS AND TECHNIQUES TO SOLVE PASSAGE COMPLETION


¾¾ It is very important to read the passage without missing any line to un-
derstand the inherent message/view/opinion the author wants to convey.
This also helps you out in drawing the topic or the central idea of the
passage which is vital while choosing the correct option.
¾¾ After reading the passage carefully, have a glance on the options and try
to find the correct sentence by matching the subject-matter/topic of the
passage with that of the given options.
¾¾ The options which appear to be irrelevant or out of context should be
ignored instantly.
¾¾ After eliminating the mismatch options, you will be left with two or
may be three options to choose from. Now, you need to judge which
of the left options matches with the tone of the author and correctly fits
the blank.
¾¾ Try to find the selected options one by one at the missing space and
judge it on the basis of tone, context, logic, sequence and symmetry. The
option which qualifies these criteria will be the right one and should be
chosen.
¾¾ After finding the correct option, read the passage for the last time to
finally conclude that after filling the blank with your selected option, the
passage reflects a complete picture/ meaning.
Chapter

23 REVISION EXERCISES

CLOZE TEST
DIRECTIONS (Qs. 1-10) : In the passage given below, there are 10 blanks,
each followed by a word given in bold. Every blank has four alternative words
given in options (a),(b),(c) and (d). You have to tell which word will best suit
the respective blank. Mark (e) as your answer if the word given in bold after
the blank is your answer i.e “No change required”.
Everyone knows that the baby-boomer generation is in the process of retiring,
and that all those ex-hippies and _1_(impish) can expect to live longer than the
Americans who retired before them. But the financial challenge this poses is less
well understood. Any lingering _2_(anxiety) ought to be exploded by two papers
in the latest Journal of Retirement.
The first*, from the Centre for Retirement Research (CRR) at Boston College,
estimates the proportion of 65-year-olds who will be able to retire without a big
hit to their _3_(boorish) income. Pensioners do not usually need as much money
coming in as workers: for a start, they no longer need to save for retirement. The
CRR estimates that 65-85% of their previous income is a reasonable “replacement
rate”, depending on the type of household.
As well as private pensions, elderly Americans receive income from the federal
government (in the form of Social Security, the public pension) and many earn
money from their accumulated wealth, particularly by taking _4 (equity) out of
their houses. Even allowing for these sources of income, the CRR estimates that
52% of Americans may not be able to maintain their standard of living (which it
defines as having an income that falls no more than 10% below the replacement
rate).
Unsurprisingly, the biggest problems face those with no private pension at all: 68%
of these Americans are expected to fall short. Those lucky enough to be covered by
defined-benefit plans—in which pensions are linked to a worker’s salary—have
the least difficulty: only 20% are _5_(risk) at risk. Of those in defined-contribution
(DC) plans—in which workers receive whatever pension _6_(deduced) they have
accumulated by retirement—53% probably will not reach the replacement rate.
At least, you might think, Social Security will provide a basic income for the
elderly. But the second paper**, by Sylvester Schieber, a former chairman of
the Social Security Advisory Board, _7_(dramatized) that there are holes in the
safety net. Mr Schieber finds that people whose total careers _8_(fronted) just 10-
EBD_7275
142 Revision Exercises
19 years comprise 8% of pensioners, but just over half of the poorest tenth of the
elderly. Such workers could hardly have saved more for their retirement; they had
barely _9_(intolerable) income in the first place.
That could be a _10_ (languid) problem, given Social Security’s finances. Payroll
taxes on current workers no longer exceed the benefits paid out, prompting the
government to tap the surplus of past years. On current projections, this will run out
in 2034. After that, the cost of pensions could still be met mainly by contributions
from workers. But the politicians of the day may be forced to consider benefit cuts.
1. (a) brats (b) punks
(c) bleats (d) bonnets
(e) No Change Required
2. (a) modest (b) burden
(c) altruism (d) complacency
(e) No Change Required
3. (a) turbid (b) essential
(c) disposable (d) enhance
(e) No Change Required
4. (a) partial (b) empathy
(c) liquidity (d) dispense
(e) No Change Required
5. (a) deemed (b) doubted
(c) regarded (d) nundated
(e) No Change Required
6. (a) box (b) pot
(c) dribble (d) dethrone
(e) No Change Required
7. (a) kindled (b) evoke
(c) draw out (d) points out
(e) No Change Required
8. (a) perished (b) arched
(c) strived (d) lasted
(e) No Change Required
9. (a) shy (b) skimpy
(c) adequate (d) waiting
(e) No Change Required
10. (a) isolating (b) looming
(c) receding (d) holding
(e) No Change Required
Revision Exercises 143
DIRECTIONS (Qs. 11-15) : In the following passage, some of the words
have been left out, each of which is indicated by a number. Find the suitable
word from the options given against each number and fill up the blanks with
appropriate words to make the paragraph meaningfully complete.
Land problems in India continue to attract equal attention from policy-makers
and academics. The renewed interest in land issues _11_ from the _12_ impact of
liberalisation and _13_  the economy. Tenancy, land ceiling and land administration
are being _14_ with a new perspective. Among the issues under renewed focus,
legalizing tenancy, revising the ceiling limits, quality of land, meeting the challenge
of miniscule holdings that are _15_ of marginalisation and land administration
are dominating the debate.
11. (a) clears (b) finds
(c) tools (d) stems
(e) fires
12. (a) commenced (b) perceived
(c) migrated (d) less
(e) pioneer
13. (a) agitate (b) materialize
(c) opening up (d) suffer
(e) slope
14. (a) revisited (b) renowned
(c) used (d) termed
(e) havocked
15. (a) start (b) level
(c) status (d) inception
(e) consequences
DIRECTIONS (Qs. 16-20) : In the following passage, some of the words
have been left out, each of which is indicated by a number. Find the suitable
word from the options given against each number and fill up the blanks with
appropriate words to make the paragraph meaningfully complete.
A good percentage of the population of India is tribal. They live in the hills
and forests of the country; and have been little…….(16)……. by the …….
(17)……. currents of the plains. Practically all the states of India have their tribal
population. The tribes are numerous, computed to be about 200, some living
in…….(18)…….regions in dense forests, and others on the borders of villages.
Some tribes are …….(19)…….to a few souls, while others like the Santhals run
into millions and are steadily…….(20)…….in numbers.
16. (a) affected (b) domiciled
(c) motivated (d) deprived
(e) favoured
EBD_7275
144 Revision Exercises
17. (a) financial (b) proud
(c) cultural (d) unruly
(e) swift
18. (a) comfortable (b) marshy
(c) wild (d) unpopulated
(e) inhospitable
19. (a) devoted (b) confined
(c) susceptible (d) related
(e) attached
20. (a) constant (b) deteriorated
(c) developing (d) increasing
(e) decreasing

DOUBLE FILLERS
DIRECTIONS (Qs. 21-35) : In question given below there are two statements,
each statement consists of two blanks. You have to choose the option which
provides the correct set of words that fits both the blanks in both the
statements appropriately and in the same order making them meaningful and
grammatically correct.
21. (1) With a firm _____, the government should be able to focus more on the
strategy to further _____ growth and demand.
(2) They can’t accept the attack on their electoral ______ which has the
chances to negatively ______ their voters.
(a) edict, deter (b) mandate, stimulate
(c) directives, appease (d) denial, prompt
(e) authority, dissuade
22. (1) It is a great misfortune that an internal ______ has moved ______ towards
a full-blow crisis.
(2) While ______ still exists between the status of men and women in the
world, the tide of history is flowing ______ in favour of women.
(a) controversy, wrongly (b) unity, adamantly
(c) cleft, impeccably (d) concordance, doggedly
(e) rift, inexorably
23. (1) He appeared to be the ______ as everyone in the party was ______ about
him.
(2) Being the ______ of the meeting, he was busy ______ about the new
project laid by his company.
(a) cynosure, talking (b) nonentity, discussing
(c) hotshot, sceptical (d) linchpin, aporetic
(e) boss, incredulous
Revision Exercises 145
24. (1) As the sun rose higher in the sky, the temperature ______ into hundreds
and Martha felt her energy begin to ______.
(2) The ______ prices of onion in the last month resulted in ______ its
consumption.
(a) increased, improve (b) drifted, lessen
(c) ascended, enhance (d) soared, dwindle
(e) towered, increase
25. (1) The soldier refused to accept a ______ for his bravery because he felt he
was ______ performing his duty.
(2) The inexperienced singer was surprised to receive a ______ for ______
singing in the chorus.
(a) compliment, ambiguously
(b) reward, effortlessly
(c) plaudit, simply
(d) bestowal, normally
(e) citation, callously
26. (1) In order to ______ himself from the beating of his father, the student
thought to ______ a way to cheat in the examination.
(2) To ______ their businesses, the black marketers have ______ with the
police in a shoddy business.
(a) protect, cogitate (b) secure, confederate
(c) save, connive (d) bulwark, plot
(e) defend, scheme
27. (1) His ______ towards his brother led him to ______ his car.
(2) Since I bear no ______ towards you, I don’t understand what provoked
you to ______ the boundary of my campus.
(a) animosity, eradicate (b) grudge, repair
(c) antipathy, devastate (d) malice, vandalize
(e) benevolence, build
28. (1) The dollar has been gradually ______ against major currencies ______
the beginning of the last year.
(2) The increasing difference between the rich and poor has resulted in
______ of people’s faith in our democracy ______ independence.
(a) depreciating, since (b) growing, from
(c) undermining, from (d) crippling, since
(e) weakening, since
29. (1) Monika used to walk ______ foot for five kilometers to look after her
father who was ______ in the city hospital.
(2) The chief minister in the party meeting stressed _____ the view shared
by one of his ministers as two days ago, he in a press conference ______
that the state government was not doing its work up to the mark.
(a) on, accepted (b) over, abstained
(c) on, admitted (d) over, put
(e) at, repudiated
EBD_7275
146 Revision Exercises
30. (1) The increasing burden of loans on the company has forced it to ______
from ______ to CSR.
(2) As rising oil prices put pressure on domestic consumers, the government
will have to ______ from ______ to subsidies.
(a) desist, resorting (b) cease, addressing
(c) relinquish, dodging (d) endure, procuring
(e) recover, providing
31. (1) A ___________ driver was arrested yesterday on a drunk driving
__________ on the Delhi-Chandigarh Highway.
(2) An erstwhile ___________ department in the ministry of commerce,
the Department of Industrial Policy and Promotion (DIPP), was put in
___________ of this transformation.
(a) inactive, headlong (b) lazy, impeach
(c) headstrong, arraign (d) sleepy, charge
(e) slumber, contact
32. (1) Economic growth, which had been ___________ down for some time
but had been given a boost by strong governance reforms was given a
demand boost to facilitate ___________ acceleration.
(2) Forced __________ is when political parties hire PR agencies which
___________ employ social media ‘influencers’ who release the same
tweets with the hashtag at a high intensity.
(a) hover, advance (b) trending, further
(c) going, reduce (d) trading, more
(e) shot, progress
33. (1) _________ remains below targets for many Central Bank; and is generally
set to stay that way in the year ahead even though the world economy in
general is set to do much _________ in 2018.
(2) India did not have _________ targeting then and so was able to deal with
the shocks _________ than many countries.
(a) consumed, more (b) inflation, better
(c) deflation, good (d) prices, honed
(e) reduced, enhanced
34. (1) U.S. consumer confidence jumped to a near 17-year high in October, with
households _________ about the labor market and business _________ ,
which could underpin consumer spending and boost the economy.
(2) The services sector was the most _________ about business prospects in
over three years with about 9 per cent of firms expecting more favourable
business _________ in the October to March period.
(a) upbeat, conditions (b) regular, placid
(c) optimism, excellent (d) downtrodden, environment
(e) vulnerable, situation
Revision Exercises 147
35. (1) We expect crude oil to _________ to USD 65 per barrel along with
commodities standing their _________ against the temporary headwinds.
(2) Should quarterly growth _________ and BJP make gains in serial state
elections it would salvage lost _________ indicating a positive shift in
perceptions.
(a) increase, floor (b) depressed, topsy
(c) rebound, ground (d) decline, space
(e) slip, grip
DIRECTIONS (Qs. 36-40) : In each of the following sentences there are
blank spaces. Below each sentence there are five pairs of words denoted by
the numbers. Find out which pair of words can be filled up in the blanks in the
sentence in the same sequence to make it grammatically correct.
36. While the technology and state of knowledge in medicine allows the gadgets
to _____ data and give recommendation, it will be a while before we are ready
to _____ doctors for serious illnesses.
(a) analyse, trust (b) evaluate, credence
(c) figure out, confidence (d) describe, distrust
(e) estimate, credit
37. Just because momentum is _____ away coal in china does not mean that the
country is no longer part of the global coal _____.
(a) altering, bang (b) changing, collapse
(c) stagnation, rise (d) shifting, boom
(e) staying, failure
38. There’s plenty of _____ that transferring kids to the adult criminal justice
system for trial and conviction has _____ to prevent repeat offences.
(a) disproof, developed   (b) indication, finished
(c) information, rise   (d) evidence, failed
(e) data, declined
39. However, ‘Digital India’ will not happen even if the _____ is in place unless
equal _____ is paid to what is made available at the end of the pipeline.
(a) framework, thinking (b) groundwork, scrutiny
(c) infrastructure, attention (d) backbone, indifference
(e) support, thought
40. The national disaster management authority should come up with national
guidelines for the _____ of such deaths for immediate _____ by the states.
(a) prevention, implementation
(b) aid, effect
(c) deterrence, operation
(d) avoidance, usage
(e) obstruction, assistance
EBD_7275
148 Revision Exercises
SPOTTING ERRORS
DIRECTIONS (Qs. 41-60) : The following question consists of a sentence
which is divided into three parts which contain grammatical errors in one
or more than one part of the sentence. If there is an error in any part of the
sentence, find the correct alternatives to replace those parts from the three
options given below each question to make the sentence grammatically correct.
If there is an error in any part of the sentence and none of the alternatives is
correct to replace that part, then choose (d) i.e. None of the (I), (II) and (III)
as your answer. If the given sentence is grammatically correct or does not
require any correction, choose (e) i.e. No correction required as your answer.
41. Concern is rising across the security establishment over the worsening
situation (I)/ along the Indo-Pakistan border, where casualties from (II)/ cross-
border firing have now climbed beyond all recent trends. (III)
(I) Concern is arising across the security establishment over the worsening
situation
(II) in the Indo-Pakistan border, where casualties from
(III) cross-border firing has now climbed beyond all recent trends
(a) Only (I) (b) Only (II)
(c) Both (I) and (III) (d) None of the (I), (II) and (III)
(e) No correction required
42. The center had issued a statement on a mutual decision for Indian and Chinese
troop (I)/ to disengage and withdraw from the part of the Doklam plateau (II)/
disputed between China and Bhutan that has been the scene of the stand-off.
(III)
(I) The center had issued a statement on a mutual decision for Indian and
Chinese troops
(II) to disengage and withdraw the part of the Doklam plateau
(III) disputed between China and Bhutan that had been the scene of the stand-
off
(a) Only (I) (b) Only (II)
(c) Both (I) and (III) (d) None of these
(e) No correction required
43. The quality of education could be enhanced (I)/ by granting flexibility to
States, making teachers accounting, and adopting (II)/ blended learning
approaches and computer-aided learning. (III)
(I) The quality of education could enhance
(II) by granting flexibility to States, making teachers accountable, and
adopting
(III) blended learning access and computer-aided learning
(a) Only (I) (b) Only (II)
(c) Both (I) and (III) (d) None of the (I), (II) and (III)
(e) No correction required
Revision Exercises 149
44. The writer believes that the work experience should focus more on the
development (I)/ in students of the ability to work with the materials of their
environment (II)/ and solve problems as close to reality as is practicable. (III)
(I) The writer believes that the work experience should focus more in the
development
(II) in students of the ability to work with the materials of his environment
(III) and solve problems too close to reality as is practicable
(a) Only (I) (b) Only (II)
(c) Both (I) and (II) (d) None of the (I), (II) and (III)
(e) No correction required
45. If you had waited for me, (I)/ I would have (II)/ gone to the party too. (III)
(I) If you have waited for me
(II) I would have had
(III) gone to the party also
(a) Only (I) (b) Only (II)
(c) Both (I) and (III) (d) None of the (I), (II) and (III)
(e) No correction required
46. Nearly four years after the Supreme Court recognizes the rights of transgender
person,(I)/ and a few months after the approval of a policy by the State Cabinet,
(II)/ the first marriage of a transgender person was registered yesterday. (III)
(I) Nearly four years after the Supreme Court recognized the rights of
transgender person,
(II) and a few month after the approval of a policy by the State Cabinet,
(III) the first marriage of a transgender person were registered yesterday.
(a) Only (I) (b) Only (II)
(c) Both (I) and (II) (d) None of the (I), (II) and (III)
(e) No correction required
47. The government on Wednesday announced the detailed contours of the
recapitalising plan for public sector banks (I)/ it announced in October 2017,
including that a reforms package across six themes that cover 30 action points
(II)/ such as customer responsiveness, responsible banking, and credit off
take. (III)
(I) The government on Wednesday announced the detailed contours of the
recapitalisation plan for public sector banks
(II) it had announced in October 2017, including that a reforms package
across six themes that cover 30 action points
(III) such as customer responsiveness, responsible banking, and credit off
take.
(a) Only (I) (b) Only (II)
(c) Both (I) and (II) (d) None of the (I), (II) and (III)
(e) No correction required
EBD_7275
150 Revision Exercises
48. The centre has decided to borrow an additional sum of money in the last three
months (I)/ of this financial year, a move that some economists said could
result in (II)/ the government missing its budgeted fiscal deficit target. (III)
(I) The centre has decided to borrow additional sum of money in the last
three months
(II) of this financial year, a move that some economists said may result in
(III) government missing its budget fiscal deficit target.
(a) Only (II) (b) Only (III)
(c) Both (II) and (III) (d) None of the (I), (II) and (III)
(e) No correction required
49. Given the recent media reports about fresh transgressions by the Chinese
troops on the Line of Actual Control (LAC) since the Doklam crisis, (I)/ it is
now safe to say that the high-octane national security rhetoric by the Indian
government (II)/ has done little to strengthen India’s military stance at the
disputed border. (III)
(I) Given the recent media reports about fresh transgressions by the Chinese
troops on the Line of Actual Control (LAC) since the Doklam crisis,
(II) it is now safe to say that the high-octane national security rhetoric by the
Indian government
(III) has done little to strengthen India’s military stance on the disputed border.
(a) Only (I) (b) Only (II)
(c) Only (III) (d) None of the (I), (II) and (III)
(e) No correction required
50. The company will set up ten laboratories in Northern India to test, verify and
calibrate (I)/ the work and reference standards of different types of balances,
(II)/ weights and measuring equipments used in shops or establishments. (III)
(I) The company will set up ten laboratories in Northern India to test, verify
and calibrate
(II) the working and reference standards of different types of balances,
(III) weights and measuring equipments used in shops or establishments.
(a) Only (I) (b) Only (II)
(c) Both (I) and (III) (d) None of the (I), (II) and (III)
(e) No correction required
51. To the translation and interpretation of the Scriptures men might bring
a fallible judgment, (I)/ but this would be assisted by (II)/ the direct action of
the Spirit of God in proportion to their faith. (III)
(I) To the translation and interpretation of the Scriptures men might
bring fallible judgments,
(II) but this will be assisted by
(III) the direct action of the Spirit of God on proportion to their faith.
(a) Only (I) (b) Only (II)
(c) Both (I) and (III) (d) None of the (I), (II) and (III)
(e) No correction required
Revision Exercises 151
52. It is a true fact that in the next 5-6 years, (I)/ by when the high-speed trains
would start running, (II)/ the project will more than prove itself even to its
wildest of detractors. (III)
(I) It is a fact that in the next 5-6 years,
(II) by then the high-speed trains would start running,
(III) the project will more than prove itself even to its wildest of detractor.
(a) Only (I) (b) Only (II)
(c) Both (I) and (III) (d) None of the (I), (II) and (III)
(e) No correction required
53. Raman was a good teacher but despite this fact, (I)/ he had no influence (II)/
on his pupils. (III)
(I) Despite of Raman’s being a good teacher,
(II) he didn’t had any influence
(III) over his pupils..
(a) Only (I) (b) Only (II)
(c) Only (III) (d) None of the (I), (II) and (III)
(e) No correction required
54. Though the place was very scary, (I)/ he insisted to go there but we were
fortunate (II)/ that we reached our home safely. (III)
(I) In spite the place was very scary,
(II) he insisted to going there but we were fortunate
(III) that we reached our home safe.
(a) Only (I) (b) Only (II)
(c) Both (II) and (III) (d) None of the (I), (II) and (III)
(e) No correction required
55. Since he was a translator, it was duck soup (I)/ for him to translate 15 pages
from (II)/ Hindi to English in a single day. (III)
(I) Him being a translator, it was duck soup
(II) to translate 15 page from
(III) Hindi into English in a single day.
(a) Only (I) (b) Only (II)
(c) Only (III) (d) None of the (I), (II) and (III)
(e) No correction required
56. NHRC notice, terming the case (I)/a serious violation of the right of life of
patients (II)/came even as 41 more children died of since Saturday. (III)
(I) NHRC noticed, terming the case
(II) a seriously violation of the right to life of patients
(III) came even as 41 more children died from Saturday.
(a) Only (I)   (b) Both (I) and (II)  
(c) All (I), (II) and (III)   (d) None of (I), (II) and (III)
(e) No correction required
EBD_7275
152 Revision Exercises
57. Heavy rain in the last three days triggered flash floods (I)/ in parts of Bihar,
and inundated large areas in Assam and North Bengal (II)/ paralysed normal
life and snapping rail link to the North-East from the rest of the country.(III)
(I) Heavy rain in last three days triggered flash floods
(II) in parts of Bihar and inundated larger area in Assam and North Bengal
(III) paralysing normal life and snapping rail link to the North-East from the
rest of the country.
(a) Only (I) (b) Only (III)
(c) Both (II) and (III) (d) All (I), (II) and (III)
(e) No correction required
58. The effects of the crisis are not just (I) / seen in the dry economic data, (II) /
they are felt as well as in the gut. (III)
(I) The effects of crisis have not just
(II) seen in the dried economic data,
(III) they are felt in the gut as well.
(a) Both (I) and (II) (b) Both (II) and (III)
(c) Only (III) (d) All (I), (II) and (III)
(e) No correction required
59. The country’s post-war Constitution stipulates that (I)/the emperor is no god-
king above the law (II)/as he was before the country’s defeat in 1945. (III)
(I) The country’s post-war Constitution has been stipulated that
(II) the emperor is no god like over the law
(III) as it was before the country’s defeat in 1945.
(a) Only (I) (b) Both (I) and (II)
(c) Both (II) and (III) (d) None of the (I), (II) and (III)
(e) No correction required
60. It proved to be a blunder mistake (I)/ by the Indian team to choose for (II)/
bowling after winning the toss. (III)
(I) It proved to be a blunder
(II) by the Indian team to choosing for
(III) bowling after win the toss.
(a) Only (I)   (b) Both (I) and (II)  
(c) Both (II) and (III)   (d) None of the (I), (II) and (III)
(e) No correction required
Revision Exercises 153
DIRECTIONS (61-80) : Which of the phrases (a), (b), (c) and (d) given below
each question should replace the phrase printed in bold type to make the
sentence grammatically correct? If the sentence is correct as it is given and no
correction is required, mark (e) as the answer.
61. We have hired an advertising agency to prepare a campaign to encourage
people votes.
(a) people from voting (b) voting for people
(c) people to vote (d) people for voting
(e) No correction required
62. During the training program, the new recruits will be briefed about how their
role in the new company.
(a) what their roles (b) about their roles
(c) for its roles (d) which are their role
(e) No correction required
63. The machine is in such poor condition that we have no alternative to buy new
one.
(a) many alternative like (b) any alternative except
(c) no other alternative (d) no alternative but
(e) No correction required
64. Since the deadline has been changed from next week to this Monday, you
should give this work priority.
(a) be given this work priority
(b) prioritized this work
(c) priority this work
(d) not give priority this work
(e) No correction required
65. After the success of our project, we have been receiving more requests than
we do not have the resources to handle them.
(a) most of the requests (b) too many requests
(c) many requests but (d) much requests than
(e) No correction required
66. The Eunuchs Act, enacted in the Nizam’s dominions, has been in force since
1919 explicitly to control “eunuchs”, that is, people who are both “males in
female dress” and those who had undergone “emasculation”.
(a) who were both (b) who are either
(c) who were either (d) who use to ne
(e) No correction required
67. Section 5 provides for the punishment of a eunuch with imprisonment if it
could be found that he “has with him or in his house under his control” a boy
who is less than 16 years old.
(a) it has to be found (b) it found to be that
(c) it’s been found that (d) it is found that
(e) No correction required
EBD_7275
154 Revision Exercises
68. The purpose of elevating certain rights to the stature of guaranteed fundamental
rights is to insulate their exercise to disdain the majorities, whether legislative
or popular.”
(a) for disdaining of (b) for the disdaining of
(c) from the disdain of (d) the disdain of
(e) No correction required
69. Sexual orientation is an essential component of identity. Equal protection
demands protection the identification of every individual without
discrimination.
(a) the identity of (b) for the identification of
(c) of the identity of (d) as the identification of
(e) No correction required
70. There is widespread evidence that the existence of the Eunuchs Act which
resulted in a pervasive and continuing practice of criminalization, illegal
detention, torture in custody, and extreme coercion, which includes a perennial
threat of arrest.
(a) resulted (b) had resulted
(c) have resulted (d) has resulted
(e) No correction required
71. The police were barking on the wrong tree.
(a) barking upon the wrong tree
(b) barking up the wrong tree
(c) baring along the wrong tree
(d) barking over the wrong tree
(e) No correction required
72. Last year, a military plane carrying five army personnel crashed after taking
off from its base camp.
(a) plane that has carried (b) plane that had carried
(c) plane was carrying (d) plane carried
(e) No correction required
73. Special commandos were rushed out from Udhampur to Jammu for the final
assault.
(a) was rushed over
(b) was being rushed in from
(c) had been rushed away from
(d) were rushed in from
(e) No correction required
74. I am looking forward to meeting her in the next weekends.
(a) looking forward to meet
(b) looking forward upon meeting
(c) looking forward for meeting
(d) looking forward into meeting
(e) No correction required
Revision Exercises 155
75. The population of Tokyo is greater than all other town in the words.
(a) greater than that of any other
(b) greatest among any other
(c) greater than those of any other
(d) greater than any other
(e) No correction required
76. The performance of our team was rather worst that I had expected.
(a) bad as I had expected (b) worse than I had expected
(c) worst than was expected (d) worse than expectation
(e) No correction required
77. The intruder stood quietly for few moments.
(a) for few time (b) for the few moments
(c) for moments (d) for a few moments
(e) No correction required
78. The policy has so far succeeded in recovering only a part of the stolen
property.
(a) thus far succeeded for recovery
(b) so far succeeded in recovery of
(c) as for as succeeded in recovery of
(d) so far succeeded to recover
(e) No correction required
79. He asked the crowd if they thought he was right and the crowd shouted that
he did.
(a) that they did (b) that they had
(c) that he is (d) that he didn’t
(e) No correction required
80. Joseph, unnecessarily picked up a quarrel with Manu and left the party.
(a) has picked up (b) picked on
(c) picked (d) picking up
(e) No correction required

READING COMPREHENSION
DIRECTIONS (Qs. 81-90) : Read the following passage carefully and answer
the questions that follow.

Two principles are involved in the controversy about the presence of foreign
controlled media in the country; the free flow of ideas and images across national
borders and the need to safeguard the national interest and preserve cultural
autonomy. Both are valid but both are at loggerheads because each has been used
to promote less lofty goals.
EBD_7275
156 Revision Exercises
The first principle conforms to a moral imperative: freedom to expression cannot
rhyme with restrictions imposed by any government. But the free flow rhetoric also
clouds the fact that the powerful Western, and especially American media, can
and often do present, subtly or brazenly, news in a manner that promotes Western
political, ideological and strategic interests. Besides, Western entertainment
programs present lifestyles and values that run counter to the lifestyles and
values cherished by traditional societies. All this explains why so many Indian
newspapers, magazines and news agencies have sought protection from the courts
to prevent foreign publications and news agencies from operating in the country.
Their arguments are weak on two counts. As the bitter debate on a new world
information and communication order demonstrated in the late seventies and early
eighties, many of those who resent Western ‘invasion’ in the fields of information
and culture are no great friends of democracy. Secondly, the threat of such an
‘invasion’ has been aired by those media groups in the developing countries that
fear that their business interests will be harmed if Western groups, equipped with
large financial and technological resources and superior management skills, are
allowed to operate in the country without let.
The fear is valid but it goes against the grain of the economic reform program. The
presence of foreign newspapers and television channels will increase competition,
which, in the course of time, can only lead to the upgradation of dynamic Indian
newspapers and television channels, even while they drive the rest out of the
market. One way to strike a balance between the two antagonistic principles would
be to allow foreign media entry into the country, provided the India state treats
them at par with the domestic media on all fronts. On the import of technology, for
instance, foreign media cannot be allowed duty concessions denied to their Indian
counterparts. Foreign media will also have to face legal consequences should they
run foul of Indian laws. Why, for example, should the BBC, or Time magazine
or The Economist get away by showing a map of Kashmir, which is at variance
with the official Indian map? Why should they go scot-free when they allow
secessionists and terrorists to air their views without giving the government the
right to reply, or when they depict sexually explicit scenes, which would otherwise
not be cleared by the Censor Board? Since the government can do precious little
in the matter, especially about satellite broadcasts, what if it should consider
attaching the properties of the offending parties? Demands of this kind are bound
to be voiced unless New Delhi makes it clear to the foreign media that they will
have to respect Indian susceptibilities, especially where it concerns the country’s
integrity and its culture. It may be able to derive some inspiration from France’s
successful attempts in the recent GATT to protect its cinematography industry.
81. Which of the following is one of the points weakening the argument to prevent
the entry of foreign media?
(a) Such entry would be against traditional culture
(b) The threat being voiced by those whose business will be harmed by such
an entry
(c) The arguments being put forth are at loggerheads
(d) The foreign media may not be treated on par with the domestic media
(e) None of these
Revision Exercises 157
82. What will be the impact of increasing competition?
(a) The domestic media will not be able to withstand it
(b) The foreign media will not be allowed duty concessions on import of
technology
(c) It will improve Indian newspapers and television
(d) The Indian newspapers and news agencies will seek protection from the
court
(e) None of these
83. Which of the following has been cited as having succeeded in protecting
country?
(a) GATT (b) News Agencies
(c) Television (d) Cultural traditions
(e) None of these
84. Which of the following has been the major recommendation regarding the
entry of foreign media?
(a) It should not be allowed.
(b) It should be welcomed without putting any restrictions.
(c) Allow entry, treating them on par with domestic media.   
(d) Allow entry, provided they do not ask for duty concessions on import of
technology.
(e) None of these
85. In the controversy involving two principles regarding allowing foreign media,
which of the following is against its entry?
(a) Free flow of ideas (b) Preserve culture   
(c) Government restrictions (d) Security across national borders
(e) Western ideology
86. According to the passage, which media in particular promotes Western
interests?
(a) American    (b) Foreign
(c) French (d) Western
(e) None of these
87. Which of the following is the meaning of the phrase “without let”, as used in
the passage?
(a) with no difficulty (b) without confinement
(c) with strings (d) without restrictions   
(e) conducive environment
88. Why would the entry of foreign media harm local interests?
(a) They are better equipped managerially and technologically
(b) Our cultural heritage will be lost
(c) Economic reform programmes will get a setback
(d) Different sets of laws and rules were made applicable for foreign media
(e) None of these
EBD_7275
158 Revision Exercises
89. Which of the following is the meaning of the phrase “at variance”, as used in
the passage?
(a) discrepancy (b) at large
(c) in conformity (d) variable
(e) differing
90. Which of the following seems to be the most likely purpose of writing this
passage?
(a) To criticize foreign media
(b) To highlight the exploitation by developed nations
(c) To highlight the steps and caution to be taken about the entry of foreign
media
(d) To make the public aware of the technological and managerial superiority
of western media
(e) To prevent foreign media from entering our country
DIRECTIONS (Qs. 91-95) : Read the following passage divided into number
of paragraphs carefully and answer the questions that follow it.
Paragraph 1: Judiciary has become the centre of controversy, in the recent past,
on account of the sudden ‘Me’ in the level of judicial intervention. The area of
judicial intervention has been steadily expanding through the device of public
interest litigation. The judiciary has shed its pro-status-quo approach and taken
upon itself the duty to enforce the basic rights of the poor and vulnerable sections
of society, by progressive interpretation and positive action. The Supreme Court
has developed new methods of dispensing justice to the masses through the public
interest litigation.
Paragraph 2: Former Chief Justice P. N. Bhagwati, under whose leadership public
interest litigation attained a new dimension comments that “the Supreme Court
has developed several new commitments. It has carried forward participative
justice. It has laid just standards of procedure. It has made justice more accessible
to citizens”. The term ‘judicial activism’ is intended to refer to, and cover, the
action of the court in excess of, and beyond the power of judicial review. From one
angle it is said to be an act in excess of, or without, jurisdiction. The Constitution
does not confer any authority or jurisdiction for ‘activism’ as such on the Court.
Paragraph 3: Judicial activism refers to the interference of the judiciary in
the legislative and executive fields. It mainly occurs due to the non-activity of
the other organs of the government. Judicial activism is a way through which
relief is provided to the disadvantaged and aggrieved citizens. Judicial activism
is providing a base for policy making in competition with the legislature and
executive. Judicial activism is the rendering of decisions, which are in tune with
the temper and tempo of the times.
Paragraph 4: In short, judicial activism means that instead of judicial restraint,
the Supreme Court and other lower courts become activists and compel the
authority to act and sometimes also direct the government regarding policies and
also matters of administration.
Revision Exercises 159
Paragraph 5: Judicial activism has arisen mainly due to the failure of the executive
and legislatures to act. Secondly, it has arisen also due to the fact that there is a
doubt that the legislature and executive have failed to deliver the goods. Thirdly,
it occurs because the entire system has been plagued by ineffectiveness and
inactiveness. The violation of basic human rights has also led to judicial activism.
Finally, due to the misuse and abuse of some of the provisions of the Constitution,
judicial activism has gained significance.
91. What does the author want to convey in Paragraph 1?
(I) Certain personal issues and agendas in recent past in the level of judicial
intervention have put a question mark on the credibility of the apex court.
(II) The Supreme Court is very concerned about the under-privileged
sections of the society and thus, has come up with an innovative idea to
dispensing justice.
(III) Public Interest Litigation is a boon for the poor and vulnerable sections
of the society as far as the enforcement of their basic rights is concerned.
(a) Only (I) (b) Only (II)
(c) Both (II) and (III) (d) All (I), (II) and (III)
(e) None of these
92. What is the meaning of the sentence ‘From one angle, it is said to be an act in
excess of, or without jurisdiction’ in the context of paragraph 2?
(I) Judicial activism can be exercised by the Supreme Court as it is beyond
the power of judicial review.
(II) Judicial activism does not find any mention in the constitution as far as
its authority or jurisdiction is concerned.
(III) The constitution has not limited the authority or jurisdiction to ‘activism’
hence, it can be exercised over the courts as per the say of the Supreme
Court.
(a) Only (I) (b) Only (II)
(c) Only (III) (d) Both (II) and (III)
(e) Both (I) and (II)
93. What can’t be said about judicial activism in accordance to paragraph 3?
(a) It emerges when the legislative and the judiciary do not do justice with
their work.
(b) It has the right to put question mark on the legislative and executive part
of the government.
(c) It is exercised in accordance with the demand of the situation.
(d) It has given the citizenry ample power to demand for their rights from the
judiciary.
(e) It is giving a foundation for policy making in competition with the other
organs of the government.
EBD_7275
160 Revision Exercises
94. Which of the following statements does not relate to Paragraph 5?
(a) Violation of basic human rights has led to judicial activism.
(b) There is a suspicion on the executive and the legislative organ of the
government as far as their working is concerned.
(c) Judicial activism has carried forward participative justice thereby
justifying that the entire system has been plagued by ineffectiveness and
inactiveness.
(d) Some of the constitutional provisions have been misused.
(e) The failure of the other organs of the government has led to the emergence
of judicial activism.
95. What does the author mean by the phrase ‘judicial restraint’ in Paragraph 4?
(I) The limiting of the exercise of their own power by the courts.
(II) The restriction of power which Supreme Court can impose on other
lower courts.
(III) The liberation of judicial power of the Supreme Court and other courts.
(a) Only (I) (b) Only (II)
(c) Only (III) (d) Both (I) and (II)
(e) All (I), (II) and (II)
DIRECTIONS (Qs. 96-100) : Read the following passage divided into number
of paragraphs carefully and answer the questions that follow it.
Paragraph 1: India is land of Diversity. Our country has various languages,
religion, culture, tradition etc. various elements of Indian culture such as Indian
books on philosophy, Indian cuisine, yoga etc. have created an impact all over the
world. Western culture is also called European civilization, Western civilization or
Western lifestyle. It is based on certain belief, systems, traditional customs, moral
and ethical values. The term not only applies to European countries but to places
where we see spread of European culture.
Paragraph 2: India is a country rich in its heritage and culture, but we are seeing
fading of Indian culture at many places of India, especially at the urban societies
of India. The effect of western culture is greatly seen in our customs, tradition,
social and moral behavior, our love and respect for others. These days a person
loves to live in freedom, he does not want to bind themselves in Indian customs
and traditions. Day by day we see breaking of joint family and more and more
development of nuclear family.
Paragraph 3: Very few are interested in making adjustments and share their
things with other family members, the word privacy is given greater importance
and the love and respect towards other especially elders are decreasing day by day.
In this information age people are too busy to care for others. Western culture has
brought with it the seeds of selfishness in the minds of Indian.
Paragraph 4: These are contradictory to Indian culture which has always taught
to live in harmony with each other and always love and respect everyone at home.
Revision Exercises 161
With lack in experience of a nuclear family due to the absence of grandfather
and grandmother, and both parents working a child fails to learn ethical or moral
values, and learn whatever little he sees and understands from the world and his
teachers. In this way we end up bringing up a child who has little ethical values
and do not hesitate in doing any unfair practices, because no one is there to teach
him good or bad or stop him from doing something bad.
Paragraph 5: We should know what is right and wrong for us. Western culture is
not altogether bad, although it has made our life faster but enhanced technology
has also made our life easier and comfortable. We need to give importance to our
Indian culture which taught us to live in peace and harmony with other by way of
increasing our tolerance and patience. Many people of other countries are realizing
the importance of Indian heritage and are adapting the goodness of Indian culture
such as practice of Yoga and meditation, wisdom and teachings passed by the
ancient saints. The knowledge of Indian wisdom helps human being of any race
to enrich their life.
96. Why is Western culture also known as European culture as given in
paragraph1?
(I) Because it has created impact all over the western countries.
(II) Because it is based on certain belief, systems, traditional customs, moral
and ethical values.
(III) Because Europe is located in the western part of the earth.
(a) Only (I) (b) Only (II)
(c) Only (III) (d) Both (I) and (II)
(e) None of the three
97. How according to the author has Western culture impacted India in Paragraph
2?
(I) Modern Indians, in the contemporary era do not value their customs and
traditions.
(II) Western culture has impacted Indian culture adversely.
(III) Western construct is liked by the Indians very much and they are
practising it in full swing.
(a) Only (I) (b) Only (II)
(c) Both (I) and (II) (d) Both (II) and (III)
(e) All (I), (II) and (III)
98. What can be inferred by the phrase ‘In this information age’ as given in
Paragraph 3?
(I) In today’s world, information is transferred from one person to another
very fast.
(II) This is the era of technological advancement.
(III) Betterment of technology has led to faster mode of accessing information.
(a) Only (I) (b) Only (II)
(c) Both (II) and (III) (d) All (I), (II) and (III)
(e) None of the three
EBD_7275
162 Revision Exercises
99. Which of the following words is opposite to the meaning of the word ‘ethical’
as used in Paragraph 4?
(I) Virtuous
(II) Kosher
(III) Humane
(a) Only (I) (b) Only (II)
(c) Only (III) (d) All (I), (II) and (III)
(e) None of the three
100. What has been delineated in Paragraph 5?
(I) The author is sceptical about Western culture.
(II) Though Western culture has impacted India but, Indian culture has an
edge over it as many people of other countries are realizing the importance
of Indian heritage and are adapting the goodness of Indian culture such
as practice of Yoga and meditation, wisdom and teachings passed by the
ancient saints.
(III) Western culture has changed our lifestyle.
(a) Only (I) (b) Only (II)
(c) Only (III) (d) Both (II) and (III)
(e) None of these

PARAJUMBLES
101. If Sentence (C), “There appears to be no end in sight to the cycle of boom and
bust in the prices of agricultural goods.” is the first sentence, what is the order
of other sentences after rearrangement?
A. Curiously, potato prices were many times higher just months ago amid
scarce supply.
B. With the price of a kilogram of potato dropping as low as under a rupee
in certain wholesale markets, many distressed farmers have left their
produce to rot on the roads, and in cold storage facilities.
C. There appears to be no end in sight to the cycle of boom and bust in the
prices of agricultural goods.
D. Over the last few weeks, across India the price of potatoes has fallen
sharply after a year of bumper production.
E. Last year, the price of other produce like red chilli, tur dal and tomato
witnessed a similar trend of steep falls compared to the previous season.
F. The sharp swing in prices has been explained by the Cobweb phenomenon.
(a) EDABF (b) EFADB
(c) DABEF (d) DBAEF
(e) BDAEF
Revision Exercises 163
102. If Sentence (C), “Ever since the price of bitcoins skyrocketed from a little
under $1,000 (around `63,400 now) in 2016 to touch nearly $20,000 last year,
people have been posing a Hamlet kind of question: to buy or not to buy? ” is
the first sentence, what is the order of other sentences after rearrangement?
A. Some have explicitly warned investors to exercise caution while those
like the Indian government have begun imposing a tax on the gains from
the sale of bitcoins.
B. Their dilemma is understandable.
C. Ever since the price of bitcoins skyrocketed from a little under $1,000
(around `63,400 now) in 2016 to touch nearly $20,000 last year, people
have been posing a Hamlet kind of question: to buy or not to buy?
D. On the other hand, there is the nagging feeling that one may end up
investing in a virtual currency whose price is extremely volatile and
whose true value cannot be assessed.
E. Adding to the confusion, most governments remain non-committal on
the legality of bitcoins since it is not regulated by central banks.
F. On the one hand, there is the fear of losing out on an opportunity to make
money hand over fist by investing or trading in bitcoins since their price
rose by around 20-fold since the start of 2017.
(a) BFDAE (b) BFAED
(c) BFDEA (d) FDBEA
(e) FDEBA
103. If Sentence (C), “There is one strong message from the findings of the Annual
Status of Education Report (Rural) 2017, it is that the Right of Children to
Free and Compulsory Education Act should cover the entire spectrum of 18
years, and not confine itself to those aged 6 to 14. ” is the first sentence, what
is the order of other sentences after rearrangement?
A. Guaranteed inclusion will empower those in the 14-18 age group who are
not enrolled anywhere, and help them acquire finishing education that is
so vital to their participation in the workforce.
B. It is absolutely essential for all of them to get an education that equips
them with the skills, especially job-oriented vocational capabilities,
C. There is one strong message from the findings of the Annual Status of
Education Report (Rural) 2017, it is that the Right of Children to Free
and Compulsory Education Act should cover the entire spectrum of 18
years, and not confine itself to those aged 6 to 14.
D. Unfortunately, the state of rural elementary education is far from
encouraging.
E. if the expectation of a demographic dividend is to be meaningful.
F. The ASER sample study estimates that 14% of this age group — a total
of 125 million young Indians in this category — are not enrolled.
(a) AFBED (b) BEAFD
(c) DAFBE (d) AFEBD
(e) FABED
EBD_7275
164 Revision Exercises
104. If Sentence (F), “The sharp rise in bond yields has hit banks with losses
on treasury operations dominated by sovereign bond holdings.” is the first
sentence, what is the order of other sentences after rearrangement?
A. The yield on Indian 10-year benchmark government bonds has risen
steeply, from about 6.5% at the end of August to 7.56% on January 16.
B. Bankers have pleaded that the Reserve Bank of India allow them to
stagger the reporting of these losses over several quarters.
C. Rating agency ICRA believes the fall in bond prices on expectation of
the Central government breaching its fiscal deficit target has led to banks
suffering a loss on paper of over Rs.15,500 crore in the quarter that ended
in December.
D. In seeking leeway, they have pointed to the huge burden imposed on their
balance sheets by non-performing assets clogging the banking system.
E. Even the yield on newly issued 10-year bonds that would mature in 2028
has inched up 27 basis points since January 5.
F. The sharp rise in bond yields has hit banks with losses on treasury
operations dominated by sovereign bond holdings.
(a) ABECD (b) ABCED
(c) CAEBD (d) BCADE
(e) CEBAD
105. If Sentence (F), “The winter session of Parliament saw more political
positioning than appraisal of a legislation to make instant triple talaq a
criminal offence. ” is the first sentence, what is the order of other sentences
after rearrangement?
A. The Opposition has raised three concerns: whether a civil wrong, mainly
a breach of a marriage contract in an arbitrary manner, ought to be treated
as a crime; whether it is not a contradiction of sorts for the law to jail a
husband for pronouncing instant talaq and also mandate that he pay a
subsistence allowance to the wife; and whether making it a cognizable
and non-bailable offence would lead to it being misused against Muslim
men.
B. Further, some see an internal contradiction in the way the law is sought
to be framed.
C. The core question is whether resorting to an illegal and arbitrary form
of divorce should necessarily lead to a prison term for the offending
husband.
D. With the Muslim Women (Protection of Rights on Marriage) Bill
pending in the Rajya Sabha, the best option would be to refer it to a
select committee to help bring about a consensus on how to address the
problem of talaq-e-biddat
E. A three-year prison term, besides a fine, also raises the issue of
proportionality.
Revision Exercises 165
F. The winter session of Parliament saw more political positioning than
appraisal of a legislation to make instant triple talaq a criminal offence.
(a) ABCED (b) EDCBA
(c) CDAEB (d) DABCE
(e) DCEAB
106. If sentence (C) “The decision to use Aadhaar as proof of identity in the
annual collation of data on teachers employed in higher education has led
to an uncomfortable discovery: nearly a tenth of them turned out to be ghost
teachers.” is the first sentence, what is the order of other sentences after
rearrangement?
(A) Around 130,000 teachers were found to be fake.
(B) The human resource development (HRD) ministry in 2017 told colleges
and universities across India that while furnishing data they need to
give the Aadhaar number of the faculty members to authenticate their
presence.
(C) The decision to use Aadhaar as proof of identity in the annual collation of
data on teachers employed in higher education has led to an uncomfortable
discovery: nearly a tenth of them turned out to be ghost teachers.
(D) the bad news is that the country has just found out that understaffing in
higher education institutes is far greater than what has been estimated so
far.
(E) While the good news is that this will lead to a focus on improving the
quality of teaching,
(F) India has about 1.4 million teachers in colleges and universities.
(a) DFAEB (b) BAFDE
(c) BEDAF (d) AFEDB
(e) DAEFB
107. If sentence (C) “The Reserve Bank of India will shortly issue new ` 10 notes
under the Mahatma Gandhi series.” is the first sentence, what is the order of
other sentences after rearrangement?
(A) With chocolate brown colour as the base, the new note will bear the
picture of Konark Sun Temple.
(B) The central bank has already printed around 1 billion pieces of the new `
10 note.
(C) The Reserve Bank of India will shortly issue new ` 10 notes under the
Mahatma Gandhi series.
(D) The change in design in the old ` 10 note was last made in 2005.
(E) The design received the go-ahead from the government last week, said
the two of the people cited earlier.
(F) In August last year, RBI had introduced the new ` 200 and ` 50 notes
under the Mahatma Gandhi series.
(a) DFAEB (b) AEFDB
(c) FEDAB (d) BDEFA
(e) BAEDF
EBD_7275
166 Revision Exercises
108. If sentence (C) “The continuing failure of the Myanmar government to act
decisively and urgently to protect civilians from the raging crossfire between
the security forces and insurgents is shocking.” is the first sentence, what is
the order of other sentences after rearrangement?
(A) The latest flare-up began last Friday when militants suspected to be from
the Arakan Rohingya Salvation Army attacked military and police out-
posts.
(B) That should have served as a caution against an excessive counter-insur-
gency operation, a real possibility given the history of systematic perse-
cution of the Muslim minorities in Rakhine.
(C) The continuing failure of the Myanmar government to act decisively and
urgently to protect civilians from the raging crossfire between the secu-
rity forces and insurgents is shocking.
(D) Most of the victims are women and children, according to the UN’s Inter-
national Organisation for Migration, which has called for additional aid
to cope with Dhaka’s refugee situation.
(E) The military crackdown that followed has been widely condemned as
disproportionate and the government accused of being an onlooker.
(F) The recent clashes in the western State of Rakhine have claimed over
70 lives and forced thousands of Rohingya to flee across the border into
Bangladesh, in a rapidly deteriorating humanitarian crisis.
(a) DAFEB (b) FDABE
(c) FEDBA (d) BDEAF
(e) BEDFA
109. If sentence (C) “The success of the first developmental flight of GSLV Mark
III will enable indigenous launching up to 4 tonne class of communication
satellites in the future.” is the first sentence, what is the order of other sen-
tences after rearrangement?
(A) Next developmental launch is in first half of 2018.
(B) ISRO has been providing commercial launch services for earth obser-
vation satellites and small satellites onboard the Polar Satellite Launch
Vehicle (PSLV) through Antrix Corporation Limited since 1999.
(C) The success of the first developmental flight of GSLV Mark III will en-
able indigenous launching up to 4 tonne class of communication satel-
lites in the future.
(D) The successful launch of GSLV Mark III is a step ahead in building cred-
ibility in launching.
(E) Some more launches of GSLV-MK III will be needed before GSLV MK
III gets recognised internationally as a vehicle for transportation to space.
(F) ISRO has just completed one launch of GSLV-MK III.
(a) BDFAE (b) FBEDA
(c) FEDBA (d) AEDBF
(e) BFADE
Revision Exercises 167
110. If sentence (C) “The Central Government is the competent authority to initiate
disciplinary proceedings against IAS officers for misconducts while working
in the affairs of Government of India.” is the first sentence, what is the order
of other sentences after rearrangement?
(A) The Central Government is also the competent authority in respect of
disciplinary proceedings initiated by the State Government where subse-
quent to inquiry, a major penalty has been proposed.
(B) The Central Government also considers proposals for sanction for prose-
cution against IAS Officers for offences under P.C. Act, 1988, subsequent
to completion of investigation and filing of chargesheet.
(C) The Central Government is the competent authority to initiate disciplin-
ary proceedings against IAS officers for misconducts while working in
the affairs of Government of India.
(D) Department of Post and Telegraph in exercise of powers conferred under
sub-rule 3 of the Rule 16 of the All India Services (Death-cum-retirement
Benefits) Rules 1958 has prematurely retired, in public interest, 4 (four)
IAS officers since 2014.
(E) There are 36 disciplinary proceeding against IAS officers (State and Cen-
tral cases) currently in progress at various stages.
(F) In the past one year 8 cases for prosecution sanction have been granted
by the Central Government.
(a) AEDBF (b) FADBE
(c) FEDBA (d) AEBFD
(e) BFADE

IDIOMS & PHRASES


DIRECTIONS (Qs. 111-120) : In the following questions, a part of the sentence
is given in bold, it is then followed by three sentences which try to explain
the meaning of the phrase given in bold. Choose the best set of alternatives
from the five options given below each question which explains the meaning
of the phrase correctly without altering the meaning of the sentence given as
question.

111. The dichotomy is the striking feature. An estimated 20 per cent of our pop-
ulation is economically advanced with access to the latest technology, while
the rest wallows in inhuman conditions
(I) The contrast can clearly be understood by the fact that an estimated one-
fifth of our population is economically sound with access to the latest
technology while the rest are still in critical condition.
(II) It is quite attractive that around 20 per cent of our population has access
to the latest technology unlike the rest of the population thereby, contrib-
uting more in the economic development.
EBD_7275
168 Revision Exercises
(III) It is shameful that the majority of our population is unaware of advanced
technology and thus, they are not able to compete with the technologi-
cally advanced people of our country.
(a) Only (I) is correct
(b) Only (III) is correct
(c) Both (II) and (III) are correct
(d) Both (I) and (III) are correct
(e) None is correct
112. It may have taken 20 years to develop the export market for UK garden fur-
niture, but if there is to be a price war, the industry could be back to square
one within 12 months.
(I) It took two decades to develop the export market for UK garden furni-
ture, however, in case of high competition among its peers, it will always
emerge victorious and that too within 1 year.
(II) The development of the export market of UK garden furniture took 20
years and it has reached to such a position that in case of price war, it can
defeat all other industries within 12 months.
(III) Despite 20 years of hard work done to develop the export market of UK
garden furniture, still the industry is sceptical about it success. It may fall
down in case of a price war.
(a) Only (I) is correct (b) Only (II) is correct
(c) Only (III) is correct (d) Both (I) and (II) are correct
(e) None is correct
113. As the situation got out of control during the match, the captain of the team
tried to put oil over troubled waters.
(I) Seeing the situation going out of control, the captain of the team tried to
calm down the players.
(II) As the situation was out of control, the captain of the team had no other
option than to support his teammates and fight with the opposition.
(III) Taking advantage of the bad situation, the captain of the team too argued
along with his teammates and tried to worsen the situation.
(a) Only (I) is correct (b) Only (II) is correct
(c) Only (III) is correct (d) Both (II) and (III) are correct
(e) None is correct
114. The manager had to eat a humble pie after the workers decided to go on
strike to protest against the biased salary hike and promotions.
(I) The manager politely refused to take his decision back despite knowing
that the employees would go for a strike against the nepotism in salary
hike and promotions.
(II) Seeing the urgency of the situation and the threat of strike given by the
workers, the manager withdrew his decision of giving salary hike and
promotions to his favourite employees.
Revision Exercises 169
(III) Knowing the fact that the employees would go for a strike against the
biased salary hike and promotions, the manager gave up his pride and
apologized for his mistake.
(a) Only (I) is correct (b) Only (II) is correct
(c) Only (III) is correct (d) Both (II) and (III) are correct
(e) None is correct
115. The teacher tried his best to explain the importance of the chapter to the stu-
dents but soon realized that he was casting pearls before swine.
(I) The teacher soon realized that whatever he was explaining to the students
about the chapter was grasped by them the way he expected.
(II) Despite the hard work done by the teacher to explain the importance of the
chapter, the students were busy making mockery of him.
(III) After trying his best to make the students understand the importance of
the chapter, the teacher soon realized that they are not recognising its
worth.
(a) Only (I) is correct (b) Only (II) is correct
(c) Only (III) is correct (d) Both (II) and (III) are correct
(e) None is correct
116. Any curb on access to higher education, for instance, would run counter to
the prevailing mood among the middle class, which wants further improve-
ments to the overall standard of living.
(I) Any restriction on access to higher education would barely affect the mood
of the middle class which expects further improvements to the overall
standard of living.
(II) The present spirit of the middle class which wants further improvements
to the overall standard of living will worsen if the access to higher educa-
tion is paused for any reason.
(III) Any clampdown on the access to higher education is likely to alter the
mood of the middle class which wants further improvements to the over-
all standard of living.
(a) Only (I) is correct (b) Only (II) is correct
(c) Only (III) is correct (d) Both (II) and (III) are correct
(e) None is correct
117. We were all set for the picnic but the sudden change in the plan by my dad to
go for the movie instead of picnic came out of the blue.
(I) The sudden change in the plan to go for the movie instead of picnic was a
well thought idea by my dad.
(II) The sudden change made in the plan by my father to go for the movie
instead of picnic was unexpected for us.
EBD_7275
170 Revision Exercises
(III) My dad had already made his mind to change the plan to go for the movie
instead of picnic, however, we had intuitions about the change.
(a) Only (I) is correct (b) Only (II) is correct
(c) Only (III) is correct (d) Both (I) and (III) are correct
(e) None is correct
118. The Supreme Court has struck a blow for the rights of the disabled, with a
direction to the Central and State governments to provide full access to public
facilities, such as buildings and transport, within stipulated deadlines.
(I) The Supreme Court has ordered all the states to provide full access to
public facilities like building and transport to the disabled and they must
not ignore the true spirit and purpose of the law made for these people.
(II) The Supreme Court has ordered the central government to provide the
basic amenities to the disabled within the stipulated time period.
(III) The Supreme Court has warned the Central and State governments for
the rights of the disabled which they are deprived of and has ordered
them to provide full access to public facilities, such as buildings and
transport, within stipulated deadlines.
(a) Only (I) is correct (b) Only (II) is correct
(c) Only (III) is correct (d) All are correct
(e) None is correct
119. At the UN General Assembly, USA and India accused Pakistan of funding
terrorism in South-East Asia but Pakistan kept on beating around the bush.
(I) USA and India accused Pakistan of funding terrorism in South-East Asia
at the UN General Assembly however, Pakistan kept on refusing it.
(II) Pakistan was accused of funding terrorism in South-East Asia at the UN
General Assembly by India and USA but Pakistan kept on avoiding this
topic.
(III) The rising terror in the South-East Asia was the major concern at the UN
General Assembly. USA and India accused Pakistan of funding terrorism
in the area which was partially accepted by Pakistan.
(a) Only (I) is correct (b) Only (II) is correct
(c) Only (III) is correct (d) All are correct
(e) None is correct
120. Manish was in doldrums after he was badly scolded by the teacher in front
of all the students.
(I) Manish was in huge anger after being badly scolded by the teacher in
front of all the students.
(II) Manish felt so insulted after being badly scolded by the teacher in front
of all his colleagues that he started planning to take the revenge of the
insult.
(III) Manish was depressed after he was badly scolded by the teacher in front
of all the students.
(a) Only (I) is correct (b) Only (II) is correct
(c) Only (III) is correct (d) Both (I) and (II) are correct
(e) None is correct
Revision Exercises 171
SENTENCE FILLERS
DIRECTIONS (Qs. 121-130) : In each of the following questions, a paragraph
with a blank is given. From the five choices given below, select the sentence
which can go into the blank to make the paragraph logically coherent.

121. Srinagar is the capital of Kashmir. There are very beautiful scenes all round.
The Dal Lake is one among them. [__________] We can hire one of them and
voyage along the length and breadth of the lake or live in it for a week or so.
(a) There are several guides to take us round.
(b) There, we can see a number of house boats waiting to be hired by the
tourists.
(c) It is difficult to count them.
(d) It was dark inside and bright outside.
(e) There are several tourists looking around the place.
122. Polio-affected children are found everywhere in India. Recently, the Gov-
ernment has started Polio-eradication Scheme. [__________] Unless we take
care to co-operate, the purpose cannot be fulfilled.
(a) We must help children to take the proper vaccine.
(b) We must take the children to get vaccinated.
(c) Children must remain without any movement.
(d) We must make children exercise.
(e) We need experts to undertake such projects.
123. Many film-stars have recently migrated to the area of television. [__________]
As the viewers increase their popularity also increases. T.V serials appear to
be more paying than the feature films.
(a) Television gives a better chance of action.
(b) There they have a greater number of viewers.
(c) Television is a house hold affair.
(d) If we do not like a TV program we can turn it off.
(e) Television is useful in many ways.
124. Man is trying to find out modern means of producing electric power. The solar
panel is one of them. [__________] This electricity can light lamps, turn fans
or work small household appliances.
(a) It is very cheap and affordable.
(b) We ourselves can make one such device.
(c) It converts sunlight into electricity.
(d) It is easy to work and beautiful to look at.
(e) Electricity is a good substitute for sunlight.
EBD_7275
172 Revision Exercises
125. Air-pollution is one of the gravest problems faced by city-dwellers. Foul gas
liberated from heaps of waste matter is one source of pollution. [__________]
Recently, the Government has made arrangements to measure it. If it is above
the allowable limit, the vehicles will be prohibited from plying along the pub-
lic roads.
(a) Another source is the smoke emitted by vehicles.
(b) Foul water in the channels is another.
(c) The smell from toddy shops pollutes the air.
(d) Open drainage pollutes the air.
(e) People suffer due to smoke emitted by vehicles.
126. Providing benefits for women on maternity leave and children is a societal re-
sponsibility which can be funded in a large country through a combination of
general taxation and contributory payments from those who have the means.
Health care should be treated as a right and deliveries handled without cost
to women. ________________________________ Such a policy would har-
monise the varying maternity benefit provisions found in different laws that
govern labour at present.
(a) The income guarantees during the pregnancy period can be ensured
through a universal social insurance system.
(b) Beneficiaries covered by the latest amendment must be protected from
discrimination through clear provisions.
(c) Mandating creche facilities to help women workers under the changed
law is a forward-looking move.
(d) Women’s empowerment can be achieved through universal initiatives,
not by imposing conditionalities to avail benefits.
(e) Access to welfare support has become even more critical as workers mi-
grate frequently due to economic changes.
127. Fringe elements affiliated with the BJP have been in the news ever since the
party came to power. Activists who pretend to be associated with the Trin-
amool Congress in West Bengal and the Samajwadi Party in Uttar Pradesh
too have had their day in the news. _________________________. First, the
“fringe” is encouraged by the party as a strategy to appeal to more extreme
elements within the party and to polarise politics. Second, and more specific
to the right wing, some believe that these individuals’ provocative actions and
remarks are indicative of the government’s tacit support for Hindutva prin-
ciples.
(a) There are two reasons of the palpable polarization in politics.
(b) There are also local politicians who have misperceived favourable re-
sponses or miscalculated the impacts of their actions.
(c) There are two widely held explanations for such activism.
(d) In our view, this results from a systemic problem with our politics.
(e) Their two actions have served to embarrass the BJP and the Central gov-
ernment.
Revision Exercises 173
128. Traditional pharmacies have been knocking at the doors of the govern-
ment for some time now as they face intense competition from e-phar-
macies. Their profit margins and market share have faced pressure in re-
cent years from e-pharmacies that often offer medicines at cheaper prices.
____________________________. The AIOCD has repeatedly accused
e-pharmacies of a wide range of malpractices, including selling fake drugs
and enabling self-medication. The organisation has been citing these issues to
seek a ban on the sale of drugs online.
(a) While this has improved the accessibility of drugs to a wider population,
the concern of traditional pharmacists too is easy to understand.
(b) The Ministry of Health and Family Welfare proposed the setting up of an
e-portal to track and regulate the sale of drugs across the retail chain.
(c) The risks associated with e-pharmacies, especially when it comes to the
dispensation of prescription drugs without the necessary checks, cannot
be taken lightly.
(d) However, the Ministry’s plan on regulating e-pharmacies is a rather out-
dated one.
(e) But perhaps the only thing clear from the All India Organisation of
Chemists and Druggists (AIOCD)’s demands is its intention to protect
the business interests of traditional brick-and-mortar pharmacies
129. With the direction of global headwinds remaining uncertain, growth in gov-
ernment spending budgeted to be lower this year compared to last year, and
private investment virtually absent, the lowering GDP numbers should serve
as a reality check. _____________________________. While the govern-
ment has vigorously underlined its reform achievements of the last three
years, such as the Goods and Services Tax that rolls out in July, a mission-
mode reform reboot is urgently needed and that can only begin if the problem
is suitably acknowledged by policymakers.
(a) While lower inflation and growth may soften the RBI’s outlook, there is
little that monetary policy alone can do at this juncture to revive animal
spirits.
(b) Returning to the 8% growth mark is going to be a big challenge.
(c) In fact, the only reason the 7.1% estimate has held up is because growth
for the previous quarters was revised upwards.
(d) Private consumption grew at the slowest pace in five quarters, even as
construction and manufacturing activities dipped sharply.
(e) Yet, whichever way one looks at it, the note ban seems to have exacer-
bated the problem, particularly for India’s large informal economy that
the poor depend on, as even the World Bank has now noted.
EBD_7275
174 Revision Exercises
130. The United States currently gives an impression of being at war with it-
self. This stems from a series of charges and countercharges levied against
President Donald Trump and his advisers, including that of collusion with
the Russians, who are accused of meddling with the presidential election.
_____________________________. Meanwhile, the kaleidoscopic nature
of the changes taking place in the top echelons of the new administration is
hardly helping matters. The peremptory actions of the president, such as the
dismissal of Federal Bureau of Investigation Director James Comey, have
only aggravated this situation. Almost every step taken by the new adminis-
tration is leading to partisan rows.
(a) One of the principal charges against members of the Trump team is that
they maintained improper contact with Russian diplomats.
(b) The media and intelligence agencies are far from impartial in their be-
haviour.
(c) Several probes have already been launched in this connection.
(d) Barack Obama, Mr. Trump’s predecessor, is by contrast credited cur-
rently with many more virtues than at any time when he was in office.
(e) What has led to a fractured society in the U.S. today carries a message for
democracies everywhere.

EVALUATING INFERENCE
DIRECTIONS (Qs. 131-140) : In each of the given questions, an inference is
given in bold which is then followed by three paragraphs. You have to find
the paragraph(s) from where it is inferred. Choose the option with the best
possible outcome as your choice.
131. Triple talaq is banned now
(I) The Supreme Court said triple talaq violates the fundamental rights of
Muslim women as it irrevocably ends marriage without any chance of
reconciliation. Triple talaq,or verbal divorce, is practiced by some in the
Muslim community to instantly divorce their wives by saying talaq three
times.
(II) By ruling the discriminatory practice of instant triple talaq as unconsti-
tutional and unlawful, the Supreme Court has sent out a clear message
that personal law can no longer be privileged over fundamental rights.
Three of the five judges on the Constitution Bench have not accepted the
argument that instant talaq, or talaq-e-biddat, is essential to Islam and,
therefore, deserves constitutional protection under Article 25.
(III) The Centre’s proposal to make instant triple talaq an offence punishable
with three-year imprisonment and a fine is an unnecessary attempt to
convert a civil wrong into a criminal act. By a three-two majority, the
Supreme Court has already declared, and correctly, that the practice of
talaq-e-biddat, or instant divorce of a Muslim woman by uttering the
word ‘talaq’ thrice, is illegal and unenforceable.
Revision Exercises 175
(a) Only (I) is correct (b) Only (II) is correct
(c) Only (III) is correct (d) Both (II) and (III) are correct
(e) All are correct
132. It is good to invest in Bitcoin
(I) India’s policy on Bitcoin regulation is still evolving and no legal frame-
work exists. The RBI has cautioned against its use, informing users,
holders, investors and traders dealing with virtual currencies that they
are doing so at their own risk.
(II) One lakh rupees invested in bitcoin in 2010 would be worth a few hun-
dred crore rupees today. That is the kind of extraordinary return the digi-
tal currency has given investors as its price has witnessed a meteoric rise,
from just a few cents in 2010 to hit a lifetime high of over $11,000 last
week.
(III) Even if you become a bitcoin miner, there is no guarantee that you would
be able to mine a certain number of bitcoins. Any scheme related to bit-
coins promising a fixed return is likely a tall promise best avoided.
(a) Only (I) is correct (b) Only (II) is correct
(c) Only (III) is correct (d) Both (II) and (III) are correct
(e) All are correct
133. Cyber security is a major concern for country like India
(I) India is one of the key players in the digital and knowledge-based econ-
omy, holding more than a 50% share of the world’s outsourcing mar-
ket. Pioneering and technology-inspired programmes such as Aadhaar,
MyGov, Government e-Market, DigiLocker, Bharat Net, Startup India,
Skill India and Smart Cities are propelling India towards technological
competence and transformation. India is already the third largest hub for
technology-driven startups in the world and its Information and Commu-
nications Technology sector is estimated to reach the $225 billion land-
mark by 2020.
(II) To encourage development of new technologies in the field of cyber se-
curity, the Ministry of Electronics and Information Technology will offer
challenge grants of up to Rs 5 crore to start-ups to spur research and
development, Minister for Electronics and IT said on the previous day.
“We are in the process of working with Data Security Council of India to
conduct challenge grant for cyber security…,” the Minister added.
(III) Two things set aside India’s digital spaces from that of major powers
such as the United States and China: design and density. India is a net
information exporter. Its information highways point west, carrying with
them the data of millions of Indians. This is not a design flaw, but simply
reflects the popularity of social media platforms and the lack of any seri-
ous effort by the Indian government to restrict the flow of data.
(a) Only (I) is correct (b) Only (II) is correct
(c) Only (III) is correct (d) Both (II) and (III) are correct
(e) None is correct
EBD_7275
176 Revision Exercises
134. Demonetisation has benefitted the Indian Economy
(I) While the jury is still out on whether last year’s demonetisation has
harmed the Indian economy, the government’s Chief Statistician main-
tains that the picture will become clear in the current fiscal only after
data from the government and company accounts come in. He maintains
he has reservations about “making quick” judgements about the note
ban decision that had affected the economy at multiple levels, and that it
should not be seen just from the perspective of cash replacement, but as
one that produced many benefits too.
(II) Listing out the advantages of demonetisation, Union Finance Minister
said that direct tax collections had risen 15.7% till September 18, adding
that undisclosed income of ` 5,400 crore was also detected. “Net collec-
tions up to September 18 in the current financial year rose to ` 3.7 lakh
crore, a growth of 15.7%. The revenue collections in case of direct taxes
rose to ` 8,49,818 crore during 2016-17, a growth of 14.5%,” he said.
(III) Indian Economy has witnessed close to 20% decline in currency in cir-
culation, number of tax payers has considerably increased and a large
number of shell companies have been identified.
(a) Only (I) is correct (b) Only (II) is correct
(c) Only (III) is correct (d) Both (II) and (III) are correct
(e) All are correct
135. The government is trying hard to lure FDI in India
(I) In yet another significant move to attract Foreign Direct Investment
(FDI), the government has opened the door wider in several major sec-
tors of the Indian economy, through what it calls “path-breaking” amend-
ments in the extant FDI policy.
(II) In less than a year, the Government of India has announced yet another
set of “radical changes” in foreign direct investment (FDI) policies. The
earlier announcement in November 2015 introduced changes in 15 major
sectors, and the latest announcement covers nine sectors which seek to
further simplify the regulations governing FDI in the country and make
India an attractive destination for foreign investors”.
(III) For India, the servicing burden of FDI in terms of repatriations, dividend
payments and payments for use of intellectual property is now showing
up prominently. About half of the inflows into India during the past six
years were balanced by outflows.
(a) Only (I) is correct (b) Only (II) is correct
(c) Only (III) is correct (d) Both (I) and (II) are correct
(e) All are correct
136. Does India need Bullet Trains?
(I) The proposed bullet train project is just a piece of stone that has been
laid by our Prime Minister and Japanese premier, but it has already been
written off as a white elephant by most analysts and commentators. They
are probably right. The project, of course, is alarmingly expensive.
Revision Exercises 177
(II) The government had set an ambitious deadline to complete the bullet
train project on August 15, 2022 when India marked 75 years of Indepen-
dence. The project will be executed through a special purpose vehicle,
the National High Speed Rail Corporation Ltd.“The bullet train project
will take care of high speed, high growth and high-end technology,” Mr.
Modi said, describing it as “a symbol of New India” that his government
wants to build by 2022.
(III) Bullet train in India is a vanity project which has little or no justifica-
tion on the grounds of economic viability or public service. Even the
vanity angle — looking to position India among the ranks of developed
countries — is a huge overreach. Only a handful of high-income coun-
tries with specific demographics have high-speed rail (HSR), while many
have failed in their efforts, others have abandoned it after studying it. The
main problem is viability, given the huge costs involved.
(a) Only (I) is correct (b) Only (II) is correct
(c) Only (III) is correct (d) Both (I) and (III) are correct
(e) All are correct
137. Indian art and culture is far ahead from Western art and culture
(I) Art in India is still very European centric. Why isn’t Indian art given its
due importance? “Indian art is spiritual, but it has nothing to do with any
particular sect. A common misconception among society is that Indian art
is religious. “The two should not be confused. It is more spiritual and less
about rituals. For instance, the character Krishna means so many things.
It is more of a symbol than just a god.” Similarly, Arjuna is not just the
character Arjuna, when depicted in a painting. He represents valour, ac-
tion and mind.
(II) Assertive cultural pride is understandable, even justified only when a
group is breaking away from prolonged cultural subjugation and humili-
ation, as was the case in mid-19th century India, when profound distor-
tions were introduced by cultural imperialism in our self-understandings.
But already by early 20th century, in the expressions of Vivekananda,
Tagore and Gandhi, we see an articulation of legitimate cultural pride
that behoves a confident cultural community. What then is the need for
such vociferous assertion now?
(III) During the peak of the trade in mid-17th century, millions of yards of In-
dian cloth were being sold in markets as far as Japan, Africa, Middle-East
and Europe. India’s central location in the Indian Ocean basin was ideal
for trading textiles to both East and West, with Gujarat, the Coromandel
Coast and Bengal being the major trading centres.
(a) Only (I) is correct (b) Only (II) is correct
(c) Only (III) is correct (d) Both (I) and (III) are correct
(e) None is correct
EBD_7275
178 Revision Exercises
138. The Doklam issue between India and China no more exists
(I) The resolution of the Sino-Indian military stand-off at Doklam, that last-
ed close to two and a half months, is a much-awaited and welcome de-
velopment where patient statecraft and deft diplomacy seem to have paid
off. Even as several significant questions remain unanswered about the
terms and conditions of the resolution, it provides New Delhi and Beijing
an opportunity to reflect over what went wrong and rejig this important
bilateral relationship.  
(II) Our Prime Minister at the BRICS summit was confident that the resolu-
tion of two-month old Doklam stand-off between the People’s Liberation
Army (PLA) and the Indian Army in Bhutan has gone into annals of
military history. The resolution of the Doklam stand-off has now become
a case study on how to deal with China, the rising global power, with our
government employing deft, principled diplomacy and steely military re-
solve to checkmate hardline PLA generals in Beijing.
(III) Our Union Home Minister said that India has become a powerful nation
and that is why it was able to resolve the standoff at Doklam with China.
“Had India remained weak, the Doklam standoff would not have been
resolved till now. It was possible only because India has become a world
power,” he said. Troops from India and China had been locked in a face
off in the Doklam region for over two months. “Everyone was expecting
that relation between China and India will deteriorate due to the Doklam
issue, but both the countries resolved the issue with comprehension,” he
added.
(a) Only (I) is correct (b) Only (II) is correct
(c) Only (III) is correct (d) Both (I) and (III) are correct
(e) All are correct
139. There is no provision for banning a film in the certification rule
(I) Constitutional guarantees of freedom of expression are under threat in
India. How can some people threaten to kill or maim persons associated
with films they don’t like and haven’t even seen? Censorship is nothing
but a dictatorial weapon used by people who do not want the public to
know what is really happening in the country. Banning a movie is clearly
not only illegal but shockingly naive and unwise.
(II) Some of the burning issues that confront us are: How does the Consti-
tution of India define freedom of speech and expression? What are the
limits on the said freedom? Why are films banned? Are these bans consti-
tutionally valid? What views have been expressed by the final interpreter
of the Constitution, the Supreme Court of India, about these bans on the
films? Are we on the right constitutional path when we ban films? What
consequences would these bans have on our freedom of speech and ex-
pression and on the rule of law?
Revision Exercises 179
(III) To ban a film in India, reacting to demands from some, is grave consti-
tutional impropriety. We tolerate such foolish and sometimes dangerous
appeals not because they may prove true but because freedom of speech
is indivisible. That liberty cannot be denied to some ideas and saved for
others. The endeavour here is to highlight that banning a film is not only
unconstitutional and illegal but also imprudent.
(a) Only (I) is correct (b) Only (II) is correct
(c) Only (III) is correct (d) Both (I) and (III) are correct
(e) All are correct
140. Pollution is adversely affecting the health of children.
(I) A study conducted on Delhi children and released recently in the Jour-
nal of Indian Paediatrics provides powerful evidence that shows chil-
dren growing up in polluted environments like the Capital have reduced
lung growth compared to children in developed countries like the United
States.
(II) Bad food habits is taking a toll on our children, warn doctors. A group of
doctors have published a multi-centric study to drive home the ill effects
of moving away from healthy eating habits and opting for easy-to-use
and widely accessible processed food.
(III) Children living in big cities such as Delhi, are likely to grow susceptible
to allergic ailments, more than adults, due to urban pollution, especially
air, health experts said. Infants and children living in metro cities are
inhaling polluted air and therefore their resistance to allergic ailments is
lowered at a very young age, making them more susceptible to contract
various allergies when they grow up, compared to adults.
(a) Only (I) is correct (b) Only (II) is correct
(c) Only (III) is correct (d) Both (I) and (III) are correct
(e) All are correct

SENTENCE FORMATION
DIRECTIONS (Qs. 141-150) : There are sets of four statements in question
given below which when connected using the correct sentence structure forms
a complete single sentence without altering the meaning of the sentences given
in the question. There are four options given below the question, choose the
sentence that forms the correct formation of single sentence which is both
grammatically correct and contextually meaningful. If none follows, choose
(e) as your answer.
141. It spent some percentage of its GDP on research and development; A chapter
in the Economic Survey has data about India’s spending on GDP; Last year,
it spent only 0.5% of its GDP; It was far below its economic capacity on
research.
(a) India spent far below its economical capacity on research last year,
according to a chapter in the Economic Survey as its spending was only
0.5% of its GDP in research and development.
EBD_7275
180 Revision Exercises
(b) As India spent only 0.5% of its GDP in research and development last
year as per the data revealed by a page of the Economic Survey, it can be
concluded that it spent far below its economical capacity on research.
(c) A page in the Economic Survey states that, India’s expenditure on research
and development was far below its economic capacity on research as it
spent only 0.5% of its GDP on research and development.
(d) According to a chapter in the Economic Survey, India spent only 0.5% of
its GDP on research and development in the last year which is far below
its economic capacity on research.
(e) None of the above is correct
142. GST uses a unique modified deterrent behavioural model; It promotes tax
compliance behaviour as a compulsion along with a self-promoting motive;
But to shift from noncompliance to compliance we need a paradigm shift in
culture; It is completely overlooked in any discussion on GST.
(a) To shift from non-compliance to compliance we need a paradigm shift
in culture which is completely overlooked in any discussion on GST
which uses a unique modified deterrent behavioural model that promotes
tax compliance behaviour as a compulsion along with a self-promoting
motive.
(b) GST uses a unique modified deterrent behavioural model, wherein it
promotes tax compliance behaviour as a compulsion along with a self-
profiting motive, however, to shift from non-compliance to compliance,
we need a paradigm shift in culture, which is completely overlooked in
any discussion on GST.
(c) To promote tax compliance behaviour GST uses a unique modified
deterrent behavioural model which shifts from non-compliance to
compliance as a paradigm shift in culture, is completely overlooked in
any discussion on GST.
(d) A paradigm shift in culture of non-compliance to compliance is
completely overlooked in any discussion on GST; that’s why GST uses
a unique modified deterrent behavioural model which promotes tax
compliance behaviour.
(e) None of the above is correct
143. He invites as many technical people and academicians as he can for lunch; He
spends an hour a day trawling the internet; He wants to act as a beacon for his
version of the learning organization; He looks forward to learning about new
technological developments in his field.
(a) In order to becoming a beacon for his version of the learning organization,
he spent an hour a day trawling the internet so that he could find as many
technical people and academicians as he can for lunch to learn about new
technological developments in his field.
Revision Exercises 181
(b) Since he wants to become a beacon for his version of the learning
organization, he not only spends an hour a day trawling the internet to
learn about new technological developments in his field but he too makes
as many luncheon appointments as he can with technical people and
academicians.
(c) To act as a beacon for his version of the learning organization, he not
only spends an hour a day trawling the internet to learn about new
technological developments in his field, he also makes as many luncheon
appointments as he can with technical people and academicians.
(d) By inviting as many technical people and academicians as he can for
lunch after trawling the internet for one hour every single day, he wishes
to become a beacon for his version of the learning organization as he
looks forward to learn about new technological developments in his field.
(e) All of the above are correct
144. Our law is not a monolith; It is not handed to us by our founding fathers
as an edifice constructed brick-by-brick through an incremental series of
decisions; It is not based on the judgements that preceded it; It is in aggregate
a composite, well-integrated whole.
(a) Our law is not handed to us as a monolith constructed brick-by-brick
through an incremental series of decisions based on the judgements
preceding it but it is in aggregate a composite, well-integrated whole.
(b) Our law is well-integrated whole and not a monolith handed to us by
our founding fathers constructed brick-by-brick through an incremental
series of decisions based on the judgements that preceded it in aggregate.
(c) Our law is not a monolith handed to us by our founding fathers as an
edifice constructed brick-by-brick through an incremental series of
decision based on the judgements that preceded it but it is in aggregate a
composite, well-integrated whole.
(d) The monolithic edifice constructed brick-by-brick by our forefathers,
based on incremental series of decisions, is like our law which is in
aggregate a composite, well-integrated whole.
(e) None of the above is correct
145. Fans are eagerly awaiting the finale of Season 7 of Game of Thrones; It will
be aired on Monday in India; A south Delhi firm has begun work on the props
and costumes; These props and costumes will be used in the next and final
season of the popular television series.
(a) A South Delhi firm has began work on the props and costumes to be used
in the next and final season of the popular television series which will be
aired on Monday in India being waited eagerly by fans.
EBD_7275
182 Revision Exercises
(b) The finale of Season 7 of Game of Thrones to be aired on Monday in
India and props and costumes to be prepared by a South Delhi firm and
to be used in the next and final season of the popular television series are
being waited eagerly.
(c) The props and costumes to be used in the next and final season of the
popular television series are being prepared by a South Delhi firm, which
is being eagerly awaited by the fans to be aired on Monday in India.
(d) While fans are eagerly awaiting the finale of Season 7 of Game of
Thrones, which will be aired on Monday in India, a South Delhi firm has
begun work on the props and costumes to be used in the next and final
season of the popular television series.
(e) None of the above is correct
146. It has continued with its underlying humour; Film ‘Ant-Man was’ successful;
Marvel studios has made another movie whose lot of promotions were done
earlier; Another superhero – The Wasp is added in the upcoming sequel which
contains extra action.
(a) As the film ‘Ant-Man’ was successful, Marvel studios has come up
with another movie whose lot of promotions were done earlier which
introduces another superhero – The Wasp who will make sure that the
underlying humour is maintained along with extra action.
(b) Another film with another superhero – The Wasp with extra humour
and action has been made by Marvel studios which is a sequel to the
successful movie ‘Ant-Man’.
(c) Seeing the success of the film ‘Ant-Man’, Marvel studios has made
another epic promotional sequel that contains new superhero- The Wasp,
extra actions and underlying humour.
(d) Since the film ‘Ant-Man’ was successful, Marvel studios has made a
sequel to it by adding another superhero – The Wasp, extra action and its
underlying humour.
(e) None of the above is correct.
147. It was a hospital; The accident happened in West Delhi; Some patients were
injured and suffered minor burns; On Sunday, a major fire broke out.
(a) In a accident of fire at a hospital in West Delhi on Sunday, some patients
suffered minor burns and injuries.
(b) A major fire broke out on Sunday at a hospital in West Delhi in which
some patients suffered injuries and minor burns.
(c) Some patients, in an accident on Sunday that occurred in a hospital in the
Western part of Delhi suffered injuries and minor burns.
(d) Due to a fire broke out at a hospital in West Delhi on Sunday, some
patients suffered injuries and minor burns.
(e) None of the above is correct
Revision Exercises 183
148. She was weeping; They were returning home in a dark night; It was a late
night party; They saw an old lady sitting on the road side.
(a) A weeping old lady sitting on the road side in a dark night was seen by
them who were returning back to home from a late night party.
(b) As they were returning home in a dark night from a late night party, they
saw an old lady which was sitting on the road side and weeping.
(c) It was a dark night when they saw a weeping old lady sitting on the road
side while they were on their way back home from a late night party.
(d) An old weeping lady was seen by them who were on their way back to
home from a late night party.
(e) None of the above is correct
149. I hope to see him in the concert tonight; we were on the flight; I met him after
eight years, he was sitting next to me; He will perform in the concert.
(a) I met him after eight years on the flight as he was sitting next to me and
I look forward to seeing him perform in the concert tonight.
(b) I was on the flight after years where the person sitting next to me was him
and I hope to see him perform in tonight’s concert.
(c) I met him after eight years on the flight, he is a performer and he would
be performing in tonight’s concert where I wish to see him.
(d) After eight years, we met each other on the flight as we were fellow-
passengers; moreover, I look forward to seeing him perform in the
concert tonight.
(e) None of the above is correct
150. England lost to India; Indian batsmen played extremely well; It was a duck
soup for India; The total runs scored by them were only 102.
(a) England outplayed India as they scored only 102 runs and Indian batsmen
batted extremely well.
(b) India managed to win the match against England as the target was only
102 and Indian batsmen batted extremely well.
(c) It was an easy win against England as Indian batsmen played well enough
to achieve the small target of 102.
(d) India won the match against England easily because the total runs scored
by them were only 102 and Indian batsmen batted superbly.
(e) None of the above is correct
EBD_7275
184 Revision Exercises

HINTS & SOLUTIONS


1. (b) 26. (c) 51. (e) 76. (b) 101. (d) 126. (a)
2. (d) 27. (d) 52. (a) 77. (d) 102. (c) 127. (c)
3. (c) 28. (e) 53. (c) 78. (e) 103. (a) 128. (e)
4. (e) 29. (c) 54. (c) 79. (a) 104. (c) 129. (b)
5 (a) 30. (a) 55. (c) 80. (c) 105. (e) 130. (c)
6. (b) 31. (d) 56. (d) 81. (b) 106. (d) 131. (d)
7. (d) 32. (b) 57. (b) 82. (c) 107. (e) 132. (b)
8. (d) 33. (b) 58. (c) 83. (e) 108. (b) 133. (e)
9. (c) 34. (a) 59. (e) 84. (c) 109. (a) 134. (d)
10. (b) 35. (c) 60. (a) 85. (b) 110. (d) 135. (d)
11. (d) 36. (a) 61. (c) 86. (a) 111. (a) 136. (d)
12. (b) 37. (d) 62. (b) 87. (d) 112. (e) 137. (e)
13. (c) 38. (d) 63. (d) 88. (a) 113. (a) 138. (e)
14. (a) 39. (c) 64. (e) 89. (e) 114. (c) 139. (d)
15. (e) 40. (a) 65. (c) 90. (c) 115. (c) 140. (d)
16. (a) 41. (e) 66. (a) 91. (e) 116. (c) 141. (d)
17. (c) 42. (c) 67. (d) 92. (b) 117. (b) 142. (b)
18. (e) 43. (b) 68. (c) 93. (d) 118. (e) 143. (c)
19. (b) 44. (e) 69. (c) 94. (c) 119. (b) 144. (c)
20. (d) 45. (e) 70. (d) 95. (a) 120. (c) 145. (d)
21. (b) 46. (a) 71. (b) 96. (e) 121. (b) 146. (e)
22. (e) 47. (c) 72. (e) 97. (c) 122. (b) 147. (b)
23. (a) 48. (e) 73. (d) 98. (a) 123. (b) 148. (c)
24. (d) 49. (c) 74. (e) 99. (e) 124. (c) 149. (a)
25. (c) 50. (b) 75. (a) 100. (c) 125. (a) 150. (d)
Shortcuts in

QUANTIATIVE
APTITUDE
for Competitive Exams
EBD_7382
• Corporate Office : 45, 2nd Floor, Maharishi Dayanand Marg, Corner
Market, Malviya Nagar, New Delhi-110017
Tel. : 011-49842349/49842350

DISHA PUBLICATION
All Rights Reserved
© Copyright Publisher

No part of this publication may be reproduced in any form without prior permission of the
publisher. The author and the publisher do not take any legal responsibility for any errors
or misrepresentations that might have crept in. We have tried and made our best efforts to
provide accurate up-to-date information in this book.

For further information about books from DISHA,


Log on to www.dishapublication.com or email to info@dishapublication.com
Index
1. Number System & Simplification 1-12

2. HCF & LCM 13-24

3. Fractions and Decimals 25-32

4. Square and Square Roots, Cube and Cube 33-48


Roots, Indices and Surds

5. Fundamental Operations, Vbodmas Rule, Algebraic 49-62


Identities, Approximation and Simplification
(Including Basic Calculations)

6. Polynomials and Rational Expressions (Inducing 63-66


Factorization of Polynomials, Factor Theorem &
Remainder Theorem)

7. Linear Equation, Quadratic Equations & Inequalities 67-74


(Including Simultaneous Linear Equations)

8. Ratio, Proportion, Variation and Partnerships 75-86

9. Average 87-98

10. Percentage 99-108

11. Profit, Loss and Discount 109-122

12. Simple Interest & Compound Interest and Instalments 123-144

13. Sequence and Series 145-154

14. Allegation and Mixture 155-164


EBD_7382
15. Time, Work and Wages 165-182

16. Time, Speed and Distance 183-203

17. Clock and Calendar 204-210

18. Permutation & Combination and Probability 211-219

19. Geometry-I 220-225

20. Geometry-II 226-235

21. Coordinate Geometry 236-243

22. Mensuration Plane Figures 244-252

23. Mensuration - Solid Figures 253-259

24. Set Theory 260-265

25. Trigonometry 266-276

26. Logarithm 277-280


01 Number System &
Chapter Simplification

Shortcut Approach - 1
When two numbers are divided by a third number, leave the same remainder,
then the difference of these two numbers is always perfectly divisible by
third number.
1. 24345 and 33334 are divided by certain number of three digits and
the remainder is the same in both the cases. Find the divisor and
the remainder.
(a) 103, 6 (b) 809, 3 (c) 101, 4 (d) 109, 5
Sol. (c) Difference = 33334 – 24345 = 8989
Since, 8989 = 101 × 89
\ 101 is the required 3 digit divisor
On dividing any of the given numbers by 101, we get 4 as
remainder.

Shortcut Approach - 2
To find the greatest n digits number completely divisible by a divisor.
Find the remainder by dividing greatest n digits number by the divisor.
Required number = (Greatest n digits number) – (Remainder)
2. What is the largest number of 4 digits which is completely divisible
by 18?
(a) 9990 (b) 9984 (c) 9992 (d) None of these
Sol. (a) Greatest four digits number = 9999
Divisor = 18
Remainder = 9
So, required number = 9999 – 9 = 9990

18 9999
90 ( 555
99
90
99
90
9
EBD_7382
2 Number System & Simplification
3. What is the greatest odd number of 4-digits which is completely
divisible by 17?
(a) 9996 (b) 9979 (c) 9981 (d) 9991
Sol. (b) Greatest four number number = 9999
Divisor = 17
Remainder = 3
So, number = 9999 – 3 = 9996

17 9999
85 ( 588
149
136
139
136
3
But, 9996 is even number, so subtract
17 from 9996.
Difference = 9996 – 17 = 9979.
(Which is odd number)
Hence, the required number is 9979

Shortcut Approach - 3
To find the least n digits number completely divisible by a divisor. Find the
remainder by dividing least n digits number by the divisor.
Required number = (Least n digits number) + (Divisor – Remainder)
4 . What is the smallest number of 4-digits which is completely divisible
by 22?
(a) 1008 (b) 1010 (c) 1002 (d) 1012
Sol. (d) The least four digit number = 1000
Divisor = 22
So, required number
= 1000 + (22 – 10) = 1012
22 1000
88 ( 45
120
110
10

Shortcut Approach - 4
If sum and difference of two numbers are given, then
Product of the two numbers
(Sum + Difference) × (Sum – Difference)
=
4
Number System & Simplification 3
5. The product of two terms is 4640 and their difference is 22. Find
the sum of their reciprocals.
14 69 19 11
(a) (b) (c) (d)
2025 2320 3941 1347
Sol. (b) Let the numbers be x and y
\ Product = é ( Sum + Difference )(Sum - Difference ) ù
ê ú
ë 4 û

é ( Sum + 22 )(Sum - 22 ) ù
4640 = ê ú
ë 4 û

é ( Sum )2 - ( 22 )2 ù
4640 = ê ú
ê 4 ú
ë û
(Sum)2 = 4640 × 4 + (22)2
= 18560 + 484
Sum = (19044)1/2 = 138
1 1 x+y Sum 138 69
Now, + = = = =
x y xy Pr oduct 4640 2320

Shortcut Approach - 5
Sum of the digits of a given two digit number is S. When its digits are
interchange their places, the number decreased by D. Then,
æ Dö 1 æ Dö
Given number = 5 ç S + ÷ + ç S - ÷
è 9 ø 2 è 9ø
6. Sum of the digits of a given 2–digit number is 12. When its digits
interchange their places, the number decreases by 54. Find the
number.
(a) 93 (b) 84 (c) 75 (d) 66
Sol. (a) \ Given number
é Dù 1 é Dù
= 5 êS + ú + êS - ú
ë 9û 2ë 9û

é 54 ù 1 é 54 ù
= 5 ê12 + ú + ê12 - ú
ë 9 û 2ë 9û

1
= 5 ´ 18 + ´ 6 = 93
2
EBD_7382
4 Number System & Simplification

Shortcut Approach - 6
Sum of the digits of a given two digit number is S. When its digits
interchange their places, the number is increased by I. Then
æ Iö 1æ Iö
Given number = 5 çè S - ÷ø + çè S + ÷ø
9 2 9
7. Sum of the digits of a given 2-digit number is 13. When its digits
interchange their places, the number increases by 9. Find the given
number.
(a) 76 (b) 94 (c) 67 (d) 85
Sol. (c) \ Given number
æ I ö 1æ Iö
= 5ç S - ÷ + ç S + ÷
è 9 ø 2 è 9 ø

æ 9ö 1æ 9ö
= 5 ç13 - ÷ + ç13 + ÷
è 9ø 2è 9ø
1
= 5 ´12 + ´ 14
2
= 60 + 7 = 67

Shortcut Approach - 7
If difference between a two-digit number and the number obtained by
interchanging the digits is given, then difference of the two digits of the two
Difference in original and new number
digits number =
9
8. If the difference between a two-digit number and the number obtained
by interchanging digits is 36, then find the difference of the digits
of the original number.
(a) 1 (b) 2 (c) 3 (d) 4
Sol. (d) (\ Difference of the digits of the number)
æ Difference of originalö
çè and new number ÷ø
=
9
36
= =4
9
Number System & Simplification 5

Shortcut Approach - 8
If ratio of the sum and the difference of two numbers is a : b, then
a+b
Ratio of the two numbers =
a -b
9. Ratio of the sum and difference of two numbers is 9 : 1. Find the
ratio of these two numbers.
(a) 5 : 4 (b) 9 : 5 (c) 2 : 1 (d) 3 : 2
æa +bö
Sol. (a) \ Ratio of the two numbers = ç ÷
èa -bø
æ 9 + 1 ö 10
=ç ÷ = = 5:4
è 9 -1 ø 8

Shortcut Approach - 9
(i) (an + bn) is divisible by (a + b) when n is odd
(ii) (an – bn) is divisible by both (a + b) and (a – b) when n is even
(iii)(an – bn) is divisible by only (a – b) when n is odd
10. If (6767+ 67) is dividing by 68, the remainder is :
(a) 61 (b) 67 (c) 63 (d) 66
Sol. (d) \ (xn + yn) is divisible by
(x + y) when n is odd
So, ((6767 + 167) + 66)
(6767 + 167) is divisible by 68,
Hence remainder is 66.

Shortcut Approach - 10
When (xn + k) is divided by (x – 1),
(a) Remainder = 1 + k; if k < (x – 1)
(b) Remainder = 1 + (Remainder obtained when
k is divided by x – 1); if k > x – 1
11. Find the remainder on dividing (916 + 6) by 8.
(a) 5 (b) 7 (c) 2 (d) 3
Sol. (b) Here k = 6 and x – 1 = 8
\ k < (x – 1)
So, Remainder = 1 + k = 1 + 6 = 7

Shortcut Approach - 11
A number when divided by d1 and d2 successively, leaves the remainder r1
and r2 respectively. If the number is divided by d1 × d2, then
Remainder = d1 × r2 + r1
EBD_7382
6 Number System & Simplification
12. A certain number when successively divided by 3 and 5 leaves
remainder 1 and 2. What is the remainder if the same number be
divided by 15?
(a) 7 (b) 8 (c) 9 (d) 10
Sol. (a) \ Remainder = (d1 × r2 + r1)
= (3 × 2 + 1) = 7

Shortcut Approach - 12
To find the number of zeros at the end of a product, write the product as
prime factorisation like (in the prime factorise bases are different prime
numbers and powers are natural numbers)
Product = 2m × 5n × ......
Here m and n are natural numbers. Now you know that product of each
pair of 2 and 5 gives a zero (0), therefore
The product has m zeros, if m < n and
The product has n zeros, if m > n.
The product has m or n zeros, if m = n.
13. Number of zeroes at the end in the product 1 × 2 ×3 × 4 × ...... ×
25 is
(a) 5 (b) 6 (c) 10 (d) 25
Sol. (b) Let 1 × 2 × 3 × ... × 25 = 2x5yz where Z is the which is neither
divisible by (2) nor by (5)
5, 10, 15, 20, 25 are the multiplier of 5 where 25 has two multiples
of 5. (i.e. : 5 × 1 = 5, 5 × 2 = 10, 5 × 3 = 15, 5 × 4 = 20, 5 × 5 = 25) So
y = 6. It is clear that indices of 2 will be more than 5 means x >
6 means there will be maximum 6 pairs of 2 × 5. So their will be
6 zeros at the end of the given numbers.
14. If 2 × 4 × 6 × 8 × 10 × 12 × ........... × 60 will be divisible by 10 n, then
what can be the maximum value of n?
(a) 12 (b) 10 (c) 14 (d) 15
Sol. (c) It is similar previous Question
Here, total power on 5 will be 12 + 2 =14.

Shortcut Approach - 13
(a) Unit digit of any power of the numbers with unit digit 0, 1, 5 and 6 are
respectively 0, 1, 5 and 6 i.e.
(. . . . . . 0)n = (. . . . . . . 0)
(. . . . . . . 1)n = (. . . . . . . 1)
(. . . . . . . 5)n = (. . . . . . . 5)
(. . . . . . . 6)n = (. . . . . . . 6)
(b) If unit digit of any number is other than 0, 1, 5 and 6; then multiply its
unit digit by itself least number of times till you get the unit digit as
1, 5 or 6.
Number System & Simplification 7
Count the number of times you multiply the unit digit by itself.
Let's see
(2)4 = 16 (3)4 = 81
(4)2 = 16 (7)4 = 2401
4
(8) = 4096 (9)2 = 81
Therefore, if n is a natural number, then
Unit digit of (2)4n = 6 Unit digit of (3)4n = 1
Unit digit of (4)2n = 6 Unit digit of (7)4n = 1
Unit digit of (9)2n = 1
(c) Unit digit of a product a × b × c × ...... is the unit digit of the product
of the unit digit of a, b, c, .......
Using the facts given above in section (a), (b) and (c), you can find
the unit digit of any power of a number.
15. Find the unit digit of (624)50.
(a) 9 (b) 6 (c) 4 (d) 3
Sol. (b) Digit at unit place in 624 = 4
Now, (4)50 = (4)2 × 25
So, unit digit of (4)2 × 25 = 6
Hence, unit digit of (624)50 is 6

Shortcut Approach - 14
If N is a composite number and N = ap bq cr ....where a, b, c are different
prime numbers and p, q, r are natural numbers. Then
The number of divisors (factors) of N including 1 and the number N itself
= (p + 1) (q + 1) (r + 1) ........
16. Find the different divisors of 37800, excluding unity.
(a) 95 (b) 94 (c) 93 (d) 92
Sol. (c) 37800 = 2 × 3 × 5 × 7
3 3 2 1

The number of divisors = (p + 1)(q + 1)(r + 1)


= (3 + 1) × (3 + 1) × (2 + 1) × (1 + 1) = 96
Therefore number of divisor, excluding unity and the number
= 96 – 1 = 95.

Shortcut Approach - 15
If the sum of squares of two numbers is x and the square of their difference
is y, then the product of the two numbers
EBD_7382
8 Number System & Simplification

æ x - yö

è 2 ÷ø

Shortcut Approach - 16
Two different numbers when divided by the same divisor, leaves remainder
x and y respectively and when their sum is divided by the same divisor,
remainder is z. Then the divisor = x + y – z
17. If two numbers are each divided by the same divisor, the remainders
are respectively 3 and 4. If the sum of the numbers be divided by the
same divisor, the remainder is 2. The divisor is :
(a) 9 (b) 7 (c) 5 (d) 3
Sol. (c) \ Divisor = x + y – z
=3+4–2=5

Shortcut Approach - 17
If N be a composite number such that
N = (x)a (y)b (z)c .......; where x, y, z, ....... are different prime numbers, then
sum of divisors or factors of
x a +1 - 1 y b +1 - 1 z c +1 - 1
z= ´ ´ ´ .......
x -1 y -1 z -1
18. Find the sum of the factors of 90.
(a) 214 (b) 234 (c) 224 (d) 226
Sol. (b) (90) = [2 × (3)2 × (5)]
Hence, sum of divisor

z=
( x( a +1) - 1)
´
(y (b +1)
-1) ´ (z (c +1)
-1) ´ ....
(x - 1) ( y - 1) ( z - 1)
22 - 1 33 - 1 52 - 1 4 - 1 27 - 1 24
z= ´ ´ Þ ´ ´
2 - 1 3 -1 5 - 1 1 2 4
= 3 × 13 × 6
= 234

Shortcut Approach - 18
If the product of the numbers is x and the sum of these two numbers is y,
then the numbers are given by
æ y + y2 - 4x ö
The larger number = ç ÷ and
çè 2 ÷ø
Number System & Simplification 9

æ y - y 2 - 4x ö
The smaller number = ç ÷
çè 2 ÷ø
19. If the sum of two numbers is 78 and their product is 1352, then find
the larger number.
(a) 52 (b) 26 (c) 25 (d) 62
Sol. (a) \ Larger number
ì y + y2 - 4x ü ì 78 + (78)2 - 4 ´1352 ü
ï ï ï ï
=í ý= í ý
2 ïî 2 ïþ
îï þï
78 + 26 104
= = = 52
2 2

Shortcut Approach - 19
If the product of two numbers is x and the difference between these two
numbers is y, then the

y 2 + 4x + y
Large number = and
2

y 2 + 4x - y
Smaller number =
2
20. If the product of two numbers is 600 and their difference is 10, the
find the smaller number.
(a) 30 (b) 20 (c) 10 (d) 40
Sol. (b) Sum of two number
é y2 + 4x + y y2 + 4x - y ù
ê + ú
=ê 2 2 ú
ë û

= 42 + 4 x = (10)2 + 4 ´ 600 = 50
EBD_7382
10 Number System & Simplification

Exercise
1. The digit in the unit’s place 7. The last digit of the
of the number represented by expression 4 + 92 + 43 + 94 + 45
(795 –358) is: + 96 + ............. + 499 + 9100 is :
(a) 0 (b) 4 (a) 0
(c) 6 (d) 7 (b) 3
2. Find the remainder when (c) 5
(d) None of these
.66 8. Find the number of zeros at
6..
6 6 the end of the product 15 ×
66 (100 times) divided
25 × 35 × .............. × 325
by 10? (a) 25 (b) 30
(a) 6 (b) 2 (c) 32 (d) 35
(c) 4 (d) 8 9. Which of the following will not
3. If N is sum of factorials of all completely divide (341 + 782)?
the prime numbers less than (a) 4 (b) 52
100, find the last two digits (c) 17 (d) 26
of N 10. Find the digit at the unit’s
(a) 48 (b) 58 place of
(c) 68 (d) 78 (377)59 × (793)87 × (578)129 ×
32 (99)99
4. Find the last digit of 3232
(a) 1 (b) 2
(a) 4 (b) 8
(c) 6 (d) 2 (c) 7 (d) 9
5. Find the number of zeros at the 11. A number which when divided
end of the product 100! + 200! by 32 leaves a remainder of 29.
(a) 24 If this number is divided by 8
(b) 25 the remainder will be
(c) 49 (a) 0 (b) 1
(d) None of these (c) 5 (d) 3
6. Two numbers, when divided 12. If two numbers when divided
by 17, leaves remainder 13 by a certain divisor give
and 11 respectively. If the remainder 35 and 30
sum of those two numbers is respectively and when their
divided by 17, the remainder sum is divided by the same
divisor, the remainder is 20,
will be :
then the divisor is
(a) 13 (b) 11
(a) 40 (b) 45
(c) 7 (d) 4
(c) 50 (d) 55
Number System & Simplification 11

Hints & Solution


1. (b) Use Short Approach -13 40 + 8 + 1 = 49 trailing zeros in 200!
2. (a) 6n (where n is a natural 20 + 4 = 24 trailing zeros in 100!.
number) will always leaves
the remainder 6 when If we add two numbers with 'm'
divide by 10. trailing zeros and 'n' trailing zeros,
3. (c) Since last two digits of all the resultant number will end with 'm'
the factorials more than 9 zeros if (m < n) or 'n' zeros if (m > n).
is 00 hence required last Here, 24 < 49
two digits is given by Hence 24 trailing zeros.
2! + 3! + 5! + 7! = 2 + 6 + 120 6. (c) Use Short Approach -16
+ 5040 = 5168,
4. (c) Use Short Approach -13 7. (a) 4ODD UNIT DIGIT
¾¾¾¾¾¾
® 4
5. (a) 100! + 200! 4EVEN ¾¾
® 6
9ODD ¾¾
® 9
100! 200!
EVEN
9 ¾¾
® 1
5 100 5 200
5 20 5 40
4 5 8
1
2 3 4 5 6 99 100
4 + 9 + 4 + 9 + 4 + 9 + ..........+ 4 + 9
UNIT DIGIT

4 + 1 4 + 1 4 + 1 4 + 1
5 5 5 5
Sum of unit digits = (5 + 5 + 5 +
........... 50 terms) Any number of the form
Hence, unit digit is 0. 'A n + Bn ' is always divisible
8. (d) Use Short Approach -12 by 'A + B', if n is an odd
number.
9. (c) 341 + 782 = 341 + ( 7 2 )
41

So, the given expression


41 41
3 + 49 will be completely divisible
= by 52 and its factors, since
17 is not a factor of 52 .
3 + 49 = 52
EBD_7382
12 Number System & Simplification
10. (b) Use Short Approach -13 8(Q2 – 4Q1) = 29 – R
11. (c) Let this number be N then
29 - R
N = 32 × Q1 + 29 ...(1) or(Q2 – 4Q1) =
8
Again N = 8 × Q2 + R…(2)
From (1) and (2) Since Q1, Q2, R are integers
32Q1 + 29 = 8Q2 + R also Q2 – 4Q1 is an integer.
(where R is the remainder) Therefore 29 – R must be
8Q2 – 32Q1 = 29 – R divisible by 8.
12. (b) Use Short Approach -16
02 HCF & LCM
Chapter

Shortcut Approach - 1
For Finding HCF or GCD
To find the HCF of any number of given numbers, first find the difference
between two nearest given numbers. Then find all factors (or divisors) of
this difference. Highest factor which divides all the given numbers is the
HCF.
1. Find the HCF of 132, 204 and 228.
(a) 24 (b) 12 (c) 36 (d) 48
Sol. (b) Difference of two nearest numbers
= 228 – 204 = 24
All factors of 24 = 1, 2, 3, 4, 6, 8, 12, 24
Among all the factors of 24, only 12 is the highest factor which
divides all of the three numbers 132, 204 and 228 completely
\ HCF of 132, 204 and 228 = 12
2. Find the LCM of 8, 15, 24 and 72.
(a) 240 (b) 720 (c) 180 (d) 360
Sol. (d) Here, 8 and 15 are co-prime to each other.
Now, prime factorisation of (8 × 15) = 2 × 2 × 2 × 3 × 5
Prime factorisation of 24 = 2 × 2 × 2 × 3
Prime factorisation of 72 = 2 × 2 × 2 × 3 × 3
Here, all the prime factors of 24 are present in the prime
factorisation of (8 × 15).
But one prime factor 3 among the factors of 72 is not present in
that prime factorisation of (8 × 15).
\ Required LCM = (8 × 15) × 3 = 360

Shortcut Approach - 2
26
Using idea of co-prime, you can find the LCM by the following shortcut
method:
LCM of 9, 10, 15 and 36 can be written directly as 9 × 10 × 2.
The logical thinking that behind it is as follows:
Step (i): If you can see a set of 2 or more co-prime numbers in the set of
numbers of which you are finding the LCM, write them down by multiply them.
EBD_7382
14 HCF & LCM
In the above situation, since we see that 9 and 10 are co-prime to each
other, we start off writing the LCM by writing 9 × 10 as the first step.
Step (ii): For each of the other numbers, consider what prime factor(s) of it
is/are not present in the product (if factorised into primes) taken in step 1.
In case you see some prime factors of each of the other given numbers
separately are not present in the product (if factorised into primes) taken in
step 1, such prime factors will be multiplied in the product taken in step 1.
Prime factorisation of 9 × 10 = 3 × 3 × 2 × 5
Prime factorisation of 15 = 3 × 5
Prime factorisation of 36 = 2 × 2 × 3 × 3
Here we see that both prime factors of 15 are present in the prime
factorisation of 9 × 10 but one prime factor 2 of 36 is not present in the
product taken in step 1. So to find the LCM of 9, 10, 15 and 36; we multiply
the product taken in step 1 by 2.
Thus required product = 9 × 10 × 2 = 180
Step (iii): The work of finding the HCF may sometimes be simplified by the
following devices:
(a) Any obvious factor which is common to both numbers may be
removed before the rule is applied. Care should however be taken to
multiply this factor into the HCF of the quotients.
(b) If one of the numbers has a prime factor not contained in the other, it
may be rejected.
(c) At any stage of the work, any factor of the divisor not contained in the
dividend may be rejected. This is because any factor which divides
only one of the two cannot be a portion of the required HCF.
3. Find the HCF of 1365, 1560 and 1755.
(a) 195 (b) 13 (c) 15 (d) 65
Sol. (a) Difference of two nearest numbers = 1560 – 1365
= 195
All factors of 195 = 1, 3, 5, 13, 15, 39, 65, 195
Among all the factors, 195 is the highest factor that divides
1560, 1365 and 1755 completely.
\ Required HCF = 195

Shortcut Approach - 3
26
The greatest number that will exactly divide x, y and z = HCF of x, y and z
4. Find the greatest possible length which can be used to measure
exactly the lengths 7m, 3m 85 cm, 12m 95cm.
(a) 35 cm (b) 45 cm (c) 1 m 15 cm (d) 90 cm
Sol. (a) 700 = 7 × 5 × 4
2

385 = 5 × 7 ×11
1295 = 5 × 7 × 37
HCF = 5 × 7 = 35
\ Required length = 35 cm
HCF & LCM 15
5. Which is the largest number which can exactly divide each of the
numbers 264, 312 and 456?
(a) 26 (b) 24 (c) 28 (d) 32
Sol. (b) 264 = 23 × 3 × 11
312 = 23 × 3 × 13
456 = 23 × 3 × 19
HCF = 23 × 3 = 24
\ Largest number = 24

Shortcut Approach - 4
26
Least number which is exactly divisible by x, y and z = LCM of x, y and z
6. How many numbers are there between 4000 and 6000 which are
exactly divisible by 32, 40, 48 and 60?
(a) 1 (b) 2 (c) 3 (d) 4
Sol. (d) 32 = 25
40 = 23 × 5
48 = 24 × 3
60 = 22 ×3 × 5
\ LCM = 25 × 3 × 5 = 480
The numbers divisible by 480 between 4000 and 6000 are 4320,
4800, 5280 and 5760.
\ Required number of numbers = 4
7. Find the least number which is exactly divisible by 12, 15, 90, 108,
135 and 150.
(a) 1500 (b) 3000 (c) 2700 (d) 2250
Sol. (c) 12 = 22 × 3
15 = 3 × 5
90 = 2 × 32 × 5
108 = 22 × 33
135 = 33 × 5
150 = 52 × 3 × 2
\ LCM = 22 × 33 × 52 = 2700

Shortcut Approach - 5
26
The greatest number that will divide x, y and z leaving remainders a, b and
c respectively = HCF of (x – a), (y – b) and (z – c)
8. Find the greatest number that divides 130, 305 and 245 leaving
remainders 6, 9 and 17 respectively?
(a) 4 (b) 5 (c) 14 (d) 24
Sol. (a) Required number = HCF of (130 – 6), (305 – 9) and
(245 – 17)
= HCF of 124, 296 and 228
EBD_7382
16 HCF & LCM
124 = 22 × 31
296 = 23 × 37
228 = 22 × 3 × 19
\ Required number (HCF) = 2 × 2 = 4
9. The numbers 400, 536 and 646; when divided by a number N, give
the remainders of 22, 23 and 25 respectively. Find the greatest such
number N.
(a) 40 (b) 45 (c) 27 (d) 22
Sol. (c) Required number, N = HCF of (400 – 22), (536 – 23)
and (646 – 25)
= HCF of 378, 513 and 621
378 = 33 × 2 × 7
513 = 33 × 19
621 = 33 × 23
\ N (HCF) = 33 = 27

Shortcut Approach - 6
26
Greatest number that will divide x, y, z leaving the same remainder 'r' in
each case
= HCF of (x – r), (y – r) and (z – r)
10. What is the greatest that divides 13850 and 17030 leaves a
remainder 17?
(a) 477 (b) 159 (c) 107 (d) 87
Sol. (b) Greatest number = HCF of (13850 – 17)
and (17030 – 17)
= HCF of 13833 and 17013

13833 17013 1
–13833
3180 13833 4
–12720
1113 3180 2
–2226
954 1113 1
–954
159 954 6
–954
×
\ Greatest number = 159
HCF & LCM 17
11. What will be the greatest number that divides 1356, 1868 and 2764
leaving 12 as remainder in each case?
(a) 64 (b) 124 (c) 156 (d) 260
Sol. (a) Greatest number = HCF of (1356 – 12), (1868 – 12)
and (2764 – 12)
= HCF of 1344, 1856 and 2752 = 64
Hence, required number = 64

Shortcut Approach - 7
26
Least number which when divided by x, y and z leaves the same remainder
'r' in each case
= (LCM of x, y and z) + r
12. The least number which when divided by 12, 16 and 18 leaves 5 as
remainder in each case. Find the number.
(a) 139 (b) 149 (c) 159 (d) 169
Sol. (b) Here, x = 12, y = 16, z = 18, r = 5
\ Requir ed least number = (LCM of x, y an d z)
+ r = (LCM of 12, 16, 18) + 5
2 12, 16, 18
2 6, 8, 9
3 3, 4, 9
1, 4, 3
\ LCM = 2 × 2 × 3 × 3 × 4 = 144
Hence, required least number = 144 + 5 = 149
13. What is the least number which when diminished by 7, is divisible
by each one of 21, 28, 36 and 45?
(a) 1267 (b) 1157
(c) 1077 (d) 1347
Sol. (a) 2 21, 28, 36, 45
2 21, 14, 18, 45
3 21, 7, 9, 45
3 7, 7, 3, 15
7 7, 7, 1, 5
5 1, 1, 1, 5
1, 1, 1, 1
\ LCM = 2 × 2 × 3 × 3 × 7 × 5 = 1260
Hence, required number = 1260 + 7 = 1267
EBD_7382
18 HCF & LCM

Shortcut Approach - 8
26
Greatest number that will divide x, y and z leaving the same remainder in
each case (but remainder is not given)
= HCF of | x – y |, | y – z | and | z – x |.
14. Find the largest number which divides 1305, 4665 and 6905 leaving
same remainder in each case. Also, find the common remainder.
(a) 1210, 185 (b) 1120, 185
(c) 1005, 175 (d) 1100, 175
Sol. (b) Here, x = 1305, y = 4665, z = 6905
Now, |x – y| = |1305 – 4665| = 3360
|y – z| = |4665 – 6905| = 2240
|z – x| = |6905 – 1305| = 5600
\ Greatest number
= HCF of |x – y|, |y – z| and |z – x|
= HCF of 3360, 2240 and 5600 = 1120
Again, on dividing 1305 by 1120, we get
remainder = 185.
On dividing 4665 by 1120, we get remainder = 185.
On dividing 6905 by 1120, we get remainder = 185.
Hence, largest number = 1120
and common remainder = 185

Shortcut Approach - 9
26
Least number which when divided by x, y and z leaves the remainder a, b
and c respectively
= (LCM of x, y and z) – k, where k = x – a = y – b = z – c.
15. What is the smallest positive integer which when divided by 4, 5, 8, 9
leaves remainder 3, 4, 7, 8 respectively?
(a) 119 (b) 319 (c) 359 (d) 719
Sol. (c) Here, K = 4 – 3 = 5 – 4 = 8 – 7 = 9 – 8 = 1
\ Required number = (LCM of 4, 5, 8 and 9) – K
= 360 – 1 = 359
16. What is the sum of the digits the least number which, when divided by 52,
leaves 33 as remainder; when divided by 78, leaves 59 and when divided by
117, leaves 98 as reaminder?
(a) 17 (b) 18 (c) 19 (d) 21
Sol. (a) Here, K = 52 – 33 = 78 – 59 = 117 – 98 = 19
\ Required number = (LCM of 52, 78, 117) – 19
= 468 – 19 = 449
\ Sum of the digits of the least number
HCF & LCM 19

Shortcut Approach - 10
26
To find the n-digits greatest number divisible by x, y and z; follow the
steps :
Step (i) : Find the LCM of x, y and z.
Let LCM of x, y and z = L.
Step (ii) :Find the remainder by dividing n-digits greatest number by L.
Let the remainder = R.
Step (iii) : Required number = (n-digits greatest number) – R.
17. Find the greatest number of five digits which is completely divisible
by 12, 16, 20, 24 and 28.
(a) 99120 (b) 99908
(c) 99899 (d) 99446
Sol. (a) Greatest number of 5-digits = 99999
LCM of 12, 16, 20, 24, 28, = 1680
On dividing 99999 by 1680, we get 879 as remainder

1680 99999 59
8400
15999
15120
879
Hence, required number = 99999 – 879 = 99120
18. Find the greatest number less than 900, which is divisible by 8, 12
and 28.
(a) 880 (b) 800 (c) 860 (d) 840
Sol. (d) LCM of 8, 12, 28 = 168
On dividing 900 by 168, we get 60 as remainder.
\ Required number = 900 – 60 = 840
168 900 5
840
60

Shortcut Approach - 11
To find the n-digits greatest number, which when divided by x, y and z
leaves the same remainder K in each case; follow the steps:
Step (i) : Find the LCM of x, y and z.
Let LCM x, y and z = L.
EBD_7382
20 HCF & LCM
Step (ii) : Find the remainder by dividing n-digits greatest number by L.
Let the remainder = R.
Step (iii) : Required number = (n-digits greatest number) – R + K
19. Find the greatest number of 4 digits which when divided by 8, 9, 12
and 15 leaves 6 remainder in each case.
(a) 9980 (b) 9726 (c) 9920 (d) 9626
Sol. (b) Greatest number of 4-digits = 9999
LCM of 8, 9, 12, 15 = 360
On dividing 9999 by 360, we get 279 as remainder.
\ Required number = 9999 – 279 + 6 = 9726
20. Find the largest number of 5 digits which when divided by 3, 5, 8, 12
leaves the same remainder 2 in each case.
(a) 99722 (b) 99958 (c) 99960 (d) 99962
Sol. (d) Largest number of 5-digits = 99999
LCM of 3, 5, 8 and 12 = 120
On dividing 99999 by 120, we get 39 as remainder
\ Required number = 99999 – 39 + 2 = 99962

Shortcut Approach - 12
26
To find the smallest n-digits number divisible by x, y and z; follow the
steps :
Step (i) : Find the LCM of x, y and z.
Let LCM of x, y and z = L.
Step (ii) : Find the remainder by dividing n-digits smallest number by L.
Let the remainder = R.
Step (iii) : Required number = n-digits smallest number + (L – R)
21. Find that smallest 6-digit number which when divided by 9, 12, 21
and 25 leaves no remainder
(a) 100005 (b) 100225 (c) 100800 (d) 100100
Sol. (c) Smallest 6-digits number = 100000
LCM of 9, 12, 21, 25 = 6300
On dividing 100000 by 6300, we get 5500 as
remainder.
\ Required number
= 100000 + (6300 – 5500) = 100800

Shortcut Approach - 13
26
To find smallest n-digits number which, when divided by x, y and z leaves
K in each case; follow the steps :
Step (i) : Find the LCM of x, y and z.
Let LCM of x, y and z = L.
HCF & LCM 21
Step (ii) : Find the remainder by dividng n-digits
smallest number by L.
Let the remainder = R.
Step (iii) : Required n-digits smallest number
= n-digits smallest number + (L – R) + K.
22. Find the least possible 5-digit number, which when divided by 10,
12, 16 and 18 leaves remainder 27.
(a) 10107 (b) 10100 (c) 10027 (d) 10507
Sol. (a) LCM of 10, 12, 16, 18 = 720
Smallest 5 – digit number = 10000
On dividing 10000 by 720, we get 640 as remainder.
\ Requir ed n umber = 10000 + (720 – 640) + 27
= 10107

Shortcut Approach --14


26
Least number which when divided by x1, x2 and x3 leaves the remainder
a1, a2 and a3 respectively
= (LCM of x1, x2, x3) – (x1 – a1) or (x2 – a2) or (x3 – a3)
Note that here, x1 – a1 = x2 – a2 = x3 – a3
23. Find that smallest number which when divided by 16, 24 and 30
leaves remainder as 6, 14 and 20 respectively.
(a) 116 (b) 224 (c) 300 (d) 230
Sol. (d) Here, 16 – 6 = 24 – 14 = 30 – 20 = 10
\ LCM of 16, 24 and 30 = 240
Hence, required number = 240 – 10 = 230

Shortcut Approach - 15
26
If product of two numbers and their HCF are given, then to find all possible
pairs of numbers follow the steps :
Product
Step- (i): Find the value of .
(HCF)2
Step-(ii): Find the all possible pairs of co-prime
numbers, whose product is equal to the value
obtained in step-I.
Step-(iii): Multiply each number of the possible pairs
obtained in step-II by HCF.
The new pairs obtained in this step is the required pairs.
24. Product of two numbers is 1440 and their HCF is 12. How many
pairs of such numbers are possible?
(a) 2 (b) 3 (c) 1 (d) 4
EBD_7382
22 HCF & LCM

Product 1440
Sol. (a) Step – (i) : = = 10
( HCF) 2
(12 )2
Step – (ii) : All possible pairs of co–prime numbers
= (1, 10) ; (2, 5)
\ Only 2 pairs can be possible.
25. The product of two whole numbers is 1500 and their HCF is 10. Find
the numbers.
(a) (70, 80) (b) (50, 38)
(c) (10, 150) (d) (75, 20)
Product 1500
Sol. (c) Step (i) : = = 15
( HCF) 2
(10)2
Step (ii) : All possible pairs of co–prime numbers
= (1, 15) ; (5, 3)
Step (iii) : (1 × 10, 15 × 10) and (5 × 10, 3 × 10)
HCF & LCM 23

Exercise
1. A hall is 13 metres 53 cm long 7. How many pairs of factors of
and 8 metres 61cm broad is to number 7200 have HCF 20?
be paved with minimum (a) 10 (b) 44
number of square tiles. The (c) 53 (d) 18
number of tiles required is: 8. Find that number by which
(a) 123 (b) 77 when 1050, 1250 and 1650 are
(c) 99 (d) 57 divided respectively leaves
2. There are 4 numbers. The 43, 31 and 7 as remainders.
H.C.F. of each pair is 3 and (a) 47 (b) 51
the L.C.M. of all the 4 numbers (c) 53 (d) 57
is 116. What is the product of 9. The least number, which
4 numbers? when divided by 2, 3, 4, 5 and
(a) 9782 (b) 9396 6, leaves in each case, a
(c) 9224 (d) 9100 remainder 1, but when
3. What will be the least number divided by 7 leaves no
which when doubled will be remainder. The number is
excactly divisible by 12,18,21 (a) 121 (b) 181
and 30? (c) 241 (d) 301
(a) 196 (b) 630 10. One pendulum ticks 57 times
in 58 seconds and another 608
(c) 1260 (d) 2520 times in 609 seconds. If they
4. Four bells ring at an interval started simultaneously, find
of 30 min, 1 hour, 1 hour 30 the time after which they will
min and 1 hour 45 min. All tick together.
bells started ring together at
12 noon. At what they will 211 1217
ring again? (a) s (b) s
19 19
(a) 12 O' clock mid – night
(b) 3 O' clock next morning 1218 1018
(c) 6 O' clock next morning (c) s (d) s
19 19
(d) 9 O' clock next morning 11. If the product of the HCF and
5. The largest number which the LCM of 3 natural numbers
will divide the numbers 104, p, q, r, equals pqr, then p, q, r
221 and 377 leaving the same must be :
remainder in each case is : (a) such that (p, q, r) = 1
(a) 56 (b) 13 (b) prime number
(c) 39 (c) odd number
(d) It does not exist (d) such that (p, q) = (p, r) = 1
6. The sum of two numbers is 12. Find the largest number of 5
462 and their highest common digits which when divided by
factor is 22. What is the 3, 5, 8, 12 leaves the same
maximum number of pairs that remainder 2 in each case.
satisfy these conditions ? [SSC CHSL 2016]
(a) 1 (b) 3 (a) 99722 (b) 99958
(c) 5 (d) 6 (c) 99960 (d) 99962
EBD_7382
24 HCF & LCM

Hints & Solution


1. (b) Use Short Approach -2 q + r) + 2(pq + pr + qr) +
2. (b) Q Product of ‘n’ 4pqr here p = 3, q = 2 and r
numbers = 1 hence required
= (HCF for each pair)n × number of such pairs are
LCM of ‘n’ numbers 1 + 6 + 22 + 24 = 53
= (3)4 × 116 = 9396 8. (c) Use short approach -5
3. (b) Use Short Approach -1 9. (d) LCM (2, 3, 4, 5, 6) = 60
4. (d) The L.C. M of 30, 60, 90 \ Required number is
and 105 min = 1260 min = of the form 60 k + 1
21 hours means bells Least value of k for
will ring next morning at which 60k + 1 is divisible
9' O Clock. by 7 is k= 5
5. (c) Use short approach -8 \ Required number = 60
6. (d) There are 6 such pairs :
× 5 + 1= 301
(22, 440), (44, 418), (88, 374), (110, 352) 10. (c) Time gap between two
(176, 286), (220, 242) consecutive ticks
7. (c) Sin ce HCF of two 58 609
sec. and sec.
numbers are 20 hence 57 608
numbers must be in the \ Required time = LCM
form of 20a and 20b where
58 609
a and b are co-prime to of and
57 608
each other. 20a and 20b are
factors of 7200 hence a LCM of 58 and 609 1218
= = sec
and b must be a factor of HCF of 57 and 608 19
7200/20 = 360 = 233251 11. (a) It is possible only when
Now condition is that a, there is no common
and b are co-prime to each factor among p, q and r.
other and they are factors Hence, the HCF of p, q,
of 7200, similar type of r=1
questions we have solve and the LCM of p, q, r
in chapter 2 (factor theory) = p.q.r
concept 7 and there we \ (HCF × LCM) of p, q, r =
have seen a formula total 1 × p. q. r = pqr
number of pairs is 1 + (p + 12. (d) Use Short Approach -2
03 Fractions and
Chapter Decimals

Shortcut Approach - 1
26
a c
To compare two fractions and , follow as
b d
a c
b d

ad bc

a c
If ad > bc, then >
b d
a c
If ad < bc, then <
b d
15 8
1. If a = , b = then which is true
16 9
(a) a > b (b) a < b
(c) a = b (d) can’t be determine
Sol. (a) 15 ´ 9 = 135, 16 ´ 8 = 128
Here 135 > 128
\ a>b

Shortcut Approach - 2
26
If in any given sequence of fractions, the difference in consecutive
numerators is equal or greater than the difference in consecutive
denominators, then the fraction having larger numerator is larger and the
fraction having smaller numerator is smaller.
This is not applicable for compound fractions i.e. improper fractions
2. Which of the following fractions is smallest?
5 7 9 11
(a) (b) (c) (d)
12 13 14 15
EBD_7382
26 Fractions and Decimals

æ Difference in ö æ Difference in ö
Sol. (a) ç consecutive ÷ > ç consecutive ÷
ç ÷ ç ÷
è numerators ø è denominatorsø
or 2 > 1
Hence the fraction having smaller numeratoer is the smallest
5
i.e. .
12

Shortcut Approach - 3
26
If in any given sequence of fractions, numerator is increasing by a definite
value and denominator is also increasing by a different definite value but
increase in the value of numerator is less than increase in the value of
denominator, then
increase in numerator
(i) If > First fraction, the fraction with
increase in denominator
greatest numerator is greatest.
increase in numerator
(ii) If < First fraction, the fraction with
increase in denominator
greater numerator is least.
2 5 8 11
3. Which of the fraction is largest among , , , ?
5 11 17 23
2 11 5 8
(a) (b) (c) (d)
5 23 11 17
Increase in numerator
Sol. (b) Here, first fraction
Increase in denominator
3 2
or >
6 5
Hence the fraction with greatest numerator is the greatest i.e.
11
.
23

Shortcut Approach - 4
26
x1 x2 x1 + x2
A fraction between any two given fractions y and y is y + y .
1 2 1 2
Using this you can find any number of fractions between any two given
fractions.
Fractions and Decimals 27

3 2
4. Which of the following fractions lie between and ?
5 3
2 1 1 5
(a) (b) (c) (d)
5 3 15 8
3 2 3+ 2 5
Sol. (d) Fraction lie between and = =
5 3 5+ 3 8

Shortcut Approach - 5
26
x1 x2
If a man spends y part of his salary on food, y part of his salary on
1 2

x3
clothing, y part of his salary on education and so on. After these
3
expenditures,
é æ x1 x2 x3 öù
Balance salary = (Salary) × ê1 - ç + + + ......÷ ú
ë è y1 y2 y3 øû
1
5. A person gave ` 500 to his eldest son. Then the gave part of his
10
total wealth to his second son and the amount of money received by
his third son equal to the total amount received by his first and
second son. How much money the person had?

(a) 1250 (b) 1750 (c) 1500 (d) 1000


Sol. (a) Let the total money be ` x.
500
Fraction for eldest son’s share =
x
æ 500 1 ö
Fraction for third son’s share = ç + ÷
è x 10 ø
Now remaining money
é æx x öù
= Total money × ê1 - ç 1 + 2 + ... ÷ú
êë è y1 y2 øúû

é æ 1 æ 500 1 ö ö ù
Þ 500 = x × ê1 - ç + ç + ÷ ÷ú
ë è 10 è x 10 ø ø û
EBD_7382
28 Fractions and Decimals

é 2 x + 5000 ù
Þ 500 = x ´ ê1 -
ë 10 x úû
é10 x - 2 x - 5000 ù
Þ 500 = x ´ ê ú
ë 10 x û
Þ 5000 = 8x – 5000
Þ 10000 = 8x Þ x = `1250

Shortcut Approach - 6
26
For any single activity,
(i) Part done + Part remained = 1
(ii) Total amount
Amount spent Amount remained
Total amount = =
Balance part Part remained

1 1
6. part of a pencil is black and part of the remaining is white. If
8 2
1
the remaining part is blue and length of this blue part is 3 cm,
2
then find the length of the pencil.
(a) 6 cm (b) 7 cm (c) 8 cm (d) 9 cm
Amount remained
Sol. (c) Length of the pencil =
Part remained
1 1
3 3
2 = 2
=
é 1 1 æ 1 öù æ1 7 ö
1 - ê + ç1 - ÷ ú 1 - ç + ÷
ë 8 2 è 8 øû è 8 16 ø
1
3
2 = 7 = 7 = 8cm
=
9 æ7ö 7
1- 2ç ÷
16 è 16 ø 8

Shortcut Approach - 7
26
The denominator of a fraction (or rational number) is 'D' more than its
numerator. If the numerator is increased by x and the denominator is
decreased by y, we get P, then the fraction (or rational number) is given by
x - P ( D - y)
.
x + ( yP - D)
Fractions and Decimals 29
7. The numerator of a fraction is 4 less than its denominator. If the numerator
is decreased by 2 and the denominator is increased by 1, then the
denominator becomes eight times the numerator, then find the
fraction
3 4 2 3
(a) (b) (c) (d)
7 8 7 8
1
Sol. (a) Here, D = 4, x = (–2), y = (–1) and P =
8
x - P(D - y )
\ Required fraction =
x + ( yP - D)
1 5
-2 - [4 - (-1)] -2 -
= 8 = 8
é æ 1 ö ù -2 - 33
-2 + ê(-1) ç ÷ - 4 ú
ë è8ø û 8

-21
8 = -21 = 3
=
-49 -49 7
8
EBD_7382
30 Fractions and Decimals

Exercise
1. Find the value of (SSC CGL 2011)
1 1 1 1 1 2 11
+ + + + ..... +
2 ´ 3 3´ 4 4 ´ 5 5 ´ 6 9 ´10 (a) (b)
5 23
3 2
(a) (b) 5 8
2 5 (c) (d)
2 3 11 17
(c) (d) 5. If the numerator of a fraction
3 5
2. The ascending order of is increased by 200% and the
denominator of the fraction
3 5 11 17 is increased by 150%, the
, , and is
4 12 18 24 resultant fraction is 9/35.
17 11 5 3 What is the original fraction?
(a) , , ,
24 18 12 4 3
(a)
3 17 11 5 10
(b) , , ,
4 24 18 12 2
(b)
5 11 17 3 15
(c) , , ,
12 18 24 4 3
11 17 5 3 (c)
, , , 16
(d)
18 24 12 4 (d) None of there
3. Which of the following 6. What is value of the
fractions are in ascending expression
order?
11 7 4 ?
(a) , , 7 7 7 7 .. .. .. .. .. .¥
52 26 13 (a) 49 (b) 7 (c)
7 4 11 1 (d) 0
(b) , , ,
26 13 52
7. Evaluate 6+ 6+ 6+ --
4 7 11
(c) , , , -----µ
13 26 52 (a) –2 (b) 3
7 11 4 (c) 6 (d) – 6
(d) , , , 8. The product of 2 numbers is
26 52 13
4. Which of the fraction is 1575 and their quotient is 9/7.
largest among Then the sum of the numbers
is–
2 5 8 11
, , , ? (a) 74 (b) 78
5 11 17 23
(c) 80 (d) 90
Fractions and Decimals 31

1 2 3 4 11. For all real values of x and y


9. 99 + 99 + +99 + 99 if ( 4x – y) is divisible by 3,
7 7 7 7
then (4x2 + 7 xy – 2y2) is
5 6 always divisible by
+99 + 99 is equal to
7 7 (a) 12 (b) 3
(a) 603 (b) 600 (c) –11 (d) 1
(c) 598 (d) 597 12. Find the value of
10. A man reads 3/8 of a book on 1 1 1
a day and 4/5 of the + +
1+ 2 2+ 3 3+ 4
remainder on the second
day. If the number of pages 1
+.... + is
still unread are 40, then how 99 + 100
many pages did the book (a) 8 (b) 6
contain? (c) 10 (d) 9
(a) 300 (b) 500
(c) 320 (d) 350

Hints & Solution


1. (b) Given expression 150y 100y + 150y
= y+ =
100 100
æ1 1ö æ1 1ö æ 1 1ö æ1 1ö
= ç - ÷+ç - ÷+ç - ÷+ç - ÷+ 250y
è 2 3ø è 3 4ø è 4 5ø è 5 6ø =
100
æ1 1 ö æ1 1 ö 4 2
..... + ç - ÷ = ç - ÷= = Then, according to the
è 9 10 ø è 2 10 ø 10 5
question,
2. (c) Use Shortcut Approach-2
3. (d) Let the original fraction 300x /100 9
=
x 250y /100 35
be .
y 300x 9
Þ =
Numerator is increased 250y 35
by 200%.
\ Numerator = x + 200% of x x 9 250 3
\ = ´ =
200x 100x + 200x y 35 300 14
=x+ = = 300 4. (a) Use Shortcut Approach-2
100 100
x / 100 5. (b) Let
And denominator of the
fraction is increased by x= 7 7 7 ........ ¥
150%.
Denominator
EBD_7382
32 Fractions and Decimals

Þ x = 7x =
4 5
´ x= x
1
Squaring both sides 5 8 2
and solving for x, we Now, remaining pages
get x2-7x = 0 or x = 0, 7.
æ 3x x ö
= x-ç +
But x can’t be zero, so x è 8 2 ÷ø
= 7.
6. (b) Use Shortcut Approach-3
7. (b) \ Two consecutive æ 7x ö - 7x ö x
æ 8x
= ç x -= ç ÷ ÷=
è è
8 ø 8 ø 8
factor of 6 = 3 × 2
According to the question,
\ 6+ 6+ 6+ ------- x
= 40 Þ x = 40 × 8 Þ x =
µ = +3, –2 8
8. (c) Use Shortcut Approach-6 320
9. (d) Required sum 11. (b) 4x2 + 7 xy – 2y2 = ( 4x –
y) ( x + 2y)
æ 1 6ö æ 2 5ö (4x – y ) is divisible by 3
= ç 99 + 99 ÷ + ç 99 + 99 ÷
è 7 7 ø è 7 7ø 4x 2 + 7 xy – 2y2 is also
divisible by 3
æ 3 4ö 12. (d)
+ ç 99 + 99 ÷
è 7 7 ø 1 1
=
= 199 × 3 = 597 1+ 2 2+ 1
10. (c) Let the number of pages 2- 1
+ = 2- 1
be x. 2- 1
Number of pages read
1 1 1
3x + +
in first day = 1+ 2 2+ 3 3+ 4
8
1
Remaining pages ...
99 + 100
3x 5
=x- = x = 2 - 1+ 3- 2 +
8 8
Number of pages read .... + 100 - 99
on second day
= 100 - 1 = 10 - 1 = 9
04 Square and Square Roots,
Cube and Cube Roots,
Chapter
Indices and Surds

Shortcut Approach - 1
26
Easier method to find the square root of 3 or 4 digits number:
Step-(i) : Square root of 3 or 4 digits number is always a 2 digits number.
Ten's digit of the square root is the largest digit whose square is less than
or equal to the number form from the digit(s) remains after leaving the two
digits from right from the given number.
Step-(ii) :Guess the unit digit of the square root on the basis of unit digit of
the given number as explained in the following examples.
(i) 3481 = ?
(5)2 < 34 < 36 Þ Ten's digit of 3481=5
Since unit digit of 3481 = 1
Therefore, unti digit of 3481 is 1 or 9.
Thus 3481 is 51 or 59.
Now 51 is close to 50 and 59 is close to 60.
Now (50)2 = 2500 and (60)2 = 3600.
But 3481 is closer to 3600.
∴ 3481=59
(ii) 324 = ?
(1)2 < 3 < (2)2 Þ Ten's digit of 324 is 1.
Since unit digit of 324 = 4.
Therefore, unit digit of 324 is 2 or 8.
Thus 324 = 12 or 18.
Now, 12 is close to 10 and 18 is close to 20.
Now (10)2 = 100 and (20)2 = 400.
But 324 is closer to 400.
\ 324 =18
1. Find the square root of 2304
(a) 48 (b) 42
(c) 47 (d) None of these
EBD_7382
34 Square and Square Roots, Cube and Cube Roots......

Sol. (a) (4)2 < 23 < (5)2 Þ Ten’s digit of 2304 = 4


Since unit digit of 2304 = 4
Therefore, unit digit of 2304 is 2 or 8
Thus, 2304 is 42 or 48
Now 42 is close to 40 and 48 is close to 50
Now, (40)2 = 1600
(50)2 = 2500
\ 2304 = 48
2. Find the square root of 5184
(a) 78 (b) 71
(c) 72 (d) None of these
Sol. (c) (7)2 < 51 < (8)2 Þ Ten’s digit of 5184 = 7
Since unit digit of 5184 = 4
Therefore, unit digit of 5184 is 2 or 8
Thus, 5184 is 72 or 78
Now, 72 is close to 70 and 78 is close to 80
Now, (70)2 = 4900,
(80)2 = 6400,
\ 5184 = 72

Shortcut Approach - 2
Approximate Square Root of a number :
Divide the given number by a divisor around approximate square root of
the number. Find the average of the divisor and quotient. This average is
the approximate square root of the given number.
(i) 850 = ?
Take 30 as divisor.
850 ÷ 30 = 28.33
Average of 30 and 28.33 = 29.17.
Hence, approximate square root of 850 = 29.17.
(ii) 1171 = ?
Take 35 as divisor.
1171 ÷ 35 = 33.46
35 + 33.46
Average of 35 and 33.46 = = 34.23
2
Hence approximate square root of 1171 = 34.23.
Square and Square Roots, Cube and Cube Roots...... 35
Note that, we can take divisor as any other number nearby the square
root of 1171. If we take 30 as divisor, then
1171 ÷ 30 = 39.03.
30 + 39.03
Average of 30 and 39.03 = = 34.52
2
Hence approximate square root of 1171 = 34.52.
3. Find the approximate square root of 624.
(a) 23.98 (b) 24.98
(c) 25.98 (d) None of these.
Sol. (b) Consider 25 as divisor
624 ¸ 25 = 24.96
Average of 25 and 24.96

25 + 24.96 49.96
= = = 24.98
2 2
Hence, approximate square root of 624 is = 24.98
4. Find the Approximate square root of 1550.
(a) 37. 375 (b) 38. 375
(c) 39. 375 (d) None of these.
Sol. (c) Consider 40 as divisor
1550 ¸ 40 = 38.75
Average of 40 and 38.75
40 + 38.75 78.75
= = = 39.375
2 2
Hence, approximate square root of 1550 is 39.375

Shortcut Approach - 3
If in a given number, number of digits is n and if n is even, then square
n
root of that number will have digits and if n is odd, then number of
2
n +1
digits will be .
2
5. How many digits are there in square root of 6594624.
(a) 7 (b) 6 (c) 5 (d) 4
EBD_7382
36 Square and Square Roots, Cube and Cube Roots......
Sol. (d) Since number of digits in 6594624 is 7 and 7 is odd number
Hence, number of digits in square root
7 +1
= = 4 digits
2
6. How many digits are there in square root of 20820969.
(a) 4 (b) 5 (c) 6 (d) 8
Sol. (a) Since number of digits in 20820969 is 8 and 8 is even number
Hence, number of digits in square root
n 8
= = = 4 digits
2 2

Shortcut Approach - 3 (a)


7. Find the greatest 8 digit number which is a perfect square.
(a) 99970001 (b) 99980001
(c) 99990001 (d) None of these

Sol. (b) Greatest 8 digits number = 99999999

9999
9 99 99 99 99
9 81
189 1899
9 17 01
1989 19899
9 17 901
19989 199899
9 179901
19998 19998

Clearly greatest 8-digits number which is a perfect square will


be 9999, less than 99999999. Therefore, required number
= 99999999 – 19998 = 99980001
8. Find the greatest 9 digit number which is a perfect square.
(a) 999970884 (b) 999960884
(c) 999950884 (d) None of these
Square and Square Roots, Cube and Cube Roots...... 37
Sol. (c) Greatest 9 digits number = 999999999
31622
3 999999999
3 9
61 99
1 61
626 3899
6 3756
6322 14399
2 12644
63242 175599
126484
49115
Clearly greatest 9-digits number which is a perfect square will
be 31622 less than 999999999.
Therefore, required number
= 999999999 – 49115 = 999950884

Shortcut Approach - 4
To calculate cube of a number the following two formulae are used
(i) (a + b)3 = a3 + 3ab (a – b) + b3
(ii) (a – b)3 = a3 – 3ab (a – b) – b3
You can easily understand it by the following illustrations.
9. Find the cube of 19.
(a) 6559 (b) 6659 (c) 6759 (d) 6859
Sol. (d) (19)3 = (10 + 9)3
= (10)3 + 3 × 10 × 9 (10 + 9) + (9)3
= 1000 + 270 × (19) + 729
= 1729 + 5130 = 6859
10. Find the cube of 28.
(a) 23952 (b) 22952
(c) 21952 (d) None of these
Sol. (c) (28)3 = (30 – 2)3
= (30)3 – 3 × 30 × 2 × (30 – 2) – (2)3
= 27000 – 180 × (28) – 8
= 27000 – 5040 – 8 = 21952

Shortcut Approach - 5
Cube Root By Successive Subtraction:
If we subtract 1, 7, 19, 37, 61, 91 ............ from a given number, till we get zero
then the number of subtractions will give the cube root of the given number.
EBD_7382
38 Square and Square Roots, Cube and Cube Roots......
The numbers 1, 7, 19, 37 ....... are obtained by putting m = 1, 2, 3, 4.......... in
[1 + m × (m –1) × 3]
Example : 64 – 1 = 63, 63 – 7 = 56, 56 – 19 = 37,
37 – 37 = 0.
Here number of subtractions till we get zero is 4. Therefore 3
64 = 4
Cube root of any number upto 3-digits is a single digit number. We can
easily find the cube root of any perfect cube number upto 3 digits. Using
the relation between unit digit of a perfect cube number and unit digit of its
cube root. Cube root of any 4, 5 or 6 digits number is a 2-digits number.
To find the cube root of any 4, 5 or 6 digits number follow the steps :
(i) Put a slash before the last three digits from right of given perfect
cube number. 103823 can represent as 103 / 823, 1331 can represent
as 1 / 331.
(ii) The number on right side of slash will give us the unit place digit of
cube root and that on left hand side will give ten's digit of cube root.
(iii) The unit place digit of cube root can be found by looking at unit
place digit of the given number and the relation between unit digit of
a perfect cube number and unit digit of its cube root . For example,
cube root of 287496 will have 6 at units place in its cube root because
the number itself has 6 in its unit place.
(iv) To find the ten's place digit of the cube root, we take the number
which lies to the left of the slash. Now we find two perfect cubes
between which the left hand part i.e., number before slash, lies. For
example in 287/496, left part 287 lies between 216 (cube of 6) and 343
(cube of 7).
(v) Now out of the numbers 6 and 7 obtained above we take the smaller
number as ten's place digit of the cube root. Thus cube root of
287496, which is a perfect cube, is 66.
Thus by simple observation we will be able to find cube root of any
given perfect cube number upto 6 digits.
11. Find the cube root of 729, by successive substraction method.
(a) 9 (b) 11
(c) 13 (d) None of these.
Sol.. (a) 729 – 1 = 728, 728 – 7 = 721,
721 – 19 = 702, 702 – 37 = 665,
665 – 61 = 604, 604 – 91 = 513,
513 – 127 = 386, 386 – 169 = 217,
217 – 217 = 0
Here number of subtractions fill we get zero is 9. Therefore
3 729 = 9
Square and Square Roots, Cube and Cube Roots...... 39
12. Find the cube root of 343, by successive substraction method.
(a) 9 (b) 8
(c) 7 (d) None of these.
Sol. (c) 343 – 1 = 342, 342 – 7 = 335,
335 – 19 = 316, 316 – 37 = 279,
279 – 61 = 218, 218 – 91 = 127,
127 – 127 = 0
Here number of subtraction fill we get zero is 7. Therefore
3 343 = 7

Shortcut Approach - 6
Finding cube root by Approximation
Divide the given number by a divisor around approximate cube root of the
number. Divide the quotient by the same divisor again. Take average of
the two divisors and final quotient. This is approximate cube root of the
number.
13. Find the cube root of 3400.
(a) 15.037 (b) 14.037
(c) 16.037 (d) 13.037
Sol. (a) Let, take 15 as divisor,
3400 ¸ 15 = 226.666
226.666 ¸ 15 = 15.111
So, average of 15, 15, 15.111 is 15.037
14. Find the cube root of 1750.
(a) 15.05 (b) 14.05 (c) 13.05 (d) 12.05
Sol. (d) Let take 12 as divisor,
1750 ¸ 12 = 145.833
145.833 ¸ 12 = 12.152
So, average of 12, 12, 12.152 is 12.05

Shortcut Approach - 7
If x is a positive real number and a, b and c are real number; then
a b c
æ xb ö æ x c ö æ x a ö
(i) ç c ÷ gç a ÷ gç b ÷ = 1
èx ø èx ø èx ø

a +b b+ c c+ a
æ xa ö æ xb ö æ xc ö
(ii) ç b÷ gç c ÷ gç a ÷ =1
èx ø èx ø èx ø
EBD_7382
40 Square and Square Roots, Cube and Cube Roots......

( a 2 + b 2 + ab ) (b 2 + c 2 + bc )
æ xa ö æ xb ö
(iii) ç b÷ gç ÷
èx ø è xc ø

(c 2 + a 2 + ca )
æ xc ö
gç ÷ =1
è xa ø
15. Find the value of the following:
2 3 6
æ 133 ö æ 136 ö æ 132 ö
ç 6÷ .ç 2 ÷ .ç 3 ÷
è 13 ø è 13 ø è 13 ø
(a) 11 (b) 111
(c) 1 (d) None of thses
æ (13)6 ö æ (13)18 ö æ (13)12 ö
Sol. (c) ç ÷ .ç ÷.ç ÷= 1
ç (13)12 ÷ ç (13)6 ÷ ç (13)18 ÷
è øè øè ø
16. Find the value of the following.
7+8 8+9 9+7
æ (121)7 ö æ (121)8 ö æ (121)9 ö
ç ÷ .ç ÷ .ç ÷
çè (121)8 ÷ø è (121)9 ø è (121)7 ø

(a) 121 (b) 1


(c) 11 (d) None of these

(121)7´15 (121)8´17 (121)9´16


Sol. (b) . . =1
(121)8´15 (121)9´17 (121)7´16
17. Find the value of the following.

(32 + 92 + 3 ´ 9) (92 + (27)2 + 9 ´ 27)


æ ( 49) 3 ö æ (49) ö 9
ç ÷ .ç ÷
çè (49)9 ÷ø è (49)27 ø

((27)2 +(3)2 + 27 ´ 3)
æ (49)27 ö
.ç ÷
è (49)3 ø
(a) 1 (b) 49
(c) (49)2 (d) None of these.
Square and Square Roots, Cube and Cube Roots...... 41

( 49 )3.(3 ) ( 49)9(92 +( 27 )2 +9´27 )


2 +9 2 +3´9

Sol. (a) .
9.( 32 +9 2 +3´9 )
( 49 )27( 9 +( 27) +9´27 )
2 2
( 49 )

( 49 )27(( 27) )
2 +32 + 27´3

. =1
3( ( 27 ) +32 + 27´3)
2
( 49 )

Shortcut Approach - 8
If the sum of a number and its square is x, then the number is given by

é 1 + 4 x - 1ù
ê ú.
ë 2 û
18. If the sum of a number and its square is 110, what is the number?

(a) 10 (b) 21
(c) 11 (d) None of these.

1 + 4 ´ 110 - 1
Sol. (a) The required number =
2

441 - 1 21 - 1
= Þ = 10
2 2
19. If the sum of a number and its square is 240, what is the number?

(a) 13 (b) 14
(c) 15 (d) None of these
Sol. (c) The required number

1 + 4 ´ 240 - 1 961 - 1
= =
2 2

31 - 1
= = 15
2
EBD_7382
42 Square and Square Roots, Cube and Cube Roots......

Shortcut Approach - 9
The difference between the squares of two consecutive numbers is the
sum of the two consecutive numbers and it is always an odd number.
20. Find the value of
(47)2 + (31)2 + (21)2 – (46)2 – (30)2– (20)2
(a) 190 (b) 195 (c) 205 (d) 215
Sol. (b) (47) + (31) + (21) – (46) – (30)2 – (20)2
2 2 2 2

= ((47)2 – (46)2) + ((31)2 – (30)2)


+ ((21)2 – (20)2)
= (47 + 46) + (31 + 30) + (21 + 20)
Þ 93 + 61 + 41 = 195
21. Find the value of
(91)2 + (81)2 + (71)2 – (90)2 – (80)2– (70)2
(a) 480 (b) 485 (c) 481 (d) 483
Sol. (d) (91)2 + (81)2 + (71)2 – (90)2 – (80)2 – (70)2
= ((91)2 – (90)2) + ((81)2 – (80)2) + ((71)2 – (70)2)
= (91 + 90) + (81 + 80) + (71 + 70)
= 181 + 161 + 141 = 483

Shortcut Approach - 10
A number is divided by a certain number N1 and gives a remainder R. If
the same number is divided by another number N2, where N1 > N2 and N1
is divisible by N2, then the new remainder is obtained by dividing R by
N2.
22. A number when divided by 357, gives a remainder 39. What
remainder would be obtained by dividing the same number by 17.

(a) 6 (b) 3 (c) 5 (d) 8


Sol. (c) \ Required number = 5

17 ) 39 (2
34
5
Square and Square Roots, Cube and Cube Roots...... 43
23. A number when divided by 899, gives a remainder 63. What
remainder would be obtained by dividing the same number by 29.
(a) 0 (b) 3 (c) 5 (d) 8
Sol. (a) \ Required number = 5

29 ) 63 (2
58
5

Shortcut Approach - 11
If the sum of two numbers be x and difference is y, then the difference of
their squares is xy.
24. The sum of two number be 100 and their difference is 37. The
difference of their square is:
(a) 2700 (b) 3700 (c) 3900 (d) 2900
Sol. (b) The required number
= xy = 100 × 37 = 3700
25. The sum of two number is 291 and their difference is 89, the
difference of their square is
(a) 25799 (b) 25699 (c) 25599 (d) 25899
Sol. (d) The required number
= xy = 291 × 89 = 25899

Shortcut Approach - 12
If the difference between the squares of two consecutive numbers is x,
æ x - 1ö æ x + 1ö
then the numbers are çè ÷ø and çè ÷.
2 2 ø
26. The difference between the squares of two consecutive number is
31, find the numbers.
(a) 15, 16 (b) 14, 15
(c) 13, 14 (d) None of these
Sol. (a) The required number

æ x - 1ö æ x + 1ö
=ç ÷ and ç
è 2 ø è 2 ÷ø
EBD_7382
44 Square and Square Roots, Cube and Cube Roots......

31 - 1 31 + 1
= and
2 2
= 15 and 16
27. The difference between the squares of two consecutive number 201,
find the numbers.
(a) 101, 102 (b) 102, 103
(c) 100, 101 (d) None of these
Sol. (c) The required number

æ 201 - 1 ö æ 201 + 1 ö
=ç ÷ and ç ÷ = 100 and 101
è 2 ø è 2 ø

Shortcut Approach - 13
If the two consecutive numbers are x and y, then the difference of their
squares is given by (x + y).
28. Two consecutive numbers are 171 and 172, then the difference of
their squares.
(a) 345 (b) 343
(c) 353 (d) None of these
Sol. (b) The required number = x + y
= 171 + 172 = 343
29. Two consecutive numbers are 1178 and 1179 then the difference of
their squares.
(a) 2357 (b) 2457
(c) 2557 (d) None of these
Sol. (a) The required number = x + y
= 1178 + 1179 = 2357

Shortcut Approach - 14
If the product of the two numbers is x and the sum of
their squares is y, then
(i) the sum of the two numbers is given by y + 2x and

(ii) the difference of the two numbers is given by y - 2x .


Square and Square Roots, Cube and Cube Roots...... 45
30. The product of two numbers is 3976. The sum of their squares is
8177. Find the difference of the two numbers.
(a) 17 (b) 18 (c) 16 (d) 15
Sol. (d) As given that x = 3976, y = 8177
The required difference = y - 2x

= 8177 - 2 ´ 3976
= 225 = 15
31. The product of two numbers is 5429. The sum of their squares is
11642. Find the sum of the two numbers.
(a) 151 (b) 152 (c) 150 (d) 153
Sol. (c) As given that x = 5429, y = 11642
The required sum = y + 2x

= 11642 + 2 ´ 5429
= 22500 = 150
EBD_7382
46 Square and Square Roots, Cube and Cube Roots......

Exercise
1. Find the value of x if (a) 0 (b) 1
..
.¥ (c) x
x
x
x a 2 +b2 +c2
x =9 (d) x
1 1 1 1
(a) 3 (b) 3 4 7. If x 3 + y 3 = z 3 , then {(x +
1 4 y – z)3 + 27 xyz} will be equal
(c) 36 (d) 39 to
2. What number must be added (a) –1 (b) 1
to the expression 16a2 – 12a (c) 0 (d) 27
to make it a perfect square? 8. If P = 124, then
P ( P 2 + 3P + 3) + 1 = ?
(a) 9/4 (b) 11/2 3
(c) 13/2 (d) 16
3. Which one among 21/2, 31/3, (a) 5 (b) 7
(c) 123 (d) 125
41/4 , 61/6 and 121/12 is the
largest? 9. If a = 2 + 3 , then what is the
(a) 21/2 (b) 31/3 value of (a2 + a–2)?
(c) 41/4 (d) 61/6 (a) 12 (b) 14
4. Arrange the following in (c) 16 (d) 18
descending order. 10. If p, q, r be integers such that
p2 = q2. r then :
3 4 6 12 (a) p is an even number
3, 4, 6, 12
(b) q is an even number
(a) 4
4 > 12 12 > 6 6 > 3 3 (c) r is an even number
(d) r is a perfect square
(b) 12
12 > 6 6 > 4 4 > 3 3 11. What is the least number
which must be subtracted
(c) 3
3 > 4 4 > 6 6 > 12 12 from 369 to make it a perfect
(d) 4 4 > 6 6 > 3 3 > 12 12 cube?
(a) 8 (b) 26
5. The largest number among (c) 2 (d) 25
7 - 5, 5 - 3, 9 - 7 , 11 - 9 is 12. Simplify:
(a) 7- 5 éïì ïü ù
(b) 5- 3 êí 4 3 6 2572 ý ú
êîï þï ú
(c) 9- 7 ê ú
ê ìï üï ú
(d) 11 - 9 ê ´ í 2 3 4 948 ýú
6. The value of éë x ( b - c ) ùû
b+c ëê îï þï ûú
(a) 75 (b) 45
( c+ a ) ( a +b ) (c) 1/5 (d) 15
éë x ( c-a ) ùû éë x ( a - b ) ùû is
Square and Square Roots, Cube and Cube Roots...... 47

Hints & Solution


1. (d) From the given 5. (b) If b – a = d – c = f – e
information we can say that .................then
putting smaller value in
( x)
2 4
9
= 9 = 32 hence x = 39 or x = 39
b - a , d - c, f - e
2. (a) 16a2 – 12a = (4a)2 –
we get the largest difference.
2(4a)(3/2)
\ The number is (3/2)2 So, 5 - 3 here will be the
= (9/4). largest
3. (b) In this question it is
adv[isable to raise all 2
Trick : 7- 5=
the numbers to the 7+ 5
power of 12, so the
numbers become, 2
11 - 9 = etc .
1/ 2 12 1/ 3 12 1/ 4 12, 1/ 6 12 11 + 9
(2 ) , (3 ) ,(4 ) (6 )
Q Numerator is same, so
6 4 3 2 that number will be largest
or 2 ,3 , 4 , 6
or 64, 81, 64, 36 whose denominator is the
So, 31/3 is the largest. smallest.
6. (b) Use Short Approach -7
4. (c) 3
3 4
4 6
6 12
12 7. (c) If a + b + c = 0, then a3 +
¯ ¯ ¯ ¯ b3 + c3 = 3abc
1 1 1 1 After putting
33 44 66 12 12 1 1 1
LCM of (3, 4, 6, 12) = 12. a= x3 ,b = y 3 ,c = -z 3
1
´12 1 1 1
33 = 34 = 81 x + y – z = – 3x 3 y 3 z 3
1
1 ´12 or, (x + y – z)3 = – 27 xyz
4 Þ 44 = 43 = 64
4 or, (x + y – z)3 + 27 xyz = 0

P ( P 2 + 3P + 3) + 1
1 3
1 ´12 8. (d)
6 Þ 66 = 62 = 36
6
3
1 = P3 + 3P2 + 3P + 1
´12
12 Þ 1212 = 121 = 12 3 3
12 = ( P + 1) = P + 1
81 > 64 > 36 > 12 Q P = 124 Þ P + 1 = 125
31/3 > 41/4 > 61/6 > 121/12.
EBD_7382
48 Square and Square Roots, Cube and Cube Roots......

9. (b) Given that, a = 2 + 3 11. (b) Go through option. If


you subtract 2, 8 or 25 from
1 369, then you will not obtain
Then =2- 3 [by
a a perfect cube number. So (b)
conjugate property] is the correct answer.
Now, we have, Alternatively : If you write
2 down the nearest perfect
æ 1ö cube such that k £ 369, where
a 2 + a -2 = ç a + ÷ - 2
è aø k is a perfect cube, then
73 = 343, which is just below
= (2 + 3 + 2 - 3) - 2
2
369.
So 369 – 343 = 26
2
= ( 4) - 2 = 16 - 2 = 14 12. (b)
2 ´ 72
10. (d) Options (a), (b) and (c)
are irrelevent. 4 3 6 2572 = 5 4´ 3´ 6 = 52 = 25

as (25)2 = (5)2 × 25 and


2 ´ 48
or (27)2 = (9)2 × 9
2 3 4 948 = 3 4 ´3´ 2 = 34 = 81
or (6)2 = (4)2 × 9
or (6)2 = (3)2 × 4 etc. Hence required value is
81 ´ 25 = 2025 = 45
Fundamental Operations, Vbodmas Rule,
05 Algebraic Identities, Approximation and
Simplification (Including Basic
Chapter Calculations)

Shortcut Approach - 1
26
Application of the formula :
(a + b)2 = a2 + 2ab + b2
(or)
(a + b) = a2 + b2 + 2ab
2

1. 16 × 16 + 128 + 4 × 4 = ?
(a) 20 (b) 100 (c) 400 (d) 500
Sol. (c) Put a = 16 and b = 4, then the
given expression becomes
16 × 16 + 128 + 4 × 4 = (a)2 + 2 × a × b + (b)2
= (a + b)2 = (16 + 4)2 = 400
2. If the sum of two numbers is 3 and the sum of their squares is 12,
then their product is equal to:
3 2
(a) (b)
2 3
3 2
(c) - (d) -
2 3
Sol. (c) Here, x + y = 3 and x2 + y2 = 12
Now (x + y)2 = (3)2 Þ x2 + y2 + 2xy = 9
-3
Þ 12 + 2xy = 9 Þ xy =
2

Shortcut Approach - 2
26
Application of the fomrula :
(a – b)2 = a2 – 2ab + b2
(or)
(a – b)2 = a2 + b2 – 2ab
3. The sum of two numbers is 42 and their product is 437. Then the
absolute difference between the numbers is:
(a) 3 (b) 4 (c) 5 (d) 7
EBD_7382
50 Fundamental Operations, VBODMAS Rule......
Sol. (b) x + y = 42 and xy = 437
Now, (x – y)2 = x2 + y2 – 2xy = x2 + y2 + 2xy – 4xy
= (x + y)2 – 4xy = (42)2 – 4 (437) = 16
Þ x–y=4
4. The product of two numbers is 120. The sum of their squares is
289. The difference of these two numbers is
(a) 9 (b) 7 (c) 8 (d) 6
Sol. (b) xy = 120 and x + y = 289
2 2

\ (x – y)2 = x2 + y2 – 2xy = 289 – 2 (120) = 49


Þ x–y=7
5. Find the value of

0.5 ´ 0.5 + 0.1 ´ 0.1 - 0.2 ´ 0.5


0.4
(a) 0.2 (b) 0.3 (c) 0.4 (d) 0.6

0.5 ´ 0.5 + 0.1 ´ 0.1 - 2 ´ 0.1 ´ 0.5


Sol. (c)
0.5 - 0.1
Substitute a = 0.5 and b = 0.1
2
a 2 + b2 - 2ab ( a - b )
Þ = = a - b = 0.4
a -b a -b

Shortcut Approach - 3
26
Application of the formula :
a2 – b2 = (a + b) (a – b)

é (0.1) 2 - (0.01)2 ù
6. ê ú + 1 is equal to
ë 0.0001 û
(a) 1010 (b) 110 (c) 101 (d) 100
é ( 0.1)2 - ( 0.01) 2 ù
Sol. (d) ê + 1ú
ê 0.0001 ú
ë û

é ( 0.1 + 0.01)( 0.1 - 0.01) ù é 0.11´ 0.09 ù


Þê + 1ú = ê + 1ú
ë 0.0001 û ë 0.0001 û
= [99 + 1] = 100
Fundamental Operations, VBODMAS Rule...... 51
7. The product of two consecutive odd numbers is 4899. What is the
smaller number?
(a) 67 (b) 69 (c) 71 (d) 73
Sol. (b) Let the numbers be (x – 1) and (x + 1)
\ (x – 1)(x + 1) = 4899
Þ x2 – 1 = 4899
Þ x = 4900 = 70
\ The smaller number = x – 1 = 69

9982 - 9972 - 45
8. =?
982 - 972
(a) 1995 (b) 195 (c) 95 (d) 10
( 998 + 997 )( 998 - 997 ) - 45
Sol. (d)
( 98 + 97 )( 98 - 97 )
1995 - 45 1950
= = = 10
195 195

Shortcut Approach - 4
26
Application of the formula :
(a + b)2 + (a – b)2 = 2(a2 + b2)
(835 + 378)2 + (835 - 378)2
9. Solve:
835 ´ 835 + 378 ´ 378
(a) 1 (b) 2 (c) 3 (d) 4

(835 + 378)2 + (835 - 378)2


Sol. (b) Þ
( 835 ´ 835 + 378 ´ 378)

=
( 2
2 ( 835 ) + ( 378 )
2
) =2
((835) + ( 378 ) )
2 2

Shortcut Approach - 5
26
Application of the formula:
(a + b)2 – (a – b)2 = 4ab
EBD_7382
52 Fundamental Operations, VBODMAS Rule......

(999 + 588)2 - (999 - 588)2


10. Solve:
999 ´ 588
(a) 4 (b) 3 (c) 2 (d) 1
( 999 + 588)2 - ( 999 - 588)2
Sol. (a) Þ
999 ´ 588
4 ´ 999 ´ 588
= =4
999 ´ 588
2 2
æ 4 3ö æ 4 3ö
11. çè + ÷ø - çè - ÷ø = ?
3 4 3 4
(a) 1 (b) 2 (c) 3 (d) 4
Sol. (d) The given expression is
æ 4 öæ 3 ö
(a + b)2 – (a – b)2 = 4ab = 4 ç ÷ç ÷ = 4
è 3 øè 4 ø
Shortcut Approach - 6
26
Application of the formula :
(a + b)3 = a3 + 3a2b + 3ab2 + b3
(a + b)3 = a3 + b3 + 3ab (a + b)
1
12. If xy (x + y) = 1, then the value of 3 3
- x3 - y3 .
x y
(a) 3 (b) – 3 (c) 1 (d) – 1
1
Sol. (a) xy (x + y) = 1 Þ x + y =
xy
1 1
Þ (x + y)3 = Þ x3 + y3 + 3xy (x + y) = 3 3
3 3 x y
x y
1
Þ = – x3 – y3 = 3xy (x + y) = 3
3 3
x y

Shortcut Approach - 7
26
Application of the formula :
(a – b)3 = a3 – 3a2b + 3ab2 – b3
or
(a – b)3 = a3 – b3 – 3ab (a – b)
13. If x – y = 1, then x3 – y3 – 3(xy – 2) is equal to
(a) 7 (b) 6 (c) 10 (d) 4
Fundamental Operations, VBODMAS Rule...... 53
Sol. (a) As given expression is
Þ x–y=1
Þ (x – y)3 = 13
Þ x3 – y3 – 3 (xy)(x – y) = 1
Þ x3 – y3 – 3 xy = 1
Þ x3 – y3 – 3 xy + 6 = 1 + 6 = 7
14. If p – 2q = 4, then the value of p3 – 8q3 – 24pq – 64 is
(a) 0 (b) 3 (c) – 1 (d) 2
Sol. (a) As given expression is
Þ p – 2q = 4
Þ (p – 2q)3 = (4)3
Þ p3 – (2q)3 – 3p(2q) (p – 2q) = (4)3
Þ p3 – 8q3 – 24pq = 64 (Q p – 2q = 4)
Then the value of
p3 – 8q3 – 24pq – 64 = 0

Shortcut Approach - 8
26
Application of the formula :
a3 + b3 = (a + b) (a2 – ab + b2)
a b
15. If + = 1, then the value of a3 + b3 is
b a
(a) 0 (b) 1 (c) 2 (d) 3
a b
Sol. (a) + =1
b a
Þ a2 + b2 = ab
Þ (a2 + b2 – ab) = 0 ...(i)
\ a3 + b3 = (a + b)(a2 + b2 – ab) = 0 (Q From eq (i))
16. The value of

( ) {( ) ( ) } is:
2 2
3
3.5 + 3 2.5 3
3.5 - 3 8.75 + 3
2.5
(a) 5.375 (b) 1
(c) 6 (d) 5
Sol. (c) As given expression

( 3 3.5 + 3 2.5 ) ìíî( 3 3.5 ) ( 3 2.5 ) üýþ


2 2
Þ - 3 8.75 +

Let a = 3 3.5, b = 3 2.5


EBD_7382
54 Fundamental Operations, VBODMAS Rule......
Þ (a + b)(a2 – ab + b2)
æ
( ) ( )
3 3ö
Þ (a3 + b3) = ç 3 3.5 + 3 2.5 ÷ = 6
è ø

Shortcut Approach - 9
26
Application of the formula :
a3 – b3 = (a – b) (a2 + ab + b2)
(0.96) 3 - (0.1)3
17. The value of is
(0.96) 2 + (0.096) + (0.01)
(a) 0.86 (b) 1.06 (c) 0.95 (d) 0.97

( 0.96 )3 - ( 0.1)3
Sol. (a)
( 0.96 )2 + ( 0.096 ) + ( 0.01)

=
(
( 0.96 - 0.1) ( 0.96 )2 + ( 0.96 ) ´ ( 0.1) + ( 0.1)2 )
( 0.96) 2
+ ( 0.096 ) + ( 0.01)

( 0.96 - 0.1)(10.96 )2 + 0.096 + 0.01)


=
(
( 0.96)2 + ( 0.096) + 0.01 )
= 0.86

Shortcut Approach - 10
26
Application of the formula :
a3 + b3 + c3 = 3abc, if a + b + c = 0
( m - n) 3 + ( n - r ) 3 + ( r - m ) 3
18. =?
6( m - n)( n - r )( r - m )
1 1 1 1
(a) (b) (c) (d)
2 10 5 6

Sol. (a)
( m - n )3 + ( n - r )3 + ( r - m )3
6 ( m - n )( n - r )( r - m )

3 ´ ( m - n )( n - r )( r - m ) 1
Þ =
6 ( m - n )( n - r )( r - m ) 2
Fundamental Operations, VBODMAS Rule...... 55
19. Find the value of
( a - b) 2 ( b - c )2 (a - c ) 2
+ + .
(b - c )(c - a ) (a - b )(c - a ) (a - b )(b - c )
(a) 0 (b) 1 (c) 2 (d) 3
( a - b )2 ( b - c )2 ( a - c )2
Sol. (d) + +
( b - c )( c - a ) ( a - b )( c - a ) ( a - b )( b - c )
( a - b )3 + ( b - c )3 + ( c - a )3
=
( a - b )( b - c )( c - a )
3. ( a - b )( b - c )( c - a )
= =3
( a - b )( b - c )( c - a )

Shortcut Approach - 11
26
Application of the formula:
a3 + b3 + c3 – 3abc = (a + b + c)(a2 + b2 + c2 – ab – bc – ca)
or a3 + b3 + c3 – 3abc
1
= (a + b + c) {(a – b)2 + (b – c)2 + (c – a)2}
2
20. If a + b + c = 6 and a2 + b2 + c2 = 20, then the value of a3 + b3 + c3 –
3abc is
(a) 64 (b) 70 (c) 72 (d) 76
Sol. (c) (a + b + c)2 = (6)2
Þ a2 + b2 + c2 +2 (ab + bc + ca) = 36
Þ (ab + bc + ca) = 8 (Q a2 + b2 + c2 = 20)
Now, a + b + c – 3abc
3 3 3

= (a + b + c) (a2 + b2 + c2 – ab – bc – ca)
= 6 (20 – 8) = 72
21. If (a – b) = 3, (b – c) = 5 and (c – a) = 1, then the value of
a 3 + b3 + c 3 - 3abc
is
a +b+c
(a) 17.5 (b) 20.5 (c) 10.5 (d) 35

a3 + b3 + c3 - 3abc
Sol. (a)
a +b +c
EBD_7382
56 Fundamental Operations, VBODMAS Rule......

1
( a + b + c ) éêë( a - b )2 + ( b - c )2 + ( c - a )2 ùúû
= 2
a+b+c
1
= éê( 3) + ( 5 ) + (1) ùú
2 2 2
2ë û
1
= ´ ( 9 + 25 + 1) = 17.5
2

Shortcut Approach - 12
26
Application of the formula:
(a + b + c)2 = a2 + b2 + c2 + 2ab + 2bc + 2ca

x2 + y2 + z2 -64
22. If = – 2 and x + y = 3z, then the value of z is
xy - yz - zx
(a) 2 (b) 3
(c) 4 (d) None of these
Sol. (c) As given that,
x 2 + y 2 + z 2 - 64
= -2
xy - yz - zx
Þ x2 + y2 + z2 – 64 = –2(xy – yz – zx)
Þ [x + y + (–z)]2 = 64
Þ (3z – z)2 = 64 [Q x + y = 3z]
Þ 2z = 8 Þ z = 4
1 1 1
23. If a + b + c = 3, a2 + b2 + c2 = 6 and + + = 1, where a, b, c are
a b c
all non-zero, then 'abc' is equal to
2 3 1 1
(a) (b) (c) (d)
3 2 2 3
Sol. (b) As given that
a +b+c=3
(a + b + c)2 = 9
Þ (a2 + b2 + c2 + 2ab + 2bc + 2ca) = 9
Þ (6 + 2 (ab + bc + ca) = 9 [ Q (a2 + b2 + c2 = 6)]
Þ 2 (ab + bc + ca) = 3
Þ ab + bc + ca = 3/2
Fundamental Operations, VBODMAS Rule...... 57
dividing both side by (abc)
é1 1 1ù 3 é 1 ù
Þ ê a + b + c ú = 2 ê abc ú
ë û ë û
3 é 1 ùæ 1 1 1 ö
Þ 1= ç\ + + = 1÷
2 êë abc úû è a b c ø
Þ abc = 3/2

Shortcut Approach - 13
26
To simplify a continued Fraction :
You can easily understand this through the following
example,
1
5+
1
6+
1
6+
10
To simplify a continued fraction, begin at the bottom and work upwards
as follow :
1
5+
1
6+
1
6+
10
1
First take up the lowest complex fraction 1
6+
10
10
Multiply the numerator and denominator by 10 and thus we get .
61
1
Next multiply the numerator and denominator of the fraction by
10
6+
61
61
61 and thus we get .
376
61 61
Hence, the given fraction = 5 + or 5
376 376
2
24. Find the value of 1 +
4
3+
6
5+
7
EBD_7382
58 Fundamental Operations, VBODMAS Rule......

82 31 49 49
(a) 1 (b) 2 (c) 2 (d) 1
151 151 159 150

2 2
Sol. (a) 1+ = 1+
4 28
3+ 3+
6 41
5+
7
82 82
= 1+ =1
151 151

Shortcut Approach - 14
26
To find the value of x + x + x + ..... , find the factors of x, such that
the difference between the factors is 1, then the larger factor will be the
result.

25. 6 + 6 + 6 + ... is equal to


(a) 3 (b) 4 (c) 5 (d) 6

Sol. (a) 6 + 6 + 6 + .....


The factors of 6 with difference one are 2 and 3
Here 3 is the larger factors.

Hence 6 + 6 + 6 + ..... = 3

26. 12 + 12 + 12 + 12..... = ?
(a) 3 (b) 4 (c) 6 (d) 12

Sol. (b) 12 + 12 + 12 + 12 + ...


The two factors of 12 with difference one are 4 and 3.
Here, 4 is the bigger factor.

Hence, 12 + 12 + 12 + 12...... = 4

27. The value of 20 + 20 + 20 + ..... is


(a) 4 (b) 5
(c) 6 (d) greater than 6
Fundamental Operations, VBODMAS Rule...... 59
Sol. (b) As given expression is

Þ 20 + 20 + 20 + ......
The factors of 20 with difference 1 are 4 and 5.
Here 5 is the larger factor.

Hence 20 + 20 + 20 + ...... = 5

æ ö 2
28. çè 56 + 56 + 56 + ..... ÷ø ¸ 2 = ?
(a) 0 (b) 1 (c) 2 (d) 8
Sol. (c) As given expression
æ ö 2
Þ ç 56 + 56 + 56 + .... ÷ ¸ 2
è ø
The factor of 56 with difference one are 7 and 8.
Here 8 is the larger factor

Hence, æ 56 + 56 + 56 + .... ö = 8
ç ÷
è ø
Þ 8 ¸ 22 = 2

Shortcut Approach - 15
26
To find the value of x - x - x - ..... find the factors of x, such that the
difference between the factors is 1, then the smaller factor will be the result.

29. 2 - 2 - 2 - 2 ..... = ?
(a) 0 (b) 1 (c) 2 (d) 3

Sol. (b) As given expression is

2- 2- 2- 2
The factors of 2 with difference one are 1 and 2.
Here 1 is the smaller factor

Hence, 2- 2- 2- 2 =1

Shortcut Approach - 16
26
2n -1
x x x ........... n times = ( x ) 2n
EBD_7382
60 Fundamental Operations, VBODMAS Rule......

30. The value of 2 2 2 2 2 will be

(a) 2 (b) 215/32 (c) 231/32 (d) 4


é 25 -1 ù
ê ú
5
ëê 2 ûú
( 31/ 32)
Sol. (c) 2 2 2 2 2 = ( 2) = 2

Shortcut Approach - 17
26

x x x x .......¥ = x

31. 3 3 3... is equal to


(a) 3 (b) 3 (c) 2 3 (d) 3 3

Sol. (b) 3 3 3..... = 3


Alternate Method

Let 3 3 3..... = x
or, 3x = x
Squaring both side
3x = x2
0 = x2 – 3x
0 = x (x – 3)
x=3

32. If 7 7 7 7... = (343) y–1, then y will be equal to


(a) 2/3 (b) 1 (c) 4/3 (d) 3/4

Sol. (c) 7 7 7 7..... = (343)


y– 1

7 7 7 7..... = 7
7 = (343)y –1 Þ (7)1 = (7)3(y – 1)
Þ 3 (y – 1) = 1 Þ 3y – 3 = 1
Þ 3y = 4 Þ y = 4/3
Fundamental Operations, VBODMAS Rule...... 61

Exercise
1. If (X + (1/X)) = 4, then the 6. The value of
value of X 4 + 1/X 4 is 0.051´ 0.051´ 0.051
(a) 124
(b) 64 + 0.041´ 0.041´ 0.041
(c) 194 0.051´ 0.051 - 0.051´ 0.041 is
(d) Can’t be determined + 0.041´ 0.041
1
2. If x4 + 4 = 119 and x > 1, (a) 0.092 (b) 1.002
x (c) 0.0092 (d) 1.0002
then the value of 7. Calculate the number of
1 digits in the product of
x3 – 3 is 411111 × 522222
x
(a) 22222 (b) 22220
(a) 54 (b) 18
(c) 72 (d) 36 (c) 22221 (d) 22223
8. If (x + y + z) = 6 and (xy + yz +
3. If ‘a’ be a positive number,
zx) = 11, then the value of (x3
1 + y3 + z3 – 3xyz) is :
then the least value of a + is (a) 81
a
(a) 1 (b) 0 (b) 54
(c) 18
1 (d) none of these
(c) 2 (d)
2
4. If æ 1ö
9. If çè x + ÷ø = 2 3 , then the
x
1 1
x= a+ ,y = a - , æ 3 1ö
a a value of çè x + 3 ÷ø is :
then the value of x4 + y4 – x
2x2y2 is (a) 12 3
(a) 16 (b) 20 (c)
(b) 18
10 (d) 5
5. The value of (c) 18 3
(d) None of these
æ 1ö æ 3ö æ 5ö 10. If x = 997, y = 998, z = 999,
ç 2 - ÷´ç2 - ÷´ç2 - ÷
è 3ø è 5ø è 7ø then the value of
æ 999 ö x2 + y2 + z2 – xy – yz – zx will
´ ... ´ ç 2 - ÷ be
è 1001 ø (a) 3 (b) 9
1001 (c) 16 (d) 4
(a) (b) 999 11. If x2 + y2 + 2x + 1 = 0, then the
5
value of x31 + y35 is
1003 (a) – 1 (b) 0
(c) 1003 (d)
3 (c) 1 (d) 2
EBD_7382
62 Fundamental Operations, VBODMAS Rule......
12.
2 3
1 1 1 1 1 1 1 1 1 1 1 1 (a) (b)
. . + . . - 3. . . + . . 3 4
3 3 3 4 4 4 3 4 5 5 5 5
1 1 1 1 1 1 æ 1 1 1 1 1 1ö 47 49
. + . + . -ç . + . + . ÷ (c) (d)
3 3 4 4 5 5 è 3 4 4 5 5 3ø 60 60
is equal to
Hints & Solution
1. (c) 8. (c) Use Short Approach -11
2 9. (c) Use Short Approach -8
é 1ù 2 1 10. (a) x2 + y2 + z2 – xy – yz – zx
ê X + X ú = X + 2 + 2 = 16
ë û X 2
= ( x 2 + y 2 + z 2 - xy - yz - zx)
1 2
or X 2 + = 14
X2 1
= (2x2 + 2y2 + 2z2 - 2xy - 2yz - 2zx)
Now, 2
1 1 1
X4 + 4
+ 2 = 196 or X 4 + = 194. = (x2 ++y2 –-2xy + y+2 + z2+– 2yz-+ x2 + + -
X X4 2
2. (d) Use Short Approach -9 + z2 – 2xy)
1
3. (c) The least value of = [( x - y )2 + ( y - z )2 + ( z - x) 2 ]
2
1 1
a+
is 2 where a = 9. = [(997 -– 998)2 ++ - + -
a 2
4. (a) Use Short Approach -2 (998 – 999)2 + (999 – 997)2]
5. (d)
1 1
æ 1ö æ 3ö æ 5ö = [12 + 12 + 22 ] = ´ 6 = 3
ç 2 - ÷´ç2 - ÷´ç 2 - ÷ 2 2
è 3ø è 5ø è 7ø 11. (a) x2 + y2 + 2x + 1 = 0
æ 999 ö Þ (x + 1)2 + y2 = 0
×´..... ´ ç 2 - ÷ Þ x + 1 = 0 Þ x = – 1 and
è 1001 ø y =0
5 7 9 1003 1003 \ x31 + y35 = – 1
= ´ ´ ´ ........... ´ = 12. (c) Q a3 + b3 + c3 – 3abc =
3 5 7 1001 3
6. (a) Use Short Approach -8 (a + b + c)
7. (d) 411111 × 522222 (a2 + b2 + c2 – ab – bc – ca)
Here
= ( 22 )
11111
´ 522222 1 1 1
a = ,b = ,c =
é n ù 3 4 5
= 222222 ´ 522222 ë( a m ) = a mn û So,
= 1022222 [am × bm = (ab)m] 1 1 1 47
Hence, number of digits a+b+c= + + =
= 22222 + 1 = 22223. 3 4 5 60
06
Polynomials and Rational Expressions
(Inducing Factorization of
Polynomials, Factor Theorem &
Chapter Remainder Theorem)

Shortcut Approach (Remainder Theorem) - 1


Let f (x) be a Polynimial of degree greater than or equal to 1. Then if f (x) is
divided by (x–a) where ‘a’ be any real number then the nemainder = f (a).
1. Find the remainder when x3 + 2x2 – 5x + 3 is divided by x – 2.
(a) 7 (b) 9 (c) 1 (d) 0
Sol. (b) Let p (x) = x3 + 2x2 – 5x + 3
Remainder = p (2) = 23 + 2 × 22 – 5 × 2 + 3 = 8 + 8 –10 + 3 = 9
2. Find the remainder when x4 + x2 +1 is divided by x + 1.
(a) 0 (b) 5 (c) 2 (d) 3
Sol. (d) Let p (x) = – x4 – x2 + 1
Now x + 1 = x– (–1)
Hence, remainder = p (–1) = (–1)4 – (–1)2 + 1 = 3

Shortcut Approach (Factor Theorem) - 2


Let p (x) be a polynimial of degree greater than or equal to 1 and ‘a’ be any
real number such that p (a) = 0 then (x – a) is a factor of p (x)
3. Is (x – 2) is a factor of x3 + 3x2 – 12x + 4?
Sol. Let p (x) = x3 +3x2 – 12x + 4
Now p (2) = (2)3 + (2)2 – 12× 2 + 4 = 8 + 12 – 24 + 4 = 0
Hence x – 2 is a factor of x3 + 3x2 – 12x + 4

Shortcut Approach - 3
If a number a is zero of a linear polynomial f (x), if f (a) = 0
4. If a is the zero of 2x + 5 then what is velue of a?
5 -5 -2
(a) (b) (c) (d) 0
2 2 5
Sol. (b) Let f(x) = 2x + 5
Since a is the zero of the given polynomial,
Therefore f (a) = 0
-5
Þ 2a + 5 = 0 Þ a=
2
EBD_7382
64 Polynomials and Rational Expressions...

Shortcut Approach - 4
If a and bare zeros of a Quarditic Polynomial ax2 +bx +c with a ¹ 0 then
-b c
a+b= and a. b=
a a
5. Find is sum of zeros of 5x2 + 34x +24?
-34 -24 34
(a) (b) (c) –5 (d) -
5 5 5
- b -34
Sol. (a) sum of zero of Quarditic Polynomial = =
a 5

Shortcut Approach - 5
If a, b and g are zeros of a cubic Polynomial ax3 + bx2 + cx + d,
a ¹ 0, then
-b c
(i) a+b+g= (ii) ab+gb+ga=
a a
-d
(iii) abg=
a
6. Find the Product of zeros of the polynomial 5x3 +17x2 +19x +25
-19
(a) (b) –5 (c) 5 (d) None of these
5
-d -25
Sol. (b) Product of zeros of a cubic polynomial = = = -5
a 5
Shortcut Approach - 6
If p (x) and q (x) are two different polynomials then
L.C.M × H.C.F = P (x) × q (x)
7. If L.C.M and H.C.F of two polynomials are (x – 1)2 (x + 2)3(x + 3) and
(x – 1) (x + 3) and one of the two polynomials is (x–1) (x+2)3 (x + 3)
then second polynomial is
(a) (x + 3) (x – 1)2 (b) (x + 3)2 (x – 1)
(c) (x + 3)2 (x – 1)2 (d) None of these
L.C.M´ H .C.F
Sol. (a) Second Polynomial =
First Polynomial

( x - 1)2 ( x + 2)3 ( x + 3)( x - 1)( x + 3)


= = (x + 3)(x – 1)2
( x - 1)( x + 2)3 ( x + 3)
Polynomials and Rational Expressions... 65

Exercise
1. Factorize : (a – b)3 + (b – c)3 + 8. If the polynomials (2x3 + ax2
(c – a)3 + 3x – 5) and (x3 + x2 – 2x + a)
(a) 3(a + b) (a – b) (a + c) leave the same remainder
(b) 3(abc) when divided by (x – 2), find
(c) 3(a – b) (b – c) (c – a) the value of a.
(d) 3(a + b) (b + c) (c + a) (a) – 8 (b) 4
2. If (x3 – 5x2 + 4p) is divisible 6
by (x + 2), then the value of p (c) – 3 (d)
3
is 9. If (x – a) and (x – 2) are two
(a) 7 (b) – 2 factors of x3 – ax2 + 14x + b,
(c) 3 (d) – 7 the values of a and b are
3. If (x + 2) and (x – 1) are the (a) a = 6, b = 7
factors of (x3 + 10x2 + mx + (b) a = –8, b = –7
n), the values of m and n are : (c) a = 7, b = –8
(a) m = 5, n = –3 (d) a = 7, b = 8
(b) m = 17, n = – 8 10. Let r1 and r2 be the remainders
(c) m = 7, n = – 18 when the polynomials
(d) m = 23, n = – 19 p(x) = x3 + x2 – 5kx – 7
4. Find the remainder when 5p3 and q(x) = x3 + kx2 – 12x + 6
– 13 p2 + 21p – 14 is divided are divided by x + 1 and x – 2
by (3 – 2p + p2). respectively.
(a) –5 (b) –15 If 2r1 – 2r2 = 10, the value of k is.
(c) –10 (d) 5 7 3
5. If p(y) = 3y4 – 5y3 + y2 + 8, (a) (b)
then p(–1) will be 3 7
(a) 2 (b) 15 5 7
(c) 17 (d) –17 (c) (d)
7 5
6. For what value of k is the
polynomial 2x4 + 3x3 + 2kx2 + 4
11. If x = is a root of the
3x + 6 exactly divisible by 3
(x +2)? polynomial
(a) 1 (b) –1 f (x) = 6x3 – 11x2 + kx – 20, then
(c) 2 (d) –2 find the value of k.
7. If 4x4 – 3x³ – 3x² + x – 7 is di- (a) 10 (b) 19
vided by 1 – 2x then remain- (c) – 5 (d) 3
der will be – 12. If two zeroes of the
polynomial x4 – 6x3 –26 x2
57 59
(a) (b) - + 138x – 35 are 2 ± 3 , then
8 8 the other zeroes are
55 55 (a) 5, 7 (b) –5, 7
(c) (d) – (c) –5, –7 (d) 5, –7
8 8
EBD_7382
66 Polynomials and Rational Expressions...

Hints & Solution


1. (c) Q (a – b) + (b – c) + (c Now, f(2) = (2 × 23 + a × 22 + 3
– a) = 0 × 2 – 5) = (17 + 4a).
\ (a – b)3 + (b – c)3 + (c – a)3 And, g(2) = (23 + 22 – 2 × 2 +
= 3(a – b)(b – c) (c – a) a) = (8 + a)
2. (a) Since (x + 2) divides the \ 17 + 4a = 8 + a Þ 3a = –
polynomial (x3 – 5x2 + 4p) 9 Þ a = – 3.
Hence, 9. (c) Use Shortcut Approach -1
f (– 2) = 0. 10. (a) Here, we will use
f (x) = x3 – 5x2 + 4p remainder theorem to find r 1
Þ f (–2) = (–2)3 – 5. (–2)2 + and r2. Remainder theorem
4p = 0 states that if a polynomial f(x)
Þ f (–2) = – 8 – 20 + 4p = 0 is divided by (x – a), then the
Þ p = 27 remainder is given by f(a).
3. (c) Use Shortcut Approach -2 Let us perform synthetic
4. (a) division of p(x) and q(x) by
(x + 1) an d (x – 2)
5p – 3 respectively.
2 3 2
p – 2p + 3 5p – 13p + 21p –14 –1 1 1 –5k –7
3 2
5p – 10p + 15p –1 0 5k
2
–3p + 6p – 14 1 0 –5k 5k – 7 = r1
2
–3p + 6p – 9 and
2 1 k –12 6
–5
2 2k + 4 4k – 16
Hence, the remainder is –5.
5. (c) p(–1) = 3(–1)4 – 5(–1)3 1 k + 2 2k – 8 4k – 10 = r2
+ (–1)2 + 8 Given that 2r 1 – r 2 = 10
= 3 + 5 + 1 + 8 = 17 Þ 2(5k – 7) – (4k – 10) = 10
6. (b) f(–2) = 2 × (–2)4 + 3 (–2)3 + 2k Þ 10k – 14 –4k + 10 = 10
× (–2)2 + 3 × (–2) + 6 Þ 6k = 14
= 32 – 24+ 8k – 6 + 6 = 8k + 8 7
\ f(–2) = 0 Þ 8k + 8 = 0 Þ k \ k=
3
= –1 11. (b) Let f (x) = 6x3 – 11x2 + kx – 20
7. (b) Use Shortcut Approach -1 3 2
æ4ö æ4ö æ4ö æ4ö
8. (c) Let f(x) = 2x3 + ax2 + 3x – f ç ÷ = 6 ç ÷ - 11 ç ÷ + k ç ÷ - 20 = 0
è3ø è3ø è3ø è3ø
5 and g(x)
= x3 + x2 – 2x + a. 64 16 4 k
Þ 6. - 11. + - 20 = 0
When f(x) is divided by (x – 2), 27 9 3
remainder = f(2). Þ 128 – 176 + 12k – 180 = 0
When g(x) is divided by (x – 2), Þ 12k + 128 – 356 = 0 Þ k = 19
remainder = g(2). 12. (b) Use Shortcut Approach -4
07
Linear Equation, Quadratic
Equations & Inequalities (Including
Chapter Simultaneous Linear Equations)

Shortcut Approach - 1
Shortcut Approach to solve quadratic equation
ax2 + bx + c = 0, if b2 – 4ac ³ 0,

Divide ac in two parts p and q such


that their sum is equal to coefficient
b of x and product equal to ac

Divide p and q by a

Change the sign

-p -q
Here and are two roots or solutions of quadratic equation ax2
a a
p q
+ bx + c = 0 i.e. x = - or - .
a a
1. Solve : 2x2 + 6 = 7x

3 1
(a) ,2 (b) ,2
2 2

5
(c) ,2 (d) None of these
2
Sol. (a) 2x2 + 6 = 7x
Þ 2x2 – 7x + 6 = 0
EBD_7382
68 Linear Equation, Quadratic Equations ...

2 x2 – 7x + 6 = 0

12 2 × 6 = 12

(–4) + (–3) = – 7
–4 –3
(–4) × (–3) = 12

–4 –3 Divide by coefficient 2 or x2
2 2

2 3
Change the sign
2
\ Solutions or roots of given quadratic equation
3
2x2 + 6 = 7x are and 2.
2
1 1
2. Solve x – =1
x 2
1 -1
(a) ,2 (b) ,2
2 2
3 -3
(c) ,2 (d) ,2
2 2
1 1
Sol. (b) x - =1
x 2
x2 - 1 3
Þ = Þ 2(x 2 - 1) = 3x
x 2
2 – 2 = 3x 2x2 – 3x – 2 = 0
Þ 2x Þ
2 x2+ (–3)x+ (–2) = 0

–4 2 × (–2) = –4

–4+1=–3
–4 1 –4×1=–4

1 Divide by 2
–2
2

2 –1 Change the sign


2
Linear Equation, Quadratic Equations ... 69
Either 2x + 1 = 0 or x – 2 = 0
Þ 2x = – 1 or x = 2
-1
Þ x= or x = 2
2
-1
\ x= , 2 are solutions.
2

Shortcut Approach - 2
If present age of the father is F times the age of his son. T years hence,
the father’s age become Z times the age of son then present age of his
(Z - 1)T
son is given by
(F - Z)
3. Present age of the father is 9 times the age of his son. One year
later, father’s age become 7 times the age of his son. What are the
present ages of the father and his son.
(a) 27 years (b) 30 years
(c) 36 years (d) 45 years
Sol. (a) By the formula
(7 - 1) 6
Son’s age = ´ 1 = ´ 1 = 3 years.
(9 - 7) 2
So, father’s age = 9 × son’s age = 9 × 3 = 27 years.

Shortcut Approach - 3
If T1 years earlier the age of the father was n times the age of his son, T2
years hence, the age of the father becomes m times the age of his son
then his son’s age is given by
T1 (n - 1) + T2 (m - 1)
Son’s age =
n -m
4. 10 years ago, Shakti’s mother was 4 times older than her. After 10
years, the mother will be twice older than the daughter. What is
the present age of Shakti?
(a) 15 years (b) 20 years
(c) 24 years (d) 30 years
Sol. (b) By using formula,
10(4 - 1) + 10(2 - 1)
Shakti’s age = = 20 years.
4-2
EBD_7382
70 Linear Equation, Quadratic Equations ...

Shortcut Approach - 4
Present age of Father : Son = a : b
After / Before T years = m : n
Then son’s age = b × T(m - n)
an - bm
T(m - n)
and father’s age = a ×
an - bm
5. The ratio of the ages of the father and the son at present is 3 : 1.
Four years earlier, the ratio was 4 : 1. What are the present ages
of the son and the father?
(a) 10 years, 30 years
(b) 20 years, 50 years
(c) 12 years, 36 years
(d) None of these
Sol. (c) Ratio of present age of father and son = 3 : 1
4 years before = 4 : 1
4(4 - 1)
Son’s age = 1 ´ = 12 years.
4 ´1 - 3 ´1
4(4 - 1)
Father’s age = 3 ´ = 36 years.
4 ´1 - 3 ´1
Linear Equation, Quadratic Equations ... 71

Exercise
1. If ax2 + bx + c = 0 has real and
different roots, then p 2 - 4q
(c)
(a) b2 – 4ac = 0 q2
(b) b2 – 4ac > 0
(c) b2 – 4ac < 0 p 2 - 2q
(d) b2 – 4ac £ 0 (d)
q2
2. When 24 is subtracted from a
number, it reduces to its four- 10. For what value of c the
seventh. What is the sum of quadratic equation
the digits of that number ? x2 – (c + 6) x + 2(2c – 1) = 0
(a) 1 has sum of the roots as half
(b) 9 of their product?
(c) 11 (a) 5 (b) – 4
(d) Data inadequate (c) 7 (d) 3
5. Determine the value of k for
3. The roots of the equation
which the quadratic equation
1 4x2 – 3kx + 1 = 0 has equal
-
2 x + 2 x = 5 can be 2
roots :
found by solving
(a) 4x2 – 25x + 4 = 0 æ 2ö æ 4ö
(a) ± ç ÷ (b) ± ç ÷
è 3ø è 3ø
(b) 4x2 + 25x – 4 = 0
(c) 4x2 – 17x + 4 = 0 (c) ± 4 (d) ± 6
(d) None of these 6. Find the value of k so that
4. The roots of the equation ax2 the sum of the roots of the
+ bx + c = 0 will be reciprocal if quadratic equation is equal to
(a) a = b the product of the roots :
(b) a = bc (k + 1)x2 + 2kx + 4 = 0
(c) c = a (a) – 2 (b) – 4
(d) b = c (c) 6 (d) 8
7. If r and s are roots of x2 + px + 8. If – 4 is a root of the quadratic
q = 0, then what is the value equation
x 2 – px – 4 = 0 and the
1 1 quadratic equation
of
2
? +
r s2 x2 – px + k = 0 has equal roots,
(a) p2 – 4q find the value of k :
(a) 9/4 (b) 1
p 2 - 4q (c) 2.5 (d) 3
(b)
2
EBD_7382
72 Linear Equation, Quadratic Equations ...
9. Find the quadratic equation 11. If p and q are the roots of x2 +
whose roots are px + q = 0, then
3 and
(a) p = 1 (b) p = 1 or 0
2 3: (c) p = – 2 (d) p = – 2 or 0
12. A two digit number is such that
(a) x 2 + 3 3x - 6 = 0
the product of its digits is 12.
(b) x 2 - 3 3x + 6 = 0 When 9 is added to the number,
the digits interchange their
(c) x 2 + 3 3x + 5 = 0 places, find the number :
(d) None of these (a) 62 (b) 34
(c) 26 (d) 43
Linear Equation, Quadratic Equations ... 73

Hints & Solution


1. (b) Use Short Approach -1
9
2. (c) Let the number be x, \ k=
4
Then,
3. (c) Use Short Approach -1 9. (b) Method – I.
4. (c) Since roots are reciprocal, If a and b be the roots
product of the roots = 1 of a quadratic equation,
then the equation will be
c
Þ = 1 Þ c = a. (x – a) (x – b) = 0
a
5. (b) 4x2 – 3kx + 1 = 0 \ ( x - 3 )( x - 2 3 ) = 0
D = b2 – 4ac = 0
\ 9k2 – 4 × 1 × 4 = 0 Þ x 2 - 3 3x + 6 = 0
Method – II.
2 16 4
Þk = Þk=± The required equation
9 3
x2 – (a + b)x + (a. b) = 0
6. (a) a + b = ab
Þ x 2 - ( 3 + 2 3 ) x + ( 3.2 3 ) = 0
-b c
Þ = Þ -b = c Þ
a a x 2 - 3 3x + 6 = 0
\ – 2k = 4 Þ k = –2 10. (c) Use Short Approach -1
7. (d) Use Short Approach -1 11. (b) Since p and q are the
8. (a) Since – 4 is a root of roots of given equation
x2 – px – 4 = 0 x2 + px + q = 0, then p +
\ (–4)2 – p (–4) – 4 = 0 Þ q = –p Þ q = –2p
p = –3 and pq = q Þ p = 1
\ The equation becomes So, when p = 1, then q =
x2 + 3x + k = 0 Again, when q = 0, then
Since both the roots of p = 0 hence,
this equation are equal. p = 1, 0 and q = – 2, 0
\ Sum of the roots (a + a) Thus, option (b) is most
appropriate.
3
= 2a = –3 Þ a = - 12. (b) Let the tens digit be x and
2
unit digit be y. Then the
\ Product of the roots two digit number = 10 x + y
9 but
( a.a ) = a 2 = ( -3 / 2)2 = =k
4 12
x ´ y = 12 Þ y =
x
EBD_7382
74 Linear Equation, Quadratic Equations ...
Þ (x + 4) (x – 3) = 0
æ 12 ö
\ the number is çè 10x + ÷ Þ x = – 4, 3

but x cannot be negative
Again
\ x = 3 only
12 æ 12 ö
10x + + 9 = 10 ´ ç ÷ + x 12 12
x èxø y= = =4
\
x 3
Þ 10x2 + 12 + 9x = 120 + x2
Þ 9x2 + 9x – 108 = 0 \ the number = 10x + y
Þ x2 + x – 12 = 0 = 10 × 3 + 4 = 34
08 Ratio, Proportion, Variation
and Partnership
Chapter

Shortcut Approach - 1
To divide a given quantity into a given ratio
Suppose any given quantity a is to be divided in the ratio m : n, then
ma na
First part = ; Second part =
m+n m+n
1. Divide 1458 into two parts such that one may be to the other as 2 :
(a) 324, 1134 (b) 358, 1100
(c) 462, 996 (d) 458, 1000
2 ´ 1458 2 ´ 1458
Sol. (a) First part = = = 324
2+7 9
7 ´ 1458
Second part = = 1134
2+7

Shortcut Approach - 2
(i) If a : b = N1 : D1, b : c = N2 : D2
Then a : b : c = N1N2 : D1N2 : D1D2.
(ii) If a : b = N1 : D1, b : c = N2 : D2, c : d = N3 : D3
Then a : b : c : d = N1N2N3 : D1N2N3 : D1D2N3 : D1D2D3
(iii) If a : b = N1 : D1, b : c = N2 : D2, c : d = N3 : D3, d : e = N4 : D4
Then a : b : c : d : e = N1N2N3N4 : D1N2N3N4 : D1D2N3N4 : D1D2D3N4
: D1D2D3D4.
2. The ratio of A: B = 1 : 3, B : C = 2: 5 and C : D = 2 : 3. Find the value
of A : B : C: D.
(a) 3 : 9 : 12 : 15 (b) 4 : 12 : 30 : 45
(c) 5 : 10 : 15 : 20 (d) 4: 8 : 12 : 20
Sol. (b) As given that,
A : B = 1 : 3, B : C = 2 : 5 and C : D = 2 : 3
We know that
A : B : C : D = N1N2N3 : D1D2N3 : D1D2D3
= (1 × 2 × 2) : (3 × 2 × 2) : (3 × 5 × 2) : (3 × 5 × 3)
A : B : C : D = 4 : 12 : 30 : 45
EBD_7382
76 Ratio, Proportion, Variation and Partnership
3. If ratio A : B = 1 : 2, B : C = 3 : 4 and C: D = 5: 6 find D : C : B : A
=?
(a) 6 : 5 : 4 : 2 (b) 6 : 4 : 2 : 1
(c) 6 : 3 : 2: 1 (d) 48 : 40 : 30 : 15
Sol. (d) A : B = 1 : 2, B : C = 3 : 4 and C : D = 5 : 6
A : B: C : D
= (1 × 3 × 5) : (2 × 3 × 5) : (2 × 4 × 5) : (2 × 4 × 6)
= 15 : 30 : 40 : 48
D : C : B : A = 48 : 40 : 30 : 15

Shortcut Approach - 3
If the ratio between two numbers is a : b and if each number is increased by
xa(c - d )
x, the ratio becomes c : d. Then two numbers are given by and
ad - bc
xb(c - d )
ad - bc
4. Two numbers are in the ratio of 2: 3. If 15 is added to both the numbers,
11
then the ratio between two numbers becomes .
14
Find the greater number.
(a) 18 (b) 30 (c) 27 (d) 9
Sol. (c) Numbers are
15 ´ 2 (11 - 14 ) 15 ´ 3 (11 - 14 )
and
2 ´ 14 - 3 ´11 2 ´14 - 3 ´ 11
-30 ´ 3 - 45 ´ 3
Þ and Þ 18 and 27
-5 -5
Hence gretest number is 27

Shortcut Approach - 4
If two numbers are in the ratio of a : b and the sum of these numbers is x,
ax bx
then these numbers will be and .
a+b a+b
5. Two glasses A and B with their capacities in the ratio
2 : 3. If the total capacities of the glasses is 45 litres. Find the
capacity of glass A.
(a) 5 (b) 6 (c) 7 (d) 18
Ratio, Proportion, Variation and Partnership 77
Sol. (d) \ Capacity of glass
ax 2 ´ 45
( A) = = = 18
( a + b ) ( 2 + 3)
6. Amit and Sudesh have invested in the ratio of
4 : 7. If both invested an amount of ` 49500, then find the investment
of sudesh.
(a) 31500 (b) 1800
(c) 31000 (d) 18500
æ bx ö
Sol. (a) \ Investment of Sudesh = ç ÷
è a +b ø
7 ´ 49500
= = 31500
( 7 + 4)
Shortcut Approach - 5
If two numbers are in the ratio a : b and the difference between these
numbers is x, then these numbers will be
ax bx
(i) and respectively (if a > b)
a -b a -b
ax bx
(ii) and respectively (if a < b)
b-a b-a
7. The ratio of two numbers is 10:7 and their difference is 105. The
sum of the numbers is:
(a) 595 (b) 805
(c) 1190 (d) 1610
ax bx
Sol. (a) + ( Q a > b)
a -b a -b
10 ´ 105 7 ´ 105
= + = 10 ´ 35 + 7 ´ 35 = 595
(10 - 7 ) (10 - 7 )
8. A certain sum of money is divided between A and B in the ratio 3:7.
If B gets ` 800 more than A, find the total sum.
(a) ` 2400 (b) ` 3200 (c) ` 3000 (d) ` 2000
æ ax ö æ bx ö
Sol. (d) ç ÷+ç ÷ (Q b > a)
èb-aø èb-a ø
( 3 ´ 800 ) ( 7 ´ 800 ) 800
= + = ( 3 + 7 ) = 2000
( 7 - 3) ( 7 - 3) 4
EBD_7382
78 Ratio, Proportion, Variation and Partnership

Shortcut Approach - 6
If three numbers are in the ratio a : b : c and the sum of these numbers is
x, then these numbers will be
ax bx cx
, and respectively..
a +b+c a+b+ c a+b+c
9. If ` 126.50 is divided among A, B and C in the ratio of 2 : 5: 4, the
share of B exceeds that of A by
(a) ` 36.50 (b) ` 35.50
(c) ` 34.50 (d) ` 33.50
Sol. (c) Share of
bx 5 ´126.50
B= = = 57.5
a+b+c 2+5+ 4
Share of
ax 2 ´ 126.50
A= = = 23.0
a +b+c 2+5+4
then, the share of B exceeds that of A = B – A
= 57.5 – 23 = 34.50
1
10. If ` 900 is divided among A, B, C; the division is such that of
2
1
A's money = rd of B's money
3
1
= th of C's money. Find the amount (in `) received by A, B, C.
4

(a) 300, 400, 200 (b) 350, 450, 100


(c) 200, 300, 400 (d) 400, 150, 130
Sol. (c) As given that,
A B C
= = Þ A : B:C = 2:3: 4
2 3 4
Q Amount received by
æ ax ö 2 ´ 900
A=ç ÷ = = 200
è a + b + c ø ( + 3 + 4)
2
Q Amount received by
æ bx ö 3 ´ 900
B=ç ÷ = = 300
è a + b + c ø ( 2 + 3 + 4)
Q Amount received by
Ratio, Proportion, Variation and Partnership 79

cx 4 ´ 900
C= = = 400
( 2 + b + c) (a + 3 + 4)
Shortcut Approach - 7
If two numbers X and Y are in the ratio x : y. Then
X +Y x + y Y + X y+x
(i) = , if x > y. (ii) Y - X = y - x , if x < y.
X -Y x - y
11. The prices of a scooter and a moped are in the ratio of 9 : 5. If a
scooter costs ` 4200 more than a moped, find the total price of both
vehicles.
(a) `14000 (b) `14700
(c) ` 15000 (d) `15200
X+Y x+y
Sol. (b) Here, =
X-Y x-y
X+Y 9+5 14
Þ = Þ X + Y = 4200 ´
4200 9 - 5 4
Þ X +Y = ` 14700

Shortcut Approach - 8
If the sum of two numbers is A and their difference is a, then the ratio of
numbers is given by (A + a) : (A – a).
12. The sum of two numbers is 40 and their difference is 4. The ratio of
the numbers is :
(a) 21 : 19 (b) 11: 9
(c) 22 : 9 (d) 11 : 18
A + a é 40 + 4 ù = 44 = 11
Sol. (b) Ratio of two numbers = =
A - a êë 40 - 4 úû 36 9
13. The ratio between sum and difference of two numbers is
5 : 1. Find the ratio between the numbers.
(a) 5 : 2 (b) 3 : 2
(c) 2 : 3 (d) 2 : 5
Sol. (b) As given, ratio of sum and difference of two numbers
=5:1
So, the sum of two numbers = 5 and their difference is 1, the
ratio of two numbers
5 +1 6 3
= = =
5 -1 4 2
EBD_7382
80 Ratio, Proportion, Variation and Partnership

Shortcut Approach - 9
A number which, when added to the terms of the ratio a : b makes it equal
ad - bc
to the ratio c : d is .
c -d
14. What must be added to each term of the ratio 7 : 11 so as to make it
equal to 3:4 ?
(a) 1 (b) 2
(c) 3 (d) 5
ad - bc = ( 7 )( 4 ) - (11)( 3) = -5 = 5
Sol. (d) Required number =
c-d 3- 4 -1

Shortcut Approach - 10
A number which when subtracted from the terms of the ratio a : b makes it
bc - ad
equal to the ratio c : d is .
c-d
15. What number should be subtracted from both terms of the ratio 15
: 19 in order to make it 3 :4 ?
(a) 9 (b) 2
(c) 5 (d) 3
bc - ad
Sol. (d) Required number =
c-d
(19 )( 3) - (15)( 4 ) =
57 - 60
=3
=
3- 4 -1

Shortcut Approach - 11
The ratio between two numbers is a : b, if each number is increased by x,
x(a + b)(c - d )
the ratio becomes c : d. Then sum of two numbers = .
ad - bc
16. Two numbers are in the ratio 2 : 3. If 3 be added to both of them, then
their ratio becomes 3 : 4. Find the sum of the numbers.
(a) 10 (b) 15
(c) 20 (d) 25
x ( a + b )( c - d )
Sol. (b) Sum of two numbers =
ad - bc
3 ( 2 + 3)( 3 - 4 ) 3 ´ 5 ´ ( -1)
= = = 15
2 ( 4 ) - ( 3)( 3) ( -1)
Ratio, Proportion, Variation and Partnership 81

Shortcut Approach - 12
Income of two persons are in the ratio of a : b and their expenditures are
in the ratio c : d. If each of them saves ` x then their incomes are given by
xa(d - c) xb(d - c)
and .
ad - bc ad - bc
17. The ratio of monthly incomes of A, B is 6 : 5 and their monthly
expenditures are in the ratio 4 : 3. If each of them saves ` 400 per
month, find the sum of their monthly incomes.
(a) ` 2300 (b) ` 2400
(c) ` 2200 (d) ` 2500
xa ( d - c ) xb ( d - c )
Sol. (c) Sum of incomes = +
ad - bc ad - bc
400 ( 6 )( 3 - 4 ) 400 ( 5 )( 3 - 4 ) -2400 -2000
= + = +
( 6 )( 3) - ( 5)( 4 ) ( 6 )( 3) - ( 5)( 4 ) -2 2
= 1200 + 1000 = ` 2200
18. The incomes of Ram and Shyam are in the ratio
8 : 11 and their expenditure are in the ratio 7 :10. If each of them
saves ` 500, what are their incomes and expenditures ?
(a) 4000, 5500, 3500, 5000
(b) 5000, 5000, 4000, 3500
(c) 4000, 5000, 3500, 4500
(d) None of these
(11 - 8 ) ´ 500 + 7 3 ´ 500 ´ 7
Sol. (a) Expenditure of Ram = = = 3500
8 ´10 - 11 + 7 3
500 ´10(11 - 8)
Expenditure of Shyam = = 500
3
500 ´ 8(10 - 7)
Income of Ram = = 4000
8 ´ 13 - 7 ´ 11
500 ´ 10(11 - 8)
Income of Shyam = = 5000
3

Shortcut Approach - 13
Income of two persons are in the ratio a : b and their expenditure are in the
ratio c : d. If each of them saves ` x, then their expenditures are given by
xc(b - a ) xd (b - a )
and .
ad - bc ad - bc
EBD_7382
82 Ratio, Proportion, Variation and Partnership
19. The incomes of Ram and Shyam are in the ratio
8 : 11 and their expenditure are in the ratio 7 :10. If each of them
saves ` 500, what are their incomes and expenditures ?
(a) 4000, 5500, 3500, 5000
(b) 5000, 5000, 4000, 3500
(c) 4000, 5000, 3500, 4500
(d) None of these
(11 - 8 ) ´ 500 + 7 3 ´ 500 ´ 7
Sol. (a) Expenditure of Ram = = = 3500
8 ´10 - 11 + 7 3
500 ´10(11 - 8)
Expenditure of Shyam = = 500
3
500 ´ 8(10 - 7)
Income of Ram = = 4000
8 ´ 13 - 7 ´ 11
500 ´ 10(11 - 8)
Income of Shyam = = 5000
3

Shortcut Approach - 14
Two candles of the same height are lighted at the same time. The first is
consumed in T1 hours and the second in T2 hours. Assuming that each
candle burns at a constant rate, the time after which the ratio of first candle
æx ö
T1T2 ç - 1÷
èy ø
to second candle becomes x : y is given by hours.
æ xö
çè y ÷ø T1 - T2

20. Two candles of the same height are lighted at the same time. The
first is consumed in 7 hours and the second is consumed in 4 hours.
Assuming that each candle burns at a constant rate, in how many
hours, after being lighted, was the first candle four times the height
of the second ?
(a) 3.5 hrs (b) 2.5 hrs
(c) 2 hrs (d) 3 hrs
Sol. (a) If out of two candles of the same height, the first burns in T 1
æx ö
T1T2 ç - 1÷
hour and the second burns in T2 hour, then after èy ø
æxö
ç ÷ ( T1 ) - T2
è yø
Ratio, Proportion, Variation and Partnership 83
hours, the ratio of the height of remaining parts will be 4 : 1
Here, T1 = 7hrs, T2 = 4hrs

\ Required time =
( 7 )( 4 )( 4 - 1) = 84 = 3.5 hrs
( 4 )( 7 ) - 4 24
Shortcut Approach - 15
If on adding x to four numbers a, b, c and d; the resulting numbers become
bc - ad
proportional, then x = .
(a + d ) - (b + c)
21. What must be deducted from each of the numbers 7, 10, 12 and 18
so that the resultant numbers are in proportion ?
(a) 1 (b) 2
(c) 3 (d) 4
bc - ad (10 )(12 ) - ( 7 )(18) -6
Sol. (b) Required number = ( a + d ) - ( b + c ) = ( 7 + 18) - (10 + 12 ) = 3 = ( -2 )
Thus, 2 must be subtracted from each of the numbers.

Shortcut Approach - 16
Two numbers are in the ratio a : b. If p is added to the first number and q
is added to the second number respectively, then ratio of the resulting
numbers become m : n. Then,
a( mq - np )
The original first number = and the original second number
an - bm
b( mq - np )
=
an - bm
22. In a class, the number of boys and girls is in the ratio of 4: 5. If 10
more boys join the class, the ratio of numbers of boys and girls
becomes 6 : 5. How many girls are there in the class?

(a) 20 (b) 30
(c) 25 (d) Couldn't be determined
b ( mq - np )
Sol. (c) Number of girls =
an - bm
5 é( 6 )( 0 ) - ( 5 )(10 ) ùû 5 ´ ( -50 )
= ë = = 25
( 4 )( 5 ) - ( 5)( 6 ) ( -10 )
EBD_7382
84 Ratio, Proportion, Variation and Partnership

Exercise
1. Two numbers are in the ratio 6. The ratio of males and
of 3 : 4. If 5 is subtracted from females in a city is 7 : 8 and
each, the resulting numbers the percentage of children
are in the ratio 2 : 3. Find the among males and females is
numbers 25% and 20% respectively. If
(a) 12, 16 (b) 24, 32 the number of adult females
(c) 60, 80 (d) 15, 20 in the city is 156800 what is
2. Find the fourth proportional the total population?
to 12X 3, 9aX 2, 8a 3X. (a) 245000 (b) 367500
(a) 4a3 (b) 6a4 (c) 196000 (d) 171500
(c) 5a (d) 7a5 7. A and B invested ` 12,000
3. In a partnership, A invests and ` 18,000 respectively in
1 1 a business for the whole year.
of the capital for of the At the year-end, there was a
6 6
total profit of ` 2,000. What
1 is the share of A in the profit?
time, B invests of the capital
3 (a) ` 800
1 (b) ` 1, 200
for of the time and C, the (c) ` 1,600
3
rest of the capital for whole (d) None of these
time. Find A’s share of the total 8. A bag contains an equal
profit of ` 2,300. number of one rupee, 50 paise
(a) ` 100 (b) ` 200 and 25 paise coins
(c) ` 300 (d) ` 400 respectively. If the total value
4. If 12 men can build a wall 100 is ` 35, how many coins of
m long, 3 m high, and 0.5 m each type are there?
thick in 25 days, in how many (a) 20 coins
days will 20 men build a wall (b) 30 coins
60 m × 4 m × 0.25 m? (c) 28 coins
(a) 6 days (b) 8 days (d) 25 coins
(c) 12 days (d) 15 days 9. Three friends A, B and C
5. In a camp, there is a meal for started a business by
120 men or 200 children. If 150 investing a sum of money in
children have taken the meal, the ratio of 5 : 7 : 6. After 6
how many men will be catered months C withdraws half of
to with the remaining meal ? his capital. If the sum invested
(a) 20 (b) 30 by ‘A’ is ` 40,000, out of a
(c) 40 (d) 50
Ratio, Proportion, Variation and Partnership 85
total annual profit of ` 33,000, would get and the 3rd queen
C’s share will be would get 3/5th of what the
(a) ` 9,000 (b) ` 12,000 2nd queen would get. 1st
(c) ` 11,000 (d) ` 10,000 queen got 60 gold coins more
10. In a journey of 45 km than the 3rd queen. How many
performed by tonga, rickshaw gold coins were distributed
and cycle in that order, the to the three queens?
distance covered by the three (a) 120 (b) 175
ways in that order are in the (c) 225 (d) 250
ratio of 8 : 1 : 3 and charges 12. Anil started a manufacturing
per kilometre in that order are unit with a certain amount of
in the ratio of 8 : 1 : 4. If the money. After a few months,
tonga charges being 24 paise Dheeraj became his partner,
per kilometre, the total cost of contributing three times of
the journey is what Anil had contributed. At
(a) ` 9.24 the end of the year, each was
(b) ` 10 entitled to half the total profit.
(c) ` 12 If Anil started the unit in
(d) None of these January, then when did
11. King Dashrath decided to Dheeraj join as a patner?
distribute gold coins to his (a) August
three queens in the following (b) September
way : 2nd queen would get 5/ (c) July
7th of what the 1st queen (d) October
EBD_7382
86 Ratio, Proportion, Variation and Partnership

Hints & Solution


1. (d) Use Short Approach -8 \ Total population
2. (b) Let r be the 4th propor- = 196000 + 171500 = 367500
tional. 7. (a) Use Short Approach -22
Then 8. (a) Let number of each type
of coin = x. Then,
12 X 3 8a 3 X 4
= Þ r = 6a 1 × x + .50 × x + .25 x = 35
9aX 2 r
Þ 1.75x = 35 Þ x = 20
3. (a) Use Short Approach -21 coins
4. (a) Let the required days be x. 9. (a) Use Short Approach -22
More men, less days
10. (a) Total distances covered
(Indirect proportion)
under each mode = 32, 4
More size, more days
(Direct proportion) and 12 km respectively.
Total charges = 32 × 24
Men 20 12 ü
size 100×3×0.5
ý :: 25 : x
60×4×0.25 þ
+ 4 × 3 + 12 × 12
= 924 paise = ` 9.24.
\ 20 × 100 × 3 × 0.5× x = 12
× 60 × 4 × 0.25 × 25 11. (c) Let x gold coins were
Þ x = 6 days given to 1st queen. Then
5. (b) There is a meal for 200 no. of gold coins given
children. 150 children 5
have taken the meal. to 2nd queen = x
7
Remaining meal is to be And no. of gold coins
catered to 50 children. given to 3rd queen
Now, 200 children º 120
men 3
= x
\ 50 children 7
then,
æ 120 ö
ºç ´ 50 ÷ men = 30 men. 3 4x
è 200 ø x - x = 60 Þ = 60
7 7
6. (b) Number of females x = 105
100 Required no.
= 156800 × = 196000
80 5 3
7 = 105 + ´105 + ´105 = 225
\ Number of males = × 7 7
8
196000 = 171500 12. (b) Use Short Approach -22
09 Average
Chapter

Shortcut Approach - 1
If average of n consecutive odd numbers x, then the difference between the
smallest and the largest numbers is given by 2 (n – 1).
Note that the above formula is independent of x. Therefore, this formula
always holds good irrespective of the value of x.
1. If A, B, C and D are four consecutive odd numbers and their average
is 52, what is the difference between A and D?
(a) 6 (b) 7 (c) 8 (d) 9
Sol. (a) Difference = 2 (n – 1) = 2 (4 – 1 ) = 6

Shortcut Approach - 2
If the average of n numbers is A and on rechecking, it is noticed that some
of the numbers n (i.e. x1, x2, x3, ....., xn) are wrongly taken as (x'1, x'2, x'3,
( x1 + x 2 + x3 + .... + xn ) -
....., x'n), then their correct average = A + ( x '1 + x '2 + x '3 + ..... + x 'n )
n
2. The mean of 50 observations was 36. It was found later that an
observation 48 was wrongly taken as 23. The corrected (new)
mean is:
(a) 35. 2 (b) 36.1 (c) 36. 5 (d) 39.1
Sol. (c) Correct average
48 - 23
= 36 +
50
25
= 36 + = 36.5
50
3. The arithmetic mean of 100 numbers was computed as 89.05. It
was later found that two numbers 92 and 83 have been misread
as 192 and 33 respectively, what is the correct arithmetic mean
of the numbers?
(a) 88.55 (b) 87.55
(c) 89.55 (d) Data Insufficient
EBD_7382
88 Average
Sol. (a) Correct average

= 89.05 +
( 92 + 83) - (192 + 33)
100
175 - 225 50
= 89.05 + = 89.05 - = 89.05 - 0.5 = 88.55
100 100

Shortcut Approach - 3
Let the average weight of n balls is a units. If the weight of a bag is included,
the average weight increases by b units, then weight of the bag = a + b (n +
1).
4. The average run scored by a batsman in 20 innings is 32. After
21st innings, the runs average becomes 34. How much runs does
the batsman score in his 21st innings?
(a) 70 (b) 72 (c) 74 (d) 76
st
Sol. (c) After 21 innings, the runs average increases by 2.
\ Runs scored in 21st innings = a + b (n +1)
= 32 + 2 (20 + 1) = 32 + 42 = 74
5. The average weight of 21 boys was recorded as 64 kg. If the weight
of the teacher was added, the average increased by one kg. what
was the teacher's weight?
(a) 86 kg (b) 64 kg (c) 72 kg (d) 98 kg
Sol. (a) Teacher’s weight = a + b (n + 1)
= 64 + 1 (21 + 1) = 64 + 22
= 86 kg

Shortcut Approach - 4
Let there were n students in a hostel. If the number of students increases
by a, the expenses of mess increase by ` b per day while the average
expenditure per head diminishes by ` c, then the original expenditure of
é c( n + a ) + b ù
the mess = n ê ú
ë a û
6. There were 52 students in a hostel. If the number of students
increases by 13, the daily expenses of the mess increases by `65,
while average expenses per student decreases by `1. Find the
original expenditure of the mess.
(a) `480 (b) `500 (c) `520 (d) `545
Average 89

é c (n + a) + b ù
Sol. (c) Original expenditure = n ê ú
ë a û

é1( 52 + 13) + 65 ù
= 52 ê ú
ë 13 û
130
= 52 ´ = ` 520
13
7. There were 35 students in a hostel. If the number of students
increases by 7, the daily expenses of the mess increases by `42,
while average expenses per student decreases by `1. Find the
original expenditure of the mess.
(a) `420 (b) `500
(c) `520 (d) `545
é1( 7 + 35 ) + 42 ù
Sol. (a) Original expenditure = 35 ê ú
ë 7 û
84
= 35 ´ = ` 420
7

Shortcut Approach - 5
If the average of (a + b + 1) result is x, that of the first a is y and that of
the last b is z, then value of the (n + 1)th result = Total of (a + b + 1)
results – (Total of first a results + Total of last b results)
= (a + b + 1) x – (ay + bz)
8. The average of 9 numbers is 30. The average of first 5 numbers
is 25 and that of the last 3 numbers is 35. What is the 6th
number?
(a) 20 (b) 30
(c) 40 (d) 50
Sol. (c) The value of 6th number
= (a + b + 1) x – (ay + bz)
= (5 + 3 +1) 30 – (5 × 25 + 3 × 35)
= 270 – (125 + 105) = 40

Shortcut Approach - 6
If the average age of m boys is x and the average age of n boys out of m
mx - ny
boys is y, then the average age of the rest of the boys =
m-n
EBD_7382
90 Average
9. The average age of 30 girls is 13 years. The average of first 18
girls is 15 years. Find out the average age of remaining 12 girls.

(a) 12 yrs (b) 10 yrs (c) 16 yrs (d) 10.5 yrs


Sol. (b) Average age
é mx - ny ù
=ê ú
ë m-n û
é 30 ´13 - 18 ´ 15 ù
=ê ú
ë 30 - 18 û
( 390 - 270 )
= = 10 years
12
10. The sum of five numbers is 555. The average of first two numbers
is 75 and the third number is 115. What is the average of last two
numbers?
(a) 145 (b) 290
(c) 265 (d) 150
Sol. (a) Average of last two number
é 555 - ( 75 ´ 2 ) - (115 ´ 1) ù
=ê ú = 145
ë 5 - 2 -1 û

Shortcut Approach - 7
If the average of n quantities is equal to x. When a quantity is removed,
the average becomes y. Then the value of the removed quantity = n (x – y)
+ y.
11. The average age of a family of 6 members is 25 years. The
average age of the family after the demise of a 40 years old
member will be
(a) 20 yrs (b) 21 yrs (c) 22 yrs (d) 23 yrs
Sol. (c) 40 = n (x – y) + y
40 = 6 (25 – y) + y
40 = 150 – 5y
5y = 110
y = 22 years
12. The average monthly income of a family of 4 earning members
was `7350. One of the earning members died. Thereby the average
came down to `6500. The monthly income of the deceased was:
(a) `8500 (b) `8200
(c) `9900 (d) `10750
Sol. (c) Average monthly income = 4 (7350 – 6500) + 6500
= 3400 + 6500 = ` 9900
Average 91

Shortcut Approach - 8
If the average of the first and the second of three numbers is x more or less
than the average of the second and the third of these numbers, then the
difference between the first and the third of these three numbers is given
by 2x.
Note that, here only 2 numbers (i.e. first and second or second and third)
are involved in calculating average, therefore, we multiply x by 2. If n
numbers are involved, for getting answer, we multiply x by n.
13. Average rainfall on Monday, Tuesday, Wednesday and Thursday
is 420.5 cm and average on Tuesday, Wednesday, Thursday and
Friday is 440.5 cm. If the ratio of rainfall for monday and Friday
is 20:21, find the rainfall in cm on Monday and Friday.

(a) 1700, 1470


(b) 1682, 1762
(c) 1800, 1890 (d) 1600, 1680
Sol. (d) The difference between the rainfall on Monday
and Friday = 4 × (440.5 – 420.5) = 80
So, 21 x – 20x = 80 \ x = 80
Hence, rainfall on Monday and Friday
= 20 × 80 and 21 × 80
= 1600 cm and 1680 cm
14. The mean temperature from Monday to Wednesday was 37°C and
from Tuesday to Thursday was 34°C. If the temperature on
Thursday was 4/5th that of Monday, the temperature on Thursday
was:
(a) 34°C (b) 35°C
(c) 36°C (d) 37°C
Sol. (c) Difference between the temperature on Monday
and Thursday = 3 × (37°C – 34°C) = 9°C
4x
So, x - = 9 Þ x = 45°C
5
Hence, temperature on Thursday
4
= ´ 45° = 36°C
5

Shortcut Approach - 9
If a batsman in his nth innings makes a score of x and thereby increases
his average by y, then the average after n innings = x – y (n – 1).
EBD_7382
92 Average
15. Sachin Tendulkar has a certain average for 11 innings. In the
12th innings, he score 120 runs and there by increases his
average by 5 runs. His new average is
(a) 60 (b) 62
(c) 65 (d) 66
Sol. (c) Average = x – y (n – 1)
= 120 – 5 (12 – 1)
= 120 – 5 × 11
Þ 120 – 55 = 65
16. A cricketer had a certain average of runs for his 64 innings. In
his 65th innings, he is bowled out for no score on his part, This
brings down his average by 2 runs his new average of runs is:

(a) 130 (b) 128


(c) 70 (d) 68
Sol. (b) Average = x – y (n – 1)
= 0 – (– 2)(65 – 1)
= 0 + 2 × 64 = 128

Shortcut Approach - 10
If a cricketer has completed n innings and his average is x runs. The number of
runs, he must make in his next innings so as to raise his average to y = n (y – x) + y.
17. A cricketer has completed 10 innings and his average is 21.5
runs. How many runs must be made in his next innings so as to
raise his average to 24?
(a) 47 (b) 48
(c) 49 (d) 50
Sol. (c) Required runs
= n (y – x) + y
= 10 (24 – 21.5) + 24 = 49 runs

Shortcut Approach - 11
The average of marks obtained by n candidates in a certain examination is
T. If the average marks of passed candidates is P and that of the failed
candidates is F. Then the number of candidates who passed the examination

Total candidates
(Total Average – Failed Average) n (T - F )
= =
Passed Average – Failed Average P-F
Average 93
18. The average of marks obtained by 120 candidates in a certain
examination is 35. If the average mark of passed candidates is 39
and that of the failed candidates is 15, what is the number of
candidates who passed the examination?
(a) 90 (b) 85
(c) 100 (d) 120
Sol. (b) No. of passed students

n ( T - F ) = 140 ( 35 - 20 )
= = 105
P-F ( 40 - 20 )
No. of failed students
= 140 – 105 = 35

Shortcut Approach - 12
The average weight of n persons is increased by x kg. when some of them
whose weight [y1 + y2 + ...... where, y1 + y2 + ...... = y kg] are replaced by
the same number of persons. Then the weight of the new persons
= Weight of removed person (s) + (No. of persons) ×
(Increase in average)
= y + nx.
19. The average age of 8 persons is increased by 2 years, when one
of them, whose age is 24 years, is replaced by a new person. The
age of the new person is:
(a) 42 yrs (b) 40 yrs
(c) 38 yrs (d) 45 yrs
Sol. (b) The age of the new man = y + nx
= 24 + 8 × 2 = 40 years
20. The average age of 8 men is increased by 2 years when two of them
whose ages are 21 and 23 years are replaced by two new men. The
average age of the two new men is:
(a) 22 yrs (b) 24 yrs
(c) 28 yrs (d) 30 yrs
Sol. (d) The average of the two new men is

=
Total age of new persons ( y + nx )
=
No. of new persons 2

( 21 + 23) + 8 ( 2 ) =
60
= 30 years
=
2 2
EBD_7382
94 Average

Shortcut Approach - 13
The average age of n persons is decreased by x years. When some of them
aged [y1 + y2 + ..... where, y1 + y2 + .... = y years] are replaced by the same
number of persons. Then the total age of the new persons
= Total age of removed persons – (No. of persons) × (Decrease in average)
= y – nx.
21. Out of 10 teachers of a school, one teacher retires and in his
place, a new teacher of age 25 years joins. As a result, average
age of teachers is reduced by 3 years. The age (in years of the
retired teacher is:
(a) 50 (b) 58 (c) 60 (d) 55
Sol. (d) Let, the age of the retired teacher = x
25 = x – (10 × 3)
x = 25 + 30 = 55 years

Shortcut Approach - 14
If the average of n results (where n is an odd number) is a and the average
n +1 n +1
of first results is b and that of last is c.
2 2

æ n + 1ö
Then çè ÷ th result
2 ø

n +1
= (b + c ) - na.
2
or (the middle value)
22. The average of 11 numbers is 10.8. If the average of the first six
numbers be 10.4 and that of the last six is 11.5, the middle (6th)
number is:
(a) 10.3 (b) 12.6 (c) 13.5 (d) 15.5
n +1
Sol. (b) Middle number (6th) = ( b + c ) - na
2
11 + 1
= (10.4 + 11.5 ) - 11´10.8
2
= 131.4 – 118.8 = 12.6
Average 95

Shortcut Approach - 15
If the average value of all the members of group is x, the average value of
the first part of member is y, the average value of the remaining part of
members is z and the number of the first part of members is n, then the
n( x - y )
number of the other part of members is .
z-x
or

No. of 1st part members


(Total average of whole group -
Average of 1st part)
=
Average of other part - Total
average of whole group

23. The average salary of the entire staff in a office is ` 120 per
month. The average salary of officers is ` 460 and that of non-
officers is ` 110. If the number of officers is 15, then find the
number of non-officers in the office.
(a) 510 (b) 520 (c) 530 (d) 540
Sol. (a) No. of non-officers
15 (120 - 460 ) 15 ´ ( -340 )
= = = 15 × 34 = 510
110 - 120 ( -10 )
EBD_7382
96 Average

Exercise
1. The average age of 24 (a) 8.8 km/h
students and the class
teacher is 16 years. If the class (b) 9.6 km/h
teacher’s age is excluded, the (c) 10.2 km/h
average reduces by one year.
What is the age of the class (d) 11.4 km/h
teacher? 5. The average marks of 40
(a) 50 years (b) 45 years students in an English exam
(c) 40 years is 72. Later it is found that
(d) Data inadequate three marks 64, 62 and 84 were
2. A person covers half his wrongly entered as 68, 65 and
journey by train at 60 kmph, 73. The average after
the remaining half by bus at mistakes are rectified as–
30 kmph and the rest by cycle (a) 70 (b) 72
at 10 kmph. Find his average (c) 71.9 (d) 72.1
speed during the entir e 6. The average of 7, 14, 21,
journey. 28..........77 is :
(a) 36 kmph
(a) 7 (b) 11
(b) 24 kmph
(c) 48 kmph (c) 42 (d) 66
(d) None of these 7. Three years ago, the average
3. The average of 10 numbers is age of a family of 8 members
40.2. Later it is found that two was 30 years. If one child is
numbers have been wrongly also included in the family,
copied. The first is 18 greater the present average age of
than the actual number and the family remained the same
the second number added is as three years ago. The
13 instead of 31. Find the present age of the child is
correct average. (a) 6 yrs (b) 1 year
(a) 40.2 (b) 40.4 (c) 3 yrs (d) 4 yrs
(c) 40.6 (d) 40.8 8. The average salary of all the
workers in a workshop is
4. Rahul Ghosh walks from A to
`8,000. The average salary of
B at 8 km/h and comes back
7 technicians is `12,000 and
from B to A at 12 km/h. What
the average salary of the rest
is his average speed for the
is ` 6,000. The total number of
entire journey?
workers in the workshop is :
Average 97
(a) 21 (b) 20 11. Brian Lara, the famous
(c) 23 (d) 22 batsman, scored 6,000 runs in
9. A car covers 1/5 of the certain number of innings. In
distance from A to B at the the next five innings he was
speed of 8 km/hour,1/10 of the out of form and hence, could
distance at 25 km per hour and make only a total of 90 runs,
the remaining at the speed of as a result of which his
average fell by 2 runs. How
20km per hour. Find the
many innings did he play in
average speed of the whole
all, if he gets out in all the
journey- innings?
(a) 12.625 km/hr (a) 105 (b) 95
(b) 13.625 km/hr (c) 115 (d) 104
(c) 14.625 km/hr 12. The batting average for 40
(d) 15.625 km/hr innings of a cricketer is 50
10. The average weight of 47 runs. His highest score
balls is 4 gm. If the weight of exceeds his lowest score by
the bag (in which the balls 172 runs. If these two innings
are kept) be included, the are excluded, the average of
calculated average weight the remaining 38 innings is 48
increases by 0.3 gm. What is runs. The highest score of
the weight of the bag? the player is-
(a) 14.8 gm (b) 15.0 gm (a) 165 (b) 170
(c) 18.6 gm
(c) 172 (d) 174
(d) None of these
EBD_7382
98 Average

Hints & Solution


1. (c) Use short approach -7
2. (b) Recognise that the Alternatively : Since all
journey by bus and that the numbers are in A.P
Further there are odd
by cycle are of equal number of numbers
distance. Hence, we can (i.e., 11) in the sequence.
use the short cut Thus the middle most
illustrated earlier to term is the average of
solve this part of the the sequence, which is
42.
problem. 7. (a) The present average
Using the process age of 8 members
explained above, we get will be 33 years.
average speed of the Let the present age of
second half of the child be ‘x’. So,
journey as 8 ´ 33 + x
= 30 Þ x
10 + 1 × 5 = 15 kmph 8 +1
Then we employ the = 6 years.
same technique for the 8. (a) Use short approach -6
first part and get 15 + 1 9. (d) If the whole journey be
× 9 = 24 kmph x km. The total time
taken
3. (a) Use short approach -2
4. (b) Average speed æx x 7x ö
ç5 10 10 ÷
2 ´ 8 ´12 192 =ç + + ÷ hrs
= =
8 + 12 20 çç 8 25 20 ÷
÷
= 9.6 km/h è ø
5. (d) Use short approach -2 = (x/40 + x/250 + 7x/
200) hrs
7 + 14 + 21 + ....77 = (25x + 4x + 35x)/1000
6. (c)
11 = 64x/1000 hrs
7 (1 + 2 + ......11) x
= \ Average speed =
11 64 x
7 ´ 11 ´ 12 1000
= = 42 = 15.625 km/hr
11 ´ 2
10. (d) Use short approach -3
é n ( n + 1) ù 11. (a) Use short approach -10
êëQ1 + 2 + 3........n = 2 úû 12. (d) Use short approach -2
10 Percentage
Chapter

Shortcut Approach - 1
a
If a is x% of b, then x = ´ 100%
b
1. 50 kg is what percent of 250 kg?
(a) 26% (b) 15%
(c) 20% (d) 18%
æ 50 ö
Sol. (c) Required percent = ç ´ 100 ÷ % = 20%
è 250 ø

Shortcut Approach - 2
If the price of a commodity increases by r%, then the reduction in
æ r ö
consumption so as not to increase the expenditure, is ç ´ 100 ÷ %
è 100 + r ø
2. The price of sugar is increased by 25%. If a family wants to keep its
expenses on sugar unaltered, then the family will have to reduce the
consumption of sugar by
(a) 20% (b) 21%
(c) 22% (d) 25%
Sol. (a) Required percentage
æ r ö
=ç ´100 ÷ %
è 100 + r ø

æ 25 ö
=ç ´ 100 ÷ % = 20%
è 100 + 25 ø
3. The price of cooking oil is increased by 25%. By what percent a
family should reduce the consumption of cooking oil so as not to
increase the expenditure on this account?
(a) 20 (b) 25 (c) 50 (d) 40
EBD_7382
100 Percentage
Sol. (a) Required percentage
æ 25 ö æ 25 ö
=ç ´100 ÷ % = ç ´ 100 ÷ % = 20%
è 100 + 25 ø è 125 ø
4. The price of commodity rises from `6 per kg to `7.50 per kg. If the
expenditure cannot increase, then percentage of reduction in
consumption is :
(a) 15 (b) 20 (c) 25 (d) 30
Sol. (b) % increase in price

æ (7.50 - 6) ö æ 1.50 ö
=ç ´ 100 ÷ % = ç ´ 100 ÷ % = 25%
è 6 ø è 6 ø
\ Required percentage
æ 25 ö
=ç ´ 100 ÷ % = 20%
è 100 + 25 ø

Shortcut Approach - 3
If the price of a commodity decreases by r%, then, increase in consumption,
é r ù
so as not to decrease expenditure on this item is ê 100 - r ´ 100ú % ,
ë( ) û
5. If the price of tea falls down by 6% by how much percent must a
householder increase its consumption, so as not to decrease
expenditure?
16 18 18 17
(a) 5 % (b) 4 % (c) 6 % (d) 6 %
47 67 47 47
Sol. (c) Required percentage
æ r ö æ 6 ö 18
=ç ´100 ÷ % = ç ´100 ÷ % = 6 %
è 100 - r ø è 100 - 6 ø 47

Shortcut Approach - 4
If the price of a commodity is changed by x% and its consumption changed
by y%, then

New expenditure
=
(100 - x )(100 + y )
Initial expenditure (100) 2
Percentage 101

Put x as ( + x ) and y as ( + y ) in the case of 'increase' and x as ( - x )

and y as ( - y ) in the case of 'decrease'.


6. The price of consumer goods increased by 50% and its consumption
decreased by 25%. Find the ratio of new expenditure to initial
expenditure.
(a) 8 : 9 (b) 9 : 8 (c) 7 : 8 (d) 11 : 8
Sol. (b) Required ratio
New expenditure (100 + 50)(100 - 25)
= =
Initial expenditure (100)2

150 ´ 75 9
= = = 9:8
2 8
(100)

Shortcut Approach - 5
If three successive discounts of x%, y% and z% are allowed on an
amount then a single discount that equivalent to the three successive
é xy + yz + zx xyz ù
discounts will be êë x + y + z - 100
+
1002 úû
%.

or
Finding single discount rate equivalent to the series of two discounts
= 1st discount + 2nd discount
1st discount ´ 2nd discount

100
7. Successive discount of 50% and 50% is equivalent to
(a) 100% (b) 75% (c) 50% (d) 25%
Sol. (b) Required equivalent single discount
é (50 ´ 50) ù
= ê (50 + 50) - %
ë 100 úû
= (100 – 25)% = 75%

Shortcut Approach - 6
If the value is increased successively by x% and y% then the final
é xy ù
increase is given by ê x + y + %,
ë 100 úû
EBD_7382
102 Percentage
8. A number is increased by 20% and then again by 20%. By what per
cent should the increased number be reduced so as to get back the
original number?
5 11
(a) 30 % (b) 19 % (c) 40% (d) 44%
9 13
Sol. (a) % Increase in number
æ 20 ´ 20 ö
= ç 20 + 20 + ÷%
è 100 ø
= (40 + 4) % = 44%
Hence, the increased number should be reduced by 44% so as to get
back the original number.

Shortcut Approach -7
If the value is decreased successively by x% and y% then the final
é xy ù
decrease is given by ê x + y - %,
ë 100 úû
9. The population of a city is decreased by 20% in a year. In the next
year, the population is again decreased by 30%. Find per cent decrease
in population of the city from the original population.
(a) 44% (b) 50% (c) 55% (d) 56%
Sol. (a) Population is decreased by
é 20 ´ 30 ù æ 600 ö
= ê 20 + 30 - % = ç 50 - ÷ % = 44%
ë 100 úû è 100 ø

Shortcut Approach - 8
If the value is first increased by x% and then decreased by y%, then
æ xy ö
there is ç x - y - ÷ % increase or decrease, according to the +ve or
è 100 ø
–ve sign respectively.
10. A number is increased by 20% and then it is decreased by 10%. Find
the net increase or decrease per cent.
(a) 10 % increase
(b) 10% decrease
(c) 8% increase
(d) 8% decrease
Percentage 103
Sol. (c) Change in the number
æ 20 ´10 ö
= æç x - y -
xy ö
÷ % = ç 20 - 10 - 100 ÷ %
è 100 ø è ø
= 8% increase
11. If price of a book is first decreased by 25% and then increased by
20%, the net change in the price of the book will be :
(a) 10 % decrease (b) 5% decrease
(c) No change (d) 5% increase
Sol. (a) Change in the price
æ 20 ´ 25 ö
= ç 20 - 25 - ÷%
è 100 ø
= (–5 – 5)% = (–10)%
= 10% decrease

Shortcut Approach - 9
A man spends x% of his income. His income is increased by y% and his
expenditure also increases by z%, then the percentage increase or
decrease in his savings is given by
é 100 y - xz ù
êë 100 - x úû % ,
according to the +ve or –ve sign.
12. A man spends 75% of his income. His income is increased by 20% and
he increases his expenditure by 10%. His savings are increased by :
(a) 10% (b) 25% (c) 37% (d) 50%
Sol. (d) % increase in savings
é (100 ´ 20) - (75 ´100) ù æ 2000 - 750 ö
= êë 100 - 75 ú% = ç ÷ % = 50%
û è 25 ø
13. The monthly income of a person was `13500 and his monthly
expenditure was `9000. Next year his income increased by 14% and
his expenditure increased by 7%. The percentage of increase in his
savings is :
(a) 7% (b) 21% (c) 28% (d) 35%
Sol. (c) Percentage expenditure
æ 9000 ö 200
=ç ´100 ÷ % = %
è 13500 ø 3
\ % increase in saving
EBD_7382
104 Percentage

é æ 200 ö ù é 2800 ù
ê (100 ´14) - ç 3 ´ 7 ÷ ú
è ø ú % ê 3 ú % = 28%
= ê =ê ú
ê 100 -
200 ú ê 100 ú
ê 3 ú ë 3 û
ë û

Shortcut Approach - 10
The pass marks in an examination is x%. If a candidate who secures y
marks fails by z marks, then the maximum marks, is given by
é 100 ( y + z ) ù
ê ú.
ë x û
14. In an examination a candidate must secure 40% marks to pass. A
candidate who gets 220 marks, fails by 20 marks. What are the maximum
marks for the examination?
(a) 1200 (b) 800 (c) 600 (d) 450
Sol. (c) Maximum marks
100(220 + 20)
= = 600
40

Shortcut Approach - 11
Theorem : If one factor is increased by x% and the other increases by

y% then the effect on the product is given by é x + y + xy ù % increase.


ê 100 úû
ë
15. The number of seats in a cinema hall is increased by 25%. The cost of
a ticket is also increased by 10%. The overall percentage increase in
the revenue is :
(a) 10. 5% (b) 27. 5% (c) 37. 5% (d) 40. 5%
Sol. (c) Overall percentage increase in revenue
é xy ù
= êx + y + %
ë 100 úû

é 250 ù
= ê 25 + 10 + %
ë 100 úû
= 37.5%
Percentage 105

Shortcut Approach - 12
If one of the sides of a rectangle is increased by x% and the other is
decreased by y%, then the increase or decrease per cent in area is given
æ xy ö
by ç x - y - ÷ % , according to the +ve or –ve sign.
è 100 ø
16. If the length is increased by 20% and the breadth is decreased by
25%, then what will be the effect on the area?
(a) 10% decrease
(b) 10% increase
(c) 20% increase
(d) 20% decrease
Sol. (a) Effect on area
æ xy ö æ 20 ´ 25 ö
= çx- y- ÷ % = çè 20 - 25 - ÷%
è 100 ø 100 ø
= ( -10)% = 10% decrease

Shortcut Approach - 13
If one of the sides of a rectangle is increased by x% and the other is
increased by y% then the per cent value by which area changes is given
é xy ù
by ê x + y + % increase.
ë 100 úû
17. If the sides of a square are increased by 30%, find the per cent increase
in its area.
(a) 70% (b) 68% (c) 69% (d) 71%
Sol. (c) % increase in are
é 30 ´ 30 ù
= ê30 + 30 + % = (60 + 9)% = 69%
ë 100 úû
EBD_7382
106 Percentage

Exercise
1. If the price of sugar is expenditure to initial
increased by 7%, then by expenditure.
how much per cent should a (a) 8 : 9 (b) 9 : 8
housewife reduce her (c) 7 : 8 (d) 11 : 8
consumption of sugar, to
6. 10 litres of water is added to
have no extra expenditure?
50 litres of a solution
(a) 7 over 107%
containing 20% of alcohol in
(b) 107 over 100%
water. What is the
(c) 100 over 107%
concentration of alcohol in
(d) 7%
the solution now?
2. A positive number is by
mistake divided by 6 instead 2
of being multiplied by 6. What (a) 20% (b) 16 %
3
is the % error on the basis of
correct answer? 1 1
(c) 12 % (d) 33 %
(a) 3 (b) 97 2 3
(c) 17 (d) 83
7. p is six times as large as q.
3. When the price of a radio was The percent that q is less
reduced by 20%, its sale than p, is :
increased by 80%. What was
the net effect on the sale? 2
(a) 16 (b) 60
(a) 44% increase 3
(b) 44% decrease
(c) 66% increase 1
(c) 83 (d) 90
(d) 75% increase 3
4. A store raised the price of an 8. In a class, the no. of boys is
item by exactly 10 per cent. more than the no. of girls by
Which of the following could 12% of the total strength. The
not be the resulting price of ratio of boys to girls is :
the item ? (a) 15 : 11 (b) 11 : 14
(a) ` 5.50 (b) ` 7.60 (c) 14 : 11 (d) 8 : 11
(c) ` 11.00 (d) ` 12.10
5. The price of consumer goods 9. In some quantity of ghee,
increased by 50% and its 60% is pure ghee and 40% is
consumption decreased by vanaspati. If 10 kg of pure
25%. Find the ratio of new ghee is added, then the
Percentage 107
strength of vanaspati ghee 11. In an examination 80% of
becomes 20%. The original student passed in English
quantity was : 85% in mathematics and 75%
(a) 10 kg (b) 15 kg in both English and
(c) 20 kg (d) 25 kg mathematics. If 40 student
10. The ratio of salary of a worker failed in both the subject find
in July to that in June was total number of students.
(a) 350 (b) 400
1 1
2 : 2 , by what % the (c) 450 (d) 600
2 4 12. The monthly salary of Shahid
salary of July more than salary and Kareena together is $ 28,
of June. Also find by what %, 000. The salary of Shahid and
salary of June was less than Kareena is increased by 25%
that of July. and 12.5% respectively then
1 the new salary of Kareena
(a) 11 % and 10% becomes 120% of the new
9
salary of Shahid. The new (or
1 increased) salary of Shahid is :
(b) 10% and 11 %
9 (a) $ 15,000
(c) Both 10% (b) $ 18,000
1 (c) $ 14,000
(d) Both 11 % (d) $ 16,000
9

Hints & Solution


1. (a) Use Short Approach -2 5. (b) Use Shortcut Approach - 8
2. (b) Let the number be x. 6. (b) Alcohol in solution
Then, = 20% of 50l = 10l
% error Concentration of
6x - x / 6 alcohol in new solution
= ´ 100 10 2
6x = ´ 100 = 16 %
60 3
35 7. (c) Use Shortcut Approach - 4
= ´ 100 = 97.2%
36 8. (c) Boys Girls Total
3. (a) Use Short Approach -6
4. (b) 5 + 10% = 5.50 14 11 25
10 + 10% = 11 56 44 100
11 + 10% = 12.10 12
EBD_7382
108 Percentage
Since out of 100
students number of 5 9
x- x
boys are greater than 2 4 ´100
the numbers of girls by 5
x
12 i.e., 12%. 2
Alternatively:
Let the number of boy 100 100
and girls be x and y = % and %
9 10
respectively then
(x – y) 1
= 11 % and 10%
12 ´ ( x + y ) x 14 9
= Þ =
100 y 11 11. (b) Use Shortcut Approach - 16
9. (a) Let the original
quantity be x kg. S K
Vanaspati ghee in x kg
15, 000 18, 000
12. (a)
æ 40 ö 12, 000 16, 000
= ç x ÷ kg
è 100 ø
going in the reverse
æ 2x ö direction.
= ç ÷ kg.
è 5 ø Please option (a) is
2x correct
20 Alternatively :
Now, 5 =
x + 10 100
Shahid Kareena
2x 1 48x
Û = 4x
5 x + 50 5 9
25%
Û 5x = 50 Û x = 10 5x 6x
10. (a) Let the salary of July be
+120%
5
` x
2 Þ Initially
and the salary of June be 48x
S : K = 4x : =3: 4
9 9
` x.
4 Þ Shahid's initial salary
Required percentages = $ 12, 000
5 9 Shahid's changed
x- x salary = $ 15000
= 2 4 ´ 100 and
9
x
4
11 Profit, Loss and
11
Chapter Discount

Shortcut Approach - 1
If an item is bought at the rate of X items for a rupee, then the number of
é æ 100 ö ù
items sold for a rupee in order to gain x% is ê X ç ÷ú .
ë è 100 + x ø û
1. A man bought toffees at the rate of 15 for a rupee and sold them
at the rate of 12 for a rupee. His gain percent is
(a) 20% (b) 25%
(c) 33.33% (d) None of these
é 100 ù
Sol. (b) \ Item rate sold price = X ê
ë100 + x úû

é 100 ù
12 = 15 ê ú
ë100 + x û
12 [100 + x] = 1500
1500
[100 + x] =
12
1500 - 1200
x=
12
300
x= = 25%
12
2. A fruit seller buys lemons at 2 for a rupee and sells them at 5 for
three rupees. What is his gain percent?
(a) 10% (b) 15% (c) 20% (d) 25%
é 100 ù
Sol. (c) \ Item rate sold price = x ê ú
ë100 + x û
5 é 100 ù
= 2ê ú
3 ë100 + x û
EBD_7382
110 Profit, Loss and Discount

(100 + x ) = éê
200 ù
ú
ë5 / 3û
600 - 500 100
x= = = 20%
5 5

Shortcut Approach - 2
If a man purchases 'a' items for ` 'b' and sells 'c' items for ` 'd', then the
gain or loss [depending upon the respective (+ve) or (–ve) sign in the final

é ad - bc ù
result] made by him is ê ×100ú %
ë bc û
3. A person purchased 11 articles for ` 10 and at 10 articles for `11.
Find the gain percentage.
(a) 22 (b) 20
(c) 1 (d) 21
Sol. (d) \ Profit or loss
é ad - bc ù é11´11 - 10 ´10 ù
=ê ´100% ú = ê ´ 100 ú Þ 21 %
ë bc û ë 10 ´ 10 û

Shortcut Approach - 3
When there are two successive profits of x% and y %, then the
æ xy ö
resultant profit per cent is given by çè x + y + ÷.
100 ø
4. A sells a cycle to B at 20% profit. Then B sells it to C at 25%
profit. If C pays ` 225, then what is the cost price of cycle for A?
(a) `100 (b) `125 (c) `150 (d) `175
Sol. (c) \ Resultant profit

é x.y ù = é20 + 25 + 20 ´ 25 ù
= êx + y + ê
ë 100 úû ë 100 úû
= 50 %

æ 100 ö . = æ 100 ö ´ 225 = 100 ´ 225


\ C.P. = ç ÷ ´ S P çè 100 + 50 ÷ø
è 100 + P ø 150
C.P. = `150
Profit, Loss and Discount 111

Shortcut Approach - 4
If a dishonest trader professes to sell his items at CP but uses false
measurement, then

True measurement - False measurement


% gain = ×100
False measurement
5. A retailer profess to sell his goods at cost price. If using a false
weight, he still gains 25%, find the weight he uses in place of 1
kg.
(a) 200 g (b) 600 g (c) 700 g (d) 800 g
é True weight - False weight ù
Sol. (d) Profit % = ê ú ´100
ë False weight û

é1000 - False weight ù


25 = ê ú ´ 100
ë False weight û
25 false weight = 100000 – 100 false weight
125 false weight = 100000
false weight = 800

Shortcut Approach - 5
If a shopkeeper sells his goods at x% profit and uses a measurement
which is y% less, then
Total percentage profit

% profit + % less in wt
= ×100
100 - % less in wt
6. A trader sells wheat at 20% profit and uses weight 20% less than
the actual measure. His gain percent is
(a) 50% (b) 25%
(c) 10% (d) 15%
Sol. (a) \ Total percentage profit
é Profit% - loss in wt% ù
=ê ú ´ 100
ë 100 - loss in wt% û

æ 20 + 20 ö 40
=ç ÷ ´ 100 = ´ 100 = 50% ( Profit )
è 100 - 20 ø 80
EBD_7382
112 Profit, Loss and Discount

Shortcut Approach - 6
If the shopkeeper sells his goods at x% loss on cost price but uses y gm
z
instead of z gm, then his % profit or loss is [100 - x ] - 100 according as
y
the sign is +ve or –ve.
7. A merchant professes to sell goods at the loss of 5% but uses
weight of 900 grams in place of one kilogram, what is his profit
percent?
5
(a) 5% (b) 5 %
19
(c) 6% (d) 10%
z
Sol. (b) Profit % = [100 - x ] - 100
y

é 1000 ù 10
= ê[100 - 5] - 100ú = 95 ´ - 100
ë 900 û 9
950 - 900 50 5
Þ = =5 %
9 9 9

Shortcut Approach - 7
A dishonest dealer sells the goods at x% loss on cost price but uses y%
é y- x ù
less weight, then his percentage profit or loss is ê ×100 ú
ë 100 - y û
according as the sign is +ve or –ve.
8. A dishonest dealer sells his goods at 10% loss on cost price and
uses 30% less weight. What is his profit or loss percent?
4 4
(a) 28 %profit (b) 28 % profit
7 7
4 4
(c) 26 % profit (d) 26 % loss
7 7
æ y-x ö
Sol. (b) çè\ Profit or loss = 100 - y ´ 100ø÷

æ 30 - 10 ö 2000 4
=ç ÷ ´100 = = 28 % ( profit )
è 100 - 30 ø 70 7
Profit, Loss and Discount 113

Shortcut Approach - 8
If a seller uses 'X' gm in place of one kg (1000 gm) to sell his goods and
gains a profit of x% on cost price, then his actual gain or loss percentage

is éê(100 + x ) é1000 ù - 100 ùú according as the sign is +ve or –ve.


ê ú
ë ë X û û
9. A merchant professes to sell goods at 20% profit but uses weight
of 800 grams in place of a kilogram. What is his actual profit percent?
(a) 20% (b) 25% (c) 50% (d) 33.3%
é1000 ù
Sol. (c) \ Profit or loss = (100 + x ) ê - 100
ë X úû
é1000 ù 120 ´ 5 - 400 200
= (100 + 20 ) ê ú - 100 = =
ë 800 û 4 4
= 50% (Profit)

Shortcut Approach - 9
A man purchases a certain number of articles at x a rupee and the same
number at y a rupee. He mixes them together and sells them at z a rupee.
é 2 xy ù
then his gain or loss % = ê – 1ú ´ 100 according as the sign is
êë z ( x + y ) úû
+ ve or - ve .
10. A man purchases a certain number of oranges at 4 a rupee and
the same number of oranges at 5 a rupee. He mixes them together
and sells them at 4 a rupee. What is his gain or loss percent?
1 1
(a) 10 % loss (b) 10 % gain
9 9
1 1
(c) 11 % gain (d) 11 % loss
9 9

é 2 xy ù
Sol. (c) \ Profit or loss = ê - 1ú ´100
êë z ( x + y ) úû

é 2´ 4´5 ù
=ê - 1ú ´ 100
ëê 4 ( 4 + 5 ) ûú

é 40 - 36 ù 4 100 1

36 ú ´100 = 36 ´ 100 = 9 = 11 9 % ( Profit )
ë û
EBD_7382
114 Profit, Loss and Discount

Shortcut Approach - 10
If a tradesman marks his goods at x% above his cost price and allows
æ xy ö
purchasers a discount of y% for cash, then there is çè x - y - ÷%
100 ø
profit or loss according to +ve or –ve sign respectively.
x2
Note : When x = y, then formula becomes -
100
– ve sign indicates that there will be always loss.
11. What is the percentage discount (approximately) that a merchant
can offer on his marked price so that he ends up selling at no
profit or loss, if he initially marked his goods up by 40%?
[SSC CGL Tier II 2014]
(a) 33.5 % (b) 28.5%
(c) 60% (d) No discount
Sol. (b) Let the discount % = y %
æ xy ö
Profit or loss = ç x - y - ÷
è 100 ø

æ 40y ö
0 = ç 40 - y - ÷
è 100 ø
40y
y+ = 40
100
4000
y= = 28.5%
140

Shortcut Approach - 11
If a man buys two items A and B for ` P, and sells one item A so as to lose
x% and the other item B so as to gain y%, and on the whole he neither
gains nor loses, then
æ Py ö
(i) the cost of the item A is ç
è x + y ÷ø
and

æ Px ö
(ii) the cost of the item B is ç .
è x + y ÷ø
Profit, Loss and Discount 115
12. A man purchased two cows for `500. He sells the first at 12% loss
and the second at 8% gain. In this bargain he neither gains nor
loses. Find the selling price of each cow.
(a) `176, `324 (b) `324, `350
(c) `150, `300 (d) `184, `276
æ Py ö
Sol. (a) The cost of first cow is = ç
è x + y ø÷

æ 500 ´ 8 ö 4000
=ç ÷= = 200
è 8 + 12 ø 20

æ Px ö
The cost of second cow is = ç
è x + y ø÷

æ 500 ´ 12 ö 6000
=ç ÷ = = 300
è 12 + 8 ø 20
So, selling price of first cow
200 (100 - 12 )
S.P. = = ` 176
100

300 ´ (100 + 8 )
Selling price of second cow = = ` 324
100

Shortcut Approach - 12
By selling a certain item at the rate of 'X' items a rupee, a man loses x%.
If he wants to gain y%, then the number of items should be sold for a

éæ 100 - x ö ù
rupee is êçè 100 + y ÷ø X ú
ë û
13. By selling 9 articles for a rupee, a man incurred a loss of 4%. To
make a gain of 44%, the number of articles to be sold for a rupee
is
(a) 5 (b) 3 (c) 4 (d) 6
Sol. (d) \ Number of items

éæ 100 - x ö ù é 100 - 4 ù
= êç ÷ Xú = ê ú´9 = 6
ëè 100 + y ø û ë100 + 44 û
EBD_7382
116 Profit, Loss and Discount

Shortcut Approach - 13
When each of the two commodities is sold at the same price
` A, and a profit of P % is made on the first and a loss of L% is made on
100 ( P - L ) - 2 PL
the second, then the percentage gain or loss is (100 + P ) + ( 100 - L )

according to the +ve or –ve sign.


14. A dealer sold two T.V. sets for `7400 each.
On one he gained 10% and on the other he lost 10%. The dealers
loss or gain in the transaction is
(a) no profit, no loss (b) 1% gain
(c) 0.1% loss (d) 1% loss

100 ( P - L ) - 2PL
Sol. (d) \ % Profit =
(100 + P ) + (100 - L )
100 (10 - 10 ) - 2 ´10 ´ 10 -200
= = = -1
(100 + 10 ) + (100 - 10 ) 200
So, negative sign shows the 1% loss in the transaction.

Shortcut Approach - 14
If a dealer sells an item for ` A, making a profit of x%, and he sells
another item at a loss of y%, and on the whole he makes neither profit

é æ 100 ö x ù
nor loss, then the cost of the second table is ` ê A çè ÷ ú.
ë 100 + x ø y û
15. A dealer sells a table for `400, making a profit of 25%. He sells
another table at a loss of 10%, and on the whole he makes neither
profit nor loss. What did the second table cost him?
(a) `600 (b) `800
(c) `700 (d) `900

é 100 ù x
Sol. (b) \ The cost of second table = A ê ú
ë100 + x û y

é 100 ù 25
= 400 ê ú = ` 800
ë100 + 25 û 10
Profit, Loss and Discount 117

Shortcut Approach - 15
If a person buys an article with x per cent discount on the marked price
and sells the article with y per cent profit on the marked price, then his
profit per cent on the price he buys the article is given by
æ x+ y ö
çè ÷ ×100 per cent.
100 - x ø
16. Sweta bought an article with 20% discount on the labelled price.
She sold the article with 8% profit on the labelled price. What was
her percent profit of the price she bought?
(a) 35% (b) 25% (c) 20% (d) 15%
é x+y ù
Sol. (a) \ Profit = ê ú ´100
ë100 - x û

é 20 + 8 ù 2800
=ê ú ´100 = 80 = 35%
ë100 - 20 û

Shortcut Approach - 16
A certain company declares x per cent discount for wholesale buyers. If
a person buys articlesfrom thecompany for ` A after getting discount.
He fixed up the selling price of the article in such a way that he earned a
profit y% on original company price. Then the total selling price is given

æ 100 + y ö
by ` A ç ÷.
è 100 - x ø
17. Surdeep bought a machinery for `800 after getting a discount of
20% from the company. He fixed up the selling price of garments
in such a way that he earned a profit of 10% on the original
company price. Find the selling price of machinery for Surdeep?
(a) `900 (b) `1000
(c) `1100 (d) `1200

é100 + y ù
Sol. (c) \ Total selling price = A ê ú
ë100 - x û

é 100 + 10 ù 800 ´ 110


= 800 ê ú = = ` 1100
ë100 - 20 û 80
EBD_7382
118 Profit, Loss and Discount

Shortcut Approach - 17
A businessman marks an article at ` A and allows x % discount (on the
marked price). He gains y %. If the cost price of the article is ` B, then the
selling price of the article can be calculated from the equation given below
A (100 - x ) B (100 + y )
Selling price = =
100 100
M.P(100 – Dis%) CP(100 + Profit%)
i.e. =
100 100
Note : Remember discount is given on marked price, and gain is calculated
on the cost price.
18. Raman expects 20% profit by selling a fan. If the allows 40%
discount on marked price then his cost price is what percent of
marked price?
(a) 40% (b) 60%
(c) 50% (d) 100%
A (100 - x ) B (100 + y )
Sol. (c) \ Selling price = =
100 100

A (100 - 40 ) B (100 + 20 )
=
100 100
A 60 = B 120
B 60 1
So, = =
A 120 2
But, the cost price is of marked price
B 1
´100% = ´100 = 50%
A 2

Shortcut Approach - 18
A person sells articles at ` A each after giving x% discount on marked
price. Had he not given the discount, he would have earned a profit of y%
on the cost price. Then the cost price of each article is given by `
é 1002 A ù
ê ú.
ë( 100 - x )( 100 + y )û
Profit, Loss and Discount 119
19. A person sells a T.V at 9600 after giving a discount of 20%. If he
sells the T.V at marked price then his profit is 50%. What is the
cost price of T.V?
(a) `10000 (b) `12000 (c) `8000 (d) `9000
é (100 )2 A ù
Sol. (c) \ Cost price of T.V. = ê ú
ê (100 - x )(100 + y ) ú
ë û

é (100 ) 2 ´ 9600 ù
=ê ú
ê (100 - 20 )(100 + 50 ) ú
ë û

96000000
= = ` 8000
12000

Shortcut Approach - 19
A shopkeeper sold an article for ` A after giving x% discount on the
labelled price and made y% profit on the cost price. Had he not given the
é x+ y ù
discount, the percentage profit would have been ê ×100ú per cent.
ë 100 - x û
20. A person allows 30% discount on his goods and still gains 40%.
If he sells a fan on marked price then what is percentage of profit?
(a) 100% (b) 50% (c) 60% (d) 70%

é x+y ù
Sol. (a) \ Profit = ê ´100ú %
ë 100 - x û

éæ 30 + 40 ö ù
= êç ÷ ´ 100 ú %
ëè 100 - 30 ø û

é 70 ù
= ê ´100ú % = 100 %
ë 70 û
EBD_7382
120 Profit, Loss and Discount

Exercise
1. Two motor cars were sold for value of the consignment.
` 9,900 each, gaining 10% on (a) ` 15,000 (b) ` 18000
one and losing 10% on the other. (c) ` 35000 (d) ` 45000
The gain or loss per cent in the 6. A person sells 36 oranges per
whole transaction is : rupee and suffers a loss of 4%.
(a) Neither loss no gain Find how many oranges per
rupee to be sold to have a gain
1 of 8%?
(b) % gain
99 (a) 30 (b) 31
100 (c) 32 (d) 33
(c) % profit 7. Two electronic musical
99
(d) 1% loss instruments were purchased
2. A dishonest dealer sells his for ` 8000. The first was sold at
goods at the cost price but still a profit of 40% and the second
earns a profit of 25% by at loss of 40%. If the sale price
underweighing. What weight was the same in both the cases,
does he use for a kg? what was the cost price of two
(a) 750 g (b) 800 g electronic musical instruments?
(c) 825 g (d) 850 g (a) ` 2000, ` 5000
3. Three successive discounts (b) ` 2200, ` 5500
of 10%, 12% and 15% amount (c) ` 2400, ` 5000
to a single discount of: (d) ` 2400, ` 5600
(a) 36.28 % (b) 34.68% 8. Arun bought toffees at 6 for a
rupee. How many for a rupee
(c) 37 % (d) 32.68%
he should sell to gain 20%?
4. A man sold two steel chairs for (a) 3 (b) 4
` 500 each. On one he gains (c) 5
20% and on other, he loses 12%. (d) can't be determined
How much does he gain or lose 9. A firm of readymade garments
in the whole transaction? makes both men’s and
(a) 1.5% gain women’s shirts. Its average
(b) 2% gain profit is 6% of the sales. Its
(c) 1.5% loss profit in men’s shirts average
(d) 2% loss 8% of the sales and women’s
2 shirts comprise 60% of the
5. of a consignment was sold
3 output. The average profit per
at 6 % profit and the rest at a sale rupee in women shirts is
loss of 3 %. If there was an (a) 0.0466 (b) 0.0666
overall profit of ` 540, find the (c) 0.0166
(d) None of these
Profit, Loss and Discount 121
10. For a certain article, if discount 12. A fruit seller declares that he
is 25%, the profit is 25%. If sells fruits at the cost price.
the discount is 10%, then the However, he uses a weight of
profit is 450 g instead of 500 g. His
(a) 10% (b) 20% percentage profit is :
(c) 35% (d) 50%
11. By selling 66 metres of cloth a 1
(a) 10% (b) 11 %
person gains the cost price of 9
22 metres. Find the gain per cent.
1 2
(a) 22% (b) 22 % (c) 12% (d) 12 %
2 9
1
(c) 33% (d) 33 %
3

Hints & Solution


1. (d) If any two transactions of 540 ´ 100
SP is the same and also = = ` 18,000
4 -1
gain % and loss % are
same then there is always 6. (c) Use Short Approach - 17
a loss
7. (d) Here, SP1 = SP2
\ loss % =
Þ 140 CP1 = 60CP2 Þ
2
æ Common gain or loss% ö CP1 6 3
ç ÷ = =
è 10 ø CP2 14 7
2
æ 10 ö 3
= ç ÷ = 1% \ CP1 = ´ 8000
è 10 ø (3 + 7)
2. (b) Use Short Approach - 20 = ` 2400
3. (d) Applying successive and CP2 = 8000 – 2400 =
discounts of 10%, 12%
` 5600
and 15% on 100, we get
8. (c) Use Short Approach - 1
100 × 0.9 × 0.88 × 0.85
9. (a) Women's shirt comprise
= 67.32
60% of the output.
Þ Single discount \ Men's shirts comprise
= 100 – 67.32 = 32.68% (100 – 60) = 40% of the
4. (a) Use Short Approach -18 out put.
5. (b) Value of consignment \ Aver age profit from
540 ´ 100 men's shirt
= = 8% of 40
2 1
´ 6 + ( -3) = 3.2 out of 40
3 3
EBD_7382
122 Profit, Loss and Discount
Overall average profit cloth = ` 1
= 6 out of 100 then C.P. of 66 metres of
\ Average profit from cloth = ` 66
women's shirts = 2.8 out Gain = C.P. of 22 metres =
of 60 i.e. 0.0466 out of ` 22
each shirt. % gain
10. (d) For same article, 22 1
= ´ 100 = 33 %
100 - d1 100 + g1 66 3
= Shortcut method :
100 - d 2 100 + g 2
If on selling ‘x’ articles, a
100 - 25 100 + 25 man gains equal to the
Þ = C.P. of ‘y’ articles, then %
100 - 10 100 + g 2
gain
75 125 y
Þ = = ´100
90 100 + g 2 x
22
90 ´ 125 \ % gain = ´ 100
Þ 100 + g 2 = 66
75
1
= 150 = 33 %
3
Þ g2 = 50% 12. (b) Use Short Approach - 19
11. (d) Let C.P. of one metre of
Simple Interest &
12 Compound Interest and
Chapter Instalments

Shortcut Approach - 1
If a sum is put at SI at a certain rate r1 % for t1 year, again at rate r2%
at t2 years and then so on, then
Interest ×100
Principal =
t1r1 + t2r2 + t3r3 + ...
1. The rate of interest for the first 2 years is 3% per annum, for the
next 3 years is 8% per annum and for the period beyond 5 years
10% per annum. If a man gets ` 1520 as a simple interest for 6
years, how much money did he deposit?
(a) 4000 (b) 3800 (c) 3900 (d) 4100
Sol. (b) Detail Method: Let his deposit = ` 100
Interest for first 2 years = ` 6
Interest for next 3 years = ` 24
Interest for the last year = ` 10
Total interest = ` 40
When interest is ` 40, deposited amount is ` 100
\ when interest is ` 1520, deposited amount
100
= ´ 1520 =` 3800
40
Interest ´ 100
Direct formula : Principal = t r + t r + t r + ...
11 2 2 3 3

1520 ´ 100 1520 ´ 100


= = = ` 3800
2 ´ 3 + 3 ´ 8 + 1 ´ 10 40
Note: Here t3 = 6 – 5 = 1 year
2. Manish borrowed some money at the rate of 7 per cent per annum for
the first three years, 9 per cent per annum for the next six years and
10 per cent per annum for the period beyond nine years. If the total
interest paid by him at the end of fifteen years is ` 4050, how much
money did he borrow?
(a) ` 2800 (b) ` 3600 (c) ` 3000 (d) ` 3500
Sol. (b)
Interest ×100
Principal = t r + t r + t r + ...
1 1 2 2 3 3
EBD_7382
124 Simple Interest & Compound Interest and Instalments

4050 ´ 100 4050 ´ 100


= = = ` 3000.
7 ×3 + 9× 6 + 6 ×10 135

Shortcut Approach - 2
A sum was put at SI at a certain rate for t years. Had it been put at x%
higher rate, it would have fetched ` 'A' more, then the sum is
é A ×100 ù More interest ×100
` ê ú or .
ë t×x û Time × More rate
3. A sum was put at SI at a certain rate for 5 years. Had it been put at
5% higher rate, it would have fetched ` 500 more. Find the sum.
(a) ` 2500 (b) ` 2000 (c) ` 1500 (d) ` 1800
Sol. (b)
é A ×100 ù é 500×100 ù
Sum = ê ú =ê ú = ` 2000.
ë t × x û ë 5× 5 û
Shortcut Approach - 3
A sum was put at SI at a certain rate for t years. Had it been put at x%
lower rate, it would have fetched ` A less, then the sum is
é A ×100 ù Less interest ×100
` ê ú or .
ë t × x û Time × Lessrate
4. A sum was put at SI at a certain rate for 5 years. Had it been put at
2% lower rate, it would have fetched ` 150 less. Find the sum .
(a) ` 1000 (b) ` 1500 (c) ` 1800 (d) ` 2000
Sol. (b)
é A ´ 100 ù é150 ´ 100 ù
Sum = ê ú =ê ú = ` 1500
ë t ´ x û ë 5´ 2 û

Shortcut Approach - 4
If a person deposits ` x1 in a bank at r1% per annum and ` x2 in another
bank at r2% per annum, then the rate of interest for the whole sum is
é x1 r1 + x2 r2 ù
ê ú.
ë x1 + x2 û
5. A man deposits ` 1350 in a bank at 5% per annum and ` 1150 in
another bank at 6% per annum. Find the rate of interest for the
whole sum.
(a) 5.40% (b) 6.40% (c) 5.46% (d) 11%
Sol. (c)
é x r + x 2 r2 ù
Rate of Interest = ê 1 1 ú
ë x1 + x 2 û
Simple Interest & Compound Interest and Instalments 125

é1350× 5 +1150× 6 ù é 6750 + 6900 ù


=ê ú =ê ú = 5.46%
ë 1350 +1150 û ë 2500 û

Shortcut Approach - 5
A certain sum is invested for certain time. It amounts to ` A1 at r1% per
annum. But when invested at r2% per annum, it amounts to ` A2, then the
é A2 r1 - A1 r2 ù
sum is given by ` ê ú and the time is given
ë r1 - r2 û

é A - A2 ù
by ê 1 ú ×100 years.
ë A2 r1 – A1 r2 û
6. A certain sum is invested for certain time. It amounts to ` 500 at
8% per annum. But when invested at 3 % per annum, it amounts to
` 200. Find the sum.
(a) ` 20 (b) ` 50 (c) ` 25 (d) ` 35
Sol (a)
é A r – A1r2 ù é 200 ´ 8 – 500 ´ 3 ù
Sum = ê 2 1 ú =ê 8-3 ú
ë r1 – r2 û ë û
100
= = ` 20
5
7. A certain sum is invested for certain time. It amounts to ` 450 at
7% per annum. But when invested at 5% per annum, it amounts to
` 350. Find the time.
(a) 50 years (b) 60 years (c) 45 years (d) 40 years
Sol. (a)

é A – A2 ù
Time = ê 1 ú ×100
ë A 2 r1 – A1r2 û

é 450 – 350 ù é100 ´100 ù


=ê ú ×100 = ê ú = 50 years.
ë 350 × 7 – 450× 5 û ë 200 û

Shortcut Approach - 6
If a certain sum of money at simple interest amounts to ` A1 in t1 years
é A2 t1 - A1 t 2 ù
and to ` A2 in t2 years, then the sum is given by ê t - t ú.
ë 1 2 û
EBD_7382
126 Simple Interest & Compound Interest and Instalments
8. A certain sum of money amounts to ` 625 in 4 years and to `
680 in 5 years. Find the sum.
(a) ` 505 (b) ` 305 (c) ` 405 (d) ` 504
Sol. (c)

é A t – A1 t 2 ù é 680× 4 – 625× 5 ù
Sum = ê 2 1 ú = êë 4–5 ú
û
ë t 1 – t 2 û

é 2720 – 3125 ù
=ê ú = ` 405
ë –1 û

Shortcut Approach - 7
If a sum of money becomes 'n' times in 't' years at a simple interest, then
the time in which it will amount to 'm' times itself is given by
ém -1ù
ê n - 1 ú t years.
ë û
9. A sum of money becomes 5 times itself in 5 years at a simple interest.
In how many years will it amount to 7 times itself?
(a) 7 years (b) 6 years 6 months
(c) 6 years (d) 7 years 6 months
Sol. (d)
é m –1ù é 7 –1ù
Time = ê ú t years = ê ú × 5 years
ë n –1 û ë 5 –1 û
6
= ×5 years = 7.5 years = 7 years, 6 months.
4

Shortcut Approach - 8
If the difference between the simple interest received from two different
banks on Rs X for t years is ` Id , then the difference between their rates
é I d ×100 ù
is given by ê ú per cent.
ë X ×t û
10. The difference between the interest received from two different
banks on ` 600 for 6 years is ` 72. Find the difference between
their rates.
(a) 3 % (b) 3.5% (c) 4 % (d) 2 %
Sol. (d)
Simple Interest & Compound Interest and Instalments 127

é I ×100 ù
Difference between their rates = ê d ú
ë X ×t û
é 72×100 ù
=ê ú = 2%
ë 600× 6 û

Shortcut Approach - 9
A certain sum of money at SI amounts to ` A1 in t years at r% per annum,
then the time in which it will amount to ` A2 at the same rate of interest
é A2 æ 100 ö 100 ù
is given by ê A çè t + r ÷ø - r ú years.
ë 1 û
11. A certain sum of money amounts to ` 5000 in 5 years at 10% per
annum. In how many years will it amount to ` 6000 at the same
rate?
(a) 8 years (b) 6 years (c) 10 years (d) 9 years
Sol. (a)
é A æ 100 ö 100 ù
Time = ê 2 ç t + ÷– ú
ë A1 è r ø r û

é 6000 æ 100 ö 100 ù é6 ù



5000
ç5 +
è 10
÷–
ø 10 ú = ê ×15 – 10ú = 8 years.
ë û ë5 û

Shortcut Approach - 10
If a sum of money becomes 'n' times at the simple interest rate of r% per
annum, then it will become 'm' times at the simple interest rate of
é æ m - 1ö ù
ê çè n - 1 ÷ø ´ r ú per cent
ë û
12. A sum of money becomes two times at the simple interest rate of
2% per annum. At what rate per cent will it become five fold?
(a) 10% (b) 8 % (c) 6 % (d) 9 %
Sol. (b)
éæ m – 1 ö ù
Rate of Interest = ê ç ÷× rú
ëè n – 1 ø û
é æ 5 –1 ö ù
= êç ÷ ×2 ú = 8%
ëè 2 – 1 ø û

Shortcut Approach - 11
EBD_7382
128 Simple Interest & Compound Interest and Instalments

` X is divided into two parts such that if one part be invested at r1 % and
the other at r2%, the annual interest from both the investments is ` A.
é 100A - r2 X ù
Then the first part is given by ê ú.
ë r1 - r2 û
13. Aniket deposited two parts of a sum of ` 25000 in different banks at
the rates of 15% per annum and 18% per annum respectively. In
one year he got `4050 as the total interest. What was the amount
deposited at the rate of 18% per annum?
(a) ` 9000 (b) ` 18000
(c) ` 15000 (d) None of these
Sol. (d)
Applying the formula, we have the amount deposited at the rate of
15% per annum
100 ´ 4050 - 18 ´ 25000
= = `15000
15 - 18
\ Amount deposited at the rate of 18% per annum
= ` 25000 – `15000 = `10000.

Shortcut Approach - 12
Some amount out of ` P was lent at r1% per annum and the remaining at
r2% per annum. If the total simple interest from both the fractions in t
years was ` 'A' then the sum lent at r1% per annum was given by `
é 100 A - P r2 t ù
ê ú
ë ( r1 - r2 ) t û
14. Some amount out of ` 950 was lent at 6% per annum and the
remaining at 4% per annum. If the total simple interest from both
the fractions in 5 years was ` 200, find the sum lent at 6% per
annum.
(a) ` 700 (b) ` 100 (c) ` 250 (d) ` 450
Sol. (b)
é100A – Pr2 t ù é100 ´ 200 - 950 ´ 4 ´ 5 ù
Sum = ê ú =ê
(6 – 4) ´ 5 ú = `100
ë (r 1 – r2 )t û ë û

Shortcut Approach - 13
If a person borrows ` X from a bank at simple interest and after t1 years
he paid ` x1 to the bank and at the end of t2 years from the date of borrowing
Simple Interest & Compound Interest and Instalments 129
he paid ` x2 to the bank to settle the account, then the rate of interest is
é x1 + x2 - X ù
given by ê ú ×100%
ë x1 t1 + t 2 ( X - x1 ) û
15. Uday borrows ` 5000 from a bank at SI. After 5 years he paid `
1700 to the bank and at the end of 5 years from the date borrowing
he paid ` 3550 to the bank to settle the account. Find the rate of
interest.
(a) 1 % (b) 1.5% (c) 2 % (d) 10%
Sol. (a)
é x1 + x 2 – X ù
Rate of Interest = ê ú ×100%
ë x1t1 + t 2 (X – x1 ) û
é 1700 + 3550 – 5000 ù
=ê ú ×100%
ë1700 ´ 5 + 5(5000 – 1700) û
250
= ×100% = 1%
25000

Shortcut Approach - 14
1 1
A person invested of his capital at x1 %, at x2 % and the
n1 n 2

1
remainder n at x3 % . If his annual income is `A, the capital is given
3

é ù
ê A×100 ú
by ` ê x x x ú .
ê 1+ 2+ 3ú
ên n n ú
ë 1 2 3û

1 1
16. Out of a certain sum, rd is invested at 3%, th at 6% and the
3 6
rest at 8%. If the simple interest for 2 years from all these
investments amounts to ` 600, find the original sum.
(a) ` 5000 (b) ` 6000 (c) ` 5200 (d) ` 5500
Sol. (a)
600
Here A = Annual income = ` = ` 300
2
EBD_7382
130 Simple Interest & Compound Interest and Instalments

é ù é ù
ê A ×100 ú ê 300×100 ú
Original sum = ê ú =ê ú
ê x1 x 2 x 3 ú ê 3 + 6 + 8 ú
ên n+ +
ë 1 2 n 3 úû ë 3 6 2 û

é 30000 ù
=ê ú = ` 5000
ë 6 û

Shortcut Approach - 15
Let, Amount = ` A
Principal = ` P
Time = t years
é 1
×n ù
æ Aö t
Rate of interest (r) = n ×100 êêçè ÷ø - 1ú %
ú per annum.
P
êë úû
Where, n = 1, when the interest is compounded yearly
n = 2, when the interest is compounded half-yearly
n = 4, when the interest is compounded quaterly
n = 12, when the interest is compounded monthly
If the interest is compounded yearly, then n = 1
éæ A ö 1/ t ù
\ r = 100 êç ÷ - 1ú % per annum
è ø
ëê P ûú
17. At what rate per cent compound interest will ` 625 amount to ` 676
in 2 years?
(a) 3 % (b) 2 % (c) 4 % (d) 5 %
Sol. (c)
é A 1/ t
ù
Rate of Interest = 100 êæç ö÷ –1ú %
è ø
ëê P ûú

é 1
ù
ê æ 676 ö 2 ú é 26 ù
= 100 ç - 1 = 100 ê - 1ú = 4%
ê è 625 ÷ø ú ë 25 û
ë û

Shortcut Approach - 16
A sum of money, placed at compound interest, becomes n times in t years
and m times in x years. We calculate the value of x from the equation
(
1/ t
given below n = m
1/ x
. )
Simple Interest & Compound Interest and Instalments 131
18. A sum of money placed at compound interest doubles itself in 6
years. In how many years will it amount to 16 times itself?
(a) 24 years (b) 26 years (c) 22 years (d) 20 years
Sol. (c)
We can calculate the value of x by the given equation (n)1/ t = ( m )
1/ x

From question, putting the values of n, m and t

Þ (2)1/ 6 = (16 ) Þ (2)1/ 6 = ( 2)


1/ x 4/ x

By comparing both side

é1 4 ù
Þ ê = ú Þ x = 24
ë6 xû

Shortcut Approach - 17
If a certain sum becomes 'm' times in 't' years, the rate of compound interest

r is equal to éë ( m ) - 1ùû × 100.


1/t

19. At what rate per cent will the compound interest, does a sum of
money become 27 times in 3 years?
(a) 100% (b) 150% (c) 75% (d) 200%
Sol. (d)
Rate of compound interest = éë(m)1/ t – 1ùû ×100%

= éë(27)1/ 3 – 1ùû ×100% = [ (3) –1] ×100%


= 2 × 100 = 200%

Shortcut Approach - 18
Let Principal = P
Rate = r% per annum
Amount = A and

1
Note: Time must be given in the form of fraction like 2 years.
2

é r ù
r ù ê ú
2
é
Amount (A) = P ê1 + ú ´ ê1 + 2 ú
ë 100 û ê 100 ú
ë û
EBD_7382
132 Simple Interest & Compound Interest and Instalments
1
20. On what sum will the compound interest for 2 years at 10%
2
amount to ` 6352.50 ?
(a) ` 4900 (b) ` 5500
(c) ` 5000 (d) ` 5800
Sol. (c)
é r ù
r ù ê ú
2
é 2
A = P ê1+ ú × ê1+ ú
ë 100 û ê 100 ú
ë û
2
é 10 ù é 10 ù
Þ 6352.50 = P ê1 + ú ´ ê1 + 2 ´ 100 ú
ë 100 û ë û
2
æ 10 ö 20
Þ 6352.50× ç ÷ × = P Þ P = ` 5000
è 11ø 21

Shortcut Approach - 19
On a certain sum of money, the difference between compound interest
and simple interest for 3 years at r% per annum is given by
2
æ R ö æ R ö
Pç ÷ ç + 3 ÷ and the difference between CI and SI for 2 years =
è 100 ø è 100 ø
2
æ R ö SI ´ R
Pç ÷ =
è 100 ø 200

Note : SI and CI are given on the same sum, at the same rate and for the
same time duration.
21. Find the difference between the simple and compound interest on `
10000 for 3 years at 3 per cent.
(a) ` 27.8 (b) ` 27.27
(c) ` 37.27 (d) ` 37.8
Sol. (b)
Sum × r 2 (300 + r)
Difference =
(100)3

10000× (3)2 (300 + 3)


= = 27.27
(100) 3
Simple Interest & Compound Interest and Instalments 133
22. On a certain sum of money, the simple interest for 2 years is ` 160
at the rate of 5 % per annum. Find the difference in CI and SI.
(a) ` 4 (b) ` 5
(c) ` 6 (d) ` 8
Sol. (a)
æ xr ö æ 160 ´ 5 ö
Difference in C.I and S.I = ç =
è 200 ÷ø çè 200 ÷ø
= ` 4.

Shortcut Approach - 20
To find the ratio of Compound Interest (CI) to Simple Interest (SI) on a
certain sum at r% for 2 years, we use the following formula
CI r
= + 1.
SI 200

Note : This formula is applicable only for 2 years.


23. Find the ratio of CI to SI on a certain sum at 10% per annum for 2
years.
(a) 7 : 5 (b) 21 : 20
(c) 8 : 5 (d) 20 : 19
Sol. (b)
r é 10 ù 210
Ratio of CI to SI = +1 = ê + 1ú =
200 ë 200 û 200
= 21 : 20

Shortcut Approach - 21
If the compound interest on a certain sum for 2 years is ` 'C' and simple
interest is ` 'S', then the rate of interest per annum is
é 2× ( C - S ) ù 2× Diff. ×100 Diff . ´ (100)2
ê ×100 ú % per or and sum =
ë S û SI r2
24. The compound interest on a certain sum for 2 years is ` 105 and
simple interest is ` 100. Find the rate of interest per annum and
the sum.
(a) 10%, ` 500 (b) 10%, ` 1000
(c) 20%, ` 1000 (d) None of these
Sol. (a)
é 2(C – S) ù
Rate of Interest = ê ×100ú %
ë S û
EBD_7382
134 Simple Interest & Compound Interest and Instalments

é 2(105 –100) ù é2 ´5 ù
=ê ×100ú = ê ×100 ú = 10% p.a.
ë 100 û ë 100 û

Diff ×10000 5×10000


and Sum = = = ` 500
r2 (10) 2

Shortcut Approach - 22
If an amount of money grows upto ` A1 in n years and upto ` A2 in ( n + 1)

é æ A ön ù
years on compound interest, then the sum is given by ` ê A1 ç ÷ ú and
1

êë è A 2 ø úû

æ A2 - A1 ö
the rate = ç A ´ 100 ÷ %
è 1 ø
25. An amount of money grows upto ` 2750 in 2 years and upto ` 3125
in 3 years on compound interest. Find the sum
(a) ` 2129.6 (b) ` 1229.6
(c) ` 2219.6 (d) Data inadequate
Sol. (a)
é æ A ön ù é æ 2750 ö ù
2

Sum = ê A1 ç 1 ÷ ú = ê 2750 ç ÷ ú
êë è A 2 ø úû ëê è 3125 ø ûú

é 7562500 ù
= 2750 ´ ê ú = ` 2129.60.
ë 9765625 û

Shortcut Approach - 23
If the compound rate of interest for the first t1 years is r1%, for the next t2
year is r2%, for the next t3 years is r3%... and the last tn years is rn%, then
compound interest on ` x for (t 1 + t 2 + t 3 +...t n) years is
é æ t
r ö1 æ
t
r ö2 æ r önù
t

ê x ç 1 + 1 ÷ ç 1 + 2 ÷ ... ç 1 + n ÷ ú - x.
ëê è 100 ø è 100 ø è 100 ø ûú
26. Find the compound interest on ` 8000 in 2 years, the rate of interest
being 5% for the first year and 10% for the second year.
(a) ` 1340 (b) ` 1420
(c) ` 1240 (d) None of these
Simple Interest & Compound Interest and Instalments 135
Sol. (c)
Compound Interest
é æ r ö1 æ
t
r ö2
t
æ r önù
t

= ê x ç1+ 1 ÷ × ç1+ 2 ÷ ..... ç 1+ n ÷ ú – x


ëê è 100 ø è 100 ø è 100 ø ûú

é æ
1
5 ö æ 10 ö ù
1

= ê8000 ç 1 + ÷ × ç1 + ÷ ú - 8000
êë è 100 ø è 100 ø úû

= 9240 – 8000 = ` 1240

Shortcut Approach - 24
Let an article of ` P is purchased on cash down payment of ` X and n
equal montly instalments of ` x. If r% simple interest is charged in this
instalment scheme, then the value of each instalment is given by
( )
( P - X ) + P - X ´ r ´ n = x + æç x + x ´ r ´ 1 ö÷
100 ´ 12 è 100 ´ 12 ø

æ x´r´2 ö æ x ´ r ´ ( n - 1) ö
+çx+ ÷ + ..... + ç x + ÷
è 100 ´ 12 ø è 100 ´ 12 ø
or
é r × no. of instalments ù
Remaining amount ê1 + ú
ë 12 ´ 100 û
= (Value of each instalments × Number of instalments) +
Value of each instalments
(r + 2r + 3r + ... + (n – 1)r)
12×100
or
é r´nù x
(P –X) ê1 + ú = (x ´ n) + [r + 2r + 3r + ...... + (n – 1)r]
ë 1200 û 1200

27. The price of a television is ` 10,000. It can be purchased on a down


payment of ` 3000 and 6 equal monthly instalment of ` 1300. Find
the rate of interest.
2
(a) 42% (b) 42 %
3
1 1
(c) 42 % (d) 42 %
3 2
EBD_7382
136 Simple Interest & Compound Interest and Instalments

r´n ù
( P - X) éê1 +
x
Sol. (b) ú = x´n+
ë 100 ´ n û 100 ´ 12

éë r + 2r + 3r + ( n - 1) r ùû

é r´ 6 ù
(10000 - 3000 ) ê1 + ú = 1300 ´ 6 +
ë 1200 û

1300
[ r + 2r + 3r + 4r + 5r ]
1200
13
7000 + 35r = 7800 + [15r ]
12
65
700 + 35r = 7800 + r
4
65
35r - r = 7800 - 7000
4
75r
= 800
4
800 ´ 4
r=
75
128 2
r= % = 42 %
3 3

Shortcut Approach - 25
The annual payment that will discharge a debt of ` P due in t years at the
100 P
rate of simple interest r% per annum =
rt ( t - 1)
100t +
2
28. What annual payment will discharge a debt of `1092 due in 2 years
at 12% simple interest?
(a) `725 (b) `325 (c) `515 (d) `900

100P
Sol. (c) Annual payment =
rt ( t - 1)
100t +
2
Simple Interest & Compound Interest and Instalments 137

100 ´ 1092
=
12 ( 2 )( 2 - 1)
100 ( 2 ) +
2

100 ´ 1092
= =` 515.09 =` 515
200 + 12
29. The annual payment of `160 in 5 years at 5% per annum simple
interest will discharge a debt of
(a) `980 (b) `880
(c) `440 (d) `220
100P
Sol. (b) Annual payment =
rt ( t - 1)
100t +
2

100P
Þ 160 =
5 ( 5)( 5 - 1)
100 ( 5) +
2
100P
Þ 160 = Þ ` 880 = P
500 + 50

Shortcut Approach - 26
A sum of ` P is to be paid back in n equal annual instalments. If the interest
is compounded annually at r% per annum, then the value of each instalment
P
=
2 n
æ 100 ö æ 100 ö æ 100 ö
ç ÷+ç ÷ + .... + ç ÷
è 100 + r ø è 100 + r ø è 100 + r ø
30. Kamal took ` 6800 as a loan which along with interest is to be
repaid in two equal annual instalments. If the rate of interest is
1
12 % , compounded annually, then the value of each instalment is :
2

(a) `8100 (b) `4150


(c) `4050 (d) `4000
EBD_7382
138 Simple Interest & Compound Interest and Instalments

6800
Sol. (c) Each instalment = 2
æ ö æ ö
ç 100 ÷ ç 100 ÷
ç ÷+ç ÷
ç 100 + 25 ÷ ç 100 + 25 ÷
è 2 ø è 2 ø

6800 81
= 2
= 6800 ´ =` 4050
æ 8ö æ 8 ö 136
çè ÷ø + çè ÷ø
9 9
31. A man borrows ` 1820 and undertakes to pay back with compound
interest @ 20% pa in 3 equal yearly instal ments at the end of first,
second and third years. What is the amount of each instalment?
(a) `500 (b) `560
(c) `750 (d) `864
Sol. (d) Each instalment
1820
=
2 3
æ 100 ö æ 100 ö æ 100 ö
ç ÷+ç ÷ +ç ÷
è 100 + 20 ø è 100 + 20 ø è 100 + 20 ø

1820
=
2 3 1820
æ5ö æ5ö æ5ö = ´ 216 = ` 864
ç ÷+ç ÷ +ç ÷ 455
è6ø è6ø è6ø

Shortcut Approach - 27
If a man borrows some money on compound interest at r% per annum
compounded annually and returns it in n yearly equal instalments of ` x,
then
(i) Amount borrowed
éæ 100 ö æ 100 ö2 æ 100 ö ù
n
= x êç ÷+ç ÷ + ... + ç ÷ ú
êëè 100 + r ø è 100 + r ø è 100 + r ø úû
(ii) Total interest charged
é ì 100 n ù
æ 100 ö üï ú
2
ïæ ö æ 100 ö
= x ê n - íç ÷+ç ÷ + ... + ç ÷ ý
ê ïè 100 + r ø è 100 + r ø è 100 + r ø ï ú
ë î þû
Simple Interest & Compound Interest and Instalments 139
32. A man borrows a certain sum and pays it back in two equal annual
instalments. If he pays back ` 676 annually and rate of interest is
4%, what is the sum borrowed?
(a) `1352 (b) `1300
(c) `1275 (d) `1250
Sol. (c) Amount borrowed

éæ 100 ö æ 100 ö2 ù
= 676 êç ÷+ç ÷ ú
êëè 100 + 4 ø è 100 + 4 ø úû

é 25 æ 25 ö 2 ù
= 676 ê + ç ÷ ú
êë 26 è 26 ø úû

é 625 + 650 ù
= 676 ê
ë 676 úû
= ` 1275

Shortcut Approach - 28
A sum of ` P is to be paid back in n annual instalments. If the interest is
compounded annually on the balance at r% per annum and is to be included
in each instalment, then

P é r ( n - m + 1) ù
Value of mth instalment = ê1 + ú
n ë 100 û

P é rn ù
i.e. Value of 1st instalment = êë1 + 100 úû
n

P é r ( n - 1) ù
Value of 2nd instalment = ê1 + ú and so on.
n ë 100 û
33. A sum of ` 7200 has to be paid in three annual instalments of ` 2400
with interest. If the rate of interest is 8% per annum, then what
amount will be paid at the end of every year?
(a) `2976, `2784, `2592
(b) `2782, `2529, `2425
(c) `2594, `2463, `2218
(d) `1800, `1712, `1690
EBD_7382
140 Simple Interest & Compound Interest and Instalments

7200 é 8 ´ 3 ù
Sol. (a) Value of 1st instalment = 1+
3 êë 100 úû

æ 24 ö
= 2400 ç1 + =` 2976
è 100 ÷ø
Value of 2nd instalment
7200 é 8 ´ 2 ù æ 16 ö
= 1+ = 2400 ç1 + =` 2784
3 êë 100 úû è 100 ÷ø
Value of 3rd instalment
7200 é 8 ´ 1ù æ 8 ö
= ê1+ ú = 2400 ç1 + ÷ = ` 2592
3 ë 100 û è 100 ø
Simple Interest & Compound Interest and Instalments 141

Exercise
1. In how many minimum number (c) ` 12,000
of complete years, the interest (d) Data inadequate
on ` 212.50 P at 3% per annum 6. Akram Ali left an amount of `
will be in exact number of 340000 to be divided between
rupees? his two sons aged 10 years
(a) 6 (b) 8 and 12 years such that both
(c) 9 (d) 7 of them would get an equal
2. What will be the ratio of simple amount when each attain 18
interest earned by certain years age. What is the share
amount at the same rate of of elder brother if the whole
interest for 6 years and that for amount was invested at 10%
9 years? simple interest ?
(a) 1 : 3 (b) 1 : 4 (a) 120000 (b) 140000
(c) 2 : 3 (c) 160000 (d) 180000
(d) None of these
7. If the difference between S.I and
3. What will be the difference in
C.I for 2 years on a sum of
simple and compound interest
money lent at 5% is ` 6, then
on ` 2000 after three years at the sum be.
the rate of 10 percent per
(a) ` 2200 (b) ` 2400
annum?
(c) ` 2600 (d) ` 2000
(a) ` 160 (b) ` 42
8. A man borrows ` 12,500 at
(c) ` 62 (d) ` 20
4. The difference between 20% compound interest. At
interest received by A and B the end of every year he
is ` 18 on ` 1500 for 3 year. pays ` 2000 as part repayment.
What is the difference in rate How much does he still owe
of interest? after three such instalments?
(a) 1% (b) 2.5 % (a) ` 12,000
(c) 0.5% (d) 0.4% (b) ` 12,864
5. Nitin borrowed some money at (c) ` 15,600
the rate of 6% p.a. for the first (d) None of these
three years, 9% p.a. for the next
9. A person lent a certain sum
five years and 13% p.a. for the
period beyond eight years If of money at 4% simple
the total interest paid by him interest; and in 8 years the
at the end of eleven years is ` interest amounted to ` 340
8160, how much money did he less than the sum lent. Find
borrow? the sum lent.
(a) ` 8000 (a) 500 (b) 600
(b) ` 10,000 (c) 1000 (d) 1500
EBD_7382
142 Simple Interest & Compound Interest and Instalments
10. The CI on ` 5000 for 3 years at (a) ` 10000 (b) ` 50000
8% for first year, 10% for second (c) ` 1 lakh (d) ` 40000
year and 12% for third year will 12. The ratio of CI for 3 years and
be: SI for 1 year for a fixed amount
(a) ` 1750 (b) ` 1652.80 at a rate of r% is 3.64. What is
(c) ` 1575 (d) ` 1685.20 the value of r?
11. The difference between simple (a) 10 % (b) 15 %
and compound interest for the (c) 20 %
fourth years is ` 7280 at 20% (d) None of these
p.a. What is the principal sum?
Simple Interest & Compound Interest and Instalments 143

Hints & Solution


1. (b) Use Short Approach -2 8. (d) Balance =
2. (c) Let the principal be P and
rate of interest be R%. é ìï æ 20 ö üï
3

\ Required ratio ` ê í12500 ´ ç1 + ÷ ý-


ëê îï è 100 ø þï
é æ P ´ R ´ 6ö ù
ê çè 100 ÷ø ú ìï æ 20 ö
2
æ 20 ö üïù
=ê ú í 2000 ´ ç1 + ÷ + 2000 ´ ç1 + ÷ + 2000 ýú
ê æ P ´ R ´ 9ö ú îï è 100 ø è 100 ø ïú
þû
êë çè 100 ÷ø úû éæ 5 6 6ö
= ` êç12500 ´ ´ ´ ÷ -
ëè 6 5 5ø
6PR 6
= = = 2 : 3.
9PR 9 æ 6 6 6 öù
3. (c) Use Short Approach -19 ç 2000 ´ ´ + 2000 ´ + 2000 ÷ú
è 5 5 5 øû
1500 ´ 3 = ` [21600 – (2880 + 2400 +
4. (d) ( r1 - r2 ) = 18 2000)]
100
Þ r1 – r2 = 0.4 = `14320.
5. (a) Use Short Approach -1 9. (a) Let the sum be ` x.
6. (d) Go through options \ Interest
1.8 ´ 6 ´ 10 x ´ 8 ´ 4 32 x
1.8 + = =
100 100 100
1.6 ´ 8 ´ 10 32 x 68 x
= 1.6 + x- =
100 100 100
Hence (d) is correct.
Alternatively : 68 x
When interest is
P1 ´ 6 ´ 10 100
P1 + less, the sum is ` x.
100 \ When interest is
P2 ´ 8 ´ 10 ` 340 less, the sum is
= P2 +
100 x
´ 100 ´ 340
P1 9 68 x
Þ P =8 = ` 500
2
Direct Formula: Sum
Share of elder brother
100
340000 ´ 9 = ´ 340
= 100 - 8 ´ 4
17
100 ´ 340
= `180000 = = ` 500
7. (b) Use Short Approach -19 68
EBD_7382
144 Simple Interest & Compound Interest and Instalments
10. (b) Use Short Approach -23
éæ r ö
3 ù
11. (b) Difference between CI P êç1 + - 1ú
÷
and SI for n th year
12. (c) ëè 100 ø û
n -1 pr
Pr éæ r ö ù
= êç1 + ÷ - 1ú 100
100 ë è 100 ø û
3
P ´ 20 é æ r ö
çè1 + ÷ -1
ë(1.2) - 1ùû
3
7280 = 100 ø
100 = = 3.64
r
Þ P = ` 50000
Alternatively : 100
By taking the option (c),
Ist II nd IIIrd IV th r = 20%
Initially
year year year year
SI 10000 12000 14000 16000 18000 Now,
CI 10000 12000 14400 17280 20736
3
CI for 4th year æ 20 ö
çè 1 + ÷ - 1 ( )3
= 20736 – 17280 100 ø 1.2 - 1
=
= 3456 20 0.2
SI for 4th year = 2000 100
Difference between CI
and SI = 1456 for ` 0.728
= = 3.64
10000 0.2
So, the difference of Hence, r = 20%
`7280 is for ` 50000
13 Sequence and Series
Chapter

Shortcut Approach - 1
26
a, b, c, d, ........ are in A.P., if a – b = b – c = c – d = .......
1. Check whether the series 0.6, 1.7, 2.8, 3.9, ...... is
(a) an A.P (b) a G.P
(c) an H.P (d) None of these
Sol. (a) Here 0.6 – 1.7 = – 1.1, 1.7 – 2.8 = – 1.1, 2.8 – 3.9 = – 1.1
Hence, given series is an A.P.

Shortcut Approach - 2
26
In an A.P., the sum of terms equidistant from the beginning and end is
constant and equal to the sum of first term and last term.
2. Determine the sum of 6th and 19th term of A.P. 4, 9, 14, ......, 119
with 24 terms.
(a) 124 (b) 123
(c) 120 (d) 122
Sol. (b) Since the given A.P. series has 24 terms.
\ 6th term from beginning is the 19th term from end.
\ Sum of 6th and 19th terms = 6th term from the beginning +
6th term from end
= 4 + 119
= 123
= 1st term + last term = 4 + 119 = 123

Shortcut Approach - 3
26
If in an A.P., sum of first p terms is equal to sum of first q terms, then sum
of (p + q) terms is zero.
3. In a certain A.P, it the sum of m terms is the same as the sum of its
n terms, then find the sum of its (m + n) terms.
(a) – (m + n) (b) mn
1
(c) (d) 0
mn
EBD_7382
146 Sequence and Series
Sol. (a) Here, 3rd given series is obtained by adding the corresponding
terms of 1st and 2nd series,
\ Third series is an A.P.

Shortcut Approach - 4
26
If in an A.P., pth term is q and qth term is p, then nth term is (p + q – n).
4. In an A.P, it the sum of first 23 terms is 40 and sum of 40 terms is
23, then find the sum of its 63 terms.
(a) – 163 (b) – 263
(c) – 363 (d) – 63
Sol. (b) T5 = 60 s, T60 = 5, T16 = (60 + 5 – 16) = 49

Shortcut Approach - 5
26

a b c
= = = ......
a, b, c, d, ........ are in G.P., if
b c d
5. Find the correct option for the given series. 5, 10, 20, 40, ....
(a) A. P with difference 5
(b) G.P with ratio 2

æ1ö
(c) G.P with ratio ç ÷
è 2ø
(d) A.P. with difference 2
5 1 10 1 20 1
Sol. (c) Here, = ; = & =
10 2 20 2 40 2
\ Given series is a G.P with the ratio of (1/2).
6. Find the correct option for the given series.
3, 9, 27, 81, ......
(a) A. P with difference (3)
(b) A.P with difference (6)
(c) G.P with ratio (3)

æ1ö
(d) G.P with difference ç ÷
è3ø

3 1 9 1 27 1
Sol. (d) Here, = ; = & =
9 3 29 3 81 3
\ Given series is G.P with the ratio of (1/3).
Sequence and Series 147

Shortcut Approach - 6
26
If each term of a G.P. be multiplied or divided by the same quantity, the
resulting sequence will be also in G.P. with the same common ratio as before.
7. Find the correct option for the given statement. If 4, 12, 36, 108,
324 ...... is a G.P, with common ratio 3. Then, 1, 3, 9, 27, 81 .... is

(a) G. P with common ratio 4

æ1ö
(b) G.P with common ratio ç ÷
è3ø
(c) G.P with common ratio 6
(d) None of these
Sol. (b) It is clear that every element of 1st series is divided by 4, then
the 2nd series is obtained.
1
\ 2nd series is also a G.P with common ratio .
3

Shortcut Approach -726


If a1, a2, a3, ........ and b1, b2, b3, ....... are two geometric progressions, then
the sequence a1b1, a2b2, a3b3, ...... is also G.P.
8. Find the correct option for the given statement consider two GP's 6,
12, 24, 48 .... and 1, 2, 4, 8 .... then. 6, 24, 96, 384, ..... is
(a) an A.P
(b) both A.p and G.P
(c) a G.P
(d) neither an AP nor a G.P
Sol. (c) Here, 3rd series is obtained by multiplying corresponding terms
of 1st & 2nd series.
\ Third series is a G.P.

Shortcut Approach - 8
26
n ( n + 1)
Sum of first n natural numbers =
2

n ( n + 1)
i.e. 1 + 2 + 3 + 4 + 5 +........+ n =
2
EBD_7382
148 Sequence and Series
9. Find the sum of the given series
1 + 2 + 3 + 4 + ............... 120.
(a) 5260 (b) 6260
(c) 7260 (d) 8260
Sol. (c) 1 + 2 + 3 + 4 + .........120.
We know that

n ( n + 1)
The sum of n natural number =
2
Since the given series is a set of 1st 120 natural number

120 (120 + 1)
Sum = = 7260
2
10. Find the sum of the gvien series.
1 + 2 + 3 + 4 + 5 + ....... 5600
(a) 15682600 (b) 15682800
(c) 15682700 (d) 15682900
Sol. (b) 1 + 2 + 3 + 4 + ......... 5600
Since the given series is a set of 1st 5600 natural numbers.

5600 ( 5600 + 1)
So, Sum = = 15682800
2

Shortcut Approach - 9
26
Sum of first n odd natural numbers = n2
i.e. 1 + 3 + 5 + 7 + ....... + (2n – 1) = n2
11. Find the sum of the given series.
1 + 3 + 5 + 7 + ....... upto 100 terms.
(a) 10000 (b) 9000
(c) 8000 (d) 11000
Sol. (a) Since the given series is a set of first 100 odd natural numbers
\ Sum = (100)2 = 10000
Sequence and Series 149

Shortcut Approach - 10
26
Sum of first n even natural numbers = n(n + 1)
i.e. 2 + 4 + 6 + 8 + ....... + 2n = n(n + 1)
12. Find the sum of the given series:
2 + 4 + 6 + 8 + ...... upto 50 terms.
(a) 2450 (b) 2560
(c) 2460 (d) 2550
Sol. (d) Since the givcen series is a set of first 50 even natural numbers
\ Sum = 50 (50 + 1) = 2550

Shortcut Approach - 11
26
Sum of squares of first n natural numbers

n ( n + 1)( 2n + 1)
=
6

n ( n + 1)( 2n + 1)
i.e. 12 + 22 + 32 +........+ n2 =
6

x(x+ 1) (2x+ 1)
13. If 12 + 22 + 32 + ....... + x2 =
6
2 2 2 2
then 1 + 3 + 5 +....... + 19 is equal to:
(a) 1330 (b) 2100
(c) 2485 (d) 2500
Sol. (12 + 32 + 52 + .... + 192) = (12 + 22 + 32 + ... + 202)
– (22 + 42 + 62 + .... + 202)

é 20 ( 20 + 1)( 40 + 1) ù 2

ë 6
2
(2 2
ú - 2 ´ 1 + 2 + 3 + .... + 10
û
2
)
æ 20 ´ 21´ 41 ö æ 10 ´11´ 21 ö
=ç ÷ - 4´ç ÷
è 6 ø è 6 ø
= 2870 – (4 × 385) = 2870 – 1540 = 1330
EBD_7382
150 Sequence and Series

Shortcut Approach - 12
26
2
é n ( n + 1) ù
Sum of cubes of first n natural numbers = ê ú
ë 2 û

2
é n ( n + 1) ù
i.e. 13 + 23 + 33 + 43 +........+ n3 = ê ú
ë 2 û
14. Find the sum of the given series:
13 + 23 + 33 + 43 + ....... (30) 3 =
(a) 206225 (b)226225
(c) 236225 (d) 216225
Sol. (d) Here, n = 30,
2
é 30 ( 30 + 1) ù
\ Sum = ê ú = 216225
ë 2 û
15. Find the value of 53 + 63 + ....... + 103.
(a) 2295 (b) 2425
(c) 2495 (d) 2925
Sol. (d) Required sum = (53 + 63 + ..... 103)

( ) (
= 13 + 23 + 33 + .... + 103 - 13 + 23 + 33 + 43 )
2 2
é10 (10 + 1) ù é 4 ( 4 + 1) ù
=ê ú -ê ú
ë 2 û ë 2 û
= (5 × 11)2 – (2 × 5)2
= 3025 – 100 = 2925
16. If 1 + 23 + 33 + ....... + 103 = 3025, then 4 + 32 + 108 + ..... + 400 is
3

equal to:
(a) 12000 (b) 12100 (c) 12200 (d) 12400
Sol. (b) 4 + 32 + 108 + ....... + 4000
= 4 × (1 + 8 + 27 + ..... + 1000)
= 4 × (13 + 23 + 33 + ...... + 103)
2
é10(10 + 1) ù
= 4´ ê ú = 4 × 3025 = 12100
ë 2 û
Sequence and Series 151

Shortcut Approach - 13
26
If m1, m2, m3, ........, mk and n1, n2, n3, ....., nk are in A.P., then (m1 – n1), (m2
– n2), (m3 – n3), ....... (mk – nk) are also an A.P.
17. Find the correct option for the given statement consider two AP's
15 19 23 27 31 .....
2 5 8 11 14 .....
then 13, 14, 15, 16, 17 .....
(a) an A.P (b) a G.P
(c) both A.P and G.P (d) None of these
Sol. (a) Here 3rd given series is obtained by substracting terms of the
2nd series from the corresponding elements of 1st series.
\ Third series is : 13, 14, 15, 16, 17.....is also in an A.P.
EBD_7382
152 Sequence and Series

Exercise
1. What is the sum of all the two- 7. How many terms of the series 20
digit numbers which when + 16 + 12 + ........ amounts to 48?
divided by 7 gives a remainder (a) 3 (b) 5
(c) 8
of 3?
(d) both (a) and (c)
(a) 94 (b) 676 8. What is the sum of 100 terms
(c) 696 of the series
(d) None of these 1 – 2 + 3 – 4 + 5 – 6 + ........ ?
2. The sum of an infinite GP is 162 (a) 100
and the sum of its first n terms (b) 50
is 160. If the inverse of its (c) 550
common ratio is an integer, then (d) None of these
how many values of common 9. 5, 19, 41, 71, ?, 155
ratio is/are possible, common (a) 106 (b) 125
ratio is greater than 0? (c) 115 (d) 109
(a) 0 (b) 1 10. In a geometric progression the
(c) 2 (d) 3 sum of the first and the last
3. The 4th and 10th term of a GP term is 66 and the product of
the second and the last but
are 1/3 and 243 respectively.
one term is 128. Determine the
Find the 2nd term. first term of the series.
(a) 3 (b) 1 (a) 64 (b) 64 or 2
(c) 1/27 (d) 1/9 (c) 2 or 32 (d) 32
4. Find the sum of the integers 11. Find the sum of all odd
between 1 and 200 that are numbers of four digits which
multiples of 7? are divisible by 9 :
(a) 2742 (b) 2842 (a) 2784491 (b) 2478429
(c) 2642 (d) 2546 (c) 2754000 (d) 2448729
12. The sum of the infinite series
5. Find the number of terms in
the A.P. 22, 28, 34, ......616: 1 1 1
+ + + ...........¥
(a) 80 (b) 78 1.4 4.7 7.10
(c) 99 (d) 100 is equal to :
6. Find the sum of the first 1 1
(a) (b)
hundred even natural numbers 3 4
divisible by 5 : 38
(a) 50500 (b) 55000 (c)
27
(c) 50050 (d) 50005 (d) None of these
Sequence and Series 153

Hints & Solution


1. (b) Use Short Approach -13 Þ (6, 12, 18, 24, 30, ........, 618)
2. (c) a/(1 – r) = 162 and a – (6, 12, 18)
(1 – rn) / (1 – r) = 160 Þ (1, 2, 3, 4, 5, ......., 103) – (1,
2, 3)
Þ 1 – rn = 160/162
Þ 103 – 3 = 100
Þ rn = 1/81
Alternatively :
Hence, there will be onlytwo
values of n, i.e., 2 and 4. 22, 28, 34, ..., 616
Þ 24, 30, 36, ..., 618
1
3. (c) T4 = ar3 = ...(i) Þ 4, 5, 6, ..., 103
3
Þ 103 – 3 = 100
T10 = ar = 243
9

...(ii) 6. (a) Use Short Approach -11


From (i) and (ii) 7. (d) 20 + 16 + 12 = 48
Hence number of terms
T10 ar 9 243 =3
= 3 =
T4 ar 1 Again 20 + 16 + 12 + 8 + 4
3 + 0 + (– 4) + (– 8) = 48
Þ r6 = 243 × 3 Þ r6 = 36 Hence number of terms
r=3 =8
Alternatively :
1 1
Þ a (3)3 = , a= Þ T2 Sn = 48
3 81
1 1 n
= ar = .3 = = é 2 ´ 20 + ( n - 1)( -4 ) ùû
81 27 2ë
Þ 96 = 40n – 4n 2 + 4n
4. (b) Use Short Approach -11 Þ n2 – 11n + 24 = 0
5. (d) Number of terms Þ (n – 8) (n – 3) = 0
Þ n = 8 or n = 3
æ 616 - 22 ö 8. (d) Use Short Approach -16
=ç ÷ +1
è 6 ø 9. (d) The series is as follows:
594 1 + 22 = 5
= + 1 = 100
6 3 + 42 = 19
Alternatively : 5 + 62 = 41
22, 28, 34, ..... 616 = (4, 10, 7 + 82 = 71
16, 22, 28 ,....., 616) – (4, 9 + 102 = 109
10, 16) 11 + 122 = 155
EBD_7382
154 Sequence and Series
10. (b) Let a be the first term and or a2 – 66a + 128 = 0
r be the common ratio of \ a = 64, 2.
the G.P.
11. (c) Use Short Approach -11
Also assume that nth term
is the last term of the GP. 12. (a) Let
1 1 1
Then, a + ar n -1 = 66 S= + + + ....¥
1.4 4.7 7.10
… (1)
1é 3 3 3 ù
and ar.ar n -2 = 128 Þ S= + + + ....¥ ú
3 êë1.4 4.7 7.10 û
or a 2 r n -1 = 128
1 éæ 1 1 ö æ1 1ö æ1 1ö ù
…(2) Þ S = 3 ëêçè 1 - 4 ÷ø + çè 4 - 7 ÷ø + çè 7 - 10 ÷ø + .....¥ûú
From (1) and (2),
1 1
128 Þ S = [1] \S =
a+ = 66 3 3
a
14 Allegation and Mixture
Chapter

Shortcut Approach - 1
The proportion in which rice at ` x per kg must be mixed with rice at
æ y - zö
` y per kg, so that the mixture be worth ` z a kg, is given by ç ÷
è z - xø
1. In what proportion, must wheat at ` 6.20 per kg be mixed with wheat
at ` 7.20 per kg, so that the mixture be worth ` 6.50 per kg?
(a) 4 : 7 (b) 2 : 5 (c) 3:4 (d) 7 : 3
7.20 - 6.50 = 0.70 = 7 : 3
Sol. (d) Required ratio =
6.50 - 6.20 0.30

Shortcut Approach - 2
The quantity of salt m kg at ` x per kg that a man must mix with n kg
of salt at ` y per kg, so that he may, on selling the mixture at ` z per
kg, gain p% on the outlay is given by
é100 z – y (100 + p) ù
n´ê ú kg.
ë x (100 + p) – 100 z û

Note: If we suppose that the quantity of salt at `x be m, then we have


m 100z - y (100 + p )
= .
n x (100 + p ) - 100z
2. A shopkeeper buys 50 kg sugar at the rate of
40/kg and 100kg sugar at the rate 70/kg. At what price per kg
should he sell the mixture to earn 20% profit on cost price?
(a) 72/kg (b) 75/kg (c) 84/kg (d) 90/kg
m 100 z - y (100 + P)
Sol. (a) =
n x (100 + P) - 100 z
50 100 z - 70(100 + 20)
=
100 40(100 + 20) - 100 z
EBD_7382
156 Allegation and Mixture

1 100 z - 8400
=
2 4800 - 100 z
200z – 16800 = 4800 – 100z
300z = 21600
\ z = 72/kg

Shortcut Approach - 3
n gm of sugar solution has x% sugar in it. The quantity of sugar
æ y-x ö
should be added to make it y% in the solution is given by n ç
è 100 - y ø÷
gm or quantity of sugar
added =

solution
(required% value – present% value)
(100 – required% value)
3. If 50 litres of milk solution has 40% milk in it, then how much
milk should be added to make it 60% in the solution?
(a) 25 L (b) 30 L
(c) 35 L (d) 40 L
Sol. (c)
Amount of A left é Rù
= ê1 - ú
Amount of A Originally Present ë 50 û
n
15 é R ù
Þ = 1-
60 êë 50 úû
2
1 æ
= ç 1 - ö÷
R
Þ
4 è 50 ø
1 R
Þ = 1-
2 50
Þ R = 25 litres

Shortcut Approach - 4
A trader has N kg of certain item, part of which he sells at x% profit
and the rest at y% profit. He gains P% on the whole. The quantity of
éæ y - Pö ù
item sold at x% profit is êç ÷ Nú kg and the quantity of item sold
ëè y - x ø û
Allegation and Mixture 157

éæ P - x ö ù
at y% profit is given by ê çè y - x ÷ø N ú kg . .
ë û
4. A shopkeeper has 180 kg of sugar. Some sugar he sells at 10%
profit and rest of sugar at 15% profit. If his overall profit is
12%, then what is quantity of sugar sold at 15% discount?
(a) 72 kg (b) 75 kg (c) 90 kg (d) 108 kg
Sol. (a) Required quantity
æ P - xö 12 - 10
=ç ÷ n kg = ´ 180 = 2 ´ 180 = 72
è y - xø 15 - 10 5

Shortcut Approach - 5
A trader has N kg of a certain item, a part of which he sells at x% profit
and the rest of y% loss. He gains P % on the whole. Then the quantity
éæ P + y ö ù
sold at x% profit is given by êç ÷ N ú kg and the quantity sold
ëè x + y ø û
éæ x - P ö ù
at y% loss is given by êçè ÷ N ú kg
ë x + yø û
5. A person has 216 books in his stock. He sells some books at 24%
profit and remaining books at 8% loss. If he got a profit of 12%
on whole stock then what is number of books sold at 24% profit
and 8% loss?
(a) 116, 100 (b) 120, 96
(c) 135, 81 (d) 132, 84
Sol. (c) Number of books sold at 24 % profit
æ P + yö 8 + 12
=ç ÷ N= ´ 216 = 20 ´ 216 = 135
è x + yø 24 + 8 32
Number of books sold at 8 % loss
æ x - Pö 24 - 12 12
=ç N= ´ 216 = ´ 216 = 81
è x + y ÷ø 24 + 8 32

Shortcut Approach - 6
A trader has N kg of a certain item, a part of which he sells at x% profit
and the rest at y% loss. On the whole his loss is P%. Then the quantity
æ y - Pö
sold at x% profit is ç N kg and the quantity sold at y % loss
è x + y ø÷
EBD_7382
158 Allegation and Mixture

æ x + Pö
is given by ç N kg.
è x + y ø÷
6. A person has 200 litres of milk. He sold some quantity of milk
at 20% profit and some quantity at 30% loss. If his loss was 6%
on whole stock then what quantity of milk is sold at 20% profit
and 30% loss?
(a) 100 litres, 100 litres
(b) 96 litres, 104 litres
(c) 80 litres, 120 litres
(d) 125 litres, 75 litres
Sol. (b) Quantity of milk sold at 20 % profit
30 - 6 24
= ´ 200 = ´ 200
20 + 30 50
= 96 litres
Quantity of milk sold at 30 % loss
20 + 6 26
= ´ 200 = ´ 200
20 + 30 50

= 104 litres

Shortcut Approach - 7
If a person buys n kg of an item at the rate of ` P per kg. If he sells
m kg at a profit of x%, then the rate per kg, at which he should sell
the remaining to get a profit of y% on the total deal, is given by `
é ny - mx ù
P ê1 + ú
ë ( n - m ) 100 û
7. A person buys 100 kg of sugar at the rate of
` 40/kg. He sells 30 kg sugar at 8% profit. Then what is rate
per kg of sugar is he need 10% overall profit?
(a) 34.44 (b) 37.84
(c) 40.84 (d) 44.34
Sol. (d) Required rate

é ny - mx ù = éê1 + 100 ´10 - 30 ´ 8 ùú 40


= P ê1 + ú
ëê ( n - m )100 ûú êë
(100 - 30 )100 ûú
é 760 ù 776 1552
= ê1 + ú 40 = 700 ´ 40 = 35 = 44.34
ë 70 ´100 û
Allegation and Mixture 159

Shortcut Approach - 8
Amount of liquid A left after n operations, when the container originally
contains x units of liquid A from which y units is taken out and replaced
n
æ x- yö
by liquid B each time is x ç ÷ units.
è x ø
8. From a container full of milk 20 % is drawn and the container
filled with water and this process is performed two times more.
What would be the ratio of milk in the mixture in present and
original?
(a) 64 : 125 (b) 216 : 125
(c) 125 : 64 (d) 64 : 81
Sol. (a) Required ratio
n n 3 3
æ yö æx-yö æ 100 - 20 ö æ 80 ö
= ç1 - ÷ = ç =ç =ç
è 100 ÷ø è 100 ÷ø
÷ = 64: 125
è xø è x ø

Shortcut Approach - 9
Consider a container containing only ingredient 'A' of x0 unit. From this,
xr unit is taken out and replaced by an equal amount of ingredient B.
This process is repeated n times, then after n operations.
n
æ xr ö
Amount of A left çè 1 - x ÷ø
0
= n
Amount of B left æ x ö
1 - ç1 - r ÷
è x0 ø

9. In a drum there is 100 litres of milk. From this 10 litres of milk


taken out and replaced with water this process is repeated one
more time then what is respective ratio of milk and water in new
mixture?
(a) 19 : 81 (b) 81 : 19
(c) 81 : 100 (d) 100 : 8 1
Sol. (b) Required ratio
n 2
æ xr ö æ 10 ö
ç1 - ÷ ç 1- ÷
x0 ø 100 ø
= è = è 81 100
n 2 = ´ = 81:19
æ xr ö æ 10 ö 100 19
1 - ç1 - ÷ 1 - ç 1 - ÷
è x0 ø è 100 ø
EBD_7382
160 Allegation and Mixture

Shortcut Approach - 10
A person's expenditure and savings are in the ratio a : b. His income
increases by x%. His expenditure also increases by y%. His percentage
increase in saving is given by
éæ a ö a ù
êçè b + 1÷ø x - b y ú %
ë û
10. Respective ratio of expenditure and saving of Ravi is 7 : 3. His
income is increases by 12% and his expenditure is increases by
15% then what is increament in his saving?
(a) 15% (b) 8 % (c) 5 % (d) 7 %
Sol. (c) Required increment
ìïæ a öa a üï ì 7 ü
= íç + 1÷ x - y ý = íæç + 1 ö÷12 - ´ 15 ý
7
è b ø b ïþ îè 3 ø 3 þ
îï
120 105
= - = 40 - 35 = 5%
3 3

Shortcut Approach - 11
In a group, there are some 4-legged creatures and some 2-legged
creatures. If heads are counted, there are x and if leggs are counted
æ y - 2x ö
there are y, then the no. of 4 – legged creatures are given by çè ÷
2 ø
or
æ Total legs – 2× Total heads ö and the no. of
çè ÷ø
2
æ 4x - y ö
2– legged creatures are given by çè ÷ or
2 ø
æ 4 × Total heads –Total legs ö
çè ÷ø
2
11. On a place some persons and some horses are moving. If heads
are counted, there are 60 and if legs are counted there are 190.
What is number of persons?
(a) 40 (b) 20 (c) 35 (d) 25
Sol. (d) Required number of persons
4x - y 4 ´ 60 - 190 50
= = = = 25
2 2 2
Allegation and Mixture 161

Exercise
1. The average weight of girls is at ` 50 per litre, the profit will
15 and the average weight of be 25%?
boys is 30 and the average (a) 1 : 2 (b) 2 : 3
weight of boys and girls both (c) 3 : 4 (d) 4 : 5
is 25. If the number of boys are 6. There are three types of milk,
12, then the number of girls is: Parag. Amul and Nestle. The
(a) 4 (b) 6 ratio of fat to the non–fat
(c) 10 (d) 18 contents in milk is 4 : 5, 5 : 6, 6
2. In what ratio must water be : 7 respectively. If all these
mixed with milk costing `12 three types of milk is mixed in
per litre in order to get a equal quantity, the ratio of fat
to the non-fat contents in the
mixture worth of ` 8 per litre?
mixture will be :
(a) 1 : 3 (b) 2 : 2 (a) 1751 : 2110
(c) 1 : 2 (d) 3 : 1 (b) 175 : 543
3. 5 kg of superior quality of (c) 3 : 5
sugar is mixed with 25 kg of (d) 10 : 18
inferior quality sugar. The 7. Find the ratio in which rice at
price of superior quality and ` 7.20 a kg be mixed with rice
inferior quality sugar is ` 18 at ` 5.70 a kg to produce a
and ` 12 respectively. The mixture worth ` 6.30 a kg.
average price per kg of the (a) 4 : 3 (b) 3 : 4
mixture is: (c) 2 : 3 (d) 3 : 2
(a) ` 13 (b) ` 15 8. The diluted wine contains only
(c) ` 18 (d) ` 21 8 litres of wine and the rest is
water. A new mixture whose
4. If the average weight of a class
concentration is 30% is to be
of students is 15 and the formed by replacing wine. How
average weight of another many litres of mixture shall be
class of students is 30, then replaced with pure wine if there
find the ratio of the students was initially 32 litres of water
of the first class to the another in the mixture?
class of 30 students when the (a) 4 (b) 5
average weight of both the (c) 8
classes is 25 : (d) None of these
(a) 1 : 2 (b) 2 : 1 9. We have a 630 ml mixture of
(c) 1 : 3 (d) 3 : 4 milk and water in the ratio 7:2.
5. In what ratio should freely How much water must be
available water be mixed with added to make the ratio 7:3?
the wine worth ` 60 per litre (a) 70 ml (b) 60 ml
so that after selling the mixture (c) 80 ml (d) 50 ml
EBD_7382
162 Allegation and Mixture
10. A container contains 40 litres 7 : 5. When 9 litres of mixture
of milk. From this container 4 are drawn off and the can is
litres of milk was taken out and filled with B, the ratio of A and
replaced by water. This B becomes 7 : 9. How many
process was repeated further litres of liquid A were
two times. How much milk is contained by the can initially?
now contained by the 12. A dishonest milkman sells his
container? milk at cost price but he mixes
(a) 26 litres
it with water and thereby gains
(b) 29.16 litres
25%. What is the percentage
(c) 28 litres
(d) 28.2 litres of water in the mixture?
11. A can contains a mixture of (a) 25% (b) 20%
two liquids A and B is the ratio (c) 22% (d) 24%

Hints & Solution


1. (b) Use Short Approach - 1 averages of both the
2. (c) By rule of alligation, combined classes is 25.
Cost Price of Cost Price of So the difference between
1 litre water 1 litre milk 25 and 15 i.e., 10 and 30 &
0 12 25, i.e., 5 is written
diagonally opposite.
Mean Price Thus the required ratio
8 5 1
= =
10 2
12 – 8 = 4 8–0=8 5. (a) Use Short Approach - 2
3. (a) Use Short Approach - 2 6. (a) In this type of questions,
4. (a) Let the ratio of the we consider only one
students with 15 students item (viz, either fat or non-
to 30 students be x : y, then fat) as the fraction of the
15 30 total quantity. Again we
equate the denominators.
25 Let us consider fraction
of fat
5 10
1 2 Parag Amul Nestle
ü 4 5 6
Here we know that the Fraction ï
ï 9 11 13
averages of individual of ý
ï 4×11×13 5×9 ×13 6× 9×11
classes is 15 and 30. Again fat ® ï 9×11×13 11× 9×13 13× 9×11
þ
we know that th e 572 585 594
1287 1287 1287
Allegation and Mixture 163
So the total fat in the taken out and replaced by
total mixture water. After n operations,
572 + 585 + 594 quantity of pure liquid
=
1287 + 1287 + 1287 æ yö
n

= x ç1 - ÷ units.
1751 è xø
=
3861 milk contained by the
Hence, the ratio of fat to container now
the non-fat contents in 3
the mixture æ 4 ö
= 40 ç 1 - ÷
= 1751 : 2110 è 40 ø
Q (3861 – 1751 = 2110) 3
Required ratio = 4 : 8 = 1 : 2 æ 1ö
= 40 ç 1 - ÷
7. (c) Use Short Approach - 1 è 10 ø
8. (b)
9 9 9
= 40 ´ ´ ´
Wine Water 10 10 10
8L 32L
4´9´9´9
1 : 4 = = 29.16
100
20% 80% ( original ratio )
11. (c) Suppose the can initially
30% 70% ( required ratio ) contains 7x and 5x of
In this case, the percentage mixtures A and B
of water being reduced respectively.
when the mixture is being Quantity of A in mixture
replaced with wine. left
So the r atio of left æ 7 ö
quantity to the initial = ç 7x - ´ 9 ÷ litres
è 12 ø
quantity is 7 : 8.
Therefore æ 21 ö
= ç 7x - ÷ litres.
è 4 ø
7 é Kù
= 1- Quantity of B in mixture
8 êë 40 úû
left
7 é 40 - K ù
Þ = æ 5 ö
8 êë 40 úû = ç 5x - ´ 9 ÷ litres
è 12 ø
Þ K = 5 litres
9. (a) Use short approach -1 æ 15 ö
10. (b) Suppose a container = ç 5x - ÷ litres.
è 4ø
contains x units of a liquid
from which y units are
EBD_7382
164 Allegation and Mixture
error = quantity of water
æ 21 ö he mixes in the milk = x
ç 7x - ÷ 7
è 4 ø true value = true quantity
\ = Þ
æ 15 ö 9 of milk = T
ç 5x - ÷ + 9
è 4 ø So the formula becomes,
x
28x - 21 7 25 = ´ 100
= (T - x )
20x + 21 9
Þ 252x –189 = 140x + 147 x
Þ1= ´4
Þ x = 3. ( - x)
T
So, the can contained 21
Þ T - x = 4x
litres of A
12. (b) If a trader professes to sell Þ T = 5x
his goods at cost price, Percentage of water in the
but uses false weights, mixture
then
x x
Gain % = ´ 100 = ´ 100
T 5x
é Error ù
ê ´ 100ú % 1
ë ( True Value-Error ) û =
5
´ 100 = 20%
Here Gain = 25%
15 Time, Work and Wages
15
Chapter

Shortcut Approach - 1
If A and B can do a piece of work in x days, B and C in y days, C and A in z
days, then (A + B + C) working together will do the same work in
é 2xyz ù
ê xy + yz + xz ú days.
ë û
1. A and B can do a piece of work in 12 days, B and C in 8 days, C and
A in 6 days. How long would B take to do the same work alone?
(a) 24 days (b) 32 days
(c) 40 days (d) 48 days

Sol. (d) \ (A + B + C) will work together for


é 2ABC ù é 2 ´12 ´ 8 ´ 6 ù
=ê ú days = ê12 ´ 8 + 8 ´ 6 + 6 ´ 12 ú days
ë AB + BC + AC û ë û

1152 16
= = days
216 3
Now, work done by B in one day
= Work done by (A+ B + C) in one day
– Work done by (A + C) in one day.
3 1 9 -8 1
= - = =
16 6 48 48
Hence, B take 48 days to do the same work alone.

Shortcut Approach - 2
A certain number of men can do a work in 'D' days. If there were ' x ' men
less it could be finished in 'd' days more, then the number of men
é x (D + d )ù
originally are ê ú.
ë d û
EBD_7382
166 Time, Work and Wages
2. A certain number of women can do a work in 12 days. If there were 4
women less it could be finished in 16 days more. How many women
are there?
(a) 5 women (b) 6 women
(c) 7 women (d) None of these
Sol. (c) \ No. of women
é x(D + d ) ù é 4(12 + 16) ù 4 ´ 28
=ê ú = ê 16 ú = 16 = 7 women
ë d û ë û

Shortcut Approach - 3
If A is 'n' times as fast (or slow) as B, and is therefore able to finish a
work in 'D' days less (or more) than B, then the time in which they can
æ Dn ö
do it working together is given by ç 2 ÷ days.
è n -1ø
3. Two men A and B started a job in which A was thrice as good as B
and therefore took 60 days less than B to finish the job. How many
days will they take to finish the job if they start working together?

(a) 15 days (b) 20 days


1
(c) 22 days (d) 25 days
2
Sol. (c) \ Number of days

æ Dn ö é 60 ´ 3 ù
=ç ÷ days = ê ú days
è n2 -1ø ê ( 3 )2 - 1 ú
ë û

180 90 1
= = = 22 days
8 4 2

Shortcut Approach - 4
If x1 men or y1 women can reap a field in 'D' days, then x2 men and y2
é D ( x1 y1 ) ù
women take to reap it ê x y + x y ú days.
ëê 2 1 ú
1 2 û
Time, Work and Wages 167
4. 12 men or 15 women can reap a field in 14 days. Find the number of days
that 7 men and 5 women will take to reap it.
3 3
(a) 12 days (b) 13 days
11 11
3 3
(c) 14 days (d) 15 days
11 11
Sol. (d) \ Number of days
é D ( x1y1 ) ù é 14 ´ (12 ´15) ù
=ê ú =ê ú
ë x 2 y1 + x1 y 2 û ë 7 ´15 + 12 ´ 5 û
168 3
= = 15 days
11 11

Shortcut Approach - 5
If a1 men and b1 boys can do a piece of work in x days and a2 men and b2
boys can do it in y days, then the following relationship is obtained.
é yb - xb1 ù
1 man = ê 2 ú boys
ë xa1 - ya2 û
5. If 5 men and 8 boys can complete a piece of work in 8 days and 3 men
and 12 boys can complete same work in 10 days then what is
respective ratio of the work efficiency of a man to that of a boy?

(a) 5 : 24 (b) 5 : 28 (c) 28 : 5 (d) 14 : 5


Sol. (c) \ 1 man
é yb - xb1 ù
=ê 2 ú boys
ë xa1 - ya 2 û

é10 ´12 - 8 ´ 8 ù
=ê ú boys
ë 8 ´ 5 - 10 ´ 3 û
A man work é120 - 64 ù
=
A boy 's work êë 40 - 30 úû

28
= = 28 : 5
5
EBD_7382
168 Time, Work and Wages

Shortcut Approach - 6
If A and B can do a work in x and y days respectively, they began the work
together, but A left after some time and B finished the remaining work in

æ xy öæ y - z ö
z days; then the no. of days after which A left is given by ç ÷ç ÷
è x + y øè y ø
days.
6. A and B can separately complete a piece of work in 20 days and 30
days respectively. They worked together for sometime, then B left
the work. If A completed the rest of the work in 10 days, then B
worked for:
(a) 6 days (b) 8 days
(c) 12 days (d) 16 days
Sol. (a) Number of days

æ xy ö æ x - z ö æ 20 ´ 30 öæ 20 - 10 ö
=ç ÷ç ÷ =ç ÷ç ÷
è x + y ø è x ø è 20 + 30 øè 20 ø

600 1
= ´ = 6 days
50 2

Shortcut Approach - 7
A certain number of men can do a work in 'D' days. If there were 'x' men
more it could be finished in 'd' days less, then the number of men

é x(D- d)ù
originally are ê ú
ë d û
7. A certain number of men can do a work in 60 days. If there were 8
men more it could be finished in 10 days less. How many men are
there?

(a) 40 (b) 45
(c) 50 (d) 55
Time, Work and Wages 169

é x (D - d) ù
Sol. (a) \ Number of worker = ê ú
ë d û

é 8 ( 60 - 10 ) ù 8 ´ 50
=ê ú = = 40 men
ë 10 û 10

Shortcut Approach - 8
A builder decided to build a farmhouse in 'D' days. He employed 'x' men
in the beginning and 'y' more men after 'd' days and completed the
construction in stipulated time. If he had not employed the additional
men, then the men in the beginning would have finished it in

é D ( x + y ) - yd ù é y(D - d)ù
ê ú days and it would have been ê ú days behind
ë x û ë x û
the schedule.
8. A builder decided to build a farmhouse in 40 days. He employed 100
men in the beginning and 100 more after 35 days and completed the
construction in stipulated time. If he had not employed the additional
men, how many days behind schedule would it have been finished?
(a) 2 days (b) 4 days (c) 5 days (d) 6 days

é D ( x + y ) - yd ù
Sol. (c) \ No. of days = ê ú
ë x û

é 40 (100 + 100 ) - 100 ´ 35 ù 4500


=ê ú = = 45 days
ë 100 û 100
Therefore, the work would have lasted
= 45 – 5 = 5 days before scheduled time
OR
\ No. of days

é y ( D - d ) ù é100 ( 40 - 35) ù
=ê ú =ê ú
ë x û ë 100 û
= 5 days before scheduled time
EBD_7382
170 Time, Work and Wages

Shortcut Approach - 9
A team of x persons is supposed to do a work in 'D' days. After 'd1' days,
'y' more persons were employed and the work was finished 'd2' days
earlier, then the number of days it would have been delayed if 'x' more

é y { D - ( d1 + d 2 )} - d 2 x ù
persons were not employed is given by ê ú days
êë x úû

and the number of days in which the work would have been finished is

é ( x + y ) ( D - d 2 ) - d1 y ù
given by ê ú days
ëê x ûú
9. A contractor employed 30 men to do a work in 38 days. After 25
days, he employed 5 men more and the work was finished 1 day
earlier. How many days he would have behind, if he had not employed
additional men?
(a) 1 (b) 1.5 (c) 2 (d) 2.5
Sol. (a) \ No. of days
é ( x + y )( D - d 2 ) - d1y ù é ( 30 + 5)( 38 - 1) - 25 ´ 5 ù
=ê ú days = ê ú
ë x û ë 30 û
= 39 days
\ Required number of days = (39 – 38) = 1 day after the
scheduled time.

Shortcut Approach - 10
A group of men decided to do a work in x days, but 'n' if them became
absent. If the rest of the group did the work in 'y' days, then the original
æ ny ö
number of men is given by ç y - x ÷ men.
è ø
10. (b) \ Number of men
é ny ù é 10 ´110 ù
=ê ú =ê ú
ë y - x û ë110 - 100 û

é10 ´110 ù
=ê ú = 110 men
ë 10 û
Time, Work and Wages 171
Sol. (b) \ Original number of men
æ ny ö é 20 ù
=ç ÷ men êQ y = 100 ´ 125 = 25ú
è y-x ø ë û

é 4 ´ 25 ù
Total men = ê ú = 20 men
ë 25 - 20 û

Shortcut Approach - 11
A can do a work in x days and B can do the same work in y days. If they
work together for 'd' days and A goes away, then the number of days in
æ yö
which B finishes the work is given by y - ç 1+ ÷ d days.
è xø
11. A can do a piece of work in 20 days and B can do the same work in 30
days. They start work together but A left after 10 days. How many
days required to B for remaining work?
(a) 10 days (b) 2.5 days
(c) 15 days (d) 5 days

é æ yö ù
Sol. (d) \ No. of days = ê y - ç1 + ÷ d ú days
ë è xø û

é æ 30 ö ù é 50 ù
= ê30 - ç1 + ÷ ´ 10 ú = ê30 - ´10ú = 5 days
ë è 20 ø û ë 20 û

Shortcut Approach - 12
A and B can do a piece of work in x and y days respectively and both of
them starts the work together. If A leaves the work 'a' days before the
completion of the work, then the total time in which
( x + a) y
the whole work is completed =
( x + y ) days.
A and B can do a piece of work in x and y days respectively and both of
them starts the work together. If B leaves the work 'a' days before the
completion of work, then the total time, in which the whole work is
é( y + a) x ù
completed = ê ú days
ë x+ y û
EBD_7382
172 Time, Work and Wages
12. A and B can complete a piece of work in 30 days and 40 days
respectively. Both of them start the work together but A left the
work 10 days before the completion of work then what is total time?

4 1
(a) 21 days (b) 22 days
5 7

6
(c) 22 days (d) 25 days
7
Sol. (c) \ No. of days

é( x + a ) y ù
=ê ú days
ëê ( x + y ) ûú

é ( 30 + 10 ) 40 ù
=ê ú
ëê ( 30 + 40 ) ûú

1600 6
= = 22 ( days )
70 7

Shortcut Approach - 13
If A working alone takes 'x' days more than A and B, and B working
alone takes 'y' days more than A and B together then the number of days
taken by A and B

working together is given by éë xy ùû days.


13. A takes 18 days more than A and B together to complete a piece of
work and B takes 50 days more than A and B together. What is time
required to complete the work for A and B together?
(a) 30 days (b) 45 days
(c) 15 days (d) 50 days

Sol. (a) \ No. of days = xy

= 18 ´ 50 = 30 days
Time, Work and Wages 173

Shortcut Approach - 14
There is a sufficient food for 'M' men for 'D' days. If after 'd' days 'm'
men leave the place, then the rest of the food will last for the rest of the

é D-d ù
men for ê ´ M ú days.
ë M - m û
There is a sufficient food for 'M' persons for 'D' days. If after 'd' days
D-d
'm' more men join them the food will last in × M days.
M+m

14. There is a sufficient rice for 280 men for 20 days. After 18 days 200
men leave the place. For how many days will the rest of the rice last
for the rest of the men?
(a) 6 days (b) 7 days (c) 9 days (d) 11 days
Sol. (b) \ No. of days

é D-d ù
=ê ´ Mú
ë M - m û

é 20 - 18 ù
=ê ´ 280ú
ë 280 - 200 û

2
= ´ 280 = 7 days
80

Shortcut Approach - 15
A can do a work in x days and B can do the same work in y days. If the
contract for the work is ` X, and both of them work together, then the

æ X ö æ X ö
share of A and B is given by ` ç ´ y÷ and ç ´ x÷
è x+ y ø è x+ y ø
respectively.
15. A and B can do a piece of work in 20 days and 45 days respectively.
If the total of ` 910 is paid for the work then what is the share of B?
(a) 420 (b) 390
(c) 280 (d) 260
EBD_7382
174 Time, Work and Wages

X 910
Sol. (c) Share of B = ´x = ´ 20
x+y 45 + 20

910 ´ 20
= = 280
65

Shortcut Approach - 16
A person A can do a work in x days. With the help of another person B,
he can do the same work in y days. If they get ` X for that work, then the
share of A and

é Xy ù é X ( x - y)ù
B is given by ` ê ú and ` ê ú repsectively.
ë x û ë x û
16. Rakesh can complete a piece of work in 40 days. He can complete
same piece of work with help of Ranjan in 30 days. If they are paid `
1200 for the work then what is share of Ranjan
(a) 900 (b) 300 (c) 600 (d) 400
Sol. (b) Share of Ranjan
X ( x - y) 1200 ( 40 - 30 )
= = = 300
x 40

Shortcut Approach - 17
If a pipe can fill a tank in x hours and another can fill the same tank in
é xy ù
y hours, then time taken to fill the tank = ê x + y ú hours.
ë û
Or
A tap 'A' can empty a cistern in x hours and the other tap 'B' can empty
is in y hours. If both emptying taps are opened together, then the time
é xy ù
taken to empty the full cistern is given by ê ú hrs.
ë x + yû
17. Two pipes A and B can fill a tank in 18 h and 12 h, repectively. If both
the pipes are opened simultaneously, how much time will be taken to
fill the tank:

1 1 2 2
(a) 7 hr (b) 6 hr (c) 7 hr (d) 6 hr
5 5 5 5
Time, Work and Wages 175

é xy ù 18 ´12 216 36 1
Sol. (a) \ Time = ê ú = 18 + 12 = 30 = 5 = 7 5
ëx + yû

Shortcut Approach - 18
A, B and C are three pipes connected to a tank. A and B together fill the
tank in x hours. B and C together fill the tank in y hours. A and C
together fill the tank in z hours.

æ 2 xyz ö
(i) Time taken by A to fill the tank = ç ÷ hrs,
è xy + yz - xz ø

æ 2xyz ö
(ii) Time taken by B to fill the tank = ç yz + xz - xy ÷ hrs,
è ø

æ 2xyz ö
(iii) Time taken by C to fill the tank = ç xz + xy - yz ÷ hrs
è ø

18. Three pipes are connected with a tank. A and B can fill the tank in 40
hrs and B and C in 50 hrs. If A and C can fill the tank in 30 hrs then
find the time required for each pipe?

400 1200 1200


(a) hrs, hrs, hrs
11 7 17

500 200 700


(b) hrs, hrs, hrs
3 3 17
(c) 60 hrs, 50 hrs, 80 hrs
(d) 90.5 hrs, 83.3 hrs, 62.5 hrs
Sol. (a) Required time of A

2xyz 2 ´ 40 ´ 50 ´ 30
= =
xy + yz - xz 40 ´ 50 + 50 ´ 30 - 40 ´ 30

2 ´ 40 ´ 50 ´ 30 400
= = hrs
3300 11
Time of B
2xyz 2 ´ 40 ´ 50 ´ 30
= =
yz + xz - xy 50 ´ 30 + 40 ´ 30 - 40 ´ 50
EBD_7382
176 Time, Work and Wages

2 ´ 40 ´ 50 ´ 30 1200
= = hrs
700 7
Time of C

2xyz 2 ´ 40 ´ 50 ´ 30
= =
xz + xy - yz 40 ´ 30 + 40 ´ 50 - 30 ´ 50

2 ´ 40 ´ 50 ´ 30 1200
= = hrs
1700 17

Shortcut Approach - 19
One filling pipe A is n times faster than the other filling pipe B. If B can
fill a cistern in x hours, then the time in which the cistern will be full, if
æ x ö
both the filling pipes are opened together, is ç ÷ hours.
è n+1ø
Note: Value of the slower filling pipe is given.
19. Two inlet pipes A and B are connected with a tank. A is 4 times
faster than B. If B can fill the tank in 20 hrs then find the time when
tank will be full if both A and B are opened together?
2
(a) 4 hrs (b) 5 hrs (c) 16 hrs (d) 6 hrs
3
x 20
Sol. (a) Required time = = = 4hrs
n +1 4 +1

Shortcut Approach - 20
One filling pipe A is n times faster than the other filling pipe B. If A can
fill a cistern in x hours, then the time in which the cistern will be full,
æ n ö
if both the filling pipes are opened together, is ç ÷ x hours.
è n+1ø
Note: Value of the faster filling pipe is given.
20. Two inlet pipes A and B are connected with a tank. A is 3 times
faster than B. If A can fill the tank in 24 minutes, then what is time
required for A and B together to fill the tank?
(a) 15 minutes (b) 20 minutes
(c) 18 minutes (d) 12 minutes
Time, Work and Wages 177
Sol. (c) Required time
æ n ö æ 3 ö 3
=ç ÷x =ç ÷ 24 = ´ 24 =18 min
è n +1ø è 3 +1 ø 4

Shortcut Approach - 21
If one filling pipe A is n times faster and takes x minutes less time than
the other filling pipe B, then the time, they will take to fill a cistern, if

é nx ù
both the pipes are opened together, is ê ú minutes.
êë ( n 2 - 1) úû

21. Two inlets pipes A and B are connected with a tank. A is 6 times
faster than B. A takes 35 minutes less than B to fill the tank then
what is time required for A and B together to fill the tank?
(a) 35 minutes (b) 6 minutes
(c) 10 minutes (d) 7.5 minutes
Sol. (b) Required time
nx 6 ´ 35 6 ´ 35
= 2
= 2
= = 6 min
n -1 6 -1 35

Shortcut Approach - 22
If a pipe A fills a cistern in x hours and suddenly a leak develops through
which every hour n part of the water filled by the pipe A leaks out, then
æ x ö
the time in which tank is full = ç ÷ hours.
è 1- n ø
22. Pipe A can fill the tank in 35 minutes. Due to some leakage 40% water
is leakes out. What is time required to fill the tank?

(a) 40 minutes (b) 30 minutes


(c) 60 minutes (d) 45 minutes
Sol. (c) Required time
x 36 36 ´ 100
= = = = 60 min
1 - x 1 - 40 60
100
EBD_7382
178 Time, Work and Wages

Shortcut Approach - 23
A cistern is normally filled in x hrs but takes t hrs longer to fill because of
é x´( x + t)ù
a leak in its bottom. If the cistern is full, the leak will empty it in ê ú
ë t û
hrs.
23. A pipe can fill a tank in 15 hrs. Due to a leak it takes 5 hrs more to fill
it. If tank is full the leak will empty it in:
(a) 20 hrs (b) 40 hrs
(c) 45 hrs (d) 60 hrs
Sol. (d) Required time
ìï x ´ ( x + t ) üï 15 ´ (15 + 5) 15 ´ 20
=í ý= = = 60 hrs
îï t þï 5 5

Shortcut Approach - 24
If a pipe fills a tank in x hours and another fills the same tank in y hours, but
a third one empties the full tank in z hours when all of them are opened
é xyz ù
together, then the time taken to fill the tank = ê ú hours.
ë yz + xz - xy û
24. Two inlet pipes A and B and outlet pipe C is connected with a tank.
A and B can fill the tank in 30 hrs and 40 hrs and C can empty it in 20
hrs. If all pipes are opened together then what is time required to fill
the tank?
(a) 100 hrs (b) 120 hrs (c) 60 hrs (d) 80 hrs
Sol. (b) Required time
xyz 30 ´ 40 ´ 20
= =
yz + xz - xy 40 ´ 20 + 30 ´ 20 - 30 ´ 40
30 ´ 40 ´ 20
= = 120
200

Shortcut Approach - 25
A pipe can fill a tank in x hours. Due to a leak in the bottom it is filled in
y hours. If the tank is full, the time taken by leak to empty the tank
æ xy ö
=ç ÷ hrs.
è y- xø
25. A pipe can fill the tank in 20 hrs. Due to a leak it takes 25 hrs to fill the
tank then what is time required for leak to empty the full tank?
(a) 100 hrs (b) 80 hrs (c) 75 hrs (d) 50 hrs
Time, Work and Wages 179
Sol. (a) Required time
xy 20 ´ 25 500
= = = = 100 hrs
y - x 25 - 20 5

Shortcut Approach - 26
A cistern has a leak which would empty it in x hours. If a tap is turned on
which admits water at the rate of w litres per hour into the cistern, and
the cistern is now emptied in y hours, then the capacity of the cistern is
æ xy ö
w´ç ÷ litres.
è y- xø
26. A outlet can empty the tank in 20 hrs. A inlet pumps 5 litres water in
1 hr and it is now emptied in 30 hrs then what is capacity of tank?
(a) 600 litres (b) 200 litres
(c) 300 litres (d) 500 litres
Sol. (c) Required capacity
æ xy ö æ 20 ´ 30 ö 600
= wç ÷ = 5ç ÷ = 5´ = 300 litres
è y - x ø è 30 - 10 ø 10
EBD_7382
180 Time, Work and Wages

Exercise
1. A is 30% more efficient than a man to finish the work. In
B. How much time will they how many days will 5 men
take working together, to finish the same work?
complete a job which A alone (a) 6 (b) 8
could have done in 23 days? (c) 9 (d) 15
(a) 11 days 5. X can do a piece of work in 15
(b) 13 days days. If he is joined by Y who
is 50% more efficient, in what
3
(c) 20 days time will X and Y together
17
(d) None of these finish the work?
2. If m man can do a work in r (a) 10 days
days, then the number of days (b) 6 days
taken by (m + n) men to do it (c) 18 days
is: (d) Data insufficient
m+n 6. Chandni and Divakar can do
(a) a piece of work in 9 days and
mn
m+n 12 days respectively. If they
(b) work for a day alternatively,
mr
Chandni beginning, in how
mr
(c) many days, the work will be
(m + n) completed?
( m + n) r
(d) 1 1
mn (a) 10 (b) 9
3. 12 men complete a work in 18 4 5
days. Six days after they had (c) 11.11 (d) 10
started working, 4 men joined 7. If 20 persons complete one–
them. How many days will all third of a work in 20 days then
of them take to complete the how many more persons will
remaining work ? be required to complete the
(a) 10 days rest work in 25 days?
(b) 12 days (a) 6 (b) 12
(c) 15 days (c) 18 (d) 24
(d) 9 days 8. A contractor undertakes to
4. 6 childr en an d 2 men built a walls in 50 days. He
complete a certain piece of employs 50 peoples for the
work in 6 days. Each child same. However after 25 days
takes twice the time taken by he finds that only 40% of the
Time, Work and Wages 181
work is complete. How many 11. One hundred men in 10 days
more man need to be do a third of a piece of work.
employed to complete the The work is then required to
work in time? be completed in another 13
(a) 25 (b) 30 days. On the next day (the
(c) 35 (d) 20 eleventh day) 50 more men
9. A cistern has two taps which are employed, and on the day
fill it in 12 minutes and 15 after that, another 50. How
minutes respectively. There is many men must be
also a waste pipe in the cistern. discharged at the end of the
When all the three are opened, 18th day so that the rest of
the empty cistern is full in 20 the men, working for the
minutes. How long will the
remaining time, will just
waste pipe take to empty the
full cistern ? complete the work ?
(a) 10 min (a) 100 (b) 105
(b) 12 min (c) 110 (d) 115
(c) 15 min 12. A pump can be operated both
(d) None of these for filling a tank and for
10. A contractor undertakes to emptying it. The capacity of
tank is 2400 m3. The emptying
built a walls in 50 days. He
capacity of the pump is 10 m3
employs 50 peoples for the
per minute higher than its
same. However after 25 days
filling capacity. Consequently,
he finds that only 40% of the
the pump needs 8 minutes less
work is complete. How many to empty the tank to fill it. Find
more man need to be the filling capacity of pump.
employed to complete the (a) 50 m3/min
work in time? (b) 60 m3/min
(a) 25 (b) 30 (c) 58 m3/min
(c) 35 (d) 20 (d) None of these
EBD_7382
182 Time, Work and Wages

Hints & Solution


1. (b) Use Short Approach - 2 men required
2. (c) M1 × D1 = M2 × D2 = (75 – 50) = 25
m × r = (m + n) × D2 9. (a) Use Short Approach - 24
10. (a) 50 men complete 0.4
D2 = mr work in 25 days.
(m + n) Applying the work rule,
3. (d) Use Short Approach - 8 m1 ´ d1 ´ w2
4. (a) 6C + 2M = 6 days = m2 ´ d2 ´ w1
5. (b) Use Short Approach - 3 we have,
6. (a) Use Short Approach - 17 50 ´ 25´ 0.6 =
M1D1 M 2 D2 m2 ´ 25´ 0.4
7. (b) = ,
W1 W2 or m2
20 ´ 20
= 50´ 25´0.6
1 = = 75 men
3 25 ´ 0.4
Number of additional
( 20 + x ) ´ 25 men required
2 = (75 – 50) = 25
3 11. (c) Suppose that X men
or, 2 × 20 × 20 = (20 + x) × must be discharged at
25 the end of the 18th day.
or, x = 12 100 × 10 + 150 × 1 + 200 ×
8. (a) 50 men complete 0.4 7 + (200 – X) × 5
work in 25 days. = 100 × 30
Applying the work rule, 5X = 550 Þ X = 110 men
m1 ´ d1 ´ w2 12. (a) Let the filling capacity of
pump be x m3/min.
= m2 ´ d2 ´ w1 Then, emptying
we have, capacity of pump
50 ´ 25´ 0.6 = = (x + 10) m3 /min.
m2 ´ 25´ 0.4 2400 2400
\ - =8
or m2 x x + 10
Þ x2 + 10 x – 3000 = 0
= 50´ 25´0.6 = 75 men Þ (x – 50)(x + 60) = 0
25 ´ 0.4
Þ x = 50 m3/min.
Number of additional
16 Time, Speed and
16
Chapter Distance

Shortcut Approach - 1
A man takes x hours to walk to a certain place and ride back. However,
if he walks both ways he needs t hours more, then the time taken by
him to ride both ways is (x – t) hours.
1
1. I walk a certain distance and ride back and take 6 hours
2
3
altogether. I could walk both ways in 7 hours. How long would it
4
take me to ride both ways?
(a) 4 hr 25 min (b) 5 hr 25 min
(c) 4 hr 15 min (d) 5 hr 15 min
3 1 5
1. (d) Difference = 7 - 6 = hours
4 2 4
He, need 5/4 hour more to walk both ways
\ Time taken to ride both ways = (X – t) hours
æ 1 5 ö æ 13 5 ö
= ç6 - ÷ = ç - ÷
è 2 4ø è 2 4ø
21 1
= = 5 hours = 5 hours 15 minute
4 4

Shortcut Approach - 2
A person goes to a destination at a speed of x km/hr and returns to his place
at a speed of y km/hr. If he takes T hours in all, the distance between
æ xy ö
his place and destination is ç ´ T ÷ km. In other words, Required
è x + y ø
distance
EBD_7382
184 Time, Speed and Distance

Productof thetwospeeds
= Total timetaken ´
Additionof thetwospeeds
2. A man rode out a certain distance by train at the rate of 25 km per
hour and walked back at the rate of 4 km per hour. The whole
journey took 5 hours 48 minutes. What distance did he ride?

(a) 20 km (b) 29 km (c) 24 km (d) 25 km


Sol. (a) \ Distance
é xy ù
= Tê ú
ëx + yû

æ 48 ö é 25 ´ 4 ù
= ç5 + ÷ ê
è 60 ø ë 25 + 4 ûú

é 29 25 ´ 4 ù
=ê ´ = 20km
ë5 29 úû

Shortcut Approach - 3
If a certain distance is covered at x km/hr and the same distance is
covered at y km/hr, then the average speed during the whole journey is
2 xy
km/hr.
x+ y
3. Sahil covers a certain distance by car driving at 35 km/h and he
returns back to the starting point riding on a scooter with a speed
of 25 km/hr. Find the average speed for the whole journey.

(a) 57.1 km/hr (b) 16.41 km/hr


(c) 29.16 km/hr (d) 23.14 km/hr
2xy
Sol. (c) \ Speed = km / hr
x+y

2 ´ 35 ´ 25
=
35 + 25
1750
= = 29.16 km / hr
60
Time, Speed and Distance 185

Shortcut Approach - 4
If a person does a journey in T hours and the first half at S1 km/hr
2 ´ T ´ S1 ´ S2
and the second half at S2 km/hr, then the distance =
S1 + S2
4. A bus covers a certain distance in 16 hours. It covers half the distance
at 40 km/hr and the rest at 60 km/hr. Find the length of the journey.
(a) 528 km (b) 408 km
(c) 314 km (d) 768 km
Sol. (d) \ Distance
é 2T ( S1 ´ S2 ) ù
=ê ú
ë S1 + S2 û

é 2 ´16 ´ 40 ´ 60 ù
=ê ú = 768 km
ë 40 + 60 û

Shortcut Approach - 5
A person covers a certain distance between two points. Having an
average speed of x km/hr, he is late by x1 hours. However, with a speed
of y km/hr he reaches his destination y1 hours earlier. The distance
é xy( ( x1 + y1 ) ù
between the two points is given by ê ú km.
ë ( y- x ) û
5. A man riding a bicycle from his house at 10 km/ hr and reaches his
office late by 6 min. He increases his speed by 2 km/hr and reaches
6 min before. How far is the office from his house?
(a) 6 km (b) 7 km
(c) 12 km (d) 16 km
Sol. (c) \ Distance
é xy ( x1 + y1 ) ù
=ê ú km
ëê ( y - x ) ûú

é æ 6 6 öù
ê10 ´ 12 ç 60 + 60 ÷ ú
=ê è ø ú km é120 ´12 ù
ê (12 - 10 ) ú = êë 2 ´ 60 úû = 12 km
ê ú
ë û
EBD_7382
186 Time, Speed and Distance

Shortcut Approach - 6
A person walking at a speed of x km/hr reaches his destination x1 hrs
late. Next time he increases his speed by y km/hr, but still he is late
by y1 hrs. The distance of his destination from his house is given by
é xù
ê( x1 - y1 ) ( x + y ) y ú km.
ë û
6. If a train runs at 40 km/hr, it reaches its destination late by 11 min,
but if it runs at 50 km/hr, it is late by 5 min only. Find the correct
time for the train to complete its journey.
(a) 19 min (b) 20 min
(c) 21 min (d) 18 min

é xù
Sol. (a) \ Distance = ê( x1 - y1 )( x + y ) ú km
ë yû

éæ 11 5 ö 40 ù é6 40 ù
= êç - ÷ ( 40 + 10 ) ú km = ê ´ 50 ´ ú = 20 km
ëè 60 60 ø 10 û ë 60 10 û

60
Time taken @ 40 km/hr = 20 ´ = 30 minutes
40
Þ Normal time = (30 – 11) min = 19 minutes
or Time taken @ 50 km/hr
= 20 × 60/50 = 24 minutes
Þ Normal time = (24 – 5) min = 19 min

Shortcut Approach - 7
If a train travelling x km an hour leaves a place and t hours later
another train travelling y km an hour, where y > x, in the same

é t ( xy ) ù
direction, then they will be together after travelling ê ú km from
ë y- x û
the starting place.

7. Amit walks at a uniform speed of 4 km/hr and 4 hr after his start


Brijesh cycles after him at the uniform rate of 20 km/hr. How far
from the starting point will Brijesh catch Amit?
(a) 15 km (b) 18 km (c) 13 km (d) 20 km
Time, Speed and Distance 187
Sol. (d) \ Distance
é t ( xy ) ù
=ê ú km
ë y-x û

é 4 ´ 4 ´ 20 ù
=ê ú km
ë 20 - 4 û
é16 ´ 20 ù
=ê ú km = 20 km
ë 16 û

Shortcut Approach - 8
A person has to cover a distance of x km in t hours. If he covers nth
part of the journey in mth of the total time, then his speed should be
æ x 1- n ö
ç ´ ÷ km/hr to cover the remaining distance in the remaining
è t 1- m ø
time.
8. A bullockcart has to cover a distance of 80 km in 10 hr. If it covers
3
half of the journey in th time, what should be its speed to cover
5
the remaining distance in the left time?
(a) 5 km (b) 10 km
(c) 15 km (d) 20 km
Sol. (b) \ Speed

é x (1 - n ) ù
=ê ú km/hr
ëê t (1 - m ) ûú

é æ 1 öù
ê 80 ç1 - 2 ÷ ú
=ê è øú
ê æ 3öú
ê 10 çè 1 - 5 ÷ø ú
ë û

1
80 ´
= 2 = 40 = 10 km / hr
2 4
10 ´
5
EBD_7382
188 Time, Speed and Distance

Shortcut Approach - 9
A thief is spotted by a policeman from a distance of d km. When the
policeman starts the chase, the thief also starts running. Assuming
the speed of the thief x kilometres an hour, and that of the policeman
y kilometres an hour, then the thief will run before he is overtaken

é x ù
=dê ú km.
ë y- xû
9. A thief is spotted by a policeman from a distance of 200 m. When
the policeman starts chasing, the thief also starts running. If the
speed of the thief be 16 km/hr and that of the policeman be 20 km/
hr. How far the thief will have run before he is overtaken?

(a) 800 m (b) 900 m


(c) 1000 m (d) 700 m
Sol. (a) \ Distance

é x ù
=dê ú km
ëy- xû

200 é 16 ù
= ´
1000 êë 20 - 16 úû

4 4
= km = ´ 1000 = 800 m
5 5

Shortcut Approach - 10
A person covers a certain distance on scooter. Had he moved x1 km/
hr faster, he would have taken t1 hours less. If he had moved x2 km/
hr slower, he would have taken t2 hours more, then the original speed

é x x (t + t ) ù
(S) is ê 1 2 1 2 ú km/hr and the distance is given by
ë t 2 x1 - t1 x2 û

é t1 ´ S ( S + x1 ) ù
ê ú km.
ë x1 û
Time, Speed and Distance 189
10. A train covered a certain distance at a uniform speed. If the train
had been 6 km/hr faster, it would have taken 4 hours less than the
scheduled time. And, if the train were slower by 6 km/hr, the train
would have taken 6 hours more than the scheduled time. The length
of the journey is:
(a) 700 km (b) 720 km
(c) 740 km (d) 760 km

é x x (t + t ) ù
Sol. (b) Speed = ê 1 2 1 2 ú km / hr
ë t 2 x1 - t1x 2 û

é 6 ´ 6 ( 4 + 6 ) ù 36 ´ 10
=ê ú= = 30 km / hr
ë 6´ 6 - 4´6 û 12

é t ´ s ( s + x1 ) ù é 4 ´ 30 ( 30 + 6 ) ù
Distance = ê 1 ú =ê ú = 720 km
ë x1 û ë 6 û

Shortcut Approach - 11
One aeroplane starts t hours later than the scheduled time from a place D
km away from its destination. To reach the destination at the scheduled time
the pilot has to increase the speed by 'p' km/hr.

é 2 4 Dt ù
ê t + +tú
p
Then the plane takes ê ú hours in the normal case. And the
ëê 2 úû

é 2D ù
ê ú
4 Dt
normal speed of the aeroplane is ê t 2 + + t ú km/hr.
êë p úû
11. One aeroplane started 30 minutes later than the scheduled time
from a place 1500 km away from its destination. To reach the
destination at the scheduled time the pilot had to increase the speed
by 250 km/hr. What was the speed of the aeroplane per hour during
the journey?
(a) 750 km/hr (b) 500 km/hr
(c) 350 km/hr (d) 1000 km/hr
EBD_7382
190 Time, Speed and Distance
Sol. (a) \ speed
é 2 ´ 1500 ù
= ê ú
ê é 1ù ú
ê æ 30 ö
2
ê 4 ´ 1500 ´ 2 ú æ 30 ö ú
ê çè ÷ø + ê ú + çè ÷ø ú
60 ê 250 ú 60
ê ú
ë ë û û
é ù é ù
ê ú ê ú
3000 3000
=ê ú=ê ú
ê 1 æ 3000 ö 1 ú ê æ 49 1 ö ú
ê +ç ÷+ ú ê çç + ÷÷ ú
ë 4 è 250 ø 2 û ë è 4 2ø û
3000
= = 750 km / hr
4

Shortcut Approach - 12
A goods train and a passenger train are running on parallel tracks
in the same or in the opposite direction. The driver of the goods train
observes that the passenger train coming from behind overtakes and
crosses his train completely in T1 seconds. Whereas a passenger on
the passenger train marks that he crosses the goods train in T2
seconds. If the speeds of the trains be in the ratio of a : b, then the ratio

é T2 ù
of their lengths is given by ê ú.
ë T1 - T2 û
12. A goods train and a passenger train are running on the parallel
tracks in the same direction. The driver of the goods train observes
that the passenger train coming from behind overtakes and
crosses his train completely in 1 min whereas a passenger on
2
the passenger train marks that he crosses the goods train in min.
3
If the speeds of the train is in the ratio of 1 : 2, then find the ratio of
their lengths.
(a) 4 : 1 (b) 3 : 1 (c) 1 : 4 (d) 2 : 1

æ T2 ö
Sol. (d) \ The ratio of their lengths = ç ÷
è T1 - T2 ø
Time, Speed and Distance 191

é 2 ù
ê ´ 60 ú
=ê 3 ú 40
êæ 2 ú = = 2 :1
ê çè 60 - 3 ´ 60 ú 20
ë û

Shortcut Approach - 13
Two trains start at the same time from A and B and proceed towards
each other at the rate of x km/hr and y km/hr respectively. When they
meet it is found that one train has travelled d km more than the other.

é x + yù
Then the distance between A and B is ê x - y ú d km.
ë û
13. From stations M and N, two trains start moving towards each
other at speed 125 km/hr and 75 km/hr, respectively. When the
two trains meet each other, it is found that one train covers 50
km more than another. Find the distance between M and N.

(a) 190 km (b) 200 km


(c) 145 km (d) 225 km
Sol. (b) \ The distance between A and B is
æ x+ yö æ 125 + 75 ö 200
=ç ÷d = ç ÷ ´ 50 = ´ 50 = 200 km
èx-yø è 125 - 75 ø 50

Shortcut Approach - 14
A train meets with an accident 't1' hours after starting, which detains
x
it for 't' hours, after which it proceeds at of its original speed. It
y
arrives at the destination 't2' hours late. Had the accident taken place
'd' km farther along the railway line, the train would have arrived only
't 3 ' hours late. The original speed of the train is given by

é æ xö ù
ê d ç1- ÷ ú
ê è yøú
km/hr and the length of the trip is given by
êx ú
( t
êy 2 3 ú- t )
ë û
EBD_7382
192 Time, Speed and Distance

æ d öé æy öù
ç ÷ ê t 2 + t1 ç - 1 ÷ ú km.
è t2 - t3 øë è x øû
14. A train left station A for station B at a certain speed. After travelling
4
for 100 km, the train meets with an accident and could travel at th
5
of the original speed and reaches 45 minutes late at station B. Had
the accident taken place 50 km further on, it would have reached 30
minutes late at station B. What is the distance between station A
and B?
(a) 200 km (b) 250 km
(c) 300 km (d) 350 km
Sol. (b) Original speed of the train

é é xù ù é 4ù
ê d ê1 - ú ú 50 ê1 - ú é 1ù
y ê 50 ´ ú
=ê ë û ú = ë 5 û 5
=ê ú
êx ú 4 é 45 30 ù ê 4 15 ú = 50 km/hr
ê y [ 2 3 ]ú
t - t - ´
ë û 5 êë 60 60 úû ë 5 60 û

D 100 km
So Time (T) = = = 2 hours
S 50 km
\ After 2 hours, the accident took place
Thus, distance between A and B
d é æ y öù
= ê t 2 + t1 ç - 1÷ú
( t 2 - t3 ) ë è x øû

50 é 45 æ5 ù
= + 2 ç - 1ú
æ 45 30 ö êë 60 è4 û
ç 60 - 60 ÷
è ø

50 ´ 60 é 3 1ù
= + 2´ ú
15 êë 4 4û

50 ´ 60 5
= ´ = 250 km
15 4
Time, Speed and Distance 193

Shortcut Approach - 15
Two trains of the length l1 m and l2 m respectively with different
speeds pass a static pole in t1 seconds and t2 seconds respectively.
When they are moving in the opposite direction they will cross each

é (l1 + l 2 )t1 t 2 ù
other in ê t l + t l ) ú seconds.
ë 21 12 û
15. Two trains of equal length running in opposite directions, pass a
student standing by the side of railway line in 18 sec, and 12 sec
respectively. The time that the two trains take to cross each other, is

(a) 7.2 sec (b) 9.6 sec


(c) 10.8 sec (d) 14.4 sec

é (l + l ) t t ù
Sol. (d) \ Time = ê 1 2 1 2 ú
ë t 2l1 + t1l2 û

é ( l + l ) ´18 ´12 ù
=ê ú
ë 12l + 18l û

432l
Þ = 14.4 sec
30l

Shortcut Approach - 16
Two trains of the length l1 and l2 m respectively with different speeds
pass a static pole in t1 seconds and t2 seconds respectively. When they
are moving in the same direction, they will cross each other in

é (l1 + l 2 )t1 t 2 ù
ê ú seconds.
ë t 2 l1 – t1 l 2 û
16. A man standing on a railway platform notices that a train going in
one direction takes 6 seconds to pass him and other train of the
same length takes 9 seconds to pass him. Find the time taken by the
two trains to cross each other when they are running in the same
direction.
(a) 12 seconds (b) 24 seconds
(c) 36 seconds (d) 48 seconds
EBD_7382
194 Time, Speed and Distance

é (l + l ) t t ù
Sol. (c) \ Time l = ê 1 2 1 2 ú
ë t2 l1 - t1l2 û

é(l + l ) 9 ´ 6 ù
=ê ú
ë 9 ´ l - 6l û

108l
= = 36 seconds
3l

Shortcut Approach - 17
A train after travelling d1 km meets with an accident and then proceeds
x
at of its former speed and arrives at its destination t1 hours late. Had
y
the accident occurred d2 km further, it would have reached the

é æ x öù
ê d2 ç 1 - ÷ ú
è y øú
destination only t2 hours late. The speed of the train is ê km/
êx ú
ê y ( t1 - t2 ) ú
ë û

æ d t ö
hr and the distance which train travels is ç d1 + 2 1 ÷ km.
è t1 - t 2 ø
17. A train left station A for station B at a certain speed. After travelling
for 100 km, the train meets with an accident and could travel at
4
th of the original speed and reaches 45 minutes late at station B.
5
Had the accident taken plain 50 km further on, it would have reached
30 minutes late at station B. What is the distance between station A
and B?
(a) 100 km (b) 150 km
(c) 200 km (d) 250 km
Time, Speed and Distance 195

é æ x öù
ê d2 ç1 - ÷ ú
è y øú
Sol. (d) \ Speed = ê km/hr
êx ú
êy ( t1 - t 2 ) ú
ë û

é d t ù
\ Distance = ê d1 + 2 1 ú km
ë t1 - t 2 û

é 45 ù
ê 50 ´ ú
= ê100 + 60 ú km é 2250 ù
ê æ 45 30 ö ú = ê100 + km = 250 km
ê ç - ÷ú ë 15 úû
ë è 60 60 ø û

Shortcut Approach - 18
A train covers a distance between station A and B in T1 hours. If the
speed is reduced by x km/hr, it will cover the same distance in T2
hours, then the distance between the two stations A and B is

æ xT1T2 ö æ xT2 ö
ç ÷ km and the speed of the train is given by ç ÷ km/hr.
è T2 - T1 ø è T2 - T1 ø
18. A train can complete its journey from P to Q in 6 hrs with its
uniform speed. If speed of train is reduced by 20 km/h it takes 2
hrs more to reach from P to Q. What is distance between P and Q?
Also find the speed of train?
(a) 420 km, 70 km/h (b) 360 km, 60 km/h
(c) 240 km, 40 km/h (d) 480 km, 80 km/h

é x (T1T2 ) ù
Sol. (d) \ Distance = ê ú km
ë T2 - T1 û

é 20(6 ´ 8) ù
=ê ú
êë ( 8 - 6 ) úû

960
= = 480 km
2
EBD_7382
196 Time, Speed and Distance

é x.T2 ù
\ Speed = ê ú
ë T2 - T1 û

é 20 ´ 8 ù
=ê ú = 80 km/hr
ë 8-6 û

Shortcut Approach - 19
Two trains A and B start from P and Q towards Q and P respectively.
After passing each other they take T1 hours and T2 hours to reach
Q and P respectively. If the train from P is moving x km/hr, then the

æ T ö
speed of the other train is ç x 1 ÷ km/hr.
ç T ÷
è 2 ø

19. Two trains x and y start from Mumbai and Delhi towards Delhi and
Mumbai, respectively. After passing each other, they take 12 hours
30 minutes and 8 hours to reach Delhi and Mumbai, respectively. If
the train from Mumbai is moving at 60 km/hr, then find the speed
of the other train.
(a) 75 km/hr (b) 68 km/hr
(c) 72 km/hr (d) 62 km/hr

é T ù
Sol. (a) Speed = ê x 1 ú km / hr
ëê T2 úû

é 12.5 ù
= ê60 ´ ú km / hr
ë 8 û

= éë60 ´ 1.5625 ùû km / hr

= [ 60 ´1.25] km / hr
= 75 km/hr

Shortcut Approach - 20
A man rows a certain distance downstream in x hours and returns the
same distance in y hrs. If the stream flows at the rate of z km/hr then
é z( x + y ) ù
the speed of the man in still water is given by ê ú.
ë y- x û
Time, Speed and Distance 197
20. Ramesh can row a certain distance downstream in 6 hours and
return the same distance in 9 hours. If the speed of Ramesh in still
water be 12 km/hr, find the speed of the stream.
(a) 2.2 km/hr (b) 3 km/hr
(c) 2.4 km/hr (d) 3.2 km/hr
é z( x + y) ù
Sol. (c) \ Speed = ê ú km/hr
ë ( y - x) û
é z (6 + 9) ù
12 = ê ú km/hr
ë 9-6 û
é z ´15 ù
12 = ê ú km/hr
ë 3 û
36
z= = 2.4 km/hr
15

Shortcut Approach - 21
If x km be the rate of stream and a man takes n times as long to row
up as to row down the river, then the rate of the man in still water is
é n + 1ù
given by x ê ú km/hr.
ë n - 1û
21. A boat goes 6 km an hour in still water. It takes thrice as much
time going upstream as much time going downstream. The speed of
the current is:
(a) 4 km/hr (b) 5 km/hr (c) 3 km/hr (d) 2 km/hr
é n + 1ù
Sol. (c) Speed = x ê km/hr
ë n - 1úû
é 3 + 1ù
6 = xê ú
ë 3 - 1û
6´ 2
x= = 3 km/hr..
4

Shortcut Approach - 22
A man can row x km/hr in still water. If in a stream which is flowing
at y km/hr, it takes him z hrs to row to a place and back, the distance
(
æ z x2 - y2
ç
) ö÷
between the two places is ç 2x ÷.
è ø
EBD_7382
198 Time, Speed and Distance
22. In a stream, running at 2 km/hr, a motorboat goes 10 km upstream
and back again to the starting point in 55 minutes. Find the speed of
the motorboat in still water.
(a) 22 km/hr (b) 12 km/hr (c) 6 km/hr (d) 16 km/hr

Sol. (a) Distance =


(
z x 2 - y2 )
2x
55 2
10 = 60
(
x -4 )
2x

Þ
55 2
20x =
60
x -4 Þ( 55
)
1200x
= x2 - 4

Þ 11x2 – 44 – 240x = 0
Þ (x – 22) (11x + 2) = 0
\ x = 22 km/hr

Shortcut Approach - 23
If a man can row at a speed of x km/hr in still water to a certain
upstream point and back to the starting point in a river which flows at
y km/hr, then the average speed for total journey (up + down) is given
æ ( x + y )( x - y ) ö
by ç ÷ km/hr.
è x ø
23. Speed of a boat is 5 km/hr in still water and the speed of the stream
is 3 km/hr. If the boat takes 3 hours to go to a place and come back,
the distance of the place is:
(a) 3.75 km (b) 4 km
(c) 4.8 km (d) 4.25 km
é ( x + y )( x - y ) ù
Sol. (c) \ Speed = ê ú km/hr
ë x û
é (5 + 3)(5 - 3) ù 8´ 2 16
=ê ú km/hr = km/hr = km/hr
ë 5 û 5 5
Given time, t = 3 hours.
Two way distance (D) = Time (t) × Speed
16 48
= 3´ = = 9.6
5 5
Hence, distance of the place (D)
9.6
= = 4.8 km/hr
2
Time, Speed and Distance 199

Shortcut Approach - 24
In 'd' metre race, A runs at 'x' km/hr. A gives B a start of y metres
and still beats him by 't' seconds, then the speed of B is given by
18 x(d - y)
km/hr.
18d + 5xt
24. In a 400 m race, A runs at a speed of 16 m/s. If A gives B a start of 16
m and still beats him by 40s, what will be the speed of B?
(a) 6 m/s (b) 8 m/s
(c) 15 m/s (d) 15.9 m/s
Sol. (d) \ Speed
é18 x ( d - y ) ù
=ê ú
ë 18d + 5xt û

é 18 ù
ê 18 ´ 16 ´ 5 (400 - 16) ú
=ê ú = 21.26 km/hr
ê18 ´ 400 + 5 ´ 16 ´ 18 ´ 40 ú
ë 5 û

5
= 21.25 ´ m/sec. = 5.9 m/sec.
18
EBD_7382
200 Time, Speed and Distance

Exercise
1. Two runner start running 5. A car travels 25 km an hour
together for a certain faster than a bus for a journey
distance, one at 8 km/h and of 500 km. If the bus takes 10
another at 5 km/h. The former hours more than the car, then
the speeds of the bus and the
arrives one and half an hour,
car are
before the latter. The distance
(a) 25 km/h and 40 km/h
(in km) is:
respectively
(a) 12 (b) 20
(b) 25 km/h and 60 km/h
(c) 25 (d) 36 respectively
2. A dog starts chasing to a cat (c) 25 km/h and 50 km/h
2 hours later. It takes 2 hours respectively
to dog to catch the cat. If the (d) None of these
speed of the dog is 30 km/h, 6. A bullockcart has to cover a
what is the speed of cat? distance of 80 km in 10 hr. If it
(a) 10 km/h covers half of the journey in
(b) 15 km/h
(c) 20 km/h 3
th time, what should be its
(d) Can’t be determined 5
3. Excluding stoppages, the speed to cover the remaining
speed of a train is 45 km/h and distance in the left time?
including stoppages, it is 36 [SSC FCI 2010]
km/h. For how many minutes (a) 5 km (b) 10 km
does the train stop per hour ? (c) 15 km (d) 20 km
(a) 10 min. (b) 12 min. 7. A thief goes away with a
(c) 15 min. (d) 18 min. Maruti car at a speed of 40
4. If I walk at 4 km/h, I miss the km/h. The theft has been
bus by 10 minutes. If I walk discovered after half an hour
at 5 km/h, I reach 5 minutes and the owner sets off in
before the arrival of the bus. another car at 50 km/h. When
How far I walk to reach the will the owner overtake the
bus stand ? thief from the start.
(a) 5 km 1
(a) 2 hours
(b) 4.5 km 2
(b) 2 hr 20 min
1
(c) 5 km / h (c) 1 hr 45 min
4 (d) Cannot be determined
(d) Cannot be determined
Time, Speed and Distance 201
8. A car driver travels from the km/h and reaches the school 5
plains to a hill station, which minutes early. How far is the
are 200 km apart at an aver- school from his house?
age speed of 40 km/h. In the (a) 1.25 km (b) 8 km
return trip he covers the same (c) 5 km (d) 10 km
distance at an average speed 11. A gives both B and C a start
of 20 km/h. The average of 60 m in a 1500 m race.
speed of the car over the en- However, while B finishes
tire distance of 400 km is with him, C is 15 m behind
(a) 16.56 km/h (b) 17.89 km/h them when A and B cross the
(c) 26.67 km/h (d) 35 km/h finishing line. How much start
9. Two people A and B start from can B give C for the 1500 m
P and Q (distance = D) at the race course?
same time towards each other. 6 5
They meet at a point R, which (a) 7 m (b) 15 m
23 8
is at a distance 0.4 D from P.
They continue to move to and 11 5
(c) 7 m (d) 5 m
fro between the two points. 16 24
Find the distance from point 12. The driver of an ambulance
P at which the fourth meeting sees a school bus 40 m ahead
takes place. of him. After 20 second, the
(a) 0.8 D (b) 0.6 D school bus is 60 metre
(c) 0.3 D (d) 0.4 D behind. If the speed of the
10. A student rides on a bicycle at ambulance is 30 km/h, what
8 km/h and reaches his school is the speed of the school
2.5 minutes late. The next day bus?
he increases his speed to 10 (a) 10 km/h (b) 12 km/h
(c) 15 km/h (d) 22 km/h
EBD_7382
202 Time, Speed and Distance

Hints & Solution


1. (b) Required distance æ 1ö
\ 40 ´ t = 50 ç t – ÷
S1S 2 è 2ø
= ´ Time
( 1 – S2 )
S Þ 10 t = 25 Þ t
difference 5 1
8´ 5 3 = hr = 2 hr
´ = 20 km 2 2
=
3 2 8. (c) Use Shortcut approch-3
2. (b) Use Shortcut approch-10 9. (a) The ratio of speeds of A
3. (b) Due to stoppges the to B would be 2 : 3.
train travels P R Q

(45 – 36) = 9 km less in 0.4 D means ratio


of speeds = 2:3
an hour than it could The 4th meeting would
have travelled without occur after a combined
stoppages. movement of D + 6D =
Thus train stops per hour 7D. 2/5th of this distance
for would be covered by A
9 and 3/5th of this
´ 60 = 12 min. distance would be the
45 distance covered by B.
4. (a) Use Shortcut approch-6
5. (c) Let the speed of the bus Thus, distance covered
be x km / h. by A would be 2/5th of
then speed of the car 7D __: distance covered
= (x + 25) km / h by A = 2.8D – which
means that the 4th
500 500 meeting occurs at a
\ = + 10
x x + 25 distance of 0.8 D from P.
Þ x + 25x – 1250 = 0
2
10. (c) Use Shortcut approch-6
Þ x = 25 A B B
Thus speed of the bus = C C A

25 km/h 11. (b) 60 m 15 m

Speed of the car = 50 km/h 1500 m

Alternative: In the same time, when


Difference in speeds 25 km A covers 1500 m, B
/ hr is in only option (c). covers 1440 m and C
6. (b) Use Shortcut Approach-3 covers 1425 m.
7. (a) Distance to be covered So, in 1440 m race B can
by the thief and by the give a start of 15 m.
owner is same. \ In 1500 m race B will give
Let after time 't', owner a start of
catches the thief.
Time, Speed and Distance 203
15 5 18
´ 1500 = 15 m 5 m /s = 5´
1440 8 5
\
= 18 km / h
12. (b) Relative speed
Now, relative speed = 18
Total distance km/h
=
Total time = Speed of ambulance –
Speed of school bus
18 = 30 – speed of
60 + 40 school bus
= = 5 m/s
20 Speed of school bus
= 12 km/h
EBD_7382
204 Clock and Calendar

17 Clock and Calendar


Chapter

Shortcut Approach - 1
Between x and (x + 1) o'clock the two hands are at right angle at (5x ± 15)
12
× minutes past x.
11
1. At what time between 9'O clock and 10'O clock will the hands of a
clock point in the opposite directions?
4 1
(a) 16 min past 9 (b) 16 min past 9
11 11

3 2
(c) 16min past 9 (d) 16 min past 9
11 11
Sol. (a) The hands of a clock will be at right angles twice between 9’O
clock and 10’O clock
æ 12 ö
ç (\ 5 x ± 15) * ÷ minutes past x
è 11 ø

æ 12 ö
Þ ç (5*9 ± 15)* ÷ minute past 9
è 11 ø
12
= (45 ± 15)* minute past 9
11
æ 12 ö æ 12 ö
= ç 60* minute past 9 ÷ or ç 30* minute past 9 ÷
è 17 ø è 11 ø
720 360
= min past 9 or min past 9
11 11
5 8
= 65 min past 9 or 32 min past 9
11 11
Clock and Calendar 205

Shortcut Approach - 2
Between x and (x + 1) O' clock the two hands are in the same straight line
Case I : (a) When they are in opposite directions at
12
(5 x - 30) minutes past x, [where x > 6]
11

12
(b) (5 x + 30) minutes past x. [where x < 6]
11
Note : At 6 O' clock two hands will be in opposite direction.

æ 12 ö
Case II : When they coincide (or come together), at 5 x ç ÷ minutes past
è 11 ø
x.
2. At what time between 3o' clock and 4o' clock will the hands of a clock
be in opposite directions?

2 1
(a) 48 min past 3 (b) 49 min past 3
11 11

1 2
(c) 50 min past 4 (d) 50 min past 4
11 11
Sol. (b) Required time

12
= (5 x + 30) minutes past x.
11

12
= (5 ´ 3 + 30) minutes past 3
11

æ 12 ö
= ç 45 ´ ÷ minutes past 3.
è 11 ø

1
= 49 minutes past 3.
11
EBD_7382
206 Clock and Calendar

Shortcut Approach - 3
The minute hand of a clock overtakes the hour hand at certain intervals
(given in minutes) of correct time. The clock lose or gain in a day is given
by

é 720 ù é ù
ê 60 ´ 24 ú
ê 11 - given interval in minuteú × ê ú
ë û given interval
ê ú
ë in minutes û
according as the sign in +ve or –ve.
3. The minute hand of a clock overtakes the hour hand at the intervals
of 63 min of the correct time. How much does a clock gain or lose in
a day?
8 1
(a) 56 min (b) 44 min
77 11
4 1
(c) 77 min (d) 91 min
23 11
Sol. (a) Required Time
é 720 ù é 60 ´ 24 ù
Þ ê 11 - 63ú ´ ê 63 ú
ë û ë û
27 60 ´ 24
Þ ´
11 63
3 ´ 60 ´ 24 4320 8
Þ = = 56 minute.
77 77 77
Clock and Calendar 207

Exercise
1. Wh at will be th e a cut e 6. At what time between 9’O
angle between hands of a clock and 10’O clock will the
clock at 2 : 30? hands of a clock point in the
(a) 105° (b) 115° opposite directions?
(c) 95° (d) 135° 4
(a) 16 minutes past 9
2. At what time between 8 and 9 11
o’clock will the hands of a
watch be in straight line but 4
(b) 16 minutes past 8
not together? 11
11 5
(a) 10 min. past 8 (c) 55 minutes past 7
10 61
10 5
(b) 10 min. past 8 (d) 55 minutes to 8
11 61
10 8. A clock gains 15 minutes per
(c) 11 min. past 8 day. It is set right at 12 noon.
11
What time will it show at 4.00
10 am, the next day?
(d) 12 min. past 8
11 (a) 4 : 10 am
3. The calendar for the year 2005 (b) 4 : 45 am
is the same as for the year :
(a) 2010 (b) 2011 (c) 4 : 20 am
(c) 2012 (d) 2013 (d) 5 : 00 am
4. Find the exact time between 7 7. At what point of time after 3
am and 8 am when the two O' clock, hour hand and the
hands of a watch meet ? minute hand are at right
(a) 7 hrs 35 min angles for the first time?
(b) 7 hrs 36.99 min (a) 9 O' clock
(c) 7 hrs 38.18 min
1
(d) 7 hrs 42.6 min (b) 4 h 37 min
5. When do the hands of a clock 6
coincide between 5 and 6 ? 8
(a) 5 : 30 (c) 3 h 30 min
11
(b) 5 : 27 : 16
(c) 5 : 32 : 16 8
(d) 3 h 32 min
(d) 5 : 28 : 56 11
EBD_7382
208 Clock and Calendar
9. If 5th March, 1999 was Friday, (a) 4 p.m.
what day of the week was it
7
on 9th March 2000? (b) 59 min past 3
12
(a) Wednesday
7
(b) Saturday (c) 58 min past 3
11
(c) Friday
3
(d) 2 min past 4
(d) Thursday 11
10. What was the day of the week 12. A clock is set right at 1 p.m. If
on 15th August, 1947 ? it gains one minute in an hour,
(a) Wednesday then what is the true time
(b) Tuesday when the clock indicates 6
p.m. in the same day?
(c) Friday
(d) Thursday 5
(a) 55 min past 5
11. A watch which gains 5 sec- 61
onds in 3 minutes was set (b) 5 min past 6
right at 7 a.m. In the afternoon (c) 5 min to 6
of the same day, when the
watch indicated quarter past 1
(d) 59 min past 5
4 O’clock, the true time is 64
Clock and Calendar 209

Hints & Solution


1. (a) At 2'O Clock, Minute
Hand will be 10 × 6 = 60° \ ( 5 ´ 3 + 15 ) ´ 12 min past 3.
be h i n d t h e Hou r 11
Hand.
12
In 30 minutes, Minute = 30 ´ min past 3
Ha n d wi l l g a i n 11
o 8
æ 1ö = 32 min past 3
ç 5 ÷ × 30 = 150 + 15 11
è 2ø
= 165° 8
\ Angle between Hour i.e., 3 h and 32min .
11
Ha n d a n d M i n u t e
Hand = 165 – 60 = 105° 8. (a) Use Short Approach -3
2. (b) Use Short Approach -2 9. (d) 5 th March, 1999 is
3. (c) Count the number of Friday.
days from 2005 onwards Then, 5th March 2000
to get 0 odd day.
= Friday + 2 = Sunday.
Y ear 2005 2006 2007 2008 2009 2010
Odd days 1 1 1 2 1 1 {Q 2000 is a leap year
= 7 or 0 odd day. and it crosses 29th Feb
\ Calendar for the year 2000, so 2 is taken as
2005 is the same as that odd day}
for the year 2011. \ 5th March 2000 =
4. (c) Use Short Approach -2 Sunday.
5. (b) Then, 9th March 2000 =
5 ´ 30 300 3 Thursday.
= = 27 min = 27 min 16s
11/ 2 11 11 10. (c) 15th August, 1947 =
Therefore, required time (1946 years + Period
= 5 : 27 : 16 from 1st Jan., 1947 to
15th Aug., 1947)
6. (a) Use Short Approach -2
7. (d) Clock will make right Counting of odd days :
angle at 1600 years have 0 odd
day. 300 years have 1
12 odd day.
( 5n + 15 ) ´ min past n.
11 46 years = (11 leap years
Given that, n = 3 + 35 ordinary years)
= [(11 × 2) + (35 × 1)] = 57
odd days
EBD_7382
210 Clock and Calendar
Þ 1 odd day.
Jan. Feb. March April May
31 28 31 30 31
June July Aug.
30 31 15

= 227 days = (32 weeks + 3 =


1
days) = 3 odd days. 1 + hour gained in 1 hour by incorrect clock

Total number of odd


True time interval 1
days = (0 + 1 + 1 + 3) = 5 Þ =
5 1
Hence, the required day 1+
60
was ‘Friday’.
Þ True time interval
11. (a) Use Short Approach -3
12. (a) Time interval indicated 5 ´ 60 56
= = 4
by incorrect clock 61 61
= 6 p.m – 1 p.m = 5 hrs.
56
Time gained by incorrect \ True time = 1 p.m. + 4 hrs.
61
clock in one hour
56 56
1 = 5 p.m. + hrs. = 5 p.m. + ×
= + 1 min = + hr.. 61 61
60
60 min.
Using the formula,
5
True time interval = 55 minutes past 5.
61
Time interval in incorrect clock
Permutation &
18 Combination and
Chapter
Probability

Shortcut Approach - 1
Number of permutation of n different things taken all together when r
particular things ar e to be place at some r given places
= (n – r)!.
1. How many different words can be formed with the letter of the
word ‘JAIPUR’ which start with ‘A’ and end with ‘T’.
Sol. (a) Required number of words
= (6 – 2)! = 4! = 4 × 3 × 2 = 24

Shortcut Approach - 2
Number of permutation of n different things, taken r at a time when m
particular things are to be placed at m given places
= n – mPr – m
2. In how many ways 8 different cycles out of 10 can be arranged
when 2 particular cycles are always taken?
Sol. (a) Required number of arrangement

= 10- 2 P8- 2 = 8 P6

8! 8 ´ 7 ´ 6 ´5 ´ 4 ´ 3 ´ 2 ´1
= =
(8 - 6)! 2 ´1
= 20160

Shortcut Approach - 3
Number of permutation of n particular things, taken r at a time when m
particular things is never taken in each arrangement
= n – mPr
EBD_7382
212 Permutation & Combination and Probability

3. How many different 3-letter words can be formed with the letter of
word ‘JAIPUR’ when A and I are always to be excluded?
Sol. Required number of words

= 6- 2C3 = 4C3 = 4 ´ 3 ´ 2 = 24.

Shortcut Approach - 4
Number of permutation of n different things taken all at a time when m
specified things always come together
= m !´ (n - m + 1)!
4. Find the number of ways to arrange 8 students in a que when the
tallest and smallest student always come together.
Sol. Required number of arrangement
= 2!(8 - 2 + 1)! = 2!´ 7! = 10080

Shortcut Approach - 5
Number of arrangement of n beads or flowers (all different) around a
circular necklace or garland
1
= (n - 1)!
2
5. In how many ways 21 different flowers be strong into a garland?
Sol. Required number of ways

1 1 1
= ´ (10 - 1)! = (21 - 1)! = ´ (10)! = 1814400
2 2 2

Shortcut Approach - 6
Number of arrangement of n beads or flowers (all different) taken r at a time
around a circular necklace or garland
n
Pr
=
2r
6. In how many ways can 4 beads out of 6 different beads be strung
into a ring?
Permutation & Combination and Probability 213
Sol. Required number of ways

6!
6
P4
= = 2!
2´4 2´4

6´5´ 4´3
= = 45
2´ 4

Shortcut Approach - 7
Sum of the total numbers which can be formed with the given n different
digits a1, a2, a3, ……, an = (a1 + a2 + a3 + ….+ an) [(n – 1!)] . (111 ….. n
times)
7. Find the sum of all 4 digits numbers formed with the digits 1, 2, 4
and 6.
Sol. Required sum
= (1 + 2 + 4 + 6) × (3!) × (1111) = 13 ´ 6 ´ 1111 = 86658

Shortcut Approach - 8
Number of rectangles which can be formed on a grid with m rows and n
mn(m + 1)( n + 1)
columns =
4
8. Find the number of rectangles which can be form on a grid with 6
rows and 8 columns.
Sol. Number of rectangles

6 ´ 8 ´ (6 + 1) ´ (8 + 1)
= = 756
4

Shortcut Approach - 9
Number of square which can be formed on a grid with m rows and n
columns (m > n)

2n3 + 3n 2 + n (m - n) × n × (n + 1)
= +
6 2
EBD_7382
214 Permutation & Combination and Probability
9. Find the number of square on a grid with 6 rows and 5 columns.
Sol. Required number of square

2 ´ 53 + 3 ´ 52 + 5 (6 - 5) ´ 5 ´ (5 + 1)
= +
6 2
= 55 + 15 = 70

Shortcut Approach - 10
Number of rectangles which can be formed on a grid with n rows and n
columns
2
é n(n + 1) ù

ë 2 úû
10. Find the number of rectangles formed on a grid with 8 rows and 8
columns.
Sol. Required number of rectangles formed
2
æ 8 ´ 9ö
=ç = 36 ´ 36 = 1296
è 2 ÷ø

Shortcut Approach - 11
Number of squares which can be formed on a grid with n rows and n
columns
n(n + 1)(2n + 1)
=
6
11. How many possible squares are there in a chess board?
Sol. Required number of squares in a chess board
8(8 + 1)(2 ´ 8 + 1) 8 ´ 9 ´ 17
= = = 204
6 6

Shortcut Approach - 12
Let there are h number of horizontal steps and v number of vertical steps
then number of ways to reach from one corner to another corner
h+ v h+v
= Ch or Cv
Permutation & Combination and Probability 215
12. Suppose a person is at south west corner in the following grid and
he has to travel to north-east corner through the lines. Then, find
the number of ways far the person to reach at north east corner
from south west corner.

NE

SW

Sol. Required number of ways for the person to reach from south west
corner to north east corner

= 3+ 4C3 or 3+ 4
C4 = 7 ´ 5 = 35.

Shortcut Approach - 13
If a problem is given to r person A, B, C, ……, R whose chances of
1 1 1 1
solving it are a , b , c , ...., r respectively. Then the probability that
1 1 1 1

the problem will be solved


æ 1ö æ 1ö æ 1ö æ 1ö
= 1 - ç1 - ÷ çè1 - ÷ø çè1 - ÷ø .... çè1 - ÷ø
è aø b c r
13. A problem of Chemistry is given to four students P, Q, R and S;
1 1 1 2
whose chances of solving it are , , and respectively. Then
3 4 5 15
find the probability that the problem will be solved.
EBD_7382
216 Permutation & Combination and Probability
Sol. Required probability

æ 1ö æ 1ö æ 1ö æ 2ö
= 1 - ç1 - ÷ çè1 - ÷ø çè1 - ÷ø çè1 - ÷ø
è 3ø 4 5 15

2 3 4 13 26 49
=1- ´ ´ ´ =1- =
3 4 5 15 75 75

Shortcut Approach - 14
If two boys A and B are playing a game in which they throw a coin
alternatively till one of them gets a ‘head’ and wine the game. Then the
2
probability that A wins the game if he started the game, is .
3

Shortcut Approach - 15
A fact is stated by two persons A and B. If probability of speaking the
truth by A is a and by B is b, then the probability that they contradict each
other.

Shortcut Approach - 16
The probability that a leap year selected at random will have 53 Tuesday
2
(or any other particular days is .
7

Shortcut Approach - 17
The probability that a non-leap year selected at random will have 53
1
Thursday (or any other particular day) is .
7
Permutation & Combination and Probability 217

Exercise
1. If A and B are two independent 6. Three squares of a chessboard
events with P(A) = 0.6, P(B) are chosen at random, the
probability that two are of one
= 0.3, then P ( A 'Ç B ') is colour and one of another is :
equal to :
(a) 0.18 (b) 0.28 67 16
(a) (b)
(c) 0.82 (d) 0.72 992 21
2. In tossing three coins at a 31
time, what is the probability (c)
of getting at most one head? 32
(d) None of these
3 7
(a) (b) 7. An old person forgets the last
8 8 two digits of a telephone
1 1 number, remembering only
(c) (d)
2 8 that these are different dialled
3. How many words beginning at random. The probability
with vowels can be formed that the number is dialled
with the letters of the word correctly is :
EQUATION? (a) 1/90
(a) 25200 (b) 15200 (b) 81/91
(c) 25300 (d) 35200 (c) 2/99
(d) None of these
4. If three vertices of a regular
hexagon are chosen at 8. Each of A and B tosses two
random, then the chance that coins. What is the probability
they form an equilateral that they get equal number of
triangle is: heads?
1 1 3 5
(a) (b) (a) (b)
3 5 16 16
1 1
(c) (d) 4 6
10 2 (c) (d)
5. Letters of the word 16 16
DIRECTOR are arranged in 9. A speaks truth in 75% of the
such a way that all the vowels cases and B in 80% of the
come together. Find out the cases. In what percentage of
cases are they likely to
total number of ways for
contradict each other in
making such arrangement. stating the same fact?
(a) 4320 (b) 2720 (a) 15% (b) 20%
(c) 2160 (d) 1120 (c) 5% (d) 35%
EBD_7382
218 Permutation & Combination and Probability
10. From eighty cards numbered 1
1 to 80, two cards are selected (a)
3
randomly. The probability 1
that both the cards have the (b)
2
numbers divisible by 4 is
1
given by (c)
21 4
(a) (d) None of these
316 12. A bag contains 3 black, 4
19 white and 2 red balls, all the
(b) balls being different. The
316
1 number of at most 6 balls
(c) containing balls of all the
4 colours is
(d) None of these
11. If A and B are two events such (a) 36(4!)
that P (A) = 0.5, P (B) = 0.6 (b) 26 × 4!
and P (A È B) = 0.8. Find
(c) (26 – 1)(4!)
æAö
P ç ÷. (d) None of these
èBø
Permutation & Combination and Probability 219

Hints & Solution

1. (b) Since, A and B are Hence, the r equir ed


independent events probability
\ A' and B' are also
independent events
=
(
2 32 C2 ´32 C1
=
)
16
Þ P ( A 'Ç B ') = P ( A ').P ( B ') 64
C3 21
= (0.4)(0.7) = 0.28
7. (a) The last two digits can
[QP( A ') = 1 - P( A), P( B ') = 1 - P( B) ] be dialled in
2. (c) Use Short Approach - 14 10
P2 = 90 ways.
3. (a) Use Short Approach - 3 Out of these 90 cases
4. (c) Three vertices can be only one case is
selected in 6 C 3 ways. favourable.
Hence, the r equir ed
A5 A4
1
probability = .
A6 A3 90
8. (a) Use Short Approach - 14
9. (d) Contradiction can occur
A1 A2 if A speaks truth and B
The only equilateral lies, B speaks truth and
triangles possible are A lies.
A1A3A5 and A2A4A6 So, required probability
= 0.75 × 0.2 + 0.8 × 0.25
2 2 1
p= 6
=
= = 0.35
C3 20 10 Req% = 0.35 × 100
5. (c) Use Short Approach - 3 = 35%
6. (b) 3 squar es on a 10. (b) Total no. of divisible by
ch essboar d can be 4 between 1 to 80
chosen in 64C3 ways. 80 = 4 + (n – 1)4
Two squares of one 80 = 4 n Þ n = 20
colour and third square \ Required probability
of different colour can 20
C2 19
be chosen in two = =
80 316
mutually exclusive way. C2
(i) 2 white and one black (ii) 11. (b) Q P(A Ç B) = P(A) +
2 black and one P(B) – P(A È B)
white = 0.5 + 0.6 – 0.8 = 0.3
Thus the favourable
number of cases æ A ö P(A Ç B) 0.3 1
\ Pç ÷ = = =
= 32C2 × 32C1 + 32C1 × 32C2 èBø P(B) 0.6 2
= 2 (32C2 × 32C1) 12. (a) Use Short Approach - 6
EBD_7382
220 Geometry-I

19 Geometry-I
Chapter

Shortcut Approach - 1
Sum of all the interior angles of a polygon with ‘n’ sides = (n – 2) × 180°
1. Sum of all interior angles of a regular polygon is 1440° then find
the number of sides of the polygon
(a) 6 (b) 8 (c) 10 (d) 12
Sol. (c) (n – 2) 180 = 1440
Þ n – 2 =8
\ n = 8 + 2 = 10

Shortcut Approach - 2
360
Each exterior angle of a regular polygon = , where n is number of sides
n
of the regular polygon.
2. Each exterior angle of a regular polygon is 30° then find the number
of its sides.
(a) 6 (b) 8 (c) 10 (d) 12
360
Sol. (d) = 30
n
\ n = 12

Shortcut Approach - 3
n(n - 3)
Number of diagonals of a regular polygon with n sides =
2
3. Find the is number of diagonals in a regular octagon.
(a) 20 (b) 18 (c) 12 (d) 15
8(8 - 3)
Sol. (d) Number of diagonals = = 20
2

Shortcut Approach - 4
If PB is a secant of a circle which intersects the circle at A and B and PT is
a tangent at T to the circle, then
Geometry-I 221

B
A
P
PA × PB = PT2
T
4. In the given figure, chords AB and CD of a circle when produced
intersect externally at P. If AB = 6cm, CD = 3cm and PD = 5cm
then PB = ?
(a) 5 cm (b) 6.25 cm (c) 6 cm (d) 4 cm
Sol. (b) PA × PB = PC × PD A
6cm
Þ (x + 6) x = 8 × 5 B
x cm
Þ x + 6 x – 40 = 0
2
P
Þ (x + 10) (x – 4) = 0 Þ x = 4
\ PB = 4 cm D 3cm C
Shortcut Approach - 5
In the figure AB is chord of a circle PQ is a tangent at end point A of the
chord AB to the circle. C is any point on arc AB and D is any point on arc BA
C B

P A Q
Ð BAQ and Ð ACB are angles in the alternate segments.
Ð BAP and Ð ADB are angles in the alternate segments.
Angles in the alternate segments of the circle are equal.
5. In the given figure, PAQ is the tangent. BC is the diameter of the
circle. m ÐBAQ = 60°, find m ÐABC :
B
.
O

C
60°
P Q
A
(a) 25° (b) 30° (c) 45° (d) 60°
Sol. (b) ÐBAC = 90°
ÐBCA = 60° (Q ÐBCA = BAQ)
\ ÐABC = 180° – (90° + 60°)
ÐABC = 30°
EBD_7382
222 Geometry-I

Exercise
AD in terms of ‘b’ and ‘c’
1. Find the number of sides of a provided AC = b and AB = c
regular polygon whose is
interior angle measures 108°. b bc
(a) 5 (b) 6 (a) (b)
(c) 7 (d) 8 b2 c b2 c + c
2. The interior angle of a regular bc c
polygon is 135°, find number (c) (d)
b 2 + c2 b2 + 1
of sides of the polygon. 7. With the vertices DABC as
(a) 6 (b) 8 centres, three circles are
(c) 10 (d) 15
described, each touching the
3. Each interior angle of a regular
polygon is 100° greater than other two externally. If the
each exterior angle. How many sides of the triangle are 9 cm. 7
sides the polygon has ? cm. and 6 cm, find the radii of
(a) 9 (b) 10 the circle.
(c) 12 (d) 8 (a) 3 cm, 6 cm, 9 cm
(b) 4 cm, 5 cm, 2 cm
4. A polygon has 54 diagonals. (c) 2 cm, 3 cm 11 cm
The number of sides in the
(d) 9 cm, 7 cm, 3 cm
polygon is :
(a) 7 8. Two concentric circles with
(b) 9 centre O have A, B, C, D as the
(c) 12 points of intersection with the
(d) None of these line l as shown in the figure.
5. There are two circles each with If AD = 12 cm and BC = 8 cm,
radius 5 cm. Tangent AB is 26 the length of AB, CD, AC and
cm. The length of tangent CD is : BD are respectively.
C
A B

O
D
(a) 15 cm
(b) 21 cm A B M C D
(c) 24 cm
(d) can't be determined (a) 2 cm, 2 cm, 10 cm, 10 cm
6. ABC be a rt. triangle with rt. (b) 10 cm, 10 cm, 2 cm, 2 cm
angle at A. If AD ^ BC, then (c) 10 cm, 2 cm, 10 cm, 2 cm
(d) 2 cm, 10 cm, 2 cm, 10 cm
Geometry-I 223
9. In the given figure, POQ is a 11. In the figure given below,
diameter and PQRS is a cyclic radius of a greater circle is r
quadrilateral. If ÐPSR = 150°, cm. Find the area of non-
find ÐRPQ shaded portion :
R Q

O
O
S

P
(a) 50° (b) 60° pr 2
(c) 90° (d) 45° (a) sq. cm
4
10. In the figure given below, PT
is a tangent to the circle. Find 3pr 2
PT if AT = 16 cm and AB = 12 (b) sq. cm
4
cm.
(c) 2p r2 sq. cm
P
4pr 2
(d) sq. cm
3
T 12. Find the number of diagonals
of a regular polygon of which
B each entiror angle is 36°?
A (a) 30 (b) 27
(a) 5 cm (b) 6 cm (c) 35 (d) None of these
(c) 8 cm (d) 10 cm
EBD_7382
224 Geometry-I

Hints & Solution

1. (a) Shortcut Approach -1


2.
3.
(b) Shortcut Approach -1
(a) Shortcut Approach -1, 2 \
1
2 ( b + c ) AD = 12 bc .
2 2

4. (c) Use Shortcut Approach -3


5. (c) AB = PQ = 26 CM Hence
AND PO = OQ = 13CM bc
AD =
CO = (PO) - (PC) 2 2
b + c22
7. (b) Let AB = 9 cm, BC = 7
CO = (13) 2 - (5) 2 cm and CA = 6 cm
= CO = 12 cm Let, x, y, z be the radii of
\ CD = 2 CO = 24 cm circles with centre A, B,
C C respectively.
Q Then, x + y = 9, y + z = 7
P O and z + x = 6.
Adding, we get 2(x + y
D z) = 22
6. (c) In right angled DCAB, Þ x + y + z = 11
BC2
= AB2 + AC2 = c2 +b2

or BC = b 2 + c 2 A
Area of DABC = 1/2 x
base × height = 1/2 bc x
Also area of DABC = 1/ y
2 (BC) (AD) z z
B y z C
1
= b 2 + c 2 ´ AD
2
C
\ x = [(x + y + z) – (y +z)]
= (11 – 7) cm = 4 cm.
Similarly, y = (11 – 6) cm
= 5 cm and z
b D = (11 – 9) cm = 2 cm.
Hence, the radii of circle
with centres A, B, C are
4 cm, 5 cm and 2 cm
A B respectively.
c
Geometry-I 225
8. (a) Since OM ^ BC, a chord AC = AM + MC = 6 + 4 =
of the circle. 10 cm
\ it bisects BC. BD = BM + MD = 4 + 6 =
1 10 cm.
\ BM = CM = (BC) 9. (b) Shortcut Approach-5
2 10. (c) Shortcut Approach-4
1 11. (b) Required area
= (8) = 4 cm. = Area of 2 small semi-
2
Since OM ^ AD, a circles
chord of the circle. + Area of big
\ it bisects AD. semi circle
2 2
1 æ rö æ rö
\ AM = MD = pç ÷ pç ÷
2 è 2ø è 2ø pr 2
Þ + +
1 2 2 2
AD = (12) = 6 cm.
2 2
æ rö pr 2
Now, AB = AM – BM = Þ pç ÷ +
6 – 4 = 2 cm. è 2ø 2
CD = MD – MC = 6 – 4 =
2 cm pr 2 pr 2 3 pr 2
Þ + = sq.cm.
4 2 4
12. (c) Use shortcut Approach-3
EBD_7382
226 Geometry-II

20 Geometry-II
Chapter

Shortcut Approach - 1
Sum of the lengths of any two sides of a triangle is greater than the length
of the third side.
1. Lengths of one of the two equal sides and third side of an isosceles
triangle is 7 cm and 15 cm. Find the perimeter of the triangle?
(a) 29 cm (b) 37 cm (c) 33 cm (d) None of these
Sol. (b) Case-I
Sides of triangle = 7, 7, 15
Case-II
Sides of triangle = 7, 15, 15
Here case I is not possible because 7 + 7 < 15
\ Perimeter of triangle = 7 + 15 + 15 = 37 cm

Shortcut Approach - 2
Difference between the tenths of any two sides of a triangle is smaller
than the length of third side.
2. Which one of the following is not a length of the sides of a triangle?
(a) 10, 17, 6 (b) 15, 22, 8
(c) 11, 5, 15 (d) 8, 7, 14
Sol. (a) Here 17 – 10 > 6
Which is not possible.

Shortcut Approach - 3
In any triangle, line segment Joining the mid points of any two sides is
Parallel to third side and half of the length of the third side.
3. In the figure given below D and E are mid points of AB and AC
respectively and BC = 20 cm then D E = ?
A

D E

B 20 cm C
Geometry-II 227
(a) 5 cm (b) 8 cm (c) 15 cm (d) 10 cm
1 1
Sol. (d) DE = BC = ´ 20 = 10 cm
2 2

Shortcut Approach - 4
Bisector of an angle (internal or external) of a triangle divides the opposite
side (internally or eternally) in the ratio of the sides containing the angle.
4. In the given figure AD is bisector of Ð BAC which meets BC at D.
AB = 10 cm, BD = 4 cm CD = 6 cm, them AC = ?
A

B C
D
(a) 12 cm (b) 15 cm (c) 16 cm (d) None of these
Sol. (b) A D is Angle bisector then
AB BD
=
AC CD
10 4 10 ´ 6
Þ = \ AC = = 15 cm
AC 6 4

Shortcut Approach - 5
If a line is drawn parallel to one side of a triangle which intersects the other
two sides in distinct points, the other two sides are divide into same ratio.
5. In triangle ABC, DE is parallel to BC. AB = 10 cm, BD = 6 cm and
AE = 8 cm them what is length of AC?
(a) 20 cm (b) 18 cm (c) 15 cm (d) None of these
A

D E

B C
Sol. (a) In DABC ,
DE P BC
AD AE 10 - 6 8 8´6
\ = Þ = Þ EC = = 12cm
BD EC 6 EC 4
Now AC = AE + EL = 8 + 12 = 20 cm
EBD_7382
228 Geometry-II

Shortcut Approach (Angle-Angle-Angle Similarity) - 6


6. Two triangles are said to be similar, if their all corresponding angles
are equal.
Sol. For example:

In DABC and DDEF, if


ÐA = ÐD
ÐB = ÐE
ÐC = ÐF
Then DABC ~ DDEF [By AAA Similarity]
If two triangles are similar, then their corresponding sides are in the
smae ratios.

Shortcut Approach (Side-Side-Side Similarity) - 7


7. Two triangles are said to be similar, if sides of one triangle are
proportional (or in the same ratio of) to the sides of the other triangle:
Sol. For example:

In DABC and DDEF, if


AB BC CA
= =
DE EF FD
Then DABC ~ DDEF [By SSS Similarity]

Shortcut Approach (Side-Angle-Side Similarity) - 8


8. Two triangles are said to be similar if two sides of a triangle are
proportional to the two sides of the other triangle and the angles
included between these sides of two triangles are equal.
Geometry-II 229
Sol. For example:

In DABC and DDEF, if


AB BC
=
DE EF
and ÐB = ÐE
Then, DABC ~ DDEF [By SAS Similarity]
9. D and E are the points on the sides AB and AC respectively of a
DABC and AD = 8 cm, DB = 12 cm, AE = 6 cm and EC = 9 cm, then
BC is equal to
2 5
(a) DE (b) DE
5 2
3 2
(c) DE (d) DE
2 3
Sol. (b) As in DADE and DABC
AD 8 2 AE 6 2
= = , = =
AB 20 5 AC 15 5
A

D E
AD AE
So, =
AB EC
B C

and ÐA = ÐA (common)
DADE ~ DABC
DE AD DE 2
\ = Þ =
BC AB BC 5
5
Þ BC = DE
2
EBD_7382
230 Geometry-II

Shortcut Approach (Right angle-Hypotenuse-


Side Similarity)- 9
10. Two triangles are said to be similar if one angle of both triangle is right
angle and hypotenuse of both triangles are proportional to any one
other side of both triangles respectively.
Sol. For example:

In DABC and DDEF, if


ÐB = ÐE [= 90°]
AC AB
=
DF DE
Then DABC ~ DDEF [By RHS similarity]
Geometry-II 231

Exercise
1. The centroid, circumcenter, If we have AB = 4 cm, ED = 3
orthocenter in a triangle– cm, CE = 4-2 cm and CD = 4-8
(a) are always coincident. cm, then the values of CA and
(b) are always collinear. CB respectively are
(c) are always the inside the (a) 6 cm, 6.6 cm
triangular area. (b) 4.8 cm, 6.6 cm
(d) always coincide in a (c) 5.4 cm, 6.4 cm
equilateral triangle and (d) 5.6 cm, 6.4 cm
otherwise collinear. 5. In a DXYZ, if the internal
2. Two equal circles of radius 4 bisector of ÐX meets YZ at
cm intersect each other such ‘P’, then ….
that each passes through the Z
centre of the other. The
length of common chord is.
P
(a) 2 3 cm (b) 4 3 cm
(c) 2 2 cm (d) 8 cm
X Y
3. E and F are points on the
XY + XZ YZ
sides PQ and PR respectively (a) =
of a DPQR. In which of the XZ PZ
following options is EF || QR?
XY XZ
(a) PE = 3.9 cm, EQ = 3 cm, (b) =
PF = 3.6 cm, FR = 2.4 cm PZ YP
(b) PE = 4 cm, EQ = 4.5 cm, XY PZ
PF = 8 cm, FR = 9 cm (c) =
XZ YP
(c) PQ = 1.28 cm, PR = 2.56
cm, PE = 0.8 cm, XZ YP
PF = 0.52 cm (d) =
XY YZ
(d) Both (b) and (c)
6. In the given figure, express x in
4. In the figure, triangle ABC is
terms of a, b and c.
similar to triangle EDC :
L
A

D a P
B C x
46° 46°
E M N K
b c
EBD_7382
232 Geometry-II
ab 10. Two circles inscribed in a
(a) x =
a+b right angled triangle have
ac radii is 12 cm & 3 cm. Find
(b) x =
b+c length of AP.
bc
(c) x = C
b+c

. .
ac Q
(d) x =
a+c P S
T
7. ABC is an isosceles triangle R
in which AB = AC = 10 cm.
BC = 12 cm. PQRS is a A M B
L
rectangle inside the isosceles (a) 3 cm (b) 4 cm
triangle. Given PQ = SR = y (c) 5 cm (d) 7 cm
cm. and PS = QR = 2x cm. then 11. In the adjoining figure
x= ÐBAC = 60° and BC = a, AC
3y = b and AB = c, then :
(a) 6 - (b) 6 + 6y A
4
4y 7x + 8y 60° c
(c) 6 + (d) b
3 4
8. The internal bisectors of ÐB
and ÐC of DABC meet at O. C a B
If ÐA = 80° then ÐBOC is : (a) a2 = b2 + c2
(a) 50° (b) 160° (b) a2 = b2 + c2 – bc
(c) 100° (d) 130° (c) a2 = b2 + c2 + bc
9. In the figure (not drawn to (d) a2 = b2 + 2bc
scale) given below, if AD = 12. In the figure given below,
points P and Q are mid points
CD = BC, and Ð BCE = 96°, on the sides AC and BP
how much is Ð DBC? respectively. Area of each
part is shown in the figure,
E then find the value of x + y.
A
C 96°

y P
R
A Q
D B
3 x
(a) 32° 7
(b) 84° B C
(c) 64° (a) 11 (b) 4
(d) Cannot be determined (c) 7 (d) 18
Geometry-II 233

Hints & Solution

1. (d) The centroid, circumcenter, (b) PE = 4 cm, QE = 4.5cm,


orthocenter always PF = 8 cm, RF = 9cm
coincide in a equilateral
PE 4 8
triangle and otherwise = =
collinear. EQ 4.5 9
2. (b) According to question PF 8
AB is a common chord. =
FR 9
O and O’ is the centre of
the circle PE PF
So, =
In DODA EQ FR
AO2 = AD2 + OD2 AB BC AC
(4)2 = (AD)2 + (2)2 4. (d) = =
DE DC CE
AD2 = 12
4 BC AC
AD = 2 3 = =
3 4.8 4.2
\ AB
BC = 6.4 AC=5.6
= 2´ 2 3 = 4 3 5. (a) The internal bisector of
3. (b) ÐX meets YZ at P
XY YP
P =
XZ PZ
Add 1 on both the sides
XY YP
Þ +1 = +1
XZ PZ
E F
XY + XZ YP + PZ YZ
Þ = =
Q R XZ PZ PZ
6. (b) In DKPN and DKLM, we
Here, EF || QR have
By the converse of basic ÐKNP = ÐKML = 46°
proportional by ÐK = ÐK (Common)
PE PF \ DKNP ~ DKML
= (By A-A criterion of similarity)
EQ FR
Out of all the given KN NP c x
Þ = Þ =
options, the following KM ML b+c a
option fulfils the above
condition.
EBD_7382
234 Geometry-II
7. (a) Using Pythagoras 10. (b)
theorem in DABL we
PQ = ( RS) - ( QS - PR )
2 2
have AL = 8cm,
Also, DBPQ ~ DBAL 2 2
PQ = (15) - (12 - 3)
BQ BL 6-x 6 PQ = 12 cm = LM
\ = Þ = or
PQ AL y 8 Q STBM is a square
MB = BT = ST = SM =
3 12 cm.
x = 6- y
4 In DAPR & DASQ,
A
AP PR
=
y AQ SQ
P Q
10
cm

AP 3
cm
10

=
2x 2x AP + PQ 12
AP 1
y =
B S R C AP + 12 4
12 cm 4AP = AP + 12
3AP = 12
\ AP = 4 cm.
1
8. (d) ÐBOC = 90° + ÐA b 2 + c2 - a 2
2 11. (b) cos A =
= 90° + 40° = 130° 2bc
9. (c) (cosine rules)
E 1 b 2 + c2 - a 2
=
2 2bc
C (Q cos 60° = 1/2)
96°
x \ a2 = b2 + c2 – bc
12. (d) We have a theorem
which says
If ABC is a triangle in
x 2x 2x which D is the mid-point
A D B of BC and E is the mid-
Let ÐCAD = ÐACD = x point of AD then area
At point C, A
x + (180° – 4x) + 96°
= 180°
Þ 180° - 3x + 96° = 180° R P
\ x = 32° Q
Hence, ÐDBC = 2 × 32
= 64º
B C
Geometry-II 235

1 Also, ar (DBPC) = ar
( DBED) = (DBAP)
4 (Q BP is median)
area (DABC) Þ 7 + x = 3 + y Þ 4 = y – x ...(b)
So, by applying this from (a) and (b)
theorem we get 3x = 10 + (4 + x)
1 Þ 3x = 14 + x Þ x = 7
ar (DPQC) = and y = 11
4
ar (DBAC) Thus, x + y = 7 + 11 = 18
1
Þ x = (10 + x + y) Þ 3x = 10 + y
4
..(a)
EBD_7382
236 Coordinate Geometry

21 Coordinate
Chapter Geometry

Shortcut Approach (Distance Formula) - 1


The distance between two points P (x1, y1) and Q (x2, y2)
is given by PQ = ( x2 - x1 )2 + ( y2 - y1 )2

Distance of point P (x, y) from the origin = x2 + y 2


1. If distance between the point (x, 2) and (3, 4) is 2, then find the value
of x.

Sol. 2 = ( x - 3) 2 + (2 - 4) 2
Þ 2 = ( x - 3)2 + 4
Squaring both sides
4 = (x – 3)2 + 4 Þ x – 3 = 0 Þ x = 3
2. Find the distance between the points
A(– 6, –1) and B(– 6, 11)
Sol. Here the points are A(–6, –1) and B(–6, 11)
By using distance formula, we have
AB = {-6 - (-6)}2 + {11 - (-1)}2 = 02 + 12 2 = 12
Hence, AB = 12 units.

Shortcut Approach (Ratio Formula) - 2


(i) Co-ordinates of a point which divides the line segment joining two
points P (x1, y1) and Q (x2, y2) in the ratio m1 : m2 are
æ m1 x2 + m2 x1 m1 y2 + m2 y1 ö
çè m + m , m + m ÷ø
1 2 1 2

If m1 = m2, then the point will be the mid point of PQ, whose co-
ordinates
æ x + x2 y1 + y2 ö
=ç 1 ,
è 2 2 ÷ø
(ii) When we need to find the ratio in which a point on a line segment
divides it, we suppose the required ratio as k : 1 or m/n : 1.
Coordinate Geometry 237
3. Find the points of trisection of line joining the pointsA (2, 1) and B
(5, 3).
1 2
Sol. (2, 1) (5, 3)
A P1 P2 B
2 1

æ 1 ´ 5 + 2 ´ 2 1 ´ 3 + 2 ´ 1ö æ 5 ö
P1 (x1, y1) = ç
è 1+ 2
,
1+ 2 ÷ø = çè 3, 3 ÷ø

æ 2 ´ 5 + 1 ´ 2 2 ´ 3 + 1 ´ 1ö æ 7 ö
P2 (x2, y2) = ç , = ç 4, ÷
è 2 +1 2 + 1 ÷ø è 3 ø
4. Prove that points A (1, 1), B (– 2, 7) and C (3, – 3) are collinear.

Sol. AB = (1 + 2)2 + (1 - 7) 2 = 9 + 36 = 3 5

BC = ( -2 - 3)2 + (7 + 3) 2 = 25 + 100 = 5 5

CA = (3 - 1)2 + ( -3 - 1)2 = 4 + 16 = 2 5

Clearly, BC = AB + AC. Hence A, B, C are collinear.

Shortcut Approach - 3
Let A (x1, y1), B (x2, y2) and C (x3, y3) are vertices of any triangle ABC, then
Co-ordinates of centroid,
æ x + x2 + x3 y1 + y2 + y3 ö
G= ç 1 , ÷ø
è 3 3
5. The two vertices of a triangle are (6, 3) and (–1, 7) and its centroid is
(1, 5). Find the third vertex.
Sol. Let ABC be a triangle whose vertices are A = (6, 3), B = (–1, 7), C = (x, y)
and centroid G = (1, 5)

Then using the formula, for the coordinates of centroid; we get


EBD_7382
238 Coordinate Geometry

6 + (–1) + x 3+ 7 + y
1= and 5 = Þ x = – 2 and y = 5
3 3
Hence, the third vertex is C = (–2, 5)

Shortcut Approach - 4
Let A (x1, y1), B (x2, y2) and C (x3, y3) are vertices of any triangle ABC,
then co-ordinates of incentre,
æ ax + bx2 + cx3 ay1 + by2 + cy3 ö
I= ç 1 , ,
è a+b+c a + b + c ÷ø
where a, b, c are length of the sides opposite to vertices A, B, C respectively
of triangle ABC.
6. Find incentre (I) of triangle whose vertices are A (– 36, 7), B
(20, 7), C (0, – 8).
Sol. Using distance formula

a = BC = 20 2 + (7 + 8) 2 = 25

b = CA = 362 + (7 + 8)2 = 39

c = AB = (36 + 20) 2 + (7 - 7)2 = 56

æ 25( -36) + 39(20) + 56(0) 25(7) + 39(7) + 56( -8) ö


I= ç , ÷ø
è 25 + 39 + 56 25 + 39 + 56
I = (– 1, 0).

Shortcut Approach - 5
Let A (x1, y1), B (x2, y2) and C (x3, y3) are vertices of a triangle, then area
of the triangle ABC
1
= x1 ( y2 - y3 ) + x2 ( y3 - y1 ) + x3 ( y1 - y2 )
2
When one vertex is origin i.e. if the vertices are (0, 0), (x1, y1) and (x2, y2)
then area of the triangle
1
= x1 y2 - x2 y2 .
2
7. The area of a triangle is 5. Two of its vertices are (2, 1) and (3, –2).
The third vertex lies on the line y = x + 3. Find the third vertex.
Sol. Let the third vertex be (x3, y3), Area of the triangle
1
= | [ x1 ( y2 - y3 ) + x2 ( y3 - y1 ) + x3 ( y1 - y2 )] |
2
As , x1 = 2, y1 = 1, x2 = 3, y2 = – 2 and Area of D = 5
Coordinate Geometry 239
1
\ 5= | 2( -2 - y3 ) + 3( y3 - 1) + x3 (1 + 2) |
2
Þ 10 = | 3x3 + y3 – 7 | Þ 3x3 + y3 –7 = ± 10
Taking positive sign, 3x3 + y3 – 7 = 10
Þ 3x3 + y3 = 17 ... (1)
Taking negative sign, 3x3 + y3 – 7 = – 10
Þ 3x3 + y3 = –3 ... (2)
Given that (x3, y3) lies on y = x + 3
So, –x3 + y3 = 3 ... (3)
7 13
Solving eqs. (1) and (3), x3 = , y3 =
2 2
-3 3
Solving eqs. (2) and (3), x3 = , y3 = .
2 2
æ 7 13 ö æ -3 3 ö
So the third vertex are ç , ÷ or ç , ÷
è2 2ø è 2 2ø
EBD_7382
240 Coordinate Geometry

Exercise
1. The equation of the line 7. Find the co-ordinates of the
passing through (2, – 4) and centroid of a triangle whose
parallel to X – 2Y – 5 = 0, is vertices are (0, 6) (8, 12) and
(a)2X + Y + 3 = 0 (b) X – 2Y (8, 0).
– 10 = 0 æ 16 ö æ 16 ö
(a) çè , 6÷ø (b) çè 6, ÷ø
(c)X – 2Y + 8 = 0 (d) X – 2Y 3 3
+ 13 = 0
(c) (6, 5) (d) (6, 3)
2. For what value of k, the
equations 3x – y = 8 and 9x – 8. The centroid of the triangle
ky = 24 will have infinitely whose vertices are (3, 10), (7,
many solutions ? 7), (–2, 1) is
(a)6 (b) 5 (c)3 (d) 1 (a)(8/3, 6) (b) (6, 8/3)
3. The two vertices of a triangle (c)(– 4, – 7/3) (d) N o n e
ar e (3, –5) and of these
(–7, 4). If its centroid is (2, – 9. Find the third vertex of the
12), find the third vertex.
triangle whose two vertices
(a)(10, – 35) (b)(–2, 10)
are (–3, 1) and (0, –2) and the
(c)(10, 35) (d)(–3, 10) centroid is the origin.
4. The vertices of triangle ABC (a)(2, 3)
are A (4, 4), B (6, 3), C (2, –1);
then angle ABC is equal to æ -4 14 ö
(a)45° (b) 90° (b) çè , ÷ø
3 3
(c)60° (d) None of these
(c)(3, 1)
5. If two opposite vertices of a (d) (6, 4)
rectangle are (4, 2) and (10, 10. The incentre of the triangle
6) and the equation of other
diagonal is x – 3y + k = 0. with vertices (1, 3), (0, 0)
Find the value of k? and (2, 0) is
1 æ 3ö
(a) (b) 3 (c)5 (d) 4 (a) çç1, 2 ÷÷
2 è ø
6. Let the vertices of a triangle
ABC be (4, 4), (3, 5) and (–1, æ2 1 ö
(b) çè 3 , ÷ø
–1), then the triangle is : 3
(a)scalene æ2 3ö
(c) çç , ÷÷
(b)equilateral è3 2 ø
(c)right angled æ 1 ö
(d)None of hese (d) çè1, ÷ø
3
Coordinate Geometry 241

æa ö whose vertices are (–2, 3), (2,


11. If P çè , 4÷ø is the mid-point –1), and (4, 0) are
3
of the line segment joining the (a) (3, 5); 5 2
points Q(– 6, 5) and R (– 2, 3),
then the value of a is 5 2
(b) (0, 0);
(a) – 4 2
(b) – 12
(c) 12 æ1 3ö 2
(c) ç , ÷;
(d) – 6 è2 2ø 2
12. The centre and radius of the
æ3 5ö 5 2
circumcircle of the triangle (d) ç , ÷ ;
è2 2ø 2
EBD_7382
242 Coordinate Geometry

Hints & Solution

1. (b) Line parallel to X – 2Y – diagonal AC and BD


5 = 0 will be
X – 2Y + K = 0. co-ordinate
Put the point in this
equation we have æ 4 + 10 2 + 6 ö
of O = ç , ÷
2+8+K=0 è 2 2 ø
or K = – 10 or line is X – = (7, 4)
2Y – 10 = 0 Point O is also situated
2. (c) For infinite solution on line
3 -1 8 x – 3y + k = 0
= = Þ7–3×4+k=0Þ7–
9 - k 24 12 + k = 0 Þ k = 5
1 1 6. (c) Use Shortcut Approach -1
Þ = Þk = 3
3 k 7. (a) Centroid of a triangle
3. (a) Use Shortcut Approach -3
4. (d) Angle ABC is the angle æ x + x 2 + x3
between the lines AB ºç 1 ,
è 3
and BC.
Now slope of line AB = y1 + y 2 + y3 ö
3- 4 1 3 ÷ø
m1 = =-
6-4 2
and slope of line BC æ 0+8+8
ºç ,
= m2 è 3
-1 - 3 - 4 6 + 12 + 0 ö
= = =1
2-6 - 4 ÷ø
3

Now, æ 16 ö
º ç , 6÷
è 3 ø
m1 - m2
tan q = = -1
1 + m1m2 8. (a) Centroid =
Þ q = 135º æ 3 + 7 - 2 10 + 7 + 1ö
çè , ÷ø
5. (c) 3 3
D C (10, 6) æ8 ö
= ç , 6÷
è3 ø
O 9. (c) Use Shortcut Approach -3
10. (d) Clearly, the triangle is
A (4, 2) B equilateral.
O is the mid-point of
Coordinate Geometry 243

(b) As given that P æç , 4 ö÷


A (1, 3 ) a
11.
è3 ø
is the mid-point of line
2 2 segment joining the
3
point Q (–6, 5) and R (–
60° (1,0) 2, 3)
(0,0) B 2 C(2, 0)
x Since, P is the mid-point
So, the incentre is the of QR, then
same as the centroid. a æ -6 + (-2) ö
\ Incentre =ç ÷ø
3 è 2
æ 1+ 0 + 2 3 + 0 + 0ö Þ a = – 12
=ç , ÷
è 3 3 ø 12. (d) Use Shortcut Approach -2
æ 1 ö
= ç1, ÷
è 3ø
EBD_7382
244 Mensuration Plane Figures

22 Mensuration
Chapter Plane Figures

Shortcut Approach - 1
Side of the largest possible square tiles of equal size so that the tiles
exactly fit in the floor of a rectangular room
= HCF of length and breadth of the room.
1. A room, 39 m 10 cm long and 35 m 70 cm broad, is to be paved with
equal square tiles. Find the side of the largest tile so that the tiles
exactly fit in the floor of the room.
Sol. Side of the largest possible tile
= HCF of 39.10 m and 35.70 m = 1.70 m = 1 m 70 cm

Shortcut Approach - 2
When area of the path of uniform width is given, then area of the square
garden enclosed by the path
2
é Area of path - 4 ´ (Width of the path)2 ù
=ê ú
ë 4 ´ Width of path û
2. A path 3 m wide running all around a square garden has an area of
5300 sq. m. Find the area of the garden enclosed by the path.
Sol. Area of the square garden
2
é 5300 - 4 ´ (3)2 ù (5264)2
=ê ú = = 2309141.33 sq. m.
ë 4´3 û 12

Shortcut Approach - 3
If from centre of each side of a rectangular field of length ‘l’ and breadth
‘b’, a path of width ‘w’ goes across to the centre of the opposite side, then
area of the path
= w (l + b – w)
3. A rectangular field is of measures 29.35 m by 15 m 5 dm. From the
centre of each side, a path 2.5 m wide goes across to the centre of
the opposite sides. What is the area of the path?
Mensuration Plane Figures 245
Sol. Area of the path = w(l + b – w)
= 2.5 ´ (29.35 + 15.5 - 2.5) sq. m.
= 2.5 ´ 42.35 sq. m. = 105.88 sq. m.

Shortcut Approach - 4
Area of a rhombus, length of whose one side and diagonal are given
2
æ diagonal ö
= (diagonal) ´ (side) 2 - ç ÷ø
è 2
4. Find the area of a rhombus, length of whose one side is 30 cm and
length of one diagonal is 42 cm.
Sol. Area
2
æ 42 ö
= 42 ´ (30)2 - ç ÷
è 2ø

= 42 ´ 900 - 441 = 42 ´ 449


= 42 ´ 21.2 = 890.4 cm 2

Shortcut Approach - 5
If taking each vertex of an equilateral triangle as centre, a circle is drawn
with radius of length half of a side of the equilateral triangle as shown in
the figure, then area of the triangle which is not included in the circles
(shaded region)

æ pö
= ç 3 - ÷ (radius)2
è 2ø

5. There is an equilateral triangle of which length of each side is 4


cm. Taking each vertex of the triangle as centre, a circle of radius
2 cm is drawn. Find the area of the triangle which is not included in
the circles.
EBD_7382
246 Mensuration Plane Figures
Sol. Required Area
æ pö æ 22 ö
= ç 3 - ÷ (radius) 2 = ç 1.732 - ÷ ´ 4
è 2 ø è 14 ø

= (4 - 3.14) ´ (2)2 = 3.44 cm 2

= (1.732 - 1.294) ´ 4 = 1.75 cm2

Shortcut Approach - 6
If taking each vertex of a square as centre, a circle is drawn with radius of
length half of a side of the square as shown in the figure (shaded region),
then area of the square which is not included in the circles

6. Taking each vertex of a square of side length 6 cm as centre, a


circle is drawn with radius of length half of a side of the square.
Find the area of the square which is not included in the circles.
Sol. Required Area
= (4 - p) ´ (radius)2 = (4 - 3.14) ´ (3) 2
= 0.86 × 9 = 7.74 cm2

Shortcut Approach - 7
If the sides of a triangle, rectangle, square, circle, rhombus (or any 2-
dimensional figure) are increased (or decrease) by x %, then its area is
æ x2 ö
increased or decrease by ç 2 x + %
è 100 ÷ø
7. If the sides of a rhombus increased by 20%, then find the percentage
increase in area of the rhombus.
Mensuration Plane Figures 247

æ (20) 2 ö
Sol. Increase in Area = ç 2 ´ 20 + % = (40 + 4)% = 44%.
è 100 ÷ø

Shortcut Approach - 8
If the length of a rectangle increased by x %, then the percentage decrease
in width of the rectangle so that area of the rectangle remains the same
100 x
= %
100 + x
8. Let the length of a room be 60 cm and width be 40 cm. If the owner
wants to increase the length of the room by 20% without changing
the area of the room, then find the percentage decrease in the width
of the room.
Sol. Required percentage decrease in width of the room
100 ´ 20 100 ´ 20
= %= % = 16.66%
100 + 20 120

Shortcut Approach - 9
If the length of a rectangle decreased by x %, then the percentage increase
in width of the rectangle so that area of the rectangle remains the same
100 x
= %
100 - x
9. Let the length and width of a runway of an airplane be 1800 m and
100 m respectively. If length of runway is decreased by 10% without
changing the area of runway, then find the percentage increase in
width of runway.
Sol. Required percentage increase in width of runway
100 ´ 10 100 ´ 10
= %= % = 11.11%
100 - 10 90

Shortcut Approach - 10
If length and breadth of a rectangle is increased by x and y percent
æ xy ö
respectively, then area of the rectangle is increased by çè x + y + ÷%
100 ø
10. If length of a rectangle increases by 20% and breadth of the
rectangle is decreases by 16%, then find the % change in area of
the rectangle.
EBD_7382
248 Mensuration Plane Figures
Sol. Percentage change in area
æ xy ö
=çx+ y+ ÷%
è 100 ø
Here x = 20, y = –16 (as breadth decreases)
\ Percentage change in area
é 20 ´ (-16) ù
= ê 20 + ( -16) + %
ë 100 úû

æ 16 ö
= ç 4 - ÷ % = 0.8%
è 5ø
Positive sign of 0.8% means increase in area.

Shortcut Approach - 11
If the area of a square is x cm2, then area of the circle formed by the same
perimeter
4x
= cm 2
p
11. A wire is in the form of a square enclosing an area of 44 cm2. If the
same cord is bent into a circle, then find the area of that circle.
Sol. Area of the circle
4 x 4 ´ 44 ´ 7
= = = 56 cm 2
p 22

Shortcut Approach - 12
If r be the radius of a circle, then the area of a square inscribed in a circle
is 2r2.
12. Find the area of a square inscribed in a circle of radius 10 cm.
Sol. Area of the square
= 2r2 = 2 × (10)2 = 200 cm2

Shortcut Approach - 13
Area of the largest triangle inscribed in a semi-circle of radius r cm is r2
cm2.
13. Find the area of the largest triangle inscribed in semi-circle of
radius 8 cm.
Sol. Area of the largest triangle = r2 = (8)2 = 64 cm2
Mensuration Plane Figures 249

Exercise
1. One diagonal of a rhombus is 5. A circle and a rectangle have
24 cm whose side is 13 cm. the same perimeter. The sides
Find the area of the rhombus. of the rectangle are 18 cm and
(a) 25 sq. cm 26 cm. What is the area of the
(b) 312 sq. cm. circle ?
(c) 125 sq. cm. (a) 88 cm2
(d) 120 sq. cm.
(b) 154 cm2
2. The area of a square field is
576 km2. How long will it take (c) 1250 cm2
for a horse to run around at (d) 616 cm2
the speed of 12 km/h ? 7. ABCD is a square in which
(a) 12 h (b) 10 h DOBC is an equilateral
(c) 8 h (d) 6 h triangle then find ÐDOA.
3. The length and breadth of a A D
square are increased by 60%
and 40% respectively. The O
area of the resulting rectangle
exceeds the area of the 60°
I
square by :
(a) 224% (b) 24% 60° 60°
(c) 124% (d) 100% B C
(a) 60° (b) 45°
6. If every side of a triangle is
(c) 150° (d) 90°
doubled, then increase in area
of the triangle is : 8. In the given figure, AB is a
(a) 200% line of length 2a, with M as
(b) 300% mid-point. Semi–circles are
(c) 400% drawn on one side with AM,
(d) None of these and AB as diameters. A
4. The perimeter of the figure circle with centre O and
given below correct to one radius r is drawn such that
decimal place is : this circle touches all the
three semi-circles. What is
2m
the value of r?
10m

2m

2m 2m
20 m

(a) 56 m (b) 56.6 m (c) A M B


57.2 m (d) 57.9 m
EBD_7382
250 Mensuration Plane Figures
2a a
(a) (b) 8 3
3 2 (a) (b)
a a 11 11
(c) (d)
3 4 5 6
8. Three circles with centres A, (c) (d)
11 11
B and C and with unit radii
19. ABCDEF is a regular
touch each other at O, P and
hexagon of side 6 cm. What
Q. Find the area of the shaded
is the area of triangle BDF?
region .
O E D
A B
Q F C
P

C A B

(a) 32 3 cm 2
(a) 0.16 sq. units
(b) 1.21 sq. units (b) 27 3 cm2
(c) 0.03 sq. units (c) 24 cm2
(d) 0.32 units (d) None of these
18. ABCD is a square, 4 equal 30. A circle is inscribed in an
circles are just touching each equilateral triangle of side a.
other whose centres are the What is the area of any
vertices A, B, C, D of the
square inscribed in th is
square. What is the ratio of
circle?
the shaded to the unshaded
area within square? a2 a2
(a) (b)
.
D .C 3 4

a2 a2
(c) (d)
.
A .B 6 8
Mensuration Plane Figures 251

Hints & Solution


AB = BO
1. (d) Use Short Approach -1
2. (c) Area of field = 576 km2. 180 - 30
ÐBAO = ÐBOA = = 75°
Then, 2
each side of field = ÐAOB = ÐDOC = 75°
576 = 24 km ÐAOD = 360° – [ÐAOB +
Distance covered by the ÐDOC + ÐBOC]
horse = 360° – 210° = 150°.
= Perimeter of square 8. (c)
field = 24 × 4 = 96 km
\ Time taken by horse
r .
r
O
distance 96 a r
=
speed
=
12
=8h Aa C
2
. a M a
. a B
3. (c) Use Short Approach -10 2 2 2 2
4. (b) Required perimeter 2 2
æa ö æ aö 2
æ 2 ´ 22 ´ 2 ö çè + r÷ø = çè ÷ø + ( a - r )
= 4´ ç ÷ + 2 (16 + 6 ) = 56.57 m 2 2
è 4´7 ø
5. (d) Perimeter of the circle = Þ
2pr = 2(18 + 26) a2 a2
+ r 2 + ar = + a 2 + r 2 - 2ar
22 4 4
Þ 2´ ´ r = 88
7
æa ö
Þ r = 14 çè + r ÷ø
2 (a – r)
\ Area of the circle
2 22
= pr = ´ 14 ´ 14 = 616 cm 2 .
7
a
6. (b) Use Short Approach -7
7. (c) 2
360 – (75 + 75 + 60) a2
= 150° Þ 3ar = a2 Þ r = Þ
A D 3a
a
O r=
3
60° 9. (a) Use Short Approach -5
10. (b)
60° 60° 11. (b) Use Short Approach -13
B C
90° – 60° = 30°
EBD_7382
252 Mensuration Plane Figures
E D 12. (c)

F . C
A

A B
Area of DBDF 1
=
Area of hexagon 2
B C
As there is a perfect
symmetricity. Altitude of equilateral
\ Area of hexagon
3
3 3 2 triangle = a
= ´ ( 6 ) = 54 3 cm 2 2
2 Radius of incircle
\ Area of
=DBDF 27 3cm 2 3 1 = a
= a.
Alternatively: 2 3 2 3
Subtract the areas of Diameter of incircle
three triangles DEF, = Diagonal of square
BAF, BCD from the area
of hexagon. a a
Area of Diameter = ´2 =
2 3 3
1
D DEF = ´ 6 ´ 6sin120° a
2 Diagonal of square =
3
= 9 3 cm 2
\ Area of all the three æ a ö
2

triangles = 27 3 cm ( Diagonal ) 2 ç ÷


Area =
\ Required area of 2 2
DBDF= 54 3 - 27 3
a2
= 27 3 cm 2 = .
6
23 Mensuration
Chapter -Solid Figures
Shortcut Approach - 1
If length, breadth and height of a cuboid are changed by x %, y % and z %
respectively, then its volume is increased by
é xy + yz + zx xyz ù
êx + y + z + + ú%
ë 100 (100)2 û
1. If length, breadth and height of a cuboid increases by 15%, 10%
and 20% respectively, then find the percentage increase in the
volume of the cuboid.
Sol. Percentage increase in volume
é xy + yz + zx xyz ù
= êx + y + z + + ú%
ë 100 (100)2 û

é 150 + 200 + 300 15 ´ 10 ´ 20 ù


= ê 45 + + ú%
ë 100 (100)2 û
= [45 + 6.5 + 0.3]% = 51.8%.

Shortcut Approach - 2
If side of a cube or radius (or diameter) of sphere is increased by x %, then
its volume increases by
éæ x ö
3 ù
ê ç1 + ÷ - 1ú ´ 100%
êë è 100 ø úû
2. Diameter of a sphere is increased by 20%. Find the increase in
volume of the sphere.
Sol. Percentage increase in volume
éæ 20 ö
3 ù
= êç 1 + ÷ - 1ú ´ 100%
êë è 100 ø úû
6
= ´ 100% = 120%.
5
EBD_7382
254 Mensuration- Solid Figures

Shortcut Approach - 3
Three cubes of metal whose sides are x, y and z respectively are melted to
form a new cube, if there is no loss of weight in this process. Then side of
the new cube will be

x3 + y3 + z3
3

3. The sides of three cubes of metal are 30 cm, 40 cm and 50 cm


respectively. Find the side of a resulting cube formed by melting
these cubes together.
Sol. Side of resulting cube

= 3 (30)2 + (40)2 + (50)2

= 3 900 + 1600 + 2500 = 3 21600 = 60 cm

Shortcut Approach - 4
If in a cylinder or cone, height and radius both change by x%, then its
volume changes by
éæ x ö
3 ù
ê ç1 + ÷ - 1ú ´ 100%.
êë è 100 ø úû
4. Both height and radius of a cone are increased by 50%. Find the
percentage increase in its volume.
Sol. Percentage increase in volume
éæ x ö
3 ù
= êç 1 + ÷ - 1ú ´ 100%
êëè 100 ø úû

éæ 50 ö
3 ù
= êç 1 + ÷ - 1ú ´ 100%
è 100 ø
ëê ûú
9-4
= ´ 100% = 125%.
4

Shortcut Approach - 5
If radius of a cylinder or cone is changed by x % and height is changed by
y %, then volume is changes by
é x 2 + 2 xy x 2 y ù
ê2 x + y + + ú%
ë 100 100 2 û
Mensuration- Solid Figures 255
5. Radius and height of a cylinder are increased by 10% and 20%
respectively. Find the percentage increased in volume.
Sol. Percentage increase in volume
é 100 + 400 2000 ù
= ê 20 + 20 + + %
ë 100 10000 úû

æ 1ö
= ç 45 + ÷ % = 45.2%
è 5ø

Shortcut Approach - 6
If radius of a cylinder changes by x% and height remains the same, then
volume of the cylinder changes by
éæ x ö
2 ù
ê ç1 + ÷ - 1ú ´ 100%.
êë è 100 ø úû
6. Find the percentage change in volume of a cylinder if its radius
changes by 20% but height remain the same.
Sol. Percentage change in volume
éæ x ö
2 ù
= êç1 + ÷ - 1ú ´ 100%
êëè 100 ø úû

éæ 20 ö
2 ù
= êç1 + ÷ - 1ú ´ 100%
è 100 ø
ëê ûú
36 - 25
= ´ 100% = 44%.
25
EBD_7382
256 Mensuration- Solid Figures

Exercise
1. The edge of a cube is 5. If P, R, T are the area of a
in creased by 100%, the parallelogram, a rhombus and
surface area of the cube is a triangle standing on the
increased by : same base and between the
(a) 100% (b) 200% same parallels, which of the
(c) 300% (d) 400% following is true?
2. A rectangular block has (a) R < P < T
length 10 cm, breadth 8 cm (b) P > R > T
and height 2 cm. From this (c) R = P = T
block, a cubical hole of side (d) R = P = 2T
2 cm is drilled out. Find the 6. If the radius of a circle is
volume and the surface area
diminished by 10%, the area
of the remaining solid :
is diminished by
(a) 152 cm3, 512 cm2
(a) 36% (b) 20%
(b) 125 cm3, 215 cm2
(c) 19% (d) 10%
(c) 152 cm3, 240 cm2
7. The length of a cold storage is
(d) 125 cm3, 512 cm2
double its breadth. Its height
3. The length, breadth and height
is 3 metres. The area of its four
of a cuboid are in the ratio
walls (including the doors) is
1 : 2 : 3. The length, breadth
108 m2. Find its volume.
and height of the cuboid are
increased by 100%, 200% (a) 215 m3 (b) 216 m3
(c) 217 m 3 (d) 218 m3
an d 200%, respectively.
Then, the increase in the 8. A square of side 3 cm is cut
volume of the cuboid will be off from each corner of a
rectangular sheet of length 24
(a) 5 times (b) 6 times cm and breadth 18 cm and the
(c) 12 times (d) 17 times remaining sheet is folded to
4. If h, s, V be the height, curved form an open rectangular box.
surface area and volume of a The surface area of the box is
cone respectively, then (3p (a) 423 cm2 (b) 468 cm2
Vh3 + 9V2 – s2h2) is equal to: (c) 396 cm2 (d) 612 cm2
(a) 0 (b) p 9. If length, breadth and height
of a cuboid is increased by x%,
V 36
(c) (d) y% and z% respectively then
sh V
its volume is increased by
Mensuration- Solid Figures 257
(c) 126 sq. cm
é xy + xz + yz ù (d) None of these
êx + y + z + 100 ú 11. A hemispherical bowl is filled
ê ú%
(a) ê xyz ú to the brim with a beverage.
+
ê (100)2 úû The contents of the bowl are
ë
transfered into a cylindrical
xy + xz + yz ù vessel whose radius is 50%
é
(b) êë x + y + z + 100 úû % more than its height. If the
diameter is same for both the
bowl and the cylinder, the
é xyz ù
(c) êx + y + z + ú% volume of the beverage in the
ëê (100)2 ûú cylindrical vessel is:
(d) None of these 2
10. Three cubes each of edge 3 (a) 66 %
3
cm long are placed together
as shown in the adjoining 1
(b) 78 %
figure. Find the surface area 2
of the cuboid so formed : (c) 100%
(d) More than 100%
12. The ratio of the surface area
3 cm
of a sphere to the curved
m

surface area of the cylinder


3c

3 cm 3 cm 3 cm circumscribing the sphere is:


(a) 182 sq. cm (a) 1 : 2 (b) 1 : 1
(b) 162 sq. cm (c) 2 : 1 (d) 2 : 3
EBD_7382
258 Mensuration- Solid Figures

Hints & Solution


1. (c) Let each edge of smaller 4. (a) 3pVh3 + 9V2 – s2h2
cube = 1 m
\ Each edge of larger cube æ1 2 ö 3
= çè pr h÷ø h
=2 m 3
and Surface area of 2
smaller cube æ1 2 ö
+ 9 ç pr h ÷
= 6 × (1)2 = 6 m2 è3 ø
\ Surface area of larger – p2r2 (r2 + h2)h2
cube = (prh2)2 + (pr2h)2
= 6 × (2)2 = 24 m2 – (prh)2 (r2 + h2)
\ % increase in surface = (prh)2 (r2 + h2) –
area
(prh)2 (r2 + h2) = 0
24 - 6 5. (d) Parallelogram Area = l × b
= ´ 100 = 300%
6 Rhombus area = l × b
Alternatively : l´b
Triangle area =
S2 æ e 2 ö
2
S 2
4
=ç ÷ Þ 2 = Therefore R = P = 2T.
S1 è e1 ø S1 1
6. (c) Use Short Approach -2
where S = surface area, 7. (b) Let L be the length and
e =edge of cube. B be the breadth of
\ Percentage increase in cold storage.
surface area L = 2B, H = 3 metres
4 -1 Area of four walls = 2[L
= ´ 100 = 300%
1 × H + B × H] =108
2. (c) Net volume = (10 × 8 × Þ 6BH = 108 Þ B
2) – (2 × 2 × 2) = 6 \ L = 12, B = 6, H
= 3 Volume = 12 × 6 × 3
= 152 cm3
= 216 m3
Net surface area 8. (c)
= 2 (10 × 8 + 8 × 2 + 2 × 3 3
10) 3 3

+ 4 (2 × 2) – 2 (2 × 2) 12

= 240 cm2 3 3
3 3
3. (d) Use Short Approach -1 18
Mensuration- Solid Figures 259
= 18 cm, b = 12 cm, h =
3 cm 12. (b)
S = 2( h + bh) + b
.
r
{Box is open from upper
side}
9. (a) Use Short Approach -1 r
= 2 (54 + 36) + 216
r
= 396 cm2
10. (c) Required surface area Height of the cylinder =
= 2 (9 × 3 + 3 × 3 + 3 × 9) Diameter of sphere.
= 126 cm2 CSA = 2prh = 2pr . 2r = 4pr2
Total surface area of the
11. (d) Use Short Approach -2
sphere = 4pr2
Required ratio = 4pr2 : 4pr2 = 1
:1
EBD_7382
260 Set Theory

24 Set Theory
Chapter

Shortcut Approach - 1
A' = U - A = {x : x Î U but x Ï A}

1. If U = {1, 2, 3, 4, 5, 6, 7, 8} and A = {2, 4, 6,8} then A' = ?


(a) {1, 2, 3, 4} (b) {3, 4, 5, 7}
(c) {1, 3, 5, 7} (d) None of these
Sol. (c) A ' = U - A = {1, 3, 5, 7}

Shortcut Approach - 2
A - B = {x : x Î A but x Ï B}
2. If A = {2, 4, 6, 8}, B = {2, 6, 10} them A–B = ?
(a) {2} (b) f (c) {4, 6} (d) {4, 8}
Sol. (d) A–B = {4, 8}

Shortcut Approach - 3
Number of Subsets of a set having n elements = 2n
3. Find the number of subsets of A where A = {1, 3, 5, 7, 9}
(a) 16 (b) 10 (c) 32 (d) 64
Sol. (c) Number of subsets of A = 2n = 25 = 32

Shortcut Approach - 4
A È B = {x : x Î A or x Î B or x Î A and B}
4. If A = {x : x is an odd number less than 100} and B = {x : x is a even
number less than 100} then what is A È B ?
(a) {x : x is a prime number less than 100}
(b) {x : x is whole number less than 100}
(c) {x : x is a natural number less than 100}
(d) None of these
Sol. (c)
Set Theory 261

Shortcut Approach - 5
A Ç B = {x : x Î A and x Î B}
5. If (A) = {1, 2, 3, 4, 5, 6, 7, 8, 9} and B = {1, 3, 5, 7, 9} them A Ç B = ?
(a) {1, 3, 5, 7, 9} (b) {2, 4, 6, 8}
(c) {1, 2, 3, 4, 5} (d) None of these
Sol. (a) A Ç B = {1, 3, 5, 7, 9}

Shortcut Approach - 6
If A and B are disjoint sets them A Ç B = f
6. If A = {x : x is a number less than 100} and B = {x : x is a even
number less than 100} then A Ç B = ?
(a) {1, 3, 5, - - - - 99} (b) {2, 4, 6, 8 - - - - 98}
(c) f (d) None of these
Sol. (c) There A is a set of add number less than 100 and B is a set of
even number less than 100
therefore, A Ç B = f

Shortcut Approach - 7
If A and B are two finite sets then
n(AÈ B) = n(A) + n(B) - n (A Ç B)
7. If in a class, 20 students play cricket 40 students play football and
10 students play both cricket and football then find the is number of
students in the class?
(a) 40 (b) 60 (c) 70 (d) 50
Sol. (d) n (C È F) = n (C) + n (F) - n (C Ç F) = 20 + 40 –10 = 50

Shortcut Approach - 8
If A, B and C are three finite sets, then
n(AÈ BÈ C) = n(A) + n(B) + n(C) - n(A Ç B) - n(BÇ C)
- n(CÇ A) + n(AÇ BÇ C)
8. In a group 60 persons can speak in English, 70 can speak in Hindi
and 40 can speak in Bangla. 10 persons can speak in all three
languages. 20 persons can speak in Hindi and English, 10 persons
EBD_7382
262 Set Theory
can speak in English and Bangla and 30 persons can speak in
English and Bangla then find the number of persons in the group.
(a) 100 (b) 110 (c) 120 (d) 125
8. (c) Hence, required number of persons = 60+70+40-20-10-
30+10=120
n(E È H È B) = n(E) + n(H) + n(B) – n(E Ç H)
–n(H Ç B) – n(B Ç E) + n(E Ç H Ç B)
Set Theory 263

Exercise
1. If X and Y are two sets such 5. In an examination out of 100
that n ( X ) = 17, students, 75 passed in
n ( Y ) = 23 and n ( X È Y ) = 38, English 60 passed in
then n ( X Ç Y ) Mathematics and 45 passed
(a) 2 (b) 1 in both English and
(c) 3 (d) 4 Mathematics. What is the
2. In a certain office, 72% of the number of students passed in
workers prefer tea and 44% exactly one of the two
subjects?
prefer coffee. If each of them
(a) 45 (b) 60
prefers tea or coffee and 40
(c) 75 (d) 90
like both, the total number of
6. In a car agency one day 120
workers in the office is : cars were decorated with three
(a) 200 (b) 240 different accessories viz.,
(c) 250 (d) 320 power window, AC and music
3. A survey show that 63% of system. 80 cars were
the Indians like cheese decorated with power
whereas 76% like apples. If windows, 84 cars were
x% of the Indians like both decorated with AC and 80
cheese and apples, then find cars were decorated with
the range of x. music systems. What is the
(a) 0 £ x £ 23% minimum and maximum
number of cars which were
(b) 0 £ x £ 39%
decorated with all of three
(c) 4 £ x £ 35% accessories?
(d) 6 £ x £ 33% (a) 10, 61
4. In a town three newspapers (b) 10, 45
A, B and C are published. (c) 25, 35
42% of the people in that (d) None of these
7. In an examination 70% of the
town read A, 68% read B,
candidates passed in English,
51% read C, 30% read A and
65% in Mathematics, 27 %
B, 28% read B and C, 36% A failed in both the subjects. If
and C and 18% do not read 248 candidates passed in
any paper. Find the % of both the subjects, then find
population of town that the total number of
reads all the three. candidates.
(a) 15% (b) 25% (a) 200 (b) 400
(c) 20% (d) 35% (c) 300 (d) 100
EBD_7382
264 Set Theory
8. Out of 800 boys in a school, product P 1 an d 1450
224 played cricket, 240 consumers like product P2.
played hockey an d 336 What is the least number
played basketball. Of the that must have liked both
tota l 64 played both the products?
basketball and hockey, 80 (a) 1150 (b) 2000
played cricket and basketball (c) 1170 (d) 2500
and 40 played cricket and 11. In a town of 10000 families,
hockey, 24 played all the it was found that 40%
three games. The number of families buy newspaper A,
boys who did not play any 20% families buy newspaper
game is : B and 10% families buy
(a) 128 (b) 216 newspaper C, 5% buy A and
(c) 240 (d) 160 B, 3% buy B and C and 4%
9. From 50 students taking buy A and C. If 2% families
examination in Mathematics, buy all of three newspapers,
Physics and Chemistry, each then the number of families
of the students has passed which buy A only, is
in at least one of the subject, (a) 4400 (b) 3300
37 passed Mathematics, (c) 2000 (d) 500
24 Physics and 43 Chemistry. 12. A survey of 500 television
Atmost 19 pa ssed viewers produced the
Mathematics and Physics, following information, 285
atmost 29 Mathematics and watch football, 195 watch
Chemistry and atmost 20 hockey, 115 watch basket-ball,
Physics and Chemistry. 45 watch football and basket
Then, the largest numbers ball, 70 watch football and
that could have passed all hockey, 50 watch hockey and
three examinations, are basket ball, 50 do not watch
(a) 12 (b) 14 any of the three games. The
(c) 15 (d) 16 number of viewers, who
10. A market research group watch exactly one of the three
conducted a survey of 2000 games are
consumers and reported (a) 325 (b) 310
that 1720 consumers like (c) 405 (d) 372
Set Theory 265

Hints & Solution

1. (a) Use Shortcut Approach -7 8. (d) Use Shortcut Approach -8


2. (c) Use Shortcut Approach -7 9. (b) Use Shortcut Approach -8
3. (b) Use Shortcut Approach -7 10. (c) Use Shortcut Approach -7
4. (a) Use Shortcut Approach -8 11. (b) Use Shortcut Approach -8
5. (a) Use Shortcut Approach -7 12. (a) Use Shortcut Approach -8
6. (d) Use Shortcut Approach -8
7. (b) Use Shortcut Approach -7
EBD_7382
266 Trigonometry

25 Trigonometry
Chapter

Shortcut Approach - 1
26
A vertical tower stands on a horizontal ground and is surmounted by a
flagstaff. If height of the top of the flagstaff is H metre and angle of
elevation of top and bottom of the flagstaff at a point on the ground are
q1 and q2, then height of the tower
æ tan q2 ö
= H ç1 - ÷
è tan q1 ø

F lagstaff

H
To w er

q1 q2

1. An aeroplane when 1500 m high passes vertically above another


aeroplane at an instant when the angle of elevation of the planes at
the same observing point are 60° and 30° respectively. How many
metres lower is one plane than the other?
æ tan q 2 ö
Sol. Required answer = H ç1 - ÷
è tan q1 ø
æ tan 30° ö
= 1500 ç1 - ÷
è tan 60° ø
æ 1 ö
ç ÷
= 1500 ç1 - 3 ÷ = 1000 m
ç 3÷
ç ÷
è ø
Trigonometry 267

Shortcut Approach - 2
26
Angle of elevation of a lamp post changes from q1 to q2 when a man
walks towards it. If the height of the lamp post is H metres, then the
distance travelled by the man is given by
H (tan q2 - tan q1 )
tan q1 × tan q2 metres

q1 q2

2. The pilot of a helicopter, at an altitude of 1200 m finds that the two


ships sailing towards it in the same direction. The angle of
depression of the ships as observed from the helicopter are 30°
and 45° respectively. Find the distance between the two ships.
D

1200 m
Sol.
30 45°
A B C
Distance between two ships = AB
H (tan q2 - tan q1 )
=
tan q1 × tan q2
1200(tan 45° - tan 30°)
=
tan 30° × tan 45°
æ 1 ö
1200 ç1 - ÷
= è 3ø
= 1200( 3 - 1) m
1
×1
3

Shortcut Approach - 3
26
A lamp post is stands on a horizontal plane. At a point on this plane
a man finds that the angle of elevation of the top of the flagstaff is q1. On
walking x units towards the tower, he find that the angle of elevation of
EBD_7382
268 Trigonometry

x tan q1 × tan q2
the top of the flagstaff is q2, then height of the tower = units
tan q2 - tan q1

To w er

q1 q2
x
3. The angle of elevation of top of a tower changes from 30° to 60°
when a man walks 60 m towards it. What is the height of the tower.
x × tan q1 × tan q2
Sol. Height of the tower = tan q - tan q
2 1

1
60 ´ ´ 3
60 × tan 30° × tan 60° 3
= =
tan 60° - tan 30° 1
3-
3
D

H e ig h t

30° 60°
A 60 m C
B
60 3
= = 30 3 m
2
Shortcut Approach - 4
26
The horizontal distance between two towers is x unit. The angle of
depression of the first tower when seen from the top of the second tower
is q°.
(i) If the height of the second tower is y1 units then the height of the
first tower = (y1 – x tanq) units.
(ii) If the height of the first tower is given as y2 units, then the height of
the second tower = (y2 + x tanq) units.
Trigonometry 269

S eco n d to w er
q
F irst to w er y1

y2

x
4. Distance between base of the two towers is 60 3 m. The angle of
depression of the first tower when seen from the top of the second
tower is 30o. If the height of the second tower is 180 m, find the
height of the first tower.
Sol. Height of the first tower = y1 - x tan q
= 180 - 60 3 ´ tan 30°
1
= 180 - 60 3 ´ = 120 m
3

Shortcut Approach - 5
26
Two poles of equal heights stand on either sides of a roadway which is x
units wides. At a point P on the road way between the poles AB and CD,
the elevation of the tops of the pole are q1° and q°2 , then the
A C

q1 q2
D
B P
x tan q1 × tan q2
(i) Height of the pole = units.
tan q1 + tan q2

æ x tan q2 ö
(ii) (a) Distance of the point P from B = ç ÷ units
è tan q1 + tan q2 ø
EBD_7382
270 Trigonometry

æ x tan q1 ö
(b) Distance of the point P from D = ç ÷ units
è tan q1 + tan q 2 ø
5. Two poles of equal heights stand on either side of a roadway which
is 30 m wide. At a point on a roadway between the poles, the elevation
of the top of the poles are 45° and 30°. Find the height of the poles.
x tan q1 × tan q2
Sol. Height of the pole =
tan q1 + tan q2
1
30 ×
30 × tan 45°× tan 30° 3 = 30 m
= =
tan 45° + tan 30° 1 3 +1
1+
3

Shortcut Approach - 6
26
A

L ig h t
H o u se
q1 q2
B C
D
The angle of depression of two ships at B and C from the top A of a light
house AD are q1° and q°2 . If the ships are x units apart, then
x tan q1 × tan q2
(i) The height of the light house = units
tan q1 + tan q2
(ii) Distance of the ship at A from the foot of the light house
x tan q2
=
tan q1 + tan q2
(iii) Distance of the ship at B from the foot of the light house
x tan q1
=
tan q1 + tan q2
6. Two boats approach a light house in mid-see from opposite
directions. The angle of elevation of the top of the light house
from the two boats are 30o and 45o respectively. If the distance
between the two boats is 200 m, then
(i) Find the height of the light house.
(ii) Find the distance of the first boat from light house.
Trigonometry 271

Sol. 30° 45°


A D B
200 m
(i) Height of the light house
x tan q1 × tan q2 200 tan 30°× tan 45°
= =
tan q1 + tan q2 tan 30° + tan 45°
1
200 ´ ´1
3 200 3
= = m
1 3 +1
+1
3
(ii) Distance of the first boat from light house
x tan q2 200 tan 45°
= =
tan q1 + tan q2 tan30° + tan 45°

200 ´ 1 200 3
= = m
1 3 +1
+1
3

Shortcut Approach - 7
26
Angle of elevation of top of a tower from top and bottom of a building of
height h units are q1° and q20 respectively, then

q1
To w er

q2
EBD_7382
272 Trigonometry
h × tan q2
(i) Height of the tower = units
tan q2 - tan q1

(ii) Distance between the building and the tower


h
= units
tan q2 - tan q1
7. Angle of elevation of a tower from top and bottom of a building of
height 80 m are 30° and 45° respectively. Find the
(i) Height of the tower.
(ii) Distance between the building and the tower.
h tan q2
Sol. (i) Height of the tower =
tan q2 - tan q1
80 tan 45°
=
tan 45° - tan 30°
80 ´ 1 80 3
= = m
1 3 -1
1-
3
(ii) Distance between the building and the tower
h 80
= =
tan q2 - tan q1 tan 45° - tan 30°

80 80 3
= = m
1 3 -1
1-
3
Trigonometry 273

Exercise
1. The angle of elevation of the 6. A person, standing on the bank
top of a tower at point on the of a river, observes that the
ground is 30°. If on walking 20
angle subtended by a tree on
metres towards the tower, the
the opposite bank is 60°; when
angle of elevation become 60°,
then the height of the tower is he reaches 20 m far (away) from
(a) 10 metre the bank, he finds the angle to
be 30°. The height of the tree and
10
(b) metre the breadth of the river are :
3
(a) 10 3 m, 10 m
(c) 10 3 metre
(d) None of these (b) 10, 10 3 m
2
2. If cos 4 q – sin 4q = , then (c) 20 m, 30 m
3 (d) None of these
th e value of 2 cos 2q
– 1 is: 7. If a cos q + b sin q = m and a
sin q – b cos q = n, then a2 +
(a) 0 (b) 1 b2 =
2 3 (a) m2 – n2 (b) m2 n2
(c) (d)
3 2 (c) n2 – m2 (d) m2 + n2
3. If sin q + sin 2 q = 1. Find 8. In a triangle, the angles are in
the value of cos2 q + cos 4 the ratio 2 : 5 : 3. What is the
q
value of the least angle in the
(a) 0 (b) 2
radian ?
(c) 1 (d) 3
4. If the length of the shadow p p
(a) (b)
of a tower is 3 times that of 20 10
2p p
its height, then the angle of (c) (d)
5 5
elevation of the sun is 9. Find the minimum & maximum
(a) 15° (b) 30°
value of sin 6q + cos6q.
(c) 45° (d) 60°
5. The value of tan 4°. tan 43°. 1 1 1
tan 47°. tan 86° is (a) ,1 (b) ,
4 4 2
(a) 0 (b) 1
1 3 1 3
(c) ,1 (d) ,
(c) 3 (d) 4 4 4
3
EBD_7382
274 Trigonometry
10. The angles of elevation of the (b) 240 3
top of a tower standing on a
horizontal plane from two (c) 60 3
points on a line passing
(d) None of these
through the foot of the tower
at a distance 9 ft and 16 ft 12. The angles of elevation of the
respectivelyare complementary top of a tower from two points
angles. Then the height of the at distances m and n metres
tower is are complementary. If the two
(a) 9 ft (b) 12 ft points and the base of the
(c) 16 ft (d) 144 ft tower are on the same straight
line, then the height of the
11. An aeroplane flying horizon-
tower is
tally 1 km. above the ground
is observed at an elevation of (a) mn
60° and after 10 seconds the (b) mn
elevation is observed to be
30°. The uniform speed of the m
aeroplane in km/h is (c)
n
(a) 240 (d) None of these
Trigonometry 275

Hints & Solution

1. (c) Use Shortcut Approach -2


é 1 ù
2 êQ cot q = tan q ú
2. (c) cos4q – sin4q = ë û
3 6. (a) Use Shortcut Approach -2
( cos q) 7. (d) (a cos q + b sin q)2 + (a
- ( sin q )
2 2 2 2 2
=
3 sin q – b cos q) 2
= m2 + n2.
( cos2 q + sin 2 q) a2 cos2 q + b2 sin 2 q +
Þ 2ab cos q.
( cos2 q - sin 2 q) = 2 sin q + a2 sin2 q + b2
3
cos2 q – 2 ab sin q. cos q
2 = m2 + n2.
Þ cos 2 q - sin 2 q =
3 or a2 (cos2 q + sin2 q) + b2
(sin 2 q + cos 2 q)
Þ cos q - (1 - cos q ) =
2 2 2
3 = m2 + n2.
or a2 + b2 = m2 + n2
2
Þ cos 2 q - 1 + cos 2 q = 8. (d) Let angles are 2x, 5x and
3 3x.
2 2x + 5x + 3x = 180º
Þ 2 cos2 q – 1 = (sum of interior angle
3
of triangles is 180º)
3. (c) sin q + sin2 q = 1 Þ sin q 10x =180º
= 1 – sin2 q x = 18º
Þ sin q =cos2 q Now, \ Least angle in degree
cos2 q + cos4 q = cos2 q = 2x = 2 × 18 = 36º
+ (cos2 q)2 In radian
= sin q + sin2 q = 1.
4. (b) Use Shortcut Approach -2 p p
= ´ 36º =
5. (b) tan4°. tan 43°. tan 47°. 180º 5
tan 86° 9. (a) sin q + cos q
6 6

= tan 4° . tan 43° . tan = (sin2q)3 + (cos2q)3


(90° – 43°) tan (90° – 4°) = (sin 2q + cos 2q) 3 –
= tan 4° . tan 43°. cot 43° 3sin 2q. cos2q (sin 2q +
cot 4° cos2q)
[Q tan (90 – q) = cot q]
= 1 – 3 sin 2q cos2q
= tan 4° × tan 43° ×
=
1 1
´ 3
tan 43° tan 4° 1- ´ 4 sin 2 q.cos 2 q
4
EBD_7382
276 Trigonometry

3 11. (b) d = H cot 30° – H cot 60°


= 1- ( 2 sin q. cosq) 2 Time taken = 10 second
4
speed =
3
=1- ( sin 2q )2 cot 30° - cot 60°
4
Min sin 2q = 0 10
So, Max (sin 6 q + ´ 60 ´ 60 = 240 3
3
cos6q) = 1 - (0 ) = 1 - 0 = 1
4
=1–0=1
Max sin 2q = 1
So, min (sin6q + cos6q)
3 1
= 1 - (1) =
4 4
10. (b) Use Shortcut Approach -3 12. (a) Use Shortcut Approach -3
26 Logarithm
Chapter

Shortcut Approach - 1
If two boys A and B are playing a game in which they throw a coin
alternatively till one of them gets a ‘head’ and wine the game. Then the
2
probability that A wins the game if he started the game, is .
3

Shortcut Approach - 2
A fact is stated by two persons A and B. If probability of speaking the
truth by A is a and by B is b, then the probability that they contradict each
other.

Shortcut Approach - 3
The probability that a leap year selected at random will have 53 Tuesday
2
(or any other particular days is .
7

Shortcut Approach - 4
The probability that a non-leap year selected at random will have 53
1
Thursday (or any other particular day) is .
7
EBD_7382
278 Logarithm

Exercise
1. Find the value of
log a b2 ´ log b c2 ´ log c d 2 ´
log 10 100 + log 10 1000 + 7.
log1010000. ... ´ log y z 2 ´ log z a 2 s
(a) 2 (b) 9
2
(c) 24 log a b(d) 100 b c 2 ´ log c d 2 ´ ... ´ log y z 2 ´ log z a 2 is
´ log
2. Find the value of
log5 10 × log10 15 × log15 20 (a) 227 (b) 252
× log20 25.
(c) 2 54
(d) 226
(a) 5/2 (b) 5 8. The value of (log28 + log3 9 +
æ5ö log 5 25) is :
(c) 2 (d) log ç ÷ (a) 5 (b) 6
è2ø
3. If a = log8225 and b = log215, (c) 7
(d) None of these
then a, in terms of b, is :
9. If p = log12 18, q = log24 54
(a) b/2 (b) 2b/3 then th e value of
(c) b (d) 3b/2 pq + 5(p – q) is
4. If log5 éëlog3 ( log 2 x ) ùû = 1, (a) 0 (b) 4
(c) 1 (d) 9
then x is
10. Find the value of
(a) 2234 (b) 243
81 25 16
(c) 2243 3 log + 5 log + 7 log .
(d) None of these 80 24 15
5. If log102 = a and log103 (a) log 2 (b) log 3
(c) 1
= b, then log512 equals:
(d) None of these
(a) ( a + b ) / (1 + a )
11. If 3x +1 = 6 log 2 3 , then x is
(b) ( 2a + b ) / (1 + a ) (a) 2 (b) 3
(c) log32 (d) log23
(c) ( a + 2b ) / (1 + a ) 12. The value of

(d) ( 2a + b ) / (1 - a ) é 1 1 ù
ê log ( xyz ) + log ( xyz ) ú
6. Find the value of ê xy yz ú
log20100 + log201000 ê 1 ú
ê + ú
+ log2010000 ë log zx ( xyz ) û
[Assume that log 2 = 0.3] is equal to
(a) 90/13 (b) 80/13 (a) 1 (b) 2
(c) 110/13 (d) 70/13 (c) 3 (d) 4
Logarithm 279

Hints & Solution

1. (b) Use Shortcut Approach-1 Similarly


2. (c) log5 10 × log10 15 ×
4log c
log15 20 × log20 25. log b c 2 =
log b
= (log 10/log 5) × (log 15/
log 10) × (log 20/log 15) Hence given expression
× (log 25/log 20) is equal to
= log 25/log 5 = 2 log 5/ logb log c
log 5 = 2. 427 ´ ´
loga log b
3. (b) Use Shortcut Approach-1
log d
log5 éëlog3 ( log 2 x ) ùû ´ ´ ...
4. (c) log c
= 1 = log5 5 log z log a
´ ´
log3 ( log 2 x) log y log z
Þ
= 5 = log3 35 8. (c) Use Shortcut Approach-1
9. (c) We have
Þ log 2 x = 35 = 243
log 2 18
Þ 2243 = x p = log12 18 =
log 2 12
5. (d) log5 12 = log5(3 × 4)
1 + 2log 2 3
= log5 3 + log5 4 =
= log53 + 2log52 2 + log 2 3
= {(log103)/(log105)}
+ {(2log102)/(log105)} log 2 54
q = log 24 54 =
= [(log103)/{(log1010) – log 2 54
(log102)}] 1 + 3log 2 3
=
+ [(2 log102)/{(log1010) – 3 + log 2 3
(log102)}]
= {b/(1–a)} + {2a/(1– Putting x = log 2 3, we
a)} = (2a + b)/(1–a) have
6. (a) Use Shortcut Approach-3
pq + 5 ( p - q ) =
7. (c) Since
1 + 2 x 1 + 3x
. +
log a b = 4log a b
2
2+ x 3+ x
4log b
= ì1 + 2 x 1 + 3x ü
log a 5í - ý
î 2+ x 3+ x þ
EBD_7382
280 Logarithm

3
Þ
=
(
6x2 + 5x + 1 + 5 - x2 + 1 ) 3x +1 = 3log 2 ´ 3
( x + 2)( x + 3) 3x = 3log 2 3
x + 5x + 6
2 Þ
= =1 Þ x = log 2 3
( x + 2)( x + 3)
12. (b) Given expression
10. (a) Use Shortcut Approach-3 = logxyz (xy) + logxyz (yz)
+ logxyz (zx)
3x +1 = ( 3 ´ 2) 2
log 3
11. (d) = logxyz (xy × yz × zx)
= logxyz (xyz)2
3x +1 = 3log 2 3
Þ = 2 logxyz (xyz) = 2 × 1 = 2
´ 2log 2 3
EBD_7420
• Corporate Office : 45, 2nd Floor, Maharishi Dayanand
Marg, Corner Market, Malviya Nagar, New Delhi-110017
Tel. : 011-49842349 / 49842350

Typeset by Disha DTP Team

DISHA PUBLICATION
All RightS Reserved

© Copyright Publisher
No part of this publication may be reproduced in any form without
prior permission of the publisher. The author and the publisher do not
take any legal responsibility for any errors or misrepresentations that
might have crept in. We have tried and made our best efforts to provide
accurate up-to-date information in this book.

For further information about books from DISHA,


Log on to www.dishapublication.com or email to info@dishapublication.com
Verbal Reasoning

1. Analogy & Classification 1-8

2. Series 9-16

3. Alphabet & Number Test 17-28

4. Coding-Decoding 29-40

5. Blood Relations 41-47

6. Direction and Distance 48-57

7. Time Sequence, Number, Ranking Test & Position Test 58-63

8. Logical Sequence of Words 64-66

9. Number Puzzles 67-69

10. Venn Diagram 70-76

11. Mathematical Operation Arithmetical Reasoning 77-82

12. Inequalities 83-89

13. Problem Solving (Puzzles) 90-104

14. Input and Output 105-120

15. Syllogism 121-135

16. Cube & Dice 136-146

17. Analytical Decision Making 147-157


EBD_7420
Non-verbal Reasoning

18. Series 158-165

19. Mirror & Water Images 166-172

20. Paper Cutting and Folding 173-176

21. Completion of Figure 177-179

22. Hidden / Embedded Figures 180-183

23. Figure Formation and Analysis 184-188

24. Visual Reasoning 189-195

Analytical Reasoning

25. Evaluating Inferences 196-202

26. Statement & Arguments 203-218

27. Statement & Assumptions 219-227

28. Statement & Conclusions 228-232

29. Courses of Action 233-242

30. Critical Reasoning 243-256


VERBAL REASONING

Chapter
Analogy &
1 Classification
ANALOGY 2. SYNONYM BASED ANALOGY
In such type of analogy two words
The meaning of analogy is ‘similar have similar meaning.
properties’ or similarity. If an object or
EXAMPLE
word or digit or activity shows any
Huge : Gigantic
similarity with another object or word or
3. WORKER & TOOL BASED
digit or activity in terms of properties, ANALOGY
type, shape, size, trait etc., then the This establishes a relationship
particular similarity will be called analogy. between a particular tool and the
The relationship of analogy can be person of that particular profession
who uses that tool.
established in two ways :
EXAMPLE

(i) A : B :: C : D Writer : Pen


4. WORKER & PRODUCT BASED
ANALOGY
This type of analogy gives a
(ii) A : B : : C : D relationship between a person of
particular profession and his/her
creations.
TYPES OF ANALOGY
EXAMPLE
1. Word Analogy
2. Letter Analogy Writer : Book
3. Number Analogy 5. CAUSES & EFFECT BASED
4. Mixed Analogy ANALOGY
In such type of analogy 1st word
Word Analogy acts and the 2nd word is the effect
In word analogy, candidates have to find of that action.
the relationship between given words in
EXAMPLE
a pair.
Work : Tiredness
1. TOOL & OBJECT BASED
6. OPPOSITE RELATIONSHIP
ANALOGY (ANTONYM) BASED ANALOGY
This establishes a relationship In such type of analogy the two
between a tool and the object in words of the question pair are
which it works. opposite in meaning.
EXAMPLE EXAMPLE
Scissors : Cloth Poor : Rich
EBD_7420
2 Analogy & Classification
7. GENDER BASED ANALOGY 13. SYMBOLIC RELATIONSHIP
In such type of analogy, one word BASED ANALOGY
is masculine and another word is In such type of analogy, the 1st word
feminine of it or It is a ‘male and is the symbol of the 2nd word and
female’ or ‘sex’ relationship. vice-versa.
EXAMPLE EXAMPLE
Man : Woman White : Peace
8. CLASSIFICATION BASED 14. ADULT & YOUNG ONE BASED
ANALOGY ANALOGY
This type of analogy is based on
In such type of analogy, the 1st word
biological, physical, chemical or any
other classification. In such is the adult one and 2nd word is the
problems the 1 st word may be young one of the 1st word or vice-
classified by the 2nd word and vice- versa.
versa. EXAMPLE
EXAMPLE Cow : Calf
Oxygen : Gas 15. SUBJECT & SPECIALIST
9. FUNCTION BASED ANALOGY BASED ANALOGY
In such type of analogy, 2nd word In such type of analogy the 2nd
describes the function of the 1st word is the specialist of 1st word
word. (subject) or vice-versa.
EXAMPLE EXAMPLE
Singer : Sings Heart : Cardiologist
10. QUANTITY AND UNIT BASED 16. HABIT BASED ANALOGY
ANALOGY In this type of analogy 2nd word is
In such type of analogy 2nd word is the habit of 1st and vice-versa.
the unit of the first word and vice-
EXAMPLE
versa.
EXAMPLE Cat : Omnivorous
Distance : Mile 17. INSTRUMENT AND MEASURE-
11. FINISHED PRODUCT & RAW MENT BASED ANALOGY
MATERIAL BASED ANALOGY We see in this type of analogy, the
In such type of analogy the 1st word 1st word is the instrument to measure
is the raw material and 2nd word is the 2nd word and vice-versa:
the end product of that raw material EXAMPLE
and vice-versa.
Hygrometer : Humidity
EXAMPLE
18. INDIVIDUAL & GROUP BASED
Yarn : Fabric ANALOGY
12. UTILITY BASED ANALOGY Second word is the group of 1st
In such type of analogy the 2nd word (or vice-versa) in such type
word shows the purpose of the 1st of analogy.
word or vice-versa.
EXAMPLE
EXAMPLE
Pen : Writing Cow : Herd
Analogy & Classification 3
19. STATE & CAPITAL BASED 1. FORWARD ALPHABETICAL
ANALOGY SEQUENCE BASED ANALOGY
1st word is the state and 2nd word is
EXAMPLE
the capital of that state (1st word)
(or vice-versa) in the analogy like CD : FG : : PQ : UV
this. Here, CD and FG are in the natural
alphabetical sequence. Similarly, PQ &
EXAMPLE UV are in the natural alphabetical
Bihar : Patna sequence.
20. ANALOGY BASED ON 2. BACKWARD OR OPPOSITE
INDIVIDUAL & DWELLING ALPHABETICAL SEQUENCE
PLACE BASED ANALOGY
In such type of analogy 1st word is
EXAMPLE
the individual & 2nd word is the
dwelling place of that individual (1st DC : GF : : QP : VU
word) and vice-versa. In fact this case is opposite of case I
EXAMPLE 3. VOWEL – CONSONANT RELA-
Horse : Stable TION BASED ANALOGY
21. ANALOGY BASED ON EXAMPLE
WORKER AND WORKING ATL : EVX : : IPR : ORS
PLACE Here, the 1st two words start with the 1st
In this type of analogy the 1st word two vowels A & E and the next two words
represents a person of particular start with the next two vowels I & O.
profession and 2nd word represents Last two letter of every word are
the working place of that person (1st consonants.
word) and vice-versa.
4. SKIP LETTER RELATION
EXAMPLE BASED ANALOGY
Doctor : Hospital EXAMPLE
22. ANALOGY BASED ON TOPIC ABC : FGH : : IJK : NOP
STUDY Here, between ABC & FGH two letters
1st word is the study of the 2nd word skip and they are D & E. Similarly,
(or vice-versa) in the analogy like between IJK & NOP two letters skip and
this. they are L & M.
EXAMPLE
5. JUMBLED LETTERS RELATION
BASED ANALOGY
Birds : Ornithology
EXAMPLE
Letter Analogy
(i) LAIN : NAIL : : EVOL : LOVE
In letter analogy, candidate has to find
Here, the 1st term gets reveresed to
out the relationship between given
produce the 2nd term and similar
letters or group of letters.
relation is shown in between 3rd and
4th term.
EBD_7420
4 Analogy & Classification
q Shortcut Approach
I: While solving the problems based on alphabet, you must have in your mind
the exact positions of every letters of alphabet in forward order as well as in
backward or reverse order as given below:
Letters positions in forward alphabetical order:
A B C D E F G H I J K L M N O P Q R S T U V W X
1 2 3 4 5 6 7 8 9 10 11 12 13 14 15 16 17 18 19 20 21 22 23 24
M N O P Q R S T U V W X Y Z
13 14 15 16 17 18 19 20 21 22 23 24 25 26
Letters positions in backward or reverse alphabetical order:
Z Y X W V U T S R Q P O N M L K J I
1 2 3 4 5 6 7 8 9 10 11 12 13 14 15 16 17 18
K J I H G F E D C B A
5 16 17 18 19 20 21 22 23 24 25 26
II: Just keep in mind, the following positions of the letters in the English alphabet
(forward order).

E J O T Y

(i) 5 10 15 20 25

EJOTY Remember this word

(ii) C F I L O R U X

3 6 9 12 15 18 21 24

CFILORUX Remember
Analogy & Classification 5
(iii) D H L P T X

4 8 12 16 20 24

DHLPTX Remember

Number Analogy EXAMPLE AB : 12 : : CD : : 34


In this, candidate has to find out the (Here, A B C D
relationship between the numbers or ¯ ¯ and ¯ ¯
group of numbers. 1 2 3 4
1. EVEN AND ODD NUMBERS (positional (positional
BASED ANALOGY value) vlaue)
EXAMPLE 84 : 51 : : 72 : 37
CLASSIFICATION
(Here, 84 & 72 are even and 51 & 37
are odd numbers respectively) In classification we take out an element
2. ADDITION AND SUBTRACTION out of some given elements and the
OF NUMBERS BASED element to be taken out is different from
ANALOGY the rest of the elements in terms of
common properties, shapes, sizes, types,
EXAMPLE 234 : 9 : : 136 : 10 nature, colours, traits etc. In this way,
(Here, 2 + 3 + 4 = 9 and 1 + 3 + 6 = 10) the rest of the elements form a group and
3. MULT IPLICAT ION AND the element that has been taken out is
DIVISION OF NUMBERS not the member of that group as this
BASED ANALOGY single element does not possesses the
common quality to be possessed by rest
EXAMPLE 3 : 21 : : 5 : 35
of the elements.
(Here, 3 × 7 = 21 and 5 × 7 = 35)
4. SQUARES & CUBES OF TYPES OF CLASSIFICATION
NUMBERS BASED ANALOGY
1. Letter/meaningless word based
EXAMPLE 4 : 16 : : 8 : 64 classification
(here, 42 = 16 and 82 = 64) 2. Meaningful word based
Mixed Analogy classification
In this, candidate has to find out the 3. Digit based classification
relationship between the given group of 4. General knowledge based
letters and a number on one side. classification
EBD_7420
6 Analogy & Classification
1. LETTER/MEANINGLESS WORD Gr ee ts Gr i d
BASED CLASSIFICATION (c) (d)
Such classifications are based on 2 vowels 1 vowel
letters of English alphabet. So many F i ght
groups of letters are given in the (e)
question in which one group is 1 vowel
different from remaining groups and
3. DIGIT BASED CLASSIFICATION
hence the different group will be our
answer. In such type of classifications digits
or numbers are given to find out one
EXAMPLE number that is not a part of the
(a) PQT (b) UVY group of remaining numbers.
(c) DEH (d) IJN
EXAMPLE
(e) FGJ
(a) 122 (b) 128
Sol. (a) Here, P Q R S T (c) 199 (d) 200
(e) 388
2 letter gap Sol. 199 is an odd number while all the
(b) U V W X Y other options are even numbers.
4. GENERAL KNOWLEDGE
2 letter gap BASED CLASSIFICATION
Such classification is done on the
(c) D E F G H basis of our general knowledge. No
doubts that this is a word based
2 letter gap classification but without having
general knowledge this type of
(d) I J K L M N questions can not be solved.
EXAMPLE
3 letter gap
(a) Cat (b) Dog
(e) F G H I J (c) Tiger (d) Octopus
(e) Lion
Sol. Octopus is the only animal out of
2 letter gap
given options which is a water
2. MEANINGFUL WORDS BASED animal. Rest of the options are land
CLASSIFICATION animals.
In such type of classification we
have to take odd word out of the q Shortcut Approach
given group of meaningful words. Step I : See all the given options with a
EXAMPLE serious eye.
(a) Slim (b) Trims Step II : Try to make relation of similarity
(c) Greets (d) Grid among the given options.
(e) Fight Step III :Find out the one word not
having the common similarity like other
Sl i m Tr i ms
Sol. (a) Here, (b) four options and that one word will be
1 vowel 1 vowel your answer.
Analogy & Classification 7

PRACTICE EXERCISE
DIRECTIONS (Qs. 1-2) : In the 7. LMNO : NQTW :: GHIJ : ?
following six questions, select the (a) ILOR (b) ILRO
related word/letters/number from the (c) ILMO (d) LRMO
given alternatives. 8. 103 : 10609 : : 106 : ?
1. 43 : 57 : : 111 : ? (a) 10606 (b) 10306
(a) 135 (b) 133 (c) 11236 (d) 13636
(c) 134 (d) 136
2. AZBY : CXDW : : EVFU : ? DIRECTIONS (Qs. 9-12): In the
(a) GTHS (b) FUVE following questions, find the odd word/
(c) ZYEU (d) BXWD letters/number pair from the given
DIRECTION (Q. 3) : In the following alternatives.
question, select the related word pair 9. (a) DFOU (b) N P S W
from the given alternatives. (c) KMPT (d) D F I M
3. Power : Watt : : ? : ? 10. (a) 286 (b) 166
(a) Pressure : Newton (c) 495 (d) 583
(b) Force : Pascal 11. (a) Wing Commander
(c) Resistance : Mho (b) Air Marshal
(d) Work : Joule
(c) Captain
DIRECTION (Q. 4) : In the following (d) Group Captain
question, select the related word pair 12. (a) 243 - 132 (b) 183 - 54
from the given alternatives. (c) 108 - 97 (d) 99 - 63
4. 9143 : 9963 : : 6731 : ? DIRECTION (Q. 13) : In the following
(a) 1368 (b) 5666 question, select the odd word from the
(c) 8964 (d) 9694
given alternatives.
DIRECTION (Q. 5) : In the following
13. (a) Chennai (b) Daman
question, select related word pair/
(c) Raipur (d) Shimla
number from the given alternatives.
DIRECTIONS (Qs. 14-15): Choose the
5. KLMN : IJKL : : TUVW : ?
(a) RSUT (b) VWXY odd word/letters number/number pair
(c) STUV (d) RSTU from the given alternatives.
DIRECTIONS (Qs. 6-8) : In the 14. (a) Sirius
following questions, select the related (b) Proximacentauri
word from the given alternatives. (c) Deimos
6. Influenza: Virus :: Ringworm: ? (d) Alpha centauri
(a) Bacteria (b) Fungi 15. (a) 2890 (b) 3375
(c) parasite (d) Protozoa (c) 1728 (d) 1331
EBD_7420
8 Analogy & Classification

HINTS & SOLUTIONS


1. (b) As, 62 + 7 = 43 7. (a) As,
(72) + 8 = 57 L M N O
Similarly, +2 +4 +6 +8
(10)2 + 11 = 111
(11)2 + 12 = 133 N Q T W
Similarly,
2. (a) +2 +2
A C E G
+2 +2 G H I J
Z X V T +2 +4 +6 +8
+2 +2
B D F H I L O R
+2 +2
Y W U S 8. (c) As, (103)2 = 10609
3. (d) As, Power is measured by Similarly, (106)2 = 11236
Watt. 9. (a)
Similarly, Work is measured by
N P S W, K M P T
Joule.
4. (c) As, +2 +3 +4 +2 +3 +4
9143 Þ 9 + 1 + 4 + 3 = 17
9963 Þ 9 + 9 + 6 + 3 = 27 and D F I M
Similarly,
6731 Þ 6 + 7 + 3 + 1 = 17 +2 +3 +4
8964 Þ 8 + 9 + 6 + 4 = 27 But, D F O U
5. (d) As,
+2 +9 +6
K L M N : I J K L 10. (b) As, 2 8 6 = 2 + 6 = 8 , 4 9 5
–2
–2
= 4 + 5 = 9 and 5 8 3 = 5 + 3
–2 = 8
–2 but, 1 6 6 = 1 + 6 = 7 ¹ 6
Similarly, 11. (c) Captain is odd one out.
T U V W : R S T U
12. (d) 99-63 is odd one out.
–2 13. (b) Except Daman, all others are
–2 capital.
–2 14. (c) Except Deimos (It is a satellite),
–2 all others are star systems.
6. (b) As, Infuenza is caused by 15. (a) Except 2890, All are cube of a
virus. number.
Similarly, Ring worm is caused (15)3 = 3375, (12)3 = 1728,
by Fungi. (11)3 = 1331.
Series 9

Chapter

2 Series
INTRODUCTION
A series is a sequence of numbers/alphabetical letters or both which follow a
particular rule. Each element of series is called ‘term’. We have to analyse the
pattern and find the missing term or next term to continue the pattern.

TYPES OF SERIES

SERIES

Number Alphabet Alpha-numeric Mixed Continuous Pattern Correspondence


series series series series series series

A series that is A series that is A series in A series which A series of letters, A series consists of
made by only made by only which both is created by which follow a three sequence with
number or digit alphabetic letters alphabets and the combination certain pattern, is three different
of two or more given with four / elements (for ex.
numbers are than two series
used five times blank capital letters,
1. Ascending series spaces in between. numbers and small
The missing letters letters). An element
2. Descending series of each sequence is
3. Oscillating series in order is correct
answer. correspond to the
element of other
sequence on the
basis of the similarity
in position.

NUMBER SERIES · Cube and cube roots of a numbers.


Number series is a form of numbers in · Addition
a certain sequence, where some Arithmetic Subtraction
numbers are mistakenly put into the Operations Division
series of numbers and some number is
Multiplication
missing in that series, we need to
observe first and then find the accurate Types of Number Series
number to that series of numbers.
1. PERFECT SQUARE SERIES
Remember This type of series are based on
square of a number which is in
· Even and odd numbers.
same order and one square number
· Prime and composite numbers. is missing in that given series.
· Square and square roots of a
EXAMPLE 841, ?, 2401, 3481, 4761
numbers.
Sol. 292, 392, 492, 592, 692
EBD_7420
10 Series
2. PERFECT CUBE SERIES 3. MIXED NUMBER SERIES
Perfect Cube series is a arrangement Mixed number series is a
of numbers in a certain order, arrangement of numbers in a
where some number which is in certain order. This type of series
same order and one cube is missing are more than are different order
in that given series. which arranged in alternatively in
single series or created according
EXAMPLE 4096, 4913, 5832, ?, 8000 to any non conventional rule.
Sol. 163, 173, 183, 193, 203

EXAMPLE 62, 64, 30, 32, 14, 16, ?

Sol. 62 64 30 32 14 16 6

–2 ÷ 2 –2 ÷ 2 –2 ÷ 2

÷2 ÷2

4. PRIME SERIES 7–4=3


When numbers are a series of 10 – 7 = 3
prime numbers.
So, the answer is 19 + 3 = 22
EXAMPLE 2, 3, 5, 7, 11, 13, __ , 19
7. The difference between two
Sol. Here, the terms of the series are the consecutive terms will be
prime numbers in order. The prime either increasing or decreasing
number, after 13 is 17. So, the by a constant number:
answer to this question is 17.
EXAMPLE 2, 10, 26, 50, 82, __
5. ALTERNATE PRIMES
It can be explained by below Sol. Here, the difference between two
example. consecutive terms are
EXAMPLE 2, 11, 17, 23, __, 41 10 – 2 = 8
26 – 10 = 16
Sol. Here, the series is framed by taking
the alternative prime numbers. 50 – 26 = 24
After 23, the prime numbers are 29 82 – 50 = 32
and 31. So, the answer is 31. Here, the difference is increased
6. The difference of any term by 8 (or you can say the multiples
from its succeding term is of 8). So the next difference will be
constant (either increasing 40 (32 + 8). So, the answer is 82 +
series or decreasing series): 40 = 122
EXAMPLE 4, 7, 10, 13, 16, 19, __, 25 8. The difference between two
numbers can be multiplied by
Sol. Here, the differnce of any term a constant number:
from its succeding term is 3.
Series 11
12. Every succeeding term is got
EXAMPLE 15, 16, 19, 28, 55, __
by multiplying the previous
Sol. Here, the differences between two term by a constant number or
numbers are numbers which follow a
16 – 15 = 1 special pattern.
19 – 16 = 3
28 – 19 = 9 EXAMPLE 5, 15, 45, 135, __
55 – 28 = 27
Here, the difference is multiplied Sol. Here,
by 3. So, the next difference will be 5 × 3 = 15
81. So, the answer is 55 + 81 = 136 15 × 3 = 45
9. The difference can be multiples 45 × 3 = 135
by number which will be So, the answer is 135 × 3 = 405
increasing by a constant
13. In certain series the terms are
number:
formed by various rule
EXAMPLE 2, 3, 5, 11, 35, __ (miscellaneous rules). By
keen observation you have to
Sol. The difference between two
number are find out the rule and the
3–2=1 appropriate answer.
5–3=2
11 – 5 = 6 EXAMPLE 4, 11, 31, 90, __
35 – 11 = 24 Sol. Terms are,
10. Every third number can be the 4 × 3 – 1 = 11
sum of the preceding two numbers 11 × 3 – 2 = 31
: 31 × 3 – 3 = 90
So, the answer will be 90 × 3 – 4
EXAMPLE 3, 5, 8, 13, 21, __ = 266
Sol. Here, starting from third number 14. TRIANGULAR PATTERN SERIES:
3+5=8 Sometimes the difference between
5 + 8 = 13 consecutive terms of a series,
8 + 13 = 21
again form a series. The differences
So, the answer is 13 + 21 = 34
between the consecutive terms of
11. Every third number can be the
the new series so formed, again
product of the preceeding two
form a series. Th is pattern
numbers :
continues till we attain a uniform
EXAMPLE 1, 2, 2, 4, 8, 32. __ difference between the
consecutive terms of the series.
Sol. Here, starting from the third number
1×2=2 EXAMPLE
2×2=4 2, 12, 36, 80, 150, ?
2×4=8 Sol. As discussed above, we may lebel
4 × 8 = 32 the given series as I and then form
So, the answer is 8 × 32 = 256 series II to IV as shown, below:
EBD_7420
12 Series
Series-I: 2 12 36 80 150 ? Sol. The pattern is as follows
Series-II: 10 24 44 70 ? 17
Series-III: 14 20 26 ?
Series-IV: 6 6
Clearly, the pattern in series III is +6. 2 2 2
1 (2 + 1) 3 (4 + 1) 5 (6 + 1)
2 2 2
7
2

So, missing term in series Hence number 15 is wrong and


III = 26 + 6 =32 should be replaced by 17.
Missing term in series II = 70 + 32 = 102 q Shortcut Approach
Missing term in series I = 150 + 102 = 252
(1) If numbers are in ascending order
Thus the missing term = 252 in the number series, then the
(i.e. 150 + 70 + 26 + 6) numbers may be added or
multiplied by certain numbers
Remember from the first number.
Elementry Idea of Progressions: (A) 19 23 26 30 33 ?
19 23 26 30 33 37
1. ARITHMETIC PROGRESSION
+4 +3 +4 +3 +4
(A. P.):
The sequence of the form a, a + d, (B) 1 3 12 60 ?
a + 2d, a + 3d, ...... is known as an 1 3 12 60 360
A.P., whose nth term is a + (n –1) d. ×3 ×4 ×5 ×6
Here ‘a’ is first term and ‘d ’ is
common difference. (2) If numbers are in descending
order in the number series, then
2. GEOMETRIC PROGRESSION
the numbers may be subtracted or
(G. P.):
divided by certain numbers from
The sequence of the form a, ar, ar 2, the first number.
ar3, ...... is known a G.P., whose nth (A) 34 18 10 6 4 ?
term is arn–1. 34 18 10 6 4 3
3. FIND THE WRONG NUMBER: –16 –8 –4 –2 –1
In this type of questions, a series of
numbers is given which follow a (B) 720 120 24 6 2 ?
certain pattern and one its term does 720 120 24 6 2 1
not fit into the series. The candidate /6 /5 /4 /3 /2
is required to identify the pattern
involved in the formation of series (3) If numbers are in mix order
and then find out that number which (increasing and decreasing) in the
does not follow the specific pattern number series, then the numbers
of the series. This particular number may be in addition, subtraction,
is the wrong term in the series. multiplication, division, square
and cube in the alternate numbers.
EXAMPLE (A)
One number is wrong in the 200 165 148 117 104 ?
following series. Find out this 200 165 148 117 104 77
wrong number. (14)2+4 (13)2–4 (12)2 +4 (11)2 –4 (10)2 +4 (9)2 –4

1, 5, 9, 15, 25, 37, 49,


Series 13
(B) q Shortcut Approach
14 17 31 48 ? 127 · Remember all the alphabets and
14 17 31 48 79 127 their place number.
14+17=31 17+31=48 31+48=79 · Intervals like :
E J O T Y , C F I L O R U X
(4) Check the direct formula if any.
(5) Check whether all numbers are odd, 5 10 15 20 25 3 6 9 12 15 18 21 24
even or prime.
(6) Check whether all the number are ALPHA NUMERIC SERIES
perfect squares or cubes. These kind of problems used both
mathematical operation and position of
ALPHABET SERIES letters in the alphabet in forward,
A series that is made by only alphabetic backward order.
letters. EXAMPLE 2 Z 5, 7 Y 7, 14 X 9,
EXAMPLE G, H, J, M, ? 23 W 11, 34 V 13, ?

Sol. G H J M Q
+1 +2 +3 +4

Sol.
–1 –1 –1 –1 –1

2 Z 5 14 X 9 23 W 11 34 V 13 47 U 15

+5 +7 +9 +11 +13
+2 +2 +2 +2 +2

MIXED SERIES EXAMPLE Z, L, X, J, V, H, T, F, __, __


A series formed with the combination of Sol. The given sequence consists of
more than one series. two series
(i) Z, X, V, T, __
EXAMPLE A, Z, C, X, E, ? (ii) L, J, H, F, __. Both consisting
Sol. There are two interwoven series. of alternate letters in the reverse
order.
1 26 3 24 5 22 \ Next term of (i) series = R, and
A Z C X E V
Next term of (ii) series = D
Reverse Order Repetition Series:
+2 +2
–2 –2 In such series, first part is written in
\?=V reverse order of the second part of the
series.
EBD_7420
14 Series
EXAMPLE R, A, M, S, H, H; S, E, M A,?
Sol. RAME S H H S EMA R

\?=R

SERIES HAVING GROUP OF elements (usually capital letters,


LETTERS AS ITS ELEMENTS: digits and small letters). On the basis
In such ser ies, each element of the similarity in position in the
three sequences, a capital letter is
consists of group of letters instead
found to correspond with a unique
of a single letter. digit and a unique small letter,
EXAMPLE LDP, DPL, PLD, ? whenever it occurs. The candidate is
Sol. required to trace out this
correspondence and accordingly
L D P D PL P LD LDP choose the elements to be filled in at
the desired places.
\ ? = LDP EXAMPLE
C B – – D – B AB C C B
CONTINUOUS PATTERN – – 1 2 4 3 – – ? ? ? ?
SERIES a – a b – c – b – – – –
It is a series of small/capital letters that Sol. Comparing the positions of the
follow a certain pattern like repetition of capital letters, numbers and small
letters. letters, we find a corresponds to c
and 1 correspond to a. So, a and 1
EXAMPLE b a a b – a b a – b b a – – correspond to c. b corresponds to
Sol. b a a b b a / b a a b b a / b a A and 2 corresponds to b. So, b and
2 correspond to A. Also, 4
CORRESPONDENCE corresponds to D. Therefore, the
SERIES: remaining number i.e. 3
This type of series consists of three corresponds to B. Hence, BCCB
sequences with three different corresponds to 3113.
Series 15

PRACTICE EXERCISE
DIRECTIONS (Qs. 1-2): In the following 8. 4000 2000 1000 500 250
Questions, which one set of letters when 125 ?
sequentially placed at the gaps in the (a) 80 (b) 65
given letter series shall complete it? (c) 62.5 (d) 83.5
1. ccbab _ caa _ bccc _ a _ (e) None of these
(a) babb (b) bbba 9. 588 563 540 519 ?
(c) baab (d) babc 483 468
2. a_ _ dba_ _bcad__ _da__ _cd (a) 500 (b) 496
(a) bccdbcab (b) abcddcba (c) 494 (d) 490
(c) cbcddcba (d) aabbccdd (e) None of these
DIRECTIONS (Qs. 3-4): A series is 10. 121 ? 81 64 49 36 25
given,with one term missing. Choose (a) 92 (b) 114
amongst the given responses choose the (c) 98 (d) 100
meaningful one. (e) None of these
3. CUS, DVT, EWU, ____ DIRECTIONS (Qs. 11-15) : Each of the
(a) FXV (b) VXF following number series, a wrong
(c) XFV (d) XVF number is given. Find out that number.
4. 206, 221, 251, 296, ?, 431
(a) 326 (b) 356 11. 3 5 13 43 178 891 5353
(c) 311 (d) 341 (a) 43 (b) 178
5. A series is given, with one term (c) 891 (d) 5353
missing. Choose the correct (e) None of these
alternative from the given ones that 12. 80640 10080 1440 240 48 10 4
will complete the series. (a) 240 (b) 48
CAT , DBT , ECT , ? (c) 1440 (d) 10
(a) DCT (b) FDT (e) None of these
(c) FCT (d) FAT 13. 3 5 10 12 17 23 24
DIRECTIONS (Qs. 6-10) : What should (a) 5 (b) 17
come in place of the question mark (?) (c) 24 (d) 23
in the following number series? (e) None of these
14. 1, 11, 38, 78, 175, 301
6. 2 16 112 672 3360 13440 ?
(a) 3430 (b) 3340 (a) 11 (b) 78
(c) 40320 (d) 43240 (c) 175 (d) 301
(e) None of these (e) None of these
7. 4 9 19 ? 79 159 319 15. 17, 39, 85, 179, 369, 879
(a) 59 (b) 39 (a) 369 (b) 211
(c) 49 (d) 29 (c) 179 (d) 879
(e) None of these (e) None of these
EBD_7420
16 Series

HINTS & SOLUTIONS


1. (a) c c b a/b b c a/a a b c/c c b a/b 9. (a) Given series.
2. (a) a b c d/b a c d/b c a d/b c d a/ 588 563 540 519 500 483 468
a b c d.
– 25 – 23 – 21 – 19 – 17 – 15
3. (a) \ ? = 500
+1 10. (d) Given series.
+1 121 100 81 64 49 36 25
+1 ­ ­ ­ ­ ­ ­ ­
(11)2 (10)2 (9)2 (8)2 (7)2 (6)2 (5)2
C U S, D V T, E W U F XV
+1 \ ? = 100
+1 +1
11. (b) 3× 1+2=5
+1 +1
5 × 2 + 3 = 13
+1
13 × 3 + 4 = 43
4. (b) 43 × 4 + 5 = 177
Wrong No = 178
206 221 251 296 356 431 Correct Number = 177
+15 +30 +45 +60 +75 12. (d) 80640 ÷ 8 = 10080
5. (b) 10080 ÷ 7 = 1440
1440 ÷ 6 = 240
240 ÷ 5 = 48
48 ÷ 4 = 12
Wrong No = 10
Correct Number = 12
13. (e) First series: 3, 10, 17, 24
(increased by 7)
6. (c) Given series. Second Series: 5, 14, 23
(increased by 9)
Wrong Number: 12
2 16 112 672 3360 13440 40320
Correct Number: 14
×8 ×7 ×6 ×5 ×4 ×3 14. (b) 1 + 32 + 1 = 11
\ 11 + 52 + 2 = 38
? = 40320 38 + 72 + 3 = 90
7. (b) Given series. 90 + 92 + 4 = 175
175 + 112 + 5 = 301
4 9 19 39 79 159 319 Wrong Number = 78
Correct Number = 90
×2+1 ×2+1 ×2+1 ×2+1 ×2+1 ×2+1 15. (d) 7 × 2 + 3 = 17
\ ? = 39 17 × 2 + 5 = 39
8. (c) Given series 39 × 2 + 7 = 85
85 × 2 + 9 = 179
4000 2000 1000 500 250 125 62.5 179 × 2 + 11 = 369
¸2 ¸2 ¸2 ¸2 ¸2 ¸2 369 × 2 + 13 = 751
\ Wrong Number = 879
? = 62.5 Correct Number = 751
Chapter
Alphabet &
3 Number Test
INTRODUCTION English Alphabet: English Alphabet has
· 26 letters
As we know that English alphabet is a
· 5 vowels (A, E, I, O, U) and 21
group of English letters, hence the
consonants.
problems based on alphabet are the
· 13 letters in first half i.e. A to M
problems based on English letters.
· 13 letters in second half i.e. N to Z
· Linear arrangement of alphabets in forward order and their corresponding
positions:
A B C D E F G H I J K L M
· · · · · · · · · · · · ·
1 2 3 4 5 6 7 8 9 10 11 12 13
N U P Q R S T U V W X Y Z
· · · · · · · · · · · · ·
14 15 16 17 18 19 20 21 22 23 24 25 26
· Linear arrangement of alphabets in reverse order and their corresponding
positions:
Z Y X W V U T S R Q P O N
· · · · · · · · · · · · ·
1 2 3 4 5 6 7 8 9 10 11 12 13
M L K J I H G F E D C B A
· · · · · · · · · · · · ·
14 15 16 17 18 19 20 21 22 23 24 25 26

· Circular Arrangement: When we have to find out the letters


A
coming before A or after Z, then
Z B linear arrangement does not work. In
C
Y D
X 26 1 2 3 this case, we have to take help of
25 E
24 4 circular arrangement . It is clear from
W 23 5
6
F circular arrangement, the letter coming
V 22
7 G just before A is Z and just after Z is A.
U 21
H 8 TYPES OF PROBLEMS
T 20
19 I 9 1. General series of alphabet
S 10
18 11 J 2. Random series of alphabet
17
R 16 15 13
12
K
3. Problems of word formation
Q 14
P L 4. Problems of letter gap
O N M 5. Finding digits after rearrangement.
EBD_7420
18 Alphabet & Number Test
General Series of Alphabet q Shortcut Approach
EXAMPLE 1. Which of the following (a) If both the directions are same then
options is seventh to the right of the 13th subtraction of numbers takes place.
letter from the left in a forward Alphabet (b) If the directions are opposite then
series? addition of numbers takes place.
Sol. 1st of all we will write the forward SHORTCUT METHOD FOR ABOVE
alphabet series as given below: EXAMPLE :
A B C D E F G H I J K L M Now, for solving the example we apply
this rule. As we want to find out the 7th
13th letter from left letter to the right of the 13th letter from
N O P Q R S T U V W X Y Z the left, the directions are opposite and
thus shortcut (b) will be applied here.
7th letter Hence, we add 7 + 13 = 20. Therefore, the
From the above series it is clear that answer will be 20th from left. Also, 20th
M is the 13th letter from left and to from left less mean 26 – 20 + 1 = 7th from
the right of M (13th letter from left), right. We can easily see,
T is the 7th letter. \ 20th letter from left = T
Here, we have solved this problem Also 7th letter from right = T
with a general method. But this type After solving the example, you must
have noticed that the above mentioned
of problem can also be approached
trick is to calculate the actual position
through quicker method that will help
of the required letter before going to
you save some extra consumed time.
search for it.
Remember
mth element to be counted from left to right of a series of x characters is equal to
(x + 1 – m)th element to be counted from right to left of that series. This rule can be
better illustrated by an example which is given below:
Let us take the forward order alphabet series,
A B C D E F G H I J K L M N O P Q R S T U V W X Y Z
1 2 3 4 5 6 7 8 9 10 11 12 13 14 15 16 17 18 19 20 21 22 23 24 25 26
As we know that English alphabet has 26 characters, hence, we have x = 26.
Now suppose, we have to find out the position of K in the above given series
counting from right to left.
Position of ‘K’ in the English alphabet from left to right is 11. Thus m = 11
\ Position of K in the above given series from right to left would be (26 + 1 – 11) = 16
1. How to find the number of letters
1. …?… 2. …?…
in the middle of two letters?
Four situations can be created under
these type of problems. 3. …?… 4. …?…
Alphabet & Number Test 19
Let us understand through the Shortcut Sol. Number of letters in the
following examples. English Alphabet = 26

EXAMPLE 2. How many letters are From Right


there between 10th letter from left From Right
and 6th letter from right in the 12 letters
19th 6th
English Alphabets?
Shortcut Sol. Total number of letters in Required number of letters
the English Alphabets = 26 = 19 – 6 – 1 = 12 letters
10 letters
From right
EXAMPLE 5. Find the number of
From left 10th 6th letters between 21st letter from left
Required number of letters and 15th letter from right in the
= 26 – (10 + 6) = 10 letters English Alphabets.
Shortcut Sol. Number of letters in the
EXAMPLE 3. How many letters are
English Alphabets = 26
there between 18th letter from left
and 7th letter from left in the 21st
English Alphabets? From Left

Shortcut Sol. Total number of letters in 5 letters


the English Alphabet = 26 15th From Right
From left Required number of letters
= 21 – 15 – 1 = 5
12 letters
From left 7th 18th 2. How to solve problems when
Required number of letters letters are dropped or deleted
at regular intervals?
= 18 – 7 – 1 = 10 letters
EXAMPLE 4. Find the number of EXAMPLE 6. If every 3rd letter from
letters between 19th letter from left to right of English alphabet is
right and 6th letter from right in deleted, then what would be the 6th letter
the English Alphabet. from left in the new series obtained?
Sol. General method:
AB C DEF G H I J K L M N O P Q R S T U V W X Y Z
Here, deleted letters have been encircled and we find the new series as given
below:
A B D D E EG GH H J J KK M M NN PP Q Q S ST TV V W W Y ZY Z
1 21 32 34 45 5 6 6 7 7 88 99 10
10 1111 1212131314 14
15 15
16 17
16 18
17 18
It is clear, that 6th letter from left in the new series is H.
EBD_7420
20 Alphabet & Number Test
q Shortcut Approach
No doubt, above general method gives the correct answer. But we need to save
extra consumed time and this is the reason we go for a quicker approach.
As per the example, every third letter is deleted in the original series. It does
mean that we are left of two letters after every deletion. Here, ‘2’ is the key digit for
us and we have to find out 6th letter from the left in the new obtained series.
Therefore, we have to find a digit which is just less than 6 but divisible by 2. For
this question the digit just less than 6 and divisible by 2 is 4. Now, we follow the
operation given below:
4
6th letter from the left in the new series = 6 +
2
= 8th letter from the left in the original series, which is it.
In the same manners, we can find out any letter at a particular position in the
new obtained series.
14
\ 16th letter from the left in the new obtained series = 16 +
2
= 23rd letter from the left in the original series which is W.
18th letter from the left in the new obtained series
16
= 18 +
2
= 26th letter from the left in the original series which is Z.
The sample example can be asked in following way also.
“If every third letter from left to right in English alphabet is dropped (or deleted),
then find out the 13th letter from right in the new obtained series”.
To solve this, we find first of all the number of letters in the new obtained series.
As every third letter is dropped, hence we have
æ 26 ö
çè 26 – ÷ø = 26 – 8 = 18 letters in the new series.
3
26
(approximate value of is 8).
3
As per the example we have to find out 13th letter from right in the newly
obtained series. This loss mean (18 + 1 – 13) = 6th letter from left which is H.
Note that : This shortcut approach can also be applied to the dropping of every 4th,
5th, 6th, 7th..... and so on letters from left to right at regular intervals.

3. How to solve problems based on the backward (reversed) alphabet


series?
While solving problems based on general series of alphabet, we come across the
various cases. In some cases we see that whole alphabet series is reversed but in
some other cases 1st half of the series is reversed, or second half of the series is
reversed or many segments of the alphabet series are reversed.
Alphabet & Number Test 21
Let us take a case when a forward order alphabet series get reversed in three
segments. In 1st segment 8 letters get reversed; in 2nd segment the next 8 letters get
reversed and in the 3rd segment the remaining 10 letters get reversed. Just see the
presentation given below:
A B C D E F G H I J K L M N O P Q R S T U V W X Y Z

Get reversed Get reversed Get reversed

H G F E D C B A P O N M L K J I Z Y X W V U T S R Q

(8 letters) (8 letters) (10 letters)


Now if you are asked to find out the 4th letter from left in the new obtained
series, then through general method, we simply do counting from left in the new
series and find out our required answer as ‘E’ because ‘E’ is at 4th position from left
in the new obtained series. But while solving such type of problems, we have to do
some time consuming formalities like (a) writing the original series (b) writing and
reversing the letters of original series as per the question says and (c) counting
them to get the required answer. Such time consuming processes can be avoided if
we go through “Remember” and solve the question with shortcut approach.
q Shortcut Approach
It is clear that 4th letter from left in the new obtained series falls into first segment
which has 8 letters. Hence, 4th letter in the new obtained series = (8 + 1 – 4) = 5th letter
from the left in the original series. As we know that exact position of 5th letter from left
in the original alphabet series is the position of E. Hence, E is our required answer.
If we have to find out 18th letter from left in the new obtained series, then that will
be 16 + (10 + 1 – 2) = 25th letter from left in the original alphabet series (why?) which is Y.
In fact, while finding out 18th letter, we can easily see that 18th letter is the 2nd
letter of 3rd segment and hence it will be not affected by 1st two segments having 8
letters each. In other words to find out 18th letter in the new obtained series, we have
to find out the 2nd letter in the 3rd segment. This is the reason we find out the 2nd letter
in the 3rd segment and then add the 16 letters of 1st two segment to get the 18th letter
in the new obtained series. From this, we find that 18th letter from left in the new
obtained series is the 25th letter from left in the original series. As 25th letter from left
in the original series is Y. So, (Y) will be our required answer.
Readers are advised to practice such type of problems as you much as possible
and after a certain time will notice that you have got a skill to solve such problems
in a few seconds and that too, without the use of pen and paper.
4. How to solve if positions of letters are interchanged?
There is no any rule for such type of problems. Only the hard practice can given you
a skill to solve such questions in a quick time.
EBD_7420
22 Alphabet & Number Test

EXAMPLE 7. If A and C interchange their places, B and D interchange their


places, F and H interchange their places and so on, then which letter will be 5th to
the left of Q?
Sol. As per the question the interchanges take place as follows:

A B C D E F G H I J K L M N O P

Q R S T U V W X Y Z
Here we can see that Q interchanges with S. Then to left of Q, the 5th letter
would be P because P interchanges with N.
5. How to find the Middle Letter? EXAMPLE 9. Which letter will be
q Shortcut Approach midway between 8th letter from the right
Case I : Remember that if mth and nth and 16th letter from the right in the
letter from the left in the English English alphabet.
alphabet are given then
é æ 8 + 16 ö ù
æ m + nö Sol. Middle letter = ê 27 – ç th
Middle letter = çè ÷ th letter from ë è 2 ÷ø úû
2 ø
the left. letter from left in the alphabet.
or middle letter = (27 – 12) = 15th
EXAMPLE 8. Which letter will be letter from left = 0
midway between 8th letter from the left Note : In case I and case II (m + n) must
and 16th letter from the left in the be divisible by 2.
English alphabet?
Sol. Here, m = 8 and n = 16 q Shortcut Approach
8 + 16 24 Case III :Remember that if the mth letter
then middle letter = = from the left and the nth letter from the
2 2
= 12th letter from left in the alphabet right are given then middle letter
=L
é (m – n) + 27 ù
=ê úû th letter from the
q Shortcut Approach ë 2
Case II: Remember that if mth and nth left in the alphabet.
letter from the right in the English
EXAMPLE 10. Which letter will be
alphabet are given then
Middle letter midway between 8th letter from the left
and 15th letter from the right?
æ m + nö Sol. Here, m = 8 and n = 15
=ç th letter from right
è 2 ÷ø
(8 – 15) + 27 ù
é æ m + nö ù é æ m + nö ù Then middle letter = é
= ê 26 + 1 – ç êë úû
÷ø ú = ê 27 – çè ÷ th 2
ë è 2 û ë 2 ø úû
letter from the left in the English alphabet. é 20 ù
= ê ú = 10th
ë2û
Alphabet & Number Test 23
letter from left in the English manner. Further, there is also a
alphabet = J. possibility that all the 26 letters of
Note : In case III, (m – n) + 27 must be English alphabet are not available
divisible by 2. in the series. Even same letters may
be repeated in the series.
2. Random Series of
Alphabet EXAMPLE 12. How many letters in
This series is not in the proper the following series are immediately
sequence and letters take their preceded by B but not immediately
position in the series in jumbled followed by D?
R S P Q B A H M A C F B A D N O P B A C D.
× ×
Sol. R S P Q B A H M A C F B A D N O P B A C D

ü ü
\ Only the two times A fulfill the 4. Problems of Letter Gap
given condition and those A have
been marked with the correct sign Case I:
(ü). Those not fulfilling the condition EXAMPLE 13. How many pairs of
have been marked with the cross
letters are there in the word
sign (×). \ Required answer is 2.
‘DREAMLAND’ which have as many
3. Problems on Word letters between them as in the English
Formation alphabet?
In such problems, a word is given
Sol. Here, we are asked to solve problem
and you have to find out the number
according to English alphabet. In
of words to be formed out of some
letters drawn from that particular this case we have to count both
word. ways. It does mean that we have to
count from left to right and from right
EXAMPLE 11. How many meaningful
to left. Let us see the following
words can be formed from the 3rd, 4th, 6th presentation:
and 8 th letter of the word
‘CONTROVERSIAL’?
Sol.
D R E A M L A N D
C O N T R O V E R S I A L

rd
3 4th 6th 8th
The above presentation makes it
Now, from letters N, T, O and E, clear that the required pairs of letters
two words ‘NOTE’ and ‘TONE’ can are 4. (Pairs: DA, EA, ML and LN)
be formed.
EBD_7420
24 Alphabet & Number Test
Case II: Sol. According to the question,

EXAMPLE 14. How many pairs of Original Numbers : 7 1 3 3 6 1 4 5 8 9 3 2 7 2 4

letters are there in the word New Arrangement : 3 1 7 1 6 3 8 5 4 2 3 9 4 2 7


‘DREAMLAND’ which have the same So, here only one number is even
number of letters between them as in i.e., 854.
the English alphabet in the same
16. What is the difference between the
sequence.
sum of the three digits of the
Sol. Here, we are asked to solve problems
highest and that of the second
according to the alphabetical
highest number?
sequence. It does mean that we
have to do counting only from left Sol. Highest number = 932
to right. Let us, see the following Second highest number = 724
presentation:
So, the required difference
D R E A M L A N D
= (9 + 3 + 2) – (7 + 2 + 4)
The above presentation makes it = 14 – 13 = 1
clear that the required pair of letters 17. If all the three digits are arranged
is only 1 (Pair: LN) in ascending order (from left to
5. Finding Digits After right) within the number, in each
of these numbers, then which of
Rearrangement
these will be second lowest ?
In this type of problems, a specified
order or pattern is used to rearrange Sol. According to the question,
the positions of digits of the Original number : 7 1 3 3 6 1 4 5 8 9 3 2 7 2 4
number. Then, either the number of New arrangement : 1 3 7 1 3 6 4 5 8 2 3 9 2 4 7
those digits is found out whose
So, the second lowest number will
positions remain unchanged after be 137.
rearrangement or the digit at
18. If the positions of the second and
particular place from left or right of
the third digits are interchanged
the number is to be found out.
in each of these numbers, then
EXAMPLE : (Direction (Qs. 15-19) which of these will be exactly
Following questions are based on the divisibly by 2 ?
five three-digit numbers given below: Sol. According to the question,
713 361 458 932 724 Original Numbers : 7 1 3 3 6 1 4 5 8 9 3 2 7 2 4
15. If the positions of the first and the
third digits are interchanged in New Arrangement : 7 3 1 3 1 6 4 8 5 9 2 3 7 4 2
each of these numbers, then which So, two numbers will be exactly
of these will be an even number. divisible by 2, i.e., 316 and 742.
Alphabet & Number Test 25
19. If the given numbers are arranged Now, digits sum of the 3rd number
in descending order, then what will from the right
be the square of the digits sum of = 7 + 1 + 3 = 11
the third number from the right
\ Square of the digits sum
end of the new arrangement ?
= (11)2 = 121.
Sol. According to the question,
Original Numbers : 7 1 3 3 6 1 4 5 8 9 3 2 7 2 4

New Arrangement : 9 3 2 7 2 4 7 1 3 4 5 8 3 6 1

3rd from the right end

PRACTICE EXERCISE
1. If it is possible to make only one 4. Each vowel of the word BUCK-
meaningful word with the first, SHOT is changed to the next letter
second, fifth and sixth letters of the in the English alphabetical order and
word PYGMALION, which of the each consonant is changed to the
following would be the second previous letter in the English
letter of that word from the right alphabetical order. If the new
end? If no such word can be made, alphabets thus formed are arranged
give 'X' as your answer and if more in alphabetical order (from left to
than one such word can be formed, right). Which of the following will
give your answer as' Z '. be fifth from the right ?
(a) X (b) P (a) R (b) B
(c) Y (d) A (c) G (d) J
(e) Z (e) P
2. How many such pairs of letters are 5. The positions of how many alpha-
there in the word TRIBUNAL each bets will remain unchanged if each
of which has as many letters of the alphabets in the word
between them in the word as in the WORTHY is arranged in alpha-
English alphabet ? betical order from left to right ?
(a) None (b) One (a) None (b) One
(c) Two (d) Three (c) Two (d) Three
(e) More than three (e) More than three
3. How many meaningful English 6. Each odd digit in the number
words can be made with the third, 5263187 is substituted by the next
fifth, seventh and ninth letters of higher digit and each even digit is
substituted by the previous lower
the word DOWNGRADED, using digit and the digits so obtained are
each letter only once in each word? rearranged in ascending order,
(a) None (b) One which of the following will be the
(c) Two (d) Three third digit from the left end after the
rearrangement?
(e) More than three
EBD_7420
26 Alphabet & Number Test
(a) 2 (b) 4 10. If the first alphabet of each of the
(c) 5 (d) 6 words is changed to the next
alphabet in the English alphabetical
(e) None of these
series, how many meaningful
7. If the digits in the number 79246358 English words will be formed?
are arranged in descending order
(a) One (b) Two
from left to right, what will be the
difference between the digits which (c) Three (d) Four
are third from the right and second (e) Five
from the left in the new
atrangement? DIRECTIONS (Qs. 11-13) : The
following questions are based on the five
(a) 1 (b) 2
three digit numbers given below:
(c) 3 (d) 4
612 589 743 468 297
(e) 5
11. If two is added to the first digit of
DIRECTIONS (Qs. 8-10) : The following each of the numbers, how many
questions are based on five words given numbers thus formed will be
below: completely divisible by three?
RAT ONE BUT AND SAW (a) None (b) One
(c) Two (d) Three
(The new words formed after
performing the mentioned (e) Four
operations may or may not 12. If the position of the second and
necessarily be meaningful English the third digits of each of the
words.) numbers are interchanged, in how
8. If in each of the given words, each many numbers thus formed will the
alphabet is changed to the next last digit be a perfect square? ('1' is
letter in the English alphabetical also a perfect square)
series, in how many words thus (a) One (b) Two
formed have the consonan ts (c) Three (d) Four
changed to vowels?
(e) Five
(a) One (b) Two
13. What will be the resultant if the third
(c) Three (d) Four digit of the second lowest number
(e) Five is divided by the second digit of
the highest number?
9. How many such pairs of letters are
there in the word highlighted in bold, (a) 4 (b) 1
each of which has as many letters (c) 6 (d) 5
between them in the word (in both
(e) 2
forward and backward directions)
as they have between them in the DIRECTIONS (Qs. 14-15) : Study the
English alphabetical order? following arrangement carefully and
(a) None (b) One answer the questions given below.
(c) Two (d) Three P 7 3 G # R E $ 4 F K 1 U% W H
2 NI 5 BQ Y6 @ HM b 8 VD
(e) Four
Alphabet & Number Test 27
14. If all the symbols and numbers are 15. How many such numbers are there
dropped from the above in the above arrangement, each of
arrangement, which of the which is immediately preceded by
following will be the fifteenth from a symbol and immediately followed
the right end? by a letter?
(a) P (b) R (a) None (b) One
(c) E (d) F (c) Two (d) Three
(e) None of these (e) More than three

HINTS & SOLUTIONS


1. (d)
5. (e) W O R T H Y
1 2 \3 4 5 6 7 8 9
H O R T W Y
P Y G M A L I O N
6. (b) 5 2 6 3 1 8 7
Meaningful Word Þ PL A Y
6 1 5 4 2 7 8
2. (e) 1<2<4<5<6<7<8
7. (d) 7 9 2 4 6 3 5 8
20 18 9 2 21 14 1 12
9 8 7 6 5 4 3 2
T R I B U N A L
Required difference = 8 – 4 = 4
Sol. (8-10) :
3. (b) The third, fifth, seventh and 8. (e) RAT Þ SBU; ONE Þ POF;
ninth letters are W, G, A, and E BUT Þ CVU; AND Þ BOE;
respectively.
SAW Þ TBX
Only one meaningful word can
be formed using these letters 9. (b) 18 1 20
which is WAGE. R A T
4. (d) After shifting,
B U C K S H O T 10. (b) RAT Þ SAT; ONE Þ PNE;
¯ ¯ ¯ ¯ ¯ ¯ ¯ ¯ BUT Þ CUT; AND Þ BND;
A V B J R G P S
SAW Þ TAW
After arranging in alphabet
sequence. Meaningful Words
A V B J R G P S Þ SAT, CUT
¯ ¯ ¯ ¯ ¯ ¯ ¯ ¯ 11. (b) 612 Þ 812; 589 Þ 789;
A B G J P R S V 743 Þ 943; 468 Þ 668;
Hence, letter J will be fifth from 297 Þ 497
the right.
EBD_7420
28 Alphabet & Number Test
812 789 13. (e) Second lowest number
= 270.66; = 263;
3 3 Þ 46 8
943 668
= 314.33; = 222.66; Highest number
3 3
8
497 Þ 7 4 3 = =2
= 165.66 4
3
14. (c) If all the symbols and numbers
12. (c) 612 Þ 621; 589 Þ 598;
are dropped, the new
743 Þ 734 ; 468 Þ 486; arrangement is P G R E F K U
297 Þ 279 WHNI BQ YHMVD
15. (c) Only two
6 2 1; 7 3 4 : 2 7 9 b 8 V
$ 4 F
Coding-Decoding 29

Chapter

4 Coding-Decoding
INTRODUCTION have to be decoded. They are coded as
per a definite pattern/ rule which should
In this segment of commonsense
be identified first. Then the same is
reasoning, secret messages or words
applied to decode another coded word.
FORWARD ORDER POSITION (Left to Right)

A B C D E F G H I J K L MN O P Q R S T U VWX Y Z
1 2 3 4 5 6 7 8 9 10 11 12 13 14 15 16 17 18 19 20 21 22 23 24 25 26

BACKWARD ORDER POSITION (Right to Left)

A B C D E F G H I J K L MN O P Q R S T U VWX Y Z
26 25 24 23 22 21 20 19 18 17 16 15 14 13 12 11 10 9 8 7 6 5 4 3 2 1
Backward order position of any 1 2 3 4 5 6 7 8 9 10 11 12 13
letter = 27 – Forward order position
of that letter A B C D E F G H I J K L M
For example, 26 25 24 23 22 21 20 19 18 17 16 15 14
Backward order position of C
= 27– Forward position of C Z Y X WV U T S R Q P O N
= 27 – 3 = 24 CIRCULAR ARRANGEMENT
OPPOSITE LETTERS (i) Clockwise Arrangement
Two letters are called opposite ® ®
®Z A
B ®
letters, if sum of their corresponding ®Y · · · ·C ®
positions is equal to 27. · 26 1 2 3 D
®

X 25
·
®

4 E
·

24
® ® ® ®

Opposite position of any letter


·

W 23 5
® ®
·

F
= 27– corresponding position of
· · ·

V 22 6
· ·

that letter 7 G
U 21 Clockwise
® ®

For example, Movement 8 H


20
· ·

T 9 ·I
Opposite position of D = 27 19
S 10
®

– Corresponding position of D 18
·

11 J
®
·

R 17
·

12
®

= 27 – 4 = 23. 16
·

· ·
15 14 13 K
®

Q · · ·
Hence D and W are opposite letters ® P L ®
O N
® ® ® M ®
EBD_7420
30 Coding-Decoding
(ii) Anti-clockwise Arrangement Similarly, every letter in the word
‘BOLD’ will move one place in
forward alphabetical sequence as
given below:
B O L D
+1 +1 +1 +1
Anti-clockwise C P M E
Movement
\ Code for ‘BOLD’ will be ‘CPME’.
Pattern 2: Coding in backward
sequence.
EXAMPLE 2. If ‘NAME’ is coded as
‘MZLD’, then how will code 'SAME'?
Sol. Here, every letter of the word
If we have to find the letter one
‘MZLD’ moves one place in
place forward to A, then the letter backward alphabet sequence. Let us
will be B and this result can be see:
found out by using either linear or N A M E
circular arrangement but when one –1 –1 –1 –1
is asked to find the letter which is
M Z L D
one letter backward of A (Z is one
Similarly, every letter of the word
letter backward A) or one letter ‘SAME’ will move one place in
forward of Z (A is the one letter backward alphabet sequence. Let us
forward Z), then only circular see :
arrangement gives such result. S A M E
–1 –1 –1 –1
TYPE-1 CODING BY LETTER
SHIFTING R Z L D
\ Code for ‘SAME’ will be ‘RZLD’.
Pattern 1: Coding in forward Pattern 3: Coding based on skipped
sequence sequence.

EXAMPLE 1. If ‘GOOD’ is coded as EXAMPLE 3. If the word ‘FACT’ is

‘HPPE’, then how will you code ‘BOLD’? coded as ‘IDFW’; then how will you code
‘DEEP’?
Sol. Here,every letter of the word
Sol. Here, every letter of the word shifts
‘GOOD’ shifts one place in forward three place in forward alphabetical
alphabetical sequence. order.
G O O D F A C T
+1 +1 +1 +1 +3 +3 +3 +3
H P P E I D F W
Coding-Decoding 31
Similarly, ‘DEEP’ can be coded. Let Similarly. ®
us see: ®
®
D E E P
®
+3 +3 +3 +3
G H H S
Pattern 6: Opposite Letters Coding
\ Code for ‘DEEP’ will be ‘GHHS’.
In this pattern, each letter of a word
Pattern 4: Mixed Sequence Coding
is coded with its opposite letter of
(Forward and Backword)
the alphabet.
EXAMPLE 4. In a certain code
language ‘AMIT’ is written as EXAMPLE 6. If in a certain code
‘RAMA’, then how will ‘BOOT’ be language ‘NATURAL’ is coded as
coded in that language ? ‘MZGFIZO’ then how will ‘CARE’
A M I T be written in that language?
Sol. +17 –12 +4 –19
14 1 20 21 18 1 12 13 26 7 6 9 2615
R A M A N AT U RA L M ZG F I Z O
B O O T Opposite letter
Similarly +17 –12 –4 –19 Opposite letter
S C K A Opposite letter
Pattern 5: Direct Letter Coding Opposite letter
Opposite letter
In this type of coding, some letter
Opposite letter
group/words are assigned some
Opposite letter
codes which do not follow any rule
but are direct code for a particular 3 1 18 5 24 26 9
letter of the letter group/words. C AR E XZIV

These direct codes are then used


Similarly. Opposite letter
to form the codes of an another Opposite letter
Opposite letter
given word.
EXAMPLE 5. In a code language,
\ CARE Þ XZIV
‘APPLE’ is written as ‘PQQRS; ‘RIS’ q Shortcut Approach
is written as ‘ABC’ and ‘MANGO’
is written as ‘TPXYZ’. How will • Observe alphabets given in the
‘ROSE’ be written in that same code code carefully.
language? • Find the sequence it follows
® ® whether it is ascending/descending
Sol. and ®
® ® ® • Detect the rule in which the
® ® ® alphabets follow.
® ® • Fill the appropriate letter in the
® ® blank given.
EBD_7420
32 Coding-Decoding
TYPE-2 : CODING BY (i) When the number of letters of the
SUBSTITUTION word is even
In this coding, some words are replaced EXAMPLE 9. In a certain code
by some other words and on the basis of ‘TEMPLE’ is written as ‘METELP’,
these new words the code is derived. then how will ‘ACTION’ be written
EXAMPLE 7. If 'cages' are called in that code ?
'rockets', 'rockets' are called 'traps', Sol. T E M P L E ® M E T E L P
'traps' are called 'planets', 'planets' are 1 2 3 4 5 6 3 2 16 5 4
called 'aeroplanes', 'aeroplanes' are Similarly
called 'cycles' and cycles' are called ACT I O N ® TCAN O I
'cars', what is Earth 1 2 3 4 5 6 3 2 1 6 5 4
(a) Cycles (b) Rockets
(c) Planet (d) Aeroplanes (ii) When the number of letters of the
(e) Cars word is odd
Sol. Earth is a planet and here planets EXAMPLE 10. In a certain code
are called aeroplanes. So, earth will language ‘MAGICAL’ is written as
be called aeroplanes. ‘MAGILAC’, then how will
TYPE -3: CODING BASED ‘LETTERS’ be written in that
ON REARRANGEMNT OF code?
LETTERS Sol. M A G I C A L; ® M A G I L A C
In this type of coding, the letter of the 12 34 5 6 7 1 2 3476 5
original word are rearranged in a particular Similarly
manner to obtain the code. Such coding LETTERS ® LETTS RE
can be of the following types. 1 2 3 4 5 6 7 1 2 3 4 7 6 5
(a) CODING BY REVERSING (iii) When letters of the word are divided
LETTERS into two or more groups and then
In this coding, all letters of a word has all or some particular groups are
been reversed. written in reverse order.
EXAMPLE 8. If ‘TEMPERATURE’ is EXAMPLE 11. In a certain code
coded as ‘ERUTAREPMET’, then how language ‘POSITION’ is written as
will you code ‘EDUCATION’ following ‘POSTIION’ then how will
the same scheme. ‘LANGUAGE’ be written in that
Sol. Here, the word ‘TEMPERATURE’ code?
has been reversed. Hence, the code for Sol.
‘EDUCATION’ will be ‘NOITACUDE’. POSITION®POSTIION
(b) When letters of the word are divided 1 2 34 5 6 7 8 1 2 3 5 46 7 8
into two parts and then both the parts Similarly
are written in reverse order or the first
part is written in reverse order at the place L A N G U A G E ® LA N G U A G E
of second part and second part is written 1 2 3 4 56 78 1 2 35 46 7 8
in reverse order at the place of first part. (iv) When first and last letter of the
Following situations can arise in such word remain at the same place but
cases middle letters get reversed
Coding-Decoding 33
EXAMPLE 12. In a certain code Similarly.
2 4 14 1612 14 1 3
language ‘RESPONSE’ is written as 3 15 13 2
BD N PLN AC
C OM B
‘RSNOPSEE’ then how will
DAUGHTER be written in that
code?
Sol.
\ COMB Þ BDNPLNAC
R E S P O N S E® R S N O P S E E
12 3 4 56 7 8 1 7 65 43 2 8 TYPE 5 : CODING IN
FICTITIOUS LANGUAGE
Similary
In some cases of coding-decoding,
D A U G H T E R® D E T H G U A R fictions language is used to code some
1 2 34 5 6 7 8 1 76 5 4 32 8 words. In such questions, the codes for
(v) When each letter of the word is a group of words is given. In such types
written at a certain place of problems, codes for each word can be
found by eliminating the common words.
EXAMPLE 13. In certain code
EXAMPLE 14. In a certain code
language ‘TRANGLE’ is written as
‘AGTRELN’, then how will language ‘over and above’ is written as
‘MAGICAL’, be written in that ‘da pa ta’ and ‘old and beautiful’ is written
as ‘Sa na pa’. How is ‘over’ written in
code?
that code language?
Sol. T R A N G L E ® A G T R E L N
1 2 3 4 5 6 7 3 5 1 276 4 Sol. Over and above ® da Pa ta
Similarly Old and beautiful ® Sa na Pa
M A G I C A L® GC M A L A I
Clearly, ‘and’ is common in both and
1 2 3 4 5 6 7 35 1 2 7 6 4 a common code is ‘Pa’.
TYPE 4: CODING OF \ Code for ‘and’ must be ‘Pa’.
LETTERS BY THEIR LEFT Code for ‘over’ = ‘da’ or ‘ta’.
AND RIGHT LETTERS Code for above = ‘da’ or ‘ta’.
In this pattern, each letter of a letter Code for old = ‘Sa’ or ‘na’
group/word is coded by its left and right Code for beautiful = ‘Sa’ or ‘na’
letters of English alphabet. \ We can’t certainly say what will
Q6. If in a certain language ‘NIL’ is be exact code for ‘over’. But it is
written as ‘MOHJKM’ then how will sure that code for ‘over’ must be
‘COMB’ be written in that either ‘da’ or ‘ta’.
language? q Shortcut Approach
14 9 12 13 15 8 10 13
Sol. N I L MO H J K M • Firstly, write the words and their
codes as given in the question in
straight line with an arrow in
middle.
EBD_7420
34 Coding-Decoding
• Now, find the common words and EXAMPLE 16. In a certain code 3 is
their corresponding codes. coded as ‘R’, 4 is coded as ‘D’, 5 is coded
• Encircle each pair with the same as ‘N’, 6 is coded as ‘P’, then find the
shape. code for ‘53446’.
Sol. As per the given condition
• Finally, we have each word and
its corresponding code. 3 4 5 6
R D N P
TYPE-6 : CODING BASED 5 3 4 4 6
ON NUMBERS Now,
N R D D P
Pattern 1: \ Code for 53446 = NRDDP.
When numerical values are given to words. TYPE-7 : MATHEMATICAL
OPERATIONS WITH THE
EXAMPLE 15. If in a certain language
POSITION NUMBERS OF
A is coded as 1, B is coded as 2, C is LETTERS
coded as 3 and so on, then find the code
for AEECD. EXAMPLE 17. In a certain code, if

Sol. As given the letters are coded as ‘TALE’is written as 38, then how will you
below: code ‘CAME’ using the same coding
scheme?
A B C D E F G H I Sol. Look at the numbered alphabet
1 2 3 4 5 6 7 8 9 and write down the number
corresponding to the letters of the
word ‘TALE’.
A E E C D T A L E
Now, 1 5 5 3 4 20 1 12 5
The fact that the code for ‘TALE’ is
38, gives you a clue that the code is
\ Code for AEECD = 15534
probably obtained by performing an
q Shortcut Approach arithmatical operations between the
numbers related to each letter. Let
• First you have to observe the us see :
number code. 20 + 1 + 12 + 5 = 38
• Now, n otice the position of Thus, the code for ‘CAME’ is
C A M E
number.
3 + 1 + 13 + 5 = 22
• Search the common pattern. \ Code for ‘CAME’ = 22
Pattern 2 : EXAMPLE 18. If in a certain code
When alphabetical code value are given language ‘RAMAN’ is written as
for numbers. ‘23.5’, then how will ‘CAPACITY’
be written in that code?
Coding-Decoding 35
Sol. R AMA N [18 + 1 + 13 + 1 + 14] ¸ 2 Þ 47/2
= 23.5

Similarly,
CA PAC I T Y [3 + 1 + 16 + 1 + 3 + 9 + 20 + 25] ¸ 2 Þ 78/2
= 39

\ CAPACITY Þ 39
Remember Matrix I
· If more than one codes are given 0 1 2 3 4
then likely the required code can be 0 I A U E O
drived from the question itself and 1 E U O A I
you will no need to solve it
2 O A I E U
mathematically.
3 E U A O I
· If the code for a word is a one digit
4 E I O A U
number then likely the position of
the letters are added and the digits Matrix II
are summed up until the one digit
number is arrived at. 5 6 7 8 9
5 K R L M N
TYPE-8 : MATRIX CODING 6 M R K N L
In this type of questions two matrices 7 K N M L R
are given. In each matrix there are 25 cells 8 M L K R N
and these cells contain two classes of
9 N R L K M
alphabets. The columns and rows of
matrix I are numbered from 0 to 4 and MONK
that of matrix II from 5 to 9. A letter from (a) 58, 33, 67, 98
these matrices can be represented first (b) 65, 02, 59, 67
by its row number and next by its column (c) 65, 04, 89, 75
number. For example. ‘A’ Can be (d) 65, 20, 89, 68
represented by 32 or 43. Sol.
A ® 01, 13, 21, 32, 43
EXAMPLE 19. E ® 03, 10, 23, 30, 40
Directions: In this question find out the I ® 00, 14, 22, 34, 41
correct set of number pairs for the given O ® 04, 12, 20, 33, 42
word from the two matrices given above.
EBD_7420
36 Coding-Decoding
U ® 02, 11, 24, 31, 44
EXAMPLE
K ® 55, 67, 75, 87, 98
L ® 57, 69, 78, 86, 97 Directions (Q. Nos. 21-25) Read the
M ® 58, 65, 77, 85, 99 following information to answer
the questions that follow. In
N ® 59, 68, 76, 89, 95 Column I, some words are given.
R ® 56, 66, 79, 88, 96 In Column II, their codes are given
So, 65, 04, 89, 75 is correct and they are arranged in the same
TYPE 9 : SYMBOL CODING order in which they are in Column
BASED ON SIMILARITY I but the letters in the code in
Column II are not in the same order
In this coding, various symbols are in which the letters of the words
assigned to the letters of a word and are given in Column I. Study the
based upon their correlation or similarity, columns and give your answer on
you have to determine the rules or the basis of that.
pattern being followed.
Column I Column II
EXAMPLE 20. If in a certain code (1) FLOUR (A) xncap
language ‘ROPE’ is written as, ‘% 5 (2) TAP (B) ksd
7 $’, ‘DOUBT’ is written as ‘3 5 # 8 (3) ROSE (C) cmrn
«’, and ‘LIVE’ is written as ‘@ 2 4
$’, then how will ‘TROUBLE’ be (4) LOTUS (D) smcpx
written in that language? (5) SAIL (E) kptm
R O P E D O U B T 21. Find the code for F.
Sol. ¯ ¯ ¯ ¯ ¯ ¯ ¯ ¯ ¯ (a) p (b) c
% 5 7 $ 3 5 # 8 « (c) a (d) x
22. Which letter is the code for P?
L I V E
(a) k (b) s
and ¯ ¯ ¯ ¯
@ 2 4 $ (c) c (d) d
23. Find the code for L.
T R O U B L E (a) n (b) c
Similarly. (c) k (d) p
« % 5 # 8 @ $ 24. What is the code for E?
(a) c (b) m
\ TROUBLE Þ «%5# 8@$ (c) r (d) n
25. Which of the following options is
TYPE 10 : CODING BY the code for 0?
COMPARISON (a) x (b) c
In this coding, some words are given in (c) m (d) r
one column and their codes are given in Sol.
another column. But the given codes are 21. (c) Clearly, the code for F is a
not in the same order of words given. 22. (d) Clearly, the code for P is d
You have to find out the codes of words 23. (d) Clearly, the code for L is p
on the basis of comparison of their 24. (c) Clearly, the code for E is r
properties, traits, ete. 25. (b) Clearly, the code for O is c
Coding-Decoding 37
TYPE 11 : CONDITIONAL Sol.
CODING 26. (d) F rom Statement II.
N F R S C A
In this coding, letters/numbers are
given and their codes are given
0 5 3 % 2 0
right under them, you have to find
27. (a) From Statement III.
out code for a particular letter
If both the first and last letter
group/number coding to the given
are vowels, the both are coded
condition. as the last letter.
EXAMPLE A R F T H E

Directions (Q. Nos. 26-27) In each


% 3 5 8 # %
question below, is given a group
of letters followed by four TYPE 12 : DECIPHERING
combinations of digits/symbols
NUMBERS AND SYMBOLS
numbered (a), (b), (c) and (d). You
have to find out which of the four
CODES FOR MESSAGES
combinations correctly represents In this type of questions, a few
the group of letters based on the groups of numbers/symbols, each
coding a certain message are given.
following coding system and the
Through a comparison of the given
conditions that follow and mark
coded messages, taking two at a
the number of that combination as time, the candidate is required to
your answer. If none of the find the number/symbol code for
combinations correctly represents each word and then formulate the
the group of letters, mark (e), i.e. code for the given message.
‘None of these’. as your answer.
EXAMPLE 28. In a certain code
ER C FL NHKPTAS G language. ‘617’ means ‘sweet and
Code % 3 2 5 @ 7 # 6 1 8 4 % 9 hot’, ‘735’ means ‘coffee is sweet’
(i) If the first letter is a vowel and the and ‘263’ means ‘tea is hot’. Which
last letter is a consonant, both are of the following would mean ‘coffee
to be coded as O. is hot’?
(ii) If the first letter is a consonant and Sol. In the first and third statements, the
the last letter is a vowel both are to common code digit is ‘6’ and the
common word is ‘hot’, So, ‘6’ means
be coded as O.
‘hot’
(iii) If the first letter as well as the last In the second and third statements,
letter are vowels, both are to be the common code digit is ‘3’ and
coded as the code of the last letter. the common word is ‘is’. So ‘3’
26. NFRSCA means ‘is’.
(a) 753 % 20 (b) 053% 24 In the first and second statements,
(c) 0232% 0 (d) 053% 20 the common code digit is ‘7’ and the
27. ARFTHE common word is ‘sweet’. So, in the
(a) % 358#% (b) 4358#% second statement ‘5’ means ‘coffee’.
(c) 4358#4 (d) 96385#% Clearly, ‘536’ would mean ‘coffee is
hot’.
EBD_7420
38 Coding-Decoding

PRACTICE EXERCISE
1. In a certain code language, "BAD" Matrix – II
is written as "7" and "SAP" is
written as "9". How is "BAN" 5 6 7 8 9
written in that code language? 5 I R A J M
(a) 8 (b) 3
6 A J I M R
(c) 4 (d) 6
2. In a certain code language 7 J M R A I
"NIGHT" is written as "ODDGM" 8 R A M I J
an d "DARK" is wri tten as
9 M I J R A
"GOYC". How is "GREEN"
written in that code language? (a) 00, 13, 57, 76(b) 11, 04, 86, 59
(a) IABPF (b) MCBNB (c) 23, 22, 99, 95 (d) 32, 40, 66, 68
(c) OGHVL (d) FPBAI
4. In a certain code language "who are
3. A word is represented by only one
you" is written as "432" , "they is
set of numbers as given in any one
you" is written as "485" and "they
of the alternatives. The sets of
numbers given in the alternatives are dangerous" is written as "295".
are represented by two classes of How is "dangerous"written in that
alphabets as shown in the given code language?
two matrices. The columns and (a) 2 (b) 4
rows of Matrix – I are numbered (c) 5 (d) 9
from 0 to 4 and that of Matrix – II 5. In a certain code language, "RIVER"
are numbered from 5 to 9. A letter is written as "12351" and "RED" is
from these matrices can be written as "156". How is "DRIVER"
represented first by its row and next written in that code language?
by its column, for example, 'K' can (a) 612311 (b) 612531
be represented by 10, 31, etc., and (c) 621351 (d) 612351
'M' can be represented by 76, 87, 6. In a certain code language ‘GUST’
etc. Similarly, you have to identify is coded as ‘@7$2’ and ‘SNIP’ is
the set for the word "SCAM". coded as ‘957#’ and ‘GAPE’ is
Matrix – I coded as ‘b$35’. How will ‘SING’
be coded in the same code?
0 1 2 3 4 (a) 9$7# (b) 59#$
0 S P K N C (c) 9b7$ (d) 7$59
(e) $27#
1 K S C P N 7. In a certain code language, 'how can
2 P C N S K you go' is written as 'ja da ka pa,'
3 N K S C P 'can you come here' is writter as 'na
4 C N P K S ka sa ja' and 'come and go' is written
Coding-Decoding 39
as 'ra pa sa'. How is 'here' written in @D6, %R5, !N4, ?Y6
that code language ? 'Four people from USA' is written as
(a) ja (b) na @M4, %E6, #A3, @R4
(c) pa (d) Data inadequate 'Urban development programme
(e) None of these launched' is written as % E9, *T11, #N5
DIRECTIONS (Qs. 8-9) : Study the &D8
following information to answer the 'Dhaya likes forties hero' is written as
given questions. @S7, &S5, *A5, $O4
In a certain code, 'strong financial 11. The code for the word 'People' is
economy' is written as 'mo tic su', (a) @M4 (b) %E6
'financial inclusion needed' is written as (c) #A3 (d) @R4
'da ra su' and 'economy crisis inclusion' (e) None of these
is written as 'ye da mo'. 12. The code '*A5' denotes which of
8. What is the code for 'financial'? the following word ?
(a) da (b) su
(a) Likes (b) Hero
(c) mo (d) ra
(e) Can't be determined (c) Forties (d) Dhaya
9. What does 'tic' stand for? (e) None of these
(a) economy (b) financial 13. The code word of 'Four' is
(c) strong (d) needed (a) @R4 (b) %E6
(e) Either economy or strong (c) @M4 (d) #A3
10. If ‘yellow’ means ‘green’, ‘green’ (e) None of these
means ‘white’, white means ‘red’,
14. '#' denotes which letter of the given
‘red’ means ‘black’, ‘black’ means
words ?
‘blue’ and ‘blue’ means ‘violet’,
(a) N (b) F
which of the following represents
the colour of human blood ? (c) L (d) D
(a) black (b) violet (e) U
(c) red (d) blue 15. According to the given code word,
(e) None of these what will be the code for ' Data Line
reach points' ?
DIRECTIONS (Qs. 11-15) : Study the (a) *4A &4E @5H %6S
information below and answer the (b) *4A &4E !5H %6S
following question:
(c) *4A &4E #5H %6S
In a certain code language, (d) *4A &4E $5H %6S
'Thin paper neatly folded' is written as (e) None of these
EBD_7420
40 Coding-Decoding

HINTS & SOLUTIONS


1. (a) As, Similarly,
BAD = 2 + 1 + 4 Þ 7 D R I V E R
SAP = 19 + 1 + 16 = 36
Þ3+6=9 ¯ ¯ ¯ ¯ ¯ ¯
Similarly, 6 1 2 3 5 1
BAN = 2 + 1 + 14 = 17 6. (a)
Þ1+7=8
G U S T @ 7 $ 2
2. (a) As,
N I G H T S N I P 9 5 7 #

b $ 3 5
G A P E
+5 +1 Therefore, S I N G Þ 7 9 # $
+4 +3 +2 7. (b)
how can you go ja da ka pa
O D D G M
and can you sa
come here na ka ja
D A R K
come and ra pa sa
go
Codes are:
+1
+4 +3 +2 how Þ da come Þ sa
G O Y C
can Þ ja or ka here Þ na
you Þ ja or ka and Þ ra
Similarly,
go Þ pa
G R E E N The code for here is ‘na’.
8. (b) 9. (c)
10 (e) The colour of human blood is
+5 +1 red. Here white means red.
+4 +3 +2 Therefore white is our answer.
I A B P F Do not opt for black because
red means black implies that
3. (b) Code 11, 04, 86, 59 will black is called red.
resemble SCAM when 11. (b) People - %E6
matched from given two P- %
matrices. No of letters - 6
4. (d) Last letter - E
Who are you = 4 3 2 12. (d) *A5 - Dhaya
D denote *
they is you = 4 8 5 A denote Dhaya end with A 5
they are dangerous = 2 9 5 denote no of letter (Dhaya => 5)
13. (a) Four => @R4
5. (d) As, F-@
R I V E R R E D R - last letter of the word R and
Total no of letter 4
¯ ¯ ¯ ¯ ¯ ¯ ¯ ¯ 14. (e) T =!, P = %, N = ?, F = @, U =#,
1 2 3 5 1 1 5 6 D = *, L = &, H = $
15. (e)
Blood Relations 41

Chapter

5 Blood Relations
INTRODUCTION (a) Past generations of father : Great
grandfather, great grandmother,
Blood relation does mean biological grandfather, grandmother etc.
relation. Remember a wife and husband (b) Parallel generations of father:
are met biologically related but they are Uncles (Brothers of father), Aunts
biological parents of their own children. (sisters of father) etc.
Similarly, brother, sister, paternal (c) Future generations of father:
grandfather, paternal grandmother Sons, daughters, grandsons,
maternal grandfather, maternal granddaughters etc.
grandmother, grandson, granddaughter,
niece, cousin etc. are our blood relatives. BLOOD RELATION FROM
MATERNAL SIDE
TYPES OF BLOOD
This type of blood relations can also be
RELATIONS subdivided into three types:
There are mainly two types of blood (a) Past generations of mother:
relatives: Maternal great grandfather,
(i) Blood relation from paternal side maternal great grandmother,
(ii) Blood relation from maternal side maternal grandfather, maternal
grandmother etc.
BLOOD RELATION FROM (b) Parallel generations of mother:
Maternal uncles, maternal aunts etc.
PATERNAL SIDE (c) Future generations of mother:
This type of blood relation can be further Sons, daughters, grandsons,
subdivided into three types: granddaughters etc.

Table of Blood Relations


1 Son of father or mother Brother
2 Daughter of father or mother Sister
3 Brother of father Uncle
4 Brother of mother Maternal uncle
5 Sister of father Aunt
6 Sister of mother Aunt
7 Father of father Grandfather
8 Father of father's father Great grand father
9 Father of grandfather Great grandfather
EBD_7420
42 Blood Relations
10 M o th er of fath er Gran dmo ther
11 M o th er of fath er's mother Great grand mother
12 M o th er of g ran dmo ther Great grand mother
13 Fath er o f moth er M atern al g rand father
14 Fath er o f moth er's fath er Great matern al g ran d fath er
15 Fath er o f matern al g ran dfathe r Great matern al g ran dfathe r
16 M o th er of moth er M atern al g ran d mo th er
17 M o th er of moth er, mo th er Great matern al g ran dmo th er
18 M o th er of matern al g ran dmo th er Great matern al g ran dmo th er
19 W ife o f fath er M other
20 Hus ban d o f mo th er Fath er
21 W ife o f Gran dfather Gran dmo ther
22 Hus ban d o f Gran d mo th er Gran dfather
23 W ife o f s o n Dau g hter-in -law
24 Hus ban d o f daug h ter So n -in -law
25 Bro th er of Hus ban d Brother-in -law
26 Bro th er of wife Brother-in -law
27 Sis ter o f Hu s b an d Sis ter-in -law
28 Sis ter o f wife Sis ter-in -law
29 Son of b rother Nep h ew
30 Dau gh ter o f b ro ther Niece
31 W ife o f bro th er Sis ter-in -law
32 Hus ban d o f s is ter Brother-in -law
33 Son of s is ter Nep h ew
34 Dau gh ter o f s is ter Niece
35 W ife o f un cle A un t
36 W ife o f matern al u ncle A un t
37 Son /d au gh ter o f u n cle/A u nt Cou s in
38 Son /d au gh ter o f maternal
Cou s in
u ncle/maternal aun t
39 Son /d au gh ter o f s is ter of Fath ar Cou s in
40 Son /d au gh ter o f s is ter of M o th er Cou s in
41 Only s o n o f grand fath er Fath er
Blood Relations 43
42 Only daughter of maternal grandfather Mother
43 Daughter of grandfather Aunt
44 Sons of grandfather other than father Uncle
45 Son of maternal grandfather/maternal Maternal Uncle.
grand mother
46 Only daughter in law of grandfather/ Mother
grandmother
47 Daughters in law of grandfather/ Aunt other than mother
grandmother
48 Daughters-in-law of maternal Aunt maternal
grandfather/grandmother
49 Neither brother nor sister Self

SOME IMPORTANT q Shortcut Approach


INFORMATION ABOUT Ÿ While solving blood relation
BLOOD RELATION based question, first of all find out
A. Without the information of gender, that two persons between whom
no relationship can be established a relationship has to be
between two people. For example, established.
If given that R is the child of P & Ÿ Next, try to find out middle relation.
Q, then we can only say that P & Ÿ Finally, find out the relationship
Q are the parents of R. But we can between two persons to be
not find out: identified for this purpose.
(i) R is the son of P & Q or R is
the daughter of P & Q. TYPES OF PROBLEMS
(ii) Who is mother of R and who
is father of R. (1) General Problems on Blood
But if we have given that P is a Relation
male, Q is a female and R is male, (2) Blood Relation based on Family
then we can easily say that R is Tree
the son of P and Q. Further we can (3) Coded Blood Relation
also say that P is father of R and Q (1) General Problem on
is mother of R. Blood Relation
B. Gender can not be decided on the
basis of name. For example, in Sikh EXAMPLE 1. Pointing towards a
community the names like Manjit, photograph, Mr. Sharma said, “She is the
Sukhvinder etc. are the names of only daughter of mother of my brother’s
both male and female. Similarly, in
the Hindu Community ‘Suman’ is sister.” How is Mr. Sharma related to the
the name of both male and female. lady in the photograph?
EBD_7420
44 Blood Relations
Sol. Here, we have to find relationship As per the question Q is the brother of C
between Mr. Sharma & the lady in and C is the sister of Q. Hence, relation
the photograph. between C & Q has been presented as
Mother of my brother’s sister does
mean my (Mr. Sharma’s) mother. (C –
— Q+ ) where ‘–’ sign above C
Only daughter of Mr. Sharma’s
makes it clear that C is a female and ‘+’
mother does mean “sister of Mr.
sign above ‘Q’ makes it clear that Q is a
Sharma”.
male. Similarly, for R and D. The
q Shortcut Approach æ+ ö
presentation ç R — D- ÷ has been
Read the statement from right to è ø
left to develop the relation by made. Further according to the question,
using blood relation table. A and C are having a husband and wife
relationship and hence this has been
(2) Blood Relation Based
on Family Tree æ+ ö
presented as ç A « C - ÷ . As it is
Some symbols are used to draw è ø
already given that C is the sister of Q
family tree as below:
and A and C are wife and husband, this
‘ «’ is used for husband & wife. becomes clear that A is the male member
‘___’ is used for brother & sister of the family and this is the reason A has
‘ | ’ is used for parents (father or ‘+’ as its gender sign. Lastly, the vertical
mother). Parents are put on top line gives father and son relationship and
while children are put at the æ A+ ö
bottom. has been presented as çç | + ÷÷ . Now from
‘–’ or minus sign is used for female èR ø
‘+’ or plus sign is used for male. this family tree it becomes clear that C is
the mother of R and D and as Q is the
EXAMPLE 2. Q is the brother of C and brother of C, then Q will definitely be the
C is the sister of Q. R and D are brother maternal uncle of R & D. Hence, we can
and sister. R is the son of A while A & C say that Q is the maternal uncle of D and
are wife and husband. How is Q related this is the required answer for our
with D. question.
Sol. Using the symbols, we can make a
family tree and solve the given Note : In solving family tree based
problem. Let us see the family tree: relations, make sure that your diagram
Family tree : is in correct representation.
– +
A+ C Q (3) Codded Blood Relations
EXAMPLE 3. If P + Q means P is
husband of Q, P/Q means P is the sister
of Q, P*Q means P is the son of Q. How
is D related to A in D*B + C/A ?
R+ D–
Blood Relations 45
Sol. C/A – C is sister of A. Ÿ Do not assume the gender of any
B + C/A – B is brother-in-law of A person in the question just based
(Sister's husband – broter-in-law) on the names given in the
D*B + C/A – D is nephew of A question.
(Sister's husband's son means Ÿ Draw a family tree where people
sister's son i.e., nephew)
of the same generation are placed
So, D is nephew to A.
at the same level and the entire
Shortcut Method :
diagram is in the form of a
By using symbols and generation
hierarchy.
relations :
Couple Sister
B C A Remember
(+) (–)
Ÿ Concentrate on points which give
w maximum definite information.
Son phe
Ne Ÿ Read the questions carefully and
D try identifying the persons
(+)
between whom relationship is to
So, it is clearly shown that D is be established. Possibly put
nephew to A. yourself in given character so that
q Shortcut Approach it becomes easy for you to
understand.
Ÿ The best way to solve blood
relation questions, you try and Ÿ Whilst concluding the relationship
relate every statement to between two people be careful
'yourself'. The starting name of about the gender of the person
the statement could be assumed being talked about as it is possible
as your name or you. to commit mistake by assuming the
Ÿ When the statement is very long, gender of the person which is not
it can get confusing. So, break given in the data or which can't be
down every statement in the extracted from the data/
question into sub statements and information given.
solve the question.

PRACTICE EXERCISE
DIRECTIONS (Qs. 1–2): Study the 1. What is the relation between ‘J and
following information carefully to A’ in the expression
answer the given questions : ‘A @ F $ M % J + T’ ?
(a) J is the mother-in-law of A
‘P + Q’ means ‘P is the sister of Q’.
(b) A is the aunt of J
‘P @ Q’ means ‘P is the wife of Q’.
‘P $ Q’ means ‘P is the son of Q’. (c) J is the sister-in-law of A
(d) A is the husband of J
‘P % Q’ means ‘P is the mother of Q’.
(e) None of these
EBD_7420
46 Blood Relations
2. What will come in the place of (c) Uncle
question-mark, if it is provided that (d) Cannot be determined
‘J is the daughter-in-law of T’ in the (e) None of these
expression ‘J % B ? K $ T’ ? 7. If 'A $ B' means 'A is father of B', 'A
(a) @ (b) % (c) + # B' means 'A is daughter of B', 'A
(d) $ (e) % or + @ B' means 'A is sister of B', then
3. ‘A + B’ means ‘A is the son of B’, how is K related to M in H @ K $ L
‘A – B’ means ‘A is the wife of B’. #M?
‘A × B’ means ‘A is the brother of (a) Husband
B’, ‘A ¸ B’ means ‘A is the mother (b) Uncle
of B’, ‘A = B’ means ‘A is the sister (c) Father
of B’. Which of the following (d) Cannot be determined
represents P is the maternal-uncle (e) None of these
of Q?
8. Chanda is the wife of Bharat.
(a) R × P ¸ Q (b) P × R ¸ Q
Mohan is the son of Chanda.
(c) P + R ¸ Q (d) P + R × Q
(e) None of these Ashish is the brother of Bharat and
father of Dhruv. How is Mohan
DIRECTIONS (Qs. 4-5): Study the related to Dhruv?
information given below and answer the (a) Sister (b) Cousin
questions following it: (c) Brother (d) Mother
Mohan is son of Arun’s father’s sister. 9. X is sister of Y. Y is brother of Z. Z
Prakash is son of Reva, who is mother of is husband of P. O is father of Y.
Vikash and grandmother of Arun. Pranab How is P related to O?
is father of Neela and grandfather of (a) Sister
Mohan. Reva is wife of Pranab. (b) Daughter
4. How is Mohan related to Reva ? (c) Uncle
(a) Grandson (d) Daughter - in - law
(b) Son 10. P and Q are brothers, P is the father
(c) Nephew of S, R is the only son of Q and is
(d) Data inadaequate married to U. How is U related to
(e) None of these S?
5. How is Vikash’s wife related to (a) Sister – in – law
Neela ? (b) Mother – in – law
(a) Sister (c) Sister
(b) Niece (d) Mother
(c) Sister-in-law
11. Pointing to a lady, Rohit said "She
(d) Data inadaequate
is the sister of the daughter of my
(e) None of these
6. Pointing to a photograph Shubha father's wife's son". How is the lady
said, "he is the only grandson of related to Rohit?
my mother's father". How is the man (a) Daughter
in photograph related to Shuhha? (b) Sister
(a) Cousin (c) Niece
(b) Brother (d) Daughter or Niece
Blood Relations 47

HINTS & SOLUTIONS


1. (c) A @ F Þ A is wife of F. 7. (a)
F $ M Þ F is son of M. H (–) K (+) M (–)
M % J Þ M is mother of J.
J + T Þ J is the sister of T.
M(–)
L (–)
A F J T So, K is husband of M
(–) (+) (–)
8. (b)
Spouse
Sister-in-Law Chanda Bharat = Ashish
2. (d) J % B Þ J is the mother of B
B $ K Þ B is the son of K.
K $ T Þ K is the son of T. Mohan Dhruv
Therefore J is daughter-in-law Hence, Mohan is cousin of Dhruv.
of T. 9. (d)
3. (b) Consider option (b), P × R ÷
Q, it means that P is the O Dau
ght
brother of R and R is the er-i
n-la
mother of Q. So, P is the Father w
maternal uncle of Q. Sister Brother Husband
Sol 4–5 X Y Z P
Pranab Û Reva So, P is daughter -in-law of O.
(+ ) ( -) 10. (a)
¯ ¯ ¯
Brother
Neela Prakash Vikash
¯ P Q
Mohan Arun (+) (+)
(+ )
4. (a) 5. (c)
6. (b) Father U
S R
Grandson (+) (–)
Mother
(–) Therefore, U is sister – in – law of S.
Shubha Man 11. (d)
(+)
Brother
EBD_7420
Chapter
Direction and
6 Distance
INTRODUCTION Direction Map
This part of reasoning comes under the North
category of common sense reasoning.
North-West North-East
In fact, this segment gauges the sense
(N-W) (N-E)
of direction of a candidate.

West East
CONCEPT OF DIRECTION
In our day to day life, we make our
concept of direction after seeing the
South-West South-East
position of sun. In fact, this is a truth
(S-W) South (S-E)
that sun rises in the East and goes down
in the West. Thus when we stand facing
sunrise, then our front is called East q Shortcut Approach
facing while our back is called West To remember four main
facing. At this position our left hand is directions, always remember the
word 'NEWS.'
in the Northward and the right hand is in
the Southward. Let us see the following Note: On paper North is always on top
direction map that will make your while South is always in bottom.
concept more clear.

CONCEPT OF DEGREE
Let us see the following picture:
360º 360º
0º 0º
315º
Anti clockwise (ACW)

45º 45º 315º


Clockwise (CW)

270º 90º 90º 270º

225º 135º 135º 225º


180º 180º
Direction and Distance 49
Remember Sol. Raman starts from his office A,
moves 2 km West upto B, then 4
• North, East, South and West are
km to the South upto C, 3 km East
called main directions. upto D and finally 1 km West upto
• North-east, South-east, South- E, Thus his distance from the initial
west and North-west are called position AE = BC = 4 km.
subdirections. B 2 km A
• Angle between two consecutive
main directions is always 90°. 4 km
• Angle between two consecutive 1 km D
subdirections is always 90°. C 2 km E
• Angle between a main direction
and its adjucent subdirection is Remember
always 45°. • If our face is towards North, than
after left turn our face will be towards
CONCEPT OF TURN West while after right turn, it will be
towards East.
Right turn = Clockwise turn • If our face is towards South, then
Left turn = Anticlockwise turn after left turn our face will be towards
Let us understand it through East and after right turn it will be
pictorial representation: towards West.
• If our face is towards East, then after
Right turn Left turn
left turn our face will be forwards
Right turn

North and after right turn it will be


Left turn
Right turn

Left turn

towards South.
• If our face is towards West, then
after left turn our face will be towards
Right turn Left turn
(ii) South and after right turn it will be
(i)
towards North.
• If our face is towards North-West,
then after left turn our face will be
towards South-West and after right
Right turn Left turn
turn it will be towards North-East.
• If our face is towards South-West,
then after left turn our face will be
towards South-East and after right
(iii) (iv)
turn it will be towards North-West.
• If our face is towards South-East,
EXAMPLE 1. Raman walked 2 km then after left turn our face will be
West from his office and then turned towards North-East and after right
South covering 4 km. Finally, he waked turn it will be towards South-West.
3 km towards East and again move 1 km • If our face is towards North-East,
West. How far is Raman from his initial then after left turn our face will be
towards North-West and after right-
position.
turn it will be towards South-East.
EBD_7420
50 Direction and Distance
q Shortcut Approach

Direction before Direction in which the person or vehicle


taking the turn will be moving after taking the turn

Right Left
(i) North East West
(ii) South West East
(iii) East South North
(iv) West North South
(v) North-West North-East South-West
(vi) South-West North-West South-East
(vii) South-East South-West North-East
(viii) North-East South-East North-West

CONCEPT OF SHORTEST/ moves 6 km Southwards upto C,


MINIMUM DISTANCE then turns towards East and walks
3 km upto D.
The shortest distance between two Then, AC = (AB – BC) = 10 – 6 = 4
points may be different from the total km
distance covered in going from initial CD = 3km.
position to final position.
To find the minimum distance between B
initial and last point, many times we have 6 km
to use ‘Pythogoras Theorem’. A 3 km
D
C
h2 = b2 + P2
where, 10 km
h P
h = Hypotenuse
b = Base
P = Perpendicular A
B b C
\ Rashmi’s distance from starting
EXAMPLE 2. Rashmi walks 10 km point A
towards North. She walks 6 km towards
South then. From here she moves 3 km = AD = AC 2 +CD 2 = 4 2 + 32
towards East. How far and in which
direction is she with reference to her = 16 + 9 = 25 = 5km.
starting point? From figure, D is to the North-East
Sol. It is clear, Rashmi moves 10 km
of A.
from A towards North upto B, then
Direction and Distance 51
SHADOW CASE Sol. The shadow of Rajesh was falling
exactly behind him. So, he was facing
In Morning / Sunrise Time towards East. Diagram clearly shows
that Rajesh was in North-East with
(a) If a person facing towards Sun, the reference to the starting point.
shadow will be towards his back
2m
or in West.
(b) If a person facing towards South,

int
po
the shadow will be towards his

g
right. 3m

tin
r
Sta
(c) If a person facing towards West,
the shadow will be towards his
front. 5m
(d) If a person facing towards North,
the shadow will be towards his left. q Shortcut Approach
In Evening / Sunset Time • Draw four lines from a point (let o)
repersenting four main directions.
(a) If a person facing towards Sun, the • Think that you are the point o.
shadow will be towards his back • Read the statement line by line.
or in East. • Move yourself as per statement
asked and prepare a diagram as
(b) If a person facing towards North,
per line by line statement.
the shadow will be towards his • Show, check and verify the
right. direction and distance moved
(c) If a person facing towards East, from starting point.
the shadow will be towards his • Find the direction and distance
of the final position.
front.
(e) If a person facing towards South, The questions of direction and
distance asked in var ious
the shadow will be towards his left. competitive exams are classified
into the following types.
Note : At 12:00 noon there is no
shadow because the rays of the sun 1. FINDING THE
are vertically downward. DIRECTION ONLY
(a) To determine the final direction
EXAMPLE 3. Early morning after when initial direction of movement
along with various turns are given.
sunrise, Rajesh was standing infront of
his house in such a way that his shadow EXAMPLE 4. A rat runs 20' towards
as falling exactly behind him. He starts east and turns to right, runs 10'
walking straight and walks 5 m. He and turns to right, runs 9' and
turns to his left and walks 3 m and again again turns to left, runs 5' and
turning to his left walks 2m. Now in then to left, runs 12' and finally
which direction is he from his starting turns to left and runs 6'. Now,
point? which direction is the rat facing?
EBD_7420
52 Direction and Distance
(a) East (b) West Clearly, Kunal is facing East while
(c) North (d) South coming out of his shop.
(e) None of these (c) When one direction gets changed
Sol. (c) The movements of rat are as by some other direction, then other
shown in figure. Clearly, it is directions also changed
accordingly. In this case we have
finally walking in the direction
to find the new direction of any one
FG i.e. North. (or more) of the remaining
20' B directions.
A
EXAMPLE 6. Rohit walked 25 metres
9' 10' G
D C towards South. Then he turned to
5' 6' his left and walked 20 metres. He
then turned to his left and walked
E 12' F 25 metres. He again turned to his
(b) To find the initial direction when right and walked 15 metres. At
various turns taken during a what distance is he from the
journey are given along with the starting point and in which
final direction. direction?
(a) 35 metres East
EXAMPLE 5. Kunal travels 10 m (b) 35 metres North
from his shop and turns to his left. (c) 40 metres East
After that he turns to right from (d) 60 metres East
the crossing. After moving certain (e) None of these
yards, he turns to left and again Sol. (a) The movements of Rohit are
turns to left after moving some as shown in figure.
distance. Finally, he turns to right. Rohit’s distance from starting
If at final position he is facing point A = AE
= (AD + DE) = (BC + DE)
North direction, then which direction
= (20 + 15) m = 35 m.
he was facing while coming out of Also, E is to the East of A.
his shop?
(Rohit) 15 m
(a) South (b) West A E
D
(c) East (d) North
Sol. (c) According to the question, the 25 m 25 m
direction diagram will be as follows
Fina l
pos ition B 20 m C
(d) To find the direction of a person
North Rig ht when direction of a shadow form of
N a person in an action is given.
Left
Left W E EXAMPLE 7. After 4 pm on a sunny
day when Ramesh was returning
R ig ht S from his school, he saw his uncle
coming in the opposite direction.
Lef t His uncle talked to him for some
Sh op
10 m time. Ramesh saw that the shadow
Direction and Distance 53
of his uncle was to his right side. and moving again upto 30 m to
Which direction was his uncle reach a temple. In which direction
facing during their talk? is the temple with respect to
(a) North Raman’s house?
(b) South (a) North-West (b) South
(c) East (c) South-East (d) West
(d) Data inadequate
Sol. (c) According to the question,
(e) None of these
the direction diagram is as
Sol. (b) After 4 pm the shadow will
be towards East. Now, East is follows
to the right of Ramesh. So N
Ramesh faces North. And his NW NE
uncle, who is opposite to him,
faces South. W E
(e) To Find Clock Based Direction
EXAMPLE 8. If the digit 12 of a clock SW SE
is pointing towards East, then in S
which direction will digit 9 point? 30 m
(a) South (b) West
(c) North (d) North-East Right Right
15 m
Sol. (c) According to the question,
House 30 m
the diagrams will be as
follows. So
uth
-Ea
N st
Temple
Clearly, temple is towards
W E South-East with respect to
Raman’s house.
S 2. FINDING THE
9
11
12
1 8 10
DISTANCE ONLY
2 7 11
10
9 3 6 12 EXAMPLE 10. One day, Ravi left
8 4 1
7 5
5
2
home and cycled 10 km
6 4
3 southwards, turned right and
Before turn After turn cycled 5 km and turned right and
Clearly, digit 9 will point towards North. cycled 10 km and turned left and
(f) To Find Relative Direction cycled 10 km. How many
kilometres will he have to cycle to
EXAMPLE 9. Raman starts from his
reach his home straight?
house and goes towards 15 m (a) 10 km
North, then he turns his right and (b) 15 km
walks 30 m before taking right turn (c) 20 km
EBD_7420
54 Direction and Distance
(d) 25 km direction is the multiplex hall from
(e) None of these the office of his father?
Sol. (b) Here, Ravi starts from home (a) 10 2 km, North-East
at A, moves 10 km (b) 20 km, North-East
southwards up to B, turns (c) Cannot be determined
right and moves 5 km up to C, (d) 10 km, South-East
turns right again and moves Sol. (a) According to the question,
10 km up to D and finally turns the direction diagram will be
left and moves 10 km up to E. as follows
N
10 km NW NE
D A
E (Ravi) W E
SW SE
S
10 km 10 km
School 10 km East Multiplex
S M
C B 10
5 km m
2
km k

2
Thus, his distance from initial 10 km 10
South N-W
position A = AE N-E
= AD + DE
H
= BC + DE = (5 + 10) km = 15 10 km House
km.
As in a square, diagonals are equal
3. FINDING BOTH THE
\ SH = OM = 10 2 km
DISTANCE AND DIRECTION \ Required distance, OM
= Distance between office
EXAMPLE 11. Sanjay’s school is
and multiplex = 10 2 km
10 2 km in North-West direction Also, if is clear from diagram that
from his house. The office of his multiplex (M) is to the North-East
father is 10 km in South direction of O (office)
from the school. The multiplex hall
is 10 km towards East from the So, the multiplex is 10 2 km North-
school. How far and in which East from office.
Direction and Distance 55

PRACTICE EXERCISE
1. Ashok started walking towards DIRECTIONS (Qs. 4-5): Study the
South. After walking 50 metres he following information to answer the
took a right turn and walked 30 given questions :
metres. He then took a right turn
Point P is 5 m towards the South of Point
and walked 100 metres. He again
took a right turn and walked 30 M. Point Q is 3 m towards the East of
metres and stopped. How far and Point P. Point O is 3 m towards the East
in which direction was he from the of Point M. Point N is 2 m towards the
South of Point Q.
starting point?
4. A person, facing North, takes a left
(a) 50 metres South
turn from point M, walks 4m and
(b) 150 metres North
stops. He then takes another left
(c) 180 metres East turn, walks 5 m and stops at point
(d) 50 metres North R. Which of the following points,
(e) None of these including R, fall in a straight line?
2. Town D is towards East of town F. (a) M, O, R (b) N, R, P
Town B is towards North of town (c) R, O, Q (d) R, Q, N
D. Town H is towards South of (e) Q, P, R
town B. Towards which direction 5. How far and towards which
is town H from town F ? direction is Point O from Point N?
(a) East (a) 5 m towards South
(b) South-East (b) 7 m towards North
(c) North-East (c) 8 m towards West
(d) Data inadequate (d) 7 m towards West
(e) None of these (e) 5 m towards North
3. Mohan walked 30 metres towards 6. Vikas walked 10 metres towards
South, took a left turn and walked North, took a left turn and walked
15 metres. He then took a right turn 15 metres and again took a left turn
and walked 20 metres. He again took and walked 10 metres and stopped
a right turn and walked 15 metres. walking. Towards which direction
How far is he from the starting was he facing when he stopped
point? walking?
(a) 95 metres (a) South
(b) 50 metres (b) South-West
(c) 70 metres (c) South-East
(d) cannot be determined (d) Cannot be determined
(e) None of these (e) None of these
EBD_7420
56 Direction and Distance
7. While facing East, I turn to my left south direction respectively and
and walk 10 m, then turn to my left walked for 5 meter. Now P and Q
and walk 10 m. Now, I turn 45 took a right turn and walked 10m
degrees towards my right in North- each. Now P and Q took left turn
West direction and cover 25 m. At and after walking 5 meter both of
this point, in which direction am I them stopped. Find the distance
from my starting point? between them
(a) South-East (b) South-West (a) 15 (b) 25
(c) North-East (d) North-West (c) 30 (d) 35
(e) None of these (e) None of these
8. Vijay started walking towards 10. Mohan walked 30 metres towards
South. After walking 15m, he South, took a left turn and walked
turned to the left and walked 15 m. 15 metres. He then took a right turn
He again turned to his left and and walked 20 metres. He again
walked 15 m. How far is he from his took a right turn and walked 15
original position and in which metres. How far is he from the
direction? starting point?
(a) 15 m, North (b) 15 m, South (a) 95 metres
(c) 30 m, East (d) 15 m, West (b) 50 metres
(e) None of these (c) 70 metres
9. Two Person P and Q are separated (d) Cannot be determined
by a distance of 20 meter in west- (e) None of these
east direction respectively. Now P
and Q start walking in north and

HINTS & SOLUTIONS


1. (d) 2. (d)
B
D 30 m E
N

W E
100 m A
S F D H
50 m

C 30 m B
H
Required distance = (BE – AB) = So there is no sufficient data
(100 – 50) m = 50 m to determine the direction of
Direction Þ North Town H.
Direction and Distance 57
3. (b) As per conditions given 7. (d)
25
m
N 45° 10 m
N
A
W E W E
10 m
30 m S
S Start
15 m
B C So, the person is in North-West
direction from starting point
20 m 8. (e) Follow the given movements,
Starting Final
E D point point
15 m

Required distance = AE = AB
+ BE = (30 + 20) m = 50 m 15 m 15 m
4. (e) towards
south
4m 3m
left 15 m
Hence, he is 15 m to the East
5m from the starting point.
9. (e)
5M
3m
P 10M
2m 5M
20M
N
Points R, P and Q are in a
straight line. 5M
5. (b) Points O is 7 metres towards
North of Point N. 10M
6. (a) 5M
\ Required distance = 20 meter.
15 m 10. (b)
N Starting Point
10 m

10 m

W E 30m

S 15m
Starting Piont 20m
Ending Point
15m
EBD_7420
58 Time Sequence, Number, Ranking Test & Position Test

Chapter Time Sequence, Number,

7 Ranking Test & Position Test


TIME SEQUENCE In a leap year, February has 29 days.
In time sequence, we have to detect exact • A leap year has 52 weeks and 2 days.
time/a particular day/a particular day on Therefore, a leap year has 2 odd days.
the basis of several statements provided Ordinary year
in the question. • An ordinary year has 12 months.
To solve problems related to time • An ordinary year has 365 days.
sequence, let us gather first the • An ordinary year has 52 weeks
following informations : and 1 day. Therefore, an ordinary
1 Minute = 60 seconds year has 1 odd day.
1 Hour = 60 minutes Century (100 years)
1 Day = 24 hours • A century has 76 ordinary years
1 Week = 7 days and 24 leap years.
1 Month = 4 weeks • A century has 5 odd days.
1 Year = 12 months
Odd days
Odd days in an ordinary year = 1
1 Ordinary year = 365 days
Odd days in a leap year = 2
1 Leap year =366 days
Odd days in 100 years = 5
1 Century = 100 years
Odd days in 200 years = (5 × 2)
Remember = 1 week + 3 days = 3
• A day is the period of the earth’s Odd days in 300 years = (5 × 3)
revolution on its axis. = 2 weeks + 1 day = 1
• A ‘Solar year’ is the time taken the Odd days in 400 years = (5 × 4 + 1)
earth to travel round the sun. It is = 21 days
equal to 365 days, 5 hours, 48 = 3 weeks + 0 day = 0
1 Similarly, each 800, 1600, 2000, 2004, etc.
minutes and 47 seconds nearly.. has 0 odd days.
2
EXAMPLE 1. Neena returned home
• A ‘Lunar month’ is the time taken
after 3 days earlier than the time she
by the moon to travel round the
had told her mother. Neena’s sister
earth. It is equal to nearly 28 days.
Veena reached five days later than the
Leap Year day Neena was supposed to return. If
• If the number of a given year is Neena returned on Thursday, on what
divisible by 4, it is a leap year. day did Veena return ?
Hence, the years like 1996, 2008, Sol. Neena returned home on Thursday.
2012 are leap years. But years like Neena was supposed to return 3
1997, 1991, 2005, 2007 are not days later, i.e., on Sunday.
divisible by 4 and therefore, such Veena returned five days later from
years are not leap years. Sunday. i.e., on Friday.
Time Sequence, Number, Ranking Test & Position Test 59
NUMBER TEST EXAMPLE 3. N is more intelligent
In such test, generally you are given a
long series of numbers. The candidate than M. M is not as intelligent as Y. X is
is required to find out how many times a more intelligent than Y but not as good
number satifying the conditions as N. Who is the most intelligent of all?
specified in the question occurs. (a) N (b) M
(c) X (d) Y
EXAMPLE 2. How many 8s are there Sol. (a) According to the question,
in the following number sequence which N>M
are immediately preceded by 5 but not Y> M
immediately followed by 3? N>X>Y
38584583988588893 On arranging the above data,
Sol. Let use see the following : we get
38 584 5 8 3 9 88 5 8 Q N > X>Y>M
8 8 93
So, N is the most intelligent
Clearly, two such 8s are there.
among them.
Remember EXAMPLE 4. Shailendra is shorter
There is no rule as how to attempt these than Keshav but taller than Rakesh,
questions but we can practice these Madhav is the tallest. Aashish is a little
questions : shorter than Keshav and little taller than
Left < > Right Shailendra. If they stand in the order of
A B C increasing heights, who will be the
second?
(a) Aashish (b) Shailendra
A is preceding B C is the following B
(c) Rakesh (d) Madhav
RANKING TEST Sol. (b) According to the question,
Ranking involves determining the K>S>R
sequencial order of one or more
persons/objects based on comparison é K = Keshav ù
of parameters such as height, weight, êS = Shailendra ú
ê R = Rakesh ú
age, length, position, merits etc. ê M = Madhav ú
For comparison, generally êë A = Aashish úû
following symbols are used:
(i) A > B (means A is greater/heavier / Madhav is the tallest.
taller / higher /more than B) K>A> S
(ii) A < B (means A is smaller / lighter / On arranging the above data,
shorter / lower / less than B) we get
Questions based on ranking are \ M> K >A> S > R
generally given with a set of or
information in jumbled form based R < S < A< K < M
on which the candidates are
¯ ¯ ¯ ¯ ¯
required to systematically arrange
the given information and determine 1 2 3 4 5
the sequencial order of arrangement Hence, Shailendra will be the
of the various persons/objects. second.
EBD_7420
60 Time Sequence, Number, Ranking Test & Position Test
POSITION TEST EXAMPLE 6. In a vertical column of
In this type of problems, the position
of a person from either of the two 40 students, A is 13th from the top end,
ends of a row (or column) is given find the rank from bottom end.
and it is asked to determine number Sol. Total = 40
of persons to the left / right (or above
/ below) of a partienlas person or total
number of persons in the group etc.
Sometimes, such questions are A
given in the form of a puzzle 13L
involving interchanging of seats A's rank from bottom
by two or more persons. = Total + 1 – top
Position of a Person from Left/
= 40 – 13 + 1
Right/Top/Bottom
= 27 + 1
q Shortcut Approach = 28
Formulas to determine the position of New Position of a Person(s) after
a person in a horizontal row Interchanging of Positions bet-
(1) Left + Right = Total + 1 ween Two Persons
(2) Left = Total + 1 – Right In this type of questions, the
(3) Right = Total + 1 – left position s of two per sons ar e
(4) Total = left + Right – 1 interchanged and it is asked to
determine their new positions (from
EXAMPLE 5.
left or right)
Shortcut Approach
(i) New position of the first person
from left after the interchanged
1 2 3 4 5
é Difference of two ù
ê position of second ú é Initial position ù
| =ê ú + êof first person ú
3rd from left ê person from ú ê ú
ê ú ëêfrom left ûú
3rd from right ë right û
Total = 3 + 3 – 1
(ii) New position of the second person
q Shortcut Approach from right after the interchanged
Formula to determine the position of
é Difference of two ù é Initial position ù
a person in a vertical column ê ú ê ú
(1) Total + 1 = Top + Bottom = ê positions of first ú + êof second person ú
êë person from left úû êëfrom right úû
(2) Top = Total + 1 – Bottom
(3) Botom = Total + 1 – Top
EXAMPLE 7. In a row of girls Ankita
(4) Total = Top + Bottom – 1
is sixth from the left and Rashami is
Note : The above formulas are only for nineth from the right. When they
a single person's position exchange their positions, then Ankita
Time Sequence, Number, Ranking Test & Position Test 61
becomes eighteenth from left. What will number of places between these
be Rashami new position from the right? two positions is given, then
Shortcut Sol. (i) Maximum number of persons in the
Rashami new position from the row =
right
= (18 – 6) + 9 éSum of position ù é Number of places ù
êof both persons ú + ê in the middle ú
= 21 = 21st position ë û ë û
Number of Persons between
the Original and New Position Middle
places
of a Person
(ii) Minimum number of persons in the
Shortcut Approach:
Total number of persons between row
two positions = (New position) –
= éêëof both persons
Sum of positions ù - é Number of places ù - 2
(original position) – 1. úû êëin the middle úû

EXAMPLE 8. In a row of boys, Kartik


is 5th from left and Ganesh is 15th from Middle
place
right. If the position of Kartik and
Ganesh are interchanged then new
position of Kartik 20th from left. Find EXAMPLE 9. In a row, Sakashi is 10th
the number of boys between Kartik and from left and Sonia is 15th from right
Ganesh. and there are 4 girls between Sakashi
Shortcut Sol. and Sonia, then find the maximum and
Total number of boys between two minimum number of girls in the row.
position of Kartik
Shortcut Sol.
= 20 – 5 – 1 = 14
Maximum number of girls = (10 + 15)
Maximum and Minimum Number + 4 = 29
of Persons in a Row Minimum number of girls
Shortcut Approach: = (10 + 15) – 4 – 2 = 19.
If position of two persons from the
two opposite ends and the total

PRACTICE EXERCISE
1. Pratap correctly remembers that his is definitely their mother's birthday ?
mother's birthday is before twenty (a) Twentieth
third April but after nine teenth
(b) Twenty-first
April, whereas his sister correctly
remembers that their mother's (c) Twentieth or twenty-first
birthday is not on or after twenty (d) Cannot be determined
second April. On which day in April (e) None of these
EBD_7420
62 Time Sequence, Number, Ranking Test & Position Test
2. Among five friends, P, Q, R, S and right. If they interchange their
T, each scored different marks in
positions, Srinath becomes 22nd
the examination. P scored more
than Q but less than R. S scored from the left. How many boys are
more than' only T. Who amongst there in the row ?
the following scored the second (a) 19 (b) 31
highest marks?
(a) P (b) Q (c) 33 (d) 34
(c) R (d) S 7. After 9'O clock at what time
(e) T between 9 p.m. and 10 p.m. will the
3. In a class of 25 students. Lata’s rank hour and minute hands, of a clock
is 13th from the top and Parul’s rank
is 19th from the bottom. If Vishal’s point in opposite direction?
rank is exactly between Lata’s and (a) 15 min. past 9
Parul’s rank what is Vishal’s rank (b) 16 min. past 9
from the top ?
(a) 10th (b) 8th 4
(c) 9th (d) 7th (c) 16 min. past 9
11
(e) Cannot be determined
4. Among P, Q, R, S, T and U. R is
1
taller than only P and U. S is shorter (d) 17 min. past 9
11
than only T and Q. If each of them
8. If John celebrated his victory day
has a different height, who among
on Tuesday, 5th January 1965,
them will be the third from top when
when will he celebrate his next
they are arranged in descending
victory day on the same day?
order of their height ?
(a) 5th January 1970
(a) R (b) P
(c) S (d) Q (b) 5th January 1971
(e) None of these (c) 5th January 1973
(d) 5th January 1974
5. Anish's mother remember that her
9. If the 5 th date of a month is
wedding day after 10th February
Tuesday, what date will be 3 days
2010,but before 20 th February
after the 3rd Friday in the month?
2010.According to her brother after (a) 17 (b) 22
13th February 2010 but before 17th (c) 19 (d) 18
February 2010. Anish wedding day 10. In a row of students, if John , who
on which date? (Note: DATE is 16th from the left, and Johnson,
Should be odd number) who is 8th from the right,
(a) February 12 (b) February 13 interchange their positions, John
(c) February 14 (d) February 15 becomes 33rd from left. How many
students are there in a row?
(e) February 16
(a) 38 (b) 39
6. In a row of boys, Srinath is 7th from (c) 40 (d) 41
the left and Venkat is 12th from the
Time Sequence, Number, Ranking Test & Position Test 63

HINTS & SOLUTIONS


1. (c) According to Pratap his have to gain 45 – 30 = 15 minute
mother’s birthday may be on space over the hour hand.
20th, 21st or 22nd April. Q Gain of 55 minute spaces
According to Pratap’s sister equal 60 minutes.
their mother’s birthday may be \ Gain of 15 minute spaces
from 1st April to 21st April. equals
Common Dates Þ 20th and
21st 60 180 4
= ´ 15 = = 16
2. (a) R > P > Q > S > T 55 11 11
P scored the second highest Therefore, hour and minute
marks. hands of a clock point in
6 12
opposite direction after 9
3. (a) P II V II L
4
Vishal’s rank from the top is 10th. O’clock at 16 minutes
11
4. (c) R > P, U
past 9.
T, Q > S
8. (b) 5 January 1965 Þ Tuesday
T, Q > S > R > P, U
5 January 1966 Þ Wednesday
5. (d) Mother: 11 12 13 14 15 16 17 18
19 5 January 1967 Þ Thursday
5 January 1968 Þ Friday
Brother: 14 15 16
5 January 1969 Þ Sunday
15 is coming in common and
also a odd number.Hence Since, 1968 is a leap year.
Anish's wedding day is on 5 January 1970 Þ Monday
February 15. 5 January 1971 Þ Tuesday
6. (c) 9. (d) 5th date of a month is
Tuesday.
7th 22nd Friday will be on = 5 + 3
S V = 8th of a month
1st Friday is on 1st of a month
2nd Friday is on 8th of a month
12th 3rd Friday will be on 15th of a
Total number of boys in the month
row 3 days after 15th = 15 + 3 = 18
10. (c) When John interchange his
= 22 + 12 – 1 = 33 position, new
7. (c) At 9 O’clock, the minute hand
is 9 × 5 = 45 minute – spaces Position will be 33 from left
behind the hour hand. and 8 from Right
Therefore, the minute hand will Total = 33 + 8 – 1 = 40
EBD_7420
64 Logical Sequence of Words

Chapter Logical Sequence


8 of Words
INTRODUCTION Sol. Member ® Family ®
Community® Locality®
In this particular type of problems, Country
certain inter-related words are given and
(iii) Sequence in Ascending or
numbered, followed by various
Descending order
sequences of the numbers denoting
them, as alternatives. EXAMPLE 1. Furniture
2. Forest
TYPES OF SEQUENCE 3. Wood
4. Country
(i) Sequence of occurence of
events or various stages in a 5. Trees
process. Sol. Country ® Forest ® Trees®
Wood ® Furniture.
EXAMPLE 1. Consultation (iv) Sequential order of words
2. Illness According to Dictionary
3. Doctor
4. Treatment EXAMPLE 1. Direct
5. Recovery 2. Divide
Sol. Clearly illness occurs first. One 3. Divest
then goes to a doctor and after 4. Devine
consultation with him, 5. Divisons
undergoes treatment to finally Sol. Devine ® Direct ® Divest ®
attain recovery. Divide ® Divisons.
(ii) Sequence of objects in a q Shortcut Approach
class or group
• Remember all English alphabets
EXAMPLE 1. Member in forward and reverse order
2. Country • Knowledge of our nature or
3. Community surroundings
4. Family
5. Locality
Logical Sequence of Words 65

PRACTICE EXERCISE
1. Arrange the following words 6. Arrange the following words in
according to the dictionary ? their ascending order:
1. Inventory 2. Involuntary 1. Millenium
3. Invisible 4. Invariable 2. Diamond Jubilee
5. Investigate 3. Silver Jubilee
(a) 4, 2, 5, 3, 1 (b) 4, 5, 1, 3, 2 4. Centenary
(c) 2, 5, 4, 1, 3 (d) 4, 1, 5, 3, 2 5. Golden Jubilee
2. Which one of the given responses (a) 2, 3, 5, 4, 1 (b) 2, 5, 3, 1, 4
would be a meaningful order of the (c) 3, 5, 2, 4, 1 (d) 2, 3, 5, 1, 4
following words ? 7. Arrange the following words as
A. Family B. Community per order in the dictionary.
C. Member D. Locality (i) Forge (ii) Forget
E. Country (iii) Forgo (iv) Forgive
(a) C, A, D, B, E (b) C, A, B, D, E (v) Format
(c) C, A, B, E, D (d) C, A, D, E, B (a) (v), (ii), (iv) , (iii), (i)
3. Which one of the given responses (b) (i), (iv), (iii), (ii), (v)
would be a meaningful order of the (c) (iii), (iv), (v), (ii), (i)
following? (d) (i), (ii), (iv), (iii), (v)
1. Sentence 2. Word 8. Arrange the following words as per
3. Chapter 4. Phrase order in the dictionary.
5. Paragraph 1. Obscure 2. Objective
(a) 4, 3, 1, 2, 5 (b) 2, 3, 5, 4, 1 3. Objection 4. Obligation
(c) 3, 5, 1, 4, 2 (d) 1, 3, 2, 4, 5 5. Oblivion
4. Arrange the following words (a) 3, 2, 4, 5, 1 (b) 3, 2, 5, 4, 1
according to dictionary order: (c) 3, 2, 5, 1, 4 (d) 5, 2, 1, 3, 4
1. Banquet 2. Bangle 9. Arrange the following words as per
3. Bandage 4. Bantam order in the dictionary :
5. Bank 1. Command 2. Commit
(a) 3, 2, 4, 5, 1 (b) 3, 5, 2, 1, 4 3. Connect 4. Conceive
(c) 3, 2, 1, 5, 4 (d) 3, 2, 5, 1, 4 5. Conduct 6. Commerce
5. Arrange the given words in a (a) 6 2 1 5 4 3 (b) 6 1 2 4 5 3
meaningful order: (c) 1 6 2 4 5 3 (d) 1 2 6 5 3 4
1. INFANT 10. Which one of the given responses
2. ADOLESCENT would be meaningful order of the
3. CHILD following in ascending order?
4. OLD 1. Phrase 2. Alphabet
5. ADULT 3. Sentence 4. Word
(a) 3, 1, 2, 4, 5 (b) 1, 3, 2, 5, 4 (a) 2, 1, 4, 3 (b) 1, 2, 3, 4
(c) 3, 2, 4, 5, 1 (d) 5, 4, 3, 2, 1 (c) 2, 4, 1, 3 (d) 2, 4, 3, 1
EBD_7420
66 Logical Sequence of Words

HINTS & SOLUTIONS


1. (d) Arrangement of words 2. ADOLESCENT
according to the Dictionary : ¯
(4) Invariable 5. ADULT
¯ ¯
(1) Inventory 4. OLD
¯ 6. (c) Silver jublee - 25 yr.
(5) Investigate Golden jublee - 50 yr.
¯ Diamond jublee - 75 yr.
(3) Invisible Centenary - 100 yr
¯ Millennium - 1000 yr.
(2) Involuntary 7. (d) Arrangement of words as per
2. (b) Meaningful order of the dictionary:
words (i) Forge
C. Member ® A. Family ® B. ¯
Community ® (ii) Forget
D. Locality ® E. Country ¯
3. (c) Meaningful order of the (iv) Forgive
words : ¯
3. Chapter (iii) Forgo
¯ ¯
5. Paragraph (iv) Format
¯ 8. (a) The correct order is:
1. Sentence Objection > Objective >
¯ Obligation > Oblivion >
4. Phrase Obscure
¯ 9. (c) Dictionary order :
2. Word Command ® Commerce
4. (d) According to dictionary, order ® Commit
is : Bandage, Bangle, Bank, (1) (6) (2)
Banquet, Bantam. ® Conceive ® Conduct
5. (b) Meaningful order of words : ® Connect
1. INFANT (4) (5) (3)
¯ 10. (c) Alphabet ® Word ®
3. CHILD (2) (4)
¯ Phrase ® Sentence
(1) (3)
Chapter
Number Puzzles
9
INTRODUCTION
82 25
In this particular type of problems,
questions are based on different number. 80 67
This type of problem having figure which 99 ?
follows a particular rule for their different
number. We have then asked to find a
103
missing number by using same rule. 97
TYPES OF NUMBER PUZZLE (ii) (iii)
PATTERN 1 : SINGLE FIGURE Here, a series of figure is given. Checking
PATTERN the pattern in the first two figures, we
4 have to find missing number in the third.
416 10 If we observe the first two figure
685 33 properly, we get an idea of the pattern.
? As, 110 + 30 – 75 = 65, 97 + 82 – 80 = 99
Here, a clockwise pattern is being So, 103 + 25 – 67 = 61.
followed. If we move clockwise we can
see that numbers are increasing. If we q Shortcut Approach
observe it more closely, we can crack the • The first step is to observe the
pattern which is figure and check if there is any
As, 4 × 2 + 2 = 10, 10 × 3 + 3 = 33 familiar pattern in the given
So, 33 × 4 + 4 = 136 question.
PATTERN 2 : MULTIPLE FIGURE
PATTERN • The second step is finding out the
pattern.
110
75 • There is no need to memorize any
65 pattern.
• You need to understand the
30 concept and decode the pattern.
(i)
EBD_7420
68 Number Puzzles

PRACTICE EXERCISE
1. Find the missing number from the (a) 17 (b) 23
given responses: (c) 47 (d) 73
5 6 12 5. 3 9 2 8 4 7
4 3 4 7 4 ?
2 3 ? 81 5 64 6 49 5
18 27 96 (a) 1 (b) 8
(c) 6 (d) 16
(a) 4 (b) 5
6. Select the missing number from the
(c) 3 (d) 6 given responses.
2. Find the missing number from the
given response: 81 729 64 512 49 ?
3 6 2

4 27 5 5 37 2 5 ? 2

2 3 9 9 8 7
(a) 37 (b) 45 (a) 444 (b) 515
(c) 47 (d) 57 (c) 343 (d) 373
3. Select the missing number from the
given responses: DIRECTIONS (Qs. 7 - 8) : Select the
missing number from the given
responses.

100 28 52 28
7. 59 ?
25 12 36 102
(a) 50 (b) 90
(c) 218 (d) 64
(a) 132 (b) 122 8.
(c) 222 (d) 212
3 5 2
DIRECTIONS (Qs. 4 - 5) : In questions
4 1 5 7 5 7 3 ? 5
below, select the missing number from
the given responses 5 4 1
4. (a) 11 (b) 3
7 9 8 (c) 1 (d) 5
2 4 3 9. In the following questions, select the
missing number from the given
5 7 6 responses.
16 32 ?
Number Puzzles 69
the sign of question mark (?) from
14 12 9 11 the given alternatives.
36 54 4 8 12
7 3 16 ?
38 56 ?
(a) 12 (b) 17 9 6 11 9 11 13
(c) 18 (d) 16 (a) 72 (b) 78
10. In the following question, select (c) 108 (d) 90
the number which can be placed at

HINTS & SOLUTIONS

1. (d) 5 + 4 = 9 and 9 × 2 = 18 4. (b) 7 + 22 + 5 = 16


6 + 3 = 9 and 9 × 3 = 27 9 + 42 + 7 = 32
12 + 4 = 16 and ? 8 + 32 + 6 = 23
96 5. (c) 3 + 9 – 5 = 7, 2 + 8 – 6 = 4
= = 6
16 4+ 7–5= 6
2. (c) First Figure 6. (c) 81 × 9 = 729, 64 × 8 = 512
3 + 5 + 2 + 4 = 14
Þ 14 + 13 = 27 49 × 7 = 343
Second Figure 7. (b) (100 + 12) – (28 + 25) = 59
6 + 2 + 3 + 5 = 16 Similarly,
Þ 16 + 21 = 37 (102 + 52) – (36 + 28) = 90
Third Figure 8. (d) (4 + 5) – (3 + 5) = 1
2 + 2 + 9 + 5 = 18 (7 + 7) – (5 + 4) = 5
Þ 18 + 29 = 47 (3 + 5) – (2 + 1) = 5
3. (c) Moving clockwise, the terms
are : 9. (c) (36) – (14 + 12 + 7) = 3
5 × 2 + 2 = 12 (54 ) – (9 + 11 + 16) = 18
12 × 2 + 2 = 26 10. (a) As, (4 + 6 + 9) × 2 = 38
26 × 2 + 2 = 54 (8 + 9 + 11) × 2 = 56
54 × 2 + 2 = 110 Similarly,
So, missing number = 110 × 2 + 2 (12 + 13 + 11) × 2 = 72
= 222
EBD_7420
70 Venn Diagram

Chapter

10 Venn Diagram
INTRODUCTION EXAMPLE
Venn diagrams are pictorial way of
represent the set of article. There are
different regions which needs proper 2
understanding for solving problems 1
3
based on given Venn diagrams.
TYPES OF VENN DIAGRAM – represents student passed
in English
– represents student passed
in Reasoning.
Analysis Based Identification of Relation
Venn Diagram Based Venn Diagram
1 – represents student passed in
English only
(i) Analysis Based Venn 2 – represents student passed in
Reasoning only
Diagram-
3 – represents student passed in
In this type, generally a venn both English Reasoning both.
diagram comprising of different
EXAMPLE 1.
geometrical figures is given. Each
geometrical figure in the diagram Mothers
represents a certain class.
2
q Shortcut Approach
Sisters 4 5
Case - I: 7
1
Two articles: 6 3
Q Teachers
P
The diagram consists of three
groups sisters, mothers and
IA IIAB IB teachers, represented by a triangle,
a rectangle and a circle respectively.
There are seven regions represented
by numbers from 1 to 7.
Here, IA represents only P Region-1: represents only sisters
IB represents only Q
Region-2: represents only mothers
IIAB represents P and Q
Region-3: represents only teachers
Venn Diagram 71
Region-4: represents those sisters 4 ® Doctor who is farmer also
who are mothers also but not teacher 5 ® Engineer who is doctor also
Region-5: represents those mothers
6 ® Engineer who is farmer also
who are teachers also but not sister.
Region-6: represents those teachers 7 ® Person who is Engineer,
who are sisters also but not mothers doctor and farmer.
Region-7: represents those teachers
(ii) Identification of
who are sisters as well as mothers.
Relation Based Venn
q Shortcut Approach Diagram -
Case: - II Three articles In this type, some standard
P Q representations for groups of
three items with different cases
1 2 of venn diagrams are given.
5

7 q Shortcut Approach
6 4
When one class of items is completely
3 included in the another class of item
then it is represented by the given
R diagram
1 – represents P only
2 – represents Q only II
3 – represents R only
4 – represents Q and R (not P)
I
5 – represents P and Q (not R)
6 – represents P and R (not Q)
7 – represents P, Q and R

EXAMPLE 2. EXAMPLE 3.
Engineer Doctor I – Mango
II – Fruit
2 Here, all mango are fruit.
1 5
q Shortcut Approach
7
6 4 If two classes of item are completely
different from each other but they all are
3 completely included in third class then the
relationship is represent of the diagram.
Farmer
III
1 ® Engineer
2 ® Doctor I II
3 ® Farmer
EBD_7420
72 Venn Diagram
EXAMPLE 4. EXAMPLE 6.

I – represent potato Cricketer, player and farmer


II – represent onion I – Cricketer
II – Player
III – represent vegetable
III – Farmer
q Shortcut Approach All cricketers are players but
farmers not.
• If two group of items having some
common relationship and both of q Shortcut Approach
them are all included in third If three group of things are related to
class then the relationship is each other
represented by the diagram.

III I II

I II

III
EXAMPLE 5. Brother, Father, Male.
I ® Brother EXAMPLE 7.
II ® Father Graduate, Engineer and Doctor
Graduate may be Engineer and
III ® Male Doctor.
Some Brother may be Father and
q Shortcut Approach
all are male.
When two group of items are
q Shortcut Approach completely unrelated to each other
When one class of item is completely while they are partly related with third
group of item
included in another group while third
is not related to both of them then such
condition are diagrammatically
represented by I II III
II

I EXAMPLE 8. Cloth, Red, Flowers.


Some cloth are Red and also some
Flowers are red.

III
Venn Diagram 73
q Shortcut Approach EXAMPLE 9.
When group of items are completely
different from each other Red, Yellow, Black
These are all different colour.

I II

III

PRACTICE EXERCISE

1. Find out which of the diagrams


given in the alternatives correctly
represents the relationship stated (a) (b)
in the question. Sharks, Whales,
Turtles

(a)
(c) (d)

(b) 3. Which number space indicated


Indian teachers who are also
advocates?
Indians Advocates
(c) 1 2 7
3
4 6
5

(d) Teachers
(a) 2 (b) 3
(c) 4 (d) 6
2. Which one of the following 4. In the following figure, how many
diagrams represents the correct educated people are employed ?
relationship among 'Judge', 'Thief'
and 'Criminal'?
EBD_7420
74 Venn Diagram

Educated 1 2 7
people
8 3
4 5
6 6
11 3
5 7 (a) 2 (b) 3
17
(c) 4 (d) 1
Employed
Backward people
people 7. The diagram represents Teachers,
Singers and Players. Study the
Employed people diagram and find out how many
teachers are also singers.
Backward people
Players 10 Teachers
Educated people 12 5
4
(a) 18 (b) 20 17 9 Singers
(c) 15 (d) 9
28
5. In the given diagram, Circle
represents strong men, Square (a) 4 (b) 5
represents short men and Triangle (c) 9 (d) 13
represents military officers. Which 8. Identify the diagram that best
region represents military officers represents the relationship among
who are short but not strong? classes given below :
Food, Curd, Spoons

2 (a)

3 1
(b)
4

(c)

(a) 2 (b) 3
(d)
(c) 4 (d) 1
6. In the given figure, circles 9. In the given figure, which letter
represent students studying three represents carnivorous plants
different subjects. How many which are not green?
students study all the three
subjects?
a d b Plant
Carnivorous g
e f
c Green
Venn Diagram 75
(a) d (b) g (a) 168 (b) 111
(c) e (d) f (c) 125 (d) 108
10. In the given figure, How many
water are either tap or shower?
Carnivorous
31
Tap Water
43
57 Shower
68 49

HINTS & SOLUTIONS


1. (c) Sharks belong to class 3. (b) The number ‘3’ space
pisces. Whale is a mammal represents Indian teachers
and Turtle belongs to class who are also advocates as
reptiles. this number is common to
given condition.
Sharks Whale 4. (d) 3 + 6 = 9
5. (a)

2 Short
Military 3 1
2. (c) Judge is different from both 4
the thief and criminal.
The thief comes under the Strong
class criminal. 6. (b) The number ‘3’ is common to
all the three circles.
7. (c)
Judge Numbers
Persons
4 5 9 10 12 17 28
Players
Teachers
Criminal Singers

Thief Number of teachers who are also


singers = 9
EBD_7420
76 Venn Diagram
8. (d) 10. (b) According to figure,
Food Total number of water are
Curd either tap or shower = (43 +
68) = 111.

9. (a) According to figure,


'd' letter represents
carnivorous plants which are
not green.
Mathematical Operation Arithmetical Reasoning 77

Chapter
Mathematical Operation
11 Arithmetical Reasoning

INTRODUCTION EXAMPLE 1. If ‘+’ stands for division,


In this type of problem, usually ‘×’ stands for addition, ‘–’ stands for
mathematical symbol are converted into multiplication, and ‘¸’ stands for
another form by either interchanging the subtraction, then which of the following
symbol or using different symbol in equation is correct?
place of usual symbol and then calculate (a) 36 × 6 + 7 ¸ 2 – 6 = 20
the equation according to the given (b) 36 + 6 – 3 × 5 ¸ 3 = 24
(c) 36 ¸ 6 + 3 × 5 – 3 = 45
condition.
(d) 36 – 6 + 3 × 5 ¸ 3 = 74
Remember Sol. (d) 36 × 6 ¸ 3 + 5 – 3
Þ 36 × 2 + 5 – 3 = 74
While simplifying a mathematical
problem follow 'VBODMAS' rule (ii) Interchange of Signs &
Numbers
V - Viniculum bracket
In this, the given equation
B - Bracket
becomes correct and fully balanced
O - Of
when either two signs of the equation or
D - Division both the numbers and the signs of the
M - Multiplication equations are interchanged.
A - Addition EXAMPLE 2. Given interchange : sign
S - Subtraction ‘+’ and ‘–’and numbers 5 and 8. Which
of the following is correct?
TYPES OF MATHEMATICAL (a) 82 – 35 + 55 = 2
OPERATION (b) 82 – 35 + 55 = 102
(c) 85 – 38 + 85 = 132
(i) Symbol Substitution
(d) 52 – 35 + 55 = 72
In this, various mathematical Sol. (a) 52 + 38 – 88 = 2
symbols, followed by a question (iii) Balancing the Equation
involving calculation of an expression.
In this, the signs given in one of
It is required to put the real signs in the
the alternatives are required to fill up the
given equation and then solve the
blank spaces for the signs in order to
question.
balance the given equation.
EBD_7420
78 Mathematical Operation Arithmetical Reasoning
EXAMPLE 3. Select correct Sol. Here, required sum = (80 – 3 x 3)
combination of mathematical sign to years = (80 – 9) years
replace ‘*’ sign to balance the equation. = 71 years.
9 * 4 * 22 * 14
(a) × = – q Shortcut Approach
(b) × – = • If ages of n persons in a group are
(c) = – × x1, x2, x3, ... , xn yrs, then total of
(d) – × = their ages befor t years
Sol. (b) 9 * 4 * 22 * 14
9 × 4 – 22 = 14 = x1 + x 2 + x 3 + ... + x n - nt
• If ages of n persons in a group are
q Shortcut Approach
x1 , x 2 ,..., x n yrs; then total of
• Begin with replacin g coded
operators with their meanings. their ages after t years
Write the entire expressions with = x1 + x2 + .. + xn + nt
correct operators and operand.
• When sowing always remember To F ind the Resulta nt
VBODMAS. Number in a Row
• If any interchnages are suggested, In this type of questions, two rows
apply then before you start soling. of numbers are given along with
certain rules. On the basis of these
ARITHMETICAL REASONING rules, you have to find out resultant
number in each row separately and
Arithmetical Reasoning tests the ability question below the row is to be
to solve basic arithmetic problems answered.
encountered in everyday life. These Directions (Q. Nos. 5-9) In each of the
problems require basic mathematical following questions two rows of
skills like addition, subtraction, numbers are given. The resultant
multiplication, division etc. The tests number in each row is to be
include operations with whole numbers, worked out separately based on
rational numbers, average, ratio and the following rules and the
proportion, interest, percentage, and questions below the rows of
measurement. Arithmetical reasoning is numbers are to be answered. The
one factor that helps characterize operations of numbers progress
mathematics comprehension, and it also from left to right.
assesses logical thinking. Rules
EXAMPLE 4 : The total of the ages of (i) If an odd number is followed by
Amar, Akbar and Anthony is 80 years. another composite odd number,
they are to be multiplied.
What was the total of their ages three
(ii) If an even number is followed by
years ago?
an odd number, they are to be
added.
Mathematical Operation Arithmetical Reasoning 79
(iii) If an even number is followed by a 9. 14 5 19
number which is a perfect square, 24 w 88
the even number is to be subtracted
from the perfect square. If ‘w’ is the resultant of the first
(iv) If an odd number is followed by a row, what is the resultant of the
prime odd number, the first number second row?
is to be divided by the second (a) 171 (b) 283
number.
(c) 195 (d) 107
(v) If an odd number is followed by an
even number, the second one is to (e) None of these
be subtracted from the first one. Sol. (5 to 9)
5. 58 17 5 Rules
85 5 n
(i) (Odd number ) × (Composite odd
If ‘n’ is the resultant of the first row,
number)
what is the resultant of the second
row? (ii) (Even number ) + (Odd number)
(a) 255 (b) 32 (iii) (Even number ) ® (Perfect square
(c) 49 (d) 34 number), then (Perfect square
(e) None of these number) – (Even number)
6. 24 64 15 (iv) (Odd number ) ¸ (Prime odd
m 11 15 number)
If ‘m’ is the resultant of the first (v) (Odd number) – (Even number)
row, what is the resultant of the
5. (a) 1st Row 58 17 5 Þ 58 + 17 = 75
second row?
(a) 165 (b) 75 [rule (ii)]
(c) 20 (d) 3 75 ¸ 5 = 15 = n [rule (iv)]
(e) None of these 2nd Row 85 5 n Þ 85 5 15
7. 7 21 3 85 ¸ 5 = 17 [rule (iv)]
Þ
d 7 33
If ‘d’ is the resultant of the first row, Þ 17 × 15 = 255 [rule (i)]
what will be the resultant of the \ Resultant of second row = 255
second row? 6. (b) 1st Row 24 64 15
(a) 40 (b) 138
Þ 64 – 24 = 40 [rule (iii)]
(c) 231 (d) 80
(e) None of the above Þ 40 + 15 = 55 = m [rule (ii)]
8. 73 34 13 2nd Row m 11 15 Þ 55 11 15
32 p 15 55 ¸ 11 = 5 [rule (iv)]
If ‘p’ is the resultant of the first row, Þ 5 × 15 = 75 [rule (i)]
what is the resultant of the second \ Resultant of second row = 75
row? 7. (c) 1st Row 7 21 3 Þ 7 × 21 = 147
(a) 713 (b) 50 [rule (i)]
(c) 20 (d) 525 147 ¸ 3 = 49 = d [rule (iv)]
Þ
(e) None of these
EBD_7420
80 Mathematical Operation Arithmetical Reasoning
2nd Row d 7 33 Þ 49 7 33 EXAMPLE 10. If 2 + 6 + 9 = 926, 1 +
Þ 49 ¸ 7 = 7 [rule (iv)]
8 + 2 = 218, then 4+3+1 =?
Þ 7 × 33 = 231 [rule (i)] Sol.
\ Resultant of second row = 231 1 2 3 3 1 2
8. (d) 1st Row 73 34 13 Þ 73–34 = 39 2 + 6 +9 9 2 6
[rule (v)]
Þ 39 ¸ 13 = 3 = p [rule (iv)]
1 2 3 3 1 2
2nd Row 32 p 15 Þ 32 3 15
1 + 8 +2 2 1 8
Þ 32 + 3 = 35 [rule (ii)]
Þ 35 × 15 = 525 [rule (i)]
\ Resultant of second row = 525 Similarly,
9. (d) 1st Row 14 5 9 Þ 14 + 5 = 19 1 2 3 3 1 2
[rule (ii)] 4+3+1 1 4 3
19 × 9 = 171 = w [rule (i)] \ ? = 143
2nd Row 24 w 88 Þ 24 171 88
EXAMPLE 11. If 73 + 82 = 14, 19 + 21
Þ 24 + 171 = 195 [rule (ii)]
= 11, then 86 + 24 = ?
Þ 195 – 88 = 107 [rule (v)]
Sol. 73 + 82 = 14
\ Resultant of Second row = 107
Þ (7 – 3) + (8 + 2) = 14
Trick Based Mathematical
Þ 14 = 14
Operations And 91 + 21 = 11
The questions are based on simple Þ (9 – 1) + (2 + 1) = 11
mathematical operations that do
Þ 11 = 11
not come under any of the above
Similarly , 86 + 24 = (8 – 6) + (2 + 4)
given types covered. These
=2+ 6=8
questions can be based on several
different patterns.

PRACTICE EXERCISE

1. After interchanging ÷ and +, 12 and 2. If '–' stands for '¸' '+' stands for '×' ,
18, which one of the following '¸' for '–' and '×' for '+', which one of
equations becomes correct? the following equations in correct?
(a) (90 × 18) + 18 = 60 (a) 30 – 6 + 5 × 4 ÷ 2 = 27
(b) (18 + 6) ÷ 12 = 2 (b) 30 + 6 – 5 ÷ 4 × 2 = 30
(c) (72 ÷ 18) × 18 = 72 (c) 30 × 6 ÷ 5 – 4 + 2 = 32
(d) (12 + 6) × 18 = 36 (d) 30 ÷ 6 × 5 + 4 – 2 = 40
Mathematical Operation Arithmetical Reasoning 81
3. Some equations have been solved 7. If ‘+’ means ‘¸’, ‘×’means ‘+’, ‘–’
on the basis of a certain system. means ‘×’ and ‘¸’ means ‘–’, then
Find the correct answer for the which of the following equations
unsolved equation on that basis. If is correct?
9 * 7 = 32, 13 * 7 = 120, 17 * 9 = 208, (a) 36 + 6 – 3 × 2 = 20
then 19 * 11 = ? (b) 36 × 6 + 3 – 2 < 20
(a) 150 (b) 180 (c) 36 × 6 + 3 × 2 > 20
(c) 210 (d) 240 (d) 36 + 6 × 3 + 2 = 20
4. If L denotes × M denotes ¸ ; P 8. If × stands for addition, < for
denotes + ; Q denotes – then 16 subtraction, + stands for division,
P 24 M 8 Q 6 M 2 L 3 = ? > stands for multiplication, – stands
(a) 10 (b) 9 for equal, ¸ stands for greater than,
(c) 12 (d) 11 and = stands for less than, state
5. Class A has students twice that of which of the following is true?
class B. After adding 20 students (a) 3 × 2 < 4 ¸ 16 > 2 + 4
to class A and 30 students to class (b) 5 > 8 + 4 = 10 < 4 × 8
B, the total number of students in (c) 3 × 4 > 2 – 9 + 3 < 3
both the classes is 140. What is the (d) 5 × 3 < 3 ¸ 8 + 4 × 1
number of students in class A in 9. Select the correct combination of
the beginning? mathematical signs to replace *
(a) 30 (b) 60 signs and to balance the given
(c) 80 (d) 140 equation.
6. At present, the ratio between the 15 * 24 * 3 * 6 * 17
ages of Arun and Deepak is 4 : 3. (a) + × = ¸
After 6 years, Arun's age will be 26 (b) – × = +
years. What is the age of Deepak (c) – ¸ + =
at present? (d) + ¸ – =
(a) 15 years 10. If + = ×, – = ¸, × = +, ¸ = –, then
(b) 19 years which is the correct equation out
(c) 24 years of the following?
(d) 12 years (a) 18 ¸ 6 + 4 – 2 ¸ 3 = 22
(b) 18 + 6 – 4 × 2 ÷ 3 = 26
(c) 18 × 6 – 4 + 7 × 8 = 47
(d) 18 – 6 × 7 ¸ 2 + 8 = 63

HINTS & SOLUTIONS


1. (d) (12 + 6) × 18 = 36 Þ 25 + 4 – 2 Þ 27 = 27, option
Þ (18 ¸ 6) × 12 = 36 (a) is correct
30 + 6 – 5 ¸ 4 × 2 = 30
Þ 3 × 12 = 36
Þ 30 × 6 ¸ 5 – 4 + 2 = 30
2. (a) 30 – 6 + 5 × 4 ¸ 2 = 27 Þ 36 – 4 + 2 ¹ 30, option (b) is
Þ 30 ¸ 6 × 5 + 4 – 2 = 27 wrong
EBD_7420
82 Mathematical Operation Arithmetical Reasoning
30 × 6 ¸ 5 – 4 + 2 = 32 Deepak is 3x years.
Þ 30 + 6 – 5 ¸ 4 × 2 ¹ 32, 6 years hence,
option (c) is wrong Arun’s age = 4x + 6 = 26
Þ 30 ¸ 6 × 5 + 4 – 2 = 40 Þ 4x = 26 – 6
Þ 30 – 6 + 5 × 4 ¸ 2 ¹ 40 20
option (d) is wrong. x= =5
4
3. (d) 9 + 7 = 16; 9 – 7 = 2 \ Present age of Deepak = 3x =
16 × 2 = 32 15 years
13 + 7 = 20; 13 – 7 = 6 7. (a) By checking options
20 × 6 = 120 36 ¸ 6 × 3 + 2 = 6 × 3 + 2 Þ 20
17 + 9 = 26; 17 – 9 = 8 = 20
26 × 8 = 208 8. (*)
19 + 11 = 30; 19 – 11 = 8
´ Þ + <Þ - + Þ ¸ >Þ ´
30 × 8 = 240 - Þ= ¸ Þ> =Þ<
4. (a)
Option (a)
3 × 2 < 4 ÷ 16 > 2 + 4
L Þ × M Þ ¸
Þ 3 + 2 – 4 > 16 × 2 ÷ 4
P Þ + Q Þ – 16 ´ 2
Þ5–4> Þ1>8
4
16 P 24 M 8 Q 6 M 2 L 3 = ?
(not possible)
Þ ? = 16 + 24 ¸ 8 – 6 ¸ 2 × 3 Option (b)
Þ ? = 16 + 3 – 3 × 3 5 > 8 + 4 = 10 < 4 × 8
Þ ? = 16 + 3 – 9 = 10 Þ 5 × 8 ÷ 4 < 10 – 4 + 8
Þ 5 × 2 < 18 – 4 Þ 10 < 14
5. (b) Suppose, in the beginning the
Option (c)
number of students in Class
3×4>2–9+3<3
B= x
Þ3+4×2=9÷3–3
Therefore, the number of
Students in Þ3+8 ¹ 3–3
Class A = 2x Option (d)
Now, 5×3<3¸8+ 4×1
2x + 20 + x + 30 = 140 Þ5+3–3>8¸4+1
Þ 8 – 3 > 2 +1
Þ 3x = 140 – 50
Þ5>3
90 Both options (b) and (d) are
\x = = 30
3 correct.
Number of Students in Class A 9. (d) 15 * 24 * 3 * 6 * 17
= 2x = 2 × 30 = 60 Þ 15 + 24 ¸ 3 – 6 = 17
6. (a) Suppose the present age of Þ 15 + 8 – 6 = 17
Arun is 4x years and that of 10. (b) 18 × 6 ¸ 4 + 2 – 3 = 18 × 1.5 + 2
– 3 = 27 + 2 – 3 = 26
Inequalities 83

Chapter
Inequalities
12
INEQUALITIES (i) One thing > another thing.
or
As we know,
(ii) One thing < another thing.
3×3=9
where ‘>’ denotes ‘greater than’.
Now, we can say that the result of
and ‘<’ denotes ‘less than’
multiplication between 3 and 3 is equal
Hence, you can write,
to 9. Therefore, 3 × 3 = 9 is a case of
3×4>9
equality. But when we multiply 3 × 4, we
4×1<9
get 12 as a result of this multiplication. It
( 3 × 4 > 9) means ‘Product of 3 and 4 is
does mean that
greater than 9’.
3× 4¹9
(4 × 1 < 9) means ‘Product of 4 and 1 is
As 3 × 4, is not equal to 9, it is a case of
less than 9’.
inequality.
Sometimes we come across two numbers
When, we come to know that one thing
where, we do not know the exact state of
is not equal to another; there can be only inequality between them.
two possibilities:-
Let us see :
(i) One thing is greater than another
m ³ n means m is either greater than or
thing.
equal to n.
or
m £ n means n is either less or equal to m.
(ii) One thing is less than the another
Hence, we can summarise the signs to
thing.
be used in inequalities as below:
When, we denote (i) and (ii) mathematically,
then we will write.

Important Signs
Signs Meaning Example Explanation
= Equal to A=B A is equal to B.
> Greater than A>B A is greater than B.
< Less than A<B A is less than B.
³ Greater than or equal to A³B A is greater than or equal to B.
£ less than or equal to A£B A is less than or equal to B.
EBD_7420
84 Inequalities
In inequalities different sets of elements EXAMPLE 2. 17 < 19, and 19 < 20 can
are given using the inequalities symbols. be easily combined as 17 < 19 < 20.
The candidate is required to analyse the
Here,
given statements and then decide which
of the relations given an alternatives 17 < 19 < 20
follows from those given in the statements.
But before solving the problems of Common term
inequalities, first you have to learn more
Clearly, 17 < 19 and 19 < 20 have
about the inequalities
common term 19 and this common term
CHAIN OF INEQUALITIES is greater than 17 and less than 20.
Hence, 17 < 19 and 19 < 20 have been
Sometimes two or more inequalities are
combined together to create a single combined into 17 < 19 < 20 as per the
inequality having three or more terms. Such conditions I and II.
combination is called chain of inequalities. Now, let us see some examples of
i n equ a li ti es wh i ch ca n n ot be
Conditions for Combining
combined. Some such examples are
Two Inequalities
given below:
Condition I: Two inequalities will be i. 14 > 12, 19 > 18
combined if and only if
ii. 18 < 20, 22 < 25
they have a common
term. iii. 100 > 99, 80 > 77
Condition II: Two inequalities will be iv. 100 < 115, 118 < 119
combined if and only if Clearly, (i), (ii), (iii) and (iv) can not be
the common term is combined as they do not have any
greater than (or ‘greater’ common term and therefore, they do not
than or equal to’) one and follow condition I and condition II.
less than (or ‘less than or How to Derive Conclusions
equal to’) the other. from a Combined
EXAMPLE 1: 14 > 13, 13 > 12 can be Inequalities?
easily combined as ‘14 > 13 > 12’. To derive conclusion from a combined
Here, inequality, you have to eliminate the
common term.
14 > 13 > 12
For example,
Common term (a) If we have
Clearly, 14 > 13 and 13 > 12 have common m>l>n
term 13 and this common term is greater then, our conclusion is
than 12 and less than 14. Hence, 14 > 13 m>n
and 13 > 12 have been combined into
14 > 13 > 12 as per the conditions I and II. (b) When, we have
m<l<n
Inequalities 85
then, our conclusion is m = l ³ n, which gives the conclusion

m<n m ³ n — (B)

(c) When, we have ‘³’ signs in the Combining (A) and (B), we have the final
combined inequalities then you conclusion as
have to think a little bit more. Let m ³ n
us consider the combined
inequality given below: From (a), (b), (c), (d) and (e), we get a
rule for deriving conclusions from a
m³l>n combined inequality, we may say it
Here, m is either greater than l or ‘Golden Rule’.
equal to l.
Hence, the minimum value for m is
GOLDEN RULE
equal to l. But l is always greater The conclusion inequality
than n. Therefore, m is always will have an '³' sign or a '£'
greater than n. sign if and only if both the
signs in the combined
\ Our conclusion is m>n inequality are '³' or '£' sign
respectively
(d) When, we have the following
In all other cases, there will
inequalities:-
be a ' >' or a '< ' sign in the
m> l³n conclusion.
In this case, m is always greater Clearly, in (a), (b), (c) and (d) only one
than l and l is either greater than inequality and (e) (m ³ l ³ n) has ‘³’ as
n or equal to it. When l is greater its both the sign.
than n; m will obviously be greater
than n. Even when l is equal to n; Remember
m will be greater than n as m is • If m > n, then n < m must be true.
always greater than l. • If m < n, then n > m must be true.
\ Our conclusion is m>n
• If m ³ n, then n £ m must be true.
• If m £ n, then n ³ m must be true.
(e) When, we have combine inequality
EITHER CHOICE RULES
m³l³n
Here, m is either greater than l or When your derived conclusion is of the
equal to l. type m ³ n (or m £ n) then check both the
conclusions
When m is greater than l; we have m > n an d m = n (or, m < n and
m > l ³ n, which gives the conclusion. m = n).
I. If both the conclusions are true,
m>n — (A) then choice “either follows” is true.
When m is equal to l; we have II. If none of the conclusions seems
correct. Then check wheather the
EBD_7420
86 Inequalities
conclusions form a complementary (a) > (b) <
pair. The conclusions form a (c) £ (d) =
complementary pair in the 4 cases (e) Either < or £
given below: Sol. (d) On putting sign (=) in place
(i) m ³ n and m < n
of question mark (?)
(ii) m > n and m £ n
(iii) m £ n and m > n I> J³K=L£N=M
(iv) m < n and m ³ n Þ means I > L and M ³ K
If the conclusions form a complementry EXAMPLE 4 :
pair then the choice “either follows” is
Statement : H= W£R>F
correct. Conclusion : I.R = H
II.R > H Either Or
DIRECT INEQUALITY Statement : H> L= E < T
In this type of questions, direct relation Conclusion : I.H £ T
II.H > T Either Or
between two or more than two elements
are given in a meaningful inequality. Statement : S< T³R³M
Conclusion : I.M < T
Candidates are required to establish the II.M = T Either Or
relation between elements with the help Statement : I ³H=T>S£R
of used signs between the elements. Conclusion: I.I > T
Either Or
Shortcut Approach: II.I = T
(B) NEITHER NOR :
DEFINITE CONCLUSION If 2 conclusions are wrong and variables
(i) A > B = C Þ a> B in the two conclusions are different
(ii) A ³ B = C Þ A³C then write it as 'Neither Nor'
(iii) A ³ B > C Þ A>C EXAMPLE 5 :
(iv) A < B > C Þ A<C Statement : P > Q ³ S = R
(v) A £ B = C Þ A£C Conclusion : I. P ³ R
II. R > Q Neither nor
(vi) A £ B < C Þ A<C Statement : L = T £ J ³ K
(vii) A < B = C £ D Þ A<D Conclusion : I. L > K
Neither nor
(viii) A > B = C ³ D Þ A>D II. T £ K
Statement : V < L ³ J £ T
INDEFINITE CONCLUSION Conclusion : I. V < J Neither nor
(i) a > b < c Þ No conclusion II. L = T
(ii) a > b £ c Þ No conclusion Statement : G £ K £ F < M
Conclusion: I. G > F
(iii) a ³ b £ c Þ No conclusion II. K £ M Neither nor
(iv) a ³ b < c Þ No conclusion
CODED INEQUALITY
EXAMPLE 3 : Which of the following
symbols should replace the question In the coded inequalities, all the
mark in the given expression in order to signs of inequalities (>, <, =, ³, £)
make the expressions. ‘I > L’ as well as are in coded form i.e. substituted
‘M ³ K’ definitely true? symbols are used in place of real
I>J³K?L£N=M symbols. The candidates are
Inequalities 87
required to replace the codes (like New Step IV: If neither of the
@ / © / * / $ / # etc.) with real signs conclusions has been
and then solve the questions in the proved correct till now,
same way as the questions of then check ‘Either choice
inequalities are solved. Rule II’.
EXAMPLE 6: In the following question,
q Shortcut Approach
the symbols ©, @, =,* and $ are
Steps for Solving Problems used with the following meanings :
Step I: Decode the given symbols like P © Q means ‘P is greater than Q’;
@, $, d, #, *, etc. P @ Q means ‘P is greater than or
Step II: Take one conclusion at a time equal to Q’;
and make an idea that which P = Q means ‘P is equal to Q’;
statements are relevant for P * Q means ‘P is smaller than Q’;
evaluating it. P $ Q means ‘P is either smaller
Step III: Use conditions I and II and the than or equal to Q ’.
‘Golden Rule’ to combine the Now in the following question, assuming
relevant statements and derive that the given statements are true. Find
a conclusion from it. which of the two conclusions I and II
After performing the above mentioned given below them is/are definitely true.
three steps, if a conclusion is established Give answer :
and verified, it is well and good. But if (a) if only conclusion I is true;
does not happen so, then you have to (b) if only conclusion II is true;
perform 4 more new steps given below: (c) if either I or II is true;
New Step I: Check if the given (d) if neither I nor II is true.
conclusion directly (e) if both I and II are true.
follows from anyone Statements : P © T, M $ K, T = K
single statement. Conclusions : I. T © M
New Step II: Check if the conclusion
II. T = M
– inequality you get is
essentially as same as the Sol. (c) Given statements :
given conclusion but P > T, M £ K, T = K.
written differently. T = K, K ³ M Þ T ³ M
New Step III: Check if the derived Þ T > M or T = M
conclusion follows Þ T © M or T = M
‘Either choice Rule I’. So, either I or II is true.
EBD_7420
88 Inequalities

PRACTICE EXERCISE
DIRECTIONS (Qs. 1-4): In the following (a) Only I is true
questions, the symbols @, $, «, # and d (b) Only III is true
are used with the following meaning as (c) Only IV is true
illustrated below: (d) Only either III or IV is true
(e) Only either III or IV and I are
'P $ Q' means 'P is not smaller than Q'. true
'P @ Q' means 'P is neither smaller than 4. Statements :
nor equal to Q'. R « D, D $ J, J # M, M @ K
'P # Q' means 'P is neither greater than Conclusions :
nor equal to Q'.
'P d Q' means P is neither greater than I. K # J
nor smaller than Q'. II. D @ M
'P « Q' means 'P is not greater than Q'. III. R # M
Now in each of the following questions IV. D @ K
assuming the given statements to be (a) None is true
true, find which of the four conclusions (b) Only I is true
I, II, Ill and IV given below them is/are (c) Only II is true
definitely true and give your answer (d) Only III is true
accordingly. (e) Only IV is true
1. Statements : 5. Which one of the following
H @ T, T # F, F d E, E « v symbols should be placed in the
Conclusions : I. V $ F blank spaces(from left to right) in
II. E @ T order to complete the given
III. H @ V expression in such a manner that
IV. T # V both N > L and G > K definitely
(a) Only I, II and III are true
(b) Only I, II and IV are true true?
(c) Only II, III and IV are true N _G_ P_ L_ K
(d) Only I, III and IV are true (a) £, =, >, < (b) ³, £, =, <
(e) All I, II, III and IV are true (c) >, =, ³, ³ (d) <, =, £ , ³
2. Statements : (e) None of these
D # R, R« K, K @ F, F $ J 6. Which one of the following will be
Conclusions : I. J # R definitely true if the expression ‘Q
II. J # K < S >V = W ³ O ³ R ’ is definitely
III. R # F true?
IV. K @ D (a) S ³ O (b) O ³ Q
(a) Only I, II and III are true (c) W < Q (d) R £ V
(b) Only II, III and IV are true (e) None of these
(c) Only I, III and IV are true 7. In Which of the following
(d) All I, II, III and IV are true expressions does the expression
(e) None of these 'L< T' to definitely hold true?
3. Statements : (a) K > L > R = P < S £ T
N d B, B $ W, W # H, H « M (b) U ³ T ³ M = F £ A ³ L
Conclusions : I. M @ W
(c) L ³ C > Q ³ B = N £ T
II. H @ N
III. W d N (d) G ³ L = A < B £ S £ T
IV. W # N (e) T ³ E = G ³ W = Y ³ L
Inequalities 89
DIRECTIONS (Qs. 8-10): In the given 9. Statement: J ³ A > D = E; L < A < M
Conclusions:
questions, assuming the given
I. M<J
statements to be true. Find which of II. J > L
the given two conclusions numbered I, (a) Only II is true.
II is/are definitely true and give your (b) Either I or II are true.
answer accordingly. (c) Both I and II are true
(d) Only I is true.
8. Statement: M > U > L £ N; L ³ Y > A (e) None is true.
Conclusions: 10. Statement: M £ K > L = Y; P £ T > M
I. Y< N Conclusions:
II. Y = N I. P >Y
(a) Both I and II are true II. T < L
(b) Only II (a) Only II is true
(b) Only I
(c) Only I is true (c) Either I or II are true
(d) Either I or II is true. (d) Both I and II are true
(e) None is true. (e) None is true

HINTS & SOLUTIONS


1. (b) H@ T Þ H> T Therefore, N = B ³ W < H £ M
T#F ÞT<F Conclusions
F dE Þ F = E I. M @W Þ M > W : True
E V ÞE £V II. H @ N Þ H > N : Not True
Therefore, H > T < F = E £ V III. W d N Þ W = N : Not True
Conclusions IV. W # N Þ W < N : Not
I. V $ F Þ V ³ F : True True
II. E @ T Þ E > T: True
W is either smaller than or
III. H @ V Þ H > V : Not True
equal to N. Therefore either
IV. T # V Þ T < V : True
III or IV and I are true.
So conclusion I , II & IV follow
2. (e) D# R ÞD<R 4. (a) R D Þ R £ D, D $ J Þ D ³ J
R K ÞR£K J # M Þ J < M, M @ K Þ M > K
K@ F Þ K>F Therefore, R £ D ³ J < M > K
F$J ÞF³J Conclusions
Therefore, D < R £ K > F ³ J I. K # J Þ K < J : Not True
Conclusions II. D @ M Þ D > M : Not True
I. J # R Þ J < R : Not True III. R # M Þ R < M : Not True
II. J # K Þ J < K : True
III. R # F Þ R < F : Not True IV. D @ K Þ D > K : Not True
IV. K @ D Þ K > D : True So none of the conclusion
So only conclusion II & IV follows
follow 5. (c) 6. (d) 7. (d)
3. (e) N dB Þ N = B Sol. (8-10) :
B$W Þ B³W 8. (d) Either I. Y < N or II. Y = N
W # H ÞW< H 9. (a) J > L is true.
H MÞ H £ M 10. (e) None is true.
EBD_7420
90 Problem Solving (Puzzles)

Chapter
Problem Solving
13 (PUZZLES)
INTRODUCTION TYPES OF PROBLEMS
In this chapter you will see some typical 1. Simple problems (based on
problems in which you would be given a categorisation)
series of interlinked information and on 2. Problems based on arrangement
the basis of those informations you (Linear, circular, rectangular/
would be expected to reach certain square)
conclusions. 3. Problems based on comparison
4. Problems based on blood
TYPES OF INFORMATIONS relations
IN A GIVEN PROBLEM 5. Blood relations and profession
based problems
1. Basic Informations
6. Problems based on conditional
(Useful secondary informations): It selection
is given in first couple of sentences
of given data which are such that 1. Simple Problems Based on
they give you some basic Categorisation
information that is essential to give Tips to Solve Problems
you general idea of the situation.
These type of problems can easily be
2. Actual Informations solved by constructing a table.
Whatever remains after the basic
informations are known as actual EXAMPLE 1. Directions : Read the
information. following information carefully
While trying to solve a problem and answer the question that
one should begin with actual follows:
in formation keepin g useful 1. There are six cities L, M, N, O, P
secondary information in mind. and Q.
3. Negative Informations 2. L is not a hill station.
3. M and P are not historical places.
Actual informations having
4. O is not an industrial city.
negative sentences are called
negative information. A negative 5. L and O are not historical cities.
information does not inform us 6. L and M are not alike.
anything exactly but it gives a Q. Which two cities are historical
chance to eliminate a possibility. places ?
Problem Solving (Puzzles) 91
Sol. It can be solved by preparing a table in the manner given below:

L M N O P Q
Historical
place
Industrial
city
Hill station

(2), (3), (4), (5) are negative informations. Therefore as per such informations.
We put ‘x’ (not) mark wherever applicable. As a result the table looks like the
one below.

L M N O P Q
Historical
× × × ×
place
Industrial
×
city
Hill station ×

As above table gives definite informations about L and O. L is neither a


historical place nor a hill station. So, it must be an industrial city. In the same
manner O is neither a historical nor an industrial city. So, O must be a hill
station. Hence, we put ‘P’ mark at the appropriate place which give the table
following look:-

L M N O P Q
Historical × × × ×
place
Industrial P ×
city
Hill station × P

Now, as per the condition (6), L and M are not alike, hence M can not be an
Industrial city. Also M is not a historical place. Therefore, it is very obvious
that M is a hill station.
Again, in the given problem there is no negative information about N. Hence,
we can assume that N is a hill station as well as a historical place and an
industrial city. In the same way, we find the conclusion about other cities.
N, P and Q. Finally we find the following table.
EBD_7420
92 Problem Solving (Puzzles)

L M N O P Q
Historical × × P × × P
place
Industrial P × P × P P
city
Hill × P P P P P
station

Now, after analysing the given question we get the answer


'N and Q are two historical places.'
2. Problems Based On Arrangement
In such problems a group of people, objects, etc, may have to be arranged in
a row, or in a circle or any other way.
LINEAR ARRANGEMENT
One Row Sequence
(A) When direction of face is not clear, then we take ourself as base and then the
diagram will be as follows

Face Face Face Face Face

Left Right
A B C D E

Middle
From the above diagram, it is clear that
(i) B, C, D, E are right of A but only B is the immediate right of A.
(ii) D, C, B, A are left of E but only D is the immediate left of E.
(B) When direction of face is towards you, then the diagram will be as follows
A B C D E
Right Left

Face Face Face Face Face


From the above diagram, it is clear that
(i) B is immediate left of A, C is immediate left of B; D is immediate left of
C and E is immediate left of D.
(ii) D is immediate right of E; C is immediate right of D; B is immediate
right of C; and A is immediate right of B.
Problem Solving (Puzzles) 93
TWO ROWS SEQUENCE and select the possibilities
Let us see 6 persons seating in two rows which does not violate any
P Q R condition.
Right Left
EXAMPLE 2. Directions : Just read
the following information
carefully to answer the questions
Left Right given below it:
S T U
From the above diagram, it is clear that Five friends P, Q, R, S and T are
(i) P is sitting opposite S. sitting on a bench.
(ii) Q is sitting opposite T. (1) P is sitting next to Q.
(iii) R is sitting opposite U. (2) R is sitting next to S.
(iv) P and U are sitting at diagonally (3) S is not sitting with T.
opposite positions. (4) T is on the last end of the bench.
(v) S and R are sitting diagonally (5) R is on the 2nd position from the
opposite positions. right.
In arrangement problems, the actual (6) P is on the right of Q and T.
information can be classified into 2 (7) P and R are sitting together.
categories:- Q. At what position is P sitting?
(a) Definite information Sol.
A definite information is one when Her e, 4th and 5th sentences
the place of object/man is constitute definite information:
definitely mentioned. Comparative informations are: 1st,
2nd, 6th and 7th sentences while 3rd
(b) Comparative information
is a negative information.
In such information the place of Now, start with definite information,
object/man is not mentioned sketch the following arrangement:-
definitely but only a comparative T __ __ R __
position is given. In other words Now, this is the time to look for the
the positions of objects/men are comparative informations that tell
given in comparision to another
about T and R. Such informations
objects/men.
are 2nd, 6th and 7th sentences. Take
q Shortcut Approach the 7th and the 1st sentence. If P
Step I. Sketch a diagram of and R are together and also Q and
empty places P are together, then P must be
Step II. Fill up as many empty between Q and R. Now the
places as possible using all arrangement take the form as:-
the definite informations. T Q P R ____
Step III. With the help of By the virtue of the 2nd sentence:
comparative information TQP RS
consider all possibilities So, P is sitting at the middle
position.
EBD_7420
94 Problem Solving (Puzzles)
Directions (Qs. 3-5) : The following questions consist of two words having a
certain relationship to each other. Select the alternative whose words are having
the same relationship amongst them.
Q. 3 Stare: Glance
(a) Gulp: Sip (b) Confide: Tell
(c) Hunt: Stalk (d) Step: Walk
Sol. (a) Both are synonyms of each other.
Q. 4. Cloth: Texture
(a) Body: Weight (b) Silk:Cloth
(c) Wood: Grains (d) Ornaments: Gold
Sol. (a) As 'Cloth' has 'Texture' in the same way 'Body' has 'Weight'.
Q. 5. Nuts: Bolts
(a) Nitty: Gritty (b) Bare: Feet
(c) Naked: Clothes (d) Hard: Soft
Sol. (c) As 'Nuts' are covered with 'bolts', in the same way a 'Naked' is covered with
'Clothes'.
CIRCULAR ARRANGEMENT
Circle is the most important case from the exam point of view. Most of the times
Circle kind of statements are there in exams.
From the exam point of view, in most cases they give 8 persons sitting in the circle.
But before solving the important thing is their ' Sitting Position '.
Step 1. Knowing NEWS! N= North , E= East , W=West , S= South
N

W E

S
Step 2 : Picking Left & Right :
Facing Center Facing Outside
Clock wise = Left Clock wise = Right

N N
NW NE NW NE

W E W E

SW SE SW SE
S S
Problem Solving (Puzzles) 95
If it is mention in the statement that all is
EXAMPLE 6. Directions : Study the
facing outside then just do opposite of
above like this: following information carefully and
Clock wise = Right answer the question given below.
Step 3 : Solving step wise the statement Bunty, Dev, Manav, Kavya, Payal,
or Following the statement. Qasturba, Wasir and Himmat are sitting
around a circle facing at the centre.
q Shortcut Approach
Manav is to the immediate right of Bunty
• Imagine yourself as one of the who is 4th to the right of Kavya. Payal
persons given in the question. is 2nd to the left of Bunty and is 4th to
• Count how many people are the right of Wasir. Qasturba is 2nd to
mentioned in the question. Then the right of Dev who is 2nd to the right
draw a circle with those many of Himmat.
people.
Q. Who is 3rd to the right of Bunty?
Sol.
Left Right

• Imagine yourself at the position


shown by the box. Wasir
• Now your left hand is the left side Himmat Manav
and right hand is the right side.
Seating
• Now, if in question it is given, P is Kavya Arrangement
Bunty
second to the right of Q, approach
as follows. Qasturba
Dev
® Imagine yourself as Q.
Payal

Left Right
Q To give the answer, see the setting
position. You can easily found the
® Now, P is second to right of Himmat is 3rd to the right of Bunty.
Q. The right of Q is your right side.
So, place P is second place from 3. Problems Based On
Q towards its right. Comparison
In such problems comparison of
different objects or persons has to
P
be made. Such comparisions are
Left Right done on the basis of marks, ages
Q heights, etc.
EBD_7420
96 Problem Solving (Puzzles)
Method to Solve D is ranked 6th and B is 5th. This
does mean that possibilities (2) and
If you give a serious look to the problem (3) are violated. Hence, possibility
you will find that such problems are as (1) must be true. Thus, we have:
same as the arrangement problems. C ______ B D E
Therefore, we have to go like Also by virtue of (3) and (4) we can
arrangement problem while solving have only one arrangement for G,
A and B which is GAB. Accordingly,
problems based on comparison.
there are three possibilities:
EXAMPLE 7. Directions : Read the C F GAB D E
C G FAB D E
informations given below to
and C G AF B D E
answer the given question: So, if D is ranked 6th and B is ranked
(1) 7 students A, B, C, D, E, F and 5th, then F is ranked 2nd, 3rd or
G take a series of tests. 4th.
(2) No two students obtain the 4. Problem s Based On
same marks.
Blood Relation
(3) G always scores more than A.
Such problems involves analysis
(4) A always scores more than B.
of certain blood relations.
(5) Each time either C scores the
highest and E gets the least, q Shortcut Approach
or alternatively D scores the (i) Vertical/diagonal lines to
highest and F or B scores the represent parent-child
least. relationships.
Q. If D is ranked 6th and B is ranked (ii) Single/double horizontal line
5th, then what will be the rank of F? like ( « / Û) to represent
Sol. marriages.
In this case, we see there is no definite (iii) A dashed line (—) for brother
information. Sentence 5 gives a definite and sister relationship.
information but it is conditional. Still, we (iv) ‘+’ sign for male and ‘–’ sign
draw all the possibilities based on for female
sentence 5. For example.
+
(1) C __ __ __ __ __ __ E A B

or, (2) D __ __ __ __ __ __ F
or, (3) D __ __ __ __ __ __ B
We see that the two additional + –
informations (3) and (4) are inadequate D E
to reach a definite conclusion. Hence,
keeping these in mind. We move on to
the given questions. G
– +
F
or
Problem Solving (Puzzles) 97
+
All 6 members M, N, O, P, Q, R of a
A –
B family are travelling together. N is
the son of O but O is not the
mother of N. M and O are a married
couple. Q is the brother O. P is the
+ –
daughter of M. R is the brother of N.
D E Q. How many male members are there
in the family?

G F
+
Sol. Here, all the sentences are actual
or information except the first. Out of
these statements, the 2nd and the
+
A B

fifth sentences give information on
parent child relationship. We can
begin with either of the two. Let
us begin with the 6th sentence. Our
+
diagram will be as

D E
M

(?)
G F+
The above diagrams tells us:-
(–)
(a) A and B are couple; A is the P
husband while B is the wife. As, we do not want to make many
(b) D is son of A and B while E diagrams, we would prefer to only
is daughter of A and B. add to the existing diagsams.
(c) D is the brother of E and E is Therefore, we should look for
the sister of D. sentences that talk of M or P. The
(d) D has a son F 3rd sentence talks about M. Hence,
we add this information, that M
(e) F and G are couple; F is the
and O are married couple in our
husband and G is the wife.
diagram.
(f) F is the grandson of A and
B. M O
(?) (?)
(g) G is the daughter in law of
D.
(–)
(h) E is the aunt (Bua) of F P
(i) There are 3 males (A, D and Now, the 2nd sentence talks about
F) and 3 females (B, E, G) O. It says that N is the son of O
but O is not the mother of N.
EXAMPLE 8. Directions : Read the Obviously, O must be the father of
N. This means O is a male and
following information carefully
hence M must be a female. Now
and answer the question given
our diagram takes the form as
below:
following:-
EBD_7420
98 Problem Solving (Puzzles)
M
(–) (+) (5) D is the grandmother of E and
O
is a housewife.
(6) There is one engineer, one
lawyer, one teacher, one
P
(–)
N
(+) housewife and two students
in the family.
Now, we add the two sentences Q. Who is the husband of A?
‘Q is the brother of O’ and ‘R is the Sol. Here, (1), (2), and (6) are useful
brother of N’ and we get the final secondary informations. While
diagram as below:- (3), (4) and (5) are the actual
M
(–) (+) (+) informations. We start with the 3rd
O Q
sentence because it mentions a
parent child relationship. Its
diagram can be made as the
(–) following:-
P R
(+)
N
(+)

B
Number of '+' sign is four.. (+, Eng)
therefore, there are 4 male members
in the family. B is an Engineer
and father of E
5. Problem s Based On
Blood Rela tions and
Profession E
Such problems are very much (?, ?)
similar to the problems related to Now, we move on to another
blood relation. What makes it sentence that involves either B or
different is the addition of new E. You see that the 5th sentence
data:- the professions of family gives some information about E. It
members. You will get the more says that D is the grandmother E.
clear idea about this type of Point to be noted that if D is the
problem. grandmother of E, then the son of
D must be father of E and hence B
EXAMPLE 9. Directions : Read the is the son of D. Now, the diagram
following information carefully takes the following form.
and answer the question given D
below it: (–, Housewife)
(1) A, B, C, D, E and P are
members of a family.
(2) There are two married
B D is a housewife
couples. (+, Eng) and grandmother E
(3) B is an engineer and the father
of E
(4) P is the grandfather of C and
is a lawyer. E
(?, ?)
Problem Solving (Puzzles) 99
Now, the 4th sentence has the the mother (A) should be the
remaining information and diagram teacher and the two children E and
for it is given below:- C should be students. But this
P conclusion can be challenged and
(+, Lawyer)
has no reason at all.
Apart from that the sexes of E and
P is a lawyer and C can not be determined.
(?) grandfather of C From the above diagram, it is clear
that B is husband of A.

C 6. Problem s Based On
(? ?) Conditional Selection
Now, we see that we have ended In this type of problems, a group
up with two different component. of objects/persons has to be
Then how to resolve this deadlock?
selected from a given larger group,
The answer is simple: - to resolve
it, we make use of the given useful as per the given restrictions. You
secondary information (USI). will get the better idea of such type
“There are two married couple in of problems from the problem
the family.” Clearly, the two given below:-
possible pairs are of grandfather, EXAMPLE 10. Directions : Study the
grandmother and father, mother.
following information carefully
Therefore, we combine the two
diagrams into the following way. and answer the question given
below it:
P D From, amongst 6 boys J, K, L, M,
(+, Lawyer) (–, Housewife) N & O and 5 girls P, Q, R, S and
T, a team of 6 is to be selected
under the following conditions:
(i) J and M have to be together.
B A (ii) L can not go with S.
(+, Eng) (–, ?) (iii) S and T have to be together.
(iv) K can not be teamed with N.
(v) M cannot go with P.
E (vi) K and R have to be together.
C
(?, ?) (?, ?) (vii) L and Q have to be together.
Q. If there be 5 boys in the team, then
Point to be noted that the the lone girl member is ------
professions of A, E and C are
Sol. Make the group of all the pairs that
yet unknown. However, using the have to be together on one side
6th sentence with reasonable and the pairs that must not be
justification, we may assume that together on the other side. Next,
EBD_7420
100 Problem Solving (Puzzles)
read each of the questions and L, S, K, N, M, P
treat that as an additional (+) (–) (+) (+) (+) (–1) ® Group 'never
information. Finally, analyse the be together’
possibilities and choose the Here, number of boys are 5. We
possibilities that satisfies all the see than K and N can never be
conditions. Let us see the process together. Therefore, there are only
below. two ways of selecting 5 boys –
Firstly, we can summarise the JKLMO and JNLMO. But if K
conditions in the following way. would select then R should also
J, M S, T select, and if L goes than Q should
(+ )(+ ) ( -)( -) also go. Hence, JNLMO is the only
® Group possibility in which L’s friend Q
K, R L, Q ' must be together ' would be the lone girl member.
(+ )( -) (+ )( -)

PRACTICE EXERCISE
1. Who amongst the following is the
DIRECTIONS (Qs. 1-5): Study the
son of N?
following information carefully to
answer the given questions. (a) Q (b) P
(c) K (d) J
Eight persons - J, K, L, M, N, O, P, Q are
sitting around a circular table facing the (e) M
centre with equal distances between 2. How many people sit between N
each other (but not necessarily in the and K, when counted from the
same order). Each of them is also related right of K?
to N in some way or the other. (a) Five (b) Two
K sits third to the left of N. Only (c) Four (d) Three
one person sits between N and Q. N’s (e) None
sister sits to the immediate right of Q. 3. Who sits to the immediate right of
Only two people sit between N’s sister Q?
and N’s mother. J sits to the immediate (a) J (b) N’s sister
right of N’s mother. P sits to the (c) N (d) N’s Wife
immediate right of M. N’s brother sits (e) K
third to the right of P. N’s wife sits 4. Which of the following statements
second to the left of N’s brother. Only is true with respect to the given
three people sit between N’s wife and L. information?
N’s son sits second to the right of N’s (a) All the given options are true
father. Only two people sit between N’s (b) P sits to the immediate left of J
father and N’s daughter.
Problem Solving (Puzzles) 101
(c) N’s mother sits to the 8. In which of the following
immediate left of N combinations is the third person
(d) M is the mother-in-law of Q sitting between the first and the
(e) N is an immediate neighbour second person ?
of his father. (a) PWU (b) QTR
5. How is J related to L? (c) RST (d) WUP
(a) Sister (b) Uncle (e) None of these
9. If Q and R, V and U interchange
(c) Sister-in-law (d) Father
their position then how many
(e) Daughter
persons are sitting between R and V?
DIRECTIONS (Qs. 6-10): Study the (a) Four (b) Five
following information Carefully to (c) Six (d) Two
answer the given questions (e) None of these
P, Q, R, S, T, U, V, W and X are sitting in 10. Who among the following sits
a straight line, facing North. Three of third to the left of P ?
them are not males. Two females sit (a) W (b) V
adjacent to each other. Q is fourth to (c) R (d) X
the left of V, who is second to the right (e) None of these
of R, who is not the immediate DIRECTIONS (Qs. 11-15): Study the
neighbour of P. information given below and answer
· U is fourth to the right of R and is the question based on it.
second to the left of X. S is not an Seven persons P, Q, R, S, T, U and V
immediate neighbour of either X or Q. watched movies on different days starting
· S is not male. One of the persons from Monday to Sunday. They watch i.e.
sitting on the extreme ends is a Logan, Avatar, Inception, Superman, Thor,
female. T is not an immediate Avengers and Batman, but not necessarily
neighbour of either V or U. in the same order. 3 persons watched
· No female is an immediate movie between U and the one who
neighbour of U. W does not sit watched Avengers and U watched movie
second to the left of P. The before the one who watched Avengers but
immediate neighbour of S are male not on Monday. P watched Inception with
6. Which of the following is a group a gap of a day, before the one who watched
of females ? Avengers. One person watched movie
(a) QTS (b) TXP between P and the one who watched
(c) SVR (d) UWX Logan. Q watched Avatar just before U.
(e) None of these One person watched movie between Q
7. Who is sitting to the immediate and T. T didn’t watch movie on Thursday.
left of S ? S watched Batman before R who watched
(a) V (b) Q Thor.
(c) W (d) R 11. Who among the following
watched on Monday?
(e) None of these
EBD_7420
102 Problem Solving (Puzzles)
(a) P (b) Q the same order. There is only one floor
(c) T (d) U between P and the one who belongs to
(e) V Patna lives. The person who belongs to
12. T watched which of the following Patna does not live on floor numbered
movie? 1. S lives just below Q. The one who
(a) Superman
(b) Avengers belongs to Jaipur lives an even
(c) Logan numbered floor and just above the floor
(d) Batman on which the one who belongs to Pune
(e) None of these lives. The person who belongs to Raipur
13. How many persons watched lives on an even numbered floor but not
movie between P and R? on the 8th floor. Neither S nor W lives
(a) None (b) 1 on the 1st floor. Only one person lives
(c) 2 (d) 3 between the one who belongs to Ranchi
(e) 4 and S. P lives on an odd-numbered floor
14. 'Superman' is related to Monday
and T lives just above P. Q lives on the
in the same way as 'Avengers' is
fourth floor. Only two persons live
related to Thursday. Likewise,
between the person who belongs to
'Thor' would be related to?
Raipur and P. U lives just below the one
(a) Friday
(b) Saturday who belongs to Pune. S belongs to
(c) Sunday neither Pune nor Patna. The one who
(d) Tuesday belongs to Delhi does not live on an
(e) Wednesday odd-numbered floor. V does not belong
15. Which of the following is true to Kolkata. There are two floors between
regarding 'V'? the floor on which W lives and the floor
(a) 'V' watches 'Inception' on which T lives. Only two persons live
(b) 'V' does not watch the movie between the one who belongs Mumbai
on 'Saturday'.
and the one who belongs to Delhi.
(c) 'V' watches the movie on
Thursday 16. Who among the following
(d) Both 'A' & 'B' belongs to Kolkata?
(e) None of these (a) S (b) R
(c) P (d) V
DIRECTIONS (Qs. 16-20): Study the
(e) None of these
following information carefully and
17. How many persons are there
answer the questions given below:
between T and Q?
P, Q, R, S, T, U, V and W are eight friends (a) One (b) Two
who live in an eight-storey building. The (c) Three (d) Four
ground floor is numbered one and the (e) None of these
topmost floor is numbered eight. Each 18. Who among the following lives
of them belong to different cities, Jaipur, on the topmost floor?
Kolkata, Delhi, Mumbai, Pune, Raipur, (a) The one who belongs to
Ranchi and Patna but not necessarily in Jaipur
Problem Solving (Puzzles) 103
(b) The one who belongs to Pune (c) Floor no. 1- R-Kolkata
(c) The one who belongs to Delhi (d) Floor no. 8- T- Jaipur
(d) The one who belongs to (e) None of these
Mumbai 20. P belongs to which of the
(e) None of these following city?
19. Which of the following (a) Delhi (b) Mumbai
combinations is/are true? (c) Jaipur (d) Pune
(a) Floor no. 2- S- Mumbai (e) None of these
(b) Floor no. 5- U- Patna

HINTS & SOLUTIONS


Sol. (1-5):

O
K Father
Brother J
Wife

Q P
Son Mother

M
L
Daughter
Sister
N

1. (a) 2. (b) 3. (b) 4. (b) 5. (c)


Sol. (6-10) :

Q(–) T(–) R(+) S(–) V(+) P(+) U(+) W(+) X(+)

6. (a) 7. (d) 8. (a) 9. (b) 10. (c)


Sol. (11-15) :
EBD_7420
104 Problem Solving (Puzzles)
Day Pers on Movie
M o nd ay Q A v atar
Tues day U Lo gan
W edn es d ay T Su perman
Thurs day P In ception
Frid ay S Batman
Satu rd ay V A v eng ers
Sun day R Tho r

11. (b) 12. (b) 13. (c) 14. (a) 15. (e)
Sol. (16-20) :

Floor Pers on City


8 T Jaip u r
7 P Pun e
6 U Delhi
5 W Patn a
4 Q Raipu r
3 S M umb ai
2 R Kolkata
1 V Ran ch i
16. (b) 17. (c) 18. (a) 19. (d) 20. (d)
Input and Output 105

Chapter

14 Input and Output


INTRODUCTION Problem of Shifting
Problems related to input-output are We know that in such type of problems,
frequently asked questions in various a word/number processing machine
graduate level competitive examinations. generate output th rough shiftin g.
They are not very tough stuff but take a Shifting does mean an operation in which
good deal of time to be solved or words or numbers of a given input give
sometimes students do not attempt to outputs in different steps through
solve them because of time consuming shifting their place to different place as
impression of such type of questions. per a fixed pattern.
But proper understanding of the subject
Note : In shifting problems, the previous
makes you believe that such problems
step of any step can possibly be
are not as tough and time consuming as
determined, so we can move in backward
they seem.
or reverse order which is not possible
in some of the other type of problems.
CONCEPT OF INPUT-
OUTPUT PROBLEMS Methods to Solve
In such problems: Lets take an example
(a) It is imagined that there is some Input : Blue Cat Good Other Have Cake
kind of computer/word processing Step 1 : Blue Other Good Cat Have Cake
machine. Step 2 : Blue Other Have Cat Good Cake
(b) An input is given to the computer/ Step 3 : Cake Other Have Cat Good Blue
word processing machine Step 4 : Cake Cat Have Other Good Blue
(c) The computer/word processing Step 5 : Cake Cat Good Other Have Blue
machine performs repeated Step 6 : Blue Cat Good Other Have Cake
operations as per a certain pattern Shifting of element can easily be
to give different output in different understood by making them equivalent
steps. to number like
Blue = 1, Cat = 2, Good = 3, Other = 4,
TYPES OF PROBLEMS Have = 5, Cake = 6
Input can be written as
(i) Problems of shifting 1 2 3 4 5 6
(ii) Problems of arrangement Blue Cat Good Other Have Cake
Step-1 : 2 and 4 interchanged
(iii) Problems of mathematical
Step-2 : 3 and 5 interchanged
operation
Step-3 : 1 and 6 interchanged
(iv) Miscellaneous. Step-4 : 1, 2 and 3 are repeated again.
EBD_7420
106 Input and Output
) )
Input : 1 2 3 4 5 6 Step-3 : 6 4 5 2 3 1
)
) Step-4 : 6 2 5 4 3 1
Step-1 : 1 4 3 2 5 6
Step-5 : 6 2 3 4 5 1
Step-2 : 1 4 5 2 3 6 Step-6 : 1 2 3 4 5 6

Problems on Arrangements
1. Word Arrangement from Left Side:
EXAMPLE :
Input : mango tango orange banana pear
Step I: banana mango tango orange pear
Step II: banana mango orange tango pear
Step III: banana mango orange pear tango
Here, we start arrangement from the word that comes 1st in the dictionary;
then comes the word coming 2nd in the dictionary, then comes the word
coming 3rd in the dictionary and so on. In this case, the arrangement start from
left side. This is the reason in step I banana comes 1st as it comes 1st in the
dictionary. In the 2nd step, orange comes at 3rd place because after the
arrangement of step I the next word coming in the dictionary is mango but it
get arranged automatically and hence there is no need to arrange it in step II.
This is the reason after arranging banana in step I, we directly come to the
word orange (coming 3rd in the dictionary) in step II. In the 3rd step, we
arrange the word ‘pear’ (coming 4th in the dictionary) and the word tango get
arranged automatically.
2. Word Arrangement from Right:
EXAMPLE :

Input: Name Fame Game Shame Jam


Step I: Name Game Shame Jam Fame
Step II: Name Shame Jame Game Fame
Step III: Shame Name Jam Game Fame
In this case, the arrangement starts from right side. The word coming 1st in the
dictionary comes at the 1st position from right. At the 2nd position from right
comes the word coming 2nd in the dictionary and the process goes on till the
arrangement gets completed. In the above given example, ‘Fame’ is the 1st
word coming in the dictionary and hence it comes at the 1st position from right
in the step I. In the step II, the 2nd word coming in the dictionary (Game)
comes at the 2nd position from right. Point to be noted that the word coming
Input and Output 107
third in the dictionary will come at the 3rd position from right and this word is
‘Jam’. But ‘Jam’ automatically get arranged as per the given pattern when we
arrange the word ‘Game’ in II step. This is the reason why we don’t arrange
‘Jam’ in the third step and jump directly to arrange the word. ‘Name’ that
comes 4th in the dictionary. ‘Name’ occupies 4th position from right and the
word ‘Shame’ automatically get arranged in the 5th step. Hence, the word
‘Shame’ does not need to get arranged.
3. Word Arrangement from the Left-Right Alternate:
EXAMPLE :

Input: Sachin is a great cricket player


Step I: a Sachin is great cricket player
Step II: a is great cricket player Sachin
Step III: a cricket is great player Sachin
Step IV: a cricket great is player Sachin
Here, the arrangement is made by putting the alphabetically first word at 1st
place, then alphabetically last word at last place, then alphabetically second
word at second place from left and the further arrangements goes on in the
same manner. In the other words, words are positioned from the left and from
the right alternately. In the step I the word coming 1st in the dictionary is ‘a’
and it takes 1st position from left. In the step II, the last word coming
alphabetically is Sachin and it takes last position (1st from right). In step III,
the word coming 2nd in dictionary is ‘cricket’ that comes at 2nd position from
left. In step IV, the word coming 3rd last in the dictionary takes the 3rd position
from right. After the step IV, all the words get arranged in alphabetical order.
Point to be noted that after step IV, there is no need to arrange the word ‘great’
as it get arranged automatically is step IV.
4. Arrangement in Increasing or Decreasing Order:

EXAMPLE :
Input: 25 17 18 58 100 35
Step I: 17 25 18 58 100 35
Step II: 17 18 25 58 100 35
Step III: 17 18 25 35 58 100
This arrangement gives a clear idea of arrangement of numbers in increasing
order. In step I, the smallest number (17) comes at the 1st position from left
pushing the remaining to the right. In step II, the 2nd smallest number (18)
comes at 2nd position from left pushing the remaining number to the right. In
step III, the 4th smallest number (35) takes 4th position from left and the other
two numbers 58 and 100 get arranged automatically.
EBD_7420
108 Input and Output
Now, let us see decreasing order arrangement:
Input: 25 17 18 58 100 35
Step I: 100 25 17 18 58 35
Step II: 100 58 25 17 18 35
Step III: 100 58 35 25 17 18
Step IV: 100 58 35 25 18 17
The same arrangement can take place from right side (or in the reverse order)
as follow:
Input: 25 17 18 58 100 35
Step I: 25 18 58 100 35 17
Step II: 25 58 100 35 18 17
Step III: 58 100 35 25 18 17
Step IV: 100 58 35 25 18 17

5. Number Arrangment from Left-Right Alternate:


Like words left-right alternate arrangement, number arrangement also takes
place. The process of this arrangement is exactly the same as the arrangement
takes place in case of words. Just see the following cases:
Case I :
Input: 100 125 26 10 15 35
Step I: 10 100 125 26 15 35
Step II: 10 100 26 15 35 125
Step III: 10 15 100 26 35 125
Step IV: 10 15 26 35 100 125
Here, the smallest number (10) takes 1st position from left in step I. In step II
the largest number takes the last (1st from right) position. Again in step III the
2nd smallest number (15) comes at the 2nd position from left. In the step IV, the
2nd largest number (100) comes at the 2nd position from right and the remaining
number (26 and 35) get arranged automatically.
Case II :
Input: 100 125 26 10 15 35
Step I: 100 26 10 15 35 125
Step II: 10 100 26 15 35 125
Step III: 10 26 15 35 100 125
Step IV: 10 15 26 35 100 125
In case II, the arrangements take place in the same way as the arrangements
take place in case I. But the difference here is that case I is a left-right
Input and Output 109
arrangement and case II is the right-left arrangement. In case II, the arrangement
starts with the largest number (125) coming at the 1st position from right and
this is step I. In step II, the smallest number (10) comes at the 1st position from
left. In step III the 2nd largest number (100) comes at the 2nd position from
right. In step III, the third largest number (35) automatically comes at the 3rd
position from right. In 4th step, the 2nd smallest number (15) comes at the 2nd
position from left and 26 get arranged automatically coming at 3rd position
from left.
Note: Left-right (or right-left) arrangement of numbers also take place in the
same manner when numbers are arranged in decreasing order.

6. Arrangement of Words and Numbers Simultaneously:


Just see the following outputs produced by a word and number machine.
Case I
Input: 50 32 Vandana Prerna Aradhna 100
Step I: 32 50 Vandana Prerna Aradhna 100
Step II: 32 Aradhna 50 Vandana Prerna 100
Step III: 32 Aradhna 50 Prerna Vandana 100
Step IV: 32 Aradhna 50 Prerna 100 Vandana
In such case, numbers and words get arranged alternately. In step I, the smallest
number (32) comes at the 1st position from left pushing the remaining members
of input towards right. In the step II, the word coming 1st alphabetically (that
is the word ‘Aradhna’) takes the 2nd position from left pushing the remaining
member rightward. Point to be noted that the 2nd smallest number automatically
comes at the third position from left while arranging the word ‘Aradhna’ and
hence, there is no need to arrange the 2nd smallest number ‘50’. In step III, the
word (Prerna) coming 2nd alphabetically comes at the 4th position from left
pushing the other members to the right. In step IV, the largest number (100)
occupies the 5th position from left and the word (Vandana) coming last
alphabetically comes at last position automatically finishing the complete
arrangement.
Let us see some other cases of this type:
Case II:
Input: 50 32 Vandana Prerna Aradhna 100
Step I: 100 50 32 Vandana Prerna Aradhna
Step II: 100 Vandana 50 32 Prerna Aradhna
Step III: 100 Vandana 50 Prerna 32 Aradhan
EBD_7420
110 Input and Output
In this case, largest number and the word coming last alphabetically get
arranged alternately. Then the 2nd longest number and the word coming 2nd
last alphabetically get arranged alternately and the process goes on till the
arrangements of all the numbers and words get completed. In this case,
arrangement completes in step III.
Case III:
Input: 50 32 Vandana Prerna Aradhna 100
Step I: Aradhna 50 32 Vandana Prerna 100
Step II: Aradhna 32 50 Vandana Prerna 100
Step III: Aradhna 32 Prerna 50 Vandana 100
In this case, arrangement starts with the word coming 1st alphabetically and
such word is ‘Aradhna’ that comes at the 1st position from left is step I. In step
II, the smallest number (32) comes at the 2nd position from left. Then, in step
III, the word Prerna coming 2nd alphabetically comes at the 3rd position from
left and all the other members get arranged automatically.
Case IV:
Input: 50 32 Vandana Prerna Aradhna 100
Step I: Vandana 50 32 Prerna Aradhna 100
Step II: Vandana 100 50 32 Prerna Aradhna
Step III: Vandana 100 Prerna 50 32 Aradhna
Step IV: Vandana 100 Prerna 50 Aradhna 32
In this case, word coming last alphabetically comes 1st from left in step I and
such word is ‘Vandana’. In step II, the largest number (100) comes at the 2nd
position from left. In step III, the word coming 2nd last alphabetically occupies
the 3rd position from left, and such word is ‘Prerna’. As the 2nd largest number
(50) automatically get arranged as per the pattern going on and hence this is
not needed to arranged in step IV. In step VI, the word coming Ist alphabetically
comes at the 5th position from left and such word is ‘Aradhna’. The smallest
number (32) get arranged automatically coming at the last position from left in
step IV. Thus, it is clear that in this case the word coming lst alphabetically and
the greatest number get arranged alternately in 1st two steps; then 2nd last
word alphabetically and 2nd largest number get arranged alternately finishing
the whole arrangement in step IV.
Case V:
Input: 50 32 Vandana Prerna Aradhna 100
Step I: 32 50 Vandana Prerna Aradhna 100
Step II: 32 Vandana 50 Prerna Aradhna 100
Step III: 32 Vandana 50 Prerna 100 Aradhna
Input and Output 111
In this case, the smallest number comes at the 1st position from left in step I
and such number is 32. In step II, the word (Vandana) coming last alphabetically
occupies the 2nd place from left. In the 2nd step, the 2nd smallest number (50)
takes the 3rd position from left automatically and also the word coming 2nd
last alphabatically takes the 4th position from left automatically. Hence, there
is no need to arrange ‘50’ and ‘Prerna’. In the III step, the largest number (100)
occupies the 5th position from left completing the whole arrangement.
Case VI:
Input: 50 32 Vandana Prerna Aradhna 100
Step I: 100 50 32 Vandana Prerna Aradhna
Step II: 100 Aradhna 50 32 Vandana Prerna
Step III: 100 Aradhna 50 Prerna 32 Vandana
In this case, the logic is that the greatest number (100) comes at the 1st position
from left in step I. In step II the word coming 1st alphabetically takes the 2nd
position from left and the 2nd largest number (50) gets arranged automatically.
Hence, in step III, we direct arrange the word coming 2nd last alphabetically
(that word is ‘Prerna’) occupies the 4th position from left and the other two
members (32 and ‘Vandana’) get arranged automatically finishing the whole
arrangement.

7. Arrangement Based on the Number of Letters in Words:


Just have a look at the following patterns:
Case I :
Input: let pattern love fried be mature
Step I: be let pattern love fried mature
Step II: be let love pattern fried mature
Step III: be let love fried pattern mature
Step IV: be let love fried mature pattern
Here, the words get arranged as per increasing number of letters. In other
words, the word having least number of letters comes 1st from left in step I and
such word is ‘be’. The word ‘let’ is bigger than ‘be’ and smaller than other
words letterwise and hence, it takes 2nd position from left but it gets arranged
automatically when the word ‘be’ is arranged in step I. In 2nd step, the word
‘love’ comes at the 3rd position from left as it is bigger than word ‘let’ letterwise.
In step III, the letterwise bigger word (fried) than love comes at the fourth
position from left. Similarly, mature comes at the 5th position from left and
pattern comes at the last position automatically while arranging the word
‘mature’.
EBD_7420
112 Input and Output
Case II :
Input: let pattern love fried be mature
Step I: pattern let love fried be mature
Step II: pattern mature let love fried be
Step III: pattern mature fried let love be
Step IV: pattern mature fried love let be
In this case, the words get arranged in decreasing order in terms of letters. In
other words, the word having the largest number of letters comes 1st from left,
then comes the word having 2nd largest number of letters, then comes the
word having 3rd largest number of letters and the process goes on till the
word having the least number of letters occupies the last position from left.
Case III:
Input: let pattern gate a set be hope
Step I: a let pattern gate set be hope
Step II: a be let pattern gate set hope
Step III: a be let set pattern gate hope
Step IV: a be let set gate pattern hope
Step V: a be let set gate hope pattern
Have you noticed something here? Here, the words get arranged in increasing
number of letters. But when it comes to the case of two or more words having
equal number of letters the priority is given alphabetically. It does mean that
the word coming 1st as per the alphabet will be put before the word coming
2nd. Similarly, the word coming 2nd alphabetically will be put before the word
coming third. This is the reason why ‘let’ has been put before ‘set’ and ‘gate’
has been put before ‘hope’.
Case IV:
Input: let pattern gate a set be hope
Step I: pattern let gate a set be hope
Step II: pattern hope let gate a set be
Step III: pattern hope gate let a set be
Step IV: pattern hope gate set let a be
Step V: pattern hope gate set let be a
In this case, the words get arranged in decreasing number of letters. But when
it comes to the case of two or more words having equal number of letters the
priority is given to the word that comes later alphabetically. It does mean that
the word coming 1st alphabetically will be put after the word coming 2nd and
the word coming 2nd will be put after the word coming 3rd. This is the reason
why ‘hope’ has been put before ‘gate’ and ‘set’ has been put before ‘let’.
Input and Output 113
Important Note: The case of arrangement discussed so far are the cases of push. In
all the cases a new word jumps from its place in every step, occupies its new and
due place and gives the remaining words a push either towards left or right as
per the requirement of the pattern. But in some cases of arrangement interchange
does take place and that format is given below:
8. Arrangement with Interchange:
EXAMPLE
Input: the most beautiful girl is Vandana
Step I: beautiful most the girl is Vandana
Step II: beautiful girl the most is Vandana
Step III: beautiful girl is most the Vandana
In this case, the word (beautiful) coming 1st in alphabetical order comes at the
1st position from left interchanging its place with the word ‘the’ and this is
step I. In step II, the word (girl) coming 2nd in alphabetical order occupies the
2nd position from left interchanging with the word ‘most’. In step III, the word
coming 3rd (is) comes at the third position from left interchanging with the
word ‘the’ and finishing the complete arrangement in alphabetical order.
This type of cases can also be seen in number arrangements and in the
arrangements of numbers and words simultaneously. The examples of these
type of arrangements are given below:

EXAMPLE (Increasing order number arrangement)


Input: 25 11 50 20 35
Step I: 11 25 50 20 35
Step II: 11 20 50 25 35
Step III: 11 20 25 50 35
Step IV: 11 20 25 35 50
Presentation :

Step I: 11 25 50 20 35

Step II: 11 20 50 25 35

Step III: 11 20 25 50 35

Step IV: 11 20 25 35 50
EBD_7420
114 Input and Output
The presentation gives you the clear idea of how interchange takes place in
every step.

EXAMPLE (Decreasing order number arrangement)


Input: 25 11 50 20 35
Step I: 50 11 25 20 35
Step II: 50 35 25 20 11
Presentation:

Step I: 50 11 25 20 35

Step II: 50 35 25 20 11

Problems of Mathematical Operation


In this type of problems, the input has some numbers. Different steps are obtained
by taking the numbers of the input and different arithmetic operations are performed
after that.

EXAMPLE

Input : 44 35 18 67 22 28 36
Step I : 36 27 10 59 14 20 28
Step II : 16 15 8 42 4 16 18
Step III : 132 105 54 201 66 84 108
Step IV : 50 41 24 73 28 34 42
Step V : 8 8 9 4 4 1 9
Step VI : 64 64 81 169 16 100 81
Step VII : 20 19 12 46 8 20 22
In this case, in step I (each number of the input – 8). In step II, product of the
digits of each number of the input. In step III, each number of the input is
multiplied by 3. In step IV, each number of the input is added by 6. In step V,
keep adding the digits of each number of the input till they are converted into
single digit. In step VI, (digit sum of each number of input)2. In step VII, each
number of step II is added by 4.
Input and Output 115
Miscellaneous Problems
In this type of problems, there is no fixed pattern of questions coming under this
category. Infact, questions under this category comes before you as a real surprise.
EXAMPLE

Input : every now and then same


Step I : very ow nd hen ame
Step II : ever no an the sam
Step III : vry nw nd thn sm
Step IV : ee o a e ae
Step V : ery w d en me
In this case, in step I, first letter disappear. In step II, last letter disappear. In
step III, vowels disappear. In step IV, consonants disappear. In step V, first two
letters disappear.

q Shortcut Approach
1. First of all, observe the given input line of words or numbers and the last step
of rearrangement, so that candidate may get an idea about the changes effected
in various steps of rearrangement.
2. In order to know what changes have been made in each step, observe two
consecutive steps carefully.
3. Now, correlate the input, the last step and anyone of the middle steps. This
will enable you to identify the rule of arrangement.
4. In shifting problems, it is possible to determine the previous/earlier steps
including input. We can proceed/move backward or in reverse direction in
shifting problems.
5. In shifting problems for convenience, we assign numeric value to given words.
EBD_7420
116 Input and Output

PRACTICE EXERCISE
DIRECTIONS (Qs. 1-5) : Study the 3. The step II of an input is an
following information and answer the follows: ‘and Do pet to an that’.
given questions. A word arrangement Which of the following would
machine, when given an input line of definitely be the input?
words, rearrange them following a (a) Do and pet to an that
particular rule in each step. The (b) Do pet to and that on
following is an illustration of input and (c) Do on pet to and that
the steps of rearrangement. (d) Cannot be determined
(e) None of these
Input Go for to though by easy To 4. Input ‘Over Go for through at are’.
Access at Which steps will be the last step
Step I Access go for to though by of the above input?
easy To at (a) II (b) VI
Step II Access at go for to though by (c) IV (d) VII
easy To (e) III
Step III Access at by go for to though 5. Input ‘Story for around on was He
easy To at’.
Step IV Access at by easy go for to Which of the following will be step
though To IV for the given input?
Step V Access at by easy for go to (a) around at He for story on was
though To (b) around at for He story on was
Step VI Access at by easy for go (c) around at for He on story was
though to To
(d) around at for He on was story
Step VII Access at by easy for go
(e) None of these
though To to (and step VII is
the last step for this input). As DIRECTIONS (Qs. 6-10): Study the
per the rules followed in the following information carefully and
above steps, find out in the answer the given question:
following questions the
A word and number arrangement
appropriate step for the given
machine when given an input line of
output.
1. Input Together over series on words and numbers rearrange them
feast the so which of the following following a particular rule in each step.
steps will be the last but one? The following is an illustration of input
(a) II (b) III and rearrangement. (All the numbers are
(c) IV (d) V two-digit numbers).
(e) None of these Input : gate 20 86 just not 71 for 67 38
2. Input Every and peer to an for bake sun 55
which of the following steps Step I: bake gate 20 just not 71 for 67
would be ‘an and every for peer 38 sun 55 86
to? Step II: for bake gate 20 just not 67 38
(a) II (b) IV sun 55 86 71
(c) V (d) III Step III: gate for bake 20 just not 38 sun
(e) None of these 55 86 71 67
Input and Output 117
Step IV: just gate for bake 20 not 38 sun 10. Which words numbers would be
55 86 71 67 at 5th position from the right in the
Step V: not just gate for bake 20 sun 86 last step?
71 67 55 38 (a) gem (b) 63
Step VI: sun not just gate for bake 86 71 (c) 56 (d) 85
67 55 38 20 (e) does
Steps VI: is the last step of the DIRECTIONS (Qs. 11-15): Study the
arrangement the above input. following information carefully and
As per the rule followed in the above answer the given questions.
steps, find out in each of the following When a word and number arrangement
questions the appropriate step for the machine is given an input line of words
given input. and numbers, it arranges them
Input: 31 rise gem 15 92 47 aim big 25 following a particular rule. The
does 56 not 85 63 with moon following is an illustration of Input and
6. How many steps will be required rearrangement. (All the numbers are
two digit numbers).
to complete the rearrangement?
Input : Daily 79 do diverse 57 14 dear
(a) Eight (b) Six
86 63 domain 42 dog
(c) Seven (d) Five
Step I: diverse daily 79 do 57 dear 86
(e) None of these 63 domain 42 dog 41
7. Which words numbers would be Step II: domain diverse daily 79 do 57
at 7th position from the left in step dear 86 63 dog 41 24 75
IV? Step III: daily domain diverse 79 do
(a) rise (b) aim dear 86 63 dog 41 24 75
(c) big (d) 15 Step IV: dear daily domain diverse 79
(e) 47 do 86 dog 41 do 75 36
8. Which step number is the following Step V: dog dear daily domain
output? diverse do 86 41 24 75 36 97
rise not moon gem does big aim 15 Step VI: do dog dear daily domain
with 92 85 63 56 47 31 25 diverse 41 24 75 36 97 68
(a) Step V (b) Step VII Step VI: is the largest step of above
(c) Step IV (d) Step VIII arrangement as the indended
arrangement as the intended
(e) There is no such step
arrangement is obtained
9. Which of the following represents As per the rules followed in the given
the position of ‘92’ in step VI ? steps, find out the appropriate steps
(a) Ninth from the left for the Input.
(b) Fifth from the right Input : table 63 tour 19 typhoon 72 25 to
(c) Sixth from the right tea 48 tablet 56
(d) Ninth from the right 11. Which elelment comes exactly
(e) Seventh from the left between 'typhoon' and ' tour' in
step III of the given input?
EBD_7420
118 Input and Output
(a) 63 (b) to 15. If the step IV, '72' interchange its
(c) 91 (d) table position with 'table' and 'typhoon'
(e) 56 also interchanges its position
12. Which of the following with '91' then which element will
combinations represent the sixth be to the immediate right of '91'?
and eight element in Step II of the (a) 63 (b) to
given input from left end? (c) 52 (d) typhoon
(a) 63 and tablet
(e) 72
(b) tea and 63
(c) 72 and tablet DIRECTIONS (Qs. 16-20): Study the
(d) 72 and tea given information carefully and answer
(e) 48 and table the given questions.
13. If in the last step '2' is added to
each of the odd numbers and '1' is An input-output is given in different
subtracted from each of the even steps. Some mathematical operations are
numbers, then how many done in each step. No mathematical
numbers multiple of '3' will be operation is repeated in next step but it
formed? can be repeated with some other
(a) Two (b) One mathematical operation (as multiplication
(c) None (d) Three can be used with subtraction in step 1
(e) More than three and same can be used with addition in
14. Which element is fourth to the
step 2)
left of one which is ninth from the
left end in second last step?
(a) Table (b) 72
(c) typhoon (d) 91
(e) tablet

29 1 5 2 3 6 1 2 1 1 3

Step I: 4 4 86 3 4

Step II: 96 6 4

Step III: 18 12

Step IV: 3 6

As per the rules followed in the steps given above, find out in each of the
following questions the appropriate step for the given input.
1 2 4 1 1 7 19 2 3 11
Input and Output 119
16. Find the addition of the three 19. Find the square of number which is
numbers obtained in step I? obtained in Step IV?
(a) 123 (b) 124 (a) 729 (b) 676
(c) 125 (d) 126 (c) 525 (d) 625
(e) None of these
(e) 1000
17. Find the multiplication of two
numbers obtained in Step III? 20. If digit is exchanged within the each
(a) 600 (b) 500 block then find the multiplication of
(c) 550 (d) 575 two new numbers obtained in step
(e) None of these II?
18. Find the difference between the two (a) 1525 (b) 1456
numbers obtained in Step II? (c) 1460 (d) 1458
(a) 28 (b) 17 (e) None of these
(c) 37 (d) 27
(e) None of these

HINTS & SOLUTIONS


1. (d) Step I feast Together over Step III are at for over Go
series on the so through
Step II feast on Together over Step IV are at for Go over
series the so through
Step III feast on over 5. (b) The step IV for the given
Together series the so input will be ‘around at for
Step IV feast on over series He story on was’.
Together the so
Step V feast on over series Sol. (6-10) : After careful analysis of
so Together the the given input and various steps of re-
Step VI feast on over series arrangement it is evident that in each step
so the Together one word and one number are rear-
2. (d) Using the above rule, we ranged.
observe that ‘are and every The word are rearranged from left in al-
for peer to’ will be the III and phabetical order and the numbers are
last step for the given input. rearranged from the right in descending
3. (d) For the given step we cannot order but in the final step the word get
definitely find out the input rearranged in alphabetical order in re-
because the position of the serves manner and number appear in
words in input cannot be descending order.
determined. Input : 31 rise gem 15 92 47 aim big 25
4. (c) Step I are over Go for through does 56 not 85 63 with moon
at Step I: aim 31 rise gem 15 47 big 25
Step II are at over Go for does 56 not 85 63 with moon
through 92
EBD_7420
120 Input and Output
Step II: big aim 31 rise gem 15 47 25 Sol. (11-15):
does 56 not 63 with moon 92 Input: table 63 tour 19 typhoon 72 25
85 to tea 48 tablet 56
Step III: does big aim 31 rise gem 15 47 Step I: typhoon table 63 tour 72 25
25 56 not with moon 92 85 63 to tea 48 tablet 56 91
Step IV: gem does big aim 31 rise 15 47 Step II: tablet typhoon tablet 63 tour
25 not with moon 92 85 63 56 72 to tea 48 56 91 52
Step V: moon gem does big aim 31 rise Step III: table tablet typhoon 63 tour
15 25 not with 92 85 63 56 47 72 to tea 56 91 52 84
Step VI: not moon gem does big aim rise Step IV: tour table tablet typhoon 63
15 25 with 92 85 63 56 47 31 72 to tea 91 52 84 65
Step VII: rise not moon gem does big aim Step V: tea tour table tablet typhoon
15 with 92 85 63 56 47 31 25 72 to 91 52 84 65 36
StepVIII: with rise not moon gem does Step VI: to tea tour table tablet
big aim 92 85 63 56 47 31 25 15 typhoon 91 52 84 65 36 27
6. (a) 7. (d) 8. (b) 11. (a) 12. (d) 13. (a)
9. (c) 10. (c) 14. (c) 15. (a)

Sol: (16-20):

16. (a)
Addition of number in Step I is = 34 + 36 + 53 = 123
17. (b)
Multiplication of number in Step III- 25×20 = 500
18. (d)
Difference between the number obtained in Step II- 72 – 45 = 27
19. (d)
Number obtained in Step IV is 25
Square of 25 is 625
20. (d) Numbers obtained in Step II – 45, 72
If digits are exchanged then new numbers are 54, 27
Multiplication of new number = 54 × 27 = 1458
Syllogism 121

Chapter
Syllogism
15
INTRODUCTION All the sentences mentioned above are
proposition which give a relation between
Syllogism is a Greek word that does mean
subject and predicate. Here, it is clear from
‘inference’ or ‘deduction’. The problems the sentences that a subject is the part of
of syllogism are based on two parts : a sentence something is said about, while
1. Proposition / Propositions a predicate is the term in a sentence which
2. Conclusion / Conclusions drawn from is related to the subject.
given proposition/ propositions Now, let us define the proposition :
A proposition is a sentence that makes a
PROPOSITION statement giving a relation between two
Just consider the sentences given below: terms. It has three parts :
(a) The subject
(b) The predicate
(i) “All lions are pigs ”
(c) The relation between subject and
predicate
Subject Predicate
CATEGORICAL PROPOSITION
Let us see the sentences given below :
(ii) “No cat is rat ” “All M are P”
“No M are P”
Subject Predicate “Some M are P”
“Some M are not P”
What we notice in all above-mentioned
(iii) “Some girls are beautiful ” sentences that they are condition free.
These type of sentences are called
Categorical Propositions. In other
Subject Predicate
words a categorical proposition has no
condition attached with it and it makes
direct assertion. It is different from non-
(iv) “Some kites are not birds ” categorical proposition which is in the
format
Subject Predicate “If M then P”
EBD_7420
122 Syllogism
TYPES OF CATEGORICAL PROPOSITION:
Categorical proposition

Universal Particular

Positive Negative Positive Negative

All M are P No M are P Some M are P Some M are not P


(A type) (E type) (I type) (O type)

Therefore, it is clear, that universal Some M are P (I type):


propositions either completely include
the subject (A type) or completely M
M P or
exclude it (E type). On the other hand, P
particular propositions either only partly
include the subject (I type) or only partly Some M are P Some M are P
exclude the subject (O type). [All P are M]
Now, we can summarise the four types Some M are not P (O type):
of propositions to be used while solving
the problems of syllogism : P
Format Type M P or
M
All M are P A
No M are P E Some M are not P Some M are not P
Some M are P I [All P are M]
Some M are not P O
q Shortcut Approach HIDDEN PROPOSITIONS
All M are P (A type): (A) A type:

P Apart from ‘all’ it starts with every,


or M, P each and any.
M
EXAMPLE

No M are P (E type): Every girl is beautiful.


[All girls are beautiful.]
(i) A positive sen tence with a
M P
particular person as its subject is
A type.
Syllogism 123
“Is there any truth left in the
He deserves Bharat Ratna world”
[No truth is left in the world.]
Subject Predicate (C) I type:
Apart from some it also starts with
Amitabh Bacchan is a great actor.
words such as often, frequently,
almost, generally, mostly, a few,
Subject Predicate most etc.
(ii) A sentence in with a definite EXAMPLE
exception is A type : (i) Almost all the girls are beautiful.
definite exception [Some girls are beautiful].
(ii) Most of the garments are
handmade.
“All girls except Reeta are healthy.”
[Some of the garments are
handmade].
(B) E type: It is clear from the above examples
Apart from ‘no’ this type of that negative sentences begining
propositions starts from ‘no one’, with words like ‘few’, ‘rarely’,
‘seldom’, etc. (Also ‘hardly’,
‘none’, ‘not a single’ etc. ‘scarcely’, ‘little’ etc.) are to be
EXAMPLE reduced to I type.
Just see the other formates given
No one (student) is studious.
[No student is studious] below
(i) A negative sen tence with a
particular person as its subject is Not a definite exception as name of
E type propoistion. girls are not given.
He does not deserve Bharat Ratna
All girls except a few are beautiful.
Subject Predicate
[Some girls are beautiful]
Amitabh Bacchan is not a great actor.
Not a definite exception as name of
Subject Predicate girls are not given.
(ii) Sentences in following formats are
E type :
All girls except 5 have passed
definite
exception [Some girls have passed]
Therefore, a positive proposition with
"No student except Reena an indefinite exception is reduced to I
has failed" type.
EBD_7420
124 Syllogism
(D) O type : Therefore, a negative proposition with
Apart from “Some ....... not’ this an indefinite exception, is reduced to O
type of statements start with words type.
like ‘all’, ‘every’, ‘any’, ‘each’, etc.
EXCLUSIVE PROPOSITIONS
EXAMPLE Such propositions start with ‘only’,
(i) All girls are not beautiful. ‘alone’, ‘none else but’, ‘none but’ etc.
[Some girls are not beautiful] and they can be reduced to either A or E
(ii) Poor are usually not healthy. or I format.
[Some poor are not healthy] EXAMPLE
Now, it is clear from the above mentioned Only graduates are Probationary
examples that negative propositions with Officers.
words such as ‘almost’, ‘frequently’, Þ No graduate is Probationary
‘most’, ‘mostly’, ‘a few’, generally, etc. Officer (E type)
ar e to be reduced to th e O–type Þ All Probationary Officers are
propositions. graduates. (A type)
Again, positive propositions starting Þ Some graduates are Probationary
with words like ‘few’, ‘scarcely’, ‘rarely’, Officers (I type)
‘little’, ‘seldom’ etc. are said to be O– General format of sentences given in the
type. examinations :
All M are P (A type)
EXAMPLE
No M are P (E type)
Seldom are women jealous.
Some M are P (I type)
[Some women are not jealous]
Also, see the following formates : Some M are not P (O type)
Note : General format given above
are frequently asked formats in
No definite exception as name of the examinations. But students must
girls are not given. be ready for other hidden formates
of A, E, I and O types of propositions
No girls except three are beautiful. as problems in hidden formates
can also be given in question
papers.
[Some girls are not beautiful.]
CONVERSION OF
No definite exception as name of PROPOSITIONS
women are not given.
Before solving the pr oblems of
syllogism it is must to know the
No women except a few are housewife. conversion rules of all A, E, O, and I
types of propositions :
Syllogism 125
(i) Conversion of A type: (iv) Conversion of O type:

Subject Predicate O type of proposition can’t be


converted.
Note : In each conversion, subject
“All M are P ” (A type)
becomes predicate and predicate
becomes subject.
After conversion it becomes.
In fact, conversion is an immediate
Subject Predicate
inference that is drawn from a single
proposition while inference drawn from
“Some P are M ” (I type) two propositions are called mediate
Therefore, it is clear that A type of inference.
propositions get converted into I
type.
q Shortcut Approach
(ii) Conversion of E type :
Table of conversion :
Subject Predicate Type of Ge t conve rte d into
proposition
“No M are P ”(E type) A I
E E
After conversion it becomes I I

Subject Predicate O Never get converted


Rule to draw conclusion :
“ No P are M ” (E type) After knowin g con ver sion of
propositions, we must learn the rules
Therefore, E gets converted into E. to draw conclusions. In problems of
(iii)Conversion of I type : syllogism, conclusions are drawn
either from single propositions or
Subject Predicate from two proposition or from both.
But a concl usion from sin gl e
“Some M are P ” (I type) proposition is just a conversion of
t ha t proposit ion wh il e t o get
After conversion it becomes conclusion from two propositions a
certain table is used that tells us what
Subject Predicate type of conclusion (i n form of
proposition) we get out of t wo
“Some P are M ” (I type) propositions. To understand it, let us
see the following conclusion table :
Therefore, I gets converted into I.
EBD_7420
126 Syllogism
Conclusion Table
I - Proposition II - Proposition Conclusion
A A A
A E E
E A (O)R
E I (O)R
I A I
I E O

Note : EXAMPLE
(a) Apart from above 6 pairs of Statements :
propositions, no other pair will
give any conclusion. I. No pen is chair..
(b) The conclusion drawn out of two
propositions is itself a proposition II. Some tables are pen .
and its subject is the subject of
the Ist statement while its EXAMPLE
predicate is the predicate of the
Statements :
2nd statement. The common term
get disappeared. I. Some women are men .
(c) (O) R does mean that the
conclusion is O type but is in II. No men is chair..
reverse order. In this case, the
subject of the inference or In all the above mentioned example, we
conclusion is the predicate of the notice that in two statements of every
2nd proposition and the predicate example, there is a common term. In
of the conclusion is the subject of example 1 the word ‘girl’ is common; in
the Ist sentence or statement. example 2 the word ‘pen’ is common
(d) The conclusion table gives while in example 3 the word ‘men’ is
correct conclusions or inference common.
if and only if the two propositions
Now, the aligning of the two statements
are aligned properly.
(propositions) does mean that the pair
WHAT IS ALIGNING ? of statements must be written in such a
way that the common term is the
Let us see the following examples :
predicate of the 1st sentence and the
EXAMPLE subject of the 2nd.
Statements : Just think over the following examples :
Statements :
I. All girls are beautiful.
I. Some girls are cute .
II. Some girls are Indian. II. All cute are tall.
Syllogism 127
Here, the common term cute is the METHODS:
predicate of the I statement and subject (1) By Analytical Method
of the 2nd statement. Therefore, the two (2) By Venn Diagram
statements (I & II) are properly aligned. (1) Analytical method :
But see another example. This method has two main steps:
Statements : (a) Aligning the pair of sentences.
I. Some bats are chairs. (b) Using conclusion table to
draw conclusion.
II. Some cats are bats .
EXAMPLE Statements :
Here, the sentences are not aligned as
the predicate of the 1st statement is not I. All rats are cats.
the subject of the 2nd. II. All rats are men.
Then how to align it ? In such type of When aligned it takes the form as
cases we change the order of sentences. I. Some cats are rats [I type]
In another words we put I sentence in
place of II and II in place of I : II. All rats are men [A type]
II. Some cats are bats . Now we use the conclusion table
given in this chapter that says
I. Some bats are chairs. I + A = I type of conclusion.
Therefore, as per the requirement and Therefore, the drawn conclusion
nature of the sentence the alignment is must be
done. “Some cats are men”
(i) only by changing the order It is clear that the conclusion drawn
of sentences. “Some cats are men” is a mediate
or inference as it is the result of two
(ii) only by converting the propositions. But in actual problem
sentences. immediate inferences are also given in
or conclusion part and that format is given
below :
(iii) By changing the order of the
statements and then EXAMPLE : Statements:
converting on e of the I. All rats are cats.
sentences. II. All rats are men.
IEA Rule Conclusion:
(i) Some cats are men.
Alignment must be done in IEA order. It (ii) Some men are cats.
does mean that if the two statements are (iii) Some rats are cats.
I & E then the conversion must be done (iv) Some cats are rats.
for I and for E & I it will be done for E. (v) Some rats are men.
After discussing all the minute things (vi) Some men are rats.
about this chapter, now we have come Here, all the options are correct.
at the position of solving the problems conclusion (i) follows because it is the
of syllogism. mediate inference of statements I & II.
EBD_7420
128 Syllogism
Conclusion (ii) is the conversion of METHOD TO SOLVE
conclusion (i), conclusion (iii) is the (a) 1st step is sketching all possible
immediate inference (conversion) of pictorial representation for the
statement I while conclusion (iv) is the statements separately.
conversion of conclusion (iii). (b) 2nd step is combining possible
Conclusion (v) is the immediate inference pairs of these representations of
(conversion) of statement II while all the statements into one.
conclusion (vi) is the conversion of (c) 3rd and final step is making
conclusion (v). interpretation of this combined
Further, in some problems figure.
complementary pairs are also seen in the Conclusions are true if they are
conclusion part in the forms of sentence supported by all the combined
figures in 2nd step.
given below:
(a) (i) Some cats are rats. EXAMPLE
(ii) Some cats are not rats I-O pair
Statements :
(b) (i) All cats are rats.
(ii) Some cats are not rats. A-O pair A. All chairs are books.
(c) (i) Some cats are rats. B. All books are ties.
(ii) No cats are rats. I-E pair Conclusions :
Apart from I - O, A - O and I - E pair, the I. Some ties are books.
two sentences must have some subject II. Some ties are chairs.
and predicates as are the above 1st Step :
mentioned pairs. For these pairs we write
the form 'Either (i) or (ii) follows.
METHOD TO SOLVE b c t b
(a) First step is aligning the
sentences.
(b) Second step is using conclusion 1A 1B
table.
(c) Third step is checking immediate
inferences.
(d) Fourth step is checking through c, b b, t
the conversion of immediate
inferences & mediate inferences.
(e) First step is checking the 2A
2B
complementary pairs. Here, 1A and 2A are representations for
(2) Venn diagram method for solving statement A while 1B and 2B are
problems : representations for statement B. In these
Students will have to adopt three representations
steps to solve the syllogism b = books
problems through Venn diagram c = chairs
method : t = ties
Syllogism 129
2nd step : POSSIBILITY
Let us combine all the possible pairs of
this pictorial representations : Generally, the meaning of possibility is
probability, viz. possibility exists where
nothing is certain between the objects.
b c t
Let's understand below table in which
possibility exists where no definite
relation occurs between the objects and
(1A + 1B) definite or proper relation between the
objects eliminate existance of any
possibility. In simple way given
b c t condition eliminates the possibility and
improper condition favours the
possibility. Here, we can go through with
(1A + 2B) an example which will also clear the term
possibility.
t c, b Condition Possibility
Given facts cannot be determined
Imaginary facts can be determined
(2A + 1B)
EXAMPLE
Statements Some boxes are trees
c, b t
Some trees are hens.
Conclusions
I. Some boxes being hens is a
(2A + 2B) possibility
3rd step :
II. All trees being hens is a
When we interpret the pictures in step
possibility
II, we find that all the pictures support
both the conclusions. Therefore,
conclusion I : Boxes
“Some ties are books” and Trees
conclusion II. Hens
“Some ties are chairs” Hens
both are true.
Note : In the Venn diagram method,
any conclusion given with any In Conclusion I, before deciding the
problem will be true if and only if it is possibility between boxes and hens,
supported by all the combined we must notice the relation between
pictorial representations through 2nd both, we find that there is no relation
step. If any pictorial representation between boxes and hens, so possibility
contradicts the given conclusion, it exists and the conclusion I is true for
will be put in the category of incorrect possibility. Now in conclusion II we
or wrong conclusion.
must notice the relation between trees
EBD_7420
130 Syllogism
and hens. We find that both have some type of relation between them so the
possibility of ‘All between trees and hens is true. Hence, both the Conclusions
I and II follow.
q Shortcut Approach

Desired
Given Exclus ive Propos ition Pos s ibility
Propos ition
A ll A ll ´
Some So me ´
No No ´
No Some no t ´
Some A ll ü
No prop er relation Some / A ll ü

SPECIAL CASES OF EXCLUSIVE PROPOSITION

Meaningful
If the statement is of Conversion Illustration
Conversion
Much, more, many, Some Most A are B. Some A and B.
very, A few X are Y. Some X or Y.
a few, most, almost
Atleast Some Atleast some A are B. Some A and B.
Definitely No use Some A are definitely B. Some A are B.
Some X are definitely not Y. Some X are not Y.

Only Only A are B. All B are A.


1% to 99% Some 38% A are B. Some A are B.
98% X are Y. Some X are Y.
Syllogism 131

PRACTICE EXERCISE
DIRECTIONS (Qs. 1-3): In each of the 3. Statements:
following questions two/three All chairs are tables.
statements are given and these All tables are bottles.
statements are followed by two/three/ Some bottles are jars.
four conclusions. You have to take the No jar is bucket.
given statements to be true even if they Conclusions :
seem to be at variance from commonly I. Some tables being jar is a
known facts. Read the conclusions and possibility.
II. Some bottles are chairs.
then decide which of the given
III. Some bottles are not bucket.
conclusions logically follows from the
(a) Only I follows
two given statements, disregarding
(b) I and II follow
commonly known facts.
(c) All follow
1. Statements : (d) Only II follows
All petals are flowers. (e) None of these
Some flowers are buds.
DIRECTIONS (Qs. 4-5) : In each
Some buds are leaves.
questions below are given three
All leaves are plants.
statements followed by three
Conclusions:
I. Some petals are not buds. Conclusions I, II and III. You have to
II. Some flowers are plants. take the given statements to be true even
III. No flower is plant. if they seem to be at variance from
(a) Only I follows commonly known facts. Read all the
(b) Either II or III follows conclusions and then decide which of
(c) I and II follow the given conclusion logically follows
(d) Only III follows from the given statements disregarding
(e) None of the above commonly known facts.
2. Statements: 4. Statements:
Some apartments are flats. Some nurses are doctors.
Some flats are buildings. All doctors are medicines.
All buildings are bungalows. Some medicines tablets.
All bungalows are gardens. Conclusions:
Conclusions: I. Atleast some tablets are
I. All apartments being building doctors.
is a possibility. II. Some medicine being doctors
II. All bungalows are not is a possibility.
buildings III. Some medicine are definitely
III. No flat is garden. nurses.
(a) None follows (a) All follow
(b) Only I follows (b) II and III follow
(c) Either I or III follows (c) Only II follow
(d) II and III follow (d) Either III or Iv follows
(e) Only II follows (e) None of these
EBD_7420
132 Syllogism
5. Statements: 7. Conclusion: No play is field. Some
All files are folders. field is green.
All folders are boxes. Statements:
All boxes are drawers.
(a) Some play is green. Some
Conclusions:
green is field. All field is blue.
I. All folders being drawers is a
Some blue is virus.
possibility.
II. All boxes are files. (b) Some play is green. No green
III. All files are definitely is field. Some field is blue. All
drawers. blue is virus.
IV. Atleast some drawers are (c) None of these
folders. (d) All play is virus. No virus is
(a) I and II follow field. Some field is blue. All
(b) III and IV follow blue is green.
(c) II and III follow
(e) All play is green. No green is
(d) All follows
field. Some field is virus. All
(e) None of these
virus is blue.
DIRECTIONS (Qs. 6-7): In each of the
DIRECTIONS (Qs. 8-9): Question
questions below are the two conclusions
consists of five statements followed by
followed by four statements numbered a,
five conclusions. Consider the given
b, c & d , you have to take the given
statements to be true even if they seem
statements to be true even if they seem
to be at variance with commonly known
to be at variance from commonly known
facts. Read all the conclusions and then
facts. Read the answers and then decide
decide which of the given conclusions
which of the given statements definitely
does not logically follow from the given
follows from the given conclusions
statements using all statements
6. Conclusion: Some boy is good. together.
Some good is milk.
Statements: 8. Statements: All violet is orange.
(a) Some boy is book. All book Some orange is time. Some time is
is good. Some good is text. women. No women are boy. Some
Some text is milk. boy is apple.
(b) All boy is good. Some good Conclusions:
is text. All text is book. Some (a) All violet being women is a
book is milk. possibility.
(c) All boy is text. All text is (b) Some time is not boy.
good. Some good is book. All
(c) Some violet is women.
book is milk.
(d) All book is boy. Some boy is (d) All women being orange is a
text. Some text is good. All possibility.
good is milk. (e) All time being violet is a
(e) None of these possibility.
Syllogism 133
9. Statements: All wild are the animal. together in a specific order. Choose the
Some animal is the forest. No forest options that indicates a combination
is fire. Some fire is end. No end is a where the third statement can be
life. logically deduced from the first two
Conclusions: statements and that option will be your
(a) All animal being life is a answer.
possibility. 10. i. Some X is Y.
(b) All wild being the forest is a ii. All Y is Z.
possibility. iii. No Z is W
(c) All forest being life is a iv. Some W are Y
possibility.
v. All Y are T.
(d) All end being forest is a
vi. Some T are X.
possibility.
(e) All forest being end is a (a) [ii, iii, iv]
possibility. (b) [vi, i, v]
DIRECTION (Q. 10): Each of the (c) [iv, ii, iii]
following question consists of six (d) [iv, iii, ii]
statements followed by options (e) None is correct
consisting of three statements put
EBD_7420
134 Syllogism

HINTS & SOLUTIONS


1. (b) According to question, Conclusions:I. True
Conclusions:II.False
Conclusions:III.False
Hence, only conclusion I
Petals Buds Leaves follows.
Flowers Plants 3. (c) According to question,
or

Plants
Chair
Flowers
Tables Jar Bucket
Leaves Bottles

Buds OR
Conclusions:I. False
Conclusions:II.False
Conclusions:III.False or
Hence, only either II or III Jar
follows. Chair
2. (b) According to question,
Tables
Bottles
Flats
Bucket
Apartments Building

Bungalow Conclusions:I.True
Gardens Conclusions:II.True
Conclusions:III.True
Or Hence, All I, II and III follow.

Flats

Apartments Building

Bungalow
Gardens
Syllogism 135
4. (e) According to given information

Nurse Doctor Tablets


Medicine
Hence, only Conclusions III follows.
5. (b) According to given information,

Files
Folder
Box
Drawers

Hence, Conclusions III and IV follow.


Good
6. (c) Milk
Text

Book
Boy

7. (d) Green
Virus

Play Field Blue

8. (c)
Orange
Women Boy Apple
Violet Time

9. (d)
Animal
Fire End Life
Wild Forest

10. (e) There is no option in which third statement is conclusion of first two
statements.
EBD_7420
136 Cube & Dice

Chapter
Cube & Dice
16
CUBE EH, CH, GH, DG and FG.
Introduction Faces of the cube are ABCD,
A cube is three dimensional object EFGH, CDGH, BCHE, ABEF and
whose length, breadth and height are ADFG.
equal and any two adjacent faces are When a cube is painted on all of
inclined to each other at 90°. It has 6 its faces with any colour and
faces, 8 corners and 12 edges. further divided into various
smaller cubes of equal size, we
G H
get following results :
D C (i) Smaller cubes with no face painted
will present inside faces of the
undivided cube.
E
F (ii) Smaller cubes with one face painted
will present on the faces (except
A B
edges) of the undivided cube.
• Corners of the cube are A, (iii) Smaller cubes with two faces
painted will present on the edges
B, C, D, E, F, G and H.
(except corner) of undivided cube.
• Edges of the cube are AB, (iv) Smaller cubes with three faces
painted will present on the corners
BE, EF, AF, AD, CD, BC,
of the undivided cube.

Cube with Cube with


two sides three sides
painted painted

Cube with
one side
painted

The above figure may be analysed by dividing it into three horizontal layers :
Cube & Dice 137
Layer I or top layer :
The central cube has only one face coloured, four cubes at the corner have
three faces coloured and the remaining 4 cubes have two faces coloured.
Top layer

Bottom unpainted
Layer II or middle layer :
The central cube has no face coloured, the four cubes at the corner have two
faces coloured and the remaining 4 cubes have only one face coloured.
Middle layer Top unpainted

Unpainted cube

Bottom unpainted

Layer III or bottom layer :


The central cube has only one face coloured, four cubes at the corner have
three faces coloured and the remaining 4 cubes have two faces coloured.
Bottom layer

Bottom painted

Also, number of divisions on the faces of cube,


Length of the edge of undivided cube
n = .
Length of the edge of one smaller cube

q Shortcut Approach

Æ Total number of smaller cubes = (n)3


Æ Number of smaller cubes with no face painted = (n – 2)3
Æ Number of smaller cubes with one face painted = (n – 2)3 × 6
Æ Number of smaller cubes with two faces painted = (n – 2) × 12
Æ Number of smaller cubes with three faces painted = 8
EBD_7420
138 Cube & Dice

EXAMPLE 1. A cube is painted blue DICE


on all faces is cut into 125 cubes of Introduction
equal size. Now, answer the following
A dice is three-dimensional object with
question :
6 surfaces. It may be in the form of a
How many cubes are not painted on any
face? cube or a cuboid.
Sol. Since, there are 125 smaller cubes
of equal size, therefore,
n = number of divisions on the face
of undivided cube = 5.
Number of cubes with no face
painted = (n – 2)3
= (5 – 2)3 = 27

TYPES OF DICE

ORDINARY STANDARD
DICE DICE

1. Ordinary Dice:
In this type of dice, the sum of numbers on opposite faces is not 7 but the sum
of numbers on two adjacent sides are seven.

3
5
4

Ordinary Dice

4+3 = 7

2. Standard Dice:
In such type of dice, the sum of numbers on opposite faces is 7 or sum of
numbers on adjacent faces is not 7.
Cube & Dice 139

Here,
3 1 1+4 =5
5 4 4+5=9
4 5 1+5=6

Ordinary Dice Opposite of 1 ........6 (since 1+6 =7)


Opposite of 5 ........2 (since 5+2 =7)
Opposite of 3 ........4 (since 3+4 =7)

IMPORTANT RULES Hence,


2 and 1 are opposite
When Two Positions of a Single
3 and 4 are opposite
Dice are Given
5 and 6 are opposite
Case-I: Digits are different in both
Case III: When one digit is common in
position as follow.
both positions but not at same face as
1 follows
2 3
2 4
(ii) 1 4 6 3

In such case any digit in position (i) can (ii)


be opposite to any of the three digits in
position (ii) and vice-versa as given In this case list the numbers on
below: both the dices in either clockwise
1 Can be opposite to 4, 5 or 6 or anti-clockwise starting from
2 Can be opposite to 4, 5 or 6 common digit, on comparing the
3 Can be opposite to 4, 5 or 6 numbers obtained from both dices
4 Can be opposite to 1, 2 or 3 will give you the digit on opposite
5 Can be opposite to 1, 2 or 3 faces on two position.
6 Can be opposite to 1, 2 or 3 In the above figures, number 4 is
Case II: When one digit is common in common in both positions, Now
both position and at the same face as writting the digits in both position
follows. in clock-wise starting from 4, we
2 2 get
3 5 4 6 4 1 2 [Position (i)]

(ii)
4 3 6 [Position (ii)]
In this case, except the common Thus 1 is opposite to 3
digit, the digits on the other faces 2 is opposite to 6
are opposite to each other and the
face opposite to the common digit and 4 is opposite to 5
will have that digit which is invisible.
EBD_7420
140 Cube & Dice
Note that in this case digit opposite to
Top
common digit is the digit which are not
seen (invisible) in the two positions. (iv) Back Left Front Right
Case IV: When two digits are common Bott om
in both the positions as follows.
Opposite Faces of Unfolded Dice
1 4
4 2 2 6
• Top face and Bottom face are
opposite faces
(ii) • Left face and Righ t face ar e
In this case, opposite faces
(i) There is a probability of coming • Front face and Back face are
digits on the faces opposite to the opposite faces
faces having common digits are How to Fill Unfolded Faces with
invisible. Digits ?
(ii) Uncommon digits in each dice are
When the unfolded faces of any
opposite to each other. Hence
dice is filled up with digit, then any
3 is opposite to 2 or 4
question based on dice can be
5 is opposite to 2 or 4
solved very easily. Two positions
2 is opposite to 3 or 5
of a dice are given below and by
4 is opposite to 3 or 5
the help of these dice, the unfolded
1 is opposite to 6
faces have been filled up by the digits.
UNFOLDED DICE 1 6
When a dice is unfolded, then the 4 2 3 4
following four presentation of the
unfolded dice can be possible. (ii)
Top Now we will fill faces of the
following unfolded dice with digits
(i) Left Front Right with the help of the two positions
Bottom of a dice shown below
Back

Back
Top

(ii) Left Front Right


Bottom

(i) Now, write the common digit (4) at


Top
the front face.
(iii) Left Front Right Back
Bottom
Cube & Dice 141
Dice Formation
A Dice is formed by folding a sheet of
4
paper. These forms may be in the
following form.

Form 1:
(ii) Now going anti-clockwise direction 1
through the digits from th e 2 3 4
common digit 4 in position (i). We
get 4, 2 and 1 in order. Write 2 and 5
1 in unfolded dice in anti-clockwise 6
direction as shown in the figure.

1 Number 1 is opposite to 5.
1 Number 2 is opposite to 4.
4 2
4 2 Number 3 is opposite to 6.

Form 2:
1 2
(iii) Now going anti-clockwise direction
through the digits from th e 3
common digits 4 in position (ii). We 4
get 4, 6 and 3 inorder. Write 6 and 3
5 6
in unfolded in anti-clockwise
direction as shown in the figure.
Number 1 is opposite to 6.
1 Number 2 is opposite to 4.
6
6 4 2 Number 3 is opposite to 5.
3 4
3
Form 3:
1

(iv) Fill the remaining blank face with 2


the hidden digit 5 3 4
5
1
6
6 4 2
3
Number 1 is opposite to 3.
5 Number 2 is opposite to 5.
Number 4 is opposite to 6.
EBD_7420
142 Cube & Dice

Form 4: Form 5:
1 1
2 3 2 3
4 4 5
5 6 6

Number 1 is opposite to 4. Number 1 is opposite to 4.


Number 2 is opposite to 6. Number 2 is opposite to 5.
Number 3 is opposite to 5. Number 3 is opposite to 6.
Cube & Dice 143

PRACTICE EXERCISE
1. A dice is thrown four times and its four different positions are given below.
Find the number on the face opposite the face showing 2.

2 1 3 6
3 4 2 3 5 4 2 4

(a) 4 (b) 5
(c) 6 (d) 3
2. Four different positions of dice are as shown below. What number is opposite
to face 3?

2 5 6 3
3 6 2 4 3 1 5 1

(a) 4 (b) 3
(c) 2 (d) 6
3. Which one of the following box can be created by folding the given key design?
Question Figure :

Answer Figures :

(a) (b) (c) (d)


4. Which of the following cubes can be created by folding the given figure?
Question Figure.

F
AE
B
CD
EBD_7420
144 Cube & Dice
Answer Figures.

F F F D
E C A
B E D B E

(a) (b) (c) (d)


5. Two positions of a dice are given. Which number would be at the top when
bottom is 2?

3 3
2 5 6 1

(a) 4 (b) 1
(c) 5 (d) 6
6. Which one of the four boxes given below is created by folding the given key
design in the question figure?
Question figures:

Answer figures:

(a) (b) (c) (d)

7. Two positions of a dice are shown below. If 1 is at the bottom, which number
will be on top?

(a) 4 (b) 3 (c) 8 (d) 5


Cube & Dice 145
8.

The solid so formed by joining unit cubes is rotated to obtain different


positions, which of these cannot be the shape after it has turned?

(a) (b)

(c) (d)

9. The figure given on the left hand side is folded to form a box. Choose from the
alternatives (1), (2), (3) and (4) the boxes that is similar to the box formed.

(1) (2) (3) (4)

(a) (2) and (3) only (b) (1), (3) and (4) only
(c) (2) and (4) only (d) (1) and (4) only
10. In the given cubes, which colour is opposite to purple?

Purple Blue Red Green


Red Red Purple Purple
Blue Yellow Green Orange

(a) Violet (b) Red


(c) Yellow (d) Blue
EBD_7420
146 Cube & Dice

HINTS & SOLUTIONS


1. (b) 1, 3, 4 and 6 are adjacent 2. 7. (b) By looking, the dice position,
Therefore number 5 on the we can say that 2, 4, 5 and 6
face opposite to 2. are adjacent faces of 3.
2. (a) The numbers 1, 2, 5 and 6 are therefore, if 1 number is at the
on the adjacent faces of the bottom then 3 will be on the
number 3. So, the number 4 top.
lies opposite 3. 8. (a) Option (a) is correct.
3. (d) The shaded parts are narow. 9. (b) The given figure can be
So, answer figure (b) is invalid. numbered like this:
The white part is larger. So,
answer figure (c) is invalid. 1
If dot is on the top surface, 2
then the visible surface can 3 4
not be white. So, answer 5
figure (a) is invalid. 6
4. (b) When folded in the form of a
cube, then 'F' appears In this figure:
opposite 'B', 'E' appears 1 lies opposite 3; 2 lies
opposite 'C' and 'A' appears opposite 5;
opposite ‘D’.
4 lies opposite 6.
In option (a) 'F' is adjacent to
'B' When the sheet of question
In option (c) 'E' is adjacent to figure is folded to form a
‘C’. cube, then the face bearing a
In option (d) 'E' is adjacent to dot lies opposite to one of
‘A’. the shaded faces.
5. (d) The numbers 1, 2, 5 and 6 are Therefore, figure (2) which
on the adjacent faces of has both the shaded faces
number 3. Therefore. the adjacent to the face bearing
number 4 lies opposite 3. the dot, cannot be formed.
The numbers 3, 4 and 6 can
Hence, the cubes shown in
not be on the faces opposite
figures (1), (3) and (4) can be
to 1. Therefore, 5 lies
opposite 1. formed.
Now, 2 lies opposite 6. 10. (c) Purple ® Red, Blue, Yellow,
Green, Orange
6. (c)
Green, Orange, Red, Blue are
adjacent to purple
Analytical Decision Making 147

Chapter Analytical Decision


17 Making
INTRODUCTION (ii) The candidate must have work
Analytical decision making is a process experience as a teacher or
programming experience of at least
in which a final outcome is derived by
2 years.
evaluating and analysing the given
(iii) The candidate must have a PG
information. In this chapter mainly two degree in computer application,
types of questions are covered. [MCA, M.Tech. or M.Sc.
(i) Eligibility Test and (ii) Passage (computer science)] with not less
based decision making. than 60% marks.
These questions are designed to (iv) Out of total 50 marks in the
judge the decision making interview, the candidate must
capability of the condidates. obtain 50%.
In first type of questions, condition (v) In the case when a candidate
regarding the selection or non- fulfils the above conditions, he/
she shall be appointed as senior
selection of a candidate along with
teacher.
the biodata are given based on
(vi) Has less than 60% but more than
which his eligibility has to be
50% marks in his/her PG degree in
decided. In second type of
computer application, he/she will
questions, a par agr aph of be appointed as junior teacher.
informations is given based on (vii) If the age is more than 28 years
which the question are to be but less than 32 years as on 1st
answered. November, 2013, the case may be
reffered to the GM of the institute.
FORMAT OF THE QUESTION On the basis of the above mentioned
OF ELIGIBILITY TEST conditions and information about each
of the candidates in the question below,
Example (Directions): Read carefuly the
you have to decide which of the
informations given below and answer the
following courses of action should be
questions based on it:
taken against each candidate. Point to
The following are the given conditions be noted that nothing extra will be
for the recruitment of a candidate as a assumed except the given information.
family member in a computer institute: The decision must be based only on the
(i) The candidate must be in the age data provided.
range of 23 years to 28 years as on Mark your answer:
1st November, 2013. (a) If the candidate is to be selected
as a Junior teacher
EBD_7420
148 Analytical Decision Making
(b) If the candidate is to be selected conditions because they are the
as a Senior teacher original conditions.
(c) If the case will be reffered to the Additional conditions: In the given
GM of the institute. question format, there are two more
(d) If the data are inadequate conditions apart from the basic
(e) If the candidate is not to be conditions and they are (vi) and (vii).
selected. point to be noted that (v) will not be on
additional condition as it does not talk
QUESTIONS of exceptions. In fact (v) is only a
1. Mukesh Verma was born on 31st totality of the four basic or original
July, 1985. He is an M.Tech. in conditions given in the question
computer engineering with 70% format.
marks. He has been working in an What is data inadequacy?
institution as a programmer for the
As one of the answer is given as ‘data
last 7 years.
inadequate’ we must be clear about what
2. Karishma Tiwari is MCA with
exactly does data inadequacy mean?
72% marks. Her date of birth is 14th
August, 1990. She has worked as When details given about any candidate
a computer teacher for 4 years. She provide no information as required by
has got 35 marks in interview. the basic conditions/additional
conditions then this would be the case
What You See in the given
of data inadequacy, For example, let us
Question Format?
see the first question given in the format.
In the given format you can see the
No information is given about what
following things:
marks have been obtained by Mukesh
(1) Informations about some
Verma in the interview. Hence, the data
candidates have been provided.
is inadequate here.
(2) Some conditions have been given
for candidates to fulfil in order to How to solve a given
get selected for a particular job/ problem?
post. In case of the given format, Let us consider the questions given in
four conditions have been given. the format and start one stepwise
(3) When a candidate fulfils all the process.
STEP I
criteria except some, then different
Write the name of the candidates in the
course of action has to be taken left side and then write the symbols (i, ii,
for him. iii, iv) of the basic conditions to the top
Some more things to right. Now, put the symbols of the
additional conditions (vi and vii) below
understand the symbols of that basic condition with
Basic conditions: In the given question which these might be related. For
format, there are four basic conditions example, (vi) is a condition about
(i), (ii), (iii) and (iv). They are called basic educational qualification and so, it is an
Analytical Decision Making 149
exception of (iii). Hence (vi) should be answer choice (e). The answer choice
written below (iii). Similarly, (vii) should (e), which says that the candidate is not
be written below (i). Now, after the to be selected, should be chosen when
completion of step I, the following format any one or more of the given conditions
will be prepared: (a), (b), (c) is violated. The answer choice
i ii iii iv (d), which tells that the data are
(vii) (vi) inadequate, should be chosen when no
information is given about any one or
Mukesh
1 more conditions (a), (b), (c).
Verma
How to examine data?
Karishma
2 After step II, you are required to read
Tiwari all the statements carefully. Just take
Brijesh each question one by one and compare
3
Shankar then with the given conditions. Examinees
Mansi are suggested to use following
4
Ranjan symbols while doing this comparision:
Subodh I If a basic condition is fulfilled mark
5
Saxena ‘ü’ sign below it.
II If a basic condition is violated and
Note that three more names are added it is not attached with an
additional condition then mark ‘x’
in the table only for better understanding.
sign below it.
NOTE : To differentiate between basic III If a basic condition is violated but
it is attached with an additional
conditions and additional conditions.
condition, then
The additional conditions have been
(A) Mark a ‘(×)’ sign below it if
encircled.
additional condition is also violated.
STEP II (B) Mark a ‘(ü)’ sign below it if
At the 2nd step just see the given answer additional condition is fulfilled.
choices carefully and decide which IV In case of unavailability of any
combination of the conditions leads to information about any condition,
which conclusion. If we see the given a mark '?' will be put below that
condition.
question format with serious eye, we find
After comparison, you can easily
that the following combination can be
examine the data.
formed.
i + ii + iii + iv ® b [Senior teacher] STEP III
vii + ii + iii + iv ® c [Case will be (i) One by one, read the questions
very carefully and compare the
reffered to GM]
facts given with the various
i + ii + vi + iv ® a [Junior teacher] condition.
When we have decided the above three (ii) Mark the appropriate sign or ‘ü’,
combination giving answer choices (a), ‘×’ ,(ü), (×) or ? as required
(b) and (c), two answer choices remains (iii) When a ‘×’ or a (×) sign is obtained,
and they are answer choice (d) and then stop examining further and
EBD_7420
150 Analytical Decision Making
without any hesitation select the Condition (vii) is attached to (ii) while
answer choice “not to be selected” the additional condition is (vi) attached
for that particular question. It so with the basic condition (iii).
happens because, if a condition as well
as its additional condition is violated, STEP WISE EXPLANATION
it does mean that one necessary OF ABOVE TABLE
requirement is not being fulfilled.
Hence, we reach at a conclusion that Step I
the selection is not possible even if At the step I level, we read the
other conditions are fulfilled. question carefully and find out
STEP IV that there are four, basic conditions
Now, this is the time to select your (i), (ii), (iii) and (iv) and two
answer choices on the pattern additional conditions (vii) and (vi).
given below: Further, it is clear that (vii) is
(i) If find a ‘×’ or (×) below any attached to (i) and (vi) is attached
condition, go for the answer choice to (iii). Now we write the name of
“not to be selected” the candidates in extreme left and
(ii) If you find no cross mark but there then put the basic conditions (i),
is a question mark below any (ii), (iii) and (iv) at the top-right of
condition, your answer choice the candidate. Next, we write
would be “data are inadequate”. additional condition (vii) below (i)
(iii) If you find neither any cross and additional condition (vi)
mark nor any question mark, than
below (iii).
compare the combination with
Step II
the three answer combinations
obtained in step II and select the At the step II level, we look at the
answer choice accordingly. answer choices and prepare
After understanding the above answer combinations accordingly.
steps, now we are at a position of This will be:
solving the question given in the i + ii + iii + iv Þ b
question format. Let us see the vii + ii + iii + iv Þ c
solution: i + ii + vi + iv Þ a
Solution: Step III
Question No. (i) / (vii) (ii) (iii)/ (vi) (iv) At the step III level, we read every
1 Mukesh (ü) ü ü ? question carefully and compare the
Verma facts given in it with the various
2 Karishma ü ü ü ü conditions.
Tiwari
Let us see the detailed analysis of
3 Brijesh (×) ü (ü) ü
Shankar
every candidate question wise.
4 Mansi ü ü (ü) × Mukesh Verma
Ranjan He is an M.Tech in computer
5 Subodh ü ü (ü) ü engineering with 70% marks. This
Saxena
Analytical Decision Making 151
fulfills condition (iii). Hence we Step IV
write ‘ü’ mark below (iii). Next, his At the step IV level we select the
date of birth is 31st July, 1985. Here, answer choices.
we do a mental calculation that on Sol. 1. No cross mark Þ d But a
th
31st July, 2013 he turned 28 . This question mark is available. Hence,
is the reason that on 1st November data is inadequate.
2013, he is more than 28 years. Sol. 2. i + ii + iii + iv Þ b [step II]
Therefore, (i) is violated, but the So, the candidate is to be selected
additional condition of (i) is (vii) as a senior teacher.
which is fulfilled and we write (ü)
mark here. Further, Mukesh Verma q Shortcut Approach
is having a programming • For selection all basic conditions
experience of 7 years (more than 2 must be fulfilled.
years). So we mark ‘ü’ below (ii). • For rejection atleast one
Lastly, there is no information independent basic condition must
about marks of Mukesh in the be violated/basic and additional
interview. Thus the sign of condition must be violated.
question mark ‘?’ is put below (iv). • If a basic condition is violated but
Karishma Tiwari an additional condition attached
Karishma is an MCA with 72% with it is fulfilled and all other
marks. This fulfills (iii), so we put remaining basic conditions are
the mark ‘ü’ below (iii). Her date fulfilled, then the case will be
of birth is 14th August, 1990, So referred to the person given in the
on 1st November, 2013, she is questions.
more than 23 years but less than • Once the symbol ×/(×) is put in
28 years. This fulfills (i) and hence the table, there is no need to check
we put a ‘ü’ mark below (i). She is further conditions as person is
a computer teacher from last 4 declared rejected at this stage
years. This fulfils (ii) and hence only.
we put ‘ü’ mark below (ii). Lastly, • If for one basic condition, the data
she has obtained 35 marks in the is not given while all other basic
interview. This marks is more than conditions are fulfilled, it means
the required 50% (25 marks out of data is inadequate.
50 marks), therefore (iv) is also
• If any information is not given
fulfilled and we put ‘ü’mark
and answer choices don't have
below (iv).
data inadequate option, then
NOTE: Mark ‘ü’, ‘×’, ‘(ü)’, and ‘(×)’ condition related to that particular
against the three additional persons are information is supposed to be
just assumed for better understanding. violated.
EBD_7420
152 Analytical Decision Making
PASSAGE BASED DECISION II. Marathe should try and verify the
MAKING QUESTION facts from other sources as well.
III. Kale should be sacked.
DIRECTIONS (Qs. 1-3): Read the IV. Kale should be demoted.
following caselet carefully and answer V. Marathe should suggest Kale to
the questions that follow. visit a family counselor.
Marathe is a Vice-President in a Which of the following would be the
construction equipment company in the most appropriate sequence of decisions
in terms of immediacy starting from
city of Mumbai. One day, his subordinate
immediate to a longer term solution?
Bhonsle requested that Kale, a project
(a) II, I and V
manager, be transferred to the Chennai (b) I, IV and II
office from the Mumbai office. In Chennai, (c) II, III and IV
Kale would work alone as a researcher. (d) II, V and I
Bhonsle gave the following reasons for (e) II, V and IV
his request” Kale is known to frequently 2. Marathe sought an appointment
fight with his colleagues. ‘Kale is with Lakhote to find out ways to
conscientious and dedicated only when help Kale. Lakhote is of the opinion
working alone. He is friendly with seniors that the company’s responsibility
but refuses to work with colleagues, in a is restricted to the workplace and
team. He cannot accept criticism and feels it should not try to address the
personal problems of employees.
hostile and rejected. He is over bearing
If Marathe has to agree to
and is generally a bad influence on the
Lakhote’s opinion, which of the
team.’ solutions presented in th e
Marathe called upon Gore, another previous question would be
project manager and sought further weakened?
information on Kale. Gore recalled that (a) Only I (b) Only II
a former colleague, Lakhote (who was (c) Only III (d) Only IV
also Kale’s former boss) had made a few (e) Only V
remarks on his appraisal report about 3. Which of the following
Kale. In his opinion, Kale was not fit for statements, if true would weaken
further promotion as he was emotionally the decision to sack Kale the
unstable to work in groups though he
most?
had seven years of work experience.
(a) A Government of India study
Lakhote had described Kale as too
established that employees
authoritative to work under anyone.
with 5-10 year of work
Lakhote had further told Gore that Kale
experience tend to have
had an ailing wife and an old mother,
conflicting responsibilities at
who does not want to stay with his wife.
home and office. However,
1. Consider the following solutions
these conflicts wither away
to the problem mentioned above
after 10 year of experience
I. Marathe should transfer Kale to
Chennai office.
Analytical Decision Making 153
(b) Another article published in between Kale and his
the magazine, Xaviers colleague due to which he is
Quarterly, highlighted that having such bad raputation
employees’ problems at home in his office. Marathe should
affect their performance at not easily believe on Gore, he
work should verify it from other
(c) In the latest issue of a reputed sources.
journal, Xaviers Business After collecting the
Review, it was published that infromation about Kale, he
most top managers, find it should be suggested to visit
difficult to work in a group the family counselor which
(d) It was published in Xaviers might help him in improving
Management Review (another his personality. If still, he is
reputed journal) that indivi- not been able to adjust with
duals who cannot work in
his colleagues he should take
terms find it difficult to adjust
transfer to the Chennai
to a new location
office.
(e) Bhonsle was of the opinion
that emtionally unstable 2. Concerning it as a personal
persons, find it difficult to problem of employees,
get back to normal working Marathe left no right to
life interfere it.
Sol. 1. Marathe should try to verify 3. Bhosle was of the opinion
the facts from other sources that emotionally unstable
as well becasuse there might persons find it difficult to get
be some jealousy feeling back to normal working life.
EBD_7420
154 Analytical Decision Making

PRACTICE EXERCISE
DIRECTIONS (Qs. 1-5): Study the You have to indicate your decision by
following information carefully and marking answers to each question as
answer the questions given below: follows:
Give Answer:
An organization wants to recruit system
analysts. The following conditions (a) If the case is to be referred to VP
apply. (b) If the case is to be referred to GM
(c) If the data provided is not
The candidate must
sufficient to take a decision
(i) be an engineering graduate in
(d) If the candidate is to be selected
computer/IT with at least 60%
(e) If the candidate is not to be
marks.
selected
(ii) have working experience in the
1. Ms. Suneeta is an IT Engineer with
field of computer at least for 2 yr
60% marks at graduation as well
after acquiring the requisite
as in selection test. She is working
qualification.
as a Software Engineer for last 3 yr
(iii) have completed minimum 25 yr and
after completing engineering
maximum 30 yr of age as on
degree and has completed 27 yr of
1.12.2013.
age. She is willing to sign the bond
(iv) be willing to sign a bond for
of II ` 50000.
` 50000.
2. Rakesh Rao is a Computer Engineer
(v) have secured minimum 55% marks
Graduate and thereafter is working
in selection test. However, if a
as a Software Manager for last
candidate fulfils all other
6 yr. He has secured 72% marks at
conditions
graduation and 67% marks in
Except
selection test. His date of birth is
A. at (i) above, but is an Electronics 5th December, 1984. He is not
Engineer with 65% or more marks willing to sign the bond for ` 50000.
the case is to be referred to the 3. Ramkumar is an Engineering
General Manager (GM)-IT. graduate in computers with 78%
B. at (iv) above, but has an experience marks passed out in 2007 at the
of atleast 5 yr as a Software age of 23 yr. Since, then he is
Manager, the case is to be referred working as a Software Manager in
to the VP. an engineering firm. He doesn't
In each question below, detailed want to sign the bond for ` 50000.
information of candidate is given. You He has cleared the selection test
have to carefully study the information with 72% marks.
provided in each case and take one of 4. Nishant is an Electronics Engineer
the following courses of actions based passed out in June, 2010 at the age
on the information and the conditions of 22 yr. Since, then he is working
given above. You are not to assume as a Programmer in a software
anything other than the information company. He has passed th e
provided in each question. All these selection test with 66% marks and
cases are given to you as on 01.12.2013. is willing to sign the bond.
Analytical Decision Making 155
5. Kalyani is an Engineer with 72% In each question below is given details
marks in Telecommunication. She of one candidate. You have to take one
has just completed 27 yr of age. of the following courses of actions based
She has cleared the selection test on the information provided and the
with 59% marks. She is willing to conditions and subconditions given
sign the bond. above and mark your an swer
accordingly. You are not to assume
DIRECTIONS (Qs. 6-10): Study the anything other than the information
following information carefully and provided in each question. All these
answer the questions given below : cases are given to you as on 01.03.2013.
Following are the conditions for Mark answer (a) if the candidate is not
selecting Marketing Manager in an to be selected.
organisation : Mark answer (b) if the candidate is to be
The candidate must : selected.
(i) be at least 30 years old as on Mark answer (c) if the data are
01.03.2013 inadequate to take a decision.
(ii) have secured at least 55 per cent Mark answer (d) if the case is to be
marks in graduation referred to Vice President -Marketing.
(iii) have secured at least 60 per cent Mark answer (e) if the case is to be
marks in Post graduate Degree/ referred to GM-Marketing.
Diploma in Marketing. 6. Suresh Mehta has secured 58 per
(iv) have post qualification work cent marks in graduation. He was
experience of at least five years in born on 19th May 1979. He has
the Marketing Division of an secured 50 per cent marks in the
organisation. selection process. He has been
(v) have secured at least 45 per cent working for the past seven years
marks in the selection process. in the Marketing division of an
organ-isation after completing his
In the case of a candidate who satisfies
Post Graduation with 62 per cent
all other conditions except -
marks.
(A) at (iv) above, but has post
7. Sudha Gopalan has secured 50 per
qualification work experience of at
cent marks in both selec-tion
least two years as Deputy
process and graduation. She has
Marketing Manager, the case is to
been working for the past six years
be referred to GM-Marketing.
in the Marketing division of an
(B) at (ii) above, but has secured at organisation after completing her
least 65 per cent marks in Post Post Graduate Diploma in
graduate Degree/Diploma in Marketing with 70 per cent marks.
Marketing Management, the case She was born on 14th October.
is to be referred to Vice President- 1982.
Marketing.
EBD_7420
156 Analytical Decision Making
8. Divya Kohli has been working for for the past six years in the
the past five years in Marketing Marketing division of an
division of an organisation after organisation after com-pleting his
completing her Post Graduate PG Degree in Marketing. He has
Diploma in Marketing with 65 per secured 50 per cent marks in the
cent marks. She has secured 55 per selection process.
cent marks in graduation and 50 10. Varun Malhotra was born on 3rd
per cent marks in the selection July 1980. He has been working as
process. She was born on 2nd Deputy Marketing Manager in an
April 1979. organisation for the past three
9. Navin Marathe was born on 8th years after completing his Post
April 1979. He has secured 60 per Graduate Degree in Marketing with
cent marks in both graduation and 65 per cent marks. He secured 55
Post-Graduate Degree in per cent marks in both graduation
Marketing. He has been working and selection process.
Analytical Decision Making 157

HINTS & SOLUTIONS


Sol. (1-5):

Candidate i ii iii iv v (A) (B)


Suneeta P P P P P
Rakesh P P P – P P
Ram Kumar P P P – P P
Nishant – P P P P
Kalyani û P P P P

1. (d) Suneeta fulfils all conditions so, she is to be selected.


2. (a) Rakesh Rao fulfils condition (B) instead of (IV) so, his case is to be
referred to VP.
3. (a) Ramkumar fulfils condition (B) instead of (IV) so, his case is to be referred
to VP.
4. (c) Percentage marks of Nishant in graduation is not given so, data is
insufficient.
5. (e) Kalyani is telecommunication engineer so, she is not to be selected.
Sol. (6-10) :

CRITERIA
CANDIDATE
(i) (ii) or B (iii) (iv) or A (v) Ans
Suresh ü ü – ü ü – ü b
Sudha ü – ü ü ü – ü d
Divya û ü – ü ü – ü a
Navin ü ü – ü ü – ü b
Varun ü ü – ü – ü ü e

6. (b) Suresh Mehta satisfies all the conditions (i), (ii), (iii), (iv) and (v). Therefore,
he can be selected.
7. (d) Sudha Gopalan satisfies the conditions (i), (B), (iii), (iv) and (v). Therefore,
her case would be referred to Vice-president -Marketing.
8. (a) Divya Kohli does not satisfy condition (i). Therefore, she cannot be
selected.
9. (b) Navin Marathe satisfies all the conditions (i), (ii), (iii), (iv) and (v).
Therefore, he can be selected.
10. (e) Varun Malhotra satisfies the conditions (i), (ii), (iii), (A) and (v). Therefore,
his case should be referred to GM-Marketing.
EBD_7420
158 NON-VERBAL REASONING Series

Chapter

18 Series
INTRODUCTION · Positions of Elements –
Th e word “series” is defined as Top or Up middle
anything that follows or forms a specific element
pattern or is in continuation of a given Central
element
pattern or sequence. Upper left
A B C Upper right
In this type of non-verbal test, two sets element element
of figures pose the problem. The sets Middle left H I D Middle right
element element
are called Problem Figures and Answer G F E
Lower left Lower right
Figures. Each problem figure changes element element
in design from the preceding one. Bottom or Down
middle element
q Shortcut Approach · Movement of Elements Through
· Directions – There are eight Distance –
directions as follows :
Up
P Q R

N
NW NE W S
Left W E Right
V U T
SW SE
S
Clockwise Movement

Down 1
P®Q= arm/step
· Rotational Directions – 2
There are two rotational directions P ® R = 1 arm/step
as follows :
1
P®S=1 arm/step
2
P ® T = 2 arm/step
1
P®S=2 arm/step
2
Clockwise Anticlockwise P ® R = 3 arm/step
direction (CW) direction (ACW)
Series 159
1 1
P®Q=3 arm/step P®P=3 arm/step
2 2
· Directional Movement of Elements –
P Q R

P Q R
W S 45° 90°
360° 135°
W S
315° 180°
V U T
270° 225°
V U T
Anti Clockwise Movement
1 Clockwise Movement
P®W= arm/step
2
P ® V = 1 arm/step
P Q R
1
P ® U = 1 arm/step 360° 315°
2 45° 270°
W S
1 90° 225°
P ® U = 2 arm/step
2 135° 180°
P ® V = 3 arm/step V U T
1
P®W=3 arm/step Anticlockwise Movement
2

TYPES OF SERIES
TYPE-I
A definite relationship between elements in given figures.
EXAMPLE 1.
Study the problem figures marked (A), (B) and (C) carefully and try to
establish the relationship between them. From the answer figures marked
a, b, c and d, pick out the figure which most appropriately completes the
series.
Problem Figures

(A) (B) (C)


EBD_7420
160 Series
Answer Figures

(a) (b) (c) (d)


Sol. The direction of arrow which changes alternately. The dots are also changing
alternately. Hence, we are looking for a figure in which the arrow points down
and the dots and positioned as in figure (b).
TYPE II. ADDITIONS OF ELEMENTS :
In these type of questions, each figure is obtained by either sustaining the
element of preceding figure as it is or adding a part of element or one element or
more than one element of the preceding figure in a systematic way.
EXAMPLE 2.
Problem Figures

(A) 2
(B) (C) (D)
Answer Figures

(a) (b) (c) (d)


Sol. Two line segments are added in A to obtain B and one line segment is added in
B to obtain C. This process is repeated again to obtain D. Hence, answer figure
(d) continues the series.
TYPE III. INCREASING/DECREASING OF ELEMENTS:
In these questions, the items in the diagrams either increase or decrease in number.
EXAMPLE 3.
Problem Figures

(A) (B) (C)


Series 161
Answer Figures

(a) (b) (c) (d)


Sol. The small circles are decreasing consecutively and the black dots are
increasing. So, figure (c) continues the series.
TYPE IV DELETION OF ELEMENTS :
In these type of questions, each figure is obtained by either sustaining
the element of preceding figure as it is or deleting a part of an element or
one element or more than one element of the preceding figure in a
systematic way.

EXAMPLE 4.
Problem Figures

(A) (B) (C) (D) (E)


Answer Figures

(a) (b) (c) (d)


Sol. The qualitative characteristic of various elements in the diagrams change
to complete the series. So, figure (a) continues the series.
TYPE V ROTATION TYPE :
The various elements in the diagrams move in a specific manner. They
may rotate in clockwise or anti-clockwise direction.

EXAMPLE 5.
Problem Figures

+ +

+
(A) (B) (C)
EBD_7420
162 Series
Answer Figures

+ + +

(a) (b) (c) (d)


Sol. The sign of plus is rotating clockwise. The pin changes direction alternately.
So, figure (d) coninues the series.
TYPE VI REPLACEMENT OF ELEMENTS :
In these type of questions, each figure is obtained by either sustaining
the element of preceding figure as it is or replacing a part of element or
one element or more than one element by a new element of the preceding
figure in a systematic way.

EXAMPLE 6.
Problem Figures

­ ? * X D *
= X ? *
? X = * ? C D
2
(A) (B) (C) (D) (E)
Answer figures
* # C # * C
D D D D
# *
(a) (b) ( c) (d)
Sol. The elements positioned at north-east (NE) corners disappear from the odd-
numbered figures. The elements positioned at the south-west (SW) corners
disappear from the even-numbered figures. Therefore * should not appear in
the answer figure. Hence (a), (b) and (d) cannot be the answers. Also new
elements are introduced at the NE corners in even-numbered figures. Therefore,
answer figure (c) continues the given series.
Series 163

PRACTICE EXERCISE
1. Select a figure from amongst the Problem Figures:
Answer Figures which will
– T O
continue the same series as – TS
established by the five Problem S
Figures. (A) (B) (C) (D) (E)
Problem Figures:
Answer Figures:
S N L Z
O
O O –
(A) (B) (C) (D) (E)
(1) (2) (3) (4)
Answer Figures:
(a) 1 (b) 2
X O C S (c) 3 (d) 4
4. Select a figure from amongst the
Answer Figures which will continue
(1) (2) (3) (4) the same series as established by
(a) 1 (b) 2 the five Problem Figures.
(c) 3 (d) 4 Problem Figures:
2. Select a figure from amongst the
Answer Figures which will
continue the same series as
established by the five Problem (1) (2) (3) (4)
Figures. Answer Figures:
Problem Figures:

(A) (B) (C) (D) (E) (1) (2) (3) (4)


Answer Figures: (a) 1 (b) 2
(c) 3 (d) 4
5. Select a figure from amongst the
(4) Answer Figures which will
(1) (2) (3)
continue the same series as
(a) 1 (b) 2 established by the five Problem
(c) 3 (d) 4 Figures.
3. Select a figure from amongst the Problem Figures:
Answer Figures which will
continue the same series as
x

established by the five Problem x x


Figures. (A) (B) (C) (D) (E)
EBD_7420
164 Series
Answer Figures: Problem Figures:
x x
x

(1) (2) (3) (4) (A) (B) (C) (D) (E)

(a) 1 (b) 2 Answer Figures:


(c) 3 (d) 4
6. Select a figure from amongst the
Answer Figures which will continue (1) (2) (3) (4)
the same series as established by (a) 1 (b) 2
the five Problem Figures. (c) 3 (d) 4
Problem Figures: 9. Select a figure from amongst the
Answer Figures which will
continue the same series as
established by the five Problem
(A) (B) (C) (D) (E)
Figures.
Answer Figures: Problem Figures:
× ×× ×× ××
×
× ×
(1) (2) (3) (4) (A) (B) (C) (D) (E)
(a) 1 (b) 2 Answer Figures:
(c) 3 (d) 4
×× ×× ×× ××
7. Select a figure from amongst the × × × ×× ×
Answer Figures which will ×× × ×
continue the same series as (1) (2) (3) (4)
established by the five Problem (a) 1 (b) 2
Figures. (c) 3 (d) 4
Problem Figures: 10. Select a figure from amongst the
Answer Figures which will
continue the same series as
established by the five Problem
(A) (B) (C) (D) (E) Figures.
Answer Figures: Problem Figures:

(A) (B) (C) (D) (E)


(1) (2) (3) (4)
Answer Figures:
(a) 1 (b) 2
(c) 3 (d) 4
8. Select a figure from amongst the
Answer Figures which will continue (1) (2) (3) (4)
the same series as established by (a) 1 (b) 2
the five Problem Figures. (c) 3 (d) 4
Series 165

HINTS & SOLUTIONS


1. (c) In each step, element at the 5. (d) The cross line mover ACW 90°
upper-right position gets and 45° alternatively and the
enlarged, inverts vertically and pin arrow moves CW 90° and
reaches the lower-left corner; 45° alternatively.
the existing element at the 6. (c) The arrow is changing its
lower-left position, is lost and positions clock wise 90°, 45°,
a new small element appears at 135°, 45°, ....next should be 180°.
the upper-right position. So option (c) is correct answer.
7. (d) Small hand is moving anticlock
2. (a) wise 90°, 45°, 90°, 45°,... and Big
hand is moving clock wise 135°
The figure rotates sequentially constantly. So in the next
2, 1, 3, 1, 4… step in a CW (Clock figure, small hand must move
wise) direction. 90° anti clockwise, and big
3. (c) The symbol along with line lie hand must move 135°. So
option (d) is correct answer.
rotate 90° ACW (Anti clock
8. (c) Similar figure reappears in
wise) direction in each step,
every fourth step and each time
and the symbol interchange a figure reappears, it rotates
position in one step and are through 90°ACW.
replaced by new symbol in the 9. (c) In each step, one dot is lost
new step. while another dot is replaced
4. (b) The shading rotates through by a cross.
two and three steps 10. (c) All the three symbols in the
alternatively in CW (clock wise) dice are rotating clockwise.
direction. So option (c) is correct answer.
EBD_7420
166 Mirror & Water Images

Chapter

19 Mirror & Water Images


MIRROR IMAGES
INTRODUCTION
In this category, questions are based on
the criteria that a few figures are given
and you have to find out which one is
the exact image of the given figure in a Object Image
mirror placed in front of it. This image
formation is based on the principle of 3. The line joining the object point
with its image is normal to the
‘lateral inversion’ which implies that size
reflecting surface.
of the image is equal to the size of the
object but both sides are interchanged. 4. The size of the image is the same
The left portion of the object is seen on as that of the object.
the right side and right portion of the I. Mirror Images of Capital Letters
object is seen on the left side. For
example, mirror image of ABC =
A N
Note : There are ‘11’ letters in English
Alphabet which have identical mirror B O
images: A, H, I, M, O, T, U, V, W, X, Y. C P
D Q
Characteristics of Reflection by plane
mirror E R
1. Perpendicular distance of object F S
from mirror = Perpendicular G T
distance of image from mirror. H U
2. The image is laterally inverted.
I V
J W
K X
L Y
M Z
Mirror & Water Images 167
II. Mirror Images of Small Letters q Shortcut Approach
a Whenever you have to solve a mirror
n
image question, imagine a mirror placed
b o in front of the object and then try to
c p find its inverted image. The portion of
d q the object that is near the mirror will
e r now be the portion of the image near
to the mirror in the inverted form.
f s
g t EXAMPLE 1.
h u
By looking in a mirror, it appears
i v that it is 6 : 30 in the clock. What is
j w the real time ?
k x Sol. As,
l y
m z
Time = 6 : 30
III. Mirror Images of Numbers

0 6
(Fig A)
1 7
2 8
3 9
4 10 Time = 5 : 30
5
IV. Mirror Images of Clock:
(Fig B)
There are certain questions in Clearly, fig (A) shows the time
which the position of the hour- (6 : 30) in the clock as it appears in
hand and the minute-hand of a a mirror. Then its mirror-image i.e.
clock as seen in a mirror are given. Fig (B) shows the actual time in the
On the basis of the time indicated clock i.e. 5 : 30. You can solve it
quickly if you remember that the
by the mirror-image of the clock
sum of actual time and image time
we have to detect the actual time is always 12 hours.
in the clock. In the solution of
such questions we use the fact WATER IMAGES
that if an object A is the mirror- The reflection of an object as seen in
image of another object B then B water is called its water image. It is the
is the mirror-image of A. inverted image obtained by turning the
object upside down.
EBD_7420
168 Mirror & Water Images
Water-images of Capital Letters
Letters A B C D E F G H I J K L M
Water-image
Letters N O P Q R S T U V W X Y Z
Water-image
Water-images of Small Letters
Letters a b c d e f g h i j k l m
Water-image
Letters n o p q r s t u v w x y z
Water-image
Water-images of Numbers
Letters 0 1 2 3 4 5 6 7 8 9
Water-image

Note :
1. The letters whose water-images are identical to the letter itself are : C, D, E, H,
I, K, O, X
2. Certain words which have water-images identical to the word itself are :
KICK, KID, CHIDE, HIKE, CODE, CHICK

q Shortcut Approach
Whenever we have to analyze the water image of an object, imagine a mirror or a
surface that forms an image just under the given object. The portion of the object
that is near the water surface will be inverted but will be near the water surface in
the image as well.

EXAMPLE 2.
Find the correct option for the water images below:
STORE
water surface
?
Sol. In case of water image, the water reflection will usually be formed under the
object / word.
In this case, the water image of the word will be an outcome of the water
images of each of the letters like, the water images of S is , T is , O is ,
R is and E is . Thus, the water image of the word ‘STORE’ is ‘ .’
STORE
Mirror & Water Images 169
q Shortcut Approach
(i) While solving a question, try eliminating some options and solving the
questions will become easier. To eliminate options, keep in mind the pattern
used in the object (given diagram whose image is to be formed) as well as the
position of mirror or water such that the portion of the object near to the
mirror / water will produce the same portion near the mirror / water in an
inverted form.
(ii) Images are images, be it water or mirror, in both the cases an inverted image of
the alphabets / numerals / clocks / any other object are formed by inverting the
object. Inverting of the object solely depends upon the position of mirror or
water surface w.r.t. the object.
EBD_7420
170 Mirror & Water Images

PRACTICE EXERCISE
1. If the mirror is placed on the line
LM, then which of the answer
figures is the right image of the (c) (d)
given question figure?
Questions Figure :
DIRECTIONS (Q. 3) : If a mirror is
placed on the line MN, then which of
the answer figures is the right image of
the given figure ?
3. Question Figure :

N
L M
Answer Figures :
M

Answer Figures :
(a) (b)

(a) (b)

(c) (d)
(c) (d)

DIRECTIONS (Q. 2) : If a mirror is


placed on the line MN, then which of 4. Select the correct option that will
the answer figures is the correct image be the mirror reflection of the
of the given question figure ? problem figure.
Question Figure :
2. Question Figure:
N

M
Answer Figures:
f
Answer Figures :

(a) (b)
(a) (b) (c) (d)
Mirror & Water Images 171
5. If a mirror is placed on the line AB, 7. From the answer figures, find out
then which of the answer figures the figure which is the exact mirror
is the correct image of the given image of the question figure, when
question figure? the mirror is placed on the line MN.
Question Figure : Question Figure :
A M

B Answer Figures :
Answer Figures :

(a) (b)
(a) (b)

(c) (d)

(c) (d)
8. If a mirror is placed on the line MN,
then which of the answer figures
6. Choose the right water-image of is the right image of the given
the question figure from the given figure ?
answer figures. Question figure:
Question Figure : M
M N
APPROACH

N
Answer figures:
Answer Figure :

(a) (b)
(a) (b)

(c) (d)
(c) (d)
EBD_7420
172 Mirror & Water Images
9. Which one of the following is water Answer Figures :
image of COMMISSION"?
(a) COMMISSION

(b) COMMI (a) (b)


(c) CO
(d) COMMISSION
10. If a mirror is places on the line MN,
then which of the answer figures is (c) (d)
the right image of the given figure?
Question figure:
M

HINTS & SOLUTIONS


1. (a) In water image upside 7. (d)
becomes downside.

8. (c)
9. (c) Water image of

C O M M I S S I O N

2. (c)
3. (b) O I I O
4. (b) 10. (a)
5. (a) Option (a) is the correct mirror
image of given question M
figure.

6. (a) –– ––
–– ––
–– ––
–– –– M
N irror image
××× ×
××× ×
××× ×
××× ×
Paper Cutting and Folding 173

Chapter
Paper Cutting and
20 Folding
INTRODUCTION
In this section, a sheet of paper is folded in given manner and cuts are made on it. A
cut may be of verying designs. We have to analyze how this sheet of paper will look
when paper is unfolded.
Note that when a cut is made on folded paper, the designs of the cut will appear on
each fold.

EXAMPLE 1.

Directions In the following example, figures A and B show a sequence of folding a


square sheet. Figure C shows the manner in which folded paper has been cut. You
have to select the appropriate figure from alternatives which would appear when
sheet is opened.

(A) (B) (C )

(a) (b) (c) (d)


Sol. Step I-When sheet C is unfolded once, it will appear as follows

Step II -

Clearly, the circle will appear in each of the triangular quarters of the
paper. So, figure (c) would appear when sheet is opened.
EBD_7420
174 Paper Cutting and Folding
EXAMPLE 2. q Shortcut Approach
· Consider a mirror placed on the
dotted line facing the portion/part
which is to be folded and the
(A) (B) (C) mirror image thus obtained is
superimposed on the design of
the other side to get the folded
pattern.
(a) (b) (c) (d) · When more than one fold is made
Sol. Here, a circular cut is made on the before punching then virtually try
quarter circle. Hence, this sheet, to unfold each fold one by one
when completely unfolded, will and predict the complete unfolded
contain small circle on each quarter pattern.
and will appear as option (d).

PRACTICE EXERCISE
1. A piece of paper is folded and cut Question figure :
as shown below in the question
figures. From the given answer
figures, indicate how it will appear
when opened.
Question figure
Answer figures :

(a) (b)
(a) (b)

(c) (d)
(c) (d)
Answer figures
3. A sheet of paper has been folded
as shown by the question figures.
You have to figure out from
(a) (b) (c) (d)
amongst the four answer figures
2. A square sheet of paper has been how it will appear when opened?
folded and punched as shown in
the question figure. You have to Question figures :
figure out from amongst the four
answer figures, how it will appear
when opened?
Paper Cutting and Folding 175
Answer figures : Question Figures :

Answer Figures :
(a) (b) (c) (d)

4. A sheet of paper when folded,


punched and opened shows the
question figure. Choose from the
answer figures which punched (a) (b) (c)
hole pattern gives this figure. (d)
Question figure (Open pattern) : 7. A piece of paper is folded and cut
as shown below in the question
figures. From the given answer
figures, indicate how it will appear
when opened.
Question figure:

Answer figures (Punched hole


patterns) :

Answer figures:

A B C D
5. A piece of paper is folded and cut (a) (b)
as shown below in the question
figures. From the given answer
figures, indicate how it will appear
when opened.
Question Figures :
(c) (d)

8. A piece of paper is folded and cut


Answer Figures : as shown below in the question
figures. From the given answer
figures indicate how it will appear
when opened.
(a) (b) (c) (d) Question figure :
6. A piece of paper is folded and
punched as shown below in the
question figures. From the given
answer figures, indicate how it will
appear when opened.
EBD_7420
176 Paper Cutting and Folding
Answer figures :

(a) (b)
(a) (b)

(c) (d)

(c) (d) 10. A piece of paper is folded and


punched as shown below in the
question figures. From the given
9. A piece of paper is folded and answer figures, indicate how it will
punched as shown below in the appear when opened?
question figures. From the given
answer figures, indicate how it will
(a) (b)
appear when opened?

(c) (d)

HINTS & SOLUTIONS

1. (c) 2. (b) 3. (b) 4. (a) 5. (c)


6. (c) 7. (b) 8. (c)

9. (c) 10. (c)


Completion of Figures 177

Chapter

21 Completion of Figures
INTRODUCTION
In this section, an incomplete figure is given, in which some part is missing. We
have to choose the segment, given in choices, that exactly fits into the blank portion
of figure so that the main figure is completed.
Note : If you observe carefully, you notice that the missing portion may be the
mirror image of any one of the quarters.

EXAMPLE 1.
Select from alternatives the figure (X) that exactly fits in the main figure to
complete its original pattern.

?
(X)

(a) (b) (c) (d)

Sol. In this question, half shaded leaf is moved clockwise. So, option (b) is right one.

q Shortcut Approach
· If answer figures contain similar figure but in rotated forms, then the correct
answer figure is that figure which can be substituted at the missing part with
least change in orientation.
· The correct option for the missing figure can be given in any rotated from, so
student can rotate the figures to check the correctness of option.
EBD_7420
178 Completion of Figures

PRACTICE EXERCISE
1. Which answer figure completes the
form in question figure ?
Question Figures : (c) (d)

? DIRECTION (Q. 4) : Which answer


figure completes the pattern given in
the question figure?
Answer figures : 4. Question Figure :

(a) (b) (c) (d)


(a) (b) (c) (d)
DIRECTION: In question no. 2, which Answer Figures :
answer figure will complete the
question figure? (a) (b)
2. Question Figure

(c) (d)
?

Answer Figures DIRECTION (Q. 5) : In each of the


following question, which answer
figure will complete the question
figure?
(a) (b) (c) (d) 5. Question Figure:

DIRECTION (Q. 3) : In the following


question which answer figure will
complete the question figure?
3. Question Figure
Answer Figures:

(a) (b)
Answer Figure

(a) (b) (c) (d)


Completion of Figures 179
DIRECTIONS : In questions no. 6 and Answer figures
7, which answer figure will complete the
question figure ? (a) (b)
6. Question Figure :

(c) (d)
Answer Figures :
DIRECTIONS (Qs. 9-10): In the
following Two Questions, which answer
figure will complete the pattern in the
questiion figure ?
(a) (b) (c) (d)
(a) (b) (c) (d) 9. Question Figure
7. Question Figure :

Answer Figures :
Answer Figures :

(a) (b) (c) (d)


(a)
(a) (b)
(b) (c)
(c) (d)
(d) 10. Question Figure
DIRECTION: In question no. 8, which
answer figure will complete the pattern
in the question figure?
8. Question figure
Answer Figures :

(a) (b) (c) (d)

HINTS & SOLUTIONS


1. (b)
2. (d) Option (d) will complete the 4. (b) 5. (a)
question figure. 6. (b)
3. (d)

7. (c) 8. (c) 9. (a) 10. (c)


EBD_7420
Chapter
Hidden / Embedded
22 Figures
INTRODUCTION Sol. Clearly, fig. (X) is embedded fig. (b)
A figure (X) is said to be embedded in a as shown below :
figure Y, if figure Y contains figure (X) as
its part. Thus problems on embedded
figures contain a figure (X) followed by
four complex figures in such a way that
fig (X) is embedded in one of these. The
figure containing the figure (X) is your
answer. Hence, the answer is (b)
EXAMPLE q Shortcut Approach
Directions : In each of the following ex-
· There may be some questions in
amples, fig (X) is embedded in any one
which the question figure is not
of the four alternative figures (a), (b), (c)
directly embedded in any of the
or (d). Find the alternative which con-
answer figure. In these type of
tains fig. (X) as its part.
questions, change the orientation
of question figure to find the
correct answer figure.
· In some questions, the question
figure embedded in two or more
(X)
answer figures, then the most
appropriate answer is that in
which the question figure is
embedded with least change in its
orientation.
(a) (b) (c) (d)
Hidden / Embedded Figures 181

PRACTICE EXERCISE
1. From the given answer figures, Answer Figures
select the one in which the question
figure is hidden/embedded in the
same direction.
Question Figure: (a) (b) (c) (d)
(a) (b) (c) (d)
4. From the given answer figures,
select the one in which the question
figure is hidden/embedded.
Question Figure :

Answer Figures:

Answer Figures :
(a) (b) (c) (d)
(a) (b) (c) (d)
DIRECTION : From the given answer
figures, select the one in which the (a) (b)
question figure is hidden/embedded.
2. Question Figure:

(c) (d)

5. Select the answer figure in which


Answer Figures: the question figure is hidden.
Question Figure

(a) (b) (c) (d)


DIRECTION: From the given answer
figures, select the one in which the
question figure is hidden/ embedded.
Answer Figures
3. Question Figure

(a) (b) (c) (d)


(a) (b) (c) (d)
EBD_7420
182 Hidden / Embedded Figures
DIRECTIONS (6-7) : In each of the Answer figures :
following questions, select the answer
figure in which the question figure is
hidden/embedded. (a) (b)
(SSC Sub. Ins. 2013)
6. Question Figure:
(c) (d)

9. From the given answer figures,


select the one in which the
question figure is hidden/
Answer Figures :
embedded.

(a) (b)

(c) (d)
(a) (b)
7. Question Figure:

(c) (d)

Answer Figures: 10. From the given answer figures,


select the one in which the question
figure is hidden / embedded.
(a) (b)

(c) (d)

8. Which of the answer figures is


embedded in the question figure ?
Question Figure : (a) (b)

(c) (d)
Hidden / Embedded Figures 183

HINTS & SOLUTIONS


1. (d) 5. (d)

2. (c)
3. (c) 6. (a) 7. (d)
8. (c)

9. (a) The figure is

4. (b)
embedded in

10. (b)
EBD_7420
184 Figure Formation and Analysis

Chapter
Figure Formation and
23 Analysis
INTRODUCTION Sol. If figures A, B and E are fitted
together, the resultant figure will
In this topic, a question is one of the
following types : be a triangle.
I. Formation of triangles/square/
rectangle etc. either by joining of
three figures after choosing them
from the given five figures or by B
joining any other pieces after A E
selecting them from given alternatives.
II. Making up a figure from given TYPE-II : Making up a figure from
components. given components
III. Making up a three dimensional EXAMPLE 2.
figure by paper folding.
IV. Rearrangement of the parts of given Find out which of the alternatives
figure. (a), (b), (c) and (d) can be formed
V. Fragmentation of key figure into from the pieces given in box ‘X’.
simple pieces.
TYPE-I : Formation of triangles/
square/rectangle etc. either by joining
of three figures after choosing them from
the given five figures or by joining any
other pieces after selecting them from
given alternatives.
(X)

EXAMPLE 1.
A set of five figures (A), (B), (C),
(D) and (E) are followed by four
combinations as the alternatives.
Select the combination of figures (a) (b) (c) (d)
which if fitted together, will form a
Sol.
complete triangle.

Figure (b) can be formed from the pieces


(A) (B) (C) (D) (E) the given in box 'X'.
Figure Formation and Analysis 185
TYPE-III : Making up a three dimensional
figure by paper folding. In this type, we
have to analyze when a paper folded
along the lines, how a three dimensional (a) (b) (c) (d)
figure look like. Sometimes, a key figure
is given which is made by folding one of Sol. Figure (a) is the rearrangement of
the four figures given in alternatives. We the parts of the given figure 'X'.
have to determine which figure can be TYPE-V : Fragmentation of key figure
used to create the key figure. into simple pieces.
EXAMPLE 3. This type is opposite to TYPE-II. In this
type, a key figure is given and every
A figure ‘X’ is given. You have to
alternatives has different pieces. We
choose the correct figure, given in
the alternatives, when folded have to select the set of pieces that can
along the lines, will produce the make the given key figure.
given figure ‘X’. EXAMPLE 5.
Find out which of the alternatives
will exactly make up the key figure
(X)
(X)

(X)

(a) (b) (c) (d)


Sol. Figure (a) will produce the given
figure 'X' (a) (b) (c) (d)
TYPE-IV : Rearrangement of the parts
Sol. Figure (a) will exactly make up the
of given figure.
key figure 'X'
In this type of questions, a key figure is
given. We have to identify the figure from q Shortcut Approach
alternatives that is a rearrangement of
parts of key figure. · The number of elements given to
form a figure must be equal to the
EXAMPLE 4. elements present in the answer
Which figure is the rearrangement figure. This will help you to easily
of the parts of the given figure. eliminate some of the option figures.
· The size of pieces of figures in
the question figure and the size
of pieces used to form a figure may
vary but their shapes must have
(X) to be similar.
EBD_7420
186 Figure Formation and Analysis

PRACTICE EXERCISE
DIRECTIONS (Qs. 1-4): Among the four Answer Figures :
answer figures, which figure can be
formed from the cut-pieces given below
in the question figure? (a) (b)
1. Question Figure:

(c) (d)

4. Question Figure :

Answer Figures:

(a) (b) (c) (d)


Answer Figures :
(a) (b) (c) (d)
2. Questions Figure :

(a) (b) (c) (d)


5. Which answer figure includes all
the components given in the
question figure ?
Answer Figures :
Question Figure :

(a) (b) (c) (d)


3. Question Figure :
Answer Figures :

(a) (b) (c) (d)


(a) (b) (c) (d)
Figure Formation and Analysis 187
6. Identify the response figure from Answer figures :
which the question figure’s pieces
have been cut.
Question figure
(a) (b) (c) (d)
(a) (b) (c) (d)
9. Find out which of the following
answer figures will exactly make up
the question figure ?
Answer Figures. Question Figure :

(a) (b) (c) (d)


7. Which of the an swer figures
include the separate components
Answer Figures.
found in the question figure?
Question figure:

(a) (b) (c) (d)


(a) (b) (c) (d)
10. Identify the answer figure from
which the pieces given in the
Answer figure: question figure have been cut.
Question Figure

(a) (b) (c) (d)


(a) (b) (c) (d)
8. Among the for answer figures,
which figure can be formed from
the cut - pieces given below in the Answer Figure
question figure ?
Question figure :
(a) (b) (c) (d)
(a) (b) (c) (d)
EBD_7420
188 Figure Formation and Analysis

HINTS & SOLUTIONS


7. (c) All the components of
1. (c)
Question Figure are present in
Answer Figure (c)

2. (a)

8. (d)
3. (c) 4. (a) 9. (b) All the components of the
5. (a) Question Figure are present in
the answer figure (b).

6. (d) All the components of


question figure are present in
the Answer Figure (d). 10. (d)
Visual Reasoning 189

Chapter

24 Visual Reasoning
INTRODUCTION EXAMPLE 1.
Visual intelligence measures the ability Directions : In the following question,
to process visual material and to employ a group of five figures is given. Out of
both physical and mental images in which four figures are similar to each
thinking. As a result people with a high other in a certain way and one is different
visualization find it easier to comprehend from other. Find the odd figure out.
information and communicate it to
others. Your visualization skills
determine how well you perceive visual
(a) (b) (c) (d) (e)
patterns and extract information for
Sol. After examining the above figure, it
further use. Visualization also facilitates is found that except (d) all figures
the ability to form associations between can easily be obtained by clockwise
pieces of information something which and anti-clockwise movement or
helps improve long term memory. each other.
2. NUMBER OF ELEMENTS OR
TYPES OF VISUAL LINES
REASONING A group of figure may be classified
(A) Odd-Man Out Type on the basis of number of elements
or the number of lines present in
(B) Counting of Figures
figures. The figures can also be
classified on even or odd number
(A) Odd-man Out Type of lines or elements present in
1. ROTATION OF SAME FIGURE figures. Classification can also be
done on the ratio of number of lines
This is the most common type of and elements.
classification. The similar figures
EXAMPLE 2
are actually the rotated forms of
Directions : In the following question, a
the same figure in clockwise or
group of five figures is given. Out of
anti-clockwise direction. The which four figures are similar to each
figure which comes out to be other in a certain way and one is different
different from other is that figure from other. Find the odd figure out.
which cannot be obtained by
rotation of either of the other
figures, (a) (c) (d) (e)
(b)
EBD_7420
190 Visual Reasoning
Sol. All except figure (c) contains odd 5. RELATION BETWEEN
number of arrows. ELEMENTS OF FIGURE
3. DIVISION OF FIGURES In this type of classification, the
This type of classification is done elements of the figure bears a
on the equal or inequal division of certain relationship between them
figures or divisioin of figure in in which the odd figure does not
some specified ratio or parts. posses. This relation can be based
on shape of elements presents,
EXAMPLE 3. inversion of elements etc.
Directions : In the following question, a EXAMPLE 5.
group of five figures is given. Out of
which four figures are similar to each Directions : In the following question, a
other in a certain way and one is different group of five figures is given. Out of
from other. Find the odd figure out. which four figures are similar to each
other in a certain way and one is different
from other. Find the odd figure out.

(a) (b) (c) (d) (e)


Sol. Except figure (a) all figures are (a) (b) (c) (d) (e)
divided into two equal parts. Sol. Except figure (c) in all the figures,
4. SIMILARITY OF FIGURES both the inside and outside figures
Classification on the basis of are similar but differ in size.
similarity of figure is done when 6. INTERIOR-EXTERIOR
orientation, shape, measure of CONSIDERATION OF
angle or method of presentation ELEMENTS
of group is same except for the odd A figure can be formed from two
figure. or more elements, it is likely that
EXAMPLE 4. some elements may lie in interior
of other elements while some may
Directions : In the following question, a
lie in the exterior of the other
group of five figures is given. Out of
elements. This consideration can
which four figures are similar to each
be used for classification of
other in a certain way and one is different
elements from a group.
from other. Find the odd figure out.
EXAMPLE 6.
Directions : In the following question, a
group of five figures is given. Out of
(a) (b) (c) (d) (e) which four figures are similar to each
Sol. Let us consider the two adjacent other in a certain way and one is different
bent lines as a pair. Then, in each from other. Find the odd figure out.
figure except (d) there are two
straight lines between the bent pair
and the remaining bent line when
the direction of bent is considered. (a) (b) (c) (d) (e)
Visual Reasoning 191
Sol. Only figure (d) does not contain (b) Triangle –
any element present in the interior It is a closed figure bounded by
of the closed figure. three side.
(B) Counting of Figures Type A
Type-1: Counting of Straight Lines
and Triangles
(a) Straight lines
B C
A. Horizontal line A B
A
q Shortcut Approach
B. Vertical line
· Smallest triangles are counted
B
first.
A A · Now, counted those triangles
C. Slant line which are formed with the two
B B triangles and further counting
goes on in the same way.
q Shortcut Approach · Largest triangle is counted in the
· Consider a line (AB) given last.
C
A B EXAMPLE 2.
· Then, on counting, it will be How many triangles are there in the
counted as one line, i.e., AB and figure ?
not as a two straight lines AC and
CB.

EXAMPLE 1.
How many straight lines are there
Sol. A
in the figure ?

B C
Sol. A R B Smallest triangle = BOC = 1
Largest triangle = ABC = 1
O \ Total triangle = 1 + 1 = 2
P Q
Type-2 : Counting of Quadrilaterals
D C and Polygons
S
Horizontal lines = AB + PQ + DC = 3 (a) Square
Vertical lines = AD + RS + BC = 3 It has four equal sides, equal
Slant lines = 0 diagonals, and each of the four
\ Total lines = 3 + 3 + 0 = 6 angles equal to 90°.
EBD_7420
192 Visual Reasoning
q Shortcut Approach (b) Rectangle
It has four sides, and opposite
· Count smallest squares first. sides are equal. It has equal
· Now, count squares which are diagonals and each of the four
formed with two squares and angles is equal to 90°.
further counting goes on in the
same way. EXAMPLE 4.
· Largest square is counted in the last.
How many rectangles are there in
the figure?
EXAMPLE 3.
How many square are there in the
figure ?
Sol. A B C D

Sol. A G B H G F E
Smallest rectangles = ABGH +
O F BCFG + CDEF = 3
E
Rectangles formed with two rect-
D H C angles = ACFH + BDEG = 2
Smallest squares Largest rectangles = ADEH = 1
= AGOE + GBFO + EOHD + OFCH \ Total rectangles = 3 + 2 +1 = 6
=4 FORMULA FOR COUNTING OF
Square formed with four squares RECTANGLES AND PARALLELO-
= ABCD = 1 GRAMS
\ Total squares = 4 + 1 = 5
Let r be the number of rows and c be
FORMULA FOR COUNTING the number of columns.Now, total
SQUARES number of rectangles or parallelograms
Let r be the number of rows and c be = [(r + (r – 1) + (r – 2) + ..... +1]
the number of columns. × [c + (c – 1) + (c – 2) + ...... + 1]
Now, total number of squares
= (r × c) + {(r – 1) × (c – 1)
+ (r – 2) × (c – 2) + ......
The terms are continued upto the term
which is equal to zero (0). This method
is applicable only to the figure. where
each row and column is divided into
squares of equal sections.

The method is applicable only to the


figure, where each row and column is
divided into rectangle of equal
sections.
Visual Reasoning 193
Type-3 : Circle
Circle is a closed figure. It has zero
sides.

q Shortcut Approach
· Keep writing numbers one by one
inside the circles starting from 1 Sol. Here, we start counting of circles
i.e., for 1st circle put 1, for 2nd and mark them, as 1, 2 and so on
circle put 2, for 3rd circle put 3 and finally we end on getting 5
and so on. number of circles as shown below:
· The number which is put for the
last circle is the required number
of circles. 1 2 3

EXAMPLE 5.
How many circles are there in the 4 5
figure ?

PRACTICE EXERCISE
1. How many triangles are there in the
following figure ?

(a) 4 (b) 7
(c) 9 (d) 18
DIRECTION: (Q. 4): Among the four
answer figures, which figure can be
(a) 11 (b) 13
formed from the cut-pieces given below
(c) 9 (d) 15 in the question figure?
2. How many triangles are there in the 4. How many cubes are there in the
following figure ? group?

(a) 20 (b) 24
(c) 28 (d) 32
3. How many rectangles are there in (a) 16 (b) 18
the given diagram? (c) 20 (d) 10
EBD_7420
194 Visual Reasoning
5. How many triangles are there in the (a) 1 (b) 2
given figure ? (c) 3 (d) 4
8. Choose the figure which is
different from the rest.

(a) 24 (b) 26 (1) (2) (3) (4) (5)


(c) 28 (d) 30 (a) 1 (b) 2
DIRECTIONS (Qs. 6-10): In each (c) 3 (d) 4
problem, out of the five figures marked 9. Choose the figure which is
(1), (2), (3), (4) and (5), four are similar different from the rest.
in a certain manner. However, one figure
is not like the other four. Choose the
figure which is different from the rest. (1) (2) (3) (4) (5)
6. Choose the figure which is (a) 1 (b) 2
different from the rest. (c) 3 (d) 4
10. Choose the figure which is
different from the rest.
(1) (2) (3) (4) (5) × × × × × ×
(a) 1 (b) 2 × × × ×
(c) 3 (d) 4 (1) (2) (3) (4) (5)
7. Choose the figure which is (a) 1 (b) 2
different from the rest. (c) 3 (d) 4
× +
+ ­ ­ + +
­
­

(1) (2) (3) (4) (5)

HINTS & SOLUTIONS


1. (b) B C
A I
E F
A D
G G H
B D C
The triangles are : J
DABC ; DABD ; DADC ; DAFC ; F E
DFDC ; DAFB ; DFDB ; DFBC;
The simplest triangles are ABG,
DGBD ; DADE ; DGBE ; DFDG ;
DDBE ; BIG, BIC, CIH, GIH, CDH, HED,
2. (c) The figure may be labelled GHJ, HJE, FEJ, GFJ and AGF i.e. 12
as shown. in number.
Visual Reasoning 195
The triangles composed of two Thus, total number of cubes
components each are ABF, CDE, = 4 + 6 + 6 + 4 = 20 cubes
GBC, BCH, GHG, BHG, GHF, GHE, 5. (b)
HEF and GEF i.e. 10 in number.
The triangles composed of three A E B J
components each are ABH, AFH,
CDG and GDE i.e. 4 in number. I
H F L
The triangles composed of four
components each are BHF and
CGE i.e. 2 in number. C G D K
Total number of triangles in the figure
= 12 + 10 + 4 + 2 = 28. The triangles are :
Thus, there are 28 triangles. DAIH ; DAIE; DEIB; DBFI;
3. (d) DIHC; DIGC; DIGD; DDFI;
A B C D DIAB; DIBD; DICD; DIAC;
DBAC ; DACD; DBDC; DBDA;
K L
J M E DBLD; DLDK; DKLJ; DJLB;
DJBK; DBDK; DDBJ; DDKJ
I H G F DADJ; DCBK.
The rectangles are : Thus, there are 26 triangles.
ABKJ; JKHI; BCLK; 6. (a) The pins, equal in number to
KLGH; CDML; LMFG; the number of sides in the
ACGI; ACLJ; JLGI; main figure are attached to the
BDFH; BDMK; KMFH; midpoint of a side of the main
ADFI; ADMJ; JMFI figure in case of figures (2),
ABHI, BCGH and CDFG (3), (4) and (5). In fig. (1), these
are squares. We know that pins are attached to a vertex
every square is a rectanlge. of the main figure.
But its reverse is not always 7. (d) In all other figures, the arrow
true. and the V sign lie towards the
Note : By option only its black end of the main figure.
easy to analyze. 8. (a) In all other figures, the lower-
4. (c) 10 cubes are visible and 10 right quarter portion is
cubes are hidden. Clearly, shaded.
there is one column having
9. (b) Each one of the figures except
four cubes.
fig. (2), consists of five
There are two columns each
having three cubes. arrowheads.
There are three columns, each 10. (c) In each one of the figures
having two cubes. except fig. (3), the two crosses
There are four columns, each (x) appear in the diagonally
having only one cube. opposite corners.
EBD_7420
196 ANALYTICAL REASONING
Evaluating Inferences

Chapter
Evaluating Inferences
25
INTRODUCTION (e) If the inference is ‘definitely false’
i.e., it cannot possibly be drawn
This chapter makes you aware about a from the facts given or it
special type of question pattern which contradicts the given facts.
has become a regular trend of almost all
PASSAGE
type of competitive examination. An
In its most ambitions bid ever to house
inference is a logical conclusion on 6 crore slum dwellers and realise the vision
evidence. A valid inference is believable of a slum-free India, the government is
and realistic. As per the pattern, a rolling out a massive plan to build 50 lakh
passage is given followed by some dwelling units in five years across 400
inferences (conclusions) and the towns and cities. The programme could
examinee is asked to decide whether a free up thousands of acres of valuable
given inference follows or not in the light government land across the country and
of the given passage. Let us see the generate crores worth of business for real
format below: estate developers. Proliferation of slums
has had an adverse impact on the GDP
What is the Problem Like? growth for years. Slum dwellers are
Problem Format/ Sample Problem:- characterised by low productivity and
Directions (Qs 1-5): Below is given a susceptibility to poor health conditions.
passage followed by several possible The government believes that better
inferences which can be drawn from the housing facilities will address social
facts stated in the passage. You have to issues and also have a multiplier effect
examine each inference separately in the and serve as an economic stimulus.
context of the passage and decide upon Q 1. Development of land occupied by
its degree of truth or falsity. slums in cities of India will not have
Mark answer: any effect on the common public.
(a) If the inference is definitely true i.e., Q 2. Majority of the slums in cities and
it properly follows from the towns in India are on prime private
statement of facts given. properties.
(b) If the inference is ‘probably true’ Q 3. Per capita income of slum dwellers
though not definitely true in the is significantly lower than that of
light of the facts given. those living in better housing
(c) If the ‘data are inadequate’ i.e. from facilities.
the facts given you can not say Q 4. Cities and towns of developed
whether the inference is likely to countries are free from slums.
be true or false. Q 5. Health and sanitary conditions in
(d) If the inference is ‘probably false’ slums are far below the acceptable
though not definitely false’ in the norms of human habitat in Indian
light of the facts given. cities and towns.
Evaluating Inferences 197
Before solving the sample problem, conclusion. Now, ask the following
we must see the pattern of the questions from yourself.
problem and find out what it puts (A) Is the extra assumption an
before you to think. universal truth?
A minute look will make you clear (B) Can the extra assumption
that here the examiner has graded never be false?
the choices very closely. He/ If you find ‘yes’ for the question
she has given two positive choices (A) and ‘no never’ for the
instead of one. question (B), then accept it as
i. Definitely true definitely true, otherwise pick
ii. Probably true ‘Probably true’.
Further, he/ she has also given two
2. Definitely False or
negative choices instead of one:-
i. Definitely false Probably False
ii. Probably false If the given inference does not
This pattern requires a deeper follow from the passage, it falls
thinking as it leaves before you under the category of definitely
following areas of confusion:- false. But confusion may arise when
1. Definitely true or probably the given inference is not given
true directly in the passage and seems
2. Definitely false or probably ‘almost’ definitely false. But as
false related things are not mentioned
3. Data inadequate or probably clearly in the passage, you think
true that ‘probably false’ may be correct.
4. Data inadequate or probably To get rid of this confusion try to
false recheck your reasoning. If the
opposite of the inference has not
1. Definitely True or been mentioned in the passage,
Probably True then you must assume something
extra to reach your conclusion. Just
If the given inferences is a direct ask the following questions to
consequences of something given yourself.
in the passage, then it falls under (A) Is this assumption an universal
the category of definitely true. But truth?
the confusion may arise when the (B) Can this assumption never be
given inference is not directly false?
stated in the passage but it appears If you find ‘yes’ for question (A)
‘almost’ definitely true to you. and ‘no, never’ for question (B)
Since it is not clearly stated in the then select your answer as
passage, you may think that even definitely false, otherwise probably
‘Probably true’ could be the answer. false will be your correct answer.
To get rid of this confusion, you
have to recheck your reasoning. If
3. Data Inadequate or
the given inference has not been Probably True
mentioned directly in the passage, When an indirect inference is
then you must have assumed drawn from the passage, this
something ‘extra’ to draw this confusion may arise. As the given
EBD_7420
198 Evaluating Inferences
inference is not explicitly On the other hand, you should pick
mentioned, you think that data are up the choice ‘data are inadequate’
inadequate because sufficient only if you can not find any
information has not been given to acceptable assumption which,
draw a conclusion. However, the combined with what is said in the
given inference appears to be in passage, may lead to some definite
sync with the general 'tone' of the conclusion. In such case, you can
passage In such case you may go not get convinced whether the
for ‘Probably true’. given inference is likely to be true
To get rid of this conusion, recheck or false.
your general mental ability. You can Now, lets try to apply the above
declare the given inference as rules in the passage given above
probably true, if with the help of and try to solve the sample
some extra assumption, the given problems.
inference seems likely to be true. Solution to sample problems:
Thus, you can some how convince 1. (c) As we have no information
yourself that the inference is likely about how the freed up land
to be true. On the other hand, you will benefit the common pub-
can declare that data are lic, hence data inadequate’ will
inadequate if no definite be our correct answer choice.
conclusion can be drawn from the The passage do not suggest
passage even with the help of some us any related assumption.
extra assumption. 2. (e) The passage says to the
contrary getting rid of slums
4. Data Inadequate or would “Free up ..... valuable
Probably False government land”. The
inference does not follow
When the given inference is drawn from the passage.
indirectly from the passage, such 3. (b) The extra assumption that
confusion may arise. As it is not makes this option probably
explicitly said in the passage, you true is : Low productivity is
come to the conclusion that data likely to lead to low income.
are inadequate because sufficient The passage does not
information has not been provided directly talk about per capita
to draw a definite conclusion. income.
However, the given inference 4. (b) As slums have led to a lower
appears to you in contradiction with GDP growth in India. The state-
the general ‘tone’ of the passage. ment is in sync with the 'tone'
Therefore, you are tempted to pick of the passage. The extra as-
up ‘probably false’ as your answer. sumption here can be that as
To get rid of this confusion recheck develop coun tries deploy
your general mental ability. You can things that improves their GDP.
declare an inference ‘probably So it can be probably true that
false’ only if you are able to find all slums vanish.
out a reasonable assumption, 5. (a) The passage says that the
combining which with what is said slums dwellers are susceptible
in the given passage the inference “to poor health conditions”.
appears likely to be false. This is directly mentioned in
the passage.
Evaluating Inferences
q Shortcut Approach 199
EBD_7420
200 Evaluating Inferences

PRACTICE EXERCISE
DIRECTIONS (Qs. 1-5) : Below is given But for the restructuring of loans
a passage followed by several possible permitted by the Central Bank on fairly
inferences which can be drawn from the generous terms, NPAs would have been
facts stated in the passage. You have to still higher. Prudent banks that took care
examine each inference separately in while sanctioning loans and then
the context of the passage and decide monitored the post– san ction
upon its degree of truth or falsity. disbursement diligently should be able
to weather the crisis. But it is one thing
Mark answer (1) if the inference is
to have NPAs rise because of a cyclical
'definitely true,' i.e. it properly follows downturn, it is quite another to have
from the statement of facts given. NPAs rise because of policy errors that
Mark answer (2) if the inference is ' are entirely within the realm of policy
probably true' though not 'definitely true' makers. And this is what we need to
in the light of the facts given. guard against. Excessiyely low interest
Mark answer (3) if the 'data are rates skew the risk–reward equation by
inadequate, i.e. from the facts given you making projects that are actually not
cannot say whether the inference is likely viable, appear viable–till interest rates
to be true or false. reverse and the same projects cease to
Mark answer (4) if the inference is ' be viable! it is now well established that
probably false' though not 'definitely tong periods of unduly low interest rates
false' in the light of the facts given. encourage banks to take more risks. A
Mark answer (5) if the inference is low interest rate regime driven by an easy
'definitely false, i.e. it cannot possibly money policy rather than macroeconomic
be drawn from the facts given or it fundamentals leads to excessive
contradicts the given facts. expansion of credit. It incentivizes banks
to take on more risk in search of higher
The deterioration in the overall
returns and to misprice risk.
asset quality of banks– gross Non–per–
1. Higher NPAs indicate shortcom-
forming Assets (NPAs) are reportedly
ings in disbursement and follow-
27% higher at the end of December 2009
up of credit given by banks.
than at the end of December 2008– is not
surprising. Any slowdown in growth is 2. The Central Bank always allows
bound to trigger a rise in NPAs as more banks to restructure their loans in
and more companies default on loan the event of rise in NPAs.
repayments. The effect would be 3. Lower interest rate cycle projects
pronounced when the slowdown commercially unviable projects as
coincides with a severe global recession. viable.
Evaluating Inferences 201
4. Low interest rate on credit reduces one question, consider both again and
the capacity to absorb various decide which one of the two would more
unaccounted risk factors. definitely be that answer and in the same
5. Banks' NPAs occur only due to way review the others also).
economic factors. Cardiovascular disease is so
DIRECTIONS (Qs. 6-10) : Below is prevalent that virtually all businesses are
given a passage followed by several likely to have employees who suffer from,
possible interferences which can be or may develop, this condition. Research
drawn from the facts stated in the shows that between 50-80 percent of all
passage. You have to examine each people who suffer a heart attack are able
inference separately in the context of to return to work. However, this may not
the passage and decide upon its degree be possible if they have previously been
of truth or falsity. involved in heavy physical work. In such
cases, it may be possible to move the
Mark answer (a) If the inference is
employee to lighter duties, with
“definitely true” i.e. it properly follows
appropriate retraining where necessary.
from the statement of facts given.
Similarly, high-pressure, stressful work,
Mark answer (b) If the inference is even where it does not involve physical
“probably true” though not “definitely activity, should also be avoided. Human
true” in the light of the facts given. Resource managers should be aware of
Mark answer (c) If the data is inadequate the implications of job roles for
i.e. from the facts given, you cannot say employees with a cardiac condition.
whether the inference is likely to be true 6. Employees who suffer from
or false. cardiovascular disease are mostly
Mark answer (d) If the inference is unable to return to work.
“probably false” though not “definitely 7. Employees suffering from Cardio-
false” in the light of the fact given. vascular diseases are unable to
Mark answer (e) If the inference is handle stressful situations.
“definitely false” i.e., it can not possibly 8. Employees above the age of 50 are
be drawn from the facts given or it found to suffer from cardiovascular
contradicts the given facts. disease.
9. Physical and stressful work
(Note: Each of the five questions has
definitely leads to a heart attack.
only one distinct answer i.e., no two
10. Heart disease can affect employees
questions can have the same answer. If
in any type of business.
you get the same answer for more than
EBD_7420
202 Evaluating Inferences

HINTS & SOLUTIONS

1. (a) From the given data in the 6. (e) It is mentioned in the passage
passage, it is clear that the that 50 - 80 per cent of all
Inference is definitely true. people who suffer a heart
2. (b) The use of term’ always’ in the attack are able to return to
inference shows that the work. Therefore, the inference
Inference is probably true. is definitely false.
3. (a) The inference is definitely 7. (b) It is mentioned in the passage
true. Consider the following that the persons who have
line of the passage:
suffered a heart attack should
“Excessively low interest avoid high pressure, stressful
rates skew the risk reward work. Therefore, it may be
equation by making projects
concluded that the inference
that are actually not viable,
appear viable.” is probably true.
4. (e) The Inference is definitely 8. (c) There is no information about
false. Consider the following this inference.
line of the passage: 9. (d) The use of term ‘definitely’ in
“It is now well established the inference makes it doubt-
that long periods of unduly ful. Therefore, the inference is
low interest rates encourage probable false.
banks to take more risks.” 10. (a) Consider the very first line of the
5. (a) The Inference is definitely passage. It is clear from the first
true because rise in NPAs line of the passage that the
depend upon cyclic factors. inference is definitely true.
Chapter
Statement &
26 Arguments
INTRODUCTION Explanation: The foregoing example
has two parts:
In this chapter, we are going to study
Part I: “Mr. Sharma bought a large
arguments. In fact, this is the study what
quantity of sweets.”
we call the basics of all logic. Do you
know what do we do in logic? In logic, Part II: “He must have celebrated some
we advocate certain point of view with occasion.
the help of some evidences and certain Here, ‘Part II’ is the conclusion part of
assumptions and that is called the given argument. How has this
argumentation. This is a fact that almost conclusion (part II) been arrived at? In
all segments of analytical reasoning are fact, this conclusion has come out with
the help of supporting evidence or
someway associated with argumentation
premise that is part I of the argument.
and this is the reason why study of Did you notice that in this argument part
argumentation is so important for the I and part II (Premise and conclusion)
examinees preparing for various are connected by a hidden premise
competitive examinations. which is not explicitly stated. That
hidden premise is “a large quantity of
CONCEPT OF ARGUMENT sweets is bought only on occasions”
and this premise may be called an
A sequence of two or more sentences
assumption. Hence, in reality the given
(or statements)/ phrases/clauses that argument has three parts.
includes a conclusion (or claims), is called
Part I: (Premise) Mr. Sharma bought a
an argument. This conclusion of the
large quantity of sweets.
argument is based on one or more than
one statement and these statements may Part II: (Assumption or hidden premise)
be called premises (propositions). Apart A large quantity of sweets is bought
only on occasions.
from this, arguments may also have some
hidden premises. which may be called Part III: (Conclusion) He must have
assumptions. Let us see the following celebrated some occasion.
example: Point to be noted is that part II is an
Example: assumption (a hidden premise) that
connects part I (premise) and part III
Mr. Sharma bought a large quantity of (conclusion) and hence, it is a missing
sweets, he must have celebrated some link between part I and part III of the
occasion. given argument.
EBD_7420
204 Statement & Arguments
No doubt that above mentioned example (ii) Explanation for question: Just
brings to us the basic characteristics of consider the argument given as
argumentation but it also leaves some “Vandana is tall. She is slim and has
beautiful eyes. She has long hair and
questions before us like:
charming face as well. So, Vandana
(i) Is the assumption or hidden premise is a beautiful girl.”
always present in an argument? Here,
(ii) Is the number of premise only one in 1st premise: Vandana is tall.
an argument? 2nd premise: She is slim and has
Our answer for both the questions beautiful eyes.
will be a big ‘No’. Why so? Let us 3rd premise: She has long hair and
charming face as well.
see the explanations for both the
Conclusion: So, Vandana is a
questions given below: beautiful girl.
(i) Explanation for question: Just This proves that an argument can
consider an argument given as “Mr. have more than one premises.
Sharma bought a large quantity of Further this explanation is also a
sweets. A large quantity of sweets reply for question (i) as the given
is bought on occasions only. Hence, argument has no missing link. This
he must have celebrated an argument is complete in itself and
hence, it is free of hidden premise or
occasion”.
assumption.
Here, we see that this argument has Ways of Argumentation: So far, you must
no assumption (hidden premise) have understood the basic concept of
because the premise or supporting argumentation and come to the
evidence (Mr. Sharma bought a large conclusion that an argument is usually
quantity of sweets) and conclusion made to make strong a particular point
(Hence, he must have celebrated an of view in order to convince someone
about something.
occasion) are connected by an
(i) Argument based on Analogy:
explicit statement (A large quantity Analogy based arguments are often
of sweets is bought on occasions used to make strong a particular
only). Remember, an assumption is a point of view. In fact analogy is an
hidden premise. It does mean infer ence drawn out of a
assumption is a missing link in the resemblance between particular
chain of logic. Therefore, if an things, occasion or events (that are
argument is complete in itself and known) to a further (unknown)
resemblance. For example, if we find
does not have any missing link, then
a fat-woman eating very much and
it will not have any assumption. In meet in another woman who is also
the given argument, the explicit fat then, by analogy, we expect that
statement (A large quantity of sweets the other fat woman would also be
is bought on occasions only) eating very much.
connects premise or supporting EXAMPLE 1. Sachin scored a
evidence and conclusion to make the century in the 1st test against Australia
argument assumptionless. and so did Dhoni; Sachin scored more
Statement & Arguments 205
than 150 runs in the 2nd test against different games (Football and Hockey).
Australia and so did Dhoni; Sachin has Hence, the argument given in example
scored a double century in the 3rd test (2) also seems to be a weak argument.
against Australia. So, Dhoni will also hit Final comment: Analogy based
a double century in this 3rd test match arguments are weak arguments.
against Australia.
(ii) Argument based on cause: Such
EXAMPLE 2. Australia and England
arguments relate a cause with a
have both lost to India in football and result. Let us see the examples given
hockey. So, India should defeat both the below:
countries in cricket.
EXAMPLE 3. India will win the world
Findings:
cup this year because it is the most
In Example 1, Sachin and Dhoni balanced one day team in the world in
performed very well in the 1st two present day cricket.
matches against Australia. In fact, it EXAMPLE 4. He came back home late
seems that Dhoni did the same thing
night. He must have gone to watch a
what Sachin did in the 1st and 2nd test.
movie.
As Sachin has played a great inning
Findings: We see in the foregoing
scoring a double century in the 3rd test
examples that effects have been related
match, hence on the basis of similar with causes. In example (1), the cause
situation the conclusion has been made (the most balanced one day team) well
that Dhoni will also make a double supports the effect (India will win the
century. world cup) and hence, it is a good
We also know that performing good or argument. But in Example (2) it is argued
bad is a matter of chance. It is also a that since the effect (coming home late
matter of chance that two players night) has taken place, the cause
(Sachin and Dhoni) performed equally (watching movie) must have occurred.
good in the last two test matches. But the point to be noted that effect may
Therefore, we cannot say definitely that occur (he may come home late night)
Dhoni will make a double century because of the other reason as well.
because Sachin has done so. In fact, Hence, the argument given in the Example
we can say that he may or may not hit a (2) is not a good argument or it may be
double century. It can also be said that called a weak argument.
future performances can not be Final Comment: Arguments based on
predicted on th e basi s of past causes may be strong or weak or
performances. Thus, it is clear that this fallacious.
analogical argument does not seem (iii) Argument based on example:
strong. Similary, in case of example (2) Sometimes an argument is given by
we can say that India may or may not citing some example/ examples as
defeat Australia and England in the premise/ premises. Let us see the
game of cricket only because India has following examples that will illustrate
defeated both the countries in two the concept:
EBD_7420
206 Statement & Arguments
EXAMPLE 5. We should use X brand types of arguments are very common
of cold cream because X brand is used in day to day life.
by ‘Madhuri Dixit”, the famous EXAMPLE 7. Exercise is good for
bollywood actress. health and students need good health
EXAMPLE 6. We must like Roses to put hard labour in their studies. This
because Chacha Nehru loved Roses. is the reason why every educational
Findings: In example (1) we have institution must have a gym.
arrived at the conclusion (we should use
EXAMPLE 8. There should be a ban
X brand of cold cream) by using the
on strikes as they disrupt the normal life
premise as example (X brand is used by
of the common people.
Madhuri Dixit). In example (2) the
Findings: In example-1, the conclusion
conclusion (we must like roses) has come
is that every educational institution must
out by using the premise as example
have a gym because exercise is good for
(because Chacha Nehru loved it). Here,
health and students need good health.
we can say in case of Example-1 that
No doubt the good health ensures good
using certain brand by a particular
mind but it is not practically feasible for
actress, does not mean that X brand will
every educational institution to have a
be liked by all people as likes and dislikes
gym. Hence, Example-1 will be a weak
are the personal choices. In example (2),
argument. In example-2, ban on strikes is
the case is also the same. Everyone
being demanded and this demand is
cannot like the roses only because
reasonable as argument has negative
Chacha Nehru loved roses.
feature of strike. Hence, example-2 is a
Final comment: Example based strong argument.
arguments are either weak or fallacious.
Final comment: Such arguments can
Note: In Example-1 and 2, conclusion part be both weak or strong.
is the start of the arguments. Sometimes
you can also see that conclusion is given (v) Argument based on chronology:
in the middle. It does mean that Very often we see that a conclusion
conclusion part is not always in the last. is drawn only on the basis of
But it depends on the style of writing of chronological order of some events.
different writers/authors. Let us see the examples given below:
(iv) Argument based on blind advocacy: EXAMPLE 9. Computer was invented
Such argument is like a salesman’s
later than television. Therefore, television
argument who argues only for the
has a technology inferior to that of a
purpose of selling a particular
computer.
product. He speaks of the advantages
and the benefits of his product. EXAMPLE 10. Song ‘B’ was released
Hence, a salesman argument is one two months earlier than song ‘C’. So the
where a conclusion comes out former could not be the copy of the latter.
because of the positive points and Findings: In example-1, it is assumed that
the benefits that it leads to. Such a technologically inferior object always
Statement & Arguments 207
comes before the superior objects. This After learning concept of argument we
may be true most of the time but this is can easily move on to the problems of
not true in 100% cases. Hence, the reasoning which are asked in various
conclusion given in example 1 is exams wherein examinee is required to
questionable making the given argument evaluate the forcefulness of the
a weak one. In 2nd case, it is the arguments. On the basis of a statement,
possibility that song ‘C’ was recorded arguments are given in the questions
earlier although released later than the and the candidate is required to find out:
song ‘B’. Hence, in such a situation the (a) Which argument is strong.
possibility of copying can not be denied
and this makes argument given in (b) Which argument is weak.
Example-2 a weak argument. We know that “strong” arguments are
Final comment: This type of arguments those which are both important and
are usually weak and unconvincing. directly related to the question. “Weak”
By now, all the standard ways of arguments are those which are of minor
argumentation have been discussed in importance and also may not be directly
detail. We will now take a look at the key related to the question or may be related
words so that you could easily take out to a trivial aspect of the question. To find
the conclusion part from the given out if a given argument is strong or not
argument. The keywords are given we will move according to the solution
below: steps given below:
So, Hence, Solution steps
Therefore, Consequently Step I: Do the preliminary screening
Thus, of the given arguments.
Apart from above given keywords, the Step II: Fin d out if the given
conclusion part can also be identified by arguments really follow or
the certain phrases given below: not.
As a result Step III: Fin d out if the given
It can be inferred that arguments are really desirable
Which means that (in case of positive argument)
Which suggests that / harmful (in case of negative
Which proves that arguments)
Which shows that Step IV: Find out if the statement and
It follows that argument are properly related.
If you find one of these keywords/ Now, we will discuss all the steps one by
phrases before any sentence then take one.
that sentence as your conclusion. If the Step I: Preliminary screening of the
keywords/phrases are absent, then given arguments
apply your common sense and take out
At the very 1st level we test how weak
the sentences that can follow one of
an argument is. If at the very 1st level we
these keywords/ phrases and that
find the argument weak, then there is no
sentence will be your conclusion.
need to go for further steps. In many
EBD_7420
208 Statement & Arguments
cases the weak arguments are very statement and put them under the
clearly visible and we do not need to category of weak arguments.
think much before arriving at the EXAMPLE 12.
conclusion that they are weak. Such type
of arguments come under the following Statement: Cricket must be banned in
category: India.
Argument: Yes, it has no use.
(i) Doubtful/Ambiguous arguments:
These arguments do not make it clear Comment: Here, the argument does not
that how they are related to a course go deep down into the matter making
itself a weak argument.
of action. They also do not give the
clear idea about what exactly the (iii) Arguments in the form of question:
author or writer wants to say. Such arguments are very weak in
nature as the arguments given in the
EXAMPLE 11. question form are without any
Statement: One should enjoy every substance and have no technique
second of one’s life because everyone of argumentation. In fact, in such
has to die one day. arguments arguers throw back the
Argument: No, because one must think question.
about fulfilling one’s ambition in life and EXAMPLE 13.
should not think about death as one’s Statement: Should import be banned in
goal. India?
Comment: Here, statement and argument Argument: Yes, why not?
are not properly related. Statement Comment: Here, statement is given in
suggests to enjoy every second of life. the form of question and arguer throws
Enjoying life does not mean that one back the question without giving any
should not follow the path of fulfilling convincing statement in the form of
one’s ambition. In fact a person can argument. Hence, the given argument is
enjoy his/ her life in the course of very weak.
fulfilling his/her ambition. In fact, we can (iv) Very simple arguments: Such
say without enjoying work of our own arguments are very simple in nature.
choice, we can not fulfill our ambition. They are given in small sentences
Further the given statement does not but do not get any support by facts
give any indication that one should see or established notions. Further, such
death as one’s goal. Hence, in this case arguments are not ambiguous and
statement and argument leave doubtful they are properly related with the
and confusing impression on our mind statement but because of their simple
making the given argument very weak. nature they come under the category
(ii) Useless/ superfluous arguments: of weak arguments.
Such arguments do not do a deep EXAMPLE 14.
analysis of the given statement. Statement: Enjoying life should be the
They simply ‘glance’ at the principle of our life.
Statement & Arguments 209
Argument: No this thinking hardly Comment: In the above examples, all the
enable us to do anything. given arguments are expected to follow
Comment: Here, the given argument is as they all are established facts.
only a simple assertion which contains Therefore, all the arguments presented
no substance. Here, it will come under can be said to pass the test of step II.
the category of weak arguments. NOTE : Point to be noted that arguments
Step II: Finding out if the given given under Example 1, Example 2,
arguments really follow or not. Example 3 & Example 4 have passed
the step II only so far but it has not yet
If the arguments are rejected at the been determined whether these
preliminary step then we do not need to arguments are forceful or not (strong
test them further. But, if the preliminary or not). They will be called strong only
step has been cleared, then we move on when they will pass step III and step IV.
to step II.
(ii) Prediction on the basis of
Case I: When the result follows experience: Such arguments are
At the step II, the result will follow in the very near to established facts type
cases given below: of arguments. But, in reality, they are
(i) Established fact: An established fact not established facts as they are not
does mean that it must be universally yet so universally acknowledged as
acknowledged/scientifically establi- to be treated as established fact. In
shed. A result will follow a course of fact, such arguments are given on
action if it is an established fact that the basis of experiences. Just see the
this particular result follows this
following example:
particular course of action.
EXAMPLE 19.
EXAMPLE 15.
Statement: Captains should not have
Statement: Should drinking be avoided? given their say in selection of national
Argument: Yes, it contributes to bad sports teams.
health. Argument: Yes, it discourages favouritism
EXAMPLE 16. towards some particular players.
Statement: Should Tendulkar be selected Comment:The result or consequences
in the team even after 10 years from now? given in this example will be a probable
Argument: Yes, Tendulkar is one of the result as our experiences suggest this.
greatest cricketers in the world. Hence, this will go for further test.
(iii) Logically given arguments: Such
EXAMPLE 17. arguments are given on the basis of
Statement: Married people should live logic. It does mean that the emphasis
separate from their parents. here is on the logic and not on the
Argument: Yes, living separate will give established fact or experience. If we
married people a greater freedom. see such type of arguments we can
EXAMPLE 18. easily predict that such cases have
Statement: Should smoking be promoted? occurred in practice. But when we
Argument: No, smoking is injurious to think over such situations with
health. proper logic and reasoning then we
EBD_7420
210 Statement & Arguments
arrive at the conclusion that such (i) Established fact: If it is an
an argument may be true. Let us see established fact that a particular
the example given below: result will not follow a particular
EXAMPLE 20.
course of action, then the argument
will be rejected at step II. Let us see
Statement: World leaders must try for the example given below:
complete disarmament.
EXAMPLE 22.
Argument: Yes, complete disarmament
will make a war free world. Statement: Should smoking be
discouraged in the country?
Comment: The example gives an
argument that is logically convincing: Argument: No, it give relaxation when
The argument is probable as the logic one gets tired and this way contributes
behind it is that if there will be armless to health.
world then there will be a war free world. Comment: It is an established fact that
Hence, the argument passes the step II smoking is injurious to health and thus,
test and will go for further test. we can say that this argument is incorrect
(iv) Notions of truth: Such arguments are and weak enough to be rejected at step II.
unquestionable truth because of the (ii) Prediction on the basis of
simple reason of universal experiences: If the experiences say
acceptance. It does mean that they that the result will not follow then
are the ideas or thoughts already the given argument will be rejected
acknowledged by society. This is the at the step II. Let us see the example
reason why they are very similar to given below:
established facts in many ways. The Statement: Should cricketer A be
following example illustrates this appointed the next captain of the Indian
point: cricket team?
EXAMPLE 21. Argument: Yes, it will end the favouritism
Statement: Should marriages between in selection of team as cricketer A has
blood relatives be promoted? made allegations of favouritism against
Argument: No, it will promote incest the current captain.
which is a sin. Comment: In this example, the argument
suggests that cricketer A should be
Comment: No, doubt, the given
appointed captain of the Indian cricket
argument seems strong as it is based on
team because it will end the favouritism
prevailing notion of truth that our society
in the team selection. This suggestion
does not allow marriages between blood
has been given on the basis that A has
relatives and consider such marriages as
made allegation of favouritism against
a sin. As, the given argument is likely to
the current captain. But the experiences
be strong it will go for next step test. say that there have been so many cases
Case II (When the result does not follow when people did the things what they
argument will be rejected). opposed. Hence, saying one thing and
Following are the cases when results do doing other is very common. This is the
not follow and arguments are rejected at reason why it can not be made sure that
2nd level test in step II only.
Statement & Arguments 211
A will not do favouritism in team selection (v) Arguments based on examples/
only because he has criticised the current analogies: Very often it is seen that
captain for this. It is clear that the given an example or a precedent is made
argument is weak enough to be rejected the basis of an argument. But point
in step II. to be noted that analogy or example
based arguments come under the
Note : This is the exactly opposite to category of bad arguments. It must
point (ii) in step II (Case I). be cleared that just because
(iii) Argument with faulty logic: This is someone did something in the past,
exactly opposite to the point (iii) in the same can not be said as
step II (case I). Let us see the pursuable. Let us see the example
following example: given below:
Statement: Should the culprits behind Statement: Should ever yone be
the fodder scam in Bihar be punished? optimistic in Life?
Argument: No, a political vaccum will Argument: Yes, Indira Gandhi was
be created if the culprits get punishment. optimistic and this is the reason why she
became the prime minister of India.
Comment: As per the logic, punishing
Comment: Here, the example of Indira
culprits behind the fodder scam in Bihar
Gandhi is given that makes the argument
would please the public and improve the
image of the Bihar government. How can very weak. Thus, such type of arguments
are rejected in step II.
it create a political vaccum? This argument
(vi) Arguments based on individual
has been given with a faulty logic and
perceptions (or assumptions): In
hence will be rejected in step II only.
some cases it is seen that an
(iv) Argument violating prevailing
assumption or view of the author is
notions of truth: Argument that
the substance of an argument. Such
violates unquestionable notions
arguments neither have proper logic
(Ideas that are universally accepted
nor substance of established fact.
and acknowledged by society) will
These arguments are called bad
be rejected in step II. Let us see the
arguments and they can be rejected
example given below:
in step II.
Statement: Should marriage in blood Statement: Should India be declared a
relations be promoted in India? Hindu Rashtra?
Argument: Yes, if the two mature blood Argument: No, it will lead to chaos.
relatives are willing to do so, then they Comment: What message author gives
can not be prohibited from doing it.
through the argument is view of the
Comment: In our society, it is widely
author. In fact, declaring India a Hindu
accepted truth (or universally accepted
Rashtra may or may not lead to the result
truth) that the marriages between blood
relatives are considered to be a sin as it given in the argument. It means that
promotes incest. The given argument assertion made by argument may or may
violates this prevailing notion of truth and not follow in actual practice and if the
is weak enough to be rejected in step II. author has a rigid stand on this assertion,
EBD_7420
212 Statement & Arguments
it is his/ her individual perception or EXAMPLE 26. As smoking is injurious
assumption which makes the argument to health, its promotion is harmful. This
weak enough to be rejected in step II. reason makes the argument strong
Step III: Given arguments are really enough to pass the step III test.
desirable/ harmful EXAMPLE 27. It is true that favouritism
In step II, we come to the conclusion takes place on the part of captains at
that Examples 1-7, have passed the 2nd times, but that does not mean that they
level test and qualified for the step III should not be given their say while
(3rd level test). Hence, we will take the
selecting team. In fact, captains are
examples to be qualified for step III one
expected to bring positive and desired
by one:
result if given their say in team selection.
EXAMPLE 23. Here, the argument is
Further, giving their say in team selection
positive and therefore, we have to check
the desirability. As, it is a established fact makes the captains more responsible for
that drinking contributes to bad health the bad performance of the team and this
and thus it is desirable to avoid it. It is inspires the captain to draw best out of
clear row that Example 1 passed the 3rd the players in the team. Hence, the result
level test. is not desirable and the given argument
EXAMPLE 24. No doubt that at proves to be weak enough to be rejected
present Tendulkar is one of the greatest in step III.
cricketers in the world. He will also remain EXAMPLE 28. If it is possible to make
in the list of great ones in the history of world free of wars through complete
the game of cricket. But it is also a truth disarmament, it is well and good. But,
that he has spent more than 20 years in
complete disarmament does not assure
this game and is a retired cricketer. This
th at there would be no an tisocial
is the reason that after 10 years he will
elements like murderers, looters, terrorists
definitely not be in team as his selection
is impossible. Hence, despite being an and the likes. To tackle these kind of anti-
established fact the argument is not social elements, police and different
desirable and is rejected in step III. security forces are needed. How do
(Example 2 is a weak argument) police and other security forces function
EXAMPLE 25. Here, it is true that without arms? No, doubt, it is impossible
living separately from parents gives for such security providing bodies to
married people more freedom but at the work without arms. Hence, the argument
same time getting freedom at cost of given in Example 6 is weak and will be
separation from parents is undesirable. rejected in step III.
Further, separating from parents does
EXAMPLE 29. Marriages in blood
mean avoiding duty of taking care of
relatives promote incest which is a sin
parents. Hence, argument given in
and hence harmful for the established
example 3 is not desirable and is weak
enough to be rejected in step III. norm of society. On the basis of this
Statement & Arguments 213
logic, argument given in Example 7 is EXAMPLE 32. Marriages in blood
strong enough to pass the 3rd level test relatives and promotion of incest is
step III. directly and properly related. Hence,
Now, we have, th e given ar gument “No, it wi ll
Examples qualified for step IV test: promote incest which is a sin” is a
Example-1, 4 and 7. Rejected examples in strong argument and this is the final
step III: Example- 2, 3, 5 and 6. conclusion.
Now, we have come to the end of
Note : How to decide a positive this chapter. For the understanding of
argument which is really desirable or students, below is given a question
a negative argument which is really format for the examination. The question
harmful, is only the matter of common format has been made with the Example
sense. Just apply your common sense, 4 given in this chapter.
think over the argument, try to go by Question format:
proper logic and general norms of Direction: Each question given below
society. is followed by two arguments numbered
Step IV: Finding proper relation I and II. You have to decide which one of
between statement and argument. the arguments is a ‘strong’ argument and
What does proper relation between which is a 'weak' argument.
statement and argument mean? In fact, it Give answer (a) If only argument I is
does mean that argument must be strong.
pinpointed on the main issue involved (b) If only argument II is strong.
and it should not focus on any irrelevant, (c) If either I or II is strong.
insignificant or minor issues. Now, we
(d) If neither I nor II is strong.
move on to step IV or final test. As
Example-1, 4 and 7 have qualified for this (e) If both I and II strong.
test, let us check the three examples one Statement: Should smoking be
by one: promoted?
EXAMPLE 30. Drinking and bad Argument: I: No, smoking is injurious
health are properly and directly related. to health.
Hence, the given argument “Yes, it II: Yes, why not?
contributes to bad health” is a strong Solution:
argument and this is the final conclusion. I will follow (the reason already given
EXAMPLE 31. Smoking and bad see Example 4)
health (injurious to health) are directly II will not follow as it is a question back
and properly related. Hence, the given type of argument and such type of
argument “No smoking is injurious to arguments are very weak.
health” is a strong argument and this is
Hence, option (a) is the correct answer.
the final conclusion.
EBD_7420
214 Statement & Arguments
q Shortcut Approach

Step I: Preliminary Screening of argument

Passes Fails — Weak Argument

Step II: The argument follows the statement

Passes Fails — Weak Argument

Step III: The argument is desirable (for positive statements)


/ harmful (for negative statements)

Passes Fails — Weak Argument

Step IV: The argument is properly related to the statement.

Passes Fails — Weak Argument

Strong Argument
Statement & Arguments 215

PRACTICE EXERCISE
DIRECTIONS (Qs. 1-5) : In making 2. Statement : Should there be
decisions about important questions, it complete ban on setting up of
is desirable to be able to distinguish thermal power plants in India ?
between 'strong' arguments and 'weak' Arguments :
arguments. 'strong' arguments are (A) Yes, this is the only way to
those which are both important and arrest further addition to
directly related to the question. 'Weak' environmental pollution.
arguments are those which are of minor
(B) No, there is a huge shortage
importance and also may not be directly
of electricity in most parts of
related to the question or may be related
the country and hence
to a trivial aspect of the question.
generation of electricity needs
Each question below is followed by three to be augmented.
arguments numbered (A), (B) and (C). (C) No, many developed
You have to decide which of the countries continue to set up
arguments is a 'strong' argument and
thermal power plants in their
which is a 'weak' argument.
countries.
1. Statement : Should there be a cap
on drawing groundwater for (a) None is strong
irrigation purposes in India ? (b) Only (A) is strong
Arguments : (c) Only (B) is strong
(A) No, irrigation is of prime (d) Only (C) is strong
importan ce for food (e) Only either (A) or (B) is strong
production in India and it is
heavily depen den t on 3. Statement: Should there be a
groundwater in many parts of restriction on the construction of
the country. high rise buildings in big cities in
(B) Yes, water tables have gone India.
down to alarmingly low levels Arguments :
in some parts of the country (A) No, big cities in India do not
where irrigation is primarily have adequate open land
dependent on groundwater, plots to accommodate the
which may lead to serious growing population.
environmental consequences. (B) Yes, only the builders and
(C) Yes, India just cannot afford developers benefit from the
to draw groundwater any construction of high rise
further as the international buildings.
agencies have cautioned
India against it. (C) Yes, the Government should
(a) Only (A) and (B) are strong first provide adequate
(b) Only (B) and (C) are strong infrastructure facilities to
(c) Only (A) and (C) are strong existing buildings before
(d) All (A) , (B) and (C) are strong allowing the construction of
(e) None of these new high rise buildings.
EBD_7420
216 Statement & Arguments
(a) Only (B) is strong (a) Only (A) and (B) are strong
(b) Only (C) is strong (b) Only (B) and (C) are strong
(c) Only (A) and (C) are strong (c) Only (A) and (C) are strong
(d) Only (A) is strong (d) All (A), (B) and (C) are strong
(e) None of these (e) None of these
4. Statement : Should road repair
work in big cities be carried out DIRECTIONS (Qs. 6-8) : Read the
only late at night ? following statements carefully and
Arguments : answer the questions which follow.
(A) No, this way the work will 6. The ministry of sports has been
never get completed. advised by a committee to take the
(B) No, there will be unnecessary highest award in the field of sports
use of electricity. back from two players who were
(C) Yes, the commuters will face allegedly-involved in match fixing.
lot of problems due to repair Which of the following statements
work during the day.
would weaken the argument put
(a) None is strong forward by the committee to the
(b) Only (A) is strong sports ministry?
(c) Only (C) is strong
(a) A good conduct in the past
(d) Only (B) and (C) are strong and a lack of evidence against
(e) Only (A) and (B) are strong the players make the case
5. Statement : Should all the deemed against them very weak.
universities be derecognised and
attached to any of the central of (b) The ministry of sports has
state universities in India ? never declined the
recommendations made by
Arguments :
the committee earlier.
(A) Yes, many of these deemed
universities do not conform to (c) Taking the award back from
the required standards of a the players would set a good
full – fledged university and example to other players for
hence the level of education avoiding such actions in the
is compromised. future.
(B) No, these deemed universities (d) There have been past cases
have been able to introduce where the award had to be
innovative courses suitable to taken back from the players
the requirement of various owing to some misconduct
industries as they are free from
later on.
strict Government controls.
(C) Yes, many such universities (e) The committee is constituted
are basically money spinning of some of the most respected
activities and education takes from the fields of sports and
a backseat in these institutions politics.
Statement & Arguments 217
7. Many organizations have been (a) Computerisation of bank
resorting to recruitment based branches in urban areas has
upon performance at graduate helped in making their
post-graduate level exams rather performance more efficient
than conducting exams for the and fast.
same purpose. (b) Lack of skilled and qualified
manpower has been suitably
Which of the following statements
substituted by computers in
would strengthen the argument
banks.
given in the above statement?
(c) Non-computerised ban k
(a) A recent study shows no link branches in the rural areas
of past performance with the have been proved to be as
performance in recruitment efficient as their computerized
exams. counterparts.
(b) The graduate/post-graduate (d) The government has
exams are considered to be introduced a special test for
severely deficient in training computer knowledge in all
in job related environment recruitment exams for banks.
(e) Unemployment in the rural
(c) Organisations which had
areas could be controlled by
undertaken recruitment on the
training more and more
basis of graduate / post-
professionals in computers.
graduate exams report a
significant drop in the quality DIRECTIONS (Qs. 9-10) : In making
of the recruited employees. decisions about important questions, it
is desirable to distinguish between
(d) Such policies would add to
‘strong’ argument and ‘weak’ argument.
unemployment amongst A ‘strong’ argument must be both
students having below important and directly related to the
average performance in question. A ‘weak argument may not be
graduation or post- directly related to the question and may
graduation. be of minor importance or may be related
(e) Such policies could save time, to the trivial aspect of the question. Each
money and resources of the question below is followed by two
organisation which are arguments, numberd I and II. You have to
wasted in the conduct of decide which of the arguments is ‘strong’
recruitment examinations. and which is ‘weak’
Give answer
8. According to a recent government
directive, all bank branches in rural (a) if only argument I is strong
areas should be computerized. (b) if only argument II is strong
Which of the following statements (c) if either I or II strong
would weaken the government’s (d) if neither I nor II is strong and
argument? (e) if both I and II are strong.
EBD_7420
218 Statement & Arguments
9. Statement Keeping in consideration concession to any particular group
the longivitity of life in India, should of students?
the age limit for retirement in
government jobs be increased? Argument
Argument I. Yes, this will improve the
quality of the professionals
I. Yes, other countries have as they will be able to
decided so long before. complete the courses
II. Yes, it is the actual demand of successfully.
lakhs of employees. II. No, this will keep large number
10. Statement Should the admission to of socially and economically
professional courses in India be backward students out of the
given only on merit without any reach of the professional
courses.

HINTS & SOLUTIONS


1. (a) Both argument (A) and (B) are with level of education
strong. Which clearly show the cannot be done.
importance of irrigation nad 6. (a) Option (a) would weaken the
environmental consequences argument put forward by the
of reducing groundwater level.
committee to the sports
Argument (c) is not strong.
ministry.
2. (c) Only Argument (B) is strong
7. (e) Option (e) would strengthen
because thermal power plants
the argument.
in India are one way to
increase environmental 8. (c) Option (c) would strengthen
pollution so cannot be the argument of government.
completly banned. Argument 9. (d) Both the arguments are weak.
(C) is based on example which Other countries have different
is a bad argument. conditions. And in India
3. (d) Argument (A) is strong population of youth is
because due to shortage of increasing so, for the benefits
space in big cities in India of youth age limit for retirement
high rise building should be should not be increased in
encouraged. government jobs.
4. (c) Only Argument (C) is strong 10. (b) Our country seaks to support
because to avoid the educationally and economically
inconvenience of commuters, backward classes for their
repair work is advisable in overall growth and develop-
night only. ment. Therefore, argument I is
5. (c) Only Argument (A) and (C) are not strong, in Indian context.
strong because compromise
Chapter
Statement &
27 Assumptions
INTRODUCTION Statement: “A” television — the largest
selling name with the largest range” —
Assumptions are essential part of an advertisement.
analytical reasoning. This is the reason Assumptions:
why in various competitive examinations, I. There is a demand for televisions in
examinees ar e asked to iden tify the market.
assumptions. In this chapter, we will see II. ‘A’ television is the only one with
how to identify assumptions. Before we wide variations.
go ahead, we must have a look at a The given statement in the problem
common format of the problem as it will format is an advertisement. This is
give you a clear idea of the questions to the one form of statement. But the
statement may be in different forms
be asked in the examination.
like it can be in the form of a passage;
in the form of a single line; in the
PROBLEM FORMAT (SAMPLE
form of a notice; in the form of an
PROBLEM) appeal or in any other different forms.
Directions: In every question given
below a statement (or a passage) is WHAT DOES AN
followed by two assumptions number I ASSUMPTION MEAN?
& II. An assumption is something Assumption is the hidden part of an
supposed or taken for granted. You have argument. It does mean that an
to consider the statement and the assumption is something which is
following assumptions and then decide assumed, supposed and taken for
which of the assumptions is implicit in granted. In fact, when a person says
the statement. something, he does not put everything
into words and leaves some part unsaid
Mark answer:
as why does he ? so?
(a) If only assumption I is implicit.
He does so because he takes this unsaid
(b) If only assumption II is implicit.
part for granted. In other words he thinks
(c) If eith er assumption I or this unsaid part will be understood
assumption II is implicit. without saying and hence there is no
(d) If neither of the assumption is need to put this (unsaid part) into words.
implicit It does mean this unsaid part is hidden
(e) If both the assumptions are in the given statement and this hidden
implicit. part is called assumption. Let us
EBD_7420
220 Statement & Assumptions
understand it in another way. Just
EXAMPLE 1.
remember your childhood days when
you used to solve the given arithmetic Statement: Of all the mobile sets
problem without leaving any single step. manufactured in India 'M' brand has the
But what you do today? Today your largest sale.
approach is totally different. Today you Assumption: The sale of all the mobile
leave easier steps as you assume that sets manufactured in India is known.
the person who see your solution, is Comment: The given assumption is
very much aware of these elementary valid. Here the statement makes a claim
operations. Therefore, this is an example that of all the mobile sets manufactured
of assumption. in India, 'M' brand has the largest sale.
To get the concept of assumption more In fact, without knowing sale figures may
clearly just suppose a thrilling one day be rough data of all mobile brands
international cricket match is going on manufactured in India, no such claim
between India and Australia. The about M brand could be made. Hence, it
Australian team has scored 300 runs but must have been implicitly assumed in the
while chasing the score India has made given statement that sale figure of all
280 runs in 48 overs and now, the brands is known.
situation is India has to score 21 runs to EXAMPLE 2.
win the match in remaining two overs. Statement: Virat is in great form and
As Yuvraj Singh is batting, you tell your therefore, India is going to beat New
friend - “No need to worry as Yuvraj is a Zealand in upcoming test series.
big hitter. India will definitely win the Assumption:
match”. What do you find in this I. Virat will give a good performance in
statement. In fact this statement has two upcoming series against New
parts:- Zealand.
(i) No need to worry as Yuvraj is a big II. Virat will score a triple century in the
hitter. upcoming series against New
(ii) India will win the match. Zealand.
Now, this is the time to think over these Comment: Assumption I is valid as the
two parts. How do you relate them? statement says that Virat is in great form
Obviously, by assuming that a big hitter and therefore, India is going to beat New
may score 21 runs in the remaining two Zealand in the upcoming test series. It
overs. Therefore, this is another example does mean that it is assumed in the
of assumption. The above statement can statement that Virat will perform well in
be written in three parts as follows:- the upcoming test series against New
(i) No need to worry as Yuvraj is a big Zealand and on the basis of that good
hitter. performance India will beat New Zealand.
(ii) A big hitter may score 21 runs in 2 But II is invalid because if Virat is in great
overs (Hidden part/Assumption) form, that does not mean he will surely
(iii) So, India will win the match. hit a triple century. He may or may not
Let’s get more ideas about assumption do so. Hence, assumption II is not hidden
with some simple examples given below:- in the statement.
Statement & Assumptions 221
EXAMPLE 3. EXAMPLE 5.
Statement: The next meeting of the Statement: The crisis of onion has
governing body of the institute X will be worsened and the government should
held after one year.
make every effort to boost import of onion.
Assumption: Institute X will remain in
Assumption:
function after one year.
I. Import is the best solution to avert
Comment: The given assumption is valid
the onion crisis.
as we know that the common practice is
II. Import is a reasonably good solution
to hold meetings of only those bodies
to the onion crisis.
that are functional. Hence, it does mean
III. Import is the only solution to
that the announcer must be assuming
overcome the onion crisis.
that the society will remain functional
after one year. IV. The onion crisis will definitely be
averted by boosting import of onion.
EXAMPLE 4.
V. The onion crisis will probably be
Statement: The student is too clever to averted by boosting import of onion.
fail in the examination. Comment: In the above mentioned
Assumption: Very clever students do not example, the assumption II and V are
fail in the examination. valid. But I, III and IV are not valid. The
Comment: This is a valid assumption. As reason is that there is use of definitive
per the given statement the student will words (best, only and definitely) in case
not fail (This is an effect) as he / she is of I, III and IV. The given statement
very clever (This is a cause). Clearly, it mentions a fact that crisis of onion has
has been assumed in the statement that worsened and then makes a suggestion
very clever students do not fail. that imports of onion should be boosted.
In fact the statement assumes that import
HOW DOES A SINGLE WORD should help to overcome onion crisis or
OR PHRASE MAKE A that import is a good/ reasonably good
DIFFERENCE? solution to the onion crisis. But, there is
no hint that import is the only solution/
A. Definitive Words Cases: best solution/a definitely effective
Just consider the words like ‘all’, solution.
‘only’, ‘best’, ‘strongest’, ‘certainly’, Therefore, the example given above
‘definitely’, etc. These are some illustrates how a definitive word may give
words that put a greater degree of a different ‘tone’ to a sentence.
emphasis or more weight on the
sentence than some others. In fact, B. Cases of Conjunctions:
these words impart a kind of The words like ‘because’,
exclusiveness to the sentence and ‘therefore’, ‘in spite of’, ‘despite’,
thereby reduce the scope / range of ‘so’, ‘after’, ‘even’, ‘although’ ‘as’,
the sentence. In fact, some kind of ‘as a result of’ are some significant
certainty is associated with all these conjunctions. When a statement has
words. Let us consider the following two clauses and the clauses are
examples: connected by a conjunction, then
EBD_7420
222 Statement & Assumptions
the nature of conjunction helps in EXAMPLE 9.
detecting the assumption that the Statement: There was no outbreak of any
author suggests in his statement. epidemic even after the continuous
Suppose ‘x’ is one clause of a deposition of rain water for six days.
sentence that mention an event (or Valid Assumption: Deposition of rain
fact/suggestion) and ‘y’ is the water usually leads to epidemic.
another clause of the same sentence
which mentions another event (or C. Cases of Connotive Phrases:
fact/suggestion), than depending Sometimes words used by th e
upon the conjunction, we can author are slightly indirect or
conclude the following assumption. unconventional. This is the reason
(i) x because/ as a result of y Þ It is you may miss the thing which the
assumed that ‘y’ leads to x. author wants to say. Such indirect
EXAMPLE 6. or unconventional words are called
Statement: You will find improvement in connotative or connotive phrases.
your English after taking classes in For example “It is true that ....” can
institute M. be put / written as:
Valid Assumption: Institute M may (i) It can be claimed with reasonable
help in improving English. degree of truth that...
(ii) x therefore/ hence y Þ It is (ii) It would be correct to say that...
assumed that ‘x’ leads to ‘y’. (iii) Even the most sceptic of men
EXAMPLE 7. would agree that....
Statement: Sachin Tendulkar has become Similarly, “It is false” is put / written by
the 1st man to score 50th test century, the author as :
therefore all Indians must be feeling very (i) It is baseless to say that ...
proud on his achievement. (ii) It would be highly misleading
Valid Assumption: An achievement by a to say that....
fellow countryman makes other citizens (iii) Nothing could be farther from
proud. truth than...
(iii) x even after/ despite/ in spite of y Þ
It is assumed that usually x does not Note: The role of connotative phrases is
occurs when y occurs. very limited in the questions asked
because they are given so that they do
EXAMPLE 8. not escape your eyes whenever one
Statement: There was a theft in the city come across them.
mall last night inspite of the maximum
security arrangement made by the police. Rules Related to Assumptions
Valid Assumption: Maximum security (i) Any form of public interest notice/
arrangement is usually sufficient to official notice is assumed to be paid
prevent theft. attention to.
(iv) Not ‘x’ even after/ in spite of/ despite (ii) If an appeal is made, it is assumed
‘y’ Þ It is assumed that usually x that the appeal will get response.
occurs when y does.
Statement & Assumptions 223
(iii) Any form of advertisements are Invalid Assumption: Curious persons are
given by assuming that people will good persons.
respond to such materials. (c) Observation : It is slightly different
from the restatement case. In this
(iv) If X says something to Y, it means X
case, two of the trio (Subject, verb,
assumes that B will hear what he (X) predicate) are changed into negative
says. that changes the appearance of the
Conditions for Invalidity of Assumptions: sentence without changing its
(a) Restatement meaning.
If the given assumption is a EXAMPLE 12.
restatement of the given statement, Statement: Beauty is lovable.
then the given assumption will be Invalid Assumptions :
invalid. In fact, in such case, same I. Ugliness is not lovable
thing is put in different words. II. Beauty is not hateable
EXAMPLE 10. (d) Conversion : When you study the
chapter of syllogism, you see that
Statement: Of all the computer brands,
statements are converted to get
manufactured in India, brand M has the
immediate inference. In fact, there
largest sale.
are three standard cases of
Invalid Assumption: No other brand of
conversion:
computer has as high a sale as brand M.
(i) All M are N, converted into
(b) Long-drawn Conclusion: If an
Some N are M.
assumption makes too far fetched
(ii) Some M are N, converted into
logic or long drawn conclusion, then
Some N are M.
it will be considered as invalid
(iii) No M are N, converted into No
assumption.
N are M
EXAMPLE 11. Points to be noted that given
Statement: All teaching should be done assumptions will be invalid if they are
in religious spirit as religious instruction conversions of the given statements.
leads to a curiosity for knowledge.

q Shortcut Approach
Assumption will be implicit if Assumption will not be implicit if
· it is in context of passge · not in context of statement or passage
· it is not directly mentioned · it is directly mentioned in the statement
· it is a mandatory factor condition for · it is not an accepted fact or cannot be
the statement to be correct. truly inferred
· there is use of definitive words
Note : The assumption must follow all
· it is a restatement or a long-drawn
conclusion or negative rephrasing or
the above rules for it to be implicit.
a converted syllogism form.
EBD_7420
224 Statement & Assumptions

PRACTICE EXERCISE
DIRECTIONS (Qs. 1-5) : In each (c) Only (B) is implicit
questions bellow is given a statement (d) Only (C) is implicit
followed by three assumptions (A), (B)
and (C). An assumption is something (e) Only (B) and (C) are implicit
supposed or taken for granted. You have 2. Statement : The apex body
to consider the statement and the controlling universities in the
following assumptions and decide country has decided to revise the
which of the assumptions is implicit in syllabus of all the technical
the statement. courses to make them focused
1. Statement : Police authority towards the present needs of the
industr y thereby making the
cordoned of the entire locality for
technical graduates more
the entire day and stopped
employable than they are at
vehicular movement for the visit of
present.
a top functionary of th e
government in view the threat Which of the following
perception and advised all the assumption(s) is /are implicit in the
residents in the area to limit their above statement ?
movement outside their dwellings. (A) Technical colleges affiliated
Which of the following to different universities may
assumption(s) is /are implicit in the not welcome the apex body's
above statement ? decision and may continue
with the same syllabus as at
(A) Police personnel may not be
present.
able to control the vehicular
movement in the locality and (B) The industry may welcome
may seek help from the armed the decision of the apex body
forces. and scale up their hiring from
these colleges.
(B) People living in the locality
may move out of their houses (C) The Government may not
for the day to avoid allow the apex body to
inconvenient. implement its decision in all
the colleges as it may lead to
(C) The Government functionary chaos.
may request the police
authority to lift the ban on (a) None is implicit
movement of residents of the (b) Only (A) is implicit
locality outside their
dwellings. (c) Only (B) is implicit
(a) None is implicit (d) Only (C) is implicit
(b) Only (A) is implicit (e) Only (A) and (B) are implicit
Statement & Assumptions 225
3. Statement : Government has urged (B) Many private entities in the
all the citizens to use electronic coun try are capable of
media for carrying out their daily constructing highways within
activities, whenever possible reasonable time.
instead of using paper as the (C) The Government's proposal
manufacture of paper requires the of build–operate–transfer
cutting down of a large number of may financially benefit the
trees causing severe damage to the private entities.
ecosystem.
(a) Only (A) and (B) are implicit
Which of the following
(b) Only (B) and (C) are implicit
assumption(s) is /are implicit in the
above statement ? (c) Only (B) is implicit
(A) Most people may be capable (d) Only (A) and (C) are implicit
of using electronic media to (e) None of these
carry out various routines. 5. Statement : The airlines have
(B) Most people may have access requested all their bona fide
to electronic media for passengers to check the status of
carrying out their daily routine flight operations before leaving
activities. their homes as heavy fog is causing
(C) People at large may reject the immense problems to normal flight
Governments appeal and operations.
continue using paper as Which of the following
before. assumption(s) is /are implicit in the
(a) Only (A) is implicit above statement ?
(b) Only (B) is implicit (A) The majority of the air
passengers may check the
(c) Only (A) and (B) are implicit
flight status before starting
(d) Only (C) is implicit
their journey to the airport.
(e) None of these
(B) The Government may take
4. Statement : Government has serious objection to the notice
decided to auction construction of issued by the airline company.
highways to private entities in
(C) Majority of the passengers
several blocks across the country
may cancel their tickets and
on build–operate–transfer basis.
postpone their journey till the
Which of the following situation becomes normal.
assumption(s) is /are implicit in the
(a) None is implicit
above statement ?
(b) Only (A) is implicit
(A) An adequate number of
private entities may not (c) Only (B) is implicit
respond to the Government's (d) Only (C) is implicit
auctions notification. (e) Only (A) and (C) are implicit
EBD_7420
226 Statement & Assumptions
DIRECTIONS (Qs. 6-10): In each 8. Statement : “Our Europe Holiday
question below is given a statement Package costs less than some of
followed by two assumptions numbered the holiday Packages within the
I and II. An assumption is something country” - An advertisement by an
supposed or taken for granted. You have Indian travel company.
to consider the statement and the Assumptions:
following assumptions and decide
I. People may prefer to travel to
which of the assumptions is implicit in
foreign destinations than to
the statement.
the places within the country
Give answer (a) if only Assumption I is at comparable cost.
implicit.
II. People generally take their
Give answer (b) if only Assumption II is
travel decisions after getting
implicit.
information from such
Give answer (c) if either I or II is implicit.
advertisements.
Give answer (d) if neither I nor II is
9. Statement : The retail vegetable
implicit.
vendors increased the prices of
Give answer (e) if both I and II are
vegetables by about 20 percent due
implicit.
to non availability of vegetables at
6. Statement : A nationalised bank
lower prices at the wholesale
issued an advertisement in the
market.
national dailies asking the eligible
candidates for applying for 100 Assumptions:
posts of chartered accountants. I. The customers may totally
Assumptions : stop buying vegetables at
I. The eligible chartered higher prices.
accountants may respond to II. The customers may still buy
the advertisement vegetables from the retail
II. There may be adequate vendors.
number of eligible chartered 10. Statement : A large number of
accountants who may want to
students and parents stood in the
join a nationalized bank.
queue to collect forms for admission
7. Statement : The municipal
to various undergraduate courses
authority announced before the
in the college.
onset of monsoon that the roads
within the city will be free of Assumptons :
potholes during monsoon. I. The college authority may be
Assumptions: able to admit all those who
I. The roads were repaired so stood in the queue.
well that potholes may not II. The college authority may
reappear. have adequate number of
II. People may not complain forms for all those standing
even if the potholes reappear. in the queue.
Statement & Assumptions 227

HINTS & SOLUTIONS


1. (a) None of the Assumptions (A), 5. (b) Only Assumption (A) is implicit
(B) and (C) is implicit in the in the statement. Any appeal/
statement. If police authority has request is issued assuming that
cordoned off the entire locality, some people will pay heed to it.
it implies that police will ably 6. (e) Both the assumptions are implicit
control the vechicular because daily newspapers
movement in the locality. provide instant reach. Again 100
vacancies of Chartered
It is assumed that any advice
Accountants were announced
given to the people will be
assuming sufficient eligible
followed.
candidates may join the
2. (c) Only Assumption (B) is implicit nationalised bank.”
in the statement. 7. (a) Only assumption I is implicit
The apex body controlling because repairing of roads is
universities has taken the carried out so efficiently to
decision assuming that technical leave potholes. Assumption II
colleges will honour it. is not implicit because it is
people’s right to complain
3. (c) Only Assumption (A) and (B) against any pothole.
are implicit in the statement..
8. (e) Both the assumption are implicit
Government urged all the
because main consideration for
citizens to use electronic media
people is cost factor. People
assuming that most people are
would prefer foreign destination
capable of using electronic at competitive prices.
media. Advertisement provides
4. (c) Only Assumption (B) is implicit sufficient information on this.
in the statement. If the 9. (b) Only assumption II is implicit
Government has decided to because customers prefer to buy
auction construction of vegetables from retail vondors
highways to private entities, it as there is a lot of innovation in
may be assumed that many retail sector.
private entities in the country 10. (b) Only assumption II is implicit
are capable of constructing because college authority
highways within reasonable cannot admit all those standing
time. in the queue.
EBD_7420
228 Statement & Conclusions

Chapter
Statement &
28 Conclusions
INTRODUCTION EXAMPLE 2.

In this type of questions, a statement is Statement : Today, out of the world


given followed by two conclusions. We population of several thousand million,
have to find out which of these the majority of men have to live under
conclusions definitely follows from the government which refuses them personal
given statement. liberty and the right to dissent.
Conclusions :
WHAT IS A ‘CONCLUSION’? I. People are indifferent to personal
liberty and the right to dissent.
‘Conclusion’ means a fact that can be II. People desire personal liberty and
truly inferred from the contents of a given
the right to dissent.
sentence. Conclusion is the art of
Sol. (b) It is mentioned in the
judging or deciding, based on reasoning.
statement that most people
DIRECTIONS (for Examples 1 to 3) : In
are forced to live un der
each of the following questions, a
governments which refuse
statement is given followed by two
conclusions I and II. Give answer : them personal liberty and right
(a) if only conclusion I follows; to dissent. This means that
(b) if only conclusion II follows; they are not indifferent to
(c) if either I or II follows; these rights but have a desire
(d) if neither I nor II follows; for them. So, only II follows.
(e) if both I and II follows; EXAMPLE 3.
EXAMPLE 1. Statement : It has been decided by the
Statement : The oceans are a store house Government to withdraw 33% of the
of practically every mineral including subsidy on cooking gas from the
uranium. But like most other minerals, it is beginning of next month—a spokesman
found in extremely low concentration – of the Government.
about three gms per 1000 tonnes of water. Conclusions :
Conclusions : I. People no more desire or need such
I. The oceans are a cheap source of subsidy from government as they
uranium. can afford increased price of the
II. The oceans harbour radiation hazards. cooking gas.
Sol. (d) I. Uranium is found in extremely II. The price of the cooking gas will
low concentration in oceans. increase at least by 33% from the next
Hence oceans are not a cheap month.
source of uranium. II is out of Sol. (d) I does not follow because a
context of the sentence. govt’s policy is not determined
merely by people’s needs.
Statement & Conclusions 229
II does not follow. Let the II. The government of country X seems
present price be x to be serious in attracting tourists.
\ Price if subsidy is removed Sol. (e) Clearly, the government has
x taken the step to attract more
= = 1.49x tourists. So, both I and II follow.
0.67
Hence increase in price will be
around 49%
q Shortcut Approach
DIRECTIONS (for Examples 4 to 5) : In 4 GOLDEN RULES.
each of the following questions, a 1. The conclusion must be in context
statement is given followed by two of the statement. If out of context
conclusions I and II. Give answer : then it does not follow.
(a) if only conclusion I follows; 2. The conclusion must support the
(b) if only conclusion II follows; contents of the statement. If it
(c) if either I or II follows; negates then it does not follow.
(d) if both I and II follow. 3. The conclusion must be truly
(e) if neithter I nor II follows; inferred. If there is some doubt that
it may or may not be correct or truly
EXAMPLE 4. inferred, then it does not follow.
Statement : Interest rate will be fixed on 4. The conclusion must not repeat or
the basis of our bank’s rate prevailing rephrase the statement. If so, it does
on the date of deposit and refixed every not follow.
quarter thereafter. Now let us apply these rules to the 5
Conclusions: examples solved above.
I. It is left to the depositors to guard Ex. 1 I. Rule 2 applies as it negates the
their interest.
statement.
II. The bank’s interest rates are subject
II. Rule 1 applies as it is out of
to change on a day-to-day basis
depending on market position. context.
Sol. (b) I does not follow because the Ex. 2 I. Rule 2 applies as it negates the
statement is silent about the statement.
depositors. II follows from the II. Fulfils all the conditions in Rule
phrase “bank’s rate prevailing 1-4.
on the date of deposit” which Ex. 3 I. Rule 1, 2 & 4 follow but 3 does
means the rates are subject to not as there can be various
day-to-day changes. reasons to withdraw subsidy.
EXAMPLE 5.
II. Rule 1, 2 & 4 follow but 3 does
not as the price increase is
Statement : The government of country actually 49%
X has recently announced several
Ex. 4 I. Rule I applies as it is out of
concessions and offered attractive
context.
package tours for foreign visitors.
Conclusions : II. Follows all the 4 rules perfectly.
I. Now, more number of foreign Ex. 5 Both I & II follow all the 4 rules
tourists will visit the country. and hence follow the statement.
EBD_7420
230 Statement & Conclusions

PRACTICE EXERCISE
DIRECTIONS (Qs. 1-10): In each question 3. Statements: Prime age school-
below is given a statement followed by going children in urban India have
two conclusions numbered I and II. You now become avid as well as more
have to assume everything in the regular viewers of television, even
statement to be true, then consider the in households without a TV. As a
two conclusions together and decide result there has been an alarming
which of them logically follows beyond decline in the extent of readership
a reasonable doubt from the of newspapers.
information given in the statement. Conclusions:
I. Method of increasing the
Give answer: readership of newspapers
(a) If only conclusion I follows should be devised.
(b) If only conclusion II follows II. A team of experts should be sent
(c) If either I or II follows to other countries to study the
(d) If neither I nor II follows and impact of TV. on the readership
(e) If both I and II follow. of newspapers.
1. Statements: Government has 4. Statements: In Japan, the incidence
spoiled many top ranking financial of stomach cancer is very high,
institutions by appointing while that of bowel cancer is very
bureaucrats as Directors of these low. But Japanese immigrate to
institutions. Hawaii, this is reversed - the rate
Conclusions: of bowel cancer increases but the
I. Government should appoint rate of stomach cancer is reduced
Directors of the financial in the next generation. All this is
institutes taking into related to nutrition - the diets of
consideration the expertise of Japanese in Hawaii are different
the person in the area of finance. than those in Japan.
II. The Director of the financial Conclusions:
institute should have expertise I. The same diet as in Hawaii
commensurate with the financial should be propagated in Japan
work carried out by the institute. also.
2. Statements: Population increase II. Bowel cancer is less severe than
coupled with depleting resources stomach cancer
is going to be the scenario of many 5. Statements: Monitoring has
developing countries in days to become an integral part in the
come. planning of social development
Conclusions: programmes. It is recommended
I. The population of developing that Management Information
countries will not continue to System be developed for all
increase in future. programmes. This is likely to give
II. It will be very difficult for the a feedback on the performance of
governments of developing the functionaries and the efficacy
countries to provide its people with which services are being
decent quality of life. delivered.
Statement & Conclusions 231
Conclusions: 8. Statements: Applications of
I. All the social development applicants who do not fulfill
programmes should be eligibility criteria and/or who do not
evaluated. submit applications before last
II. There is a need to monitor the date will be summarily rejected and
performance of workers. will not be called for the written test.
6. Statements: In a highly centralised Conclusions:
power structure, in which even I. Those who are called for the
senior cabinet ministers are written test are those who fulfill
prepared to reduce themselves to eligibility criteria and have
pathetic countries or yesmen airing submitted their applications
views that are primarily intended before last date.
to anticipate or reflect the Prime II. Written test will be held only
Minister's own performances, there after scrutiny of applications.
can be no place for any consensus 9. Statements: Although we have
that is quite different from real or rating agencies like Crisil, ICRA,
contrived unanimity of opinion, there is demand to have a separate
expressed through a well rating agency for IT companies to
orchestrated endorsement of the protect investors.
leader's actions. Conclusions:
Conclusions: I. Assessment of financial worth
I. The Ministers play safe by not of IT companies calls for
giving anti-government views. separate set of skills, insight
II. The Prime Minister does not and competencies.
encourage his colleagues to II. Now the investors investing in
render their own views. IT companies will get protection
7. Statements: The eligibility for of their investment.
admission to the course is 10. Statements: Wind is an
minimum second class Master's inexhaustible source of energy and
degree. However, the candidates an aero-generator can convert it
who have appeared for the final into electricity. Though not much
year examination of Master's has been done in this field, the
degree can also apply. survey shows that there is vast
Conclusions: potential for developing wind as
I. All candidates who have yet to alternative source of energy.
get their Master's degree will be Conclusions:
there in the list of selected I. Energy by wind is
candidates. comparatively newly emerging
II. All candidates having obtained field.
second class Master's degree II. The energy crisis can be dealt
will be there in the list of selected by exploring more in the field of
candidates. aero-generation.
EBD_7420
232 Statement & Conclusions

HINTS & SOLUTIONS


1. (e) According to the statement, 7. (d) The statement mentions that the
Government has spoiled financial candidates who have obtained
institutions by appointing second class Master's degree or
bureaucrats as Directors. This have appeared for the final year
means that only those persons examination of Master's degree,
should be appointed as can apply for admission. This
Directors who are experts in implies that both types of
finance and are acquainted with candidates may be selected on
the financial work of the institute. certain grounds. Thus, some
So, both I and II follow. candidates of each type and not
2. (b) The fact given in I is quite all candidates of any one type,
contrary to the given may be selected. So, neither I
statement. So, I does not follow. nor II follows.
II mentions th e direct 8. (e) The statement clearly mentions
implications of the state that fulfilling the eligibility
discussed in the statement. criteria and submitting the
Thus, II follows. application before the
3. (d) The statement concentrates on stipulated date are both
the increasing viewership of essential to avoid rejection. So,
TV. and does not stress either I follows. Also, since it is given
on increasing the readership of that the candidates whose
newspapers or making studies applications are rejected shall
regarding the same. So, neither not be called for written test,
I nor II follows. so II also follows.
4. (d) The statement neither 9. (a) The need for separate rating
propagates the diet of any of agency for IT companies
the countries nor compares the clearly indicates that such
two types of cancer. So, neither assessment requires a separate
I nor II follows. set of skills. So, I follows.
5. (e) According to the statement, However, the statement
monitoring and evaluation of indicates only the need or
social development demand and neither the future
programmes - their function, course of action nor its after-
performance and efficiency - is effects can be judged. So, II
absolutely essential. So, both I does not follow.
and II follow. 10. (e) The phrase 'not much has been
6. (a) According to the statement, done in this field' indicates that
even senior cabinet ministers wind energy is a comparatively
are always ready to conform to newly emerging field. So, I
the Prime Minister's views. follows. The expression 'there
So, I follows. However, II is vast potential for developing
contradicts the given statement wind as alternative source of
and so does not follow. energy' proves II to be true.
Courses of Action 233

Chapter

29 Courses of Action

INTRODUCTION NOTE : In the examinations more than


In many competitive examinations two courses of actions may also be given.
questions related to courses of action
are frequently asked. TYPES OF PROBLEMS
The basic reason behind asking such (1) Problems based on problem and
questions is to test your ability to judge
solution relationship.
a problem correctly in order to determine
(2) Problems based on fact &
the root of the given problem and then
improvement relationship.
finding out a proper course of action for
that particular problem. 1. Problems Based on Problem
and Solution Relationship
WHAT IS THE FORMAT OF
This is a case when the given
THE PROBLEM?
statement talks of a problem and the
Directions: In the question given below
suggested course of action talks of
is given a statement followed by two
suggested courses of action number I a solution. It is very easy to find out
and II. A course of action is a step or when a suggested course of action
administrative decision to be taken for is acceptable and when it is not. In
improvement, follow up, or further action fact, the suggested course of action
in regard to the problem, policy etc. On will be acceptable if:
the basis of the information given in the (a) it solves/ reduces or minimises the
statement. Read the situation carefully given problem
and then decide which of the given (b) it gives a practical and wise solution.
courses of action follow/ follows.
Now, what to do ? Just see the given
Mark answers:
(a) If only I follows problem with a serious eye; think over
(b) If only II follows that; apply your day to day experiences;
(c) If either I or II follows apply your common sense and use your
(d) If neither I nor II follows general knowledge to judge whether a
(e) If both I & II follow. suggested course of action solves or
Statement: The sale of a particular reduces or minimises the problem given
product ‘A’ has gone down considerably, in the statement. After this step, the next
causing great concern to company ‘X’. step is checking the practicality. Here,
Courses of action : you have to check if the solution
I. Company ‘X’ should make a proper
suggested by the given course of action
study of the rival products in the market.
II. The price of product ‘A’ should be is wise enough and applicable in
reduced. practical way in day to day life.
EBD_7420
234 Courses of Action
Infact (a) is the 1st step test and after Courses of action:
passing the step I test, the given course I. The child should be sent to child
of action will have to pass step II (which welfare society. (correct action)
is (b)). If the given course of action II. The child should be put in jail and
passes both the tests [step I and step II] severly beaten (wrong action)
only then it will be called a correct action. Comment: In example I, I is rejected as it
Step I test is an irrelevant action. It does not make
To pass the step I test a suggested it clear how instructing population for
course of action must be not coming out of their houses will solve
(i) based on an established fact or or reduce the problem of spreading
(ii) based on logical prediction or malaria. But II is a proper course of action
(iii) based on experiences as it is an established biological fact that
(iv) based on prevailing notions of truth malaria can be prevented by using
Let us discuss all the conditions safeguards against mosquitoes. This is
mentioned above:- the reason that II will go for further test
(step II test) proving itself a proper course
(i) Action based on established
of action in 1st level test (step I test).
fact: -
In example 2, II is rejected on the basis
In some of the cases an action taken that it is totally illogical to beat a child
is an established fact which and put into jail as a child is not mature
suggests that the given problem can enough to decide what is right and what
be reduced or solved by this is wrong. Further, it is an established fact
particular solution. It does mean that (socially established fact) that child
the solution suggested by the given criminals must not be treated as
course of action is universally punishable wrong doer but they should
acknowledged to the given problem. be made to mend their ways and on the
Let us see the examples given below: basis of this I is the correct course of
EXAMPLE 1. action. Hence, I will qualify for the 2nd
Statement: Southern part of India has round test ( Step II test)
been coming rapidly into the grip of (ii) Action based on logical
malaria. prediction :
Courses of action: In such type of cases, solutions
I. The Southern Indian population provided for the given problems are
must be instructed not to come out neither an established fact nor they
of their houses. [wrong action] can be considered as proper action
II. Anti-mosquito liquids should be on the basis of our past experiences.
sprayed in the southern part of Hence, in such cases examinees are
India. [correct action] required to apply certain logic and
EXAMPLE 2. reasoning to find out if the given
Statement: A child was caught while course of action solves or reduces
stealing money of a respectable person or minimises the problem. Let us see
of society. the example given below:
Courses of Action 235
EXAMPLE 3. Course of action: Efforts should be made
Statement: Jammu & Kashmir is that the Indians remain united for any
experiencing, again, the rise of terrorism eventuality. [correct action]
and it is obvious that Pakistan is Comment: Our past experiences say that
we (India) became a sufferer several times
encouraging it.
because of the foreign powers and at
Course of action: India must go to the
that time we lacked our unity. In another
international bodies with all the proof of
words, India has fallen victim to foreign
Pakistani involvement in Jammu &
powers only when our country (India)
Kashmir and demand that Pakistan must
has not remained united. Hence, on the
be declared a terrorist nation. [ correct
basis of our past experience, we can
action]
conclude that the given course of action
Comment : Here, the given course of
solves or reduces the problem making
action is the correct one at step I test. In
its entry for 2nd level (step II) test.
fact, it is a matter of simple logic of
diplomacy that in case of disturbances (iv) Action based on prevailing notions
created by a hostile nation within our of truth: In such type of cases
country, we put this issue before solutions provided for the given
international bodies so that the hostile problem is as per the social norms.
nation stands at disadvantage. Thus Ex. In other words, the given course of
3 will qualify for the next step test (step action suggests a solution that is
II or practicality test). prevailing notion of truth. In fact,
(iii) Action based on experiences: In they are the ideas that are universally
certain cases, while deciding if a accepted and acknowledged by the
given course of action solves or society and hence in many ways they
reduces or minimises the given are similar to established fact. Let us
problem, our experiences work. In see the following examples:
fact, in such cases the given problem EXAMPLE 5.
may be a relatively new one. It will Statement: Mr Sharma got angry and beat
not be totally new but it will not be his son mercilesely.
very old either. This is the reason Course of action : Mr.Sharma should be
that the solution can not be said as caned publicly [ wrong action]
an established fact. However, based
EXAMPLE 6.
on our past experiences, in the similar
kind of situation, we can reach the Statement : Most of manufacturing
conclusion that the given problem companies in India are running in losses.
can be solved/ reduced/ minimised Course of action: Prospects of
by this particular action. Let us see privatisation of these companies must
the example given below: be explored. [correct action]
Comment: In example 5, the given
EXAMPLE 4. solution is against the societal norm as
Statement: Several foreign powers public beating is not considered a good
having expansionist thinking are threat punishment. In other words, it is
to India. prevailing notion of truth that public
EBD_7420
236 Courses of Action
beating is not good. Hence, on the basis EXAMPLES FOR (A)
of this the given solution is rejected and
will not go for 2nd level test (step II test). EXAMPLE 7.
In example 6, the given course of action Statement : Lack of discipline is a good
suggests privatisation for loss making reason for low productivity in India.
manufacturing companies and no Course of action : Government must take
doubts, it is a prevailing notion of truth step to make military traing compulsory
that privatisation can reduce or minimise for all Indian citizens. [ wrong action]
their losses. There is also a chance that EXAMPLE 8.
privatisation can convert a loss making
Statement: As per the report of ‘WHO’
company into a profitable one. Hence,
(World Health Organisation) the life
we conclude the given solution is
expactancy of an average Indian is
correct one and will qualify for further
continuously declining.
test (2nd level test or step II test). Now,
Course of action : A serious effort must
we can move on to step II test.
be made to prevent children from making
Step II (Test of Practicality) noises. [wrong action]
This is the 2nd part of test. In the 1st Comment: In Example 7, the given course
part we just found out whether a of action is not a good solution for the
suggested action really solves/ reduces/ given problem. No, doubt that military
training wold be a solution for lack of
minimises the given problem. But an
discipline but is it a practical solution?
important part also remains to be checked
Your answer will be a big ‘No’ (why?). In
and that is the test of practicality. Point
reality, at the 1st step test the given
to be noted that a given course of action
course of action may seem true as it
may solve/ reduce/ minimise a particular
solves the given problem but when it
problem but if it is not practically
comes to the 2nd level test, it becomes
possible, it will be consider useless. This
clear that it is too severe solution for a
is the reason why this point too, needs
relatively small problem. Hence, on this
sound checking. For this you have to
basis the given course of action is
keep the following things in your mind:
rejected finally.
A. The problem and solution must be In example 8, the given course of
well matched and must be in action suggests that problem of
proportion. In other words, if declining life expectancy can be solved
solutions are too simple for too if children are prevented from making
severe problems, they will be noises. At one stage the given course of
useless. Conversly, we can say that action reduces the problem to some
too severe solutions are not good extent as it suggests that less noise will
solutions for too simple problems. increase the chances of low blood
B. Even after passing the step I test, pressure and this will result in less
the given solution is creating a new deaths. But when we think analytically,
problem, then the given solution we come to the conclusion that the
will not be a good solution and will problem is very serious and the given
fail in practicality test. solution is very simple for it. Hence on
Courses of Action 237
this basis the given course of action In the past we have also seen that such
would be declared a wrong one and steps have been taken. Not in the past
would be rejected finally. only even today whenever it seems that
mosquito born diseases are imminent,
EXAMPLE FOR (B)
the anti-mosquito liquids are sprayed.
EXAMPLE 9. Such step is taken only because it is
Statement: In recent years, people have practical. Here, the IInd course of action
developed a tendency of tax evasion and given under example 1 passes both the
this is the reason it has increased at an test to be finally declared as proper and
alarming level. correct solution.
Course of action : Government must Step II test of Example 2 [Course of
make law to abolish taxes. [ wrong action] action I]:
Comment: Here, the given problem is Ist course of action given under example
about tax evasion. Tax evasion does 2 is “child should be sent to child welfare
mean showing less income to pay less society”. In step II, we need to check if it
tax. Why tax evasion is a problem? is a practical solution. In so many cases
Because tax evasion generates black we have seen that when a child does a
money. The given course of action crime like stealing and some other more
serious crime, then they are put under
suggests the abolition of taxes which
such atmosphere that they can
connot be a good solution as taxes are
understand the seriousness of their
taken to provide people certain indirect
crime and try to mend their ways. For
services like the facilities of roads, parks,
such children, child welfare societies and
police etc. Suppose if taxes are not
some other such kind of organisations
charged, how and where from money will are very helpful. Hence, this course of
come to provide such indirect services action passes its final test to be declared
to community. No doubts, the tax a correct course of action.
abolition will create a new problem. Step II test of Example 3 :
Hence on this basis the given course of The course of action given under
action will be rejected finally as it fails example 3 is “India must go to the
the 2nd level test (step II test) of international bodies with all the proof of
practicality. Pakistani involvement in Jammu &
Now after understanding what is a Kashmir and demand that Pakistan must
practical solution, we can test the be declared a terrorist nation” and this is
courses of action that have passed the a very practical solution. As we have
step I test and given under examples 1, seen in certain circumstances in past that
2, 3, 4 and 6. India has put such type of demand from
Step II test of Example 1 ( Course of UNO and even from some other nations
action II): on individual basis. No doubts, that on
IInd course of action given under example such demands India has got support to
1 is “Anti mosquito liquids should be some extent. Hence it is a very practical
sprayed in the southern part of India". solution and this given course of action
In step II, we need to check if it is a passes it practicality test to be declared
practical solution for the given problem. a proper and correct course of action.
EBD_7420
238 Courses of Action
Step II test of Example 4 : (i) Find out whether the suggested
The given course of action “efforts course of action will help in
should be made that the Indians remain improvement of the situation.
united for any eventualities” is a practical (ii) Find out whether the two are
one as we have shown this type of unity properly balanced.
in the past. For example, in the freedom In fact problem given under example
struggle we were united. How this unity 7 is such type of problem.
took place? Only because this was Now we have come to the end of
practically possible. Hence, this given this chapter and this is the time to
course of action, too, passed the solve the problem given under 'what
practicality test to be declared finally a is the format of the problem'? Let us
proper and correct course of action. solve it:
Step II test of Example 6: Statement : The sale of a particular
The given course of action “Prospects product ‘A’ has gone down considerably,
of privatisation of these (loss making) causing great concern to company ‘X’.
companies must be explored is not a
correct solution at the end at the 2nd Courses of action :
level test (Practicality test) because the I. Company should make a proper
course of action and the given statement study of rival products in the
are not properly linked. The statement market.
does not make it clear that it talks only II. The price of product ‘A’ should be
about public sector manufacturing reduced.
concerns as even a private sector Solution. Option (a) is the correct option
manufacturing company may be a loss as only I follows.
making company. Hence the statement Reason /Explanation: If the sale of ‘A’
and given course of action creates has gone down, then there must be some
confusion. Therefore, the given course reasons. The company X must know this
of action is rejected at 2nd level test. reason. As I suggest the similar solution,
it follows. But II does not follow. The
2. Problem Based on Fact and
company should first know if price was
Improvement Relationship
a factor behind the drop in sale. Without
This is the 2nd type of problem knowing this, reducing price may turn
related to course of action. But point out to be a wrong and harmful action.
to be noted is that this does not
require any new skill. The solving Note : If you see 'an either choice' in the
method is exactly the same as you answer options avoid it. It will be a wrong
have solved the 1st type of problem answer. Either choice can be in the form
that is problem solution based. In like “Either of I or II (or III or I etc.)
fact you have to solve this type of follows”.
problem in two steps:
Courses of Action 239
q Shortcut Approach

Study the Statement

Problem Fact
Solution Type Improvement Type

Solves/minimises/reduces Does it help in improving


the problem the situation ?

Yes No Rejected Yes No Rejected


Check if it is a balanced Check if it is a balanced
or proportionate solution solution?

Yes No Rejected Yes No Rejected


Does it creates new Accepted
problems ?

Yes No Accepted

Rejected

PRACTICE EXERCISE
DIRECTIONS (Qs. 1-5): In each relief to the problem of acute water
crisis in the state.
question below is given a statement
followed by three courses of action Courses of action :
numbered (A), (B) and (C). A course of (A) The state government should
action is a step or administrative set up a committee to review
the alarming situation.
decision to be taken for improvement,
follow-up or further action in regard to (B) The state government should
immediately remove all the
the problem, policy, etc. On the basis of restrictions on use of potable
the information given in the statement, water in all the major cities in
you have to assume everything in the the state.
siatement to be true, then decide which (C) The state government should
of the suggested courses of action send relief supplies to all the
logically follow(s) for pursuing. affected areas in the state.
1. Statement: A heavy unseasonal (a) None
downpour during the last two days (b) Only (A)
has paralysed the normal life in the (c) Only (B) and (C)
state in which five persons were (d) Only (C)
killed but this has provided a huge (e) All (A), (B) and (C)
EBD_7420
240 Courses of Action
2. Statement : A large private bank 4. Statement: The school dropout rate
has decided to retrench one-third in many districts in the state has
of its employees in view of the increased sharply during the last
huge losses incurred by it during few years as the parents of these
the past three quarters. children make them work in the
Courses of action : fields owned by others to earn
(A) The Government should enough for them to get at least one
issue a notification to general meal a day.
public to immediately stop all Courses of action :
transactions with the bank. (A) The Government should put
(B) The Government should up a mechanism to provide
direct the bank to refrain from food grains to the poor people
retrenching its employees. in these districts through
(C) The Government should ask public distribution system to
the central bank of the encourage the parents to
country to initiate an enquiry send their wards to school.
into the bank's activities and (B) The Government should
submit its report. close down some of these
(a) None schools in the district and
(b) Only (A) deploy the teachers of these
schools to nearby schools
(c) Only (B) and also ask remaining
(d) Only (C) studen ts to join these
(e) Only (A) and (C) schools.
3. Statement : Many political (C) Government should issue
activists have decided to stage arrest warrants for all the
demonstrations and block traffic parents who force their
movement in the city during peak children to work in fields
hours to protest against the steep instead of attending classes.
rise in prices of essential (a) Only (A)
commodities. (b) Only (B)
Courses of action: (c) Only (C)
(A) The Government should (d) Only (A) and (B)
immediately ban all forms of
agitations in the country. (e) None of these
(B) The police authority of the 5. Statement : One aspirant was
city should deploy additional killed due to stampede while
forces all over the city to help participating in a recruitment drive
traffic movement in the city. of police constables.
(C) The state administration Courses of action :
should carry out preventive (A) The officials in charge of the
arrests of the known criminals recruitment process should
staying in the city. immediately be suspended.
(a) Only (A) (B) A team of officials should be
(b) Only (B) asked to find out the
circumstances which led to
(c) Only (C) the death of the aspirant and
(d) Only (A) and (B) submit its report within a
(e) None of these week.
Courses of Action 241
(C) The Government should ask 7. Statement: There is an alarming
the home department to increase in the number of people
stagger the n umber of suffering from malaria in many
aspirants over more number parts of the city.
of days to avoid such
Courses of action :
incidents in future.
(a) Only (A) I. The municipal corporation
has advised all the
(b) Only (B)
government hospitals to store
(c) Only (C)
adequate supply of malaria
(d) Only (B) and (C)
drugs.
(e) None of these
II. The municipal corporation
DIRECTIONS (Qs. 6-10): In each has urged people to use
question below is given a statement mosquito repellants and keep
followed by two courses of action their premises clean.
numbered I and II. A course of action is 8. Statement : Many people have
a step or administrative decision to be encroached into the government
taken for improvement, follow-up or property and built their houses and
further action in regard to the problem,
business establishments.
policy, etc. On the basis of the
information given in the statement, you Courses of action :
have to as some everything in the I. The government should take
statement to be true, then decide which immediate steps to remove all
of the suggested courses of action unauthorised constructions
logically follow(s) for pursuing. on government land.
Give answer (a) if only course of action I II. All the encroachers should
follows. immediately be put behind
Give answer (b) if only course of action bars and also be slapped with
II follows. a hefty fine.
Give answer (c) if either course of action 9. Statement : The meteorological
I or II follows. department has predicted normal
Give answer (d) if neither course of action rainfall throughout the country
I nor II follows. during the current monsoon.
Give answer (e) if both courses of action Courses of action :
I and II follow.
I. The government should reduce
6. Statement : Drinking water supply
to many parts of town is disrupted the procurement price of
due to loss of water because of foodgrains for the current year.
leakage in pipes supplying water. II. The government should
Courses of action : reduce subsidy on fertilizers
I. The government should order for the current year.
an enquiry into the matter. 10. Statement : The number of
II. The civic body should set up dropouts in government schools
a fact-finding team to assess has significantly increased in the
the damage and take effective urban areas over the past few
step. years.
EBD_7420
242 Courses of Action
Courses of action : II. The parents of all the
I. The government should students who dropped out of
immediately close down all the government schools in
such schools in the urban urban areas should be
areas where the dropout goes punished.
beyond 20 per cent.

HINTS & SOLUTIONS


1. (d) Immediate course of action to leakage matter would not solve
avert the difficult situation is to any purpose and secondly no
send relief rupplics in affected need of setting up fact-finding
areas course of action I is a long team when cause of leakage is
term action plan which is time known to us.
consuming and course of action 7. (e) Both the course of action I
II is not feasible practically. and II should be followed
2. (a) None of course of action is because sufficient supply of
feasible because the malaria drugs is necessary to
government cannot interfere in avoid malaria. Secondly use of
bank’s internal matter but the mosquito repellant and
government can componsate cleanliness are necessary to
the huge losser incurred by avoid mosquito breeding.
bank. 8. (a) Only I course of action is
3. (b) The course of action (A) and (C) suitable to follow because the
are not feasible because in a government should check all
democratic country the unauthorised construction on
government cannot impose ban government land. II course of
on these things. action is too harsh to be
4. (a) Course of action (A) is only suitable.
feasible solution because the 9. (d) Both the courses of action are
government can neither close not suitable because when
these schools nor issue arrest there is no problem, no action
warrants for all the parents. is required.
5. (d) Clearly course of action (B) and 10. (d) Both the course of action are
(C) should be followed because not practically possible
terminating official in charge of because government cannot
recruitment procers is not a close down all such schools
solution to the problem. and also parents of such
6. (d) Both of the course of action children cannot be punished
are not necessary because because there may be genuine
ordering an enquiry into the reason for dropouts.
Chapter
Critical Reasoning
30
INTRODUCTION to manipulate the argument to weaken/
strengthen it, find the conclusion,
Critical Reasoning (CR) is ability to assumption, explanation, do an inference
reason clearly to evaluate and judge or supplement a statement, etc.
arguments. You are using this skill a lot Whatever it is that you have to do, you
during your everyday life while reading will need 2 things to succeed: know the
newspapers or watching movies. When basic structure of arguments and clearly
you think that the movie is pushing the understand the argument.
limit of the Reasonable or the news In general, most of them, arguments
sounds less reasonable than the movie consist of evidence, usually 2 pieces, a
that was pushing the limit, you are using conclusion - the main point of an
your Critical Reasoning skills to produce argument, and an assumption - the
these conclusions. The argument you bridge between the evidence and
meet can be anything from a classical conclusion. The majority of the
argument to an advertisement or a dialog. arguments you encounter on the test will
Critical Reasoning questions will ask you be 3 step arguments:
Evidence 1 + Evidence 2 = Conclusion.
sumptio
As n

E1 + E2 = C
EXAMPLE 1. Last week Mike was EXAMPLE 2. There are a lot of
detained for shoplifting at a groceries mosquitoes outside today, please do not
store near his house, but he has been a turn on the light in the room because a
Christian for 10 years, therefore, the lot of them will fly in.
police must have been wrong accusing
Note : Here the evidences are ‘there
him in stealing. are a lot of mosquitoes outside today’
Note : There are two pieces of evidence: and ‘do not turn on the light’. The
‘Mike was accused of stealing’ and that conclusion is that ‘Many will fly in’ and
‘he is a Christian’. The conclusion is the assumption is ‘mosquitoes will
that ‘the police are wrong’. Therefore, approach the light.’
our huge assumption here is that ‘a There is no set scheme for structure in
Christian could not have stolen CR, but since the majority of the
anything.’ arguments are only a few sentences long,
EBD_7420
244 Critical Reasoning
the conclusion usually comes in the first text and as simple as possible so that
or the last sentence. However, some of you would understand it easily and
the arguments encountered will not have at the same time could fully trust it.
a conclusion at all or will have just an Do not make it too general nor too
implied one. detail oriented. When you do a
paraphrase, do it in three steps:
STRATEGY TO CRACK CRITI- Evidence1, Evidence2, and
CAL REASONING QUES- Conclusion; put “therefore” word
TIONS before you start your conclusion,
This strategy is not the easiest way to this will help you to set it off.
do CR (the easiest would be read-and- 4. Read the question again (now with
answer), but it lets you get the most more understanding of what is being
questions right spending less time per asked; reading the question 2 times,
correct answer. it will also help you to make sure your
1. Read the questions first; this is answer exactly what is stated and
needed so that you would know that you understand the question.)
what to look for and what to do: find 5. Answer before reading the answer
an assumption, strengthen/weaken, choices. There are two reasons for
infer something or else; do not worry this :
about the details in the question, (i) if you can think of the correct
read for keywords, such as answer or at least the general
strengthen, deny, or explain. [Use direction that the answer choice
symbols for convenience, e.g. + for needs to be, you will identify it
strengthen or – for weaken]. among the wrong choices much
2. Read the passage very attentively faster, thus spend less time
because in contrast to Reading reading the answers, which
Comprehension, there is very little usually take 30 seconds to cover.
text here and mostly everything is (ii) Often students are seduced by
important; try to read only once. the author’s wording. One reads
Reread if required. a few words that were used in
As you read, look for the problem in the passage and the brain
the passage (evaluate how identifies this choice with the
convincing it is)
passage, thus making it seem
3. Paraphrase (reword) the passage. It
more right that it needs to be.
is a very important step because
when you do a paraphrase, you The more problems you practice
check whether you understood the with, the more chance is you will
passage and at the same time you guess the right answer even
extract the skeleton of the argument, before reading it.
making it easier to identify the 6. Go through the answers, first time
conclusion and the assumption. Very scan them for YOUR answer choice
often, the paraphrase of the passage (usually you will guess correctly in
will be pretty close to the conclusion. 60-70% of cases), if you did not find
It is not surprising, since the it, reread them more attentively.
conclusion is the main point and 7. Draw a grid to eliminate the wrong
evidence just supports it.) Your answers easier. Use “ü” for a sure
paraphrase should be as close to the answer, “û” for a definitely wrong
Critical Reasoning 245
answer choice, and “?” for an
answer th at may be right or q Shortcut Approach
questionable. This will help to How to tackle “Identify the inference /
concentrate only on a few answer Must be true questions”:
choices and will prevent you from • Read the stimulus and look for the
reading same answers several times argument.
if you get confused or keep having • Note that Must Be True questions
troubles locating the right answer. may not contain an argument. They
may just be a series of facts.
TYPES OF CRITICAL Nevertheless, try to find the
REASONING QUESTIONS argument.
• Avoid choices which contain
Critical reasoning questions will ask you absolute statements - never, always,
to: none, only etc. Although these words
1. Identify the inference / Must be true might appear in some correct choice,
question you should be very sure about them.
2. Identify the assumption. • Some of the options can be eliminated
3. Strengthen an argument. as they go beyond the scope of the
4. Weaken an argument. passage. Note that an inference can
5. Select the best conclusion/Main Point be based on only some of th e
6. Identify the paradox information provided and not the
complete passage.
7. Evaluation/ Reasoning
8. Identify a parallel argument/Structure.
EXAMPLE 1. Stimulus Argument
1. Identify the Inference /
Increases in funding for police patrols
Must be True Question often lower the rate of crimes of
These type of questions are opportunity such as petty theft and
extremely common. An Inference van dalism by providing visual
means the same thing as “must be deterrence in high-crime neighborhoods.
true”. Conclusions differ from Levels of funding for police patrols in
inferences in that conclusions are some communities are increased when
the result of premises and inferences federal matching grants are made
available.
are something that must be true. The
Question : Which of the following can
following are the typical Inference
be correctly inferred from the statements
(Must be true) based Questions: above?
• If the statements above are true, Options :
which of the following must also (a) Areas with little vandalism can never
be true? benefit from visual deterrence.
• Which of the following is (b) Communities that do not increase
[implied, must be true, implicit, their police patrols are at higher risk
most reasonably drawn] in the for crimes of opportunity late at
passage above? night.
• Which of the following inferences (c) Federal matching grants for police
is best supported by the patrols lower the rate of crimes of
statement made above? opportunity in some communities.
EBD_7420
246 Critical Reasoning
(d) Only federal matching grants are • Which of the following most
necessary to reduce crime in most accurately states a hidden
neighborhoods. assumption that the author must
(e) None of these make in order to advance the
Sol. argument above?
(c) is a summary of the information • Which of the following is an
provided; it is the logical end of a assumption that, if true, would
chain of reasoning started in the support the conclusion in the
stimulus argument. The sequence of passage above?
events goes like this :
Increased funding ® Increased q Shortcut Approach
visual deterrence ® Lower crime How to approach “Identify the
The last statement could be mapped assumption Questions”
as follows: • Look for gaps between the premises
Federal grants ® Increased patrol and the conclusion. Ask yourself
funds why the conclusion is true. Before
(c) makes the chain complete by you progress to the answer choices,
correctly stating that federal grants try to get feel of what assumption is
can lead to lower crime in some necessary to fill that gap between the
communities. Now the logical chain premises.
becomes: • Beware of extreme language in the
Federal grants ® Increased answer choices of assumption
funding ® Increased visual questions. Assumptions usually are
deterrence ® Lower crime not extreme. “Extreme” answer
The other answer choices may not choices usually contain phrases
be correctly inferred because they such as always, never, or totally.
go beyond th e scope of the
argument. They may be objectively, EXAMPLE 2. Stimulus Argument
factually correct, or they may be Traditionally, decision making by
statements that you would tend to doctors that is carefully, deductively
agree with. However, you are limited reasoned has been considered preferable
to the argument presented when to intuitive decision making. However, a
choosing a correct answer. recent study found that senior surgeons
2. Identify the Assumption used intuition significantly more than did
most residents or mid-level doctors. This
An assumption is an unstated confirms the alternative view that
premise that supports the author’s intuition is actually more effective than
conclusion. It’s the connection careful, methodical reasoning.
between the stated premises and the Question : The conclusion above is based
conclusion., which together forms on which of the following assumptions?
the passage. An assumption is Options :
something that the author ’s (a) Senior surgeons are more effective
conclusion depends upon. at decision making than are mid-level
Assumption questions are extremely doctors.
common and have types that look (b) Senior surgeons have the ability to
like this: use either intuitive reasoning or
Critical Reasoning 247
deductive, methodical reasoning in q Shortcut Approach
making decisions.
(c) The decisions that are made by mid- How to approach “Strengthen an
level and entry-level doctors can be argument”
• Once you have identified the argument
made as easily by using methodical
of the passage, i.e. the evidence(s) +
reasoning as by using intuitive
conclusion, try putting in each option
reasoning. with the argument. Check if the
(d) Senior surgeons use intuitive assumption(s) you have drawn is (are)
reasoning in making the majority of strengthened if you accept the content
their decisions. of the option as true.
(e) None of these
Sol. EXAMPLE 3. Stimulus Argument
(a) The correct answer is (a), which Three years after the Bhakra Nangal Dam
provides a missing link in the was built, none of the six fish species
author’s reasoning by making a native to the area was still reproducing
connection from the evidence: that adequately in the river below the dam.
intuition is used more by senior Because the dam reduced the average
surgeons than other, less- temperature range of the water from
exper ienced doctors, an d the approximately 40° to approximately 10°,
conclusion: that, therefore, intuition biologists have hypothesized that sharp
increases in water temperature must be
is more effective. None of the other involved in signaling the affected species
choices helps bridge this gap in the to begin their reproduction activities.
chain of reasoning. Although some Question :
of the other statements may be true, Which of the following statements, if
they are not responsive to the true, would most strengthen th e
question. In fact, they mostly focus scientists’ hypothesis?
on irrelevant factors such as Options :
appropriateness, ease of application, (a) The native fish species were still able
ability, etc. to reproduce in nearby streams
where the annual temperature range
3. Strengthen an Argument remains approximately 40°.
Assumptions connect premises to (b) Before the dam was built, the river
conclusions. An argument is annually overflowed its banks,
strengthened by strengthening the creating temporary backwaters that
were used as breeding areas for the
assumptions. Here are some local fish population.
examples of Strengthen question (c) The lowest temperature ever recorded
types : in the river prior to dam construction
• The conclusion would be more was 30°; whereas the lowest recorded
properly drawn if it were made river temperature after construction
clear that... was completed has been 40°.
• Which of the following, if true, (d) Non-native fish species, introduced
would most strengthen the after the dam was completed, have
conclusion drawn in the passage begun competing with the native
above? species for food.
(e) None of these
EBD_7420
248 Critical Reasoning
Sol. drug. It follows, then, that continued
(a) most strengthens the conclusion production of the drug must inevitably
that the scientists reached. It does lead to the raynhu’s extinction.
so a similar population, not subjected Question :
to the same change as the Which of the following, if true, most
population near the dam, did not seriously weakens the above conclusion?
experience the same type of result. Options :
Here the basic assumption about the (a) The drug made from Raynhu bark is
conclusion that scientists reached dispensed to doctors from a central
is ‘because of the reduction of authority.
average temperature range of the (b) The drug made from the Raynhu
water, the reproduction of the native bark is expensive to produce.
fish species has reduced drastically’. (c) The Raynhu generally grows in
Option (a) clearly strengthens the largely inaccessible places.
assumption. (d) The Raynhu can be propagated from
4. Weaken an Argument cuttings and cultivated by farmers.
Assumptions connect premises to (e) None of these
conclusions. An argument is Sol.
weakened by weakening the (d) provides an alternate source of the
assumptions. Here are some Raynhu bark. Even though the tree
examples of Weaken question types: is rare in the wild, the argument is
• Which of the following, if true, silent on the availability of
would weaken the conclusion cultivated trees. The author of the
drawn in the passage above? argument must be assuming that
• The argument as it is presented there are no Raynhu trees other than
in the passage above would be those in the wild, in order to make
weaken. If which of the following the leap from the stated evidence to
were true? the conclusion that the Raynhu is
headed for extinction. The option (d)
weakens the assupmtion - ‘there are
q Shortcut Approach limited raynhu trees’ - by saying that
How to approach “Weaken an there are other ways as well for the
argument” propogation of Raynhu. The other
• Once you have identified the answer choices all contain
argument of the passage, i.e. the information that is irrelevant. Note
evidence(s) + conclusion, try putting that the correct choice does not
in each option with the argument. make the conclusion of the argument
Check if the assumption(s) you have impossible. In fact, it is possible that
drawn is (are) weakened if you accept there may be domesticated Raynhu
the content of the option as true. trees and the species could still
become extinct. Answer choice (d)
EXAMPLE 4. Stimulus Argument is correct because it makes the
conclusion about extinction less
A drug that is very effective in treating likely to be true.
some forms of cancer can, at present, be
obtained only from the bark of the 5. Conclusion / Main Point
Raynhu, a tr ee that is quite rare in the Question
wild. It takes the bark of approximately In Main Point / Conlcusion questions,
5,000 trees to make one pound of the you have to identify the conclusion
Critical Reasoning 249
of an argument. You are trying to find
the author ’s point and should EXAMPLE 5. Stimulus Argument
approach this question in a similar People should be held accountable for
way to the reading comprehension their own behaviour, and if holding
main point questions. They come in people accountable for their own
several different formats: behaviour entails capital punishment,
• The main point of the passage is then so be it. However, no person should
that... be held accountable for behaviour over
• Which of the following statements which he or she had no control.
about... is best supported by the Question : Which of the following is the
statements above? most logical conclusion of the argument
• Which of the following best above?
states the author’s conclusion in Options :
the passage above? (a) People should not be held account-
• Which of the following able for the behaviour of other
conclusions can be most properly people.
drawn from the data above? (b) People have control over their own
The conclusion of arguments in behaviour.
Main Point questions is usually (c) People cannot control the behaviour
not directly stated. To find the
conclusion, identify the premises of other people.
and then identify the conclusion (d) People have control over behaviour
drawn from the premises. Main that is subject to capital punishment.
Point questions differ from the (e) None of these
other Critical Reasoning Sol.
questions in that the argument in (b) The correct response is (b). The
the stimulus is usually valid. (In argument includes the following two
most other Critical Reasoning premises:
questions the reasoning is Premise 1: People are accountable
flawed.) Conclusion questions for their own behaviour.
require you to choose the answer Premise 2: People are not
that is a summary of the argument. accountable for behaviour they
cannot control.
q Shortcut Approach Here’s the logical conclusion based
How to approach “Main Point on these two premises:
Questions”: Conclusion: People can control their
• Main Point answers must be within own behaviour.
the scope of the passage. (a) would require that people never
• Your opinions or information outside have control over the behaviour
of the passage are always outside of of other people. Yet the argument
the scope. does not provide this premise.
• Some of the options given can be out (c) would require that people
of the scope of the passage. should not be held accountable
• Knock out answers with extreme for th e behaviour of other
wording. Main Point answers people. Yet the argument does
typically do not use only, always, not provide this premise.
never, best or any strong words that (d) is not inferable.
leave little room. (e) None of these
EBD_7420
250 Critical Reasoning
6. Identify the Paradox Options :
These questions present you with (a) The business specializing in the
a paradox, a seeming contradiction rental of children’s furniture buys its
or discrepancy in the argument, and furniture from distributors outside of
ask you to resolve it or explain how Town Y.
that contradiction could exist. In (b) The few children who do reside in
other words, there are two facts that Town Y all know each other and often
are both true, and yet they appear stay over night at each other’s houses.
to be in direct conflict with one (c) Many residents of Town Y who
another. Here are some examples of move frequently prefer to rent their
the ways in which these questions fur n i t ur e r at h er t h a n buy i t
are worded: outright.
• Which of the following, if true, (d) Many residents of Town Y must
would help to resolve the apparent provide for the needs of visiting
paradox presented above? grandchildren several weeks a year.
• Which of the following, if true, (e) None of these
contributes most to an
Sol.
explanation of the apparent
discrepancy described above? (d) The correct answer (d), explains why
a town of mostly retired residents
q Shortcut Approach might need to rent children’s
furniture. The other answer choices
How to approach “Identify the paradox all contain irrelevant information.
questions” This further illustrates the fact that,
• Read the argument and find the on all question types, if you
apparent paradox, discrepancy, or eliminate the irrelevant choices, the
contradiction. remaining choice will most likely be
• State the apparent paradox, correct.
discrepancy, or contradiction in your 7. Evaluation/ Reasoning
own words. Based Questions
• Use process of elimination. The best
answer will explain how both sides Reasoning questions ask you to
of the paradox, discrepancy, or describe how the argument was
contradiction can be true. Eliminate made, not necessarily what it says.
These questions are closely related
answers that are out of scope.
to assumption, weakening, and
stren gthening questions. The
EXAMPLE 6. Stimulus Argument correct answer identifies a question
Town Y is populated almost exclusively tha t must be an swered or
by retired people and has almost no information that must be gathered
families with small children. Yet Town Y to deter mine h ow st rong the
is home to a thriving business specializing stim ulus a rgumen t is. The
in the rental of furniture for infants and information will be related to an
small children. assumption that the author is
Question : Which of the following, if true, making. Another type of question
best reconciles the seeming discrepancy that you will encounter asks you to
described above? identify a flaw in the stimulus
argument. The question tells you
Critical Reasoning 251
that there is a problem with the logic (c) The majority of the child custody
of the argument. You just have to cases that have reached the recently
choose the answer that describes elected judge’s court have been
the flaw. Here are some examples of appealed from a lower court.
the ways in which these questions (d) The evidence shows that men
are worded: should have won custody in more
• How does the author make his than 40% of the recently elected
point? judge’s cases involving divorcing
• A major flaw in the argument fathers.
above is that it... (e) None of these
• A’s response has which of the Sol.
following relationships to B’s (d) The correct answer (d), points out a
argument? flaw in the argument. Specifically, it
points out that the author of the
q Shortcut Approach argument was comparing the
How to approach Reasoning Questions recently elected judge to other judges.
• Read the argument and find the The author of the argument made an
conclusion. unwarranted assumption that the
• State the reasoning in your own recently elected judge did not rule
words. against many men in custody battles
• Check whether the reasoning given where the evidence clearly favored the
in the various options fall in line with men. As with strengthening and
the reasoning described above. weakening questions, the correct
answer in flaw questions often
involves unwarranted assumptions.
EXAMPLE 7. Stimulus Argument
EXAMPLE 8. Stimulus Argument
Some observers have taken the position
that the recently elected judge is biased Although dentures produced through a
against men in divorce cases that involve new computer-aided design process will
child custody. But the statistics reveal cost more than twice as much as ordinary
that in 40% of such cases, the recently dentures, they should still be cost
elected judge awards custody to the effective. Not only will fitting time and
fathers. Most other judges award X-ray expense be reduced, but the new
custody to fathers in only 20%–30%of dentures should fit better, diminishing
their cases. This record demonstrates that the need for frequent refitting visits to
the recently elected judge has not the dentist’s office.
discriminated against men in cases of Question : Which of the following must
child custody. be studied in order to evaluate the
Question : The argument above is flawed argument presented above?
in that it ignores the possibility that Options :
Options : (a) The amount of time a patient
(a) A large number of the recently spends in t he fitting pr ocess
elected judge’s cases involve child versus the amount of money spent
custody disputes. on X-rays
(b) The r ecently elected judge is (b) The amount by which the cost of
prejudiced against men in divorce producing dentures has declined
cases that do not involve child with the introduction of the new
custody issues. technique for producing them
EBD_7420
252 Critical Reasoning
(c) The degree to which the use of the • Which of the following is a
new dentures is likely to reduce the parallel argument to the above
need for refitting visits when compared given argument?
to the use of ordinary dentures
EXAMPLE 9. Stimulus Argument
(d) The amount by which the new
dentures will drop in cost as the It is true that it is against international
production procedures become law to provide aid to certain countries
standardized and applicable on a that are building nuclear programs. But,
larger scale if Russian companies do not provide aid,
(e) None of these companies in other countries will.
Sol. Question : Which of the following is most
(c) The correct answer (c), highlights an like the argument above in its logical
assumption in the stimulus structure?
argument. It shows that the author Options :
must be assuming that the reduction (a) It is true that it is against United
in refitting with the new dentures States policy to negotiate with
compared to ordinary dentures is kidnappers. But if the United States
significant in order to conclude that wants to prevent loss of life, it must
the difference will help offset an negotiate in some cases.
initial outlay that is twice as much. (b) It is true that it is illegal to sell
In other words, if you answer the diamonds that originate in certain
question posed by answer choice (c) countries. But there is a long tradition
with “not much,” the argument is in Russia of stockpiling diamonds.
weakened. If you answer it with “a (c) It is true that it is illegal for an
tremendous amount,” the argument attorney to participate in a
is strengthened. The other answer transaction in which there is an
choices are all irrelevant because no apparent conflict of interest. But, if
matter what the answers are, there is the facts are examined carefully, it
no impact on the relationship between will clearly be seen that there is no
the evidence presented in the stimulus actual conflict of interest in the
argument and its conclusion. defendant’s case.
(d) It is true that it is against the law to
8. Identify a Parallel steal cars. But someone else certainly
Argument / Structure. would have stolen that car if the
The last type of Critical Reasoning defendant had not done so first.
question is the parallel structure (e) None of these
question. In this type of question, Sol.
you must choose the answer that (d) The correct answer (d), has the same
has the same structure as th e structure as the stimulus argument.
stimulus argument. In other words, If you just replace “aid to developing
you have to find the argument that nuclear powers” with “car theft,” and
is analogous to the given argument “Russian companies” with the
in that it includes the same “defendant,” it is essentially the
relationship between the evidence same argument. Sometimes the
presented and the conclusion. Here parallel structure is easier to see if
are some examples of the ways in you use symbols to represent the
which these questions are worded: terms of the argument: It is true that
• Which of the following is most X is illegal. But, if Y doesn’t do it,
like the argument above in its others will. Here X is stealing cars
logical structure? and Y is the defendant.
Critical Reasoning 253
q Shortcut Approach
How to crack Parallel Argument Question?
• Read the argument and find the conclusion.
• Try to establish a reasoning structure between the premise and the condusion.
• Read out the options and look out for one having the similar reasoning structure.

PRACTICE EXERCISE
DIRECTIONS (Qs. 1-3): Study the 2. Which of the following is an
following information carefully and inference which can be made from
answer the questions given below : the facts stated in the above
paragraph ?
The centre reportedly wants to continue (a) India's fiscal deficit is
providing subsidy to consumers for negligible in comparison to
cooking gas and kerosene for five more other emerging economies in
years. This is not good news from the the world.
point of view of reining in the fiscal
(b) Subsidy on food and fertilizers
deficit. Mounting subventions for
subsidies means diversion of savings by are essential for growth of
the government from investment to Indian economy.
consumption, raising the cost of capital in (c) Reform in financial sector will
the process. The government must cut weaken India's position in the
expenditure on subsidies to create more international arena.
fiscal space for investments in both (d) Gradual withdrawal of
physical and social infrastructure. It should subsidy is essential for
outline a plan for comprehensive reform in effectively managing fiscal
major subsidies including petroleum, food deficit in India.
and fertilizers and set goal posts.
(e) None of these
1. Which of the following is a
conclusion which can be drawn 3. Which of the following is an
from the facts stated in the above assumption which is implicit in the
paragraph ? facts stated in the above
(a) Subsidy provided by th e paragraph?
government under various (a) People in India may not be
heads to the citizen increases able to pay more for petroleum
the cost of capital products.
(b) Government is unable to (b) Many people in India are rich
withdraw subsidies provided
to various items. enough to buy petroleum
(c) Government subsidy on products at market cost.
kerosene is purely a political (c) Government may not be able
decision. to create more infrastructural
(d) Govt. does not have enough facilities if the present level of
resources to continue subsidy continues for a
providing subsidy on longer time.
petroleum products. (d) Government of India has
(e) None of these sought assistance from
EBD_7420
254 Critical Reasoning
international financial (d) People living below poverty
organizations for its line remain in that position for
infrastructural projects a very long time.
(e) None of these (e) None of these
DIRECTIONS (Qs. 4-6): Study the 5. Which of the following is an
following Information carefully and assumption which is implicit in the
answer the questions given below: facts stated in the above paragraph?
(a) It may not be possible to have
Poverty measurement is an unsettled an accurate poverty
issue, both conceptually and measurement in India.
methodologically. Since poverty is a
process as well as an outcome; many come (b) Level of poverty in India is
out of it while others may be falling into it. static over the years.
The net effect of these two parallel (c) Researchers avoid making
processes is a proportion commonly conclusions on poverty
identified as the ‘head count ratio’, but measurement data in India.
these ratios hide the fundamental (d) Government of India has a
dynamism that characterises poverty in mechanism to measure level
practice. The most recent poverty of poverty effectively and
reestimates by an expert group has also accurately.
missed the crucial dynamism. In a study (e) None of these
conducted on 13,000 households which 6. Which of the following is an
represented the entire country in 1993-94 inference which can be made from
and again on 2004-05, it was found that in the facts stated in the above
the ten-year period 18.2% rural population paragraph ?
moved out of poverty whereas another
22.1% fell into it over this period. This net (a) Poverty measurement tools in
increase of about four percentage points India are outdated.
was seen to have a considerable variation (b) Increase in number of persons
across states and regions. falling into poverty varies
considerably across the
4. Which of the following is a country over a period of time.
conclusion which can be drawn
from the facts stated in the above
(c) Government of India has
paragraph ?
stopped measuring poverty
(a) Accurate estimates of number related studies.
of people living below (d) People living in rural areas are
poverty line in India is more susceptible to fall into
possible to be made. poverty over the time.
(b) Many expert groups in India (e) None of these
are not interested to measure
poverty objectively. DIRECTIONS (Qs. 7-8): Read the
following information carefully and
(c) Process of pover ty answer the questions which follow:
measurement needs to take Supermarkets are growing at a fast
into account various factors to pace than Kirana Stores. Kirana Stores
tackle its dynamic nature. are such places where customer go to
Critical Reasoning 255
purchase their necessities. In place of businesses in areas that have
five Kir an a stor es on e or two very few Kirana stores.
supermarkets are being established. It (b) People tend to trust retail
h as been foun d th a t customers’ outlets and supermarkets
requirements are looked after by when it comes to buying high
trained staff. It has been found that value products as opposed to
food products are low life products buying them from local
which are manufactured from local shops.
manufacturers. These products are (c) If there are two or more Kirana
typically purchased by the customer stores in a typical residential
on the assurance. The markets is area the competition among
appealing to supermarkets and retail them is very high.
outlet owners are setting up their
business in other areas where there are (d) Kirana stores owners are
less Kirana stores. buying to acquire franchisee
7. Which of the following can be a of supermarkets as the local
good argument in favour of shops have lost their charm
shopping, from Kirana stores and business.
instead of supermarkets? (e) Kirana stores do not sell the
(a) People prefer supermarkets products which consumers on
because they offer a larger their regular shopping lists,
ran ge of products. i.e., so they have to depend on
products other than FMCG supermarkets.
and they can buy everything
under one roof. DIRECTIONS (Qs. 9-10): Read the
(b) People end-up buying other following statements carefully and
irrelevant things along with answer the questions given below:
those on their shopping lists Mobile technology has played great role
in Supermarkets and then they in growth and development of society.
have to stand in long queues
Earlier cellphone was used as a medium
at the billing counters.
of con versation only. Now mobile
(c) Most Kirana stores are closed
atleast one day in a week phones also support a wide variety of
whereas supermarkets are other services, such as, texting, email,
open 365 days a year. internet access etc. The price of mobile
(d) Kirana stores do not accept phones is also decreasing and people
debit and credit cards. are being encouraged to buy a mobile
(e) Very few Kirana stores sell phone set at a cheaper rate. The mobile
products at a bargained price. technology and smartphones have the
8. Which of the following can be capabilities of handling video calls,
in ferred from the given sharing large files. Mobile technology
information? (An inference is had made it more efficient to conduct
something that is not directly business. Video calls and taking
stated but can inferred from the photographs have become possible as
given information). mobile phone has in built camera.
(a) Most supermarkets and retail Therefore, there is no need to carry
outlet owners choose to set up around a camera everywhere you go,.
EBD_7420
256 Critical Reasoning
9. Which of the following cannot be (e) Technological advances are
in ferred from the given taking place in fields other
information? (An inference is than cellphones as well.
something that is not directly 10. Which of the following can be
stated but can be inferred from the concluded fr om the given
given information) information?
(a) One can share photos and (a) Buying a phone with a camera
videos via mobile phones is more convenient than
provided that the other buying two different devices.
person has a similar device. (b) Mobile phones that are basic
(b) Many features are being last longer than the ones with
added to mobile phones now- added features
a-days (c) Not many people are
(c) The other features of mobile interested in clicking pictures
phones are used as useful as with mobile phones
the built in camera (d) It is possible to share pictures
(d) Mobile phones can be used with someone having a similar
for purposes other than camera
making calls (e) No one will buy camera now
onwards.

HINTS & SOLUTIONS


1. (a) Consider the following line of subsidy cannot continue for a
the passage: long time.
“Mounting subventions for 4. (c) Process of poverty measu-
subsidies means diversion of rement needs to take into
savings by the government account various factors to
from investment of tackle its dynamic nature.
consumption, raising the 5. (a) It may not be possible to have
coast of Capital in th e an accurate poverty measu-
process”. rement in India.
2. (d) Consider the following lines 6. (b) Increase in number of persons
of the passage: falling into poverty varies
“The government must cut considerably across the
expenditure on subsidies to country over a period of time.
create more fiscal space for 7. (b) Option (b) strengthens the
investments in both physical statement.
and social infrastructure”. 8. (a) Obviously option (a) can be
inferred from the given
3. (c) The government must reduce
information.
expenditure on subsidies so
9. (e) From the given information we
that to create more
cannot infer option (e)
infrastructural facilities
10. (a) Obviously option (a) is
otherwise present level of
correct.

You might also like